Олимпиадад бэлтгэгчдэд тусламж (Тэнцэтгэл биш 1)

523
Inequalities Methods and Olympiad Problems

Upload: bayarbaatar-surenjav

Post on 27-Jul-2015

91 views

Category:

Education


14 download

TRANSCRIPT

Page 1: Олимпиадад бэлтгэгчдэд тусламж (Тэнцэтгэл биш 1)

Inequalities

Methods and

Olympiad Problems

Page 2: Олимпиадад бэлтгэгчдэд тусламж (Тэнцэтгэл биш 1)

Contents

1 Part I 3

1.1 Squares are positive . . . . . . . . . . . . . . . . . . . . . . . . 3

1.2 Some special inequalities and identities for two numbers . . . . 10

1.3 Some special inequalities and identities for more numbers . . . 13

1.4 The mathematical induction . . . . . . . . . . . . . . . . . . . . 17

1.5 The AM-GM inequality . . . . . . . . . . . . . . . . . . . . . . 25

1.6 The quadratic trinomial . . . . . . . . . . . . . . . . . . . . . . 29

1.7 Cauchy-Schwartz Inequality . . . . . . . . . . . . . . . . . . . . 41

1.8 Young Inequality . . . . . . . . . . . . . . . . . . . . . . . . . . 48

1.9 Advanced techniques with Cauchy-Buniakowski-Schwarz andHolder Inequalities . . . . . . . . . . . . . . . . . . . . . . . . . 50

1.10 The principle of extremality and monotonicity . . . . . . . . . . 84

1.11 Breaking the inequality . . . . . . . . . . . . . . . . . . . . . . 86

1.12 Separating the squares . . . . . . . . . . . . . . . . . . . . . . . 92

1.13 The Dual Principle . . . . . . . . . . . . . . . . . . . . . . . . . 101

1.14 Substitutions . . . . . . . . . . . . . . . . . . . . . . . . . . . . 104

1.15 Homogenization and dehomogenization . . . . . . . . . . . . . . 109

1.16 Unimonotonic sequences . . . . . . . . . . . . . . . . . . . . . . 111

1.17 Working backwards . . . . . . . . . . . . . . . . . . . . . . . . . 128

1.18 Mixing variables . . . . . . . . . . . . . . . . . . . . . . . . . . 129

1.19 Limits in inequalities . . . . . . . . . . . . . . . . . . . . . . . . 133

1.20 Derivatives in Inequalities . . . . . . . . . . . . . . . . . . . . . 135

1.21 Convexity . . . . . . . . . . . . . . . . . . . . . . . . . . . . . . 139

1.22 Jensen’s Inequality . . . . . . . . . . . . . . . . . . . . . . . . . 141

1.23 The shrinking principle and Karamata’s Inequality . . . . . . . 144

1.24 Schur’s Inequality . . . . . . . . . . . . . . . . . . . . . . . . . . 148

1.25 The generalized Means . . . . . . . . . . . . . . . . . . . . . . . 149

1.26 Inequalities between the symmetric sums . . . . . . . . . . . . . 150

1.27 The pqr technique . . . . . . . . . . . . . . . . . . . . . . . . . 151

1.28 The tangent line technique and its extensions . . . . . . . . . . 159

1.29 Using identities to prove inequalities . . . . . . . . . . . . . . . 178

2 Problems 185

3 Solutions 225

1

Page 3: Олимпиадад бэлтгэгчдэд тусламж (Тэнцэтгэл биш 1)

4 Anexa 1 4894.1 Metoda compararii . . . . . . . . . . . . . . . . . . . . . . . . . 4894.2 Metoda majorarii si minorarii . . . . . . . . . . . . . . . . . . . 4994.3 Metoda coeficientilor nedeterminati . . . . . . . . . . . . . . . . 5044.4 Metoda normalizarii . . . . . . . . . . . . . . . . . . . . . . . . 5104.5 Metoda omogenizarii . . . . . . . . . . . . . . . . . . . . . . . . 515

2

Page 4: Олимпиадад бэлтгэгчдэд тусламж (Тэнцэтгэл биш 1)

Chapter 1

Part I

1.1 Squares are positive

Perhaps the first inequality one learns is ”The square of any real number isnonnegative”. It’s a very simple one, but despite its simplicity it is one ofthe most important inequalities, used frequently by beginners and advancedproblem solvers alike. It lies at the base of virtually every inequality: forexample, the inequality a+b

2 ≥√ab can be rewritten as 1

2(√a−√b)2 ≥ 0. But

writing an expression as a square, or as a sum of squares, is usually far fromobvious. It requires a certain level of intuition and creativity, but perhaps moreimportantly, experience. We thus start with some introductory problems.

Example 1. If a, b, c are arbitrary real numbers, then a2+b2+c2 ≥ ab+bc+ca.

Solution: Let us first see what we have here: squares of numbers versus theirpairwise products. Next, we ask ourselves: ”Have I ever seen things like thesebefore?”. Well, yes. Namely, we’ve just seen that (x − y)2 = x2 − 2xy +y2, where x, y are two (randomly chosen) real numbers. Motivated by thisidentity, we multiply by two our inequality, and see that it is equivalent with2(a2 + b2 + c2) ≥ 2(ab+ bc+ ca). We now transfer everything to the left side.In this case,

2a2 + b2 + c2 − 2(ab+ bc+ ca) =

= (a2 − 2ab+ b2) + (b2 − 2bc+ c2) + (c2 − 2ca+ a2)

=[(a− b)2 + (b− c)2 + (c− a)2

],

which is clearly nonnegative, and thus our inequality is proven.

Example 2. If a, b are real numbers such that a+ b = 2, then a4 + b4 ≥ 2.

Solution: Since 2(a4 + b4)− (a2 + b2)2 = a4 − 2a2b2 + b4 = (a2 − b2)2 ≥ 0, we

get that a4 + b4 ≥ (a2 + b2)2

2. Similarly, 2(a2 + b2)− (a+ b)2 = a2−2ab+ b2 =

(a−b)2 ≥ 0, and therefore a2 +b2 ≥ (a+ b)2

2= 2.Thus a4 +b4 ≥ (a2 + b2)2

2≥

22

2 = 2.

3

Page 5: Олимпиадад бэлтгэгчдэд тусламж (Тэнцэтгэл биш 1)

Example 3. Let a, b, c > 0. Prove that a3 + b3 + c3 + ab2 + bc2 + ca2 ≥

2(a2b+ b2c+ c2a).Solution: Note that a3 +ab2−2a2b = a(a2−2ab+b2)2 = a(a−b)2 ≥ 0. In thiscase, we analogously establish that b3+bc2−2b2c ≥ 0 and c3+ca2−2c2a ≥ 0.Summing up all these three inequalities yields our result.

Example 4. Let a, b, x, y be arbitrary real numbers. Show that (a2 + b2)(x2 +y2) ≥ (ax+ by)2.

Solution:Transferring the terms to the left side, the inequality rewrites as(a2 + b2)(x2 + y2)− (ax+ by)2 ≥ 0. Denoting the quantity from the left handside by P , we get

P = (a2 + b2)(x2 + y2)− (ax+ by)2

= a2x2 + a2y2 + b2x2 + b2y2 − (a2x2 + b2y2 + 2abxy)

= a2y2 + b2x2 − 2abxy = (ay − bx)2 ≥ 0.

Remarks. Notice that this inequality is a particular case of the following:

(a21 + a22 + . . .+ a2n)(b21 + b22 + . . .+ b2n) ≥ (a1b1 + a2b2 + . . .+ anbn)2,

where a1, a2, . . . , an and b1, b2, . . . , bn are arbitrary real numbers. Thislast one is known in literature as the Cauchy-Schwartz inequality, to which wehave dedicated the next chapter. Anyway, before this, let us see some other(. . .)2 ≥ 0 problems.

Exercises

1. Show that a2 + b2 + c2 + 3 ≥ 2(a+ b+ c).

Solution: Moving everything to the left side, we get

a2 + b2 + c2 + 3− 2(a+ b+ c) = (a2 − 2a+ 1) + (b2 − 2b+ 1) + (c2 − 2c+ 1)

= (a− 1)2 + (b− 1)2 + (c− 1)2 ≥ 0,

which proves our inequality.

2. If x, y are real numbers, show that x2 + y2 ≥ (x−y)22

Solution: This is equivalent to x2+y2+2xy2 ≥ 0 i.e. (x+y)2

2 ≥ 0.

3. Show that x4 + y4 + z2 + 1 ≥ 2x(xy2 − x+ z + 1).

Solution: Moving everything to the left side, and denoting by P thequantity from the left hand side, we get

P = x4 + y4 + z2 + 1− 2x(xy2 − x+ z + 1)

= (x4 − 2x2y2 + y4) + (x2 − 2xz + z2) + (x2 − 2x+ 1)

= (x2 − y2)2 + (x− z)2 + (x− 1)2 ≥ 0.

4

Page 6: Олимпиадад бэлтгэгчдэд тусламж (Тэнцэтгэл биш 1)

4. Show that for any real x, we have (x− 1)(x− 3)(x− 4)(x− 6) + 10 > 0.

Solution: Factoring our expression, we obtain that

(x− 1)(x− 3)(x− 4)(x− 6) + 10 = (x2 − 7x+ 6)(x2 − 7x+ 12) + 10

= (x2 − 7x+ 6)2 + 6(x2 − 7x+ 6) + 10

= (x2 − 7x+ 9)2 + 1 > 0.

5. If a and b are strictly (nonzero) positive real numbers, prove thata√b

+

b√a≥√a+√b.

Solution: Moving everything to the left side, it follows that

a√b

+b√a−√a−√b =

(a√b−√b

)+

(b√a−√a

)=

a− b√b

+b− a√a

= (a− b)(

1√b− 1√

a

)

=

(√a−√b)2 (√

a+√b)

√ab

≥ 0.

6. Show that for any positive real number x, we have x5− x2− 3x+ 5 > 0.

Solution: Note that the following chain of equalities holds:

x5 − x2 − 3x+ 5 = x2(x3 − 1)− 3(x− 1) + 2

= (x− 1)[x2(x2 + x+ 1)− 3

]+ 2

= (x− 1)[(x4 − 1) + (x3 − 1) + (x2 − 1)

]+ 2

= (x− 1)2(x3 + 2x2 + 3x+ 3) + 2.

Since this last term is nonnegative, we deduce our inequality.

7. If x, y are two real numbers, prove that x4 + y4 ≥ x3y + y3x.

Solution: Since x2 + xy + y2 = 14(x− y)2 + 3

4(x+ y)2 ≥ 0, we have that

x4 + y4 − x3y − y3x = (x4 − x3y) + (y4 − y3x) = x3(x− y) + y3(y − x)

= (x− y)(x3 − y3) = (x− y)2(x2 + xy + y2) ≥ 0.

8. Let a, b, c,m, n, p be real numbers (a, b, c 6= 0), such that ap−2bn+cm =0 and ac = b2. Show that n2 ≥ mp.

Solution: For any two real numbers x and y, we have that (x+y)2−4xy =x2 − 2xy + y2 = (x − y)2 ≥ 0, or equivalently, (x + y)2 ≥ 4xy. In thiscase,

4b2n2 = (ap+ cm)2 ≥ 4ap · cm = 4acpm = 4b2mp,

and thus n2 ≥ mp.

5

Page 7: Олимпиадад бэлтгэгчдэд тусламж (Тэнцэтгэл биш 1)

9. Show that a2 + b2 + c2 + d2 + e2 ≥ a(b+ c+ d+ e).

Solution: Moving everything to the left side, it follows that

A = a2 + b2 + c2 + d2 + e2 − a(b+ c+ d+ e)

=

(a2

4− ab+ b2

)+

(a2

4− ac+ c2

)+

(a2

4− ad+ d2

)+

(a2

4− ae+ e2

)=

(a2− b)2

+(a

2− c)2

+(a

2− d)2

+(a

2− e)2≥ 0.

10. If a, b, c are nonnegative real numbers, show that

a3

a2 + ab+ b2+

b3

b2 + bc+ c2+

c3

c2 + ca+ a2≥ a+ b+ c

3.

Solution: Sincea3

a2 + ab+ b2− 2a− b

3=

(a− b)2(a+ b)

3(a2 + ab+ b2)≥ 0, we get

a3

a2 + ab+ b2+

b3

b2 + bc+ c2+

c3

c2 + ca+ a2≥ 2a− b

3+

2b− c3

+2c− a

3

=a+ b+ c

3.

11. Let a, b, c be three arbitrary real numbers. Prove that√a2 + ab+ b2 +

√b2 + bc+ c2 +

√c2 + ca+ a2 ≥

√3(a+ b+ c).

Solution: Since a2 + ab + b2 = 14(a − b)2 + 3

4(a + b)2 ≥ 34(a + b)2, we

deduce that√a2 + ab+ b2+

√b2 + bc+ c2+

√c2 + ca+ a2 ≥

√3

2(|a+ b|+ |b+ c|+ |c+ a|) .

Now, for any real number x we have |x| ≥ x, and therefore

√3

2(|a+ b|+ |b+ c|+ |c+ a|) ≥

√3

2(a+ b+ b+ c+ c+ a)

=√

3(a+ b+ c),

which yields our result.

12. Prove that 19x2 + 54y2 + 16z2 + 36xy − 24yz − 16zx ≥ 0.

Solution: The inequality follows from the chain of equalities writtenbelow:

P = 19x2 + 54y2 + 16z2 + 36xy − 24yz − 16zx

= (9x2 + 36xy + 36y2) + (18y2 − 24yz + 8z2) + (8x2 − 16xz + 8z2) + 2x2

= 9(x+ 2y)2 + 2(3y − 2z)2 + 8(x− z)2 + 2x2 ≥ 0.

6

Page 8: Олимпиадад бэлтгэгчдэд тусламж (Тэнцэтгэл биш 1)

13. Let x, y be real numbers such that xy ≥ 1. Show that1

x2 + 1+

1

y2 + 1≥

2

xy + 1.

Solution: Note that

1

x2 + 1+

1

y2 + 1− 2

xy + 1=

(1

x2 + 1− 1

xy + 1

)+

(1

y2 + 1− 1

xy + 1

)=

x(y − x)

(xy + 1)(x2 + 1)+

y(x− y)

(xy + 1)(y2 + 1)

=x− yxy + 1

(y

y2 + 1− x

x2 + 1

)=

(x− y)2(xy − 1)

(xy + 1)(x2 + 1)(y2 + 1),

which is clearly nonnegative. This proves the inequality in question.

14. If a, b, c are positive real numbers, prove thata

b+ c+

b

c+ a+

c

a+ b≥ 3

2 .

Solution: We have

P =a

b+ c+

b

c+ a+

c

a+ b− 3

2

=

(a

b+ c− 1

2

)+

(b

c+ a− 1

2

)+

(c

a+ b− 1

2

)=

(a− b) + (a− c)2(b+ c)

+(b− c) + (b− a)

2(c+ a)+

(c− a) + (c− b)2(a+ b)

=a− b

2

(1

b+ c− 1

c+ a

)+b− c

2

(1

c+ a− 1

a+ b

)+c− a

2

(1

a+ b− 1

b+ c

)=

(a− b)2

2(b+ c)(a+ c)+

(b− c)2

2(a+ b)(a+ c)+

(c− a)2

2(a+ b)(b+ c),

which is clearly nonnegative.

15. Let a, b, c be nonnegative real numbers such that a + b + c = 1. Provethat √

a+(b− c)2

4+

√b+

(c− a)2

4+

√c+

(a− b)24

≤ 2.

Solution: Squaring both sides, the inequality to prove rewrites as

∑cyc

(a+

(b− c)2

4

)+ 2∑cyc

√(b+

(c− a)2

4

)(c+

(a− b)24

)≤ 4.

Now, notice that (b− c)2(1 + a− b− c) = 2a(b− c)2 ≥ 0, which gives usthat

2

√(b+

(c− a)2

4

)(c+

(a− b)24

)≤ b+ c+

(a− b)(a− c)2

.

7

Page 9: Олимпиадад бэлтгэгчдэд тусламж (Тэнцэтгэл биш 1)

Thus it suffices to show that∑cyc

(a+

(b− c)2

4

)+∑cyc

(b+ c+

(a− b)(a− c)2

)≤ 4,

which since∑cyc

(a − b)(a − c) = a2 + b2 + c2 − ab − bc − ca, reduces to

3(a+ b+ c) + a2 + b2 + c2 − ab− bc− ca ≤ 4, or equivalently, a2 + b2 +c2 − ab− bc− ca ≤ 1. This is obviously true, because

1 = (a+ b+ c)2 = (a2 + b2 + c2 − ab− bc− ca) + 3(ab+ bc+ ca)

≥ a2 + b2 + c2 − ab− bc− ca.

16. Let x, y be two strictly (nonzero) positive real numbers such that x2 +y3 ≥ x3 + y4. Prove that x3 + y3 ≤ x2 + y2 ≤ x+ y ≤ 2.

Solution: Since y2 − y3 − (y3 − y4) = y2(y − 1)2 ≥ 0, we have that

x2 − x3 + y2 − y3 ≥ x2 − x3 + y3 − y4 = (x2 + y3)− (x3 + y4) ≥ 0,

and therefore x3 + y3 ≤ x2 + y2. Now, since x − x2 − (x2 − x3) =x(x − 1)2 ≥ 0, and y − y2 − (y3 − y4) = y(y − 1)2(y + 1) ≥ 0, weget x2 + y2 ≤ x + y, and thus the problem reduces to proving thatx + y ≤ 2. Now, note that x − (1 − x2 + x3) = −(x − 1)2(x + 1) ≤ 0,y − (1− y3 + y4) = −(y − 1)2(y2 + y + 1) ≤ 0, and therefore, it followsthat

x+ y ≤ 1− x2 + x3 + 1− y3 + y4 = 2 + (x3 + y4)− (x2 + y3) ≤ 2.

17. Let a, b, c > 0. Show that ab

(b

c− 1

)+ bc

( ca− 1)

+ ca(ab− 1)≥ 0.

Solution: Moving everything to the left side, it follows that

P = ab

(b

c− 1

)+ bc

( ca− 1)

+ ca(ab− 1)

=ab2

c+bc2

a+ca2

b− ab− bc− ca

=

(ab2

c+ ca− 2ab

)+

(bc2

a+ ab− 2bc

)+

(ca2

b+ bc− 2ca

)=

a(b− c)2

c+b(c− a)2

a+c(a− b)2

b≥ 0.

18. Consider three positive real numbers a, b, c such that a > c and b > c.Prove that √

c(a− c) +√c(b− c) ≤

√ab.

Solution: According to Example 4, we get[√c(a− c) +

√c(b− c)

]2=

(√c ·√a− c+

√b− c ·

√c)2

≤ (c+ b− c) (a− c+ c) = ab.

8

Page 10: Олимпиадад бэлтгэгчдэд тусламж (Тэнцэтгэл биш 1)

19. Let a, b, c be nonnegative real numbers, from which at least two arenonzero. Prove that√

a(b+ c)

b2 + c2+

√b(c+ a)

c2 + a2+

√c(a+ b)

a2 + b2≥ 2.

Solution: Notice that for any two nonnegative real numbers x, y, we havex+ y− 2

√xy =

(√x−√y

)2 ≥ 0, and thus x+ y ≥ 2√xy. Now, assume

that a ≥ b ≥ c; then√a(b+ c)

b2 + c2≥

√a(b+ c)

b2 + bc=

√a

b,√

b(c+ a)

c2 + a2≥

√b(c+ a)

ca+ a2=

√b

a,

and therefore√a(b+ c)

b2 + c2+

√b(c+ a)

c2 + a2+

√c(a+ b)

a2 + b2≥√a

b+

√b

a≥ 2

√√a

b·√b

a= 2.

20. Let a, b, c be nonnegative real numbers, from which at least two arenonzero. Prove that√

a(b+ c)

a2 + bc+

√b(c+ a)

b2 + ca+

√c(a+ b)

c2 + ab≥ 2.

Solution: Assume that a ≥ b ≥ c. Since (x+y)2 = x2+y2+2xy ≥ x2+y2,for all x, y ≥ 0, we have√

a(b+ c)

a2 + bc+

√c(a+ b)

c2 + ab≥√a(b+ c)

a2 + bc+c(a+ b)

c2 + ab,

and we will now show thata(b+ c)

a2 + bc+c(a+ b)

c2 + ab≥ b2 + ca

b(c+ a), or equiv-

alently,c(a+ b)

c2 + ab≥ b2 + ca

b(c+ a)− a(b+ c)

a2 + bc. This rewrites as

c(a+ b)

c2 + ab≥

c(a+ b)(a− b)2

b(c+ a)(a2 + bc), or b(c+ a)(a2 + bc) ≥ (a− b)2(ab+ c2) which is obvi-

ously true since b(c+a) = ab+ bc ≥ ab+ c2, and a2 + bc ≥ a2 ≥ (a− b)2.Hence √

a(b+ c)

a2 + bc+

√c(a+ b)

c2 + ab≥

√b2 + ca

b(c+ a),

and therefore the problem reduces to

√b(c+ a)

b2 + ca+

√b2 + ca

b(c+ a)≥ 2. In

the proof of Exercise 19, we have showed that x + y ≥ 2√xy for any

nonnegative real numbers x, y. Therefore√b(c+ a)

b2 + ca+

√b2 + ca

b(c+ a)≥ 2

√√√√√b(c+ a)

b2 + ca·

√b2 + ca

b(c+ a)= 2,

which completes our proof.

9

Page 11: Олимпиадад бэлтгэгчдэд тусламж (Тэнцэтгэл биш 1)

1.2 Some special inequalities and identities for twonumbers

We have already met the arithmetic mean of two numbers:

AM =a+ b

2

and the geometric mean of two numbers:

GM =√ab

We can also consider the harmonic mean

HM =2

1

a+

1

b

=2ab

a+ b

and the quadratic mean of two numbers

QM =

√a2 + b2

2

For positive a, b, these means satisfy the following chain of inequalities:

QM ≥ AM ≥ GM ≥ HM.

Proof : From the transitivity of ≥ it suffices to prove

QM ≥ AM, AM ≥ GM, GM ≥ HM.

Let’s take them step by step:

a) QM ≥ AM . This is equivalent to

QM2 ≥ AM2 ora2 + b2

2≥(a+ b

2

)2

=a2 + 2ab+ b2

4.

Multiplying by 4 we must prove 2(a2+b2) ≥ a2+b2+2ab or a2+b2 ≥ 2abwhich is just the inequality AM ≥ GM for a2, b2.

b) AM ≥ GM was proven by us in the previous topic.

c) GM ≥ HM . This is equivalent to1

GM≤ 1

HM, or to

1√ab≤

1

a+

1

b2

which is just AM ≥ GM for1

a,1

b.

[Remark: This chain of inequalities can be generalized in the following form:Let a1, a2, . . . , an be positive real numbers, and for every k ∈ R consider thegeneralized means

mk =k

√ak1 + ak2 + . . .+ akn

n

10

Page 12: Олимпиадад бэлтгэгчдэд тусламж (Тэнцэтгэл биш 1)

if k 6= 0 andm0 = n

√a1a2 . . . an

Then mk is increasing as a function in k: that is

mα ≥ mβ

if α ≥ β

Our inequality chain QM ≥ AM ≥ GM ≥ HM is just a special case of thisinequality for n− 2 and k = 3, 2, 1, 0.The proof of this inequality requires more advanced methods, and we willreturn to it in the further sections.]

We now move to identities. There is one basic identity for two numbers: theNewton Binomial Theorem:

(a+ b)n =

n∑i=0

(n

i

)aibn−i

where(ni

)=

n!

i!(n− i)!(recall that n! = 1× 2× . . .× n).

Particularly(a+ b)2 = a2 + 2ab+ b2

(a+ b)3 = a3 + 3a2b+ 3ab2 + b3

(a+ b)4 = a4 + 4a3b+ 6a2b2 + 4ab3 + b4

(a− b)2 = a2 − 2ab+ b2

(a− b)3 = a3 − 3a2b+ 3ab2 − b3(a− b)4 = a4 − 4a3b+ 6a2b2 − 4ab3 + b4

etc.Another well-known identity is

an − bn = (a− b)(an−1 + an−2b+ . . .+ bn−2a+ bn−1),

and for n odd,

an + bn = (a+ b)(an−1 − an−2b+ . . .− abn−2 + bn−1).

Note: the above identities are often used for b = 1. This is because ho-mogeneous inequalities in two numbers a, b can be reduced to inequalities injust one variable x by letting x = a

b (but more on that in the section abouthomogeneity).Finally, a very simple but sometimes crucial inequality is the triangle inequal-ity

|x|+ |y| ≥ |x+ y|.

Exercises

1. Let a > b > 0. Show that 2(a2 − b2)2 ≤ a2(a− b)2 + a2(a− b)2

Proof: This is equivalent to 4(a−b)2(a+b)24a2

≤ (a−b)2+(a−b)22 which is just the

inequality QM ≥ HM squared for the numbers a+ b, a− b. Of course,

11

Page 13: Олимпиадад бэлтгэгчдэд тусламж (Тэнцэтгэл биш 1)

this inequality can also be shown by opening the brackets and usingthe AM-GM inequality or turning it into a sum of squares. That’s notsurprising since all the inequalities AM ≥ GM,GM ≥ HM,QM ≥ AMwere proven using the same method, which is essentially that the squareof the difference of two numbers is non-negative.

2. If m,n are natural numbers and x > 0 thenxmn − 1

m≥ xn − 1

x.

Solution: We can extract the common factor xn − 1 to get

xmn − 1

m− xn − 1

x=

1

nx(xn − 1)

[x(xn(m−1) + . . .+ 1)−m

]=

1

nx(x− 1)(xn−1 + . . .+ 1)

[x(xn(m−1) + . . .+ 1)−m

].

Now as the right parenthesis vanishes for x = 1, extracting x− 1 out ofit produces

1

nx(xn − 1)(xn(m−1) − 1 + xn(m−2) − 1 + . . .+ 1− 1) =

=1

nx(x− 1)2(xn−1 + . . .+ 1)

[(xn(m−1) + . . .+ 1) + (xn(m−2) + . . .+ 1) + . . .

],

and this is clearly positive.

3. For a1, a2, a3, . . . , an ∈ R show that

min1≤i<k≤n

(ak − ai)2 ≤12

n(n2 − 1)

(n∑k=1

a2k

).

Solution: Suppose min{|ak − ai|} = a and a1 < a2 < . . . < an. Then,we clearly have |al − ak| = |al − al−1|+ . . .+ |ak+1 − ak| ≥ |l − k|a.

Next we use the inequality x2 + y2 ≥ (x− y)2

2- proven in the exercises

of the previous section. So we have a2i +a2n−i+1 ≥(n−2i+1)2

2 a2. Summingover all i = 1, 2, . . . , n and dividing by 2 we get the desired result.

4. Let a, b, c be real numbers such that |ax2 + bx+ c| ≤ 1 whenever |x| ≤ 1.Show that |cx2 + bx+ a| ≤ 2 whenever |x| ≤ 1.

Solution: By setting x = 0 we get |c| ≤ 1 and by setting x = ±1 we get|a+ b+ c|, |a− b+ c| ≤ 1. Then as |x| ≤ 1, |x2 − 1| ≤ 1 and then

|cx2 + bx+ a| = |c(x2 − 1) + c+ bx+ a| ≤ |c| · |x2 − 1|+ |c+ bx+ a|.

Since c+ bx+ a is between c+ b+ a and c− b+ a we get

|c+ bx+ a| ≤ max{|c+ b+ a|, |c− b+ a|} ≤ 1

and hence |cx2 + bx+ a| ≤ 1 + 1 = 2 as desired.

12

Page 14: Олимпиадад бэлтгэгчдэд тусламж (Тэнцэтгэл биш 1)

1.3 Some special inequalities and identities for morenumbers

Naturally there are more, but we shall announce only some of them, mostlyfor three numbers.Let’s take the inequality a2 + b2 ≥ 2ab. We can also deduce b2 + c2 ≥ 2bc, a2 +c2 ≥ 2ac. Summing all this inequalities we deduce 2(a2 + b2 + c2) ≥ 2(ab +bc+ ca) so

a2 + b2 + c2 ≥ ab+ bc+ ca

a very nice inequality in three numbers.Since (a+ b+ c)2 = a2 + b2 + c2 + 2(ab+ bc+ ca) we deduce

3(a2 + b2 + c2) ≥ (a+ b+ c)2 ≥ 3(ab+ bc+ ca).

Now for positive a, b, c, we multiply a2 + b2 + c2 − ab− bc− ca by a+ b+ c toget a3 + b3 + c3 − 3abc = (a+ b+ c)(a2 + b2 + c2 − ab− ca− bc) ≥ 0 so

a3 + b3 + c3 ≥ 3abc−AM-GM inequality for three numbers.

If we let a3 = x, b3 = y, c3 = z we deduce another form for it:

x+ y + z

3≥ 3√xyz or (x+ y + z)3 ≥ 27xyz

Particularly,

a2b ≤ 2a3 + b3

3, b2c ≤ 2b3 + c3

3, c2a ≤ 2c3 + a3

3.

Summinga2b+ b2c+ c2a ≤ a3 + b3 + c3

and analogouslyab2 + bc2 + ca2 ≤ a3 + b3 + c3

.From the other side, by applying the AM-GM inequality we can see

a2b+ b2c+ c2a ≥ 3abc, ab2 + bc2 + ca2 ≥ 3abc.

The expression a3 + b3 + c3 − (a+ b+ c)3 vanishes when two of a, b, c sum tozero, so it’s divisible by (a+ b)(b+ c)(c+ a).We deduce then (by direct computation) the identity

a3 + b3 + c3 − (a+ b+ c)3 = −3(a+ b)(b+ c)(c+ a)

.We wish to compare a2b+ b2c+ c2a with ab2 + bc2 + ca2, and have the identity

a2b+ b2c+ c2a− ab2 − bc2 − ca2 = (a− b)(b− c)(a− c)

which casts a light on the relationship between the two expressions accordingto the relative order of a, b, c.An analogous identity is a3b+ b3c+ c3a− ab3 − bc3 − ca3 = (a− b)(b− c)(a−c)(a+ b+ c).

13

Page 15: Олимпиадад бэлтгэгчдэд тусламж (Тэнцэтгэл биш 1)

We note that (a+ b)(b+ c)(c+ a) = a2b+ b2a+ b2c+ c2b+ a2c+ c2a+ 2abcand

(a+b+c)(ab+bc+ca) = a2b+b2a+b2c+c2b+3abc = (a+b)(b+c)(c+a)+abc.

Since abc ≤ (a+ b)(b+ c)(c+ a)

8(by multiplying the AM-GM inequality for

the three pairs (a, b), (b, c), (c, a)), we deduce

(a+ b)(b+ c)(c+ a) ≥ 8

9(a+ b+ c)(ab+ bc+ ca).

Consider the product (b+ c−a)(a+ c− b)(a+ b− c). At most one of b+ c−a,a+ c− b, a+ b− c is negative because the sum of any two of the is positive.So, if this product is positive, then they are all positive. we can deduce√

(b+ c− a)(a+ c− b) ≤ (b+ c− a) + (a+ c− b)2

= c.

Multiplying this by the two analogous inequalities we get

(b+ c− a)(a+ c− b)(a+ b− c) ≤ abc.

This is true also when the RHS is negative. Now opening the brackets werewrite the inequality as

a3 + b3 + c3 + 3abc ≥ a2b+ b2a+ b2c+ c2b+ a2c+ c2a

which is the famous Schur’s inequality.

Examples

1. Factorize a4b+ b4c+ c4a− ab4 − bc4 − ca4

Answer : (a− b)(b− c)(a− c)(a2 + b2 + c2 + ab+ bc+ ca).

2. Factorize 2(a5 + b5 + c5)− 5abc(a2 + b2 + c2)

Answer : (a+ b+ c)[2(a2 + b2 + c2)− 5(ab+ bc+ ca)

].

3. Show that a2 + b2 + c2 − ab− bc− ca ≥ 3

4(a− b)2

Solution: We have a2+b2+c2−ab−bc−ca =1

2

[(a− b)2 + (b− c)2 + (c− a)2

]and (b− c)2 + (c− a)2 ≥ 1

2(b− c+ c− a)2 =

(a− b)2

2, and from here the

conclusion follows.

4. Show that if a, b, c are real numbers any two of which do not sum tozero, then

a5 + b5 + c5 − (a+ b+ c)5

a3 + b3 + c3 − (a+ b+ c)3≥ 10

9(a+ b+ c)2.

Solution: We already know that

a3 + b3 + c3 − (a+ b+ c)3 = −3(a+ b)(b+ c)(c+ a),

14

Page 16: Олимпиадад бэлтгэгчдэд тусламж (Тэнцэтгэл биш 1)

and we establish an analogous identity for degree five:

a5+b5+c5−(a+b+c)5 = 5(a+b)(b+c)(c+a)(a2+b2+c2+ab+bc+ca)

(prove it!). Hence the ratio is

5

3(a2 + b2 + c2 + ab+ bc+ ca) =

5

6(a2 + b2 + c2) +

5

6(a+ b+ c)2

≥ 5

18(a+ b+ c)2 +

5

6(a+ b+ c)2

=10

9(a+ b+ c)2,

as desired.

5. If a is the smallest of the positive reals a, b, c then show that

a3 + b3 + c3 − 3abc ≥ 2

(b+ c

2− a)3

.

Solution: This factorizes as

2(a+ b+ c)[(a− b)2 + (b− c)2 + (c− a)2

]≥ [(b− a) + (c− a)]3 .

If we denote x = b− a, y = c− a then a+ b+ c > x+ y and

(a− b)2 + (b− c)2 + (c− a)2 ≥ x2 + y2

and so the inequality follows from 2(x+ y)(x2 + y2) ≥ (x+ y)3 which bycancelling common terms turns to x3 + y3 ≥ x2y + xy2 which is true.

6. Let x, y, z be positive numbers such that x2 + y2 + z2 = 1. Determine

the minimum value ofxy

z+xz

y+yz

x.

Solution: We havexy

z+xz

y+yz

x=x2y2 + y2z2 + z2x2

xyz.

Now we use the famous inequality (a+b+c)2 ≥ 3(ab+bc+ca) for a = uv,b = vw, c = uw, deducing that (uv + uw + vw)2 ≥ 3(u + v + w)uvw.Applying this for u = x2, v = y2, w = z2, we deduce that

(x2y2 + z2x2 + y2z2)2 ≥ 3x2y2z2(x2 + y2 + z2).

Taking square root and keeping in mind that x2 + y2 + z2 = 1 we have

x2y2 + y2z2 + z2x2 ≥√

3xyz.

The value√

3 can be achieved, when x = y = z =1√3

.

7. Let 0 < x < y < z < t such that x+t = y+z. Show that for any naturalnumber n ≥ 2, xn + tn > yn + zn.

Solution: Let a = y − x = t− z. We rewrite the conclusion as

(x+ a)n − xn < (z + a)n − zn,

15

Page 17: Олимпиадад бэлтгэгчдэд тусламж (Тэнцэтгэл биш 1)

which by Newton Binomial Theorem is equivalent to

nxn−1a+n(n− 1)

2xn−2a+ . . .+an < nzn−1a+

n(n− 1)

2zn−2a+ . . .+an

which is clearly true from z > x.

8. Let a, b, c be length of the sides of triangle; and let t ≥ 1 be real number.Prove that:

(t+ 1)

(a2

c+b2

a+c2

b

)≥ b2

c+c2

a+a2

b+ t(a+ b+ c).

Solution: It suffices to prove it for t = 1 since

a2

c+b2

a+c2

b≥ a+ b+ c.

This latter inequality can be proven in many ways. A simple way is tonote that a2−c(2a−b) = (a−c)2 ≥ 0 hence c(2a−c) ≤ a2 so a2

c ≥ 2a−cand similarly b2

a ≤ 2b−a, c2b ≥ 2c−b and we sum these three inequalitiesto obtain the desired result.

So it suffices to show 2(a2

c + b2

a + c2

b ) ≥ c2

b + b2

c + c2

a + a + b + c. Wemultiply this latter inequality by abc to turn it into the equivalent form2(a3b+b3c+c3a) ≥ ab3+bc3+ca3+abc(a+b+c). This can be rewrittenas ab3 + bc3 + ca3− a3b− b3c− c3a ≤ a3b+ b3c+ c3a− abc(a+ b+ c) i.e.

(a− b)(b− c)(c− a)(a+ b+ c) ≤ a3b+ b3c+ c3a− abc(a+ b+ c).

The right-hand side is non-negative since we’ve just shown that a2

c + b2

a +c2

b ≥ a+b+c, thus is suffices to consider the case (a−b)(b−c)(c−a) > 0.

In this case we can suppose a < b < c (check!) and so the RHS is

b(b2 − a2)(c− a) + a(c− b)(c2 − b2)

By the AM-GM inequality this is greater than or equal to

2(c− b)√ab(c− b)(b− a)(c+ b)(b+ a)

and so by squaring both sides and canceling reducing the common factorsit remains to prove that

4ab(c+ b)(b+ a) ≥ (c− b)(b− a)(a+ b+ c)2

Indeed, a ≥ c− b, b ≥ b− a, 4(c+ b)(b+ a)− (a+ b+ c)2 = 4c(b+ a)−(a + b + c)2 + 4b(b + a) = 4b(b + a) − (a + b − c)2 ≥ 4b(b + a) − a2 > 0and by multiplying we get the result.

16

Page 18: Олимпиадад бэлтгэгчдэд тусламж (Тэнцэтгэл биш 1)

1.4 The mathematical induction

The Principle of Mathematical Induction is formulated as follows:

Suppose we have to prove an affirmation A(n) depending on n ∈ N.

If we can prove it for some k (the basis), and then we can prove that A(n+ 1)(the induction step) follows from (the induction assumption) A(n) then A(n)is true for any n.

Another variation is to show that A(n+1) follows from A(k), A(k+1), . . . A(n)- this is called ”strong induction” as opposed to the ”weak induction” methodabove. Of course, strong induction become weak induction one we replace thepropositions A(n) with the equivalent set of propositions B(n) that state thatA(k), A(k + 1), . . . , A(n) are all true.

Consider for example Bernoulli’s Inequality

If α ≥ 0 then (1 + α)n ≥ 1 + nα.

For n = 1 this is actually an identity (the basis).

Now if (1 + α)n ≥ 1 + nα (the induction assumption) then

(1+α)n+1 = (1+α)n(1+α) ≥ (1+nα)(1+α) = 1+(n+1)α+α2 ≥ 1+(n+1)α

(The induction step). Hence the affirmation is true for all n.

We may have noted that the inequality a3 + b3 + c3 ≥ 3abc resembles theAM-GM inequality, generalized for three numbers. Indeed, if we put a3 =

x, b3 = y, c3 = z we transform it tox+ y + z

3≥ 3√xyz.

A natural question appears: is this inequality true for any number of variables,

i.e., is it true that for positive x1, x2, . . . , xn we havex1 + x2 + . . .+ xn

n≥

n√x1x2 . . . xn?

We know it’s true for n = 4, and we know how to deduce it from the two-variables case:

a+ b+ c+ d ≥ 2√ab+ 2

√cd ≥ 4

√√ab√cd = 4

4√abcd.

We can prove the inequality for n = 2m the very same way using induction:

x1 + . . .+ x2m = (x1 + . . .+ x2m−1) + (x2m−1+1 + . . .+ x2m) ≥

≥ 2m−1 2m−1√x1x2 . . . x2m−1 + 2m−1 2m−1√x2m−1+1 . . . x2m .

Applying the AM-GM Inequality for two numbers, we deduce this quantity isat greater or equal than 2m 2m

√x1x2 . . . x2m . So, we can prove the inequality for

any power of 2. To pass to the general case, pick up such a m with 2m > n andconsider the system b1, b2, . . . , b2m with bi = ai if i ≤ n and bi = n

√a1a2 . . . an

otherwise. The geometric mean of the system is still n√a1a2 . . . an and so we

deduce

b1 + b2 + . . .+ b2m ≥ 2m n√a1a2 . . . an

so

a1 + a2 + . . .+ an + (2m − n) n√a1a2 . . . an ≥ 2m n

√a1a2 . . . an

17

Page 19: Олимпиадад бэлтгэгчдэд тусламж (Тэнцэтгэл биш 1)

soa1 + a2 + . . .+ an

n≥ n√a1a2 . . . an.

This inequality is called the general Arithmetic Mean-Geometric Mean (AM-GM) Inequality.

The method we used is called Cauchy Induction, which is based on provingthe inequality for an infinite class of numbers (in our case powers of two) anthen showing that if m < n and for n the proposition is true, then for m italso must be true. In a simpler way, if for n the assertion holds, then it holdsfor n− 1.

The mathematical induction is a tool that can be used in inequalities withmany variables. Sometimes the induction step is quite clear, but it can not beproven, In this case it’s good to sharpen the induction assumption.

Look at the following problem:

1

2· 3

4· . . . · 2n− 1

2n<

1√3n.

The induction seems obvious: we prove the basis and then we show that fromn−1 to n, the LHS increases less than RHS. This is wrong, however: we need to

prove2n− 1

2n<

√n− 1√n

. Squaring this is equivalent to (2n−1)2n < 4n2(n−1)

or 4n3 − 4n2 + n < 4n3 − 4n2, which is false. In this case, sharpening the

assumption saves us: we show that actually1

2· 3

4. . . · 2n− 1

2n≤ 1√

3n+ 1.

The basis still holds, and the inductive step can now be performed, as it is

equivalent to2n− 1

2n≤√

3n− 2√3n+ 1

or (2n − 1)2(3n + 1) ≤ (2n)2(3n − 2) or

12n3 − 12n2 − n− 1 ≤ 12n3 − 8n2, true.

Exercises

1. Show that 2(√n+ 1− 1) ≤ 1 +

1√2

+ . . .+1√n< 2√n.

Solution: The basis is easy to check. The induction step is to prove

2(√n+ 1−

√n) <

1√n< 2(√n−√n− 1).

These inequalities transform to 2[√

n(n+ 1)− n]< 1 < 2

[n−

√n(n− 1)

]or√n(n+ 1) < n+

1

2and

√n(n− 1) < n− 1

2which follow by squaring.

2. Show that xn =1

n+ 1+

1

n+ 2+ . . .+

1

2n<

7

10.

Solution: As xn+1−xn =1

2n+ 1+

1

2n+ 2− 1

n+ 1=

1

(2n+ 1)(2n+ 2)we

cannot apply induction in the most trivial way. So we need to strengthen

18

Page 20: Олимпиадад бэлтгэгчдэд тусламж (Тэнцэтгэл биш 1)

somehow the condition. We might replace xn by xn +k

n. For yn to be

decreasing we need to check

yn − yn+1 =1

(2n+ 1)(2n+ 2)− k

n(n+ 1)=

(1− 4k)n− 2k

2n(n+ 1)(2n+ 1)< 0,

which holds for k =1

4. For this new sequence, the induction step is

already proven and the basis holds from n = 4.

3. Assume that x1, x2, . . . , xn, y1, y2, . . . , yn are positive numbers with

y1 ≤ y2 ≤ . . . ≤ yn andx1y1≥ x2y2≥ . . . ≥ xn

yn.

Then show that

x1y1

+ . . .+xnyn≥ n · x1 + x2 + . . .+ xn

y1 + y2 + . . .+ yn.

Solution: It’s natural to denote zi =xiyi

.

For n = 2 the inequality isx1y1

+x2y2≥ 2 · x1 + x2

y1 + y2or

x1y2(y1 + y2) + x2y1(y1 + y2) ≥ 2(x1 + x2)y1y2

or x1y22 +x2y

21 ≥ (x1 +x2)(y1y2) or (y2− y1)

(x1y1− x2y2

)y1y2 ≥ 0. Note

that for n = 2 this is a necessary and sufficient condition that y2 − y1and

x1y1− x2y2

have the same sign.

Now let’s perform the induction step. By the induction assumption

x1y1

+ . . .+xnyn≥ x1y1

+ (n− 1) · x2 + . . .+ xny2 + . . .+ yn

.

So we are left to prove that

x1y1

+ (n− 1) · x2 + . . .+ xny2 + . . .+ yn

≥ n · x1 + x2 + . . .+ xny1 + y2 + . . .+ yn

,

which is in turn equivalent to

x1(y2+. . .+yn)(y1+y2+. . .+yn)+(n−1)(x2+. . .+xn)(y1+y2+. . .+yn)y1 ≥

≥ n(x1 + . . .+ xn)y1(y2 + . . .+ yn).

If we denote x =x2 + . . .+ xn

n− 1, y =

y2 + . . .+ ynn− 1

then y1 ≤ y and

x

y≤ x1y1

and the inequality to prove rewrites as

x1(n− 1)y [y1 + (n− 1)y] + (n− 1)(n− 1)x [y1 + (n− 1)y] y1 ≥

≥ n(x1 + (n− 1)x)(n− 1)yy1,

19

Page 21: Олимпиадад бэлтгэгчдэд тусламж (Тэнцэтгэл биш 1)

or x1y [y1 + (n− 1)y]+(n−1)xy1 [y1 + (n− 1)y] ≥ n [x1 + (n− 1)x] yy1.

By cancelling the common terms we get

−(n− 1)x1y1y + (n− 1)y2x1 + (n− 1)y21x− (n− 1)xyy1 ≥ 0

or (n− 1)(y − y1)(x1y1− x

y

)yy1 ≥ 0, true.

4. Show that1

22+

1

32+ . . .+

1

n2< 1.

Solution: The sequence is increasing so we must sharpen the condition.

1

22+ . . . +

1

n2< 1 − 1

nis good, because

1

(n+ 1)2<

1

n− 1

n+ 1. (Note

that this inequality can give us a direct proof without induction).

5. Show that if x1, x2, . . . , xn ∈(0, 12)

then

x1x2 . . . xn(x1 + x2 + . . .+ xn)n

≤ (1− x1) . . . (1− xn)

((1− x1) + . . .+ (1− xn))n.

Solution: This is an application for Cauchy induction. Firstly, we provethe inequality by induction for n = 2k. If n = 2 we must prove

x1x2(x1 + x2)2

≤ (1− x1)(1− x2)(2− x1 − x2)2

,

or x1x2[4− 4(x1 + x2) + (x1 + x2)

2]≤ (x1+x2)

2 [1− (x1 + x2) + x1x2]which turns to 4(x1 − x2)2(1− x1 − x2) ≥ 0 which is true.

To pass from n to 2n we see that

x1x2 . . . xnxn+1 . . . x2n(x1 + . . .+ xn + xn+1 + . . .+ x2n)2n

=

=x1x2 . . . xn

(x1 + x2 + . . .+ xn)nxn+1 . . . x2n

(xn+1 + . . .+ x2n)n·

· (x1 + . . .+ xn)n(xn+1 + . . .+ x2n)n

(x1 + . . .+ xn + xn+1 + . . .+ x2n)2n

=x1x2 . . . xn

(x1 + x2 + . . .+ xn)nxn+1 . . . x2n

(xn+1 + . . .+ x2n)n·

·

x1 + . . .+ xn

n

xn+1 + . . .+ x2nn(

x1 + . . .+ xnn

+xn+1 + . . .+ x2n

n

)2

n

≤ (1− x1) . . . (1− xn)

[(1− x1) + . . .+ (1− xn)]n(1− xn+1) . . . (1− x2n)

[(1− xn+1) + . . .+ (1− x2n)]n·

·

(

1− x1 + . . .+ xnn

)(1− xn+1 + . . .+ x2n

n

)[(

1− x1 + . . .+ xnn

)+

(1− xn+1 + . . .+ x2n

2n

)]2n

=(1− x1) . . . (1− x2n)

[(1− x1) + . . .+ (1− x2n)]2n.

20

Page 22: Олимпиадад бэлтгэгчдэд тусламж (Тэнцэтгэл биш 1)

Now the backward step: assume that our assumption holds for n and

let’s prove it for n − 1. To do this, take xn =x1 + . . .+ xn−1

n− 1(in such

exercises it’s convenient to set the ”free” variable equal to some mean ofthe other variables).Then

x1x2 . . . xn−1x1 + . . .+ xn−1

n− 1(x1 + x2 + . . .+ xn−1 +

x1 + . . .+ xn−1n− 1

)n ≤

≤(1− x1)(1− x2) . . . (1− xm−1)(1−

x1 + . . .+ xn−1n− 1

)[(1− x1) + . . .+ (1− xn−1) +

(1− x1 + . . .+ xn−1

n− 1

)]n ,which after reducing the common terms in the numerators and denomi-nators becomes exactly what we wanted.

6. Let ai ≥ 1 and |ai − ai+1| < 1 for 1 ≤ i ≤ i− 1. Show that

a1a2

+a2a3

+ . . .+an−1an

+ana1

< 2n− 1.

Solution: The idea is to use induction on n. To do this, we must deletea variable and show that the LHS decreases by at most 2. However weneed to satisfy the inequality |ai − ai+1| < 1 which can be fulfilled onlyif we remove an−1. Now if we remove an−1 from the sequence, the newsequence a1, a2, . . . , an−2, an consists of n− 1 terms. Moreover the RHShas changed by

an−2an− an−1

an− an−2an−1

=an−2an−1 − a2n−1 − an−2

an−1an

= −2 +an−1an − (an−1 − an−2)(an−1 − an)

anan−1> −2.

so the induction step is fulfilled. For n = 1 the condition clearly holds(with equality).

7. Show that for any n ≥ 4, x1, x2, . . . , xn ∈ R+ we have

(x1 + x2 + . . .+ xn)2 ≥ 4(x1x2 + x2x3 + . . .+ xn−1xn + xnx1).

Solution: For n = 4 this is (x1 + x3 − x2 − x4)2 ≥ 0.

Now let n ≥ 5 and denote

f(x1, x2, . . . , xn) = (x1 +x2 + . . .+xn)2−4(x1x2 + . . .+xn−1xn+xnx1).

Then f(x1 +x2, x3, x4, . . . , xn) = f(x1, x2, . . . , xn) +x3x1 +x2xn−x1x2.Now if x3 ≥ x2 (which we may assume as the inequality is cyclic) we get

f(x1 + x2, x3, x4, . . . , xn) ≥ f(x1, x2, . . . , xn)

and so we are done by induction.

21

Page 23: Олимпиадад бэлтгэгчдэд тусламж (Тэнцэтгэл биш 1)

8. Let a1, a2, . . . , an ∈ (0, 1) with a1a2 . . . an = An. Show that

1

1 + a1+ . . .+

1

1 + an≤ n

1 +A.

Solution: If n = 2 the inequality is equivalent to1

1 + x2+

1

1 + y2≤

2

1 + xyor by clearing denominators to (2+x2+y2)(1+xy)−2(1+x2)(1+

y2) ≤ 0 or (x− y)2(xy − 1) ≤ 0, which is true. Now set b =anan+1

A.

Then a1a2 . . . an−1b = An so by induction assumption we have

1

1 + a1+ . . .+

1

1 + an−1+

1

1 + b≤ n

1 +A

and we are left to prove that

1

1 + a1+. . .+

1

1 + an−1+

1

1 + b+

1

1 +A≥ 1

1 + a1+. . .+

1

1 + an−1+

1

1 + an+

1

1 + an+1,

or1

1 + an+

1

1 + an+1≤ 1

1 +A+

A

A+ anan+1.

By clearing denominators this reduce to (an−A)(an+1−A)(1−anan+1) ≤0. As an, an+1 < 1 this inequality will be fulfilled iff one of an, an+1 isgreater than A ad the other is smaller (or equal). But we can clearlyassume this as the inequality is cyclic and we can’t have all ai greaterthan A or all ai smaller than A.

9. Let S(n) = 11+22+. . .+nn. Show that1

nn>

1

S(n+ 1)+

1

S(n+ 2)+. . ..

Solution: The quickest idea that comes to our mind is to use ”induction”,

i.e. prove that1

S(n)<

1

nn− 1

(n+ 1)n+1. (Actually this is not induction

but telescoping sums, however it resembles induction). Surprisingly itworks! (on difficult exams, it won’t). We need to prove that

(n+ 1)n+1 − nn

nn × (n+ 1)n+1>

1

11 + 22 + . . .+ nn.

Rewrite this as(n+ 1)n+1 − nn

(n+ 1)n+1· 1 + 22 + . . .+ nn

nn> 1.

To compare nn with (n+1)n+1, we may use

(n+ 1

n

)n< e <

(n+ 1

n

)n+1

.

Therefore, as(n+ 1)n+1

nn= (n+1)

(n+ 1

n

)n= n

(n+ 1

n

)n+1

, we have

ne <(n+ 1)n+1

nn< (n+ 1)e.

So

(n+ 1)n+1 − nn

(n+ 1)n+1·1 + 22 + . . .+ nn

nn>

(1− 1

ne

)[1 +

1

ne+

1

n(n− 1)e2+ . . .

].

22

Page 24: Олимпиадад бэлтгэгчдэд тусламж (Тэнцэтгэл биш 1)

Since(1− 1

ne

)[1 +

1

ne+

1

n(n− 1)e2

]= 1− 1

n2e2+

1

n(n− 1)e2− 1

n2(n− 1)e3

= 1 +1

n2(n− 1)e2− 1

n2(n− 1)e3> 1.

And we are done.

10. If a1 ≥ a2 ≥ . . . ≥ an ≥ 0 then a21 − a22 + a23 ± . . .± a2n ≥ (a1 − a2 + a3 ±. . .± an)2

Solution: Again we want to use induction by deleting some of the vari-ables. Since we don’t really know the sign of an, it’s probably betterto start from a1. However deleting it produces an expression of oppo-site sign and applying induction to it would give an inequality also ofopposite sign, not helpful to us. So we should delete the first terms,a1, a2.

Indeed, by the induction assumption for n− 2 we can deduce that

a21 − a22 + a23 ± . . .± a2n ≥ a21 − a22 + (a3 − a4 + a5 ± . . .± an)2.

If we denote by t = a2 − a3 ± . . .± an then we must prove that

a21 − a22 + t2 ≥ (a1 − a2 + a3)2.

It resembles our inequality for n = 3 and it is indeed our inequality forn = 3 because t = (a3−a4)+ . . . = a3−(a4−a5)− . . . so 0 ≤ t ≤ a3 ≤ a2.Therefore it suffices to prove the basis, which consists of two separatecases: n = 2 and n = 3 (this is because we did an unusual inductionstep of 2 - if you want, we do induction on [n2 ]). For n = 2 we mustprove (a1 − a2)(a1 + a2) ≥ (a1 − a2)

2 which is clear. For n = 3 wemust prove a21 − a22 + a23 − (a1 − a2 + a3)

2 ≥ 0 which is equivalent to2(a1 − a2)(a2 − a3) ≥ 0, true!

11. Let x1 = 1, xn+1 =xnn

+n

xn, for n ≥ 1. Prove that the sequence (xn) is

increasing and that [x2n] = n for n ≥ 4

Solution: To perform the induction step, we will prove a stronger hy-

pothesis, that for n ≥ 3 we have√n ≤ xn ≤

n√n− 1

.

We may note that f(x) =x

n+n

xis decreasing for x ≤ n so it suffices to

prove that f(√n) ≤ n+ 1√

nand f

(n√n− 1

)≥√n+ 1. We have

xn+1 ≥ f(

n√n− 1

)=√n− 1 +

1√n− 1

=n√n− 1

>√n+ 1.

Replacing n+ 1 by n we get xn ≥n− 1√n− 2

. Therefore

xn+1 ≤ f(n− 1√n− 2

)=n2(n− 2) + (n− 1)2

n(n− 1)√n− 2

.

23

Page 25: Олимпиадад бэлтгэгчдэд тусламж (Тэнцэтгэл биш 1)

So we are left to prove thatn2(n− 2) + (n− 1)2

n(n− 1)√n− 2

<√n+ 2 which is

equivalent ton3 − n2 − 2n+ 1

n2 − n<√n2 − 4 which after squaring is equiv-

alent to 2n2(n − 3) + 4n − 1 ≥ 0, true for n ≥ 4. We thus deducen < x2n < n+ 1 so [x2n] = n.

12. Let n > 2. Find the least constant k such that for any a1, . . . , an > 0with product 1 we have

a1a2(a21 + a2)(a22 + a1)

+a2a3

(a22 + a3)(a23 + a2)+ . . .+

ana1(a2n + a1)(a21 + an)

≤ k.

Solution: We prove the answer is n− 2. If we put a1 = . . . = an−1 = xtending to 0 we get close to n− 2. Now to prove another part: Firstly

(a21 + a2)(a22 + a1) ≥ (a21 + a1)(a

22 + a2)

(this is easily deduced by opening the brackets). So we replace to prove

n∑i=1

1

(ai + 1)(ai+1 + 1)≤ n− 2.

For n = 3 it’s simple replacing a1 =x

y, a2 =

y

z, a3 =

z

x:

yz

(x+ y)(x+ z)+

zx

(y + z)(y + z)+

xy

(z + x)(z + y)<

<xy

xy + yz + zx+

yz

xy + yz + zx+

xz

xy + yz + zx= 1.

For n > 3 we prove by induction: If a1a2 ≤ 1 and a2a3 ≥ 1 (we mayassume this since the inequality is cyclic) we can replace a2 and a3 witha2a3 and prove the value decreases by at most 1 so we can use inductionstep. Indeed, the value decreases by

1

(a1 + 1)(a2 + 1)+

1

(a2 + 1)(a3 + 1)+

1

(a3 + 1)(a4 + 1)−

− 1

(a1 + 1)(a2a3 + 1)− 1

(a2a3 + 1)(a4 + 1)=

=a2(a3 − 1)

(a1 + 1)(a2 + 1)(a2a3 + 1)+

a3(a2 − 1)

(a4 + 1)(a3 + 1)(a2a3 + 1)+1− a2 + a3 + a2a3

(a2 + 1)(a3 + 1) + 1.

Now if a2 ≤ 1, a3 ≥ 1 then this quantity is at most

1 +a2a3

a2a3 + 1− (a2 + 1)(a3 + 1)− 1

(a2 + 1)(a3 + 1)≤

≤ 1 + 1− 1

a2a3 + 1− 1 +

1

a2a3 + a2 + a3 + 1≤ 1,

24

Page 26: Олимпиадад бэлтгэгчдэд тусламж (Тэнцэтгэл биш 1)

and analogously if a2 ≥ 1, a3 ≤ 1. Finally if a2, a3 > 1 then the quantityis at most

1 +a2a3

2(a2a3 + 1)+

a2a32(a2a3 + 1)

− a2a3 + a2 + a3(a2 + 1)(a3) + 1

=

= 1 +a2a3

a2a3 + 1− a2a3 + a2 + a3a2a3 + a2 + a3 + 1

≤ 1

again.

1.5 The AM-GM inequality

The inequality proven in the previous chapter:

a1 + a2 + . . .+ ann

≥ n√a1a2 . . . an

is the most important of all the inequalities used, and has enormously manyapplications.Here’s one of them:If ai, bi > 0 then

n∑i=1

aiai + bi

≥ n ·

n

√n∏i=1ai

n

√n∏i=1

(ai + bi)

and also

n∑i=1

biai + bi

≥ n ·

n

√n∏i=1bi

n

√n∏i=1

(ai + bi)

Summing these two inequalities and using the fact thatai

ai + bi+

biai + bi

= 1

we deduce

n ≥ n ·

n

√n∏i=1ai + n

√n∏i=1bi

n

√n∏i=1

(ai + bi)

or

n

√√√√ n∏i=1

(ai + bi) ≥ n

√√√√ n∏i=1

ai + n

√√√√ n∏i=1

bi

This is called Huygens Inequality.If we have the sequence a1, a1, . . . , a1, a2, . . . , a2, . . . , an, . . . , an, where ai re-peats pi times then according to AM-GM we have

p1a1 + p2a2 + . . .+ pnan ≥ (p1 + p2 + . . .+ pn) p1+p2+...+pn

√ap11 . . . apnn .

25

Page 27: Олимпиадад бэлтгэгчдэд тусламж (Тэнцэтгэл биш 1)

If we denote αi =pi

p1 + p2 + . . .+ pn, we obtain the following generalization

of AM-GM:

If α1, . . . , αn are positive numbers that sum to 1 then for any positive x1, x2, . . . , xnwe have α1x1 + . . .+ αnxn ≥ xα1

1 . . . xαnn

This is called Weighted AM-GM. Of course, in this setting αi need to berational - but in fact this condition is unnecessary.

Exercises

1. Show that for any a1, a2, . . . , an > 0 we have (a1+. . .+an)

(1

a1+ . . .+

1

an

)≥

n2.

Solution: We have a1 + a2 + . . .+ an ≥ n n√a1a2 . . . an and

1

a1+

1

a2+ . . .+

1

an≥ n n

√1

a1a2 . . . an

and by multiplying these two relations we get the result.

2. Let a1, a2, . . . , an > 0 with a1a2 . . . an = 1. Let sk = ak1 + ak2 + . . .+ akn.

a) Show that if k ≥ l ≥ 0 then sk ≥ sl;

b) Find all k > 0 for which sk ≥ s−1.

Solution: a) We have aki +k − ll≥ k

lali by weighted AM-GM. The

inequality now follows by summing, as sk ≥ n = s0 by AM-GM

b) We notice that sn−1 ≥ s−1 because

xn−12 + xn−13 + . . .+ xn−1n ≥ (n− 1)x2x3 . . . xn =n− 1

x1

and the inequality follows by summing. So using a) we get sk ≥ s−1for any k ≥ n − 1. If k < n − 1 then taking x1 = x2 = . . . = xn−1 =

x, xn =1

xn−1we have sk = (n−1)xk +

1

x(n−1)k, s−1 =

n− 1

x+xn−1 and

s−1 ≥ sk for all sufficiently big x. So k ∈ [n− 1,+∞).

3. Show that n√

(n+ 1)! > n√n! + 1

Solution: By Huygens Inequality

n√

(n+ 1)! = n√

(1 + 1)(2 + 1)(3 + 1) . . . (n+ 1) ≥ n√

1 · 2 . . . · n+1 =n√n!+1.

The equality cannot take place.

4. a) Show that

(1 +

1

n

)n<

(1 +

1

n+ 1

)n+1

;

b) Show that

(1 +

1

n

)n+1

>

(1 +

1

n+ 1

)n+2

.

26

Page 28: Олимпиадад бэлтгэгчдэд тусламж (Тэнцэтгэл биш 1)

Solution: a) By AM-GM we have n + 2 = 1 +

(1 +

1

n

)+

(1 +

1

n

)+

. . .+

(1 +

1

n

)(1+

1

nmeets n times) is bigger than n+1 n+1

√(1 +

1

n

)n,

which rising to power n+ 1 gives us exactly the conclusion.

b) By AM-GM we have n + 1 = 1 +n

n+ 1+ . . . +

n

n+ 1≥ (n +

2)n+2

√(n

n+ 1

)n+1

which after rising to power n + 2 yields the con-

clusion.

5. 2(a4 + b4) + 17 > 16ab.

Solution: 2(a4 + b4) + 17 ≥ 4a2b2 + 17 > 4a2b2 + 16 ≥ 16ab by AM-GM.

6. n n√a1a2 . . . an − (n− 1) n−1

√a1a2 . . . an−1 ≤ an, for ai > 0.

Solution: The conclusion from AM-GM applied to the n numbers an,n−1√a1a2 . . . an−1, n−1

√a1a2 . . . an−1, . . ., n−1

√a1a2 . . . an−1.

7. Show that n

(1− 1

n√n

)+ 1 > 1 +

1

2+ . . . +

1

n> n

(n√n+ 1− 1

), if

n ∈ N∗.

Solution: By AM-GM we have 1 +1

2+

2

3+ . . .+

n− 1

n≥ n 1

n√n

, or

n−(

1

2+

1

3+ . . .+

1

n

)≥ n 1

n√n,

thus n(1− 1n√n

) ≥ 1

2+

1

3+ . . .+

1

n, from where left inequality follows.

For the right inequality, we see that 2 +3

2+

4

3+ . . .+

n+ 1

n≥ n n√n+ 1,

so n+

(1 +

1

2+ . . .+

1

n

)≥ n n√n+ 1, and the rest is clear.

8. Let x1, x2, . . . , x2n be positive reals that sum to 1. Let

S = x21x22 . . . x

2n + x22x

23 . . . x

2n+1 + . . .+ x22nx

21 . . . x

2n−1.

Show that S <1

n2n.

Solution: We can notice that

S ≤ (x21 + x2n+1) . . . (x2n + x22n) ≤ (x1 + xn+1)

2 . . . (xn + x2n)2

≤(x1 + xn+1 + . . .+ xn + x2n

nn

)2

=1

n2n.

The inequality cannot hold in all cases.

9. If a, b, c > 0 then

√a

b+ c+

√b

a+ c+

√c

a+ b> 2.

27

Page 29: Олимпиадад бэлтгэгчдэд тусламж (Тэнцэтгэл биш 1)

Solution: This time we shall use the inequality between the geometric

and harmonic means:

√a

b+ c=

√a

(b+ c) · 1≥ 2

b+ c

a+ 1

=2a

a+ b+ c,

and the inequality now follows by summation.

10. For a, b, c > 0 we have

a√(a+ b)(a+ c)

+b√

(b+ a)(b+ c)+

c√(c+ a)(c+ b)

≤ 3

2.

Solution: By AM-GM we have

a√(a+ b)(a+ c)

=

√a

a+ b

a

a+ c≤ 1

2

(a

a+ b+

a

a+ c

)and summing with the analogous results we get the conclusion.

11. For x, y, z are positive numbers such that x+ x2 + y + y2 + z + z2 = 6,find the maximum of x2y + y2z + z2x

Solution: We see that

x3 + y3 + z3 + xy2 + yz2 + zx2 = (x3 + xy2) + (y3 + yz2) + (z3 + zx2)

≥ 2(x2y + y2z + z2x).

This means x2y + y2z + z2x ≤ (x+y+z)(x2+y2+z2)3 = a(6−a)

3 ≤ 3. Theequality can hold for x = y = z = 1. So the maximum is 3.

12. If a1, a2, . . . , an > 0 then

1

a1+

2

a1 + a2+ . . .+

n

a1 + a2 + . . .+ an< 2

(1

a1+

1

a2+ . . .+

1

an

).

Solution: We havek

a1 + a2 + . . .+ ak=

k

a1 +a22

+a22

+a33

+ . . .+akk

(so

we get 1 + 2 + . . .+ k =k(k + 1)

2terms at the denominator). In virtue

of exercise 1, it’s smaller than

4k

k2(k + 1)2

(1

a1+

2

a2+

2

a2+ . . .+

k

ak

)=

4

k(k + 1)2

(1

a1+

4

a2+

9

a3+ . . .+

k2

ak

).

By summing all these expressions, we see that this sum is smaller than

4

a1

(1

4+

1

18+ . . .

)+

4

a2

(4

18+ . . .

)+ . . . .

So we need to prove 4m2

(1

m(m+ 1)2+ . . .

)< 2. But we can prove

that1

m(m+ 1)2<

1

2

[1

m2− 1

(m+ 1)2

]and hence by telescoping

4m2

[1

m(m+ 1)2+ . . .

]< 4m2 · 1

2m2= 2

and we are done.

28

Page 30: Олимпиадад бэлтгэгчдэд тусламж (Тэнцэтгэл биш 1)

13. If a1, a2, . . . , an > 0 then

a1 +√a1a2 + 3

√a1a2a3 + . . .+ n

√a1a2 . . . an < 3(a1 + a2 + . . .+ an).

Solution: As in the above exercise, we assign ”weights” to ai. We have

k√a1a2 . . . ak =

1k√k!

k√a1 × 2a2 × . . .× kak ≤

1

k k√k!

(a1+2a2+. . .+kak).

After summation, the term alongside am will be

m

[1

m√m!m

+1

m+1√

(m+ 1)!(m+ 1)+ . . .+

1n√n!n

].

Now we prove that k√k! ≥ k + 1

3which will imply

m

[1

m√m!m

+1

m+1√

(m+ 1)!(m+ 1)+ . . .+

1n√n!n

]≤

≤ m

[3

m(m+ 1)+

3

(m+ 1)(m+ 2)+ . . .+

3

n(n+ 1)

]= m

(3

m− 3

m+ 1+

3

m+ 1− 3

m+ 2+ . . .+

3

n− 3

n+ 1

)= m

(3

m− 3

n+ 1

)= 3− 3m

n+ 1< 3

so our initial assertion. The inequality k√k! ≥ k + 1

3is proven by induc-

tion on k. For k ≤ 7 this is proven manually. The induction step followsfrom the fact that(

k + 2

3

)k+1

(k + 1

3

)k =k + 2

3

(k + 1

k

)k+1

<k + 2

3

(8

7

)8

< k + 1 =(k + 1)!

k!.

1.6 The quadratic trinomial

Consider a second-degree equation

ax2 + bx+ c = 0

Its discriminant isD = b2 − 4ac.

If D > 0, this equation has two different solutions, hence it changes sign.If D < 0, this equation has no solutions, so it has the same sign for all x. Thissign is the sign of a. Thus, we can enounce the following principle:

a) For a > 0, we have ax2 + bx+ c ≥ 0 for any x if and only if b2 ≤ 4ac.b) For a < 0, we have ax2 + bx+ c ≤ 0 for any x if and only if b2 ≤ 4ac.

29

Page 31: Олимпиадад бэлтгэгчдэд тусламж (Тэнцэтгэл биш 1)

We can also make the inequalities strict:

a) For a > 0, we have ax2 + bx+ c > 0 for any x if and only if b2 < 4ac.b) For a < 0, we have ax2 + bx+ c < 0 for any x if and only if b2 < 4ac.

This principle can be used for solving many problems or deducing theorems.

Consider the quadratic trinomial f(x) =

n∑i=1

(aix+ bi)2. It is always positive,

and hence must have non-positive discriminant. However, by opening thebrackets we see that

f(x) =n∑i=1

a2ix2 + 2

(n∑i=1

aibi

)x+

n∑i=1

b2i

and so the discriminant is

4

(n∑i=1

aibi

)2

− 4

(n∑i=1

a2i

)(n∑i=1

b2i

).

Therefore we have the following inequality (Cauchy-Buniakovsky-Schwartz):For any reals a1, a2, . . . , an, b1, b2, . . . , bn we have(

n∑i=1

a2i

)(n∑i=1

b2i

)≥

(n∑i=1

aibi

)2

.

If we take the equation f(x) = (a1x−b1)2−(a2x−b2)2− . . .−(anx−bn)2 thendoes takes non-positive values. Therefore if the leading coefficient is positive,it must have roots and so the discriminant must be positive, which leads toAczel’s Inequality:If a21 > a22 + . . .+ a2n then

(a1b1 − a2b2 − . . .− anbn)2 ≥ (a21 − a22 − . . .− a2n)(b21 − b22 − . . .− b2n).

If the dominant coefficient of the trinomial f(x) = ax2 + bx + c is positive,

then as f(x) can be written as a

(x+

b

2a

)2

+D

a2, it has its minimal value at

x = − b

2a, called the vertex of the trinomial (or parabola). We can also see

that f is increasing for x > − b

2aand decreasing otherwise. It can be therefore

deduced that a quadratic trinomial with positive dominant coefficient attainsits maximum at the extremities of an interval.For a negative, all properties above change sign.

The last principle deserves more a more detailed discussion. Let us firstlyconsider the case a > 0, then for all x ∈ [t1, t2] , we have

f(x)− f(t1) = a(x2 − t21) + b(x− t1) = (x− t1)(ax+ at1 + b),

f(x)− f(t2) = a(x2 − t22) + b(x− t2) = (x− t2)(ax+ at2 + b).

From this, we have that: if ax+at1 + b ≤ 0 then f(x) ≤ f(t1), and in the caseax + at1 + b ≥ 0, we have ax + at2 + b ≥ ax + at1 + b ≥ 0, and we get that

30

Page 32: Олимпиадад бэлтгэгчдэд тусламж (Тэнцэтгэл биш 1)

f(x) ≤ f(t2). Or in the other words, in any cases, the following inequality isvalid:

f(x) ≤ max {f(t1), f(t2)} ∀x ∈ [t1, t2] .

In the case a < 0, we proceed by the same way as the above for g(x) =−f(x) = −ax2 − bx− c, and we also have that

g(x) ≤ max {f(t1), f(t2)} ∀x ∈ [t1, t2] , or −f(x) ≤ max {−f(t1),−f(t2)}∀x ∈ [t1, t2] , which is equivalent to f(x) ≥ min {f(t1), f(t2)} ∀x ∈ [t1, t2] .

Finally, let us consider the case a = 0, then if b ≥ 0, it is trivial that bx1 + c ≤bx + c ≤ bx2 + c, or f(t1) ≤ f(x) ≤ f(t2). And if b ≤ 0, it is also trivial thatbt2 + c ≤ bx+ c ≤ bt1 + c, or f(t2) ≤ f(x) ≤ f(t1).Therefore, in any cases, wehave

min {f(t1), f(t2)} ≤ f(x) ≤ max {f(t1), f(t2)} ∀x ∈ [t1, t2] .

All these results can be summarized in the following proposition:

Proposition. Consider the following polynomial f(x) = ax2 + bx + c witha, b, c ∈ R and x ∈ [t1, t2] , then

1. If a > 0, then f(x) ≤ max {f(t1), f(t2)} ∀x ∈ [t1, t2] .

2. If a < 0, then f(x) ≥ min {f(t1), f(t2)} ∀x ∈ [t1, t2] .

3. If a = 0, then min {f(x1), f(x2)} ≤ f(x) ≤ max {f(x1), f(x2)} ∀x ∈[t1, t2] .

This theorem provides us a very interesting way to prove inequalities. Forexample, we need to prove that f(x) = ax2 + bx+ c ≤ m for all x ∈ [t1, t2] andwe know that a ≥ 0, then from the above theorem, it suffices to prove thatmax {f(t1), f(t2)} ≤ m. It means that we only need to consider the originalinequality in the cases x = t1 and x = t2 which is not hard to prove. For moreclearly, see the following examples

Example 1. Let a, b, c ∈ [1, 2] , show that a2 + b2 + c2 ≤ 3abc.

Solution: We need to prove that P (a, b, c) = a2− 3abc+ b2 + c2 ≤ 0. This is aquadratic polynomial in term of a with the highest coefficient it positive, there-fore, from the above theorem, we only have to prove that max {P (1, b, c), P (2, b, c)} ≤0. Now, we see that P (1, b, c), P (2, b, c) are also quadratic polynomials in termsof b, hence it suffices to prove the inequality max{max {P (1, 1, c), P (1, 2, c)} ,max {P (2, 1, c), P (2, 2, c)}} ≤0, which is equivalent to max {P (1, 1, c), P (1, 2, c), P (2, 1, c), P (2, 2, c)} ≤ 0.Again, we see that P (1, 1, c), P (1, 2, c), P (2, 1, c), P (2, 2, c) are also quadraticpolynomials in terms of c, hence it suffices to prove the inequality max{P (1, 1, 1),P (1, 1, 2), P (1, 2, 1), P (1, 2, 2), P (2, 1, 1), P (2, 1, 2), P (2, 2, 1), P (2, 2, 2)} ≤ 0.But our inequality is symmetric, therefore we only need to consider the fol-lowing cases

i) If a = b = c = 1, then a2 + b2 + c2 − 3abc = 0.

ii) If a = b = 1, c = 2, then a2 + b2 + c2 − 3abc = 0.

iii) If a = 1, b = c = 2, then a2 + b2 + c2 − 3abc = −3 < 0.

iv) If a = b = c = 2, then a2 + b2 + c2 − 3abc = −12 < 0.

Our proof is complete.

Example 2. If a, b, c ≥ 0 and a+b+c = 1. Prove that 1−4(ab+bc+ca)+9abc ≥0.

31

Page 33: Олимпиадад бэлтгэгчдэд тусламж (Тэнцэтгэл биш 1)

Solution: Put x = ab, then 0 ≤ x ≤ (a+ b)2

4=

(1− c)2

4. And our in-

equality becomes (9c − 4)ab + 1 − 4c(a + b) ≥ 0, or equivalently, f(x) =(9c − 4)x + 1 − 4c + 4c2 ≥ 0. This is a quadratic polynomial with zeroquadratic coefficient, therefore from the above theorem, it suffices to prove

that min

{f(0), f

((1− c)2

4

)}≥ 0. But it is true since f(0) = 1− 4c+ 4c2 =

(1− 2c)2 ≥ 0, and

f

((1− c)2

4

)= (9c− 4) · (1− c)2

4+ 1− 4c+ 4c2 =

1

4c(1− 3c)2 ≥ 0.

Remark. This is Schur’s inequality for third degree.

In the Example 2, if we put P (a, b, c) = 1 − 4(ab + bc + ca) + 9abc, then we

have showed that we only need to prove that min

{f(0), f

((a+ b)2

4

)}≥ 0.

But it is easy to see that

f(0) = P (a+ b, 0, c), f

((a+ b)2

4

)= P

(a+ b

2,a+ b

2, c

).

Therefore, our statement becomes min

{P (a+ b, 0, c), P

(a+ b

2,a+ b

2, c

)}≥

0. From this, we can obtain the following interesting idea: in order to prove theoriginal inequality, it suffices to consider the cases there is one variable equalto 0 or there are two variables equal. This is a very helpful idea in solvinginequalities.

Example 3. Let a, b, c, d be nonnegative real numbers such that a+b+c+d = 4.Show that

a2b2c2 + b2c2d2 + c2d2a2 + d2a2b2 + abc+ bcd+ cda+ dab ≤ 8.

Solution: Put x = ab, then 0 ≤ x ≤ (a+ b)2

4, our inequality becomes

f(x) = (c2 + d2)x2 + (c+ d− 2c2d2)x+ cd(a+ b) + (a+ b)2c2d2 − 8 ≤ 0,

This is a quadratic polynomial of x with the highest coefficient is nonnegative,

therefore, it suffices to prove that max

{f(0), f

((a+ b)2

4

)}≤ 0. It shows

that it suffices to consider the cases a = b or ab = 0. Similarly, we also obtainthat it suffices to consider the cases c = d or cd = 0. Combining both thesestatements, we obtain that, in order to prove the original inequality, it enoughto prove it in the following cases

Case 1. b = 0, d = 0, then the inequality is trivial since

a2b2c2 + b2c2d2 + c2d2a2 + d2a2b2 + abc+ bcd+ cda+ dab = 0.

Case 2. a = b, d = 0, then c = 4 − 2a ≥ 0, and our inequality becomesa4(4 − 2a)2 + a2(4 − 2a) ≤ 8, which is true since by AM-GM inequality, we

32

Page 34: Олимпиадад бэлтгэгчдэд тусламж (Тэнцэтгэл биш 1)

have a2(4−2a) ≤(a+ a+ 4− 2a

3

)3

=64

27, hence a4(4−2a)2+a2(4−2a)−8 ≤

642

272+

64

27− 8 = − 8

729< 0.

Case 3. a = b, c = d, then c = 2 − a ≥ 0, and our inequality becomes2a4(2 − a)2 + 2(2 − a)4a2 + 2a2(2 − a) + 2(2 − a)2a ≤ 8, or equivalent toa6−6a5+14a4−16a3+7a2+2a−2 ≤ 0, which is (a2−2a−1)(a2−2a+2)(a−1)2 ≤0. The last inequality is valid since a ≤ 2.

The property of always attaining the maximal (respectively minimal) valueat the extremities of the interval is not limited to quadratic polynomials - itis characteristic for convex (respectively) concave functions, but we will dealwith these concepts later.

Exercises

1. If x, y, z ∈ R then 3(x2y2 + 1)(z2 − y2 + 1) ≥ 2(x2y2z2 + xyz + 1)

Solution: Grouping by the powers of z we get a quadratic trinomial inz with positive leading coefficient: (x2y2 + 3)z2 − (3x2y2 + 2xy + 3)z +3x2y2 + 1 ≥ 0. Its discriminant can be computed to be −3(xy− 1)4 ≤ 0and from here we deduce the result.

2. If x, y, z ∈ [0, 1] then x2 + y2 + z2 ≤ x2y + y2z + z2x+ 1.

Solution: Let f(t) = t2(1− y)− z2t+ y2 + z2 − y2z − 1. We must provethat f(x) ≤ 0. If y = 1 then f(t) = −z2t+z2+1−z−1 = z2(1−t)−z ≤z2 − z ≤ 9. If y < 1 then f is a quadratic in t with positive dominantcoefficient and attains its maximum either at t = 0 or at t = 1. But

f(1) = 1− y − z2 + y2 + z2 − y2z − 1 = y2(1− z)− y ≥ y2 − y ≤ 0

and f(0) = y2 + z2 − y2z − 1 = (1− z)(y2 − 1) ≤ 0.

3. If a < b < c < d then (a+ b+ c+ d)2 > 8(ac+ bd)

Solution: This time, we can try two different approaches. The first is tolook at (a+b+c+d)2−8(ac+bd) as at a quadratic expression in, say, a.It is written as a2−2(3c−b−d)a+(b+c+d)2−8bd. It’s discriminant isD = 32(c− b)(c− d) < 0 so the expression is always positive (moreoverwe could restrict the condition to just b < c < d or just c between b andd).

The second is to write (a+ b+ c+ d)2 − 8(ac+ bd) as D = B2 − 4AC.It’s convenient to set B = a + b + c + d,A = 2, C = ac + bd. Then thequadratic function f(x) = Ax2 +Bx+C = (x+a)(x+c)+(x+b)(x+d)satisfies f(−a) = (b− a)(d− a) > 0 and f(−b) = (a− b)(c− b) < 0 so fmust have a root and hence its discriminant is positive.

4. Show that a2 + b2 + c2 − ab− bc− ca ≥ 3

4(a− b)2.

Solution: Regarding this as a quadratic trinomial in c, we write it as

c2 − (a+ b)c+(a+ b)2

4≥ 0 or

(c− a+ b

2

)2

≥ 0.

33

Page 35: Олимпиадад бэлтгэгчдэд тусламж (Тэнцэтгэл биш 1)

5. Let 0 < a < b and xi ∈ [a, b]. Show that

(x1 + x2 + . . .+ xn)

(1

x1+

1

x2+ . . .+

1

xn

)≤ (a+ b)2

4abn2.

Solution: If xi ∈ [a, b] then (xi− a)(xi− b) ≤ 0 so x2i + ab ≤ (a+ b)xi or

xi +ab

xi≤ a + b. We conclude that

n∑i=1

xi + abn∑i=1

1

xi≤ (a + b)n and we

use AM-GM to conclude the proof.

6. If ai ∈ [a,A], bi ∈ [b, B] where 0 < a < A, 0 < b < B then

(a21 + . . .+ a2n)(b21 + . . .+ b2n)

(a1b1 + . . .+ anbn)2≤ 1

4

(√AB

ab+

√ab

AB

).

Solution: As this inequality has a form similar to the Cauchy-Schwartz,but has a inverse sign, we might try to find a quadratic function with azero and therefore positive discriminant. We can take

f(x) =n∑i=1

(a2ix

2 −

(√AB

ab+

√ab

AB

)x+ b2i

)

=n∑i=1

a2i

(x− bi

ai

√AB

ab

)(x− bi

ai

√ab

AB

).

To have f(t) ≤ 0 it suffices to find a t s.t.biai

√ab

AB≤ t ≤ bi

ai

√AB

ab, so

the intervals

[biai

√ab

AB;biai

√AB

ab

]must have non-empty intersection, or

biai

√ab

AB≤ bjaj

√AB

ab

for any i, j. This is equivalent toajbiaibj

≤ AB

ab, and is true from the

condition.

7. If a1, a2, . . . , an ∈ [1, n− 1] then

n(a1 + a2 + . . .+ an)2 ≥ 4(n− 1)(a21 + a22 + . . .+ a2n).

Solution: We have (ai−1) [ai − (n− 1)] ≤ 0 so a2i +(n−1) ≤ nai. Thus(a21 + a22 + . . .+ a2n) + n(n− 1) ≤ n(a1 + a2 + . . .+ an). Thus

4(n− 1)(a21 + a22 + . . .+ a2n) ≤ 4n(n− 1)(a1 + a2 + . . .+ an)− 4n(n− 1)2.

But 4n(n− 1)(a1 + a2 + . . .+ an)− 4n(n− 1)2 ≤ n(a1 + a2 + . . .+ an)2

as this is equivalent to n [a1 + a2 + . . .+ an − 2(n− 1)]2 ≥ 0.

34

Page 36: Олимпиадад бэлтгэгчдэд тусламж (Тэнцэтгэл биш 1)

8. Show that a2 + b2 + c2 ≥√

3(a3b+ b3c+ c3a), where a, b, c ≥ 0.

Solution: If all of a, b, c are equal, equality holds. If not, since theinequality is cyclic, we may assume that b − a > 0. As the inequalityis obvious we can set b − a = 1. Let x = c − a. Then f(a) = (a2 +b2 + c2)2 − 3(a3b + b3c + c3a) can be written as (x2 − x + 1)a2 + (x3 −5x2 + 4x+ 1)a+x4−3x3 + 2x2 + 1. Its discriminant can be evaluated as−3(x3−x2−2x−1)2 so is non-positive and thus f is always non-negative,QED.

9. Let a, b, c ∈ [0, 1] . Determine the greatest value of P = a+ b+ c− ab−bc− ca.Solution: We have that P (a, b, c) = a+ b+ c−ab− bc− ca is a quadraticpolynomial of a with the highest coefficient is 0, therefore, to find themaximum of P , we only need to consider the cases a = 0 or a = 1.Similarly, P is also a quadratic polynomial of b and c, therefore it sufficesto consider the cases b = 0 or b = 1, and c = 0 or c = 1. Combiningall cases with notice that the expression P is symmetric, we see that itsuffices to consider the following cases

Case 1. If a = b = c = 0, then P = 0.

Case 2. If a = b = 0, c = 1, then P = 1.

Case 3. If a = 0, b = c = 1, then P = 1.

Case 4. If a = b = c = 1, then P = 0.

Therefore, we obtain that maxP = 1.with equality holds when a = b =0, c = 1 or a = 0, b = c = 1 and their cyclic permutations.

10. Let a, b, c ∈[

1

3, 3

], then show that

a

a+ b+

b

b+ c+

c

c+ a≥ 7

5.

Solution: Since the inequality is cyclic, we may assume that c = max {a, b, c} ,

then since a, b, c ∈[

1

3, 3

], we have that

c

9≤ a ≤ c. We will change the

inequailty into a quadratic polynomial of a in order to use our theorem.

Put x =7

5− b

b+ c, then our inequality becomes

a

a+ b+

c

c+ a≥ x,

which is equivalent to a2 + ac + ac + bc ≥ x(a2 + ac + ab + bc), or(1− x)a2 + [(2− x)c− xb] a+ (1− x)bc ≥ 0.

If 1 − x ≤ 0 or eqivalently c ≥ 3

2b, we have that the last inequality is a

quadratic polynomial of a with the highest coefficient is not greater than

0, therefore, it suffices to consider the cases a =c

9or a = c to prove the

original inequality. If a = c9 , then we have that

a

a+ b+

b

b+ c+

c

c+ a=

c

9b+ c+

b

b+ c+

9

10=

7

5+

(3b− c)2

2(9b+ c)(b+ c)≥ 7

5,

and if a = c, then we have thata

a+ b+

b

b+ c+

c

c+ a=

c

c+ b+

b

b+ c+

1

2=

3

2>

7

5. So our inequality is proved in this case.

35

Page 37: Олимпиадад бэлтгэгчдэд тусламж (Тэнцэтгэл биш 1)

Now, let us consider the case 1 − x ≥ 0 or b ≥ 2

3c, then we have that

(2 − x)c − xb ≥ (2 − x)c − xc = 2(1 − x)c ≥ 0. Therefore, since a ≥ c

9,

we have

(1− x)a2 + [(2− x)c− xb] a+ (1− x)bc ≥

≥ (1− x)( c

9

)2+ [(2− x)c− xb]

( c9

)+ (1− x)bc.

And it suffices to prove that (1 − x)( c

9

)2+ [(2− x)c− xb]

( c9

)+ (1 −

x)bc ≥ 0. Or in another word, it means that we only need to prove the

original inequality in the case a =c

9, but this case has been already

proved in case 1. Our proof is now complete.

11. Let a, b, c ∈ [1, 2] , then show that (a+ b+ c)

(1

a+

1

b+

1

c

)≤ 10.

Solution: We can rewrite the original inequality as P (a, b, c) = (a+ b+c)(ab + bc + ca) − 10abc ≤ 0 which is a quadratic polynomial of a withthe highest coefficient is (b + c) > 0, therefore, it suffices to considerthe cases a = 1 or a = 2. Similarly, by the same manner, we also seethat P is also a quadratic polynomial of b and c, and thus, it sufficesto consider the cases b = 1 or b = 2, and c = 1 or c = 2. Combiningall cases with notice that the inequality is symmetric, we obtain that itsuffices to consider the following cases

Case 1. If a = b = c = 1, then P (1, 1, 1) = −1 < 0.

Case 2. If a = b = 1, c = 2, then P (1, 1, 2) = 0.

Case 3. If a = 1, b = c = 2, then P (1, 2, 2) = 0.

Case 4. If a = b = c = 2, then P (2, 2, 2) = −1 < 0.

Therefore, our inequality is proved.

12. Let a, b, c be nonnegative real numbers such that a + b + c = 3, thena2 + b2 + c2 + abc ≥ 4.

Solution: Put x = ab, then 0 ≤ x ≤ (a+ b)2

4, and our inequality becomes

f(x) = (c − 2)x + 2c2 − 6c + 5 ≥ 0 which is a quadratic polynomial ofx with the highest coefficient is 0, therefore, it suffices to prove that

min

{f(0), f

((a+ b)2

4

)}≥ 0. Now, putting P (a, b, c) = a2 + b2 + c2 +

abc, then we see that

f(0) = P (a+ b, 0, c), f

((a+ b)2

4

)= P

(a+ b

2,a+ b

2, c

).

which means that we have to prove min

{P (a+ b, 0, c), P

(a+ b

2,a+ b

2, c

)}≥

0, or in the other words, we only need to consider the original inequalityin the cases there is one variable equal to 0 and there are 2 variables

36

Page 38: Олимпиадад бэлтгэгчдэд тусламж (Тэнцэтгэл биш 1)

equal. Since the inequality is symmetric, therefore from the above state-ments, we have 2 cases

Case 1. If a = 0, then b = 3−c, and our inequality becomes (3−c)2+c2 ≥4, or equivalently 2c2 − 6c + 5 ≥ 0, which is true since by AM-GMinequality, we have 2c2 + 5 ≥ 2

√10c ≥ 6c.

Case 2. If a = b, then c = 3 − 2a ≥ 0, and our inequality becomes2a2 + (3 − 2a)2 + a2(3 − 2a) ≥ 4, or equivalently (5 − 2a)(a − 1)2 ≥ 0which is true since a ≤ 3

2 .This ends our proof.

13. Let a, b, c, d ∈ [0, 1] . Prove that a2b+b2c+c2d+d2a−ab2−bc2−cd2−da2 ≤8

27.

Solution: Since the inequality is cyclic, we may assume that a = max {a, b, c, d} ,then from a, b, c, d ∈ [0, 1] , we have that 0 ≤ d ≤ a. Now, notice that wecan rewrite our inequality as

f(d) = (a− c)d2 + (c2 − a2)d+ a2b+ b2c− ab2 − bc2 − 8

27≤ 0

which is a quadratic polynomial of d with the highest coefficient is non-negative, therefore, it suffices to prove that max {f(0), f(a)} ≤ 0. Wehave

f(0) = a2b+ b2c− ab2 − bc2 − 8

27= b(a− c)(a− b+ c)− 8

27

≤[b+ (a− c) + (a− b+ c)

3

]3− 8

27=

8

27(a3 − 1) ≤ 0,

and

f(a) = (a− c)a2 + (c2 − a2)a+ a2b+ b2c− ab2 − bc2 − 8

27

= (a− b)c2 + (b2 − a2)c+ a2b− ab2 − 8

27

= (b− c)(a− c)(a− b)− 8

27≤ b(a− c)(a− b)− 8

27

≤ ab(a− b)− 8

27≤ b(1− b)− 8

27≤[b+ (1− b)

2

]2− 8

27= − 5

108.

Therefore, our inequality is proved.

14. If a, b, c are the side-lengths of a triangle, then

3

(a

b+b

c+c

a

)≥ 2

(b

a+c

b+a

c

)+ 1.

Solution: Without loss of generality, we may assume that a ≥ b ≥ c,then b ≤ a ≤ b+ c. The inequality is equivalent to f(a) = (3c− 2b)a2 +(3b2 − 3bc− 2c2)a+ 3bc2 − 2b2c ≥ 0. If 3c ≤ 2b, then f(a) is a quadraticpolynomial of a with the highest coefficient is 3c − 2b ≤ 0, thereforeaccording to our theorem, it suffices to prove that min {f(b), f(b+ c)} ≥

37

Page 39: Олимпиадад бэлтгэгчдэд тусламж (Тэнцэтгэл биш 1)

0. But this inequality is true because f(b) = b(b−c)2 ≥ 0, and f(b+c) =c(b − c)2 ≥ 0. Now, let us consider the case 3c ≥ 2b, then by AM-GMinequality, we have that a2 + b2 ≥ 2ab, therefore

f(a) ≥ (3c− 2b)(2ab− b2

)+ (3b2 − 3bc− 2c2)a+ 3bc2 − 2b2c

= a(b− c)(2c− b) + b(2b− 3c)(b− c)≥ b(b− c)(2c− b) + b(2b− 3c)(b− c) = b(b− c)2 ≥ 0.

Remark. We still have to justify why we can assume that a ≥ b ≥ cbecause the inequality is not symmetric, it is only cyclic. Put P (a, b, c) =

3

(a

b+b

c+c

a

)−2

(b

a+c

b+a

c

)−3, then since the inequality is cyclic,

we may assume that a = max {a, b, c} . If b ≥ c, then the statement istrivial. If c ≥ b, then we have that

P (a, b, c)− P (a, c, b) = 5

(a

b+b

c+c

a− b

a− c

b− a

c

)=

5(a− b)(a− c)(c− b)abc

≥ 0.

Hence, it suffices to prove that P (a, c, b) ≥ 0 which is equivalent toP (a, b′, c′) ≥ 0 with a ≥ b′ ≥ c′. Therefore, we only need to consider thecase a ≥ b ≥ c in order to prove the original inequality. We may usesimilar methods to explain the reason why we can assume that a ≥ b ≥ cor c ≥ b ≥ a in proving cyclic inequalities.

15. If a, b, c are the side-lengths of a triangle, then

3

(a2

b2+b2

c2+c2

a2

)≥(a2 + b2 + c2

)( 1

a2+

1

b2+

1

c2

).

Solution: Put x = a2, y = b2, z = c2 and assume that x ≥ y ≥ z (wecan argue this like in the previous problem), then we have that y ≤ x ≤(√y +√z)2

since a, b, c are the side-lengths of a triangle. Our inequality

is equivalent to 3

(x

y+y

z+z

x

)≥ (x+ y + z)

(1

x+

1

y+

1

z

),which can

be rewritten as

f(x) = (2z − y)x2 + (2y2 − 3yz − z2)x+ 2yz2 − y2z ≥ 0.

If y ≥ 2z, then f(x) is a quadratic polynomial of x with the highest coeffi-

cient is 2z−y ≤ 0, therefore, it suffices to prove that min{f(y), f

((√y +√z)2)} ≥

0 which is true because f(y) = y(y − z)2 ≥ 0, and

f((√

y +√z)2)

=

= (2z − y)(√y +√z)4

+ (2y2 − 3yz − z2)(√y +√z)2

+ 2yz2 − y2z= (2c2 − b2)(b+ c)4 + (2b4 − 3b2c2 − c4)(b+ c)2 + 2b2c4 − b4c2

= b6 − 6b4c2 − 2b3c3 + 9b2c4 + 6bc5 + c6

= (b3 − 3bc2 − c3)2 ≥ 0.

38

Page 40: Олимпиадад бэлтгэгчдэд тусламж (Тэнцэтгэл биш 1)

Now, let use consider the case 2z ≥ y, then by AM-GM inequality, wehave that x2 + y2 ≥ 2xy. Therefore

f(x) = (2z − y)x2 + (2y2 − 3yz − z2)x+ 2yz2 − y2z≥ (2z − y)(2xy − y2) + (2y2 − 3yz − z2)x+ 2yz2 − y2z= xz(y − z) + y(y − z)(y − 2z) ≥ yz(y − z) + y(y − z)(y − 2z)

= y(y − z)2 ≥ 0.

Our proof is complete.

16. If a, b, c, d are nonnegative real numbers such that a+ b+ c+d = 1, then

abc+ bcd+ cda+ dab ≤ 1

27+

176

27abcd.

Solution: Put x = ab, then 0 ≤ x ≤ (a+b)2

4 , our inequality becomes

f(x) =

(c+ d− 176

27cd

)x + cd(a + b) − 1

27≤ 0. This is a quadratic

polynomial of x with the highest coefficient is 0, therefore, it suffices to

prove that max

{f(0), f

((a+ b)2

4

)}≤ 0. It shows that it suffices to

consider the cases a = b or ab = 0. Similarly, we also obtain that itsuffices to consider the cases c = d or cd = 0. Combining both thesestatements, we obtain that, in order to prove the orginal inequality, itenough to prove it in the following cases

Case 1. If b = d = 0, then the inequality is trivial since abc+bcd+cda+

dab− 176

27abcd = 0.

Case 2. If a = b, d = 0, then c = 1 − 2a ≥ 0, the inequality becomes

a2(1 − 3a) ≤ 1

27which is true since by AM-GM inequality, we have

a2(1− 3a) ≤[a+ a+ (1− 2a)

3

]3=

1

27.

Case 3. If a = b, c = d, then c = 1−2a2 , the inequality becomes a2(1 −

2a) + 12a(1 − 2a)2 ≤ 1

27 + 4427a

2(1 − 2a)2, which is equivalent to (4a −1)2(22a2 − 11a + 2) ≥ 0. The last inequality is valid since by AM-GMinequality, we have 22a2 + 2 ≥ 4

√11a ≥ 11a. This ends our proof.

17. If a, b, c, d are nonnegative real numbers such that a+ b+ c+d = 4, then

(1 + 3a)(1 + 3b)(1 + 3c)(1 + 3d) ≤ 125 + 131abcd.

Solution: Proceeding similar to the previous problem, we see that itenough to prove it in the following cases

Case 1. If b = d = 0, then the inequality is trivial since

(1 + 3a)(1 + 3b)(1 + 3c)(1 + 3d) = (1 + 3a)(1 + 3c) ≤ 1

4(2 + 3a+ 3c)2

= 49 < 125 = 125 + 131abcd.

39

Page 41: Олимпиадад бэлтгэгчдэд тусламж (Тэнцэтгэл биш 1)

Case 2. If a = b, d = 0, then c = 4− 2a ≥ 0, and the inequality becomes(1 + 3a)2(13 − 6a) ≤ 125 which is true since by AM-GM inequality, wehave that

(1 + 3a)2(13− 6a) ≤[

(1 + 3a) + (1 + 3a) + (13− 6a)

3

]3= 125.

Case 3. If a = b, c = d, then c = 2− a ≥ 0, and the inequality becomes(1 + 3a)2(7− 3a)2 ≤ 125 + 131a2(2− a)2, which is equivalent to (25a2−50a+ 38)(a− 1)2 ≥ 0 which is true. Our proof is complete.

18. Let a, b, c, d be nonnegative real numbers with sum 4, prove that 3(a2 +b2 + c2 + d2) + 4abcd ≥ 16.

Solution: Again, proceeding similar to the previous problem, we see thatit enough to prove it in the following cases

Case 1. If b = d = 0, then the inequality is trivial since

3(a2 + b2 + c2 + d2) + 4abcd = 3(a2 + c2) ≥ 3

2(a+ c)2 = 24 > 16.

Case 2. If a = b, d = 0, then c = 4 − 2a and the inequality becomes3[2a2 + (4− 2a)2

]≥ 16, which is true since by Cauchy Schwartz in-

equality (see at the next chapter), we have that

2a2 + (4− 2a)2 ≥ 1

3[a+ a+ (4− 2a)]2 =

16

3.

Case 3. If a = b, c = d, then c = 2 − a ≥ 0 and the inequality becomes6a2+6(2−a)2+4a2(2−a)2 ≥ 16, or equivalently (a2−2a+2)(a−1)2 ≥ 0which is trivial. The proof is complete.

19. Let a, b, c be nonnegative real numbers such that a + b + c = 3. Provethat

1

6− ab+

1

6− bc+

1

6− ca≤ 3

5.

Solution: By expanding, the inequality is equivalent to 108 + 13abc(a+b+ c)− 48(ab+ bc+ ca)− 3a2b2c2 ≥ 0, or 36 + 13abc− a2b2c2 − 16(ab+

bc+ ca) ≥ 0. Put x = ab, then 0 ≤ x ≤ (a+ b)2

4, our inequality becomes

f(x) = −c2x2 + (13c − 16)x + 36 − 16c(3 − c) ≥ 0. This is a quadraticpolynomial of x with the highest coefficient is −c2 ≤ 0, therefore, it

suffices to prove that min{f(0), f

((a+b)2

4

)}≥ 0. It shows that it suffices

to consider the cases a = b or ab = 0.

Case 1. If b = 0, then a = 3−c, and the inequality becomes 36−16c(3−c) ≥ 0, or equivalently 4(3− 2c)2 ≥ 0.

Case 2. If a = b, then c = 3 − 2a ≥ 0, and the inequality becomes36 + 13a2(3− 2a)− a4(3− 2a)2− 16a2− 32a(3− 2a) ≥ 0, or equivalently−4a6 + 12a5− 9a4− 26a3 + 87a2− 96a+ 36 ≥ 0, or (3− 2a)(a+ 2)(2a2−3a+ 6)(a− 1)2 ≥ 0. Our proof is complete.

40

Page 42: Олимпиадад бэлтгэгчдэд тусламж (Тэнцэтгэл биш 1)

20. Let a, b, c, d be nonnegative real numbers, no three of which are zero.Show that√

a

a+ b+ c+

√b

b+ c+ d+

√c

c+ d+ a+

√d

d+ a+ b≤ 4√

3.

Solution: By Cauchy Schwarz inequality, we have(∑cyc

√a

a+ b+ c

)2

[∑cyc

(a+ b+ d)(a+ c+ d)

][∑cyc

a

(a+ b+ c)(a+ b+ d)(a+ c+ d)

]

=2[2(a+ b+ c+ d)2 + (a+ c)(b+ d)][(a+ c)(b+ d) + ac+ bd]

(a+ b+ c)(b+ c+ d)(c+ d+ a)(d+ a+ b).

We need to prove that

P (a, b, c, d) = 8(a+ b+ c)(b+ c+ d)(c+ d+ a)(d+ a+ b)

−[2(a+ b+ c+ d)2 + (a+ c)(b+ d)][(a+ c)(b+ d) + ac+ bd] ≥ 0.

It is easy to see that P (a, b, c, d) is a quadratic polynomial of x = ac withthe highest coefficient is 0, therefore, in order to prove P (a, b, c, d) ≥ 0,it suffices to prove it in the case a = c or ac = 0. Similarly, P (a, b, c, d)is also a quadratic polynomial of t = bd with the highest coefficient is 0,therefore, in order to prove P (a, b, c, d) ≥ 0, it suffices to prove it in thecase b = d or bd = 0. Combining all cases, we see that it is enough toprove the inequality in the following cases

Case 1. c = d = 0, then the inequality becomes 8ab(a+ b)2 ≥ 3ab(2a2 +5ab+ 2b2), or equivalently ab(2a2 + ab+ 2b2) ≥ 0.

Case 2. a = c, d = 0, then the inequality becomes 16a(2a+ b)(a+ b)2 ≥6a(a+ b)(a+ 2b)(4a+ b), or equivalently 2a(a+ b)(4a2− 3ab+ 2b2) ≥ 0.

Case 3. a = c, b = d, then the inequality becomes 8(2a+ b)2(a+ 2b)2 ≥12(2a2+5ab+2b2)(a2+4ab+b2), or equivalently 4(a+2b)(2a+b)(a−b)2 ≥0.Our proof is now complete. Equality holds if and only if a = b = c = d.

1.7 Cauchy-Schwartz Inequality

The inequality

(a21 + a22 + . . .+ a2n)(b21 + b22 + . . .+ b2n) ≥ (a1b1 + a2b2 + . . .+ anbn)2

has incredibly many applications.Especially useful is the following form of the inequality:CBS Lemma: For yi > 0,

x21y1

+ . . .+x2nyn≥ (x1 + x2 + . . .+ xn)2

y1 + y2 + . . .+ yn

41

Page 43: Олимпиадад бэлтгэгчдэд тусламж (Тэнцэтгэл биш 1)

It can be easily noticed that it’s equivalent to CBS inequality, and it can beproven also by induction on n.

With this lemma, inequalities like the following become straightforward:

Example 1. If a, b, c > 0 then

a

a+ 2b+

b

b+ 2c+

c

c+ 2a≥ 1.

Indeed, multiplying the denominator and numerator of the first fraction witha, of the second with b, and of the third with c, we get

a2

a2 + 2ab+

b2

b2 + 2bc+

c2

c2 + 2ca≥ (a+ b+ c)2

a2 + 2ab+ b2 + 2bc+ c2 + 2ca= 1.

The importance of this inequality is revealed by the multitude of exercises ithelps solve.

Exercises

1. If x, y, z ≥ 0 show that√

(x+ y)(x+ z) ≥ x+√yz.

Solution: Immediate from Cauchy-Schwartz for√x,√y and

√x,√z.

2. Show that n(x21 + x22 + . . .+ x2n) ≥ (x1 + x2 + . . .+ xn)2

Solution: In the form (1 + 1 + . . . + 1)(x21 + x22 + x23 + . . . + x2n) ≥(x1 + . . .+ xn)2, Cauchy-Schwartz is clearly seen.

3. For a, b, c > 0, show thata

a+ 2b+ c+

b

b+ 2c+ a+

c

c+ 2a+ b≥ 3

4.

Solution: Amplify the first fraction by a, the second by b, the third byc and apply CBS Lemma.

4. For a, b, c, d, e, f > 0 show thata

b+ c+

b

c+ d+

c

d+ e+

d

e+ f+

e

f + a+

f

a+ b≥ 3.

Solution: We again amplify by the corresponding numerators and applyCBS Lemma, obtaining

a

b+ c+

b

c+ d+

c

d+ e+

d

e+ f+

e

f + a+

f

a+ b≥

≥ (a+ b+ c+ d+ e+ f)2

a(b+ c) + b(c+ d) + c(d+ e) + d(e+ f) + e(f + a) + f(a+ b).

It remains to show that

a(b+c)+b(c+d)+c(d+e)+d(e+f)+e(f+a)+f(a+b) ≤ (a+ b+ c+ d+ e+ f)2

3.

However this can be written as (a+d)(b+e)+(b+e)(c+f)+(c+f)(a+d)

and we apply the inequality uv + vw + wu ≤ (u+ v + w)2

3.

42

Page 44: Олимпиадад бэлтгэгчдэд тусламж (Тэнцэтгэл биш 1)

5. If1

x+

1

y+

1

z= 2 then

√x+ y + z ≥

√x− 1 +

√y − 1 +

√z − 1.

Solution: By CBS,(√x− 1 +

√y − 1 +

√z − 1

)2≤

≤ (x+ y + z)

(x− 1

x+y − 1

y+z − 1

z

)= x+ y + z.

6. If abc = 1, a, b, c > 0 thena

1 + b+ c+

b

1 + c+ a+

c

1 + a+ b≥ 1.

Solution: Amplifying the fractions by a, b, c we get

a2

a+ ab+ ac+

b2

b+ ab+ bc+

c2

c+ ac+ bc≥ (a+ b+ c)2

a+ b+ c+ 2(ab+ bc+ ca)

≥ (a+ b+ c)2

(a+ b+ c)2= 1.

The last inequality follows from the fact that a2 + b2 + c2 ≥ a+ b+ c asa2 + 1 + b2 + 1 + c2 + 1 ≥ 2a+ 2b+ 2c and a+ b+ c ≥ 3.

7. Let n ≥ 3, a1, a2, . . . , an ≥ 0 with a21 + a22 + . . .+ a2n = 1. Prove that

a1a22 + 1

+a2

a23 + 1+ . . .+

ana21 + 1

≥ 4

5(a1√a1 + a2

√a2 + . . .+ an

√an)2 .

Solution: Amplifying the fractions by a2i and applying the CBS Lemmawe get

a1a22 + 1

+a2

a23 + 1+ . . .+

ana21 + 1

≥(a1√a1 + a2

√a2 + . . .+ an

√an)2

a21a22 + a21 + . . .+ a2na

21 + a2n

.

So we are left to show that a21a22 + . . .+a2na

21 ≤

1

4which we have already

proven in exercise 7 at induction.

8. Show that

x

x+√

(x+ y)(x+ z)+

y

y +√

(y + z)(y + x)+

z

z +√

(z + x)(z + y)≤ 1.

Solution: We have√

(x+ y)(x+ z) =√

(x+ y)(z + x) ≥√xz +

√xy

hence

x

x+√

(x+ y)(x+ z)≤ x

x+√xy +

√xz

=

√x√

x+√y +√z.

Summing with the analogously deduced inequalities we get the result.

43

Page 45: Олимпиадад бэлтгэгчдэд тусламж (Тэнцэтгэл биш 1)

9. If a, b, c ≥ 0 thena

1 + a+

b

1 + b+

c

1 + c+

1

1 + a+ b+ c≥ 1.

Solution:

a2

a2 + a+

b2

b2 + b+

c2

c2 + c+

1

a+ b+ c+ 1≥

≥ (a+ b+ c+ 1)2

a2 + a+ b2 + b+ c2 + c+ a+ b+ c+ 1= 1.

10. If m ≥ n ≥ p ≥ q are integers numbers with m + q = n + p anda1, a2, . . . , ak are positive reals then(

k∑i=1

ami

)(k∑i=1

aqi

)≥

(k∑i=1

ani

)(k∑i=1

api

).

Solution: Let sr =

k∑i=1

ari . We must prove that smsq ≥ snsp. For

n = p = n− 1 = q+ 1 we have sn+1sn−1 ≥ s2n, which is true in virtue of

CBS inequality. Rewriting this assn+1

sn≥ snsn−1

. Therefore rn =sn+1

snis increasing in n. Hence rm−1rm−2 . . . rn ≥ rp−1rp−2 . . . rq (recall that

m− n = p− q) which means thatsmsn≥ spsq

or smsq ≥ snsp.

11. If n, k > l ∈ N, a1, a2, . . . , an ≥ 0 thenk

√ak1 + . . .+ akn

n≥ l

√al1 + . . .+ aln

n.

Solution: We shall use induction on k − l. If k = l this is obvious.Rephrasing the problem using the notation in the previous exercise weneed to show that slkn

k−l ≥ skl . Now remember that we proved that

ri =si+1

si≥ ri−1 =

sisi−1

. From here we deduce that

rlk ≥ rl−1rl−2 . . . r0 or

(sk+1

sk

)l≥ sln

or

(sk+1

sk

)ln ≥ sl,

and this relation performs the induction step on k − l: multiplying itslkn

k−l ≥ skl we get slk+1nk−l+1 ≥ sk+1

l .

12. (Minkowski) If ai, bi are reals then

n∑i=1

√a2i + b2i ≥

√√√√( n∑i=1

ai

)2

+

(n∑i=1

bi

)2

.

Solution: If n = 2 the inequality is equivalent to√a2 + b2 +

√c2 + d2 ≥

√(a+ c)2 + (b+ d)2,

which after squaring and cancelling common terms becomes√(a2 + b2)(c2 + d2) ≥ ac+ bd,

which is CBS. The general case follows by induction on n.

44

Page 46: Олимпиадад бэлтгэгчдэд тусламж (Тэнцэтгэл биш 1)

13. If a, b, c > 0 add up to 1, thena+ 1

c(2− b)+

b+ 1

a(2− c)+

c+ 1

b(2− a)≥ 36

5.

Solution:

1

2− a=

1

2 (a+ b+ c)− a=

1

2b+ 2c+ a≤ 1

52

(2

b+

2

c+

1

a

),

where the last inequality is because

(2

b+

2

c+

1

a

)(2b+ 2c+ a) ≥ 52

(Cauchy-Schwartz). Similarly,

1

2− b≤ 1

52

(2

c+

2

a+

1

b

),

1

2− c≤ 1

52

(2

a+

2

b+

1

c

).

Summing up results in

1

2− a+

1

2− b+

1

2− c≤

≤ 1

52

(2

b+

2

c+

1

a

)+

1

52

(2

c+

2

a+

1

b

)+

1

52

(2

a+

2

b+

1

c

)=

1

52

(5

a+

5

b+

5

c

)=

1

5

(1

a+

1

b+

1

c

).

Thus, using a+ 1 = 2− (1− a) = 2− (b+ c) = (2− b)− c and similarlyb+ 1 = (2− c)− a and c+ 1 = (2− a)− b, we get

a+ 1

c (2− b)+

b+ 1

a (2− c)+

c+ 1

b (2− a)=

=(2− b)− cc (2− b)

+(2− c)− aa (2− c)

+(2− a)− bb (2− a)

=

(1

c− 1

2− b

)+

(1

a− 1

2− c

)+

(1

b− 1

2− a

)=

(1

a+

1

b+

1

c

)−(

1

2− a+

1

2− b+

1

2− c

)≥

(1

a+

1

b+

1

c

)− 1

5

(1

a+

1

b+

1

c

)=

4

5

(1

a+

1

b+

1

c

).

So it remains to show that 45

(1

a+

1

b+

1

c

)≥ 36

5, i. e. that

1

a+

1

b+

1

c≥ 9.

But this is clear, since Cauchy-Schwartz gives (a+ b+ c)

(1

a+

1

b+

1

c

)≥

9, and a+ b+ c = 1. This completes the proof.

14. Let a, b, c, x, y, z be reals such that

(a+ b+ c)(x+ y + z) = 3 and (a2 + b2 + c2)(x2 + y2 + z2) = 4.

Show that ax+ by + cz ≥ 0

Solution: Set u = ax + by + xz. As (a, b, c) and (x, y, z) are unrelatedin the condition, we can also take its ”brothers”: v = ay + bz + cx,w = az + bx+ cy. Then u+ v + w = (a+ b+ c)(x+ y + z) = 3.

u2 + v2 + w2 = (a2 + b2 + c2)(x2 + y2 + z2) + 2(ab+ bc+ ca)(xy + yz + zx)

= 4 + 2(ab+ bc+ ca)(xy + yz + xz).

45

Page 47: Олимпиадад бэлтгэгчдэд тусламж (Тэнцэтгэл биш 1)

Assume now that u < 0, then v+w ≥ 3 so a(y+z)+b(x+z)+c(x+y) > 3.Applying CBS we get

(a2+b2+c2)[2(x2 + y2 + z2) + 2(xy + yz + zx)

]≥ 9

4(a2+b2+c2)(x2+y2+z2).

This means that xy+ yz+ zx ≥ x2 + y2 + z2

8and analogously ab+ bc+

ca ≥ a2 + b2 + c2

8. This estimates are too rough, however they tell us

that ab + bc + ca, xy + yz + zx are positive, and so we can apply CBSin the following form:[

a2 + b2 + c2 + 2(ab+ bc+ ca)] [x2 + y2 + z2 + 2(xy + yz + zx)

]≥

≥[√

(a2 + b2 + c2)(x2 + y2 + z2) + 2√

(ab+ bc+ ca)(xy + yz + zx)]2

or 9 ≥[2 + 2

√(ab+ bc+ ca)(xy + yz + zx)

]2which implies

(ab+ bc+ ca)(xy + yz + zx) ≤ 1

4.

It suffices to derive a contradiction now: we have

u2 + v2 + w2 = 4 + 2(ab+ bc+ ca)(xy + yz + xz) ≤ 9

2,

however from the other side u2 + v2 + w2 > v2 + w2 ≥ (v + w)2

2>

9

2.

15. If a, b, c, x, y, z > 0 then

a

b+ c(y + z) +

b

a+ c(x+ z) +

c

a+ b(x+ y) ≥ 3

xy + yz + zx

x+ y + z.

Solution: We add (y+ z) + (x+ z) + (x+ y) to both sides to get a morecomfortable form of the LHS:

(a+b+c)

(y + z

b+ c+z + x

c+ a+x+ y

a+ b

)=

2(x+ y + z)2 + 3(xy + yz + zx)

x+ y + z.

Now from CBS we have

(a+ b+ c)

(y + z

b+ c+z + x

c+ a+x+ y

a+ b

)=

=1

2[(b+ c) + (c+ a) + (a+ b)]

(y + z

b+ c+z + x

c+ a+x+ y

a+ b

)≥ 1

2

(√y + z +

√z + x+

√x+ y

)2.

So it suffices to prove that(√y + z +

√z + x+

√x+ y

)2 ≥ 4(x+ y + z) + 6xy + yz + zx

x+ y + z,

46

Page 48: Олимпиадад бэлтгэгчдэд тусламж (Тэнцэтгэл биш 1)

or

2(x+ y + z) + 2∑cyc

√(x+ y)(x+ z) ≥ 4(x+ y + z) + 6

xy + yz + zx

x+ y + z

or√(x+ y)(x+ z)+

√(y + z)(y + x)+

√(z + x)(z + y) ≥ (x+y+z)+3

xy + yz + zx

x+ y + z.

However√(x+ y)(x+ z) +

√(y + z)(y + x) +

√(z + x)(z + y) =

=√x2 + (xy + yz + zx) +

√y2 + (xy + yz + zx) +

√z2 + (xy + yz + zx)

≥√

(x+ y + z)2 + 9(xy + yz + zx)

from Minkowski. The fact that√(x+ y + z)2 + 9(xy + yz + zx) ≥ (x+ y + z) + 3

xy + yz + zx

x+ y + z

follows by squaring.

16. If a1, a2, . . . , an are positive numbers that sum to 1, then(a1

a22 + a2+

a2a23 + a3

+ . . .+an

a21 + a1

)(a1a2+a2a3+. . .+ana1) ≥

n

n+ 1.

Solution: We may note thatai

a2i+1 + ai+1=

aiai+1

− aiai+1 + 1

, thus

a1a22 + a2

+a2

a23 + a3+ . . .+

ana21 + a1

=

=

(a1a2

+a2a3

+ . . .+ana1

)−(

a1a2 + 1

+ . . .+an

a1 + 1

).

But

(a1a2

+a2a3

+ . . .+ana1

)(a1a2 + . . .+ana1) ≥ 1 so it suffices to prove

that

a1a2

+a2a3

+ . . .+ana1≥ (n+ 1)

(a1

a2 + 1+ . . .+

ana1 + 1

).

However we may note thata1a2

+ . . .+ana1≥ n but as

(a2 + 1)

(1

n2a2+ 1

)≥(n+ 1

n

)2

(CBS)

we deducea1

a2 + 1≤ 1

(n+ 1)2a1a2

+n2

(n+ 1)2a1. Summing with the anal-

ogous relations we get the result.

47

Page 49: Олимпиадад бэлтгэгчдэд тусламж (Тэнцэтгэл биш 1)

1.8 Young Inequality

In the AM-GM inequality put m terms equal a and n terms equal b. Wededuce

ma+ nb ≥ (m+ n)am

m+n bn

m+n .

Now setting p =m+ n

m, q =

m+ n

n, x = a

mm+n , y = b

nm+n we rewrote the

inequality asxp

p+yq

q≥ pq for any a, b > 0, p, q ∈ Q with 1

p + 1q = 1. We can

extend this inequality to any positive real p, q: taking sequences of rationals

pn, qn with1

pn+

1

qn= 1 and pn tending to p, qn tending to q we deduce

xpn

pn+yqn

qn≥ xy. Passing to limit we have

Theorem. If p, q > 0 with 1p + 1

q > 0 then for any positive reals x, y > 0

xp

p+yq

q≥ xy.

This is called Young’s inequality.

Exercise: a) Prove that Young Inequality changes sign when one of p, q is

negative. (Hint: if, for example, q < 0, use normal Young Inequality for bq

1−q ,

(ab)qq−1 and weights 1− q, 1− q

q).

b) Prove the generalization of Young Inequality:

If p1, p2, . . . , pn, x1, x2, . . . , xn > 0 and p1 + p2 + . . .+ pn = 1 then

p1x1 + . . .+ pnxn ≥ xp11 . . . xpnn

(hint: use induction).

Take now some A,B, a1, a2, . . . , an, b1, b2, . . . , bn positive reals. Then

aibiAB≤ 1

p

(aiA

)p+

1

q

(biB

)q.

Summing this over all i we have

a1b1 + . . .+ anbnAB

≤ 1

p

n∑i=1

api

A

p

+1

q

n∑i=1

bpi

B

q

.

Taking now A =

(n∑i=1

api

)1p

, B =

(n∑i=1

nbqi

)1q

we deduce

Theorem. If p, q > 0 and 1p+1

q = 1 then for any positive reals a1, . . . , an, b1, . . . , bnthe following inequality holds:

48

Page 50: Олимпиадад бэлтгэгчдэд тусламж (Тэнцэтгэл биш 1)

n∑i=1

aibi ≤

(n∑i=1

api

)1p(∑i=1

nbqi

)1q

This is Holder’s Inequality. Like Young’s Inequality, when one of p, q isnegative it changes sign. [Note: for p = q = 2 this is Cauchy-Schwartz].Take p > 1 and apply Holder for the numbers ai, (ai + bi)

p−1 with weights

p,p

p− 1. We deduce

n∑i=1

ai(ai + bi)p−1 ≤

(n∑i=1

api

)1p[

n∑i=1

(ai + bi)p

]p−1p

.

Writing the analogous inequality for bi, (ai + bi)p−1 and summing we obtain

Minkovski’s Inequality:(n∑i=1

(ai + bi)p

)1p

(n∑i=1

api

)1p

+

(n∑i=1

bpi

)1p

.

When 0 < p < 1 it changes sign.

Exercises

1. Show that for any x ≥ y ≥ 0,

(ax1 + . . .+ axn)(a−x1 + . . .+ a−xn ) ≥ (ay1 + . . .+ ayn)(a−y1 + . . .+ a−yn ).

Solution: By Holder,

(ax1 + . . .+ axn)yx (1 + 1 + . . .+ 1)

x−yx ≥ (ay1 + . . .+ ayn),

and analogously

(a−x1 + . . .+ a−xn )yx (1 + 1 + . . .+ 1)

x−yx ≥ (a−y1 + . . .+ a−yn ).

The inequality follows from multiplying these two and using the factthat

(ax1 + . . .+ axn)(a−x1 + . . .+ a−xn ) ≥ n2.

2. For a, b, c > 0√a− ab+ b2 +

√a2 − ac+ c2 ≥

√b2 + bc+ c2.

Solution: we write it as√(b

2− a)2

+

(b

2

)2

+

(b

2

)2

+

(b

2

)2

+

√(a− c

2

)2+( c

2

)2+( c

2

)2+( c

2

)2≥

√(b

2− c

2

)2

+

(b

2+c

2

)2

+

(b

2+c

2

)2

+

(b

2+c

2

)2

,

and it now follows from Minkowski.

49

Page 51: Олимпиадад бэлтгэгчдэд тусламж (Тэнцэтгэл биш 1)

3. (a2 + ab+ b2)(b2 + bc+ c2)(c2 + ca+ a2) ≥ (ab+ bc+ ca)3

Solution: Write it as

(a2 + ab+ b2)(ac+ a2 + c2)(c2 + b2 + bc) ≥ (ab+ bc+ ca)3.

By CBS

(a2 + ab+ b2)(ac+ a2 + c2) ≥(a32 c

12 + a

32 b

12 + bc

)2

and

(a32 c

12 + a

32 b

12 + bc

)2

(c2 + b2 + bc) ≥ (ab+ bc+ ca)3 by Holder for

p =3

2, q = 3.

Note: by the very same method we can prove by induction the general-ized version of Holder for more that two sequences of variables.

1.9 Advanced techniques with Cauchy-Buniakowski-Schwarz and Holder Inequalities

In the previous parts, we have presented the Cauchy-Schwartz inequality andthe Holder’s inequality with their basic techniques in use. These inequalitiesare so important, however, that they deserve an additional chapter for finerapplications.

Firstly, let us recall the Cauchy-Schwartz inequality and the Holder’s inequal-ity.

Theorem 1. (Cauchy-Schwartz inequality) For any real numbers (a1, a2, . . .,an) and (b1, b2, . . ., bn), we have

(a1b1 + a2b2 + . . .+ anbn)2 ≤ (a21 + a22 + . . .+ a2n)(b21 + b22 + . . .+ b2n).

The equality holds if and only if ai : aj = bi : bj ∀i, j ∈ {1, 2, . . . , n} .

Corollary 1. (CBS lemma) For any real numbers (a1, a2, . . ., an) and (b1, b2,. . ., bn) with bi > 0 ∀i = 1, 2, . . . , n, we have

a21b1

+a22b2

+ . . .+a2nbn≥(a1 + a2 + . . .+ an)2

b1 + b2 + . . .+ bn.

The equality holds if and only if ai : aj = bi : bj ∀i, j ∈ {1, 2, . . . , n} .

Theorem 2. (Generalized Holder’s inequality) Given positive reals xij (i =1,m, j = 1, n). Then, for all ω1, . . ., ωn ≥ 0 satisfy ω1 + . . .+ ωn = 1, we have

n∏i=1

m∑j=1

xij

ωj

≥m∑j=1

(n∏i=1

xωj

ij

).

50

Page 52: Олимпиадад бэлтгэгчдэд тусламж (Тэнцэтгэл биш 1)

A special case of Holder’s inequality which we used to apply is(m∑i=1

ani

)(m∑i=1

bni

)n−1≥

(m∑i=1

aibn−1i

)n,

for all ai, bi ≥ 0. The equality holds in this inequality whena1b1

= . . . =ambm

.

In most advanced inequalities it is not immediately clear how to apply to CBSor Holder. One needs to do some more work to ”prepare” it for their use. Inthe following, we will describe several common techniques that help one findthe way to apply Cauchy-Schwartz and Holder to problems. These techniquesare not restricted only to Cauchy-Schwartz and Holder, they work for otherinequalities as well, but in this paragraph we will only focus on these twoparticular inequalities.

Balancing coefficients

Grouping the coefficients to use classical inequalities is not always easy, espe-cially in non-symmetric inequalities. In these cases, the coefficients of similarterms are normally not equal to each other and therefore there is confusionabout not only the use basic inequalities properly but also taking care of thecase of equality so that this case is valid throughout the process of solution.One way to deal with this is to use additional variables (”weights”) in orderto apply some inequality with unknown parameters, and the solve a system ofequations to find assign the corresponding values to the weights. Knowing anequality case or what should be in the right-hand side of the inequality usuallygives information that is helpful in finding them. This method is called thebalancing coefficients technique. It is mainly applied in conjunction with twoinequalities the AM-GM inequality and Cauchy Schwartz/Holder inequality.In some sense, Holder as deduced from Young’s inequality is a consequence ofthis method for AM-GM: in the inequality ap

p + bq

q ≥ ab, p and q are ”weights”

with respect to which weighted AM-GM is applied (i.e. ap appears ”1p times”

and bq appears ”1q times”).

We will focus on applying the technique for Cauchy-Schwarts/Holder inequal-ities in the examples.

Example 1. Given nonnegative real numbers x, y, z satisfying 2x+ 3y+ z = 1.Determine the minimum value of P = x3 + y3 + z3.

Solution: According to the Holder’s inequality, we have that for all a, b, c ≥ 0,

(x3 + y3 + z3)(a3 + b3 + c3)2 ≥ (xa2 + yb2 + zc2)3.

It follows that

P = x3 + y3 + z3 ≥ (xa2 + yb2 + zc2)3

(a3 + b3 + c3)2.

Now, let us choose a, b, c, so that we can apply the given hypothesis 2x+ 3y+z = 1 to the above inequality, and therefore, naturally, we will choose a, b, c,

51

Page 53: Олимпиадад бэлтгэгчдэд тусламж (Тэнцэтгэл биш 1)

such thata2

2=b2

3=c2

1= 1, or a =

√2, b =

√3, c = 1. The equality holds

whenx

a=y

b=z

c, or

x

a=y

b=z

c=

2x+ 3y + z

2a+ 3b+ c=

1

2a+ 3b+ c, or

x =a

2a+ 3b+ c, y =

b

2a+ 3b+ c, z =

c

2a+ 3b+ c.

From now, we have

P ≥ (xa2 + yb2 + zc2)3

(a3 + b3 + c3)2=

(2x+ 3y + z)3[(√2)3

+(√

3)3

+ 13]2 =

1(2√

2 + 3√

3 + 1)2 .

Since the equality can be attained, the minimum value of P is1(

2√

2 + 3√

3 + 1)2 .

Example 2. Given nonnegative real numbers x, y such that x3 + y3 = 1. Findthe maximum value of P = x+ 2y.

Solution: This is in a sense a converse to the previous example. By Holder’sinequality, we have that for all a, b ≥ 0

(x3 + y3)(a3 + b3)2 ≥ (xa2 + yb2)3,

hence

xa2 + yb2 ≤ 3

√(x3 + y3)(a3 + b3)2.

Naturally we will choose a, b so that xa2 + yb2 ≡ P, which gives us a = 1, b =√

2. The equality holds whenx

a=y

b, or

x3

a3=y3

b3=x3 + y3

a3 + b3=

1

a3 + b3, or

x3 =a3

a3 + b3, y3 =

b3

a3 + b3.

Then

P ≤ 3

√(x3 + y3)

[13 +

(√2)3]2

=3

√(2√

2 + 1)3.

Again equality is attained and we obtain that the maximum of P is3

√(2√

2 + 1)3.

Example 3. Let a1, a2, . . . , an be positive real numbers. Prove that

n∑k=1

kk∑j=1

aj

≤ 2

n∑k=1

1

ak.

Solution: By the Cauchy Schwartz inequality,

(a1+a2+ . . .+ ak)

(b21a1

+b22a2

+ . . .+b2kak

)≥ (b1 + b2 + . . .+ bk)

2,

52

Page 54: Олимпиадад бэлтгэгчдэд тусламж (Тэнцэтгэл биш 1)

where bi are arbitrary positive real numbers for all i = 1, 2, . . . , n. Therefore

kk∑j=1

aj

≤ k

(b1 + b2 + . . .+ bk)2

(b21a1

+b22a2

+ . . .+b2kak

),

from which we deduce thatn∑k=1

kk∑j=1

aj

≤n∑i=1

ciai,

where

ck =kb2k

(b1 + b2 + . . .+ bk)2+

(k + 1)b2k(b1 + b2 + . . .+ bk+1)2

+. . .+nb2k

(b1 + b2 + . . .+ bk+1)2,

for all k = 1, 2, . . . , n. Choosing now bk = k, we get

ck =k3(k∑i=1

i

)2 +k2(k + 1)(k+1∑i=1

i

)2 + . . .+k2n(n∑i=1

i

)2

= 4k2(

1

k(k + 1)2+

1

(k + 1)(k + 2)2+ . . .+

1

n(n+ 1)2

)= 4k2

(1

k(k + 1)+ . . .+

1

n(n+ 1)− 1

(k + 1)2− . . .− 1

(n+ 1)2

)< 4k2

(1

2

(1

k2+

1

(k + 1)2

)+ . . .+

1

2

(1

n2+

1

(n+ 1)2

)− 1

(k + 1)2− . . .− 1

(n+ 1)2

)= 4k2

(1

2k2+

1

2(n+ 1)2+

1

(k + 1)2+ . . .+

1

n2− 1

(k + 1)2− . . .− 1

(n+ 1)2

)= 4k2

(1

2k2− 1

2(n+ 1)2

)< 2,

for all k = 1, 2, . . . , n. In this case, it follows that

n∑k=1

kk∑j=1

aj

≤ 2

n∑i=1

1

ai,

which ends the proof.

Example 4. Given positive real numbers a1, a2, . . . , an. Prove that

4(a21 + a22 + . . .+ a2n) ≥ a21 +

(a1 + a2

2

)2

+ . . .+

(a1 + a2 + . . .+ an

n

)2

.

Solution: By the Cauchy Schwartz inequality, for all k = 1, 2, . . . , n, bi > 0, wehave (

a21b1

+a22b2

+ . . .+a2kbk

)(b1 + b2 + . . .+ bk) ≥ (a1 + a2 + . . .+ ak)

2,

53

Page 55: Олимпиадад бэлтгэгчдэд тусламж (Тэнцэтгэл биш 1)

It follows that(a1 + a2 + . . .+ ak

k

)2

≤ b1 + b2 + . . .+ bkk2

·(a21b1

+a22b2

+ . . .+a2kbk

),

orn∑k=1

(a1 + a2 + . . .+ ak

k

)2

≤n∑k=1

cka2k,

where

ck =b1 + b2 + . . .+ bk

k2bk+b1 + b2 + . . .+ bk+1

(k + 1)2bk+ . . .+

b1 + b2 + . . .+ bnn2bk

.

Now, choosing bk =√k −√k − 1, we obtain

ck =b1 + b2 + . . .+ bk

k2bk+b1 + b2 + . . .+ bk+1

(k + 1)2bk+ . . .+

b1 + b2 + . . .+ bnn2bk

=1√

k −√k − 1

(1

k3/2+

1

(k + 1)3/2+ . . .+

1

n3/2

)

≤ 1√k −√k − 1

2

1√k − 1

2

− 1√k + 1

2

+ . . .+ 2

1√n− 1

2

− 1√n+ 1

2

=

1√k −√k − 1

1√k − 1

2

− 1√n+ 1

2

<2√

k − 12

(√k −√k − 1

)=

2√

2(√

k +√k − 1

)√

2k − 1≤ 4.

The inequality is proved.

This last example, by far the hardest, requires some explanation - namely,how to arrive to the choice bk =

√k −√k − 1? There is no universal method

to choose these coefficients, and hard problems require a certain degree ofcreativity from the solver. In this case, there are several reasons that point tothe correct solution.One way would be to look at the equality case. We don’t know it (and infact it is never achieved unless all variables are zero), but we can produce an”asymptotic” case, i.e. an examples where the two expression are close to eachother. The choice to take all ai equal fails. Then, because the inequality isnot symmetric, one can try to choose ai as a function of i, and the easiestone is ai = ik. This is good, because the sum of ik is approximately ik+1

k+1 fork > −1 (this is done by tricks as the integral approximation or the Bernoulli

polynomials), so that a1 + . . .+ an would be approximately nk+1

k+1 .

One therefore gets 4∑i2k ∼ 4

2k+1n2k+1 (for 2k > −1) and we compare it to∑

( ik

k+1)2 ∼ 1(k+1)2(2k+1)

n2k+1. Therefore, in order for them to be equal we

need (k + 1)2 = 14 so k = −1

2 . Note that we cannot do any better because weneed 2k > −1 - in fact we are already on the border and violate this conditionfor k = −1

2 . This case is still very close to the equality anyway, as checkeddirectly.

54

Page 56: Олимпиадад бэлтгэгчдэд тусламж (Тэнцэтгэл биш 1)

So now if we want the method to work, we better choose bi that ”approxi-mately” respect the inequality case, i.e. that are ”approximately proportional”to ai = 1√

i. Such a choice is bi =

√i.

This one isn’t too nice however - the reason being that ck involves the sum1 + 1√

2+ . . .+ 1√

kwhich is approximately 2

√k but however is not precise. So

perhaps a nicer choice would be the one that makes the partial sum b1+. . .+bkactually equal to

√k (forgetting the unnecessary constant of 2) i.e. bk =√

k −√k − 1. These numbers are still approximately proportional to 1√

kas

√k −√k − 1 = k−(k−1)√

k+√k−1 = 1√

k+√k−1 ∼

12√k, and this is how the weights are

found.

Another idea is to apply the ”power guess” to the coefficients bk: one could tryto set bk = kα and then solve for α using the same estimates to get α = −1

2 .

Make it nonnegative Suppose one has to prove an inequality of the forma1b1

+a2b2

+ . . .+anbn≥ k, bi > 0 ∀n = 1, 2, . . . , n

It is similar to Cauchy Schwartz and Holder. However these inequalities maynot be applicable if the quantities ai are not positive. In some cases, it ispossible to resolve this inconvenience. The suggestion is to add up ai

bito mi so

that ai +mibi ≥ 0 and as small as possible (it is great if we choose mi so thatthe inequality ai +mibi ≥ 0 has equality case), then the inequality becomes

a′1b1

+a′2b2

+ . . .+a′nbn≥ k +m1 +m2 + . . .+mn.

where a′i ≥ 0, bi > 0 ∀i = 1, 2, . . . , n.

In this case, the Cauchy-Schwartz or Holder inequality can be applied. In thecase that ai ≥ 0 ∀n = 1, 2, . . . , n, one can subtract a positive quantity mi

from aibi

such that ai −mibi ≥ 0 and as small as possible, then we can applythe Cauchy Schwartz-Holder inequality more effectively than if we apply itdirectly (in some cases).

Example 5. Let a, b, c, d be real numbers such that a2 + b2 + c2 +d2 = 1. Provethat

1

1− ab+

1

1− bc+

1

1− cd+

1

1− da≤ 16

3.

Solution: Rewrite the inequality in the form(2− 1

1− ab

)+

(2− 1

1− bc

)+

(2− 1

1− cd

)+

(2− 1

1− da

)≥ 8

3,

or1− 2ab

1− ab+

1− 2bc

1− bc+

1− 2cd

1− cd+

1− 2da

1− da≥ 8

3.

Now, since 1− 2ab = a2 + b2 + c2 + d2− 2ab = (a− b)2 + c2 + d2 ≥ 0, applying

55

Page 57: Олимпиадад бэлтгэгчдэд тусламж (Тэнцэтгэл биш 1)

the Cauchy Schwartz inequality, we have

1− 2ab

1− ab+

1− 2bc

1− bc+

1− 2cd

1− cd+

1− 2da

1− da≥

≥ [(1− 2ab) + (1− 2bc) + (1− 2cd) + (1− 2da)]2

(1− 2ab)(1− ab) + (1− 2bc)(1− bc) + (1− 2cd)(1− cd) + (1− 2da)(1− da)

=4[2− (a+ c)(b+ d)]2

4− 3(a+ c)(b+ d) + 2(a2 + c2)(b2 + d2).

It suffices to prove that

3[2− (a+ c)(b+ d)]2 ≥ 2[4− 3(a+ c)(b+ d) + 2(a2 + c2)(b2 + d2)],

or

3(a+ c)2(b+ d)2 − 6(a+ c)(b+ d) + 4− 4(a2 + c2)(b2 + d2) ≥ 0,

3[1− (a+ c)(b+ d)]2 + 1− 4(a2 + c2)(b2 + d2) ≥ 0.

By the AM-GM inequality, we have

4(a2 + c2)(b2 + d2) ≤ (a2 + b2 + c2 + d2)2 = 1.

The inequality is proved. Equality holds if and only if a = b = c = d = ±12 .

Example 6. For any real numbers a, b, c, prove that

a2 − bca2 + 2b2 + 3c2

+b2 − ca

b2 + 2c2 + 3a2+

c2 − abc2 + 2a2 + 3b2

≥ 0.

Solution: Rewrite the inequality as

∑cyc

4(a2 − bc)a2 + 2b2 + 3c2

≥ 0,

or ∑cyc

[4(a2 − bc)

a2 + 2b2 + 3c2+ 1

]≥ 3,

2∑cyc

(b− c)2

a2 + 2b2 + 3c2+∑cyc

5a2 + c2

a2 + 2b2 + 3c2≥ 3.

Since 2∑cyc

(b− c)2

a2 + 2b2 + 3c2≥ 0, it suffices to show that

∑cyc

5a2 + c2

a2 + 2b2 + 3c2≥ 3,

but it follows from the Cauchy Schwartz inequality because(∑cyc

5a2 + c2

a2 + 2b2 + 3c2

)(∑cyc

(5a2 + c2)(a2 + 2b2 + 3c2)

)≥

[∑cyc

(5a2 + c2)

]2

= 36

(∑cyc

a2

)2

.

56

Page 58: Олимпиадад бэлтгэгчдэд тусламж (Тэнцэтгэл биш 1)

and

12

(∑cyc

a2

)2

−∑cyc

(5a2 + c2)(a2 + 2b2 + 3c2) = 4∑cyc

a4 − 4∑cyc

a2b2 ≥ 0.

Equality holds if and only if a = b = c.

Example 7. Let a, b, c be nonnegative real numbers, no two of which are zero.Prove that the following inequality holds

1

a2 + bc+

1

b2 + ca+

1

c2 + ab≥ 3(a+ b+ c)2

2(a2 + b2 + c2)(ab+ bc+ ca).

Solution: The inequality is equivalent to each of the following inequalities∑cyc

a2 + b2 + c2

a2 + bc≥ 3(a+ b+ c)2

2(ab+ bc+ ca),

∑cyc

(a2 + b2 + c2

a2 + bc− 1

)≥ 3(a+ b+ c)2

2(ab+ bc+ ca)− 3,

∑cyc

b2 + c2 − bca2 + bc

≥ 3(a2 + b2 + c2)

2(ab+ bc+ ca).

Now, by Cauchy Scwhartz inequality, we have that

∑cyc

b2 + c2 − bca2 + bc

(2∑cyc

a2 −∑cyc

ab

)2

∑cyc

(a2 + bc)(b2 + c2 − bc)

=

(2∑cyc

a2 −∑cyc

ab

)2

(∑cyc

ab

)2

+

(∑cyc

ab

)(∑cyc

a2

)− 4abc

∑cyc

a

.

It suffices for us to prove that(2∑cyc

a2 −∑cyc

ab

)2

(∑cyc

ab

)2

+

(∑cyc

ab

)(∑cyc

a2

)− 4abc

∑cyc

a

≥ 3(a2 + b2 + c2)

2(ab+ bc+ ca).

To prove this inequality, we will use the pqr technique (refer to the appropriateparagraph for a general description of the method). Because the inequality ishomogeneous, we can assume that a+b+c = 1. and set q = ab+bc+ca, r = abc.Then this inequality becomes

(2− 5q)2

q2 + q(1− 2q)− 4r≥ 3(1− 2q)

2q,

57

Page 59: Олимпиадад бэлтгэгчдэд тусламж (Тэнцэтгэл биш 1)

or(2− 5q)2

q − q2 − 4r≥ 3(1− 2q)

2q.

If 1 ≥ 4q, then we have

(2− 5q)2

q − q2 − 4r− 3(1− 2q)

2q≥ (2− 5q)2

q − q2− 3(1− 2q)

2q=

(5− 11q)(1− 4q)

2q(1− q)≥ 0.

If 4q ≥ 1, then from Schur’s inequality for fourth degree, we see that r ≥(4q−1)(1−q)

6 . Hence,

(2− 5q)2

q − q2 − 4r≥ (2− 5q)2

q − q2 − 4 · (4q−1)(1−q)6

=3(2− 5q)

1− q.

It remains to prove that

3(2− 5q)

1− q≥ 3(1− 2q)

2q,

which can be easily simplified to

3(4q − 1)(1− 3q)

2q(1− q)≥ 0.

Sometimes one has to prove the inequalitya1b1

+a2b2

+ . . . +anbn≤ k, bi > 0

∀n = 1, 2, . . . , n. We can prove this inequality by adding aibi

to mi such thatai +mibi is a complete square of a expression, then we can apply the Cauchy-Schwarts to each of these terms to obtain an upper bound.Example 8. Let a, b, c be real numbers. Prove that the following inequalityholds

a2 − bc4a2 + 4b2 + c2

+b2 − ca

4b2 + 4c2 + a2+

c2 − ab4c2 + 4a2 + b2

≥ 0.

Solution: Rewrite the inequality in the form

4(bc− a2)4a2 + 4b2 + c2

+4(ca− b2)

4b2 + 4c2 + a2+

4(ab− c2)4c2 + 4a2 + b2

≤ 0,

or[4(bc− a2)

4a2 + 4b2 + c2+ 1

]+

[4(ca− b2)

4b2 + 4c2 + a2+ 1

]+

[4(ab− c2)

4c2 + 4a2 + b2+ 1

]≤ 3,

or(2b+ c)2

4a2 + 4b2 + c2+

(2c+ a)2

4b2 + 4c2 + a2+

(2a+ b)2

4c2 + 4a2 + b2≤ 3.

Now, using Cauchy Schwartz inequality, we have

(2b+ c)2

4a2 + 4b2 + c2=

(b+ b+ c)2

(a2 + 2b2) + (a2 + 2b2) + (c2 + 2a2)

≤ b2

a2 + 2b2+

b2

a2 + 2b2+

c2

c2 + 2a2

=2b2

a2 + 2b2+

c2

c2 + 2a2.

58

Page 60: Олимпиадад бэлтгэгчдэд тусламж (Тэнцэтгэл биш 1)

Similarly, we have

(2c+ a)2

4b2 + 4c2 + a2≤ 2c2

b2 + 2c2+

a2

a2 + 2b2,

(2a+ b)2

4c2 + 4a2 + b2≤ 2a2

c2 + 2a2+

b2

b2 + 2c2.

Adding up these inequalities, we get the result.

Make it symmetric It is usually the case that a symmetric inequality iseasier than cyclic inequality. This technique is based on this intuitive idea.We will use the Cauchy Schwartz-Holder inequality to make the inequalitysymmetric - and then it will be easier to prove.

Example 9. Let a, b, c be nonnegative real numbers, no two of which are zero.Prove that √

a

a+ b+

√b

b+ c+

√c

c+ a≤ 3√

2.

Solution: By the Cauchy Schwartz inequality, we have(∑cyc

√a

a+ b

)2

[∑cyc

(a+ c)

][∑cyc

a

(a+ b)(a+ c)

]

=

4

(∑cyc

a

)(∑cyc

ab

)(a+ b)(b+ c)(c+ a)

.

Moreover, by the AM-GM inequality,

(a+ b)(b+ c)(c+ a) =

(∑cyc

a

)(∑cyc

ab

)− abc ≥ 8

9

(∑cyc

a

)(∑cyc

ab

).

Hence (∑cyc

√a

a+ b

)2

≤ 9

2.

Equality holds if and only if a = b = c.

Example 10. Let a, b, c be positive real numbers. Prove that√a

4a+ 4b+ c+

√b

4b+ 4c+ a+

√c

4c+ 4a+ b≤ 1.

Solution: By the Cauchy Schwartz inequality, we have(∑cyc

√a

4a+ 4b+ c

)2

[∑cyc

(4a+ b+ 4c)

][∑cyc

a

(4a+ 4b+ c)(4a+ b+ 4c)

]

=

9

(∑cyc

a

)(∑cyc

a2 + 8∑cyc

ab

)(4a+ 4b+ c)(4b+ 4c+ a)(4c+ 4a+ b)

.

59

Page 61: Олимпиадад бэлтгэгчдэд тусламж (Тэнцэтгэл биш 1)

It suffices to prove that

9

(∑cyc

a

)(∑cyc

a2 + 8∑cyc

ab

)≤ (4a+ 4b+ c)(4b+ 4c+ a)(4c+ 4a+ b),

or

7∑cyc

a3 + 3∑cyc

ab(a+ b) ≥ 39abc.

The last inequality is trivial by the AM-GM inequality. Equality holds if andonly if a = b = c.

Substitutions This technique is simply the application of the ubiquitoussubstitution method to the Cauchy-Schwartz and Holder inequalities.

Example 11. Let a, b, c be positive real numbers. Prove that

a3

a3 + abc+ b3+

b3

b3 + abc+ c3+

c3

c3 + abc+ a3≥ 1.

Solution: Set x =b

a, y =

a

c, z =

c

b, then x, y, z > 0, xyz = 1 and the inequality

becomes ∑cyc

1

x3 +x

y+ 1≥ 1,

or ∑cyc

1

x3 + x2z + 1≥ 1,

which can be rewritten as ∑cyc

yz

x2 + yz + zx≥ 1.

By the Cauchy Schwartz inequality, we have

∑cyc

yz

x2 + yz + zx≥

(∑cyc

yz

)2

∑cyc

yz(x2 + yz + zx)= 1.

Example 12. Let a, b, c be positive real numbers. Prove that√a2

a2 + 7ab+ b2+

√b2

b2 + 7bc+ c2+

√c2

c2 + 7ca+ a2≥ 1.

Solution: Set x =b

a, y =

c

b, z =

a

c, then x, y, z > 0, xyz = 1 and the inequality

becomes ∑cyc

1√x2 + 7x+ 1

≥ 1.

60

Page 62: Олимпиадад бэлтгэгчдэд тусламж (Тэнцэтгэл биш 1)

Again, since x, y, z > 0, xyz = 1 then there exist m,n, p > 0 such that x =n2p2

m4, y =

p2m2

n4, z =

m2n2

p4, the inequality is transformed into

∑cyc

m4√m8 + 7m4n2p2 + n4p4

≥ 1

By Holder’s inequality, we have(∑cyc

m4√m8 + 7m4n2p2 + n4p4

)2 [∑cyc

m(m8 + 7m4n2p2 + n4p4)

]≥

(∑cyc

m3

)3

.

It suffices to prove that(∑cyc

m3

)3

≥∑cyc

m(m8 + 7m4n2p2 + n4p4),

or ∑sym

(5m6n3 + 2m3n3p3 − 7m5n2p2) +∑sym

(m6n3 −m4n4p) ≥ 0.

The last inequality is obviously true from the AM-GM inequality.

Example 13. Let a, b, c be positive real numbers. Prove that

1

a(a+ b)+

1

b(b+ c)+

1

c(c+ a)≥ 3

23√a2b2c2

.

Solution: Due to the homogeneity, we may assume abc = 1, then there existx, y, z > 0 such that a = x

y , b = zx , c = y

z . The inequality becomes

∑cyc

y2

x2 + yz≥ 3

2.

By the Cauchy Schwartz inequality, we have

∑cyc

y2

x2 + yz≥

(∑cyc

y2

)2

∑cyc

x2y2 +∑cyc

y3z.

Moreover (∑cyc

y2

)2

− 3∑cyc

x2y2 =1

2

∑cyc

(x2 − y2)2 ≥ 0,

and (∑cyc

y2

)2

− 3∑cyc

y3z =1

2

∑cyc

(x2 − z2 − 2xy + yz + zx)2 ≥ 0.

61

Page 63: Олимпиадад бэлтгэгчдэд тусламж (Тэнцэтгэл биш 1)

Therefore

∑cyc

y2

x2 + yz≥

(∑cyc

y2

)2

∑cyc

x2y2 +∑cyc

y3z≥

(∑cyc

y2

)2

13

(∑cyc

y2

)2

+ 13

(∑cyc

y2

)2 =3

2.

Equality holds if and only if a = b = c.

The CYH technique This is the most advanced technique that we wantto present in this paper. It may not be applicable to such a wide range ofinequalities as others - however it solves some very hard problems. Here is, forexample, an inequality of Jack Garfunkel, famous for creating many difficultinequalities.

Example 14. Let a, b, c be nonnegative real numbers, no two of which are zero.Prove that

a√a+ b

+b√b+ c

+c√c+ a

≤ 5

4

√a+ b+ c.

Solution: By the Cauchy Schwartz inequality, we have(∑cyc

a√a+ b

)2

[∑cyc

a(5a+ b+ 9c)

][∑cyc

a

(a+ b)(5a+ b+ 9c)

]

= 5

(∑cyc

a

)2 [∑cyc

a

(a+ b)(5a+ b+ 9c)

].

It suffices to prove that(∑cyc

a

)[∑cyc

a

(a+ b)(5a+ b+ 9c)

]≤ 5

16.

This holds since since

5

16−

(∑cyc

a

)[∑cyc

a

(a+ b)(5a+ b+ 9c)

]=A+B

C.

where

A =∑cyc

ab(a+ b)(a+ 9b)(a− 3b)2 ≥ 0,

B = 243∑cyc

a3b2c+ 835∑cyc

a2b3c+ 232∑cyc

a4bc+ 1230a2b2c2 ≥ 0,

C = 16(a+ b)(b+ c)(c+ a)(5a+ b+ 9c)(5b+ c+ 9a)(5c+ a+ 9b) > 0.

Equality holds if and only if a3 = b

1 = c0 or any cyclic permutations (of course,

the expression c0 in the equality simply means c = 0).

62

Page 64: Олимпиадад бэлтгэгчдэд тусламж (Тэнцэтгэл биш 1)

Of course, this solution has a strategy behind it:

Similar to the symmetric technique, we want to apply the Cauchy Schwartzinequality like this

(∑cyc

a√a+ b

)2

[∑cyc

a(ma+ nb+ pc)

][∑cyc

a

(a+ b)(ma+ nb+ pc)

].

where m,n, p are nonnegative real numbers that we will choose later.

Now, note that the original inequality has equality for a = 3, b = 1, c = 0 thenwe must choose m,n, p so that this step, the equality also holds for this point.Moreover, due to the equality of Cauchy Schwartz inequality, this step, wehave equality if and only if

√a(ma+ nb+ pc)√

a

(a+ b)(ma+ nb+ pc)

=

√b(mb+ nc+ pa)√

b

(b+ c)(mb+ nc+ pa)

=

√c(mc+ na+ pb)√

c

(c+ a)(mc+ na+ pb)

.

We must choose m,n, p such that this equation has a root (a, b, c) = (3, 1, 0),that is√

3 · (3 ·m+ 1 · n+ 0 · p)√3

(3 + 1)(3 ·m+ 1 · n+ 0 · p)

=

√1 · (1 ·m+ 0 · n+ 3 · p)√

1

(1 + 0)(1 ·m+ 0 · n+ 3 · p)

=

√0 · (0 ·m+ 3 · n+ 1 · p)√

0

(0 + 3)(0 ·m+ 3n+ 1 · p)

,

or

2(3m+ n) = m+ 3p,

or

5m+ 2n = 3p.

Moreover, note that if the expression∑cyc

a(ma + nb + pc) = m∑cyc

a2 + (n +

p)∑cyc

ab has the form k

(∑cyc

a

)2

, then after using the Cauchy Schwartz in-

equality, it will be simpler thus it might be easier to proceed. Hence, let uschoose m,n, p such that n + p = 2m. Now, due to the homogeneity, we canchoose m = 5, n = 1, p = 9. That is the reason why we have the above solutionfor this very hard inequality.

Unfortunately, the second part of the proof does not have such a generalmethod behind it. Still, this method of choosing m,n, p can reduce the in-equality to something one may have more freedom to tackle. For example,with the the same Jack Garfunkel’s inequality, we can proceed in a slightlydifferent way :

From the same Cauchy Schwartz inequality (this time for (m,n, p) = (3, 3, 1)),

63

Page 65: Олимпиадад бэлтгэгчдэд тусламж (Тэнцэтгэл биш 1)

we have that(∑cyc

a√a+ b

)2

[∑cyc

a(3a+ 3b+ c)

a+ b

](∑cyc

a

3a+ 3b+ c

)

=

[2∑cyc

a+∑cyc

a(a+ b+ c)

a+ b

](∑cyc

a

3a+ 3b+ c

)

=

(∑cyc

a

)(∑cyc

a

3a+ 3b+ c

)(∑cyc

a

a+ b+ 2

),

hence it suffices for us to prove that(∑cyc

a

3a+ 3b+ c

)(∑cyc

a

a+ b+ 2

)≤ 25

16.

The detailed proof of the last inequality can be found in the following book:Old And New Inequalities 2, Vo Quoc Ba Can - Cosmin Pohoata, GIL pub-lishing house, 2008.

Example 15. Let a, b, c be nonnegative real numbers, no two of which are zero.Prove that

(ab+ bc+ ca)

(1

(b+ c)2+

1

(c+ a)2+

1

(a+ b)2

)≥ 9

4.

Solution: The appearance of∑cyc

1

(b+ c)2suggests the Cauchy Schwartz in-

equality (since it is the sum of squares). Let us use the above idea to attackthis inequality.

By the Cauchy Schwartz inequality, we have[∑cyc

(ma+ nb+ nc)2

][∑cyc

1

(b+ c)2

]≥

(∑cyc

ma+ nb+ nc

b+ c

)2

.

The equality holds if and only if

ma+ nb+ nc1

b+ c

=mb+ nc+ na

1

c+ a

=mc+ na+ nb

1

a+ b

.

We notice that the original inequality has an equality case for a = b = 1, c = 0,hence the above solution must be satisfied at this point, that is

m+ n

1=m+ n

1=

2n12

⇔ m = 3n⇒ m = 3, n = 1.

And now, we have the solution as follows:

64

Page 66: Олимпиадад бэлтгэгчдэд тусламж (Тэнцэтгэл биш 1)

By the Cauchy Schwartz inequality, we have(11∑cyc

a2 + 14∑cyc

ab

)[∑cyc

1

(b+ c)2

]=

[∑cyc

(3a+ b+ c)2

][∑cyc

1

(b+ c)2

]

(∑cyc

3a+ b+ c

b+ c

)2

= 9

(1 +

∑cyc

a

b+ c

)2

.

It suffices to prove that

4

(1 +

∑cyc

a

b+ c

)2

≥11∑cyca2 + 14

∑cycab∑

cycab

.

Due to the homogeneity, we may assume a+ b+ c = 1, setting q =∑cyc

ab, r =

abc, then by Schur’s inequality for third degree, we obtain r ≥ max

{0,

4q − 1

9

}.

The inequality becomes

4

(1 + q

q − r− 2

)2

≥ 11− 8q

q.

If 1 ≥ 4q, then

4

(1 + q

q − r− 2

)2

−11− 8q

q≥ 4

(1 + q

q− 2

)2

−11− 8q

q=

(4− 3q)(1− 4q)

q2≥ 0.

If 4q ≥ 1, then

4

(1 + q

q − r− 2

)2

− 11− 8q

q≥ 4

1 + q

q − 4q − 1

9

− 2

2

− 11− 8q

q

=(1− 3q)(4q − 1)(11− 17q)

q(5q + 1)2≥ 0.

Equality holds if and only if a = b = c or a = b, c = 0 and any cyclicpermutations.

Example 16. Let a, b, c, d be positive real numbers such that

(a+ b+ c+ d)

(1

a+

1

b+

1

c+

1

d

)= 20.

Prove that

(a2 + b2 + c2 + d2)

(1

a2+

1

b2+

1

c2+

1

d2

)≥ 36.

65

Page 67: Олимпиадад бэлтгэгчдэд тусламж (Тэнцэтгэл биш 1)

Solution: By the Cauchy Schwartz inequality, we have(∑cyc

1

a2

)[∑cyc

(b+ c+ d− a)2

]≥

(∑cyc

b+ c+ d− aa

)2

= 144.

Moreover, we see that

4∑cyc

a2 =∑cyc

(b+ c+ d− a)2.

Sometimes, the above way to choose m,n, p, . . . cannot be used, and we needto another way to choose them. A nice idea to apply the Cauchy Schwartzinequality is by taken note at some well-known inequalities or taken note atthe special form of the given inequalities. Here are some examples.

Example 17. Let a, b, c be positive real numbers. Prove that for all k ≥ 2, wehave ∑

cyc

√a2 + kab+ b2 ≤

√4∑cyc

a2 + (3k + 2)∑cyc

ab.

Solution: By the Cauchy Schwartz inequality, we have(∑cyc

√a2 + kab+ b2

)2

[∑cyc

(a+ b)

](∑cyc

a2 + kab+ b2

a+ b

)

= 2

(∑cyc

a

)(∑cyc

a2 + kab+ b2

a+ b

).

We need to prove that

2∑cyc

a2 + kab+ b2

a+ b≤

4∑cyc

a2 + (3k + 2)∑cyc

ab∑cyc

a,

This inequality is equivalent to each of the following

2∑cyc

(a+ b) + 2(k − 2)∑cyc

ab

a+ b≤ 4

∑cyc

a+

3(k − 2)∑cyc

ab∑cyc

a,

2∑cyc

ab

a+ b≤

3∑cyc

ab∑cyc

a,

2∑cyc

ab(a+ b+ c)

a+ b≤ 3

∑cyc

ab,

66

Page 68: Олимпиадад бэлтгэгчдэд тусламж (Тэнцэтгэл биш 1)

2abc∑cyc

1

a+ b≤∑cyc

ab.

And the last inequality follows from the Cauchy Schwartz inequality

2abc∑cyc

1

a+ b≤ 2abc

∑cyc

(1

4a+

1

4b

)=∑cyc

ab.

Equality holds if and only if a = b = c.

Example 18. Let a, b, c be positive real numbers such that a+ b+ c = 1. Provethat

a√a+ 2b

+b√

b+ 2c+

c√c+ 2a

≤√

3

2.

Solution: At this point, we can easily show that

a

2a+ 4b+ c+

b

2b+ 4c+ a+

c

2c+ 4a+ b≤ 1

2,

with equality holding when abc = 0. (this is left as an exercise)Thus, using the Cauchy Schwartz inequality, we have that(∑

cyc

a√a+ 2b

)2

(∑cyc

a

2a+ 4b+ c

)[∑cyc

a(2a+ 4b+ c)

a+ 2b

]

≤ 1

2

∑cyc

a(2a+ 4b+ c)

a+ 2b

=1

2

(2∑cyc

a+∑cyc

ca

a+ 2b

)

≤ 1

2

(2∑cyc

a+∑cyc

ca

a

)=

3

2

∑cyc

a =3

2.

Exercises

1. Let x, y ≥ 0 such that x3 + y3 = 1. Prove that

√x+ 2

√y ≤ 6

√(1 + 2

5√

2)5.

Solution: By Holder’s inequality, we have that for all a, b ≥ 0,(a6 + b6

)5 (x3 + y3

)≥(a5√x+ b5

√y)6.

Hence,

a5√x+ b5

√y ≤ 6

√(a6 + b6)5 (x3 + y3) =

6

√(a6 + b6)5.

Choosing a = 1, b = 5√

2, then we have

√x+ 2

√y ≤ 6

√[16 +

(5√

2)6]5

=6

√(1 + 2

5√

2)5.

67

Page 69: Олимпиадад бэлтгэгчдэд тусламж (Тэнцэтгэл биш 1)

2. Let a, b, c ≥ 0 such that a2 + b2 + c2 = 1. Find the minimum value of

P = a3 + 3b3 + 2c3.

Solution: By Holder’s inequality, we have that for all m,n, p ≥ 0,

(a3 + 3b3 + 2c3)2(m3 + n3 + p3) ≥(a2m+ b2n

3√

9 + c2p3√

4)3.

Choosing m = 1, n = 13√9, p = 1

3√4, then we have

(a3 + 3b3 + 2c3)2 ≥ (a2 + b2 + c2)3

1 +1

9+

1

4

=36

49,

or

P = a3 + 3b3 + 2c3 ≥ 6

7.

We have equality when

a

m=b 3√

3

n=c 3√

2

p,

ora2

m2=b2 3√

9

n2=c2 3√

4

p2=

a2 + b2 + c2

m2 +n2

3√

9+

p2

3√

4

=36

49,

or

a =6

7, b =

6

7 3√

9, c =

6

7 3√

4.

Hence

minP =6

7.

3. Given a, b, c ≥ 0 satisfying a+ b+ c = 3. Determine the minimum valueof

P = a4 + 2b4 + 4c4.

Solution: By Holder’s inequality, we have that for all m,n, p ≥ 0,

(a4 + 2b4 + 3c4)(m4 + n4 + p4)3 ≥(am3+bn3

4√

2+cp34√

3)4.

Choosing m = 1, n = 112√2

, b = 112√3

, then we have

P = a4 + 2b4 + 3c4 ≥ (a+ b+ c)4[14 +

(1

12√

2

)4

+

(1

12√

3

)4]3

=1(

1 +13√

2+

13√

3

)3 .

68

Page 70: Олимпиадад бэлтгэгчдэд тусламж (Тэнцэтгэл биш 1)

We have equality when

a

m=b 4√

2

n=c 4√

3

p=

a+ b+ c

m+n4√

2+

p4√

3

=1

1 +13√

2+

13√

3

,

or

a =1

1 +13√

2+

13√

3

, b =1

3√

2

(1 +

13√

2+

13√

3

) , c =1

3√

3

(1 +

13√

2+

13√

3

) .Hence

minP =1(

1 +13√

2+

13√

3

)3 .

4. Let x1, x2, . . . , xn be real numbers. Prove that

x21+(x1+x2)2+. . .+(x1+x2+. . .+xn)2 ≤ 1

4 sin2 π

2(2n+ 1)

·(x21+x22+. . .+x2n).

Solution: By Cauchy Schwartz inequality, we have that for all ci > 0,

n∑k=1

(k∑i=1

xi

)2

≤n∑k=1

[(k∑i=1

ci

)(k∑i=1

x2ici

)]

=

n∑k=1

[Sk

(k∑i=1

x2ici

)]=

n∑k=1

Si + . . .+ Snci

x2i .

Choosing ci such that

S1 + S2+ . . .+Snc1

=S2 + . . .+ Sn

c2= . . . =

Sncn,

then we obtain ci = siniπ

2n+ 1− sin

(i− 1)π

2n+ 1, and

S1 + S2+ . . .+Snc1

=S2 + . . .+ Sn

c2= . . . =

Sncn

=1

4 sin2 π

2(2n+ 1)

.

Thusn∑k=1

(k∑i=1

xi

)2

≤ 1

4 sin2 π

2(2n+ 1)

n∑k=1

x2i .

5. Let a, b, c ≥ 0 such that a+ b+ c = 1. Determine the minimum value of

P =

√a2 +

1

b2+

√b2 +

1

c2+

√c2 +

1

a2.

69

Page 71: Олимпиадад бэлтгэгчдэд тусламж (Тэнцэтгэл биш 1)

Solution: We guess that the minimum attains equality when a = b = c =1

3. Hence, by Cauchy Schwartz Inequality, we have that for all m,n ≥ 0,√(

a2 +1

b2

)(m2 + n2) ≥ ma+

n

b,√(

b2 +1

c2

)(m2 + n2) ≥ mb+

n

c,√(

c2 +1

a2

)(m2 + n2) ≥ mc+

n

a.

Therefore

P√m2 + n2 ≥ m(a+ b+ c) + n

(1

a+

1

b+

1

c

)≥ m(a+ b+ c) +

9n

a+ b+ c= m+ 9n,

or

P ≥ m+ 9n√m2 + n2

.

Choosing m,n > 0 such that

a = b = c =1

3,

a

m=

1

bn,

b

m=

1

cn,

c

m=

1

an.

Then n = 9m, and we may choose m = 1, n = 9 to obtain

P ≥ 1 + 9 · 9√12 + 92

=√

82.

Thus minP =√

82.

6. Let a, b, c > 0 such that a2 + b2 + c2 = 3. Prove that

1

2− a+

1

2− b+

1

2− c≥ 3.

Solution: The inequality is equivalent to(2

2− a− 1

)+

(2

2− b− 1

)+

(2

2− c− 1

)≥ 3,

ora

2− a+

b

2− b+

c

2− c≥ 3.

By the Cauchy Schwartz inequality and AM-GM inequality, we have

a

2− a+

b

2− b+

c

2− c=

a4

2a3 − a4+

b4

2b3 − b4+

c4

2c3 − c4

≥ (a2 + b2 + c2)2

2a3 + 2b3 + 2c2 − a4 − b4 − c4

≥ (a2 + b2 + c2)2

(a4 + a2) + (b4 + b2) + (c4 + c2)− a4 − b4 − c4

= a2 + b2 + c2 = 3.

70

Page 72: Олимпиадад бэлтгэгчдэд тусламж (Тэнцэтгэл биш 1)

7. Let a, b, c be the side lengths of a triangle. Prove that

a(a− b)a2 + 2bc

+b(b− c)b2 + 2ca

+c(c− a)

c2 + 2ab≥ 0.

Solution: The inequality is equivalent to∑cyc

[a(a− b)a2 + 2bc

+ 1

]≥ 3,

or ∑cyc

2a2 − ab+ 2bc

a2 + 2bc≥ 3.

Since a, b, c are the side lengths of a triangle, we have c ≥ b− a, then

2a2 − ab+ 2bc ≥ 2a2 − ab+ 2b(b− a) = 2(a− b)2 + ab ≥ 0.

Hence, by the Cauchy Schwartz inequality, we have

∑cyc

2a2 − ab+ 2bc

a2 + 2bc≥

[∑cyc

(2a2 − ab+ 2bc)

]2∑cyc

(2a2 − ab+ 2bc)(a2 + 2bc)

It suffices to prove that[∑cyc

(2a2 − ab+ 2bc)

]2≥ 3

∑cyc

(2a2 − ab+ 2bc)(a2 + 2bc),

or equivalently,

7∑cyc

a3b+ 4∑cyc

ab3 ≥ 2∑cyc

a4 + 3∑cyc

a2b2 + 6∑cyc

a2bc.

Again, since a, b, c are the side lengths of a triangle, there exist x, y, z > 0such that a = y + z, b = z + x, c = x+ y. The inequality becomes

2∑cyc

x4 + 2∑cyc

xy(x2 + y2) + 3∑cyc

xy3 ≥ 6∑cyc

x2y2 + 3∑cyc

x2yz,

But it follows from the AM-GM inequality since

2∑cyc

x4 ≥2∑cyc

x2y2, 2∑cyc

xy(x2 + y2) ≥ 4∑cyc

x2y2, 3∑cyc

xy3 ≥ 3∑cyc

x2yz.

Equality holds if and only if a = b = c.

8. Let a, b, c be nonnegative real numbers, no two of which are zero. Provethat

2a2 − bcb2 − bc+ c2

+2b2 − ca

c2 − ca+ a2+

2c2 − aba2 − ab+ b2

≥ 3.

71

Page 73: Олимпиадад бэлтгэгчдэд тусламж (Тэнцэтгэл биш 1)

Solution: Rewrite the inequality as

∑cyc

[2a2 − bc

b2 − bc+ c2+ 1

]≥ 6,

or ∑cyc

2a2 + (b− c)2

b2 − bc+ c2≥ 6.

By the Cauchy Schwartz inequality, we have

∑cyc

2a2 + (b− c)2

b2 − bc+ c2≥

[∑cyc

[2a2 + (b− c)2]

]2∑cyc

[2a2 + (b− c)2](b2 − bc+ c2).

It suffices to prove that[∑cyc

[2a2 + (b− c)2]

]2≥ 6

∑cyc

[2a2 + (b− c)2](b2 − bc+ c2),

or

2∑cyc

a4 + 2abc∑cyc

a+∑cyc

ab(a2 + b2) ≥ 6∑cyc

a2b2,

that is

2∑cyc

a2(a− b)(a− c) + 3∑cyc

ab(a− b)2 ≥ 0.

which is true by Schur’s inequality for fourth degree. Equality holds ifand only if a = b = c or a = b, c = 0 and its cyclic permutations.

9. Let a, b, c be nonnegative real numbers, not all are zero. Prove that

3a2 − bc2a2 + b2 + c2

+3b2 − ca

2b2 + c2 + a2+

3c2 − ab2c2 + a2 + b2

≤ 3

2.

Solution: Rewrite the inequality as

∑cyc

[3− 2(3a2 − bc)

2a2 + b2 + c2

]≥ 6,

or ∑cyc

3b2 + 2bc+ 3c2

2a2 + b2 + c2≥ 6,

that is

3∑cyc

(b− c)2

2a2 + b2 + c2+ 8

∑cyc

bc

2a2 + b2 + c2≥ 6.

72

Page 74: Олимпиадад бэлтгэгчдэд тусламж (Тэнцэтгэл биш 1)

If (a−b)2+(b−c)2+(c−a)2 = 0, it is trivial. If (a−b)2+(b−c)2+(c−a)2 >0, then by the Cauchy Schwartz inequality, we have

∑cyc

(b− c)2

2a2 + b2 + c2≥

[∑cyc

(b− c)2]2

∑cyc

(b− c)2(2a2 + b2 + c2)

=

4

(∑cyc

a2 −∑cyc

ab

)2

[∑cyc

(b− c)2](∑

cyc

a2

)+∑cyc

a2(b− c)2

=

2

(∑cyc

a2 −∑cyc

ab

)2

(∑cyc

a2 −∑cyc

ab

)(∑cyc

a2

)+∑cyc

b2c2 −∑cyc

a2bc

,

and

∑cyc

bc

2a2 + b2 + c2≥

(∑cyc

bc

)2

∑cyc

bc(2a2 + b2 + c2)=

(∑cyc

bc

)2

(∑cyc

bc

)(∑cyc

a2

)+∑cyc

a2bc

.

It suffices to prove that

6

(∑cyc

a2 −∑cyc

ab

)2

(∑cyc

a2 −∑cyc

ab

)(∑cyc

a2

)+∑cyc

b2c2 −∑cyc

a2bc

+

8

(∑cyc

bc

)2

(∑cyc

bc

)(∑cyc

a2

)+∑cyc

a2bc

≥ 6.

Due to the homogeneity, we may assume a + b + c = 1. Putting q =∑cyc

bc, r = abc, then the inequality becomes

3(1− 3q)2

(1− 3q)(1− 2q) + q2 − 3r+

4q2

q(1− 2q) + r≥ 3,

or(3r + q − 4q2)2

(1− 5q + 7q2 − 3r)(q − 2q2 + r)≥ 0,

which is true.

10. Let a, b, c be the side lengths of a triangle. Prove that

a(b+ c)

a2 + 2bc+b(c+ a)

b2 + 2ca+c(a+ b)

c2 + 2ab≤ 2.

73

Page 75: Олимпиадад бэлтгэгчдэд тусламж (Тэнцэтгэл биш 1)

Solution: The inequality is equivalent to∑cyc

[1− a(b+ c)

a2 + 2bc

]≥ 1,

or ∑cyc

a2 − ab− ac+ 2bc

a2 + 2bc≥ 1

We will show that a2 − ab − ac + 2bc ≥ 0, or a(a − b) + (2b − a)c ≥ 0.Indeed, if 2b ≥ a, we have

a(a− b) + (2b− a)c ≥ a(a− b) + (2b− a)(b− a) = 2(a− b)2 ≥ 0.

If a ≥ 2b, we have

a(a− b) + (2b− a)c ≥ a(a− b) + (2b− a)(a+ b) = 2b2 ≥ 0.

Now, by the Cauchy Schwartz inequality, we have

∑cyc

a2 − ab− ac+ 2bc

a2 + 2bc≥

[∑cyc

(a2 − ab− ac+ 2bc

)]2∑cyc

(a2 − ab− ac+ 2bc

)(a2 + 2bc)

=

(∑cyc

a2

)2

∑cyc

(a2 − ab− ac+ 2bc

)(a2 + 2bc)

.

It suffices to show that(∑cyc

a2

)2

≥∑cyc

(a2 − ab− ac+ 2bc

)(a2 + 2bc),

which can be easily simplified to∑cyc

ab(a− b)2 ≥ 0.

which is obviously true. Equality holds if and only if a = b = c ora = b, c = 0 and its cyclic permutations.

11. Let a, b, c be the side lengths of a triangle. Prove that

a

b+ c+

b

c+ a+

c

a+ b+ab+ bc+ ca

a2 + b2 + c2≤ 5

2.

Solution: The inequality is equivalent to

∑cyc

(1− a

b+ c

)≥ 1

2+

∑cyc

ab∑cyc

a2,

74

Page 76: Олимпиадад бэлтгэгчдэд тусламж (Тэнцэтгэл биш 1)

or

∑cyc

b+ c− ab+ c

(∑cyc

a

)2

2∑cyc

a2.

By the Cauchy Schwartz inequality, we have

2

(∑cyc

a2

)(∑cyc

b+ c− ab+ c

)=

[∑cyc

(b+ c)(b+ c− a)

](∑cyc

b+ c− ab+ c

)

[∑cyc

(b+ c− a)

]2=

(∑cyc

a

)2

.

Equality holds if and only if a = b = c or a = b, c = 0 or any cyclicpermutations.

12. Let a, b, c be the side lengths of a triangle. Prove that

a

3a− b+ c+

b

3b− c+ a+

c

3c− a+ b≥ 1.

Solution: We have4a

3a− b+ c=

a+ b− c3a− b+ c

+ 1. Hence, it suffices to

prove thata+ b− c3a− b+ c

+b+ c− a3b− c+ a

+c+ a− b3c− a+ b

≥ 1,

which is obviously true since by Cauchy Schwartz inequality, we have

a+ b− c3a− b+ c

+b+ c− a3b− c+ a

+c+ a− b3c− a+ b

≥ (a+ b+ c)2∑cyc

(a+ b− c)(3a− b+ c)

=(a+ b+ c)2∑

cyc

a2 + 2∑cyc

ab= 1.

13. Let x1, x2, . . . , xn be positive real numbers such thatn∑i=1

xi = 1. Prove

that the following inequality holds

n∑i=1

√x2i + x2i+1 ≤ 2− 1

√2

2+

n∑i=1

x2ixi+1

.

Solution: The original inequality is equivalent to

n∑i=1

(xi + xi+1 −

√x2i + x2i+1

)≥ 1√

2

2+

n∑i=1

x2ixi+1

,

75

Page 77: Олимпиадад бэлтгэгчдэд тусламж (Тэнцэтгэл биш 1)

orn∑i=1

x2ix2ixi+1

+ xi +xixi+1

√x2i + x2i+1

≥ 1

√2 + 2

n∑i=1

x2ixi+1

.

By the Cauchy Schwartz inequality, we have that

n∑i=1

x2ix2ixi+1

+ xi +xixi+1

√x2i + x2i+1

(n∑i=1

xi

)2

n∑i=1

(x2ixi+1

+ xi +xixi+1

√x2i + x2i+1

)=

1n∑i=1

(x2ixi+1

+ xi +xixi+1

√x2i + x2i+1

) .It suffices to prove that

n∑i=1

(xixi+1

√x2i + x2i+1 −

x2ixi+1

)≤√

2− 1,

orn∑i=1

xixi+1√x2i + x2i+1 + xi

≤√

2− 1.

Since√x2i + x2i+1 ≥

xi + xi+1√2

, it remains to show that

n∑i=1

xixi+1(1 +√

2)xi + xi+1

≤ 1−√

2

2,

or

n∑i=1

xixi+1(1 +√

2)xi + xi+1

≤n∑i=1

(3− 2

√2

2xi +

√2− 1

2xi+1

),

that isn∑i=1

(√2− 1

)(xi − xi+1)

2

2[(

1 +√

2)xi + xi+1

] ≥ 0.

The proof is complete. Equality holds if and only if x1 = x2 = . . . =

xn =1

n.

14. Let a, b, c be positive real numbers. Prove that

a4

a2 + ab+ b2+

b4

b2 + bc+ c2+

c4

c2 + ca+ a2≥ a3 + b3 + c3

a+ b+ c.

Solution: Rewrite the inequality in the form∑cyc

(a4

a2 + ab+ b2+ ab− a2

)≥ a3 + b3 + c3

a+ b+ c+ab+bc+ca−a2−b2−c2,

76

Page 78: Олимпиадад бэлтгэгчдэд тусламж (Тэнцэтгэл биш 1)

or ∑cyc

ab3

a2 + ab+ b2≥ 3abc

a+ b+ c,

or ∑cyc

b2

a2 + b2 + ab

ab

≥ 3abc

a+ b+ c.

By the Cauchy Schwartz inequality, we have∑cyc

b2

a2 + b2 + ab

ab

≥ (a+ b+ c)2∑cyc

a2 + b2 + ab

ab

=abc(a+ b+ c)

ab+ bc+ ca.

It suffices to prove that

abc(a+ b+ c)

ab+ bc+ ca≥ 3abc

a+ b+ c,

or(a+ b+ c)2 ≥ 3(ab+ bc+ ca),

which is obviously true by the AM-GM inequality.

15. Let a, b, c be positive real numbers such that a+ b+ c = 1. Prove that

ab√ab+ bc

+bc√

bc+ ca+

ca√ca+ ab

≤ 1√2.

Solution: By the Cauchy Schwartz inequality, we have(∑cyc

ab√ab+ bc

)2

=

[∑cyc

√a+ b ·

√a2b

(a+ b)(a+ c)

]2

[∑cyc

(a+ b)

][∑cyc

a2b

(a+ b)(a+ c)

]

= 2∑cyc

a2b

(a+ b)(a+ c),

hence it suffices to prove that

4∑cyc

a2b

(a+ b)(a+ c)≤ a+ b+ c,

which is equivalent to

4a2b(b+ c) + 4b2c(c+ a) + 4c2a(a+ b) ≤ (a+ b)(b+ c)(c+ a)(a+ b+ c),

this last one being true, because it can be written as

ab(a− b)2 + bc(b− c)2 + ca(c− a)2 ≥ 0.

77

Page 79: Олимпиадад бэлтгэгчдэд тусламж (Тэнцэтгэл биш 1)

16. Let a, b, c be nonnegative real numbers, no two of which are zero. Provethat √

a2

4a2 + ab+ 4b2+

√b2

4b2 + bc+ 4c2+

√c2

4c2 + ca+ 4a2≤ 1.

Solution: By the Cauchy Schwartz inequality, we have(∑cyc

√a2

4a2 + ab+ 4b2

)2

[∑cyc

(4a2 + ac+ 4c2)

][∑cyc

a2

(4a2 + ab+ 4b2)(4a2 + ac+ 4c2)

].

It suffices to prove that[∑cyc

(4a2 + ac+ 4c2)

][∑cyc

a2

(4a2 + ab+ 4b2)(4a2 + ac+ 4c2)

]≤ 1.

By expanding, we can see the inequality is equivalent to

8∑cyc

a3b3 + 8∑cyc

a4bc+ 3abc∑cyc

ab(a+ b) ≥ 66a2b2c2.

which follows from the AM-GM inequality.

17. Let a, b, c be positive real numbers. Prove that

1

a√a+ b

+1

b√b+ c

+1

c√c+ a

≥ 3√2abc

.

Solution: Due to the homogeneity, we may assume abc = 1, then there

exist x, y, z > 0 such that a =x

y, b =

z

x, c =

y

z. The inequality becomes

∑cyc

y√y√

x(x2 + yz)≥ 3√

2

By the Cauchy Schwartz inequality,

∑cyc

y√y√

x(x2 + yz)≥

(∑cyc

y

)2

∑cyc

√xy(x2 + yz)

,

and∑cyc

√xy(x2 + yz) ≤

√√√√(∑cyc

xy

)(∑cyc

x2 +∑cyc

xy

)

=1√2

√√√√(2∑cyc

xy

)(∑cyc

x2 +∑cyc

xy

)

≤ 1

2√

2

(∑cyc

x2 + 3∑cyc

xy

)≤ 2

3√

2

(∑cyc

x

)2

.

78

Page 80: Олимпиадад бэлтгэгчдэд тусламж (Тэнцэтгэл биш 1)

Hence ∑cyc

y√y√

x(x2 + yz)≥ 3√

2.

Equality holds if and only if a = b = c.

18. Let a, b, c be positive real numbers such that abc = 1. Prove that

1

a2 − a+ 1+

1

b2 − b+ 1+

1

c2 − c+ 1≤ 3.

Solution: First of all, we will show that for any x, y, z > 0 which satis-fying xyz = 1

1

x2 + x+ 1+

1

y2 + y + 1+

1

z2 + z + 1≥ 1.

Indeed, since x, y, z > 0 and xyz = 1 then there exist m,n, p > 0 such

that x =np

m2, y =

pm

n2, z =

mn

p2. The inequality is transformed into

∑cyc

m4

m4 +m2np+ n2p2≥ 1.

From the Cauchy Schwartz inequality and the known∑cyc

n2p2 ≥∑cyc

m2np,

we have

∑cyc

m4

m4 +m2np+ n2p2≥

(∑cyc

m2

)2

∑cyc

(m4 +m2np+ n2p2)≥

(∑cyc

m2

)2

∑cyc

m4 + 2∑cyc

n2p2= 1.

Back to the original problem, from the statement above, we have∑cyc

11

a4+

1

a2+ 1≥ 1,

or ∑cyc

a4

a4 + a2 + 1≥ 1,

∑cyc

2(a2 + 1)

a4 + a2 + 1≤ 4,

∑cyc

(a2 + a+ 1) + (a2 − a+ 1)

(a2 + a+ 1)(a2 − a+ 1)≤ 4,

∑cyc

1

a2 + a+ 1+∑cyc

1

a2 − a+ 1≤ 4.

79

Page 81: Олимпиадад бэлтгэгчдэд тусламж (Тэнцэтгэл биш 1)

Using the above statement again, we have∑cyc

1

a2 + a+ 1≥ 1, and so

∑cyc

1

a2 − a+ 1≤ 3.

Equality holds if and only if a = b = c = 1.

19. Let a, b, c be nonnegative real numbers, no two of which are zero. Provethat

(b+ c)2

a2 + bc+

(c+ a)2

b2 + ca+

(a+ b)2

c2 + ab≥ 6.

Solution: By the Cauchy Schwartz inequality, we obtain[∑cyc

(b+ c)2

a2 + bc

][∑cyc

(b+ c)2(a2 + bc)

]≥

[∑cyc

(b+ c)2

]2

= 4

(∑cyc

a2 +∑cyc

ab

)2

.

Hence, it suffices to show that

2

(∑cyc

a2 +∑cyc

ab

)2

≥ 3∑cyc

(b+ c)2(a2 + bc),

or equivalently,

2∑cyc

a4 + 2abc∑cyc

a+∑cyc

ab(a2 + b2)− 6∑cyc

a2b2 ≥ 0.

This inequality follows by summing Schur’s inequality of fourth degree

2∑cyc

a4 + 2abc∑cyc

a ≥ 2∑cyc

ab(a2 + b2)

to the inequality

3∑cyc

ab(a2 + b2) ≥ 6∑cyc

a2b2.

which follows from the AM-GM inequality. Equality holds if a = b = cor a = b, c = 0 and its cyclic permutations.

20. Let a, b, c be nonnegative real numbers, no two of which are zero. Provethat

1√4a2 + bc

+1√

4b2 + ca+

1√4c2 + ab

≥ 2√ab+ bc+ ca

.

Solution: By Holder’s inequality, we have(∑cyc

1√4a2 + bc

)2 [∑cyc

(b+ c)3(4a2 + bc)

]≥

[∑cyc

(b+ c)

]3

= 8

(∑cyc

a

)3

.

80

Page 82: Олимпиадад бэлтгэгчдэд тусламж (Тэнцэтгэл биш 1)

It suffices to prove that

2

(∑cyc

a

)3(∑cyc

ab

)≥∑cyc

(b+ c)3(4a2 + bc),

or ∑cyc

ab(a3 + b3)−∑cyc

a2b2(a+ b) + 14abc∑cyc

a2 ≥ 0,

or ∑cyc

ab(a− b)2(a+ b) + 14abc∑cyc

a2 ≥ 0,

which is trivial. Equality holds if and only if a = b, c = 0 and its cyclicpermutations.

21. Let a, b, c be nonnegative real numbers, not all are zero. Prove that

(a+ b)2

a2 + 2b2 + 3c2+

(b+ c)2

b2 + 2c2 + 3a2+

(c+ a)2

c2 + 2a2 + 3b2≥ 3

2.

Solution: By the Cauchy Schwartz inequality, we have

∑cyc

(a+ b)2

a2 + 2b2 + 3c2≥

[∑cyc

(a+ b)(2a+ b)

]2∑cyc

(2a+ b)2(a2 + 2b2 + 3c2)

=

9

(∑cyc

a2 +∑cyc

ab

)2

∑cyc

(2a+ b)2(a2 + 2b2 + 3c2).

It suffices to prove that

6

(∑cyc

a2 +∑cyc

ab

)2

≥∑cyc

(2a+ b)2(a2 + 2b2 + 3c2),

which can be easily simplified to

4∑cyc

a3b+ 2∑cyc

ab3 − 3∑cyc

a2b2 + 6abc∑cyc

a ≥ 0,

or

2∑cyc

ab(a− b)2 + 2∑cyc

a3b+∑cyc

a2b2 + 6abc∑cyc

a ≥ 0,

which is obviously true. Equality holds if and only ifa

1=b

0=c

0and its

cyclic permutations.

81

Page 83: Олимпиадад бэлтгэгчдэд тусламж (Тэнцэтгэл биш 1)

22. Let a, b, c be nonnegative real numbers satisfying a + b + c = 1, andmoreover from which at least two are nonzero. Prove that

a√

4b2 + c2 + b√

4c2 + a2 + c√

4a2 + b2 ≤ 3

4.

Solution: With the help of the Cauchy Schwartz inequality, we get(∑cyc

a√

4b2 + c2

)2

[∑cyc

a(2b+ c)

][∑cyc

a(4b2 + c2)

2b+ c

]

= 3

(∑cyc

ab

)[∑cyc

a(4b2 + c2)

2b+ c

],

hence it suffices to prove that

3

16(ab+ bc+ ca)≥ a(4b2 + c2)

2b+ c+b(4c2 + a2)

2c+ a+c(4a2 + b2)

2a+ b,

or equivalently,

3

16(ab+ bc+ ca)+ 4abc

(1

2b+ c+

1

2c+ a+

1

2a+ b

)≥ 3(ab+ bc+ ca)

Using again the Cauchy Schwartz inequality we see that

1

2b+ c+

1

2c+ a+

1

2a+ b≥ 3

a+ b+ c= 3,

and so, we are left to show that

1

16(ab+ bc+ ca)+ 4abc ≥ ab+ bc+ ca.

Setting q = ab + bc + ca, notice that 0 ≤ q ≤ 1

3. From the Schur’s

inequality, applied for the fourth degree, we have

a4 + b4 + c4 + abc(a+ b+ c) ≥ ab(a2 + b2) + bc(b2 + c2) + ca(c2 + a2),

and thus, in terms of q, we get

abc ≥ (4q − 1)(1− q)6

.

Therefore

1

16(ab+ bc+ ca)+ 4abc− ab− bc− ca =

1

16x− q + 4abc

≥ 1

16q− q +

2

3(4q − 1)(1− q)

=(3− 8q)(1− 4q)2

48q≥ 0.

82

Page 84: Олимпиадад бэлтгэгчдэд тусламж (Тэнцэтгэл биш 1)

23. Let n > 2 and x1, x2, . . . , xn > 0 such that(n∑k=1

xk

)(n∑k=1

1

xk

)= n2 + 1.

Prove that (n∑k=1

x2k

)(n∑k=1

1

x2k

)> n2 + 4 +

2

n(n− 1).

Solution: Put s =

n∑k=1

xk. Then by Cauchy-Schwartz we have that

[n∑k=1

(s− n

2xk

)2]( n∑k=1

1

x2k

)≥

(n∑k=1

s− n2xk

xk

)2

=

[(n∑k=1

xk

)(n∑k=1

1

xk

)− n2

2

]2

=

(n2 + 1− n2

2

)2

=(n2 + 2)2

4.

Now, notice that

n∑k=1

(s− n

2xk

)2= ns2 − ns

n∑k=1

xk +n2

4

n∑k=1

x2k =n2

4

n∑k=1

x2k.

Hence

n2

4

(n∑k=1

x2k

)(n∑k=1

1

x2k

)≥ (n2 + 2)2

4,

It follows that (n∑k=1

x2k

)(n∑k=1

1

x2k

)≥ (n2 + 2)2

n2.

It suffices to prove that

(n2 + 2)2

n2> n2 + 4 +

2

n(n− 1),

which is true because

(n2 + 2)2

n2− n2 − 4− 2

n(n− 1)=

2(n− 2)

n2(n− 1)> 0.

Therefore, our inequality is proved.

83

Page 85: Олимпиадад бэлтгэгчдэд тусламж (Тэнцэтгэл биш 1)

1.10 The principle of extremality and monotonicity

To prove an inequality of the form f(x1, x2, . . . , xn) ≥ (≤)0 it suffices to proveit for the multiset of x1, x2, . . . , xn which minimizes (maximizes) f on the givendomain.

This is actually a very powerful idea, because there are often methods todetermine when a set of variables minimize f . However, the bulk of suchexamples comes from analysis, which is too advanced for the purposes of thischapter (but will be dealt with later).

There are still simpler instances of this method, for which we can find thepoints of minimal/maximal values using elementary methods.

For example, to prove the inequality

a2 + b2 + c2 ≥ ab+ bc+ ca

we can take the function

f(a) = a2 + b2 + c2 − ab− ac− bc

as a quadratic function in a and we must prove f(a) ≥ 0. It suffices to prove

it for a being the vertex of the parabola a =b+ c

2which minimizes f . But

f

(b+ c

2

)=

3

4(b− c)2 ≥ 0.

If the function is monotone the extremal values are always attained on theends on an interval. Therefore if for each of xi the domain is an interval, itsuffices to check the inequality for xi being the extremities of the intervals.We can actually consider just one of the extremities: If f is increasing and theinequality is f ≥ 0 or f is decreasing and the inequality is f ≤ 0 we can takejust the smallest extremity, otherwise just the largest extremity.

For example, consider the inequality x2 − xy + y2 ≤ 4 for x, y ∈ [0, 2]. Thefunction

f(x, y) = x2 − xy + y2

is increasing in both x and y, because its a quadratic trinomial and x >y

2which is the vertex of g(x) = x2 − xy + y2, y >

x

2which is the vertex of

h(y) = y2 − yx+ x2.

Therefore the inequality must be proven only for the extremities of [1, 2]. How-ever, as the function is increasing, we can consider only the largest extremities,which are x = y = 2, for which we get the value 4.

84

Page 86: Олимпиадад бэлтгэгчдэд тусламж (Тэнцэтгэл биш 1)

Exercises

1. If a, b, c ∈ [0, 1] then 1 ≤ a+ b+ c+ 3(1− a)(1− b)(1− c) ≤ 3.

Solution: If we fix b, c then E = a + b + c + 3(1 − a)(1 − b)(1 − c) is alinear function in a so attains its extremal value at the extremes, so weneed to consider only the case when a ∈ {0, 1}. Analogously it sufficesto consider that b, c ∈ {0, 1}. This cases are easy to handle to obtain thedesired result.

2. If x1, x2, x3, . . . , xn ∈ [0, 1] then

x1 + x2 + x3 + . . .+ xn − x1x2 − x2x3 − . . .− xnx1 ≤⌊n

2

⌋.

Solution: Again this function is linear in all xi, so it suffices to look at thecases xi ∈ {0, 1}. Now assume that there are k ones among x1, x2, . . . , xnand n−k zeroes. Then x1+x2+. . .+xn = k and x1x2+x2x3+. . .+xnx1is the number of pairs of consecutive ones in the sequence. How smallcan the number of such pairs be? As there are n−k zeroes and each zeroenters in two pairs of consecutive numbers, there are at most 2(n − k)pairs of consecutive numbers at least one of which is zero, so there areat least n− 2(n− k) = 2k − n pairs of consecutive ones. So our sum isat most x1 + x2 + . . .+ xn = k and at most k − (2k − n) = n− k, so atmost max{k, n− k} ≤

⌊n2

⌋.

3. If a, b, c ∈ [0, 1]; p, q, r ∈[0, 12]; a+ b+ c = p+ q + r = 1 then

abc ≤ 1

8(pa+ qb+ rc).

Solution: we have r = 1− p− q so the expression

abc− 1

8(pa+ qb+ rc) = abc− 1

8[pa+ qb+ (1− p− q)c]

is linear in q for fixed p, and so takes its minimal value at the extremities.

The extremities are1

2− p (which yields r =

1

2) and

1

2. So it suffices to

look only at this case, which gives us that one of q, r is1

2.

Analogously we can suppose one of p, q is1

2and one of q, r is

1

2. It’s

easy to see that in this case two of p, q, r are1

2and one is 0. WLOG

assume p = q =1

2. Then we have to prove 16abc ≤ a + b. If we let

x = a+ b ∈ [0, 1] then 16abc ≤ 16

(a+ b

2

)2

c = 4x2(1− x). So we need

to show that 4x2(1− x) ≤ x or 4x(1− x) ≤ 1 or 0 ≤ (1− 2x)2, which istrue.

85

Page 87: Олимпиадад бэлтгэгчдэд тусламж (Тэнцэтгэл биш 1)

1.11 Breaking the inequality

Most complicated inequalities can be broken into simpler ones, by summingwhich we get the original inequality. We have already seen an example wehave broken the inequality

a2 + b2 + c2 ≥ ab+ bc+ ca

intoa2 + b2

2≥ ab, b

2 + c2

2≥ bc, and

c2 + a2

2≥ ca.

There are different tips for breaking inequalities.If we have a homogeneous inequality of degree one, like the one

a2

b+ c+

b2

a+ c+

c2

b+ a≥ a+ b+ c

2, (a, b, c > 0),

we can try to compare each term from the left sides with a part of the rightside:

a2

b+ c≥ αa+ βb+ γc.

We can see that α+β+γ = 12 as for a = b = c = 1 we get equality. Moreover,

sincea2

b+ cincreases with respect to a and decreases with respect to b, c we

must have α ≥ 0, β, γ ≤ 0. Moreover we must have β = γ because of thesymmetry of b, c. So, we must have an inequality of the form

a2

b+ c≥(

1

2+ 2x

)a− x(b+ c)

or

a2 ≥(

1

2+ 2x

)a(b+ c)− x(b+ c)2 or a2−

(1

2+ 2x

)a(b+ c) + x(b+ c)2 ≥ 0.

Since equality occurs when a = b = c, this quadratic is probably a perfectsquare that vanishes when 2a = b+ c, thus it must be (a− b+c

2 )2, and indeed

for x =1

4it is, thus we get the inequality

a2

b+ c≥ a− b+ c

4

with analogous ones which break up the original inequality. The most usedinequalities are xy ≥ 0 if x, y have the same sign, and especially the inequalityx2 ≥ 0. since every inequality can be transformed to E ≥ 0, a common way isbreaking E into sum of products of squares with positive numbers.

For example, the above inequality

a2 + b2 + c2 ≥ ab+ bc+ ca

is equivalent toa2 + b2 + c2 − ab− bc− ca ≥ 0

86

Page 88: Олимпиадад бэлтгэгчдэд тусламж (Тэнцэтгэл биш 1)

which breaks into

1

2(a− b)2 +

1

2(b− c)2 +

1

2(c− a)2 ≥ 0,

clearly true.Whereas most inequalities break by sum, some of them can be broken byproduct. Look at this problem from Saint-Petersburg 2005:(

x2 +3

4

)(y2 +

3

4

)(z2 +

3

4

)≥√

(x+ y)(y + z)(z + x).

Although the inequality is in three variables, it can be easily broken down intothree inequalities in just two variables:√(

x2 +3

4

)(y2 +

3

4

)≥√x+ y

and analogous.This inequalities are actually true: keeping in mind that x = y = z = 1

2 givethe equality, we can deduce(

x2 +1

4+

1

2

)(1

4+ y2 +

1

2

)≥(x+ y + 1

2

)2

but (x+ y + 1)2 ≥ 4(x+ y) by AM-GM, and from here the conclusion.

Some inequalities in many variables, like Cauchy–Schwartz, can be brokendown into simpler ones according to pairs of indices. For example, the in-equality

(a21 + . . .+ a2n)(b21 + . . .+ b2n)− (a1b1 + . . .+ anbn)2 ≥ 0,

if opened the brackets, can be grouped by pairs i, j of indices:∑1≤i<j≤n

(a2i b

2j + a2jb

2i − 2aibiajbj

)=

∑1≤i<j≤n

(aibj − ajbi)2 ≥ 0.

That’s how we established Lagrange’s Identity:

(a21 + . . .+ a2n)(b21 + . . .+ b2n)− (a1b1 + . . .+ anbn)2 =∑

1≤i<j≤n(aibj − ajbi)2.

Analogously one can prove the following inequality:If x1, x2, . . . , xn > 0 then

(x1 + x2 + . . .+ xn)

(1

x1+ . . .+

1

xn

)≥ n2.

Indeed, opening the brackets the inequality is equivalent to∑1≤i<j≤n

(xixj

+xjxi− 2

)≥ 0

which follows from AM-GM for two variables.This method of extracting squares leads to another way of proving inequali-ties, conventionally called the SOS (Sum of Squares) method, which shall bepresented in a section to come.

87

Page 89: Олимпиадад бэлтгэгчдэд тусламж (Тэнцэтгэл биш 1)

Exercises

1. If a, b, c > 0 then (a+ b)(b+ c)(c+ a) ≥ 8abc

Solution: The inequality breaks by product into

a+ b ≥ 2√ab, b+ c ≥ 2

√bc, c+ a ≥ 2

√ca.

2. (a+ b− c)2 + (a+ c− b)2 + (b+ c− a)2 +3

4≥ a+ b+ c.

Solution: we break it by sum into

(a+b−c)2+1

4≥ a+b−c, (a+c−b)2+1

4≥ a+c−b, (b+c−a)2+

1

4≥ b+c−a.

3. If a, b, c, d > 0 then a4b+ b4c+ c4d+ d4a ≥ abcd(a+ b+ c+ d)

Solution: We wish to break the inequality into

ma4b+ nb4c+ pc4d+ qd4a ≥ a2bcd,

where m+n+ p+ q = 1. Combined with the analogous cyclic relations,this would yield the conclusion.

Now according to weighted AM-GM we have

ma4b+ nb4c+ pc4d+ qd4a ≥ a4m+qb4n+mc4p+nd4q+p.

Thus we need 4m+ q = 2, 4n+m = 1, 4p+n = 1, 4q+p = 1. We thendeduce q = 2− 4m, p = 1− 4q = 1− 4(2− 4m) = 16m− 7, n = 1− 4p =1 − 4(16m − 7) = 29 − 64m. Then 4n + m = 1 or 116 − 255m = 1 som = 115

255 = 2351 . So we compute then n = 7

51 , p = 1151 , q = 10

51 and we’vegot the breaking.

4. If a, b, c > 0 then

(a+b+c)

(a

(2a+ b+ c)(b+ c)+

b

(2b+ c+ a)(c+ a)+

c

(2c+ b+ a)(b+ a)

)≥ 9

8.

Solution: We may notice that

a

(2a+ b+ c)(b+ c)=

1

2(b+ c)− 1

2(a+ b) + 2(a+ c)

≥ 1

2(b+ c)− 1

8(a+ b)− 1

8(a+ c).

By summing with the analogous terms we have

a

(2a+ b+ c)(b+ c)+

b

(2b+ c+ a)(c+ a)+

c

(2c+ b+ a)(b+ a)≥

≥ 1

4

(1

a+ b+

1

b+ c+

1

c+ a

)≥ 1

4

9

2(a+ b+ c)=

9

8(a+ b+ c).

88

Page 90: Олимпиадад бэлтгэгчдэд тусламж (Тэнцэтгэл биш 1)

5. Let a, b, c ≥ 0 with a2+b2+c2 = 1. Show thata

1− a2+

b

1− b2+

c

1− c2≥

3√

3

2.

Solution: we notice the inequalityx

1− x2≥ 3√

3

2x2, which is equivalent

to 3√

3x(1− x2) ≤ 4 or 27x2 · 1−x22 · 1−x22 ≤ 1, which follows by AM-GM

for x2, 1−x2

2 , 1−x2

2 . This inequality solves the problem.

6. If a, b, c ≥ 0 and a2 + b2 + c2 = 1 then a1−a4 + b

1−b4 + c1−c4 ≥

5 4√54 .

Solution: Again we break the inequality according to the relation

x

1− x4≥ 5 4√

5

4x2.

This relation is equivalent to5 4√

5

4x(1− x4) ≤ 1, or raised to the fourth

power to x4 · 1−x44 · 1−x44 · 1−x44 · 1−x44 ≤ 55, which follows by AM-GM.

7. Let x, y, z > 0. Then

(x2+y2−z2)(y2+z2−x2)(x2+z2−y2) ≤ (x+y−z)2(y+z−x)2(x+z−y)2.

Solution: It suffices to consider the case when all x2+y2−z2, x2+z2−y2,y2 + z2 − x2 are positive, because at most one of them can be negative(the pairwise sums are positive, so we can’t have two negative numbersamong them), and when one of them is negative, LHS is negative andRHS is positive.

Now we shall break the inequality into

(x2 + y2 − z2)(x2 + z2 − y2) ≤ (x+ y − z)2(x+ z − y)2

and the analogous ones.

This is equivalent to x4 − (y2 − z2)2 ≤[x2 − (y − z)2

]2or

x4 − (y + z)2(y − z)2 ≤ x4 − 2(y − z)2x2 + (y − z)4,

or that (y − z)4 + (y + z)2(y − z)2 − 2(y − z)2x2 ≥ 0 or

(y − z)2[(y + z)2 + (y − z)2 − 2x2

]≥ 0

or 2(y − z)2(y2 + z2 − x2) ≥ 0, which is true.

8. For a, b, c > 0 we havea2

b2+b2

c2+c2

a2≥ a

b+b

c+c

a.

Solution: As in example 3, we have a breaking that can be computed by

constructing a system of equations:2

3

a2

b2+

1

6

b2

c2+

1

6

c2

a2≥ a

b.

89

Page 91: Олимпиадад бэлтгэгчдэд тусламж (Тэнцэтгэл биш 1)

9. For x, y, z > 0,

1

3x+ y+

1

3y + z+

1

3z + x≥ 1

2x+ y + z+

1

2y + z + x+

1

2z + x+ y.

Solution: We seek an inequality of form

m

3x+ y+

n

3y + z+

p

3z + x≥ 1

2x+ y + z

where m+ n+ p = 1.

We have

m

3x+ y+

n

3y + z+

p

3z + x=

m2

3mx+my+

n2

3ny + nz+

p2

3pz + xp

≥ 1

(3m+ p)x+ (3n+m)y + (3p+m)z.

By solving the system we get m = 47 , n = 1

7 , p = 27 .

10. Let 0 ≤ x ≤ y, a1, a2, . . . , an, b1, b2, . . . , bn > 0, n ≥ 2.

Then(n∑i=1

a1+xi b1−xi

)(n∑i=1

a1−xi b1+xi

)≤

(n∑i=1

a1+yi b1−yi

)(n∑i=1

a1−yi b1+yi

).

Solution: Like CBS, we break the inequality into pairs of indices. Indeedthe LHS is

n∑i=1

a2i b2i +

∑1≤i<j≤n

(a1+xi b1−xi a1−xj b1+xj + a1−xi b1+xi a1+xj b1−xj )

and the RHS is

n∑i=1

a2i b2i +

∑1≤i<j≤n

(a1+yi b1−yi a1−yj b1+yj + a1−yi b1+yi a1+yj b1−yj ).

So we have to show that

a1+xi b1−xi a1−xj b1+xj +a1−xi b1+xi a1+xj b ≤ a1+yi b1−yi a1−yj b1+yj +a1−yi b1+yi a1+yj b1−yj .

By dividing by aibiajbj we transform this to a nicer form(aibjbiaj

)x+

(biajaibj

)x≤(aibjbiaj

)y+

(biajaibj

)y,

of ux+1

ux≤ uy +

1

uy, or

u2x + 1

ux≤ u2y + 1

uyor (ux+y−1)(xy−x−1) ≥ 0,

which is true.

90

Page 92: Олимпиадад бэлтгэгчдэд тусламж (Тэнцэтгэл биш 1)

11. If x1, . . . , xn > 0 such that x1x2 . . . xn = 1 and p, q > 0 such thatq

p≥ n(n− 2), then

√px21 + q +

√px22 + q + . . .+

√px2n + q ≤

√p+ q(x1 + x2 + . . .+ xn).

Solution: We try to break the inequality into√mx21 + n ≤ ax1 +

√m+ n− an− 1

(x2 + x3 + . . .+ xn).

As the LHS depends only on x1, we apply AM-GM to the RHS to get

the inequality ax1 +

√p+ q − an− 1

(x2 + x3 + . . .+ xn) ≥ ax+

√p+ q − a

x1

n−1(we let x = x1 for simplicity).

Now square to get

a2x2+2a(√p+ q − a

)x1− 1

n−1+(p+ q − 2

√p+ qa+ a2

)x− 2n−1 ≥ px2+q,

or (a2−p)x2+2a (√p+ q − a)x

1− 1n−1+

(p+ q − 2

√p+ qa+ a2

)x− 2n−1 ≥

q. We apply now weighted AM-GM to get

(a2 − p)x2 + 2a(√p+ q − a

)x1− 1

n−1 +(p+ q − 2

√p+ qa+ a2

)x− 2n−1 ≥

≥[(a2 − p) + 2a

(√p+ q − a

)+(p+ q − 2

√p+ qa+ a2

)]·

·x2(a2−p)+

(1− 1

n−1)2a(√p+q−a)+

(− 2n−1

)(p+q−2

√p+qa+a2)

(a2−p)+2a(√p+q−a)+(p+q−2

√p+qa+a2)

= qx

2nn−1

√p+qa−2

(pnn−1 + q

n−1)

q .

So now for the desired inequality to take place, we must take

a =

n

n− 1p+

1

n− 1q

n√p+ q

=√p+ q

np+ q

n(p+ q).

We can see that√p < a <

√p+ q so for this value the desired breaking

holds and the conclusion follows.

12. If a, b, c > 0 and abc = 1 then1

1 + a+ b+

1

1 + b+ c+

1

1 + c+ a≤ 1.

Solution: Let’s try to find a breaking of the form1

1 + a+ b≤ ct

at + bt + ct,

or at+bt+ct ≤ ct+ct(a+b), so at+bt ≤ ct(a+b) so a2tbt+b2tat ≤ a+b.

This is achieved for t =1

3.

13. If a, b, c > 0 thena√

a2 + 8bc+

b√b2 + 8ac

+c√

c2 + 8ab≥ 1

91

Page 93: Олимпиадад бэлтгэгчдэд тусламж (Тэнцэтгэл биш 1)

Solution: Again let’s break the inequality intoa√

a2 + 8bc≥ as

as + bs + cs

or a2s(a2 + 8bc) ≤ a2(as + bs + cs)2. We must have

a2s + 8a2s−2bc ≤ a2s + b2s + c2s + 2bscs + 2as(bs + cs)

so 8a2s−2bc ≤ b2s + c2s + 2bscs + 2as(bs + cs).

As b2s + c2s ≥ 2bscs and bs + cs ≥ 2bs2 c

s2 , it suffices to have

8a2s−2bc ≤ 4bscs + 4asbs2 c

s2 .

If we let d =√bc this amount to 2a2s−2d2 ≤ d2s + asds. However by

AM-GM d2s+asds ≥ 2as2d

3s2 , and for s =

4

3we get the desired relation.

1.12 Separating the squares

Sometimes it’s hard to break a inequality into sum of squares, and we must use

some special tricks. One very good trick is the easy principle a− b =a2 − b2

a+ b,

which helps us deal with inequalities involving radicals. Look at√a2 + 2b2 +

√b2 + 2a2 ≥ 2

√a2 + ab+ b2.

The first idea is squaring, selecting the radical, and squaring again, whichproduces a terrible mess of computations.

For another way, we first observe that if

a = b,√a2 + 2b2 =

√b2 + 2a2 =

√a2 + ab+ b2,

so we want to extract a− b as a factor. To do this, use our trick:√a2 + 2b2 −

√a2 + b2 + ab =

a2 + 2b2 − a2 − b2 − ab√a2 + 2b2 +

√a2 + b2 + ab

=b(b− a)√

a2 + 2b2 +√a2 + b2 + ab

.

We can factor√2a2 + b2 −

√a2 + b2 + ab =

a(a− b)√2a2 + b2 +

√a2 + b2 + ab

,

and so the inequality is equivalent to

(b− a)

(b√

a2 + 2b2 +√a2 + b2 + ab

− a√2a2 + b2 +

√a2 + b2 + ab

)≥ 0.

We have selected just b − a, which is not a square, and so cannot help us:in fact it can take both positive and negative values. But, if we think thatthe original expression is symmetric in a, b, we can expect to factor it alsosymmetrically in a, b. As b − a is not symmetric but (b − a)2 is, we can be

92

Page 94: Олимпиадад бэлтгэгчдэд тусламж (Тэнцэтгэл биш 1)

pretty sure that we can extract b−a once again (this is just intuitive reasoning,but it leads us to the correct result). First, let’s clear the denominators:

(b− a)b(√

a2 + 2b2 +√a2 + b2 + ab

)− a

(√2a2 + b2 +

√a2 + b2 + ab

)(√

2a2 + b2 +√a2 + b2 + ab

)(√a2 + 2b2 +

√a2 + b2 + ab

) ≥ 0.

The denominator is clearly positive, so let’s look at the numerator which canbe written as

(b− a)√a2 + b2 + ab+

√2a2b2 + b4 −

√2a2b2 + a4.

We have no problem in extracting b− a from√2a2b2 + b4 −

√2a2b2 + a4

using our trick:√2a2b2 + b4 −

√2a2b2 + a4 =

(b− a)(b4 + b3a+ b2a2 + ba3 + a4)√2a2b2 + b4 +

√2a2b2 + a4

.

That’s why the numerator is

(b− a)2(√

a2 + b2 + ab+b4 + b3a+ b2a2 + ba3 + a4√

2a2b2 + b4 +√

2a2b2 + a4

),

thus positive, which finishes the proof. [Note: actually this inequality can be

easily handled by Minkowski’s Inequality for p =1

2].

When we have such inequalities, symmetric in a few variables, and that becomeequalities when all the variables are equal, its useful to select out the squaresof the differences ((a− b)2, (b− c)2), perhaps using the trick above. But thesedifferences help us even sometimes when the inequalities do not transform intoequalities when all the variables are equal:Consider the inequalityIf a, b, c are positive numbers which form a triangle (that if a < b+c, b < c+a,c < a+ b), then

2ab+ 2bc+ 2ca > a2 + b2 + c2.

We don’t see an obvious method of writing it as a sum of squares, but sincewe see a2, b2, c2, 2ab, 2bc and 2ca We try to get the squares

(a+ b)2, (b+ c)2, (c+ a)2 or (a− b)2, (b− c)2, (c− a)2.

We transform the inequality into

0 > a2 + b2 + c2 − 2ab− 2bc− 2ca

or intoa2 + b2 + c2 ≥ (a− b)2 + (b− c)2 + (c− a)2

which breaks into the inequalities

a2 ≥ (b− c)2, b2 ≥ (c− a)2, c2 ≥ (a− b)2.

93

Page 95: Олимпиадад бэлтгэгчдэд тусламж (Тэнцэтгэл биш 1)

They are true: since b < a+c we get a > b−c; since c < a+b we get a > c−b,hence |a| ≥ |b− c| and a2 ≥ (b− c)2, the other inequalities being analogous.When trying to extract the squares we must reason logically and usually thisleads to a solution. For example, let’s try to break the inequality

a3 + b3 + c3 + 3abc− a2b− b2a− b2c− c2b− a2c− c2a ≥ 0.

If we write it in the form

P1(a, b, c)(a− b)2 + P2(a, b, c)(b− c)2 + P3(a, b, c)(c− a)2

then P1, P2, P3 must be linear, thus we must have P1(a, b, c) = ax + by + cz,and from the symmetry in a, b ce suppose x = y. Further, from the totalsymmetry we may assume

P2(a, b, c) = x(b+ c) + za, P3(a, b, c) = x(a+ c) + zb.

To compute, x, z, set a = b. Then the relation reduces to

c3−2c2a+ca2 = c(a−c)2 = (P2(a, a, c)+P3(a, a, c))(a−c)2 = (2(x+z)a+2xc)(c−a)2.

This yields us x =1

2, z = −1

2and so the inequality is equivalent to

(b+ c− a)(b− c)2 + (a+ c− b)(a− c)2 + (a+ b− c)(a− b)2.

Actually one of these terms may be negative, but the problem is simpler tosolve in this form.Indeed, as it’s symmetric, we may assume that a ≥ b ≥ c. If b+ c ≥ a, we aredone. Otherwise, we must prove that

(a− b− c)(b− c)2 ≤ (a+ c− b)(a− c)2 + (a+ b− c)(a− b)2.

However a−b−c < a+c−b and b−c ≤ a−c and from here the result follows.

The method of extracting squares of differences is conventionally called SOS(Sum of Squares) method, and is usually used to solve hard inequalities.

Sometimes it’s hard to see the square. In this case it’s good to find an in-

termediary expression. For example, if we want to express√ab − 2ab

a+ bwith

respect to (a− b)2, we might remember that

a+ b

2−√ab =

(√a−√b)2

2=

(a− b)2

2(√

a+√b)2

anda+ b

2− 2ab

a+ b=

(a− b)2

a+ b

hence√ab− 2ab

a+ b=

(a− b)2

2

(1

a+ b− 1

a+ b+ 2√ab

)≥ 0.

94

Page 96: Олимпиадад бэлтгэгчдэд тусламж (Тэнцэтгэл биш 1)

Exercises

1. If a2 + b2 = 1 and a, b > 0 then a+ b+1

ab≥ 2 +

√2

Solution: The inequality can be rewiteen as√

2− a− b ≤ a2 + b2

ab− 2 or

as√

2(a2 + b2)−a−b ≤ a

b+b

a−2 or as

(a− b)2

a+ b+√

2≤

(√a

b−√b

a

)2

=

(a− b)2

abwhich transforms to ab ≤ a+ b+

√2 which is obviously true.

2. If a, b, c > 0 and a2 + b2 + c2 = 1 then a+ b+ c+1

abc≥ 4√

3

Solution: We write the conclusion as√

3− (a+ b+ c) ≤ 1

abc− 3√

3. It’s

clear that both sides are positive (from AM-GM). This is equivalent to

3− (a+ b+ c)2√3 + a+ b+ c

≤ 1− 27a2b2c2

abc(1 + 3

√3abc

) .Now we remember that a2 + b2 + c2 = 1 and we use the identities

3(a2 + b2 + c2)− (a+ b+ c)2 = (a− b)2 + (b− c)2 + (c− a)2

and (x+y+z)3−27xyz =∑cyc

(x−y)2x+ y + 7z

2(for x = a2, y = b2, z =

c2) to transform our inequality into∑cyc

(a− b)2(a+ b)2 a2+b2+7c2

2

abc(1 + 3

√3abc

) ≥

∑cyc

(a− b)2

√3 + a+ b+ c

.

However (a + b)2 ≥ 4ab,a2 + b2 + 7c2

2=

1 + 6c2

2≥√

6c from AM-

GM, and abc ≤ 13√3, implying that LHS is at least

∑cyc

2√

6(a − b)2 ≥

∑cyc

(a− b)2√3

, which is, in turn, greater than RHS.

3. Let a, b, c be nonnegative numbers. Show that

a+ b+ c

3− 3√abc ≤ max

{(√a−√b)2, (

√a−√c)2, (

√b−√c)2}.

Solution: Set a = x6, b = y6, c = z6. WLOG suppose x ≥ y ≥ z. Wemake use of the identity

u3 + v3 + w3 − 3uvw =1

2(u+ v + w)

[(u− v)2 + (v − w)2 + (w − u)2

],

rewriting the conclusion as

1

6(x2 + y2 + z2)

[(x2 − y2)2 + (y2 − z2)2 + (x2 − z2)2

]≤ (x3 − z3)2,

95

Page 97: Олимпиадад бэлтгэгчдэд тусламж (Тэнцэтгэл биш 1)

or

(x2 + y2 + z2)(x+ y)2(x− y)2 + (x2 + y2 + z2)(y + z)2(y − z)2 +

+(x2 + y2 + z2)(x+ z)2(x− z)2 ≤ 6(x2 + z2 + zx)(z − x)2.

However we have

x2 + y2 + z2 ≤ 2(x2 + z2 + xz),

(x− y)2 + (y − z)2 ≤ (x− z)2,(y − z)2 ≤ (x− z)2,

Therefore we have

6(x2 + xz + z2)2(x− z)2 − [(x2 + y2 + z2)(x+ y)2(x− y)2 +

+(x2 + y2 + z2)(y + z)2(y − z)2 + (x2 + y2 + z2)(x+ z)2(x− z)2] ≥≥ 2(x2 + zx+ x2)

[(2x2 + xz + 2z2)(x− z)2 − (y + z)2(y − z)2 − (x+ y)2(x− y)2

].

Therefore it suffices to show that

(2x2 + 2z2 + xz)(x− z)2 ≥ (x+ y)2(x− y)2 + (y + z)2(y − z)2.

Now set u = x − y, v = y − z, u + v = x − z. The inequality to showtransforms then into[2(z + u+ v)2 + 2z2 + z(z + u+ v)

](u+v)2 ≥ (2z+2u+v)2u2+(2z+v)2v2,

or

z2[5(u+ v)2 − 4u2 − 4v2

]+[5(u+ v)3 − 4(2u+ v)u2 − 4v3

]z +

+2(u+ v)4 − (2u+ v)2v2 − v4 ≥ 0.

It is now pretty clear that 5(u+v)2−4u2−4v2, 5(u+v)3−4(2u+v)u2−4v3,2(u+ v)4 − (2u+ v)2v2 − v4 are positive, so the proof is finished.

4. Let a, b, c > 0. Show that

4(ab+ bc+ ca)

(1

(a+ b)2+

1

(b+ c)2+

1

(c+ a)2

)≥ 9.

Solution: We have

4(ab+bc+ca) = (a+b)2+(b+c)2+(c+a)2−(a−b)2−(b−c)2−(c−a)2

so

4(ab+ bc+ ca)

[1

(a+ b)2+

1

(b+ c)2+

1

(c+ a)2

]− 9 =

= −[(a− b)2 + (b− c)2 + (c− a)2

] [ 1

(a+ b)2+

1

(b+ c)2+

1

(c+ a)2

]+

+[(a+ b)2 + (b+ c)2 + (c+ a)2

] [ 1

(a+ b)2+

1

(b+ c)2+

1

(c+ a)2

]− 9

= −[(a− b)2 + (b− c)2 + (c− a)2

] [ 1

(a+ b)2+

1

(b+ c)2+

1

(c+ a)2

]+

+(a− b)2(a+ b+ 2c)2

(a+ c)2(b+ c)2+

(b− c)2(b+ c+ 2a)2

(a+ b)2(a+ c)2+

(a− c)2(a+ 2b+ c)2

(a+ b)2(b+ c)2

96

Page 98: Олимпиадад бэлтгэгчдэд тусламж (Тэнцэтгэл биш 1)

(we used here the identity (x + y + z)

(1

x+

1

y+

1

z

)=

(x− y)2

xy+

(x− z)2

xz+

(y − z)2

yz.)

As(a+ b+ 2c)2

(a+ c)2(b+ c)2=

1

(a+ c)2+

1

(b+ c)2+

2

(a+ c)(b+ c), we finally

write

4(ab+ bc+ ca)

[1

(a+ b)2+

1

(b+ c)2+

1

(c+ a)2

]− 9 =

= (a− b)2[

2

(a+ c)(b+ c)− 1

(a+ b)2

]+ (a− c)2

[2

(a+ b)(b+ c)− 1

(a+ c)2

]+

+(b− c)2[

2

(a+ b)(a+ c)− 1

(b+ c)2

].

Assume that a ≥ b ≥ c. Then we have2

(a+ b)(b+ c)− 1

(a+ c)2≥ 0 and

2

(a+ c)(b+ c)− 1

(a+ b)2≥ 0. If

2

(a+ b)(a+ c)− 1

(b+ c)2≥ 0 then we

are done. So assume that2

(a+ b)(a+ c)− 1

(b+ c)2< 0. Then we rewrite

the inequality as

(a− b)2[

2

(a+ c)(b+ c)− 1

(a+ b)2

]+ (a− c)2

[2

(a+ b)(b+ c)− 1

(a+ c)2

]≥

≥ (b− c)2[

1

(b+ c)2− 2

(a+ b)(a+ c)

].

We shall prove that

(a−c)2[

2

(a+ b)(b+ c)− 1

(a+ c)2

]≥ (b−c)2

[1

(b+ c)2− 2

(a+ b)(a+ c)

].

After clearing denominators and cancelling common terms we reducethis to

(a− c)2

a+ c(2a2+3ac+2c2−b2−ab−bc) ≥ (b− c)2

b+ c(a2+ab+ac−2b2−2c2−3bc).

However (a−c)2a+c ≥

(b−c)2b+c as this is equivalent to

(a− b)[(a+ b)(b+ c)− 4c2

]≥ 0

and 2a2 + 3ac + 2c2 − b2 − ab − bc ≥ a2 + ab + ac − 2b2 − 2c2 − 3bc asthis is equivalent to a2 + b2 + 2c2 − 2ab+ 2ac+ 2bc ≥ 0, clearly true.

5. If a, b, c > 0 then show that (a3 + b3 + c3)2 ≥ (a4 + b4 + c4)(ab+ bc+ ca).

Solution: We need to write the difference

(a3 + b3 + c3)2 − (a4 + b4 + c4)(ab+ bc+ ca)

in SOS style?.

97

Page 99: Олимпиадад бэлтгэгчдэд тусламж (Тэнцэтгэл биш 1)

We might remember that (a3 + b3 + c3)2 − (a4 + b4 + c4)(a2 + b2 + c2)can be written in this form according to Lagrange Identity, and clearly

(a4 + b4 + c4)(a2 + b2 + c2 − ab− bc− ca),

can be written in that form, too.

So

(a3 + b3 + c3)− (a4 + b4 + c4)(ab+ bc+ ca) =

= (a3 + b3 + c3)2 − (a4 + b4 + c4)(a2 + b2 + c2) +

+(a4 + b4 + c4)(a2 + b2 + c2 − ab− bc− ca)

= −(a− b)2a2b2 − (b− c)2b2c2 − (c− a)2c2a2 +

+(a4 + b4 + c4)

[(a− b)2 + (b− c)2 + (c− a)2

2

]=

1

2[(a− b)2(a4 + b4 − 2a2b2 + c4) + (b− c)2(b4 + c4 − 2b2c2 + a4) +

+(c− a)2(c4 + a4 − 2a2c2 + b4)] ≥ 0.

6. If a, b, c > 0 then 9(a4 + b4 + c4)2 ≥ (a5 + b5 + c5)(a+ b+ c)3.

Solution: We use the same method as in the previous problem. Keepingin mind that

(a4+b4+c4)2−(a5+b5+c5)(a3+b3+c3) = −a3b3(a−b)2−b3c3(b−c)2−c3a3(a−c)2

and

9(a3 + b3 + c3)− (a+ b+ c)3 =

= 2(a3 + b3 + c3 − 3abc) + 3(a3 + b3 − a2b− ab2) + 3(b3 + c3 − b2c− bc2) +

+3(a3 + c3 − a2c− ac2)= (a− b)2(a+ b+ c+ 3(a+ b)) + (b− c)2(a+ b+ c+ 3(b+ c)) +

+(c− a)2(a+ b+ c+ 3(a+ c)),

we write 9(a4 + b4 + c4)2 − (a5 + b5 + c5)(a+ b+ c)3 as

(a− b)2[(a5 + b5 + c5)(4a+ 4b+ c)− 9a3b3] +

+(b− c)2[(a5 + b5 + c5)(4b+ 4c+ a)− 9b3c3] +

+(c− a)2[(a5 + b5 + c5)(4a+ 4c+ b)− 9a3c3].

As (a5+b5+c5)(4a+4b+c) ≥ 4(a5+b5)(a+b) = 4(a6+b6+a5b+ab5) ≥16a3b3, we conclude that all terms accompanying the squares are non-negative so we are done.

7. If a, b, c > 0 then1

a+

1

b+

1

c+

9

a+ b+ c≥ 4

(1

a+ b+

1

b+ c+

1

c+ a

).

Solution: We have

1

a+

1

b+

1

c+

9

a+ b+ c− 4

(1

a+ b+

1

b+ c+

1

c+ a

)=

=1

2

[(a− b)2

ab(a+ b)+

(a− c)2

ac(a+ c)+

(b− c)2

bc(b+ c)

]+

9

a+ b+ c− 2

a+ b− 2

b+ c− 2

a+ c.

98

Page 100: Олимпиадад бэлтгэгчдэд тусламж (Тэнцэтгэл биш 1)

Now knowing the identity

(x+ y + z)

(1

x

1

y+

1

z

)− 9 =

(x− y)2

xy+

(x− z)2

xz+

(y − z)2

yz,

we obtain

9

a+ b+ c− 2

a+ b− 2

b+ c− 2

a+ c=

=1

a+ b+ c

[(b− c)2

(a+ b)(a+ c)+

(a− b)2

(c+ a)(c+ b)+

(a− c)2

(b+ a)(b+ c)

],

so finally

1

a+

1

b+

1

c+

9

a+ b+ c− 4

(1

a+ b+

1

b+ c+

1

c+ a

)=

=1

2

[(a− b)2

[1

ab(a+ b)− 4

(a+ c)(b+ c)(a+ b+ c)

]+

+(a− c)2[

1

ac(a+ c)− 2

(a+ b)(b+ c)(a+ b+ c)

]+

+(c− b)2[

1

cb(c+ b)− 2

(a+ c)(b+ a)(a+ b+ c)

] ].

In what cases do we have the inequality1

ab(a+ b)≤ 2

(a+ c)(b+ c)(a+ b+ c)?

This is equivalent to ab(a + b) ≥ c(a2 + b2 + 3ab) + 2c2(a + b) + c3 soc < ab

a+b and c is the smallest of a, b, c.

Now assume that a ≥ b ≥ c and write the inequality as

(a− b)2[

2

(a+ c)(b+ c)(a+ b+ c)− 1

ab(a+ b)

]≤

≤ (a− c)2[

1

ac(a+ c)− 2

(a+ b)(b+ c)(a+ b+ c)

]+

+(c− b)2[

1

cb(c+ b)− 2

(a+ c)(b+ a)(a+ b+ c)

].

However we may note that (a− c) ≥ (a− b) and

1

ac(a+ c)− 2

(a+ b)(b+ c)(a+ b+ c)≥ 2

(a+ c)(b+ c)(a+ b+ c)− 1

ab(a+ b).

This is equivalent to

1

ab(a+ b)+

1

ac(a+ c)≥ 2

[1

(a+ c)(b+ c)(a+ b+ c)+

1

(a+ b)(b+ c)(a+ b+ c)

].

However we may prove that

1

ab(a+ b)+

1

ac(a+ c)≥ 2

a√bc(a+ b)(a+ c)

≥ 8

a(b+ c)(2a+ b+ c)

99

Page 101: Олимпиадад бэлтгэгчдэд тусламж (Тэнцэтгэл биш 1)

and1

(a+ c)(b+ c)(a+ b+ c)+

1

(a+ b)(b+ c)(a+ b+ c)<

<1

a(b+ c)(a+ b+ c)+

1

a(b+ c)(a+ b+ c)=

2

a(b+ c)(a+ b+ c).

So it suffices to prove that8

a(b+ c)(2a+ b+ c)≥ 4

a(b+ c)(a+ b+ c)which is equivalent to 2(a+ b+ c) ≥ 2a+ b+ c, true!

8. Let a, b, c > 0 and a+ b+ c = 1. Prove that√a+ abc+

√b+ abc+

√c+ abc ≥ 2

3

(√a+√b+√c).

Solution: Let’s substitute

a =x2

x2 + y2 + z2, b =

y2

x2 + y2 + z2, c =

z2

x2 + y2 + z2.

We have reduced the inequality to proving

3∑cyc

x√

(x2 + y2)(x2 + z2) ≥ 2(x+ y + z)(x2 + y2 + z2).

Lemma: 2(x3 + y3 + z3)−∑sym

x2y =∑sym

(x+ y)(x− y)2.

Proof : Immediately from x3 + y3 − xy(x+ y) = (x+ y)(x− y)2.

Now we see that

3∑cyc

x(x2 + y2) + (x2 + z2)

2−RHS =

1

2

∑cyc

(x+ y)(x− y)2

using Lemma. However

3∑cyc

x(x2 + y2) + (x2 + z2)

2− LHS =

3

2

∑cyc

(√x2 + y2 −

√x2 + z2

)2.

So we are left to prove that∑cyc

(x+ y)(x− y)2 ≥ 3∑cyc

x(√

x2 + y2 −√x2 + z2

)2.

We prove that

(x+y)(x−y)2 ≥ 3z(√

z2 + y2 −√z2 + x2

)2=

3z(x− y)2(x+ y)2(√z2 + y2 +

√z2 + x2

)2 .Thus we have to prove

(√z2 + y2 +

√z2 + x2

)2≥ 3z(x+ y) or

2z2 + x2 + y2 + 2√z2 + x2

√z2 + y2 ≥ 3z(x+ y).

However√z2 + x2

√z2 + y2 ≥ z2 + xy by CBS. Thus we reduce it to

4z2 + x2 + y2 + 2xy ≥ 3z(x+ y)

or 4z2 + (x + y)2 ≥ 3z(x + y). However 4z2 + (x + y)2 ≥ 4z(x + y) byAM-GM and so we are done.

100

Page 102: Олимпиадад бэлтгэгчдэд тусламж (Тэнцэтгэл биш 1)

1.13 The Dual Principle

This is very simple idea, and yet it prove to have quite a few applications - andnot only in inequalities, but also in computational geometry. In particular, itis useful in solving algebraic inequalities, as we will see in the examples.

The positive reals a, b, c are sides of a triangle if and only if there are positivereals x, y, z with a = y + z, b = x+ z, c = x+ y.

Indeed, if a = y+ z, b = x+ z, c = x+ y, then a = y+ z < 2x+ y+ z = b+ c

and analogously b < c + a, c < a + b. Conversely, setting x =b+ c− a

2,

y =a+ c− b

2, z =

a+ b− c2

we satisfy the requirements.

Let’s look at one already met inequality through the prism of dual principle:

2(ab+ bc+ ca) ≥ a2 + b2 + c2

is equivalent, by dual principle, to

2 [(x+ y)(y + z) + (x+ y)(x+ z) + (y + z)(x+ z)] ≥ (x+y)2+(z+x)2+(y+z)2 ≥ 0

or to

2(x2 + y2 + z2) + 3(xy + yz + zx) ≥ 2(x2 + y2 + z2) + 2(xy + yz + zx)

which is obvious.

101

Page 103: Олимпиадад бэлтгэгчдэд тусламж (Тэнцэтгэл биш 1)

Exercises

1. If a, b, c are sides of a triangle then

a(b− c)2 + b(c− a)2 + c(a− b)2 + 4abc > a3 + b3 + c3.

Solution: After substituting a = y+z, b = x+z, c = x+y by cancellingcommon terms the inequality becomes xyz > 0.

2. If a, b, c form a triangle thena

2a+ b+ c+

b

2b+ c+ a+

c

2c+ a+ b>

2

3.

Solution: After applying the CBS Lemma we get

a

2a+ b+ c+

b

2b+ c+ a+

c

2c+ a+ b≥ (a+ b+ c)2

2(a2 + b2 + c2) + 2(ab+ bc+ ca)

so we left to prove that 3(a+b+c)2 > 2[2(a2 + b2 + c2) + 2(ab+ bc+ ca)

]or 2(ab+ bc+ ca) > a2 + b2 + c2 which we have already proven.

3. If a, b, c are the sides of a triangle and ax+by+cz = 0 then xy+yz+zx ≤0.

Solution: We can suppose that x, y > 0, because the inequality is sym-metric and doesn’t change if we replace x, y, z by −x, −y, −z. Wehave

xy + yz + zx = xy + z(x+ y) = xy − (ax+ by)(x+ y)

c.

So we have to prove that cxy ≤ (ax+ by)(x+ y). But

cxy ≤ (a+ b)xy ≤ (ax+ by)(x+ y).

We see that in this case we managed better without using the dualprinciple.

4. Show that in a triangle, the length of a median from a vertex is not lessthan the length of the bisector from the same vertex.

Solution: We use the formula ma =

√2b2 + 2c2 − a2

4and

la =

√bc(b+ c)(b+ c− a)(b+ c+ a)

(b+ c)2. So we have to prove

2b2 + 2c2 − a2

4≤ bc(b+ c− a)(b+ c+ a)

(b+ c)2.

Now by using the dual principle we have

2b2 + 2c2 − a2

4=

2(x+ z)2 + 2(y + z)2 − (x+ y)2

4

=x2 + y2 − 2xy + 4z2 + 4zy + 4zx

4≥ z2 + zy + zx = z(x+ y + z).

102

Page 104: Олимпиадад бэлтгэгчдэд тусламж (Тэнцэтгэл биш 1)

However

l2a =bc(b+ c− a)(b+ c+ a)

(b+ c)2≤

(b+ c)2

4(b+ c− a)(b+ c+ a)

(b+ c)2

=(b+ c− a)(b+ c+ a)

4= z(x+ y + z).

5. In a triangle, show thatrarbmamb

+rbrcmbmc

+rcramcma

≥ 3.

Solution: We pass by dual principle to x, y, z. Then we must prove∑cyc

x(x+ y + z)

mbmc≥ 3.

We prove more∑cyc

x(x+y+z)m2

b+m2c≥ 3

2. Which transforms to

∑cyc

x(x+ y + z)

(y + z)(x+ y + z) + (x− z)2 + (y − z)2≥ 3

2

or ∑cyc

x2

y + z + [(x− z)2 + (x− y)2]x

x+ y + z

≥ 3

2.

Now applying (Cauchy) this reduces to

(x+ y + z)2 ≥ 3(xy + yz + zx) +∑cyc

x+ y

x+ y + z(x− y)2

which is just∑cyc

z

x+ y + z(x− y)2 ≥ 0 true.

6. Prove that: In the triangle ABC:

ma · cosA

2+mb · cos

B

2+mc · cos

C

2≥ 3

4(a+ b+ c).

Solution: We pass to dual principle. It’s easy to see that

m2a = x(x+ y + z) +

(y − z)2

4, cos

A

2=

√x(x+ y + z)

(x+ y)(x+ z).

Now let’s prove the following: ma cosA

2≥ p

(x

x+ y+

x

x+ z

)- where

p = x + y + z = a+b+c2 . The inequality would follow then by summing

the desired terms.

First, we consider the expression

(1) ma−√x(x+ y + z) =

m2a −

√x(x+ y + z)

2

ma +√x(x+ y + z)

=(y − z)2

4[ma +

√x(x+ y + z)

]103

Page 105: Олимпиадад бэлтгэгчдэд тусламж (Тэнцэтгэл биш 1)

Denote this expression by E.

Then we must have

(2)[√

x(x+ y + z) + E] √x(x+ y + z)√

(x+ y)(x+ z)− p

(x

x+ y+

x

x+ z

)=

=E√x(x+ y + z)√

(x+ y)(x+ z)− 1

2p

(√x

x+ y−√

x

x+ z

)2

.

We are to prove this expression is greater than zero.

However√x

x+ y−√

x

x+ z=

√x(x+ z)−

√x(x+ y)√

(x+ y)(x+ z)

=x(x+ z)− x(x+ y)[√

x(x+ z) +√x(x+ y)

]√(x+ y)(x+ z)

.

Squaring we get

(3)x2(y − z)2[√

x(x+ z) +√x(x+ y)

]2(x+ y)(x+ z)

.

Substituting 1) and 3) into 2) we produce√x(x+ y + z)(y − z)2

4[ma +

√x(x+ y + z)

] ≥ x2(x+ y + z)(y − z)2

2[√

x(x+ z) +√x(x+ y)

]2(x+ y)(x+ z)

.

Now cancelling common terms and grouping leads to

2[ma +

√x(x+ y + z)

]√x(x+ y + z) ≤

√(x+ y)(x+ z)

(√x+ y +

√x+ z

)2.

However√

(x+ y)(x+ z) ≥√x(x+ y + z) thus is suffices to prove that

2[ma +

√x(x+ y + z)

]≤(√x+ y +

√x+ z

)2= 2x+y+z+2

√(x+ y)(x+ z).

This breaks into the inequality 2ma < 2x+ y + z and√x(x+ y + z) ≤√

(x+ y)(x+ z) which are true (the first one is the well known ma ≤b+ c

2).

1.14 Substitutions

The Dual Principle actually consists of substituting x =b+ c− a

2, y =

a+ c− b2

, z =a+ b− c

2. There are many other substitutions, which solve

many inequalities impossible to solve in other way. Basically every inequalityis solved by substituting some variables into a known inequality, possibly morethan one time.

104

Page 106: Олимпиадад бэлтгэгчдэд тусламж (Тэнцэтгэл биш 1)

When we have some numbers with product 1, say xyz = 1, it’s convenient to

set x =a

b, y =

b

cthen z =

c

a.

For example the inequality

x

y+y

z+x

z≥ x+ y + z, xyz = 1

reduces toac

b2+ab

c2+bc

a2≥ ab2 + bc2 + ca2

abc,

ora3c3 + a3b3 + b3c3 ≥ a2b3c+ ab2c3 + a3bc2,

oru3 + v3 + w3 ≥ u2v + v2w + w2u,

where u = ab, v = ac, w = bc, and this is a known inequality - we’ve done itbefore.For the inequality

x11 + x1x2

+x2

1 + x2x3+ . . .+

xn1 + xnx1

> 1,

where x1x2 . . . xn = 1, we can substitute xi =aiai+1

, xn =ana1

, but this turns

xi1 + xixi+1

into a messyaiai+2

ai+1(ai + ai+2). The substitution xi =

ai+1

ai, however,

turns it intoai+1

ai + ai+2>

ai+1

a1 + a2 + . . .+ anand it’s done.

Another popular substitution is to work with the inverses of the numbersinstead of them.Look at this example:

If abc = 1 then1

a3(b+ c)+

1

b3(a+ c)+

1

c3(a+ c)≥ 3

2.

This time it’s convenient to set x =1

a= bc, y =

1

b= ac, z =

1

c= ab, the

inequality becoming thenx2

y + z+

y2

x+ z+

z2

x+ y≥ 3

2. However we know that

x2

y + z+

y2

x+ z+

z2

x+ y≥ x+ y + z

2≥ 3

2.

There are also trigonometric substitutions, based on the following relations:

cosA =b2 + c2 − a2

2bc

sinA

2=

√(p− b)(p− c)

bc=

√yz

(x+ y)(x+ z)

cosA

2=

√p(p− a)

bc=

√x(x+ y + z)

(x+ y)(x+ z)

and other analogous ones. (The notations are usual: a, b, c are the sides of the

triangle, A,B,C its angles, p =a+ b+ c

2its semiperimeter, x =

b+ c− a2

,

y =a+ c− b

2, z =

a+ b− c2

).

105

Page 107: Олимпиадад бэлтгэгчдэд тусламж (Тэнцэтгэл биш 1)

Via this substitutions, the inequality sinx+sin y+sin z ≤ 3

2, for x+y+z =

π

2,

for example, transforms to a purely algebraic one: if we consider the trianglewith angles 2x, 2y, 2z then

sinx+ sin y + sin z =

√y

x+ y

z

x+ z+

√x

x+ y

z

y + z+

√x

x+ z

y

y + z

≤ 1

2

(y

x+ y+

z

x+ z+

x

x+ y+

z

y + z+

x

x+ z+

y

y + z

)=

3

2.

The list of substitutions can be extended further, for example if xyz = x +

y + z + 2 then1

1 + x+

1

1 + y+

1

1 + z= 1 so

1

1 + x=

a

a+ b+ c,

1

1 + y=

b

a+ b+ c,

1

1 + z=

c

a+ b+ c

thus x =b+ c

a, y =

a+ c

b, z =

a+ b

c.

Thus, for example, we can prove that if x, y, z > 0 and xyz = x + y + z + 2

then xy + yz + zx ≥ 2(x + y + z). Indeed, after the substitution x =b+ c

a,

y =a+ c

b, z =

a+ b

c, we have to prove that

(a+ b)(a+ c)

bc+

(b+ a)(b+ c)

ac+

(c+ a)(c+ b)

ab≥ 2(

b

a+a

b+a

c+c

a+b

c+c

b),

ora2

bc+b2

ac+c2

ab+ 3 ≥ b

a+a

b+a

c+c

a+b

c+c

b.

After clearing denominators it becomes

a3 + b3 + c3 + 3abc ≥ ab(a+ b) + bc(b+ c) + ca(c+ a),

or Schur’s Inequality.

Exercises:

1. If a, b ∈ R\{−1, 1} then

∣∣∣∣(1− a2)(1− b2)(1 + a2)(1 + b2)

(1 +

4ab

(1− a2)(1− b2)

)∣∣∣∣ ≤ 1.

Solution: If a = tanx, b = tan y then∣∣∣∣(1− a2)(1− b2)(1 + a2)(1 + b2)

[1 +

4ab

(1− a2)(1− b2)

]∣∣∣∣ =

=

∣∣∣∣(1− tan2 x)(1− tan2 y)

(1 + tan2 x)(1 + tan2 y)

[1 +

4 tanx tan y

(1− tan2 x)(1− tan2 y)

]∣∣∣∣ = | cos(x− y)| ≤ 1.

2. If a, b, c > 0 thena2

(a+ b)(a+ c)+

b2

(b+ a)(b+ c)+

c2

(c+ a)(c+ b)≥ 3

4.

Solution: If we let x = b+ c, y = a+ c, z = a+ b the inequality turns to

(y + z − x)2

yz+

(x+ z − y)2

xz+

(x+ y − z)2

xy≥ 3

106

Page 108: Олимпиадад бэлтгэгчдэд тусламж (Тэнцэтгэл биш 1)

or after cancelling common terms to

x2

yz+y2

xz+z2

xy+ 3 ≥ x

y+y

x+x

z+z

x+y

z+z

y,

which multiplying by xyz comes to Schur.

3. Let a0 =1

2, and ak+1 = ak +

a2kn

. Show that 1− 1

n< an < 1.

Solution: Let bk =1

ak. Then bk+1 =

11

bk+

1

nb2k

=nb2k

nbk + 1= bk −

bknbk + 1

. Now if bk > 1 thenbk

nbk + 1∈(

1

n+ 1,

1

n

). As b0 = 2, we

can prove by induction on 1 ≤ k ≤ n that bk ∈(

2− k

n, 2− k

n+ 1

).

Particularly for k = n, bk ∈(

1,n+ 2

n+ 1

), which implies the problem.

4. If x, y, z > 0 with xy+yz+zx+2xyz = 1, then1

x+

1

y+

1

z≥ 4(x+y+z).

Solution: As1

x,

1

y,

1

zsatisfy

1

x+

1

y+

1

z+2

1

x· 1y· 1z

= 1, we have numbers

a, b, c with1

x=b+ c

a,

1

y=a+ c

b,

1

z=a+ b

c. The inequality to prove

now becomes

b+ c

a+a+ b

c+a+ c

b≥ 4

(a

b+ c+

c

a+ b+

b

a+ c

)which follows from 4

a

b+ c≤ a

b+a

cand the analogous relations.

5. Show that

√y + z

x+

√z + x

y+

√x+ y

z≥ 4 (x+ y + z)√

(y + z) (z + x) (x+ y)for

every three positive reals x, y, z.

Solution: For simplifying the solution set

x→√y + z, y →

√z + x, z →

√x+ y.

Then x2, y2, z2 are sides of a triangle and hence so do x, y, z that forman acute-angled triangle. The inequality transforms then to∑

cyc

x

y2 + z2 − x2≥ x2 + y2 + z2

xyz.

We can solve this inequality in two ways:

a) Applying cosine theorem to the acute-angled triangle formed by x, y, z

we transform the inequality to∑cyc

x

yz cosA≥ 2

x2 + y2 + z2

xyzand by mul-

tiplying it by xyz we have to prove∑cyc

x2

cosA≥ 2(x2 + y2 + z2).

107

Page 109: Олимпиадад бэлтгэгчдэд тусламж (Тэнцэтгэл биш 1)

However (x2, y2, z2) and

(1

cosA,

1

cosB,

1

cosC

)are clearly ordered the

same way hence by Chebyshev (see the appropriate chapter on ordering)we have ∑

cyc

x2

cosA≥∑cyc

x2

1

cosA+

1

cosB+

1

cosC3

and the desired inequality follows now from

1

cosA+

1

cosB+

1

cosC≥ 9

cosA+ cosB + cosC

(by CBS) and from the well-known inequality cosA+ cosB+ cos c ≤ 3

2.

b) The inequality is

∑cyc

(x

y2 + z2 − x2− x

yz

)≥ 0 or

∑cyc

x(x2 + yz − y2 − z2)yz(y2 + z2 − x2)

≥ 0.

Now using the fact that x, y, z form a triangle it’s quite clear that x(x2+

yz − y2 − z2), 1

yz(y2 + z2 − x2)are ordered the same way with x, y, z

and hence by Chebyshev inequality it suffices to prove that∑cyc

x(x2 +

yz − xy − xz) ≥ 0 which is just the renowned Schur’s Inequality.

6. If x1, x2, . . . , xn > 0 with1

x1+

1

x2+ . . .+

1

xn= n. Show that

x1 . . . xn − 1 ≥(n− 1

n

)n−1(x1 + . . .+ xn − n).

Solution: Let yi =1

xi. Then

n∑i=1

yi = n. Now multiplying the inequality

by y1 . . . yn we get a new inequality where we are allowed even to haveyi = 0.

Transform our inequality to f(y1, y2, . . . , yn) ≥ 0 where

f(y1, y2, . . . , yn) =

= nn−1(1− y1y2 . . . yn)+

+(n−1)n−1(ny1y2 . . . yn−y2y3 . . . yn−y1y3 . . . yn−. . .−y1y2 . . . yn−1).Consider y1, y2, . . . , yn with minimal value of f and of all such n-uples,one with minimal y21 + y22 + . . .+ y2n.

Now let’s try to decrease f . Pick up arbitrary non-zero yi, yj and fix theothers. Thus yi + yj is constant. Then f will have form ayiyj + b linearin yiyj and takes its minimum at the extremes, hence when yi = yj orone of them is 0. Thus we may replace yi, yj by their mean (type 1) ormake one of them 0 (type 2).

108

Page 110: Олимпиадад бэлтгэгчдэд тусламж (Тэнцэтгэл биш 1)

We keep doing transformations. We see that we can perform only finitelymany operations of type 2 because they increase the number of zeros inthe sequence. Thus at some time we shall have l zeros and some othernon-zero number every two of them we can replace by their arithmeticmean non-increasing f .

Now we can suppose the non-zero numbers are also equal(otherwise re-placing them by their mean decreases the sum of squares) hence they

aren

n− l. For this very particular case we can perform a trivial manual

checking.

7. Prove that for any seven numbers we may find two numbers x, y for

which 0 ≤ x− y1 + xy

≤ 1√3

.

Solution: Let the numbers be a1, a2, . . . , a7 and let ai = tan bi, where

0 ≤ bi ≤ π. We may partition [0, π] into six intervals of lengthπ

6. Then

some two numbers bi, bj will fit into the same interval then if bi < bj

we will have 0 ≤ bj − bi ≤π

6so 0 ≤ tan(bj − bi) ≤ tan

π

6=

1√3

, thus

0 ≤ aj − ai1 + ajai

and we may take x = aj , y = ai.

1.15 Homogenization and dehomogenization

If we have to solve a non-homogeneous inequality,we may want to make ithomogeneous to deal with more familiar homogeneous inequalities (this isbecause we already have a luggage of known inequalities to which we canreduce it). Look, for example, at the inequality(a2 +

1

b

)(b2 +

1

c

)(c2 +

1

a

)≥ 8

9(a+b+c)

(1

a+

1

b+

1

c

), for a, b, c > 0, abc = 1.

The inequality is not homogeneous, but we can use the condition abc = 1 to

make it homogeneous, by replacing1

a,

1

b,

1

cby bc, ac, ab. It becomes then

(a2 + ac)(b2 + ab)(c2 + bc) ≥ 8

9(a+ b+ c)(ab+ bc+ ca).

We can the remove abc = 1 from the left-hand side and then we get the alreadyknown homogeneous inequality

(a+ b)(b+ c)(c+ a) ≥ 8

9(a+ b+ c)(ab+ bc+ ca).

In other cases is better to dehomogenize an inequality to reduce the number ofvariables or add a good condition on them. For example, if the inequality is injust two variables, we may set one of them be 1 (for example the inequality x3+

y3 ≥ x2y+xy2 reduces to x3+1 ≥ x2+x be dividing by y3 and letting x→ x

y).

109

Page 111: Олимпиадад бэлтгэгчдэд тусламж (Тэнцэтгэл биш 1)

The inequality then may be reduced to a polynomial in one variable and solvedby algebraic methods applicable to polynomials (i.e. finding the roots). Otherexample of dehomogenization is the AM-GM inequality x1 + x2 + . . .+ xn ≥n n√x1x2 . . . xn. By replacing x1, x2, . . . , xn by

x1n√x1 . . . xn

, . . . ,xn

n√x1 . . . xn

we

can assume x1x2 . . . xn = 1 and then write x1 =y1y2, x2 =

y2y3, . . . , xn =

yny1

.

The inequality then becomes

y1y2

+y2y3

+ . . .+yny1≥ n,

and it can be solved by unimonotonic sequences (refer to the chapter on or-dering).

Exercises

1. If ai, bi, ci > 0 then

(a1b1c1 + . . .+ anbncn)3 ≤ (a31 + . . .+ a3n)(b31 + . . .+ b3n)(c31 + . . .+ c3n).

Solution: The quantity

(a1b1c1 + . . .+ anbncn)3

(a31 + . . .+ a3n)(b31 + . . .+ b3n)(c31 + . . .+ c3n)

doesn’t change if we replace each ai by tai. Therefore we may assumethat a31 + . . .+a3n = 1 and analogously b31 + . . .+b3n = 1, c31 + . . .+c3n = 1.

Then aibic≤a3i + b3i + c3i

3.

By summing this inequalities we get a1b1c1 + . . . + anbncn ≤ 1 and theconclusion follows.

Remark: This also easily follows from Holder’s inequality. The solutionwe gave above is essentially the same as the inequality aibici ≤ 1

3(pa3i +

qb3i+rc3i ) where p =

a31+...+a3n

3√

(a31+...+a3n)(b

31+...+b

3n)(c

31+...+c

3n), q =

b31+...+b3n

3√

(a31+...+a3n)(b

31+...+b

3n)(c

31+...+c

3n), r =

c31+...+c3n

3√

(a31+...+a3n)(b

31+...+b

3n)(c

31+...+c

3n)

- except this method makes it easier to

see. Of course, this is also how we prove Holder’s inequality.

2. If x, y, z > 0 then x3y + y3z + z3x ≥ xyz(x+ y + z).

Solution: Assume that x is the smallest of x, y, z. Then as the inequalityis homogeneous we can assume x = 1, y = 1 + u, z = 1 + v.

The inequality now after cancelling common terms becomes

(u− v)2 + (u3 + v3 − uv2) + u2 + v2 + 2u2v + u3v ≥ 0

and is clearly true.

3. For every positive a, b and c such that a2 + b2 + c2 = 1 prove the

inequality:∑cyc

a

a3 + bc> 3 (the inequality is strict).

110

Page 112: Олимпиадад бэлтгэгчдэд тусламж (Тэнцэтгэл биш 1)

Solution: By letting x = a2, y = b2, z = c2 and homogenizing we have

to prove the following∑cyc

x(x+ y + z)

x2 + t≥ 3 where t =

√xyz(x+ y + z).

Which is in turn equivalent to

(x+ y + z)∑cyc

x(y2 + t)(z2 + t)− 3(x2 + t)(y2 + t)(z2 + t) ≥ 0.

Now let u = x+ y + z, v = xy + yz + zx, w = xyz. We transform ourinequality to

g(x, y, z) = u(vw + (uv − 3w)√uw + u3w)− 3w2 −

−3(v2 − 2uw)√uw − 3(u2 − 2v)uw − 3uw

√uw ≥ 0.

Now let’s assume that this inequality fails for some x, y, z thus g(x, y, z) <0.

Now keep u,w fixed and let’s try to vary v in order to minimize g.It can run on a set intervals of reals that ensures that the polynomialp(x) = x3− ux2 + vx−w = 0 has solutions or equivalently that the line

y = −v intersects the graph of function x2 − ux − w

xin three points.

Now g(x, y, z) transforms to a quadratic expression in v with leadingcoefficient negative thus it takes minimum at some interval. Howeverthe extremum for v realizes when the line x = −v touches the graph ofthe function somewhere which implies that the equation has a doubleroot. This means that two of x, y, z are equal. So we need to check therelation only for x = y, z = 1. This is quite simple to check: after clearingdenominators this expression transforms to (x2+4x−2)

√2x+ 1 ≥ 4x2+

x− 2.

This inequality can be easily proved as follows: If 4x2+x−2 ≥ 0 then we

shall have x ≥√

33− 1

8> 0.6 and necessarily x2 + 4x− 2 ≥ 0 and thus

by squaring we deduce that we have to prove 2x4+x3+24x2−5x−4 > 0which is clearly true for x > 0.6. If 4x2 + x − 2 < 0 we must look onlyat the case x2 + 4x− 2 < 0 which is x <

√6− 2.

By squaring we must prove the inequality 2x4 + x3 + 24x2 − 5x− 4 < 0which again easily follows from x <

√6− 2.

1.16 Unimonotonic sequences

Suppose that x1 > x2 and y1 > y2. Then we have

(x1y1 + x2y2)− (x1y2 + x2y1) = (x1 − x2)(y1 − y2) ≥ 0.

Now take numbers x1, x2, . . . , xn, y1, y2, . . . , yn and consider the quantity

Q(y1, y2, . . . , yn) = x1y1 + . . .+ xnyn.

111

Page 113: Олимпиадад бэлтгэгчдэд тусламж (Тэнцэтгэл биш 1)

If xi > xj but yi < yj then by exchanging yi with yj we increase Q by

(xi − xj)(yj − yi) > 0.

If yi > yj then by exchanging yi with yj we decrease Q by (xi − xj)(yi − yj).Now suppose x1 > x2 > . . . > xn.

Let z1, z2, . . . , zn be the permutation that maximizes Q and let z′1, z′2, . . . , z

′n

be the permutation that minimizes Q. If zi > zj for some i < j then we couldincrease Q which contradicts the maximality of Q(z1, z2, . . . , zn). That’s whyz1 ≥ z2 ≥ . . . ≥ zn. Analogously we deduce z′1 ≤ z′2 ≤ . . . ≤ z′n (and soz′1 = zn, z

′2 = zn−1, . . . , z

′n = z1).

So, we proved the following theorem:

Theorem (Rearrangement Inequality). If x1 ≥ x2 ≥ . . . ≥ xn and y1, y2, . . . , ynare real numbers then

x1z1 + . . .+ xnzn ≥ x1y1 + x2y2 + . . .+ xnyn ≥ x1zn + . . .+ xnz1,

where z1 ≥ z2 ≥ . . . ≥ zn is a decreasing permutation of y1, y2, . . . , yn.

The sequences x1, x2, . . . , xn and y1, y2, . . . , yn are called unimonotonic if theyare ordered in the same way (i.e (xi − xj)(yi − yj) ≥ 0 for any i, j). If theyare inversely ordered (i.e. (xi − xj)(yi − yj) ≤ 0) we call them antimono-tonic. By the above theorem, the quantity x1y1 + . . .+ xnyn is maximized byunimonotonic sequences and minimized by antimonotonic sequences.

For example, if yi =1

xiwhere xi > 0 then x1, x2, . . . , xn and y1, y2, . . . , yn are

antimonotonic sequences so

Q(y1, y2, . . . , yn) ≤ Q(y2, . . . , yn, y1).

Thusx1x2

+ . . .+xnx1≥ n

which proves the AM-GM Inequality as it is equivalent to it (see the lastexample from the previous chapter - or the first exercise from this one).

If y1 ≥ y2 ≥ . . . ≥ yn and x1 ≥ x2 ≥ . . . ≥ xn then

x1y1 + x2y2 + . . . xnyn ≥ x1y1+i + x2y2+i + . . .+ xnyn+i

(we consider yn+k = yk). Summing this i = 0, 1, . . . , n− 1 we obtain

Theorem (Chebyshev’s Inequality)

n(x1y1 + . . .+ xnyn) ≥ (x1 + . . .+ xn)(y1 + y2 + . . .+ yn)

If y1 < y2 < . . . < yn, it changes sign.

A consequence of Chebyshev’s inequality is the following inequality (which canalso be proved by induction):

If x1, x2, . . . , xn and y1, y2, . . . , yn are antimonotonic then

x1y1

+ . . .+xnyn≥ nx1 + . . .+ xn

y1 + . . .+ yn.

112

Page 114: Олимпиадад бэлтгэгчдэд тусламж (Тэнцэтгэл биш 1)

Proof : By Chebyshev’s inequality

x1y1

+ . . .+xnyn≥ 1

n(x1 + x2 + . . .+ xn)

(1

y1+ . . .+

1

yn

).

Since1

y1+ . . .+

1

yn≥ n2

y1 + . . .+ yn,

the conclusion follows.

Exercises

1. Show that if x1, x2, . . . , xn > 0 then x1 + x2 + . . .+ xn ≥ n n√x1x2 . . . xn.

Solution: As the inequality is homogeneous, we can assume x1x2 . . . xn =

1 then we can write x1 =a1a2, x2 =

a2a3, . . . , xn =

ana1

.

And the inequality becomesa1a2

+ . . . +ana1≥ n, so it follows from the

antimonocity of the sequences (a1, a2, . . . , an) and

(1

a1,

1

a2, . . . ,

1

an

).

2. Show that if a, b, c > 0 thenb2 − a2

c+ a+c2 − b2

a+ b+a2 − c2

b+ c≥ 0.

Solution: We can rewrite the inequality as

b2

c+ a+

c2

a+ b+

a2

b+ c≥ a2

c+ a+

b2

a+ b+

c2

b+ c

which follows from the unimonotonicity of (b2, a2, c2) and

(1

a+ c,

1

b+ c,

1

a+ b

).

3.a3

b2 − bc+ c2+

b3

a2 − ac+ c2+

c3

a2 − ab+ b2≥ a+ b+ c.

Solution: We can suppose that a ≥ b ≥ c. Now note that we have

(b2 − bc+ c2)− (a2 − ac+ c2) = (b− a)(b+ a− c).

Hence we have two cases to consider:

a) a, b, c form a triangle. In this case (a3, b3, c3) and(1

b2 − bc+ c2,

1

a2 − ac+ c2,

1

a2 − ab+ b2

)are ordered the same way, so

2

(a3

b2 − bc+ c2+

b3

a2 − ac+ c2+

c3

a2 − ab+ b2

)≥

≥ b3 + c3

b2 − bc+ c2+

a3 + c3

a2 − ac+ c2+

a3 + b3

a2 − ab+ b2= a+ b+ c.

113

Page 115: Олимпиадад бэлтгэгчдэд тусламж (Тэнцэтгэл биш 1)

b) a > b + c. Let’s denote x =1

b2 − bc+ c2, y =

1

a2 − ac+ c2, z =

1

a2 − ab+ b2. Then x ≥ z ≥ y. Hence a3x+b3y+c3z ≥ a3y+b3x+c3z ≥

a3y+b3z+c3x and also a3x+b3y+c3z ≥ a3z+b3y+c3x ≥ a3z+b3x+c3y.

By summing these two relation we get again

2

(a3

b2 − bc+ c2+

b3

a2 − ac+ c2+

c3

a2 − ab+ b2

)≥

≥ b3 + c3

b2 − bc+ c2+

a3 + c3

a2 − ac+ c2+

a3 + b3

a2 − ab+ b2= a+ b+ c.

4. If α > 0 and x, y, z > 0 then

xα+2

(x+ y)(x+ z)+

yα+2

(y + x)(y + z)+

zα+2

(z + y)(z + x)≥ 1

4(xα + yα + zα).

Solution: We can see that the sequences

(xα, yα, zα) and

(x2

(x+ y)(x+ z),

y2

(y + x)(y + z),

z2

(z + x)(z + y)

)are ordered the same way, hence

xα+2

(x+ y)(x+ z)+

yα+2

(y + x)(y + z)+

zα+2

(z + y)(z + x)≥

≥ 1

3

[x2

(x+ y)(x+ z)+

y2

(y + x)(y + z)+

z2

(z + x)(z + y)

](xα + yα + zα).

Moreover, we havex2

(x+ y)(x+ z)+

y2

(y + x)(y + z)+

z2

(z + x)(z + y)≥ 3

4from CBS Lemma.

5. Let x1 ≤ x2 ≤ . . . ≤ xn and y1 ≤ y2 ≤ . . . ≤ yn be real numbers. Forany permutations (z1, z2, . . . , zn) of (y1, y2, . . . , yn), prove that

(x1−y1)2+(x2−y2)2+. . .+(xn−yn)2 ≤ (x1−z1)2+(x2−y2)2+. . .+(xn−nn)2.

Solution: Notice that y21+y22+. . .+y2n = z21+z22+. . .+z2n. After expansionand simplification, the desired inequality is equivalent to x1y1 + x2y2 +. . .+ xnyn ≥ x1z1 + x2z2 + . . .+ xnzn, which is just the Rearrangementinequality.

6. (Nesbitt’s Inequality) Let a, b, c be positive real numbers. Prove that

a

b+ c+

b

c+ a+

c

a+ b≥ 3

2.

Solution: Without loss of generality, we can suppose that a ≥ b ≥ c,then

1

b+ c≥ 1

c+ a≥ 1

a+ b.

114

Page 116: Олимпиадад бэлтгэгчдэд тусламж (Тэнцэтгэл биш 1)

Therefore, applying the Rearrangement inequality, we obtain

a

b+ c+

b

c+ a+

c

a+ b= a · 1

b+ c+ b · 1

c+ a+ c · 1

a+ b

≥ a · 1

a+ b+ b · 1

b+ c+ c · 1

c+ a

=a

a+ b+

b

b+ c+

c

c+ a,

and

a

b+ c+

b

c+ a+

c

a+ b= a · 1

b+ c+ b · 1

c+ a+ c · 1

a+ b

≥ a · 1

c+ a+ b · 1

a+ b+ c · 1

b+ c

=a

c+ a+

b

a+ b+

c

a+ b,

Adding up these two inequalities, we obtain

2

(a

b+ c+

b

c+ a+

c

a+ b

)≥ a+ b

a+ b+b+ c

b+ c+c+ a

c+ a= 3.

This completes the proof.

7. Let a1, a2, . . . , an be distinct positive integers. Prove that

a112

+a222

+ . . .+ann2≥ 1 +

1

2+ . . .+

1

n.

Solution: Let (a′1, a′2, . . . , a

′n) be a cyclic permutation of (a1, a2, . . . , an)

such that a′1 < a′2 < . . . < a′n. Then we have a′i ≥ i for all i = 1, 2, . . . , n.Now, we have that

a′1 < a′2 < . . . < a′n and1

12>

1

22> . . . >

1

n2.

Therefore, according to the Rearrangement inequality, we obtain

a112

+a222

+ . . .+ann2

= a1 ·1

12+ a2 ·

1

22+ . . .+ an ·

1

n2

≥ a′1 ·1

12+ a′2 ·

1

22+ . . .+ a′n ·

1

n2

≥ 1 · 1

12+ 2 · 1

22+ . . .+ n · 1

n2

= 1 +1

2+ . . .+

1

n.

This completes our proof. Equality holds if and only if ai = i for alli = 1, 2, . . . , n.

8. Let a, b, c be the side lengths of a triangle. Prove that

a2(b+ c− a) + b2(c+ a− b) + c2(a+ b− c) ≤ 3abc.

115

Page 117: Олимпиадад бэлтгэгчдэд тусламж (Тэнцэтгэл биш 1)

Solution: We will firstly show that if a ≥ b, then

a(b+ c− a) ≤ b(c+ a− b).

Indeed, we have

b(c+ a− b)− a(b+ c− a) = (a− b)(a+ b− c) ≥ 0.

Now, we see that the inequality is symmetric, hence we can assume thata ≥ b ≥ c. Then, from the above inequality, we have

a ≥ b ≥ c and c(a+ b− c) ≥ b(c+ a− b) ≥ a(b+ c− a).

Thus, according to the Rearrangement inequality, we obtain

a · a(b+ c− a) + b · b(c+ a− b) + c · c(a+ b− c) ≤≤ b · a(b+ c− a) + c · b(c+ a− b) + a · c(a+ b− c),

and

a · a(b+ c− a) + b · b(c+ a− b) + c · c(a+ b− c) ≤≤ c · a(b+ c− a) + a · b(c+ a− b) + b · c(a+ b− c).

Adding both these inequalities, we have

2[a2(b+ c− a) + b2(c+ a− b) + c2(a+ b− c)

]≤ 6abc,

which is

a2(b+ c− a) + b2(c+ a− b) + c2(a+ b− c) ≤ 3abc.

9. Let a, b, c be the side lengths of a triangle. Prove that

a2b(a− b) + b2c(b− c) + c2a(c− a) ≥ 0.

Solution: Without loss of generality, we can suppose that a = max {a, b, c}and consider 2 cases

Case 1. If a ≥ b ≥ c, then from the above problem, we have

1

a≤ 1

b≤ 1

cand a(b+ c− a) ≤ b(c+ a− b) ≤ c(a+ b− c).

And the Rearrangement inequality yields that

a+ b+ c =1

a· a(b+ c− a) +

1

b· b(c+ a− b) +

1

c· c(a+ b− c)

≥ 1

c· a(b+ c− a) +

1

a· b(c+ a− b) +

1

b· c(a+ b− c)

= a+ b+ c− a2b(a− b) + b2c(b− c) + c2a(c− a)

abc.

This implies that

a2b(a− b) + b2c(b− c) + c2a(c− a) ≥ 0.

116

Page 118: Олимпиадад бэлтгэгчдэд тусламж (Тэнцэтгэл биш 1)

Case 2. If a ≥ c ≥ b, then from the above problem, we have

1

a≤ 1

c≤ 1

band a(b+ c− a) ≤ c(a+ b− c) ≤ b(c+ a− b).

Thus, by Rearrangement inequality, we see that

a+ b+ c =1

a· a(b+ c− a) +

1

c· c(a+ b− c) +

1

b· b(c+ a− b)

≥ 1

c· a(b+ c− a) +

1

b· c(a+ b− c) +

1

a· b(c+ a− b)

= a+ b+ c− a2b(a− b) + b2c(b− c) + c2a(c− a)

abc.

Thena2b(a− b) + b2c(b− c) + c2a(c− a ≥ 0,

and our proof is complete.

10. Let a, b, c be positive real numbers. Prove that

a

b+b

c+c

a≥ a+ c

b+ c+b+ a

c+ a+c+ b

a+ b.

Solution: We rewrite our inequality as(a

b− a

b+ c

)+

(b

c− b

c+ a

)+

(c

a− c

a+ b

)≥ b

a+ b+

c

b+ c+

a

c+ a,

ca

b(b+ c)+

ab

c(c+ a)+

bc

a(a+ b)≥ b

a+ b+

c

b+ c+

a

c+ a.

Applying Cauchy Schwarz inequality,(b

a+ b+

c

b+ c+

a

c+ a

)2

≤[

ab

c(a+ b)+

bc

a(b+ c)+

ca

b(c+ a)

] [bc

a(a+ b)+

ca

b(b+ c)+

ab

c(c+ a)

].

It remains to show that

ab

c(a+ b)+

bc

a(b+ c)+

ca

b(c+ a)≤ ca

b(b+ c)+

ab

c(c+ a)+

bc

a(a+ b).

Let (x, y, z) be a cyclic permutation of (a, b, c) such that x ≥ y ≥ z, then

xy

z≥ zx

y≥ yz

xand

1

x+ y≤ 1

z + x≤ 1

y + z.

Therefore, according to the Rearrangement inequality, we have

bc

a· 1

a+ b+ab

c· 1

c+ a+ca

b· 1

b+ c≥ xy

z· 1

x+ y+zx

y· 1

z + x+yz

x· 1

y + z

=xy

z(x+ y)+

yz

x(y + z)+

zx

y(z + x)

=ab

c(a+ b)+

bc

a(b+ c)+

ca

b(c+ a).

Our proof is complete. Equality holds if and only if a = b = c.

117

Page 119: Олимпиадад бэлтгэгчдэд тусламж (Тэнцэтгэл биш 1)

11. Let a, b, c be positive real numbers. Prove that

a+ b

b+ c+b+ c

c+ a+c+ a

a+ b≤ (a+ b+ c)2

ab+ bc+ ca.

Solution: Rewrite our inequality as

(a+ b) [a(b+ c) + bc]

b+ c+

(b+ c) [b(c+ a) + ca]

c+ a+

(c+ a) [c(a+ b) + ab]

a+ b≤ (a+b+c)2,

bc(a+ b)

b+ c+ca(b+ c)

c+ a+ab(c+ a)

a+ b≤ ab+ bc+ ca.

Let (x, y, z) be a cyclic permutation of (a, b, c) such that x ≥ y ≥ z, then

x+ y ≥ z + x ≥ y + z andxy

x+ y≥ zx

z + x≥ yz

y + z.

Therefore, according to the Rearrangement inequality, we obtain

ab+ bc+ ca = xy + yz + zx

= xy · xy

x+ y+ zx · zx

z + x+ yz · yz

y + z

≥ (c+ a) · ab

a+ b+ (b+ c) · ca

c+ a+ (a+ b) · bc

b+ c

=bc(a+ b)

b+ c+ca(b+ c)

c+ a+ab(c+ a)

a+ b.

Our proof is complete.

12. Let a, b, c, d be nonnegative real numbers such that a + b + c + d = 4.Prove that

a2bc+ b2cd+ c2da+ d2ab ≤ 4.

Solution: Let (x, y, z, t) is a cyclic permutation of (a, b, c, d) such thatx ≥ y ≥ z ≥ t, then xyz ≥ xyt ≥ xzt ≥ yzt. Therefore, according to theRearrangement inequality, we obtain

x · xyz + y · xyt+ z · xzt+ t · yzt ≥ a · abc+ b · bcd+ c · cda+ d · dab= a2bc+ b2cd+ c2da+ d2ab.

That is

a2bc+ b2cd+ c2da+ d2ab ≤ x · xyz + y · xyt+ z · xzt+ t · yzt

= (xy + tz)(xz + ty) ≤(xy + tz + xz + ty

2

)2

=

[(x+ t)(y + z)

2

]2≤[

(x+ y + z + t)2

8

]2= 4.

Remark. We can also prove this inequality by AM-GM inequality asfollows:

118

Page 120: Олимпиадад бэлтгэгчдэд тусламж (Тэнцэтгэл биш 1)

We have

a2bc+ b2cd+ c2da+ d2ab = ac(ab+ cd) + bd(bc+ da).

If ab+ cd ≥ bc+ da, then

a2bc+ b2cd+ c2da+ d2ab ≤ ac(ab+ cd) + bd(ab+ cd)

= (ac+ bd)(ab+ cd) ≤(ac+ bd+ ab+ cd

2

)2

=

[(a+ d)(b+ c)

2

]2≤[

(a+ b+ c+ d)2

8

]2= 4.

If bc+ da ≥ ab+ cd, then

a2bc+ b2cd+ c2da+ d2ab ≤ ac(bc+ da) + bd(bc+ da)

= (ac+ bd)(bc+ da) ≤(ac+ bd+ bc+ da

2

)2

=

[(a+ b)(c+ d)

2

]2≤[

(a+ b+ c+ d)2

8

]2= 4.

This completes the proof.

13. Let a, b, c, d be positive real numbers. Prove that(a

a+ b+ c

)2

+

(b

b+ c+ d

)2

+

(c

c+ d+ a

)2

+

(d

d+ a+ b

)2

≥ 4

9.

Solution: Let (x, y, z, t) is a cyclic permutation of (a, b, c, d) such thatx ≥ y ≥ z ≥ t, then

1

(x+ y + z)2≥ 1

(x+ y + t)2≥ 1

(x+ z + t)2≥ 1

(y + z + t)2.

Therefore, according to the Rearrangement inequality, we obtain

x2 · 1

(x+ y + z)2+ y2 · 1

(x+ y + t)2+ z2 · 1

(x+ z + t)2+ t2 · 1

(y + z + t)2≤

≤(

a

a+ b+ c

)2

+

(b

b+ c+ d

)2

+

(c

c+ d+ a

)2

+

(d

d+ a+ b

)2

.

It remains to show that

x2

(x+ y + z)2+

y2

(x+ y + t)2+

z2

(x+ z + t)2+

t2

(y + z + t)2≥ 4

9.

The inequality being homogeneous, hence we can assume that x + y +z + t = 1. The above inequality becomes[

x2

(1− t)2+

t2

(1− x)2

]+

[y2

(1− z)2+

z2

(1− y)2

]≥ 4

9.

119

Page 121: Олимпиадад бэлтгэгчдэд тусламж (Тэнцэтгэл биш 1)

Setting u = x+ t (0 < u < 1), and v = xt, we have

x2

(1− t)2+

t2

(1− x)2=

2v2 − 2(u− 1)(2u− 1)v + u2(1− u)2

(1− u+ v)2= f(v),

and

f ′(v) =2(1− u)

[(3− 2u)v − (1− u)3

](1− u+ v)3

,

f ′(v) = 0⇔ v =(1− u)3

3− 2u.

From now, by making variation board, we obtain

f(v) ≥ f(

(1− u)3

3− 2u

)=−2u2 + 4u− 1

(2− u)2∀v > 0.

Similarly, if we put w = y + z = 1− u, then we also have

y2

(1− z)2+

z2

(1− y)2≥ −2w2 + 4w − 1

(2− w)2=−2(1− u)2 + 4(1− u)− 1

(1 + u)2=

1− 2u2

(1 + u)2.

It remains to show that

−2u2 + 4u− 1

(2− u)2+

1− 2u2

(1 + u)2≥ 4

9,

which can be easily simplified to

(1− 2u)2(11 + 10u− 10u2)

9(1 + u)2(2− u)2≥ 0,

which is clearly nonnegative and our proof is complete.

Remark. We can also prove this inequality using Holder inequality. SeeOld And New Inequalities 2, Vo Quoc Ba Can - Cosmin Pohoata, GILpublishing house, 2008.

14. Let a, b, c be positive real numbers such that a2 + b2 + c2 = 3. Prove that

a3b2 + b3c2 + c3a2 ≤ 3.

Solution: Let (x, y, z) be a cyclic permutation of (a, b, c) such that x ≥y ≥ z, then x2y2 ≥ x2z2 ≥ y2z2. Hence, the Rearrangement inequalityyields that

a3b2 + b3c2 + c3a2 = a · a2b2 + b · b2c2 + c · c2a2

≤ x · x2y2 + y · x2z2 + z · y2z2

= y(x3y + yz3 + x2z2

)≤ y

(x2 · x

2 + y2

2+ z2 · y

2 + z2

2+ x2z2

)=

1

2y(x2 + z2)(x2 + y2 + z2) =

3

2y(x2 + z2)

≤ 3

√√√√(y2 + x2+z2

2 + x2+z2

2

3

)3

= 3.

120

Page 122: Олимпиадад бэлтгэгчдэд тусламж (Тэнцэтгэл биш 1)

This completes our proof. Equality holds if and only if a = b = c = 1.

Remark. We can also prove this inequality using Cauchy Schwartz in-equality. See Old And New Inequalities 2.

15. Let a, b, c be nonnegative real numbers such that a + b + c = 1. Provethat

a2b+ b2c+ c2a ≤ 4

27.

)

Solution: Let (x, y, z) be a cyclic permutation of (a, b, c) such that x ≥y ≥ z, then xy ≥ xz ≥ yz. Hence, the Rearrangement inequality yields

a2b+ b2c+ c2a = a · ab+ b · bc+ c · ca= x · xy + y · xz + z · yz= y(x2 + z2 + xz) ≤ y(x+ z)2

≤ 1

2

(2y + x+ z + x+ z

3

)3

=4

27.

16. Let a, b, c be positive real numbers such that a2 + b2 + c2 = 3. Prove that

a2b+ b2c+ c2a ≤ 2 + abc.

Solution: Similar to the above problem, let (x, y, z) be a cyclic permuta-tion of (a, b, c) such that x ≥ y ≥ z, then by Rearrangement inequality,we also have that

a2b+ b2c+ c2a ≤ y(x2 + z2 + xz).

And we can deduce our inequality to

y(x2 + z2 + xz) ≤ 2 + xyz,

y(x2 + z2) ≤ 2,

which is true because

y(x2 + z2) = y(3− y2) = 2− (y + 2)(y − 1)2 ≤ 2.

17. Let a, b, c be nonnegative real numbers, no two of which are zero. Provethat

a2 + b2 + c2

ab+ bc+ ca+

4abc

a2b+ b2c+ c2a+ abc≥ 2.

Solution: Similar to the above problem, let (x, y, z) be a cyclic permuta-tion of (a, b, c) such that x ≥ y ≥ z, then by Rearrangement unequality,we also have that

a2b+ b2c+ c2a ≤ y(x2 + z2 + xz).

121

Page 123: Олимпиадад бэлтгэгчдэд тусламж (Тэнцэтгэл биш 1)

And we can deduce our inequality to

x2 + y2 + z2

xy + yz + zx+

4xyz

y(x2 + z2 + xz) + xyz≥ 2,

x2 + y2 + z2

xy + yz + zx≥ 2(x2 + z2)

(x+ z)2,

which can be easily simplified to

(xy + yz − x2 − z2)2 ≥ 0,

Equality holds if and only if a = b = c or a = b, c = 0 and its cyclicpermutations.

Remark. By the same manner, we can prove that

4(a2 + b2 + c2)

ab+ bc+ ca+

9abc

a2b+ b2c+ c2a≥ 7,

2(a2 + b2 + c2)

(a+ b+ c)2+

abc

a2b+ b2c+ c2a≥ 1.

18. Let a, b, c be positive real numbers. Prove that

a√a+ b

+b√b+ c

+c√c+ a

≥√a+√b+√c√

2.

Solution: Setting x =√a, y =

√b, z =

√c and squaring both sides, we

can rewrite our inequality as

2∑cyc

x4

x2 + y2+ 4∑cyc

x2y2√(x2 + y2)(y2 + z2)

≥ (x+ y + z)2.

Let (m,n, p) be a cyclic permutation of (x, y, z) such that m ≥ n ≥ p,then

m2n2√m2 + n2

≥ p2m2√p2 +m2

≥ n2p2√n2 + p2

and1√

m2 + n2≤ 1√

p2 +m2≤ 1√

n2 + p2.

Therefore, using Rearrangement inequality, we obtain∑cyc

x2y2

x2 + y2=

=m2n2

m2 + n2+

p2m2

p2 +m2+

n2p2

n2 + p2

=m2n2√m2 + n2

· 1√m2 + n2

+p2m2√p2 +m2

· 1√p2 +m2

+n2p2√n2 + p2

· 1√n2 + p2

≤ x2y2√x2 + y2

· 1√y2 + z2

+z2x2√z2 + x2

· 1√x2 + y2

+y2z2√y2 + z2

· 1√y2 + z2

=∑cyc

x2y2√(x2 + y2)(y2 + z2)

.

122

Page 124: Олимпиадад бэлтгэгчдэд тусламж (Тэнцэтгэл биш 1)

It remains to show that

2∑cyc

x4

x2 + y2+ 4∑cyc

x2y2

x2 + y2≥ (x+ y + z)2,

which is obviously true because

2∑cyc

x4

x2 + y2=

∑cyc

x4 + y4

x2 + y2+∑cyc

x4 − y4

x2 + y2

=∑cyc

x4 + y4

x2 + y2+∑cyc

(x2 − y2)

=∑cyc

x4 + y4

x2 + y2,

and ∑cyc

x4 + y4

x2 + y2+ 4∑cyc

x2y2

x2 + y2=

=∑cyc

(x4 + y4 + 4x2y2

x2 + y2− x2 + y2 + 4xy

2

)+∑cyc

x2 + y2 + 4xy

2

=∑cyc

(x− y)4

2(x2 + y2)+

(∑cyc

x

)2

(∑cyc

x

)2

.

Remark. In the same manner, we can prove that√a3

a2 + ab+ b2+

√b3

b2 + bc+ c2+

√c3

c2 + ca+ a2≥√a+√b+√c√

3.

19. Let x, y, z be positive real numbers. Prove that

x√x+ y

+y√y + z

+z√z + x

≤ 3√

3

√(x+ y)(y + z)(z + x)

xy + yz + zx.

Solution: We cam rewrite our inequality as

∑cyc

x√(x+ y)(x+ z)

· 1√(x+ y)(y + z)

≤ 3√

3

4√xy + yz + zx

.

Let (a, b, c) be a cyclic permutation of (x, y, z) such that a ≥ b ≥ c, then

a√(a+ b)(a+ c)

≥ b√(b+ c)(b+ a)

≥ c√(c+ a)(c+ b)

and

1√(a+ b)(a+ c)

≤ 1√(b+ c)(b+ a)

≤ 1√(c+ a)(c+ b)

.

123

Page 125: Олимпиадад бэлтгэгчдэд тусламж (Тэнцэтгэл биш 1)

Hence, by Rearrangement inequality, we obtain∑cyc

x√(x+ y)(x+ z)

· 1√(x+ y)(y + z)

≤ a√(a+ b)(a+ c)

· 1√(c+ a)(c+ b)

+b√

(b+ c)(b+ a)· 1√

(b+ c)(b+ a)+

+c√

(c+ a)(c+ b)· 1√

(a+ b)(a+ c)

=1√

(b+ c)(b+ a)

[1 +

b√(b+ c)(b+ a)

].

It remains to show that√ab+ bc+ ca

(b+ c)(b+ a)

[1 +

b√(b+ c)(b+ a)

]≤ 3√

3

4.

Indeed, setting u = b√(b+c)(b+a)

≤ 1, then√

xy+yz+zx(y+z)(y+x) =

√1− u2. Ap-

plying AM-GM inequality, we have√ab+ bc+ ca

(b+ c)(b+ a)

[1 +

b√(b+ c)(b+ a)

]= (1 + u)

√1− u2 =

√(1 + u)3(1− u)

=

√(1 + u) · (1 + u) · (1 + u) · 3(1− u)

3

√(3/2)4

3=

3√

3

4.

Our inequality is proved. Equality holds if and only if x = y = z.

20. Let a, b, c be nonnegative real numbers, no two of which are zero. Provethat

a√a+ b

+b√b+ c

+c√c+ a

≤ 5

4

√a+ b+ c.

Solution: After squaring, we can rewrite our inequality as∑cyc

a2

a+ b+ 2∑cyc

ab√(a+ b)(b+ c)

≤ 25

16

∑cyc

a,

2∑cyc

ab√(a+ b)(b+ c)

≤ 9

16

∑cyc

a+∑cyc

ab

a+ b.

Let (x, y, z) be a cyclic permutation of (a, b, c) such that x ≥ y ≥ z, then

xy√x+ y

≥ zx√z + x

≥ yz√y + z

,

and1√y + z

≥ 1√z + x

≥ 1√x+ y

.

124

Page 126: Олимпиадад бэлтгэгчдэд тусламж (Тэнцэтгэл биш 1)

Therefore, according to the Rearrangement inequality, we obtain

ab√(a+ b)(b+ c)

+bc√

(b+ c)(c+ a)+

ca√(c+ a)(a+ b)

=

=ab√a+ b

· 1√b+ c

+bc√b+ c

· 1√c+ a

+ca√c+ a

· 1√a+ b

≤ xy√x+ y

· 1√y + z

+zx√z + x

· 1√z + x

+yz√y + z

· 1√x+ y

=y(x+ z)√

(x+ y)(y + z)+

xz

x+ z.

From now, we can deduce our inequality to

2y(x+ z)√(x+ y)(y + z)

+2xz

x+ z≤ 9

16

∑cyc

x+∑cyc

xy

x+ y,

2y(x+ z)√(x+ y)(y + z)

≤ 9

16(x+ y + z) +

xy

x+ y+

yz

y + z− zx

z + x.

Now, using AM-GM inequality, we have that

2√(x+ y)(y + z)

≤ x+ y + z

2(x+ y)(y + z)+

2

x+ y + z.

and it remains to show that

y(x+ z)(x+ y + z)

2(x+ y)(y + z)+

2y(x+ z)

x+ y + z≤ 9

16(x+y+z)+

xy

x+ y+

yz

y + z− zx

z + x,

y(x+ z)(x+ y + z)

2(x+ y)(y + z)+

2y(x+ z)

x+ y + z≤ 9

16(x+y+z)+

y(xy + yz + 2xz)

(x+ y)(y + z)− zx

z + x,

Since this inequality is homogeneous, we may assume that x + z = 1,put t = xz, then

t− y(1− y) = xz − y(x+ z − y) = (y − x)(y − z) ≤ 0.

Hence 0 ≤ t ≤ y(1− y). The above inequality becomes

y(y + 1)

2(y2 + y + t)+

2y

y + 1≤ 9

16(y + 1) +

y(y + 2t)

y2 + y + t− t,

f(t) =9

16(y + 1)− 2y

y + 1+y(4t+ y − 1)

2(t+ y + y2)− t ≥ 0.

After expanding, we can see that our inequality has the form g(t) =At2+Bt+C ≥ 0, with A < 0. Therefore, in order to prove this inequality,we just need to prove that it holds when t = 0 and t = y(1 − y), or inthe other words, we must prove that f(0) ≥ 0 and f(y(1− y)) ≥ 0. Butit is true because

f(0) =9

16(y + 1)− 2y

y + 1+

y(y − 1)

2(y + y2)=

(1− 3y)2

16(1 + y)≥ 0,

f(y(1− y)) =9

16(y + 1)− 2y

y + 1+y [4y(1− y) + y − 1]

2 [y(1− y) + y + y2]

=5 + 2y + 13y2 − 16y3

16(y + 1)> 0 (since 1 ≥ y ≥ 0).

Thus f(t) ≥ 0 and our proof is complete.

125

Page 127: Олимпиадад бэлтгэгчдэд тусламж (Тэнцэтгэл биш 1)

21. Let a, b, c be positive real numbers such that a+ b+ c = 3. Prove that√a

b2 + 3+

√b

c2 + 3+

√c

a2 + 3≤ 3

2.

Solution: Let (x, y, z) be a cyclic permutation of (a, b, c) such that x ≥y ≥ z, then

√x ≥ √y ≥

√z and

1√z2 + 3

≥ 1√y2 + 3

≥ 1√x2 + 3

.

Therefore, by Rearrangement inequality, we obtain√a

b2 + 3+

√b

c2 + 3+

√c

a2 + 3=

=√a · 1√

b2 + 3+√b · 1√

c2 + 3+√c · 1√

a2 + 3

≤√x · 1√

z2 + 3+√y · 1√

y2 + 3+√z · 1√

x2 + 3

=

√x

z2 + 3+

√z

x2 + 3+

√y

y2 + 3.

And we can deduce our inequality to√x

z2 + 3+

√z

x2 + 3+

√y

y2 + 3≤ 3

2.

Setting t = xz, then 0 ≤ t ≤ (x+z)2

4 = (3−y)24 . By Cauchy-Schwartz

inequality, we have√x

z2 + 3+

√z

x2 + 3≤

√(x+ z)

(1

z2 + 3+

1

x2 + 3

)

=

√(3− y)(x2 + z2 + 6)

x2z2 + 3(x2 + z2) + 9

=

√(3− y)(y2 − 6y + 15− 2t)

t2 − 6t+ 3y2 − 18y + 36.

Consider the function

f(t) =y2 − 6y + 15− 2t

t2 − 6t+ 3y2 − 18y + 36

We have

f ′(t) =2[t2 − (y2 − 6y + 15)t+ 9

](t2 − 6t+ 3y − 18y + 3)2

,

f ′(t) = 0⇒

t = t1 =y2−6y+15+(3−y)

√y2−6y+21

2

t = t2 =y2−6y+15−(3−y)

√y2−6y+21

2

.

126

Page 128: Олимпиадад бэлтгэгчдэд тусламж (Тэнцэтгэл биш 1)

If y ≥ 1 then we have t1 ≥ t2 ≥ (3−y)24 , therefore since 0 ≤ t ≤ (3−y)2

4 ,

we have f ′(t) ≥ 0, and f(t) is increasing for all 0 ≤ t ≤ (3−y)24 . Thus

f(t) ≤ f(

(3− y)2

4

)=

8

y2 − 6y + 21.

Hence√x

z2 + 3+

√z

x2 + 3+

√y

y2 + 3≤

√8(3− y)

y2 − 6y + 21+

√y

y2 + 3≤ 3

2.

Since√8(3− y)

y2 − 6y + 21+

√y

y2 + 3≤

√(2 + 1)

[4(3− y)

y2 − 6y + 21+

y

y2 + 3

]

=3

2

√1− (y2 + 15)(y − 1)2

(y2 + 3)(y2 − 6y + 21)≤ 3

2.

If 1 ≥ y ≥ 0, then we have t1 ≥ (3−y)24 ≥ t2 > 0, therefore we obtain

f(t) ≤ f(t2) =2

(3− y)(√

y2 − 6y + 21 + y − 3) .

Hence√x

z2 + 3+

√z

x2 + 3+

√y

y2 + 3≤√

2√y2 − 6y + 21 + y − 3

+

√y

y2 + 3.

We need to prove√2√

y2 − 6y + 21 + y − 3+

√y

y2 + 3≤ 3

2.

We have4(y2 − 6y + 21)− (9− y)2 = 3(y − 1)2 ≥ 0,

It follows that√y2 − 6y + 21 + y − 3 ≥ 9− y

2+ y − 3 =

y + 3

2.

and it suffices to prove that

2√y + 3

+

√y

y2 + 3≤ 3

2,

which is true because

2√y + 3

+

√y

y2 + 3≤

√(2 + 1)

(2

y + 3+

y

y2 + 3

)

=3

2

√1− (y − 1)2(y + 1)

(y + 3)(y2 + 3)≤ 3

2.

Equality holds if and only if a = b = c = 1.

127

Page 129: Олимпиадад бэлтгэгчдэд тусламж (Тэнцэтгэл биш 1)

1.17 Working backwards

Consider the following hard problem:Non-negative reals x, y, z satisfy x2 + y2 + z2 + xyz = 4. Then xy + yz +zx− xyz ≤ 2.The condition is hard to use, however the conclusion is quite simple. For thisreason let us reverse the problem:Assume that xy + yz + zx− xyz = 2, we shall then prove that

x2 + y2 + z2 + xyz ≥ 4.

This new question implies the original one: if we would find a, b, c with

a2 + b2 + c2 + abc = 4

and ab+bc+ca−abc > 2, we could take a 0 < t < 1 with t2(ab+bc+ca)−t3abc =2, then setting x = ta, y = tb, z = tc we would get

x2 + y2 + z2 + xyz < a2 + b2 + c2 + abc = 4,

contradiction.So, let’s prove the new problem.

Assume that x ≤ y ≤ z. We have z =2− xy

x+ y − xy.

Now if x+ y < xy so1

x+

1

y< 1 we deduce y + z < yz, x+ z < xz so

x(y + z − yz) + y(x+ z − xz) + z(x+ y − xy) ≥ 0

so2(xy + yz + zx− xyz)− xyz ≤ 0 so xyz ≥ 4

and then

x2 + y2 + z2 + xyz ≥ xy + yz + zx− xyz + 2xyz ≥ 10 > 4.

So assume x + y − xy > 0. Set xy = a, x + y − xy = b. Then we must provethat if a+ bz = 2 then (a+ b)2 − 2a+ z2 + az ≥ 4.

By replacing z =2− ab

the relation to prove becomes(b2 − b+ 1

)a2 + 2a(b3 − b2 + b− 2) + (b2 − 2)2.

Regarded as an equation in a, its discriminant is

−b2(b− 1)(b2 − 5b+ 5)

so it can be positive for b < 1 or b ∈

(5−√

5

2,5 +√

5

2

).

If b < 1. Notice that we must have

(x+ y)2 ≥ 4xy or 4a ≤ (a+ b)2 or a2 + 2a(b− 2) + b2 ≥ 0,

128

Page 130: Олимпиадад бэлтгэгчдэд тусламж (Тэнцэтгэл биш 1)

which yields a2 ≥ 2a(2− b)− b2. And we can deduce our inequality to(b2 − b+ 1

) [2a(2− b)− b2

]+ 2a(b3 − b2 + b− 2) + (b2 − 2)2 ≥ 0,

or(1− b)(4 + 4b− b2 − 4ab) ≥ 0.

Moreover, as z ≥ y ≥ z, we have 2z ≥ x + y = a + b, therefore 2 = a + bz ≥

a+(a+ b)b

2, or 0 ≤ a ≤ 2− b. It follows that 4 + 4b− b2− 4ab ≥ 4 + 4b− b2−

4(2− b)b = −4b+ 3b2 + 4 = 2b2 + (b− 2)2 > 0.

If b ∈

(5−√

5

2,5 +√

5

2

), our inequality is trivial because b3− b2 + b−2 > 0.

This completes our proof.

Exercise:

1. If x2 + y2 + z2 = 2 then x+ y + z − xyz ≤ 2.

Solution: If one of x, y, z is negative, for example z < 0, then we have

z − xyz = z(1 − xy) ≤ 0 as from AM-GM 1 − xy ≥ 1 − x2 + y2

2=z2

2.

Therefore x+y+z−xyz = x+y+z(1−xy) ≤ x+y ≤√

2(x2 + y2) ≤ 2from Cauchy Schwartz.

If x, y, z are all non-negative. As above, we prove that if x+y+z−xyz = 2

then x2 + y2 + z2 ≥ 2. z =2− x− y

1− xy. Let a = x + y, b = xy. Then

z =2− a1− b

, and we need to prove that a2 − 2b +(2− a)2

(1− b)2≥ 2 or that

a2(b− 1)2 + (a− 2)2 − 2b(b− 1)2 ≥ 2(b− 1)2, or (a− 2)2 + a2(b− 1)2 −2(1− b)2(1 + b) ≥ 0,

If b ≥ 1 then a2 ≥ 4b ≥ 2(1 + b) and the expression in clearly non-negative.

If b < 1 then the discriminant, looked as an equation in a, is 2b2(b− 1)3,so negative, and in this case the conclusion also follows.

1.18 Mixing variables

In the previous chapter we took a quantity

Q(y1, y2, . . . , yn) = x1y1 + x2y2 + . . .+ xnyn

and increased or decreased it by exchanging some of the yj . In this chapterwe will take a similar quantity as a function of some variables (usually it’ssymmetric in these variables) and change the variables thus increasing or de-creasing the function. We distinguish two sorts of change: one is replacing avariable by a constant or a value that equals other variable (i.e. set xi = xj),and the other by replacing two (or more variables) by a new common value

129

Page 131: Олимпиадад бэлтгэгчдэд тусламж (Тэнцэтгэл биш 1)

(usually some sort of mean) - this way is particularly useful when we havesome conditions on the variables. This method is called the method of mixingvariables and it is used to reduce the number of variables until we can provethe inequality directly.

Let’s illustrate this method. Take Schur’s inequality:

a3 + b3 + c3 + 3abc− a2b− ab2 − a2c− ac2 − b2c− bc2 ≥ 0.

Take now

f(a, b, c) = a3 + b3 + c3 + 3abc− a2b− ab2 − a2c− ac2 − b2c− bc2.

Thenf(a, a, c)− f(a, b, c) = (a− b)(b2 + 2ac− a2 − bc− c2).

Since the inequality is symmetric in a, b, c, we can suppose that c < a < b.Then a− b < 0 and

b2 + 2ac− a2 − bc− c2 = (b2 − a2) + (ac− bc) + (ac− c2)= (b− a)(a+ b− c) + (ac− c2) ≥ 0

so f(a, a, c) < f(a, b, c) and so the inequality is sufficient to prove for the casewhen the two variables are equal. But in this case it reduces to c(a− c)2 ≥ 0,true!

Take now a harder inequality:

If x2 + y2 + z2 + xyz = 4, x, y, z > 0 then x+ y + z ≤ 3.The condition is quite uncomfortable to tackle. On the contrary the conclu-sion sounds simple. We have seen such inequalities in the section on workingbackwardsThat’s why let’s reverse them: prove that if x+ y + z = 3 then

x2 + y2 + z2 + xyz ≥ 4.

This would provide a solution for the original inequality: if we would have

x+y+z > 3 then dividing x, y, z by a constant a > 1 such thatx

a+y

a+z

a= 3

we would have(xa

)2+(ya

)2+(za

)2+x

a

y

a

z

a< x2 + y2 + z2 + xyz = 4

contradiction.So, let’s prove the new inequality.Let f(x, y, z) = x2 + y2 + z2 + xyz.

Since x+ y + z is fixed, consider f

(x+ y

2,x+ y

2, z

)we have

f(x, y, z)− f(x+ y

2,x+ y

2, z

)=

(x− y)2

2− z(x− y)2

4> 0

if we suppose z < 2 which we can do since x+ y + z = 3 and the inequality is

symmetric in x, y, z. So f(x, y, z) ≥ f(x+ y

2,x+ y

2, z). The inequality is just

enough to prove when two of the variables are equal, so y = x, z = 1 − 2x.The condition now becomes (x− 1)2(5− 2x) ≥ 0 and is true.

130

Page 132: Олимпиадад бэлтгэгчдэд тусламж (Тэнцэтгэл биш 1)

Finally, one can not make two variables equal but even make their differencebigger. Consider the inequality

a2b+ b2c+ c2a ≤ 4

27if a+ b+ c = 1, a, b, c ≥ 0.

Let f(a, b, c) = a2b+ b2c+ c2a.Then

f(0, a+ b, c)− f(a, b, c) = (ca2 + 2abc− c2a− a2b)= a(ac+ 2bc− c2 − ab) = a [bc− (c− a)(c− b)] .

So if c is the between b and a then f(0, a + b, c) ≥ f(a, b, c). But we canclearly assume this since the inequality is cyclic. The inequality is so sufficientto prove when one variable is 0. So if a = 0, b = x, c = 1− x it’s equivalent to

x2(1− x) ≤ 4

27which follows from the AM-GM inequality for

x

2,x

2, 1− x.

Exercises

1. If a, b, c > 0 then 2(a2 + b2 + c2) + 3(abc)23 ≥ (a+ b+ c)2.

Solution: Let’s try to mix a, b into their geometric mean, trying to

invariate the more complicated (abc)23 .

The difference 2(a2 + b2 + c2) + 2(abc)23 − (a+ b+ c)2 will then change

by

−2(a− b)2 − (2√ab+ c)2 + (a+ b+ c)2 = −(a− b)2 + 2

(a+ b− 2

√ab)c

= −(a− b)2 +2(a− b)2c

a+ b+ 2√ab≤ 0

when c is the smallest of a, b, c.

So we may assume that a = b. Take a = b = 1, c = x3 ≤ 1. Theinequality becomes 4 + 2x6 + 3x2 − (2 + x3)2 ≥ 0 or x6 − 4x3 + 3x2 ≥ 0or x2(x− 1)2(x2 + 2x+ 3) ≥ 0, which is true.

2. If a, b, c > 0 with abc = 1 then a2 + b2 + c2 +3 ≥ a+ b+ c+ab+ bc+ ca.

Solution: Let f(a, b, c) = a2+b2+c2+3−ab−bc−ca−a−b−c. To preservethe condition abc = 1, let’s mix b, c into their geometric mean. We com-

pute f(a, b, c) − f(a,√bc,√bc)

=(√

b−√c)2 [(√

b+√c)2− 1− a

].

Now if a = min{a, b, c} ≤ 1 then(√

b+√c)2≥ 4√bc ≥ 4 ≥ 1 + a, so f

decreases. So assume b = c =1

x, a = x2.

We then compute f to be

x4 + 3 +1

x2− 2

x− x2 − 2x =

x6 − x4 − 2x3 + 3x2 − 2x+ 1

x2

=(x− 1)2(x4 + 2x3 + 3x2 + 1)

x2≥ 0.

131

Page 133: Олимпиадад бэлтгэгчдэд тусламж (Тэнцэтгэл биш 1)

3. Let a, b, c, d ≥ 0 with a+ b+ c+ d = 1. Show that

abc+ bcd+ cda+ dab ≤ 1

27+

176

27abcd.

Solution: This is a classic example of mixing variables. Since the in-equality is symmetric, we remember the principle saying ”in almost allcases, the extreme values of a symmetric function are achieved whenalmost all variables are equal”. As we have that the sum of a, b, c, d, itis natural to try to mix two numbers into their arithmetic mean.

So, let f(a, b, c, d) = abc+ bcd+ cda+ dab− 1

27− 176

27abcd. Then

f

(a+ b

2,a+ b

2, c, d

)− f(a, b, c, d) =

=

[(a+ b)2

4− ab

](c+ d)− 176

27cd

[(a+ b)2

4− ab

].

Since(a+ b)2

4− ab is non-negative, f

(a+ b

2,a+ b

2, c, d

)≥ f(a, b, c, d)

when c+d ≥ 176

27cd. As cd ≤ (c+ d)2

4, f(

a+ b

2,a+ b

2, c, d) ≥ f(a, b, c, d)

when c+ d ≥ 176

27

(c+ d)2

4or

27

44≥ c+ d. This certainly happens when

a ≥ b ≥ c ≥ d, so we have f(a, b, c, d) ≤ f(m,m, c, d) where m =a+ b

2.

Then m+d ≤ 1

2<

27

44, so we can replace (m, c) with their mean n, to get

f(a, b, c, d) ≤ f(m,m, c, d) ≤ f(m,n, n, d). Analogously, we can replaceany two of the three greater numbers (except d) by their arithmeticmean.

We can now conclude that f(a, b, c, d) ≤ f(a+ b+ c

3,a+ b+ c

3,a+ b+ c

3, d

)by passing to the limit (remember the topic ”limits in inequalities”). So

we must prove that f(x, x, x, 1− 3x) ≤ 0 where1

4≤ x =

a+ b+ c

3≤ 1

3.

This is easy to do, as this equals

3x2(1− 3x) + x3 − 1

27− 176

27x3(1− 3x) =

1

27(528x4 − 392x3 + 81x2 − 1)

=1

27(3x− 1)(4x− 1)2(11x+ 1).

4. Let a, b, c, d ≥ 0. Show that

a4 + b4 + c4 + d4 + 2abcd ≥ a2b2 + b2c2 + c2d2 + d2a2 + b2b2 + a2c2.

Solution: We shall present two solutions by mixing here. The first is themix by product, which invariates the term 2abcd. Indeed, let

f(a, b, c, d) = a4+b4+c4+d4+2abcd−(a2b2+b2c2+c2d2+d2a2+b2b2+a2c2).

132

Page 134: Олимпиадад бэлтгэгчдэд тусламж (Тэнцэтгэл биш 1)

Then

f(a, b, c, d)− f(√

ab,√ab, c, d

)=

= a4 + b4 − 2a2b2 − (c2 + d2)(a2 + b2 − 2ab)

= (a− b)2[(a+ b)2 − c2 − d2

].

This is nonnegative when (a+b)2 ≥ c2+d2 so if we suppose a ≥ b ≥ c ≥ d,then f(a, b, c, d) ≥ f(m,m, c, d) where m =

√ab.

As in the problem above, we can mix any two of the larger three numberinto their geometric product, hence by passing to the limit it suffices toprove the inequality for a = b = c ≥ d. As the inequality is homogeneous,we can set a = b = c = 1, d = x ≤ 1. We have to prove that x4+3+2x ≥3 + 3x2, or that x3 + 2 ≥ 3x, which is obvious by AM-GM.

The second solution is the unusual sort of mix, in which we replace onevariable by another. Define f as above.

Let’s look at f(a, b, c, d)− f(a, a, c, d). It equals

b4 − a4 + 2acd(b− a)− (b2 − a2)(a2 + c2 + d2) =

= (b− a)[(b+ a)(b2 + a2) + 2bcd− (a2 + c2 + d2)(b+ a)

].

So it’s positive when b ≥ a and

(b2 + a2)(b+ a) + 2acd ≥ (a2 + c2 + d2)(a+ b),

or when b ≤ a and (b2 + a2)(b+ a) + 2acd ≤ (a2 + c2 + d2)(a+ b). As

(a2 + b2)(a+ b) + 2acd− (a2 + c2 + d2)(a+ b) =

= b2(a+ b) + 2acd− (c2 + d2)(a+ b)

= (b2 + ab− c2 − d2)b− (c− d)2a,

we can see that this is positive when b > a are the largest among a, b, c, dand is negative when a > b are the smallest among a, b, c, d.

Then, if we assume a ≥ b ≥ c ≥ d, by our conclusion we deduce

f(a, b, c, d) ≥ f(b, b, c, d) ≥ f(b, b, c, c).

Finally, f(b, b, c, c) = (b2 − c2)2 ≥ 0.

1.19 Limits in inequalities

Limits is a subject that apparently has nothing to do with inequalities. Itdoes, as we shall list above three applications for inequalities. All of theseapplications are based on the following crucial fact:

(F): If xi is a sequence converging to a constant a, f, g are continuous functionsand f(xi) ≥ g(xi), then f(a) ≥ g(a).

133

Page 135: Олимпиадад бэлтгэгчдэд тусламж (Тэнцэтгэл биш 1)

The proof of this fact is very simple: if we suppose that g(a) − f(a) = ε > 0then we may choose some n with

|f(xi)− a| <ε

2, |g(xi)− a| <

ε

2

and thus

(f(xi)− g(xi))− (f(a)− g(a)) ≤ |f(xi)− f(a)|+ |g(xi)− g(a)| < ε.

But this is as f(xi) − g(xi) is a nonnegative number, but f(a) − g(a) = −ε.This fact can easily be extended to functions of more variables. An importantnote here is that the inequalities f(xi) ≥ g(xi) can be strict, but it doesn’tfollow from here that the inequality f(a) ≥ g(a) is strict. Look for example

at the inequalities1

n> 0, but lim

n→∞

1

n= 0.

• Sometimes we have to prove an inequality like Young’s one:ap

p+bq

q≥ ab,

1

p+

1

q= 1, p, q > 0. We can prove them for p =

m+ n

n, q =

m+ n

nbeing

rationals, as multiplying to m + n this inequality is a direct consequence ofAM-GM. Now, as every real can be approximated by rationals, we can find

rationals pi, qi with1

pi+

1

qi= 1, |pi − p| <

1

i, |qi − q| <

1

i. As pi tend to p,

qi tend to q, from (F) we deduce that the inequality is true for p, q.

• Sometimes we have to prove an inequalityf(x1, x2, . . . , xn) ≥

≥ f(x1 + x2 + . . .+ xn

n,x1 + x2 + . . .+ xn

n, . . . ,

x1 + x2 + . . .+ xnn

)and we can prove that

f(x1, x2, . . . , xn) ≥ f(x1, x2, . . . , xi−1,

xi + xj2

, xi+1, . . . , xj−1,xi + xj

2, xj+1, . . . , xn

)for any 1 ≤ i < j ≤ n. By such operations we can actually make our sequenceof numbers tend to

a1 + a2 + . . .+ ann

, . . . ,a1 + a2 + . . .+ an

n.

Indeed, consider the invariant

E(x1, x2, . . . , xn) =n−1∑i=1

∑j=i+1n

(xi − xj)2.

Using the inequality

(x− a)2 + (x− b)2 ≥ 2

(x− a+ b

2

)2

,

we can prove that replacing xi, xj byxi + xj

2, we decrease E by at least

(xi − xj)2. Thus, taking |xi − xj | be the greatest of all possible differences

134

Page 136: Олимпиадад бэлтгэгчдэд тусламж (Тэнцэтгэл биш 1)

between two numbers, we can decrease E to at most

[1− 2

n(n− 1)

]E. So

we can make E as close to zero as possible, particularly making any of thepossible differences between two number as small as possible. Since our trans-formations preserve the sum, it’s now clear that al the numbers get as close

toa1 + a2 + . . .+ an

nas desired. Since f is continuous, we now deduce the

claim.Like this, for example, we can prove AM-GM:

a1 + a2 + . . .+ an ≥ n√a1a2 . . . an,

and even in two ways. The first way is almost identical to what we’ve told

above. However, we can do it another way: take f(x1, x2, . . . , xn) =x1x2 . . . xnn√x1x2 . . . xn

.

Then

f(x1, x2, . . . , xn) ≥ f(x1, . . . , xi−1,√xixj , xi+1, . . . , xj−1,

√xixj , xj+1, . . . , xn)

(the product of the number is the same, but the sum decreases from AM-GMfor two numbers). Next, we can proceed analogously as above (for example,work with lnxi instead of xi).

• Limits are also applicable with integral inequalities, as the integral is in factitself a limit. Like this, many of the classic inequalities translate into integralones.For example, CBS (Cauchy-Schwartz). Inequality rewrites as(∫ b

af2(x)dx

)(∫ b

ag2(x)dx

)≥(∫ b

af(x)g(x)dx

)2

.

Indeed, as

∫ b

ah(x)dx = lim

n→∞

f(x1) + f(x2) + . . .+ f(xn)

n, where xi =

(n− i)a+ ib

n,

(∫ b

af2(x)dx

)(∫ b

ag2(x)dx

)−(∫ b

af(x)g(x)dx

)2

is the limit of

f2(x1) + . . .+ f2(xn)

n

g2(x1) + . . .+ g2(xn)

n−(f(x1)g(x1) + . . .+ f(xn)g(xn)

n

)2

,

which is positive from CBS. Using (F), we deduce our claim.

1.20 Derivatives in Inequalities

When we have to prove a inequality of the form f(a) ≥ f(b), a > b, we canprove that f is increasing on [b; a]. But to prove that f is increasing on [b, a]it suffices to prove that f ′(x) ≥ 0.Consider for example the already discussed inequality

an1 + an2 + . . .+ ann ≥ na1a2 . . . an, an > 0 (AM-GM).

135

Page 137: Олимпиадад бэлтгэгчдэд тусламж (Тэнцэтгэл биш 1)

We can suppose that a1 ≥ a2 ≥ . . . ≥ an. Now let

f(a1, a2, . . . , an) = an1 + an2 + . . .+ ann − na1a2 . . . an.

Regarded as a function in a1, it is increasing, as

f ′(a1) = nan−11 − (n− 1)a2a3 . . . an ≥ 0.

So

f(a1, a2, . . . , an) ≥ f(a2, a2, a3, . . . , an).

Now we can regard it as a function in a2. It’s derivative is again positive, asit is 2an−12 − 2a2a3 . . . an. Now, the method of proof is clear: by induction onk we prove that

f(a1, a2, . . . , an) ≥ f(ak, ak, . . . , ak, ak+1, ak+2, . . . , an).

Indeed, the basis is proven, and for the induction step we note that

f(ak, ak, . . . , ak, ak+1, ak+2, . . . , an) ≥ f(ak+1, ak+1, . . . , ak+1, ak+1, ak+2, . . . , an)

because the function

g(x) = g(x, x, . . . , x, ak+1, ak+2, . . . , an)

is increasing as its derivative is

nkxn−1 − nkxk−1ak+1ak+2 . . . an ≥ 0, g(ak) ≥ g(ak+1).

Finally, because f(an, an, . . . , an) ≥ 0, we deduce our claim.

Another use of derivatives in inequalities comes from the combination of Weier-strass Theorem with Fermat’s Theorem.

Recall that Weierstrass’ theorem states that every continuous function on acompact set attains its extremes, and Fermat’s Theorem says that if x ∈ (a, b),is an extremal point of a continuous derivable function f then f ′(x) = 0.As proving an inequality f(x1, x2, . . . , xn) ≥ 0 is equivalent to min{f} ≥0, it suffices to consider x1, . . . , xn that realize this minimum (Weierstrass’stheorem ensures that it is achieved). Using Fermat’s theorem, we obtaininformation about these points, and often we can explicitly find then andverify the inequality directly.

Let’s solve the already familiar inequality

x2y + y2z + z2x ≤ 4

27, x+ y + z = 1

If we suppose x ≥ y ≥ z and substitute z = 1− x− y then we must prove

f(x, y) = x2y + y2(1− x− y) + (1− x− y)2x ≤ 4

27for x, y

in the compact set bounded by the conditions x ≥ y ≥ 0, y ≥ 1 − x − y andx+ y ≤ 1.

136

Page 138: Олимпиадад бэлтгэгчдэд тусламж (Тэнцэтгэл биш 1)

Now let (x0, y0) maximize f (this is true by Weierstrass Theorem). Therefore,from Fermat’s Theorem, either some of these conditions is an equality, eitherf ′(x) = f ′(y) = 0. In the first case, we either have one of the variables (x, y, z)zero, or two variables equal. If one is zero, we have discussed this case before, ifx = y then z = 1−2x and the inequality becomes x3+x2(1−x)+(1−2x)2x ≤4

27or f(x) = 4x3−2x2+x ≤ 4

27, for x ∈

[0,

1

2

]. As f ′(x) = 12x2−4x+1 ≥ 0,

we must prove this for x =1

2, which is true. Finally, let’s get to the cream of

the solution: the last case, when f ′x(x, y) = f ′y(x, y) = 0.

First, write out f explicitly:

x3 − y3 + 3x2y − 2x2 − 2xy + y2 + x

.

Then

f ′x(x, y) = (3x− 1) (x+ 2y − 1)

f ′y(x, y) = (3x+ 3y − 2)(x− y)

So, f ′x(x, y) = 0 when x =1

3or x+2y = 1 and f ′y(x, y) = 0 when 3x+3y−2 = 0

or x = y. If x = y, then since f ′x(x, y) = 0 we must have x = y =1

3which

gives us x = y = z =1

3and if 3x + 3y = 2 = 2(x + y + z), we will have that

x + y = 2z, which also gives us x = y = z since x ≥ y ≥ z. Therefore, in

this case, we can see that extreme attains when x = y = z =1

3but with this

value, it is trivial that f(x, y) <4

27. This ends our proof.

Exercises

1. Show that for a, b ≥ 0 we have 3a3 + 7b3 ≥ 9ab2.

Solution: If b = 0 it’s trivial so assume b > 0. By putting b = 1, a = x,we need to prove f(x) = 3x3 + 7 − 9x ≥ 0. As f ′(x) = 9(x2 − 1), itsuffices to look at the case x = 1, and in this case we get a positive value.

2. Let a, b, c > 0. Then 4

√a+ 3

√b+√c ≥ 40

√abc.

Solution: Put it as(a+ 3

√b+√c)10≥ abc. Let f(x) =

(x+ 3

√b+√c)10−

bcx. As f ′′(x) ≥ 0 we conclude f ′(x) ≥ f ′(0). Now we prove f ′(0) ≥ 0.

As f ′(x) = 10(x+ 3

√b+√c)9−bc, we have to prove that 10 (b+

√c)

3 ≥bc.

Now take g(x) = 10(x+√c)3−cx, then g′′(x) ≥ 0 and so g′(x) ≥ g′(0) =

29c ≥ 0 so g′ is positive and g is increasing so g(b) ≥ g(0) ≥ 0. Hencef ′(0) ≥ 0 so f ′ is increasing and positive and then f(a) ≥ f(0) > 0, asdesired.

137

Page 139: Олимпиадад бэлтгэгчдэд тусламж (Тэнцэтгэл биш 1)

3. Show that1

n+ 1+ . . .+

1

3n< ln 3.

Solution: We prove that ln(x+ 1)− lnx ≥ 1

x+ 1, for x ≥ 1. Indeed, by

Lagrange Theorem there is y ∈ [x, x + 1] with ln(x + 1) − ln(x) =1

y≥

1

x+ 1.

Thus

1

n+ 1+ . . .+

1

3n< ln(n+ 1)− ln(n) + . . .+ ln(3n)− ln(3n− 1)

= ln(3n)− ln(n) = ln 3.

4. Let a, b, c > 0 with a+ b+ c = 6. Find the minimum value of

P =

(a+

1

a

)(b+

1

b

)(c+

1

c

).

Solution: The expressions P is quite complicated to link with the relationa+b+c = 6, so we must to try to simplify P , looking at just two variables.

Thus, let’s compute the minimal value of

(a+

1

a

)(b+

1

b

)when a+b =

t is constant. This is a function in a:

(a+

1

a

)(t− a+

1

t− a

). When

a is close to the extreme values 0 and t, the expression tends to infinity,so the minimum is attained somewhere in the interior of the interval,thus at a = x where the derivative vanishes. The derivative is, as easilyto compute,

g(x) = t− 2x+1

t− x− 1

x− 1

x2

(t− x+

1

t− x

)+

1

(t− x)2

(x+

1

x

).

By symmetry, we see that when a = b i.e. 2x = t the derivative vanishes,so let’s extract t− 2x out of the expression:

g(x) = (t− 2x) +2x− tx(t− x)

+1

x2

(x+

1

x+ x− t− 1

t− x

)+

+

(x+

1

x

)[1

(t− x)2− 1

x2

]= (t− 2x) +

2x− tx(t− x)

+(t− 2x)(x2 − tx+ 1)

x3(t− x)− t(x2 + 1)(t− 2x)

x3(t− x)2

=(t− 2x)

[x2(x− t)2 − t2 − 1

]x2 (t− x)2

.

So, the derivative vanishes when x =t

2or when x2(t − x)2 = t2 + 1.

Thus we have established that for fixed a + b,

(a+

1

a

)(b+

1

b

)takes

its minimal value when a = b or a2b2 = (a + b)2 + 1. It’s quite hard to

138

Page 140: Олимпиадад бэлтгэгчдэд тусламж (Тэнцэтгэл биш 1)

compute a that satisfies the second relation and compare this value for

the value int

2. So we can’t actually compute the minimal value for all

t.

However, AM-GM helps us: we know that x(t−x) ≤ t2

4, so x2(t−x)2 ≤

t4

16. So when t2+1 >

t4

16, the minimum value occurs exactly when a = b.

We note that all t < 4 satisfy t2 + 1 >t4

16because t2 ≥ t4

16. Since

a+ b+ c = 6, some two of a+ b+ c sum to at most 4, that’s why we cansuppose they are equal.

Indeed, let a ≤ b ≤ c realize the minimum of P , then as a + b ≤ 4 wemust have a = b otherwise we could change a, b to keep a + b (and c)

fixed but decrease

(a+

1

a

)(b+

1

b

). Set a = b = x. If c = x then

x = 2. If not, by the same reasoning we deduce that a2c2 = (a+ c)2 + 1so x2(6 − 2x)2 = (6 − x)2 + 1, or 4x4 − 24x3 + 5x2 + 12x = 37. This

cannot actually happen for x ≤ 2, because if x <3

2then 4x4 < 24x3 and

5x2 + 12x < 5 · 1.52 + 12 · 1.5 = 29.25 < 37,

and if3

2≤ x ≤ 2 then 4x4 − 24x3 ≤ 8x3 − 24x3 = −16x3, and clearly

−16x3 + 5x2 + 12x < 0 < 37.

So, x = 2 and the minimum value realizes for a = b = c = 2 and equals125

8.

1.21 Convexity

Convexity, despite being slightly more complicated to define than monotonicityor continuity, is a very important property of a function. Here is one of themost used definitions for convexity:

Definition: A function defined on a interval I is called convex if for any 0 < α,β < 1, α+ β = 1, x, y ∈ I

f(αx+ βy) ≤ αf(x) + βf(y).

If the inequality is strict or α, β ∈ {0; 1}, the function is called strictly convex.(Note that when one of α, β is negative the inequality changes sign).When −f is convex, f is called concave.

Convexity is much more used with continuous and twice derivable functions,and here’s why:

• For a continuous twice derivable function f , f is convex if f ′′(x) ≥ 0. Iff ′′(x) > 0 for all x, the function is called strictly convex.

139

Page 141: Олимпиадад бэлтгэгчдэд тусламж (Тэнцэтгэл биш 1)

The proof of this fact goes as follows: clearly f ′(x) is increasing hence∫ a+t

af ′(x)dx ≤

∫ a+p+kt

a+pf ′(x)dx,

if a < b, p, t > 0. This means

k(f(a+ t)− f(a)) ≤ f(a+ p+ kt)− f(a+ p).

Now if we let p = t we get

kf(a+ t)− kf(a) ≥ f(a+ (k + 1)t)− f(a+ t)

or

f(a+ t) ≤ kf(a) + f(a+ kt)

k + 1,

and this is exactly the definition of convexity.

• A continuous function f is convex if

f(x) + f(y) ≥ 2f

(x+ y

2

). One implication is obvious. For the other, we firstly prove by induction that

for anyk

2n∈ [0, 1], k, n ∈ N, we have

k

2nf(x) +

2n − k2n

f(y) ≥ f(kx+ (2n − k)y

2n

).

The basis of the induction is k = 1 and follows form the condition. Now, if wehave proven the statement for n− 1, let’s prove it for n.

If k is even, thenk

2n=

k

22n−1

and we apply the induction hypothesis.

If k is odd, then we k − 1, k + 1 are even and so

k − 1

2nf(x) +

2n − k + 1

2nf(y) ≥ f

(k − 1

2nx+

2n − k + 1

2ny

)and

k + 1

2nf(x) +

2n − k − 1

2nf(y) ≥ f

(k + 1

2nx+

2n − k − 1

2ny

).

Summing these two identities and using the fact that

f

(k − 1

2nx+

2n − k + 1

2ny

)+f

(k + 1

2nx+

2n − k − 1

2ny

)≥ 2f

(k

2nx+

2n − k2n

y

),

we get the induction step done.

Therefore, we have proven

αf(x) + βf(y) ≥ f(αx+ βy), α+ β = 1

140

Page 142: Олимпиадад бэлтгэгчдэд тусламж (Тэнцэтгэл биш 1)

for α of the formk

2n. Since every real can be represented as the limit of

numbers of formk

2n, using the continuity of the function we deduce the formula

(see ”Limits in Inequalities”).

A useful reformulation of the definition goes as follows: if z is between x andy, then

f(z) ≤ z − yx− y

f(x) +z − xy − x

f(y)

(you may check thatz − yx− y

+z − xy − x

= 1 and that xz − yx− y

+ yz − xy − z

= z).

In this setting one can define convexity for functions that are not necessarilydefined on an interval (even though in practice we don’t really deal with suchfunctions).

A very important property of the convex function is that they realize themaximal value on a interval at the extremities of the interval (and henceconcave functions realize their minimal value at the extremities of the interval).This follows directly from the definition and this is sometimes used in problemslike this one:

Let a, b, c ∈ (0, 1). Then

a

b+ c+ 1+

b

c+ a+ 1+

c

a+ b+ 1+ (1− a)(1− b)(1− c) ≤ 1.

We may consider this as a function in

a : f(x) =x

b+ c+ 1+

c

x+ b+ 1+

b

x+ c+ 1+ (1− x)(1− b)(1− c).

We can easily see by taking the second derivative that it is convex, so takes itsmaximal values at 0 or 1. Analogously for b and c we conclude it’s enough toinvestigate the cases when all a, b, c are either 0 or 1, which are easy to handle.

But the most important property of convex functions is Jensen’s inequality,which is presented next.

1.22 Jensen’s Inequality

The notion of convexity gets most of its importance in lights of the famousJensen’s Inequality.

Suppose that f is continuous. We have seen from the definition of the inequal-ity that if z is a number between x and y then f(z) is smaller from that alinear combination of f(x) and f(y), and moreover this combination appliedto x and y gives z.Let’s look at this situation from a geometrical point of view:Draw the graph of f(x). Consider the pointsX(x, f(x)), Y (y, f(y)) and Z(z, f(z)).If z is between x and y, then the condition of convexity tells us that

f(z) ≤ z − yx− y

f(x) +z − xy − x

f(y)

141

Page 143: Олимпиадад бэлтгэгчдэд тусламж (Тэнцэтгэл биш 1)

which actually means that Z is below the line segment XY .From here we can deduce that every point in (XY ) is above the graph off . This means that the area above the graph of f is a convex subset (hencewhy this definition). It’s clear that the reverse implication holds: if the areaabove the graph is convex, then function if convex (the statements ”f(z) ≤z − yx− y

f(x) +z − xy − x

f(y)” and ”Z is below XY ” are equivalent). A good geo-

metric image of convex functions is therefore a kind of ”trough” representingthe graph of the function, like a parabola with positive leading coefficient.Now, take n pointsA1(x1, f(x1)), A2(x2, f(x2)), . . . , An(xn, f(xn)) on the graphof f , and assign to them positive weights λ1, . . . , λn with λ1+λ2+. . .+λn = 1.The point G = (λ1x1 + . . . + λnxn, λ1f(x1) + . . . + λnxn) is then the centerof gravity of the system of points. But it’s known that the center of grav-ity of some points (with positive weights) lies inside the convex hull of thesepoints. Indeed, if it wasn’t true, there would be a line l such that it separatesthe points A1, . . . , An and G. But then taking a system of coordinates withl being the ordinate, we would see that G has positive abscise but Ai havenegative abscises. This contradicts the obvious condition that the abscise ofG is a linear combination with positive coefficients λ1, . . . , λn of abscises ofA1, A2, . . . , An and thus must be negative. Therefore G must lie inside theconvex hull of A1, . . . , An. As the area above the graph of f is convex, G mustlie above the graph of f . This means that

λ1f(x1) + . . .+ λnf(xn) ≥ f(λ1x1 + . . .+ λnxn).

Thus, we have proved the following inequality:Theorem(Jensen’s Inequality) If f is a convex function on a interval I andx1, x2, . . . , xn ∈ I, then for any λ1, . . . , λn ≥ 0 with sum 1 we have

λ1f(x1) + . . .+ λnf(xn) ≥ f(λ1x1 + . . .+ λnxn).

Jensen’s inequality has a lot of applications in inequalities. It clearly getsreversed when f is concave.

For example, the function ex is convex, as its second derivative is ex > 0.

Taking now λi =1

nand applying Jensen we deduce

n∑i=1

exi

n≥ e

∑ni=1 xin .

Now taking ai = exi , we obtain exactly AM-GM inequality. Alternatively, onecan prove the AM-GM inequality using the concave function lnx.Many functions are either convex or concave. Here is a list of most common-used concave and convex functions:

− ax is convex for a > 1 and concave for 0 < a < 1.− lnx is concave. So is loga x when a > 1. When a < 1, loga x is convex.− xt is convex when t ∈ (−∞, 0)

⋃(1,∞) and concave when t ∈ (0, 1)

− sinx is convex on (π, 2π) and concave on (0, π).

142

Page 144: Олимпиадад бэлтгэгчдэд тусламж (Тэнцэтгэл биш 1)

− cosx is convex on

2,3π

2

)and concave on

(−π

2,π

2

).

− tanx or cotx is convex on(π

2, π)⋃(3π

2, 2π

)and concave on

(0,π

2

)⋃(π,

2

).

− |x− a| is convex, although not differentiable at a.

Exercises

1. In a triangle show that cosA+ cosB + cosC ≤ 3

2.

Solution: Suppose that A ≤ B ≤ C, then 0 < A ≤ B < π2 . As the func-

tion cosx is concave on[0,π

2

], we have cosA+ cosB ≤ 2 cos

A+B

2=

2 sinC

2.It suffices to prove that 2 sin

C

2+ cosC ≤ 3

2, which can be trans-

formed into −1

2

(2 sin

C

2− 1

)2

≤ 0, true!

2. If a, b, c ∈ (0, 1) then√a− a2 +

√2b− b2 +

√3c− c2 ≤

√6(a+ b+ c)− (a+ b+ c)2.

Solution: Set a = x, b = 2z, c = 3z. The inequality becomes

√x− x2 + 2

√y − y2 + 3

√z − z2 ≤ 6

√x+ 2y + z

6−(x+ 2y + 3z

6

)2

.

It follows from Jensen’s inequality applied to the concave function√x− x2.

3. If a, b, x, y, z > 0 and x+ y + z = 1 then(a+

b

x

)(a+

b

y

)(a+

b

z

)≥ (a+ 3b)3.

Solution: The inequality follows from Jensen applied to the convex func-

tion ln

(a+

b

x

).

4. Show that for a convex function f , we have

f(x) + f(y) + f(z) + f

(2x+ y

3

)+ f

(2y + z

3

)+ f

(2z + x

3

)≥

≥ 2

(f

(x+ 2y

3

)+ f

(y + 2z

3

)+ f

(z + 2x

3

)).

Solution: We have f(x) + f

(2z + x

3

)≥ 2f

(z + 2x

3

)by Jensen. By

summing with the analogously deduced relations we get the result.

143

Page 145: Олимпиадад бэлтгэгчдэд тусламж (Тэнцэтгэл биш 1)

5. Let a0, a1, . . . , an ∈(π

4,π

2

)with

n∑i=0

tan ai −π

4. Show that

tan a0 tan a1 . . . tan an ≥ nn+1.

Solution: Set bi = tan ai −π

4, then 0 < bi < 1 and b0 + . . .+ bn ≥

n− 1

n+ 1.

We have to show thatn∏i=0

1 + bi1− bi

≥ nn+1. This is straightforward by

Jensen, because the increasing function f(x) = ln

(1 + x

1− x

)is convex (it

has increasing derivative2

1− x2).

We can also show this by using just AM-GM: if we set xi = 1− bi thenas

n∑i=0

bi ≥ n− 1,

we get 1 + bk ≥n∑

i=0,i 6=kbi. Hence by AM-GM 1 + bi ≥ n

√√√√ n∏i=0,i 6=k

xi. Mul-

tiplying this by all i and dividing by x0x1 . . . xn, we get the conclusion.

1.23 The shrinking principle and Karamata’s In-equality

Take a convex function f and two numbers x, y. For any α, β ∈ [0, 1] withα+ β = 1 we have

αf(x) + βf(y) ≥ f(αx+ βy).

Also

βf(x) + αf(y) ≥ f(βx+ αy).

Summing we get

f(x) + f(y) ≥ f(αx+ βy) + f(βx+ αy).

Noticing the fact that (αx+ βy) + (βx+αy) = x+ y, we can reformulate thisresult as follows:

If z, t are between x, y and z + t = x+ y then

f(x) + f(y) ≥ f(z) + f(t),

for any convex f .

We see that z, t are actually obtained by taking x, y and ”pulling” them closerone to another, like ”shrinking” the interval (x; y). That’s why we can callthis result ”the shrinking principle”.

144

Page 146: Олимпиадад бэлтгэгчдэд тусламж (Тэнцэтгэл биш 1)

If we take now a sum f(x1)+ . . .+f(xn) and ”shrink” some pair of them xi, xjto a pair x′i, x

′j between xi and xj we decrease the sum f(x1)+ . . .+f(xn). We

can then perform more shrinkings. Thus, if y1, y2, . . . , yn are obtained fromx1, x2, . . . , xn by some shrinkings, we have

f(x1) + f(x2) + . . .+ f(xn) ≥ f(y1) + f(y2) + . . .+ f(yn).

It is now natural to ask how to characterize all the sequences y1, . . . , yn thatare obtained from x1, x2, . . . , xn by several applications of shrinking. A firstobservation is that the sum is constant. Another observation is, as y1, . . . , ynare ”closer” than x1, . . . , xn, the greatest of xi is bigger than the greatest ofyi, and the least of xi is smaller than the least of yi. We now come to thefollowing definition:

Definition. A sequence x1 ≥ x2 ≥ . . . ≥ xn majorizes another sequencey1 ≥ y2 ≥ . . . ≥ yn if and only if the following conditions hold:

(i) x1 + x2 + . . .+ xn = y1 + y2 + . . .+ yn(ii) x1 + x2 + . . .+ xm ≥ y1 + y2 + . . .+ ym for any 1 ≤ m ≤ n.

We claim that a sequence y1 ≥ y2 ≥ . . . ≥ yn can be obtained from

x1 ≥ x2 ≥ . . . ≥ xn

by shrinking if and only if it is majorized by x1, · · · , xn.

Let’s prove the first implication: that any sequence obtained by is majorizedby the original sequence. Since the relation of majorization is transitive, it’senough to prove that shrinking once yields a sequence majorized by the originalone.

Indeed, suppose we replace xi > xj by xi > yi > yj > xj , xi + xj = yi + yj .

Then the condition (i) clearly holds, so let’s prove (ii):

x1 + x2 + . . . + xm ≥ S, where S is the sum of m largest numbers from thenew sequence. If m < i then actually x1 + . . . + xm = S. If j > m ≥ i butthe number yj is not between the m th largest numbers of the new sequence,a strict inequality holds. Finally if m < j again equality holds.

Now let’s prove the second implication: if (yi) is majorized by (xi), it can beobtained from (xi) by some consecutive applications of shrinking.

Pick up the least k s.t. xk 6= yk and the greatest l s.t. xl 6= yl. Clearlyxk > yk, xl < yl. If xk+xl ≥ yk+yl, then we can shrink xk, xl to yk, xk+xl−ykotherwise we shrink xk, xl to xk + xl − yl, yl. It’s easy to prove that the newsequence is majorized by (xi) and majorizes (yi), but it has more commonterms with (yi). Hence, performing some such operations we can get al termsof the sequence equal to the terms of (yi), as we desired.

An immediate corollary of this reasoning is the Karamata’s inequality:

Theorem (Karamata’s Inequality) If (xi) majorizes (yi) and f is a convexfunction then

n∑i=1

f(xi) ≥n∑i=1

f(yi).

Another inequality, proved just like Karamata with the help of the shrinkingprinciple, is Muirhead’s inequality:

145

Page 147: Олимпиадад бэлтгэгчдэд тусламж (Тэнцэтгэл биш 1)

If (pi) majorizes (qi) and f is a convex function, then∑σ∈Sn

f

(n∑i=1

pixσ(i)

)≥∑σ∈Sn

f

(n∑i=1

qixσ(i)

).

The proof is left to the reader as an exercise. The suggestion is that theshrinkages that turn (pi) into (qi) induce shrinkages turn that turn (

∑pixσ(i))

into (∑qixσ(i)).

A particular case of this inequality, obtained for the function ex, is very com-monly used in solving symmetric homogeneous olympiad-style inequalities,and has the following form:

If x1, . . . , xn are positive numbers and (pi) majorizes (qi), then∑σ∈Sn

∏xpiσ(i) ≥

∑σ∈Sn

∏xqiσ(i).

For example, taking the sequence (4, 1, 0) that majorizes (3, 1, 1), we get that

x4y + xy4 + x4z + z4x+ y4z + z4y ≥ 2(x3yz + y3xz + z3xy).

One of Karamata’s applications is Popoviciu’s Inequality:

f(x)+f(y)+f(z)+3f

(x+ y + z

3

)≥ 2

(f

(x+ y

2

)+ f

(y + z

2

)+ f

(z + x

2

)).

Exercises

1. Assume that f is a convex increasing positive-valued function, and

x1 ≥ x2 ≥ . . . ≥ xn, y1 ≥ y2 . . . ≥ yn

satisfy x1 ≥ y1, x1+x2 ≥ y1+y2, . . . , x1+x2+. . .+xn ≥ y1+y2+. . .+yn.Then show that f(x1)+f(x2)+ . . .+f(xn) ≥ f(y1)+f(y2)+ . . .+f(yn).

Solution: We use induction on n. Let

ci = x1 + x2 + . . .+ xi − y1 − y2 − . . .− yi.

If one of ci is zero, then the inequality follows from Karamata applied to(x1, x2, . . . , xi) and (y1, y2, . . . , yi) and induction hypothesis applied to

(xi+1, . . . , xn) and (yi+1, . . . , yn).

If all of the ci are greater than zero, let ck be the smallest.

We can see that

f(y1)+. . .+f(yn) ≤ f(y1 +

ckk

)+. . .+f

(yk +

ckk

)+f (yk+1)+. . .+f(yn),

and(y1 +

ckn, . . . , yk +

ckn, yk+1, . . . , yn

)still satisfies the condition with

respect to (x1, x2, . . . , xn). Now the inequality follows from Karamata for

(x1, x2, . . . , xk) and(y1 +

ckn, . . . , yk +

ckn

)and induction hypothesis for

(xk+1, . . . , xn) and(yk+1, . . . , yn).

146

Page 148: Олимпиадад бэлтгэгчдэд тусламж (Тэнцэтгэл биш 1)

2. Show that 2a2

+ 2b2

+ 2c2 ≥ 2ab + 2ac + 2bc.

Solution: It follows from the previous exercise applied to the increasingconvex function 2x. Another solution is possible by applying Jensen tothe function ee

x.

3. Show that for a, b, c > 0 we have

1

a+

1

b+

1

c+

9

a+ b+ c≥ 4

(1

a+ b+

1

b+ c+

1

c+ a

).

Solution: This is just Popoviciu’s Inequality applied to the convex func-

tion1

x. Compare this solution to the one we given in the SOS section.

4. Prove that for any positive real numbers a1, . . . , an we have the inequal-ity:

n∑i=1

a21a22 + . . .+ a2n

≥n∑i=1

a1a2 + . . .+ an

.

Solution:

Lemma Consider positive reals a1 ≥ . . . ≥ an and b1 ≥ . . . ≥ bn with

a1 + . . . + an = b1 + . . . + bn andaiaj≥ bibj

for i < j. Then a1, . . . , an

majorizes b1, . . . , bn.

Proof : The proof is by induction. We see thataia1≤ bib1

for i = 1, . . . , n.

Summing we getS

a1≤ S

b1hence a1 ≥ b1 (S = a1+. . .+an = b1+. . .+bn)

Hence a1 ≥ b1. Further, we look at the new sequences a2, a3, . . . , anand b2, . . . , bn. By induction hypothesis, we get a2, a3, . . . , an majorizesb2, . . . , bn. Combined with a1 ≥ b1 the lemma is proven.

Now let xi =a2i

a21 + · · · a2n, yi =

aia1 + · · ·+ an

. We see that

a2ia21 + . . .+ a2i−1 + a2i+1 + . . .+ a2n

=xi

1− xi

andai

a1 + . . .+ ai−1 + ai+1 + . . .+ an=

yi1− yi

.

If we suppose a1 ≥ a2 ≥ . . . ≥ an then x1 ≥ . . . ≥ xn and y1 ≥ y2 ≥

. . . ≥ yn. Furtherxixj

=a2ia2j≥ ai

aj=

yiyj

for i < j. Hence by Lemma

x1, . . . , xn majorizes y1, . . . , yn and by noticing that f(x) =x

1− xis

convex, we apply Karamata QED.

147

Page 149: Олимпиадад бэлтгэгчдэд тусламж (Тэнцэтгэл биш 1)

1.24 Schur’s Inequality

A lot of contest inequalities, after homogenization, clearing denominators andother algebraic manipulations, reduce to symmetric homogeneous inequalitiesin some variables. One useful tool in handling these inequalities is, as we haveseen above, Muirhead’s inequality. Another tool, also very helpful, is Schur’sinequality:

a3 + b3 + c3 + 3abc− a2b− b2c− c2a− ab2 − bc2 − ca2 ≥ 0.

It can be rewritten as

a(a− b)(a− c) + b(b− a)(b− c) + c(c− a)(c− b) ≥ 0.

Now, a generalization follows:If f is a convex or monotonic positive valued function, then for any a, b, c fromthe value domain the following inequality holds:

f(a)(a− b)(a− c) + f(b)(b− c)(b− a) + f(c)(c− a)(c− b) ≥ 0.

Proof. Let’s suppose that a ≥ b ≥ c. Then the inequality to prove is

f(a)(a− b)(a− c) + f(c)(a− c)(b− c) ≥ f(b)(a− b)(b− c).

If f is increasing, then

f(a) ≥ f(b), a− c ≥ b− c

sof(a)(a− b)(a− c) ≥ f(b)(a− b)(b− c).

As f(c)(a− c)(b− c) ≥ 0, the inequality follows.If f is decreasing, analogously we break the inequality into

f(c)(a− c)(b− c) ≥ f(b)(a− b)(b− c)

and f(a)(a− b)(a− c) ≥ 0. Finally, when f is convex then Jensen’s inequalityfor b between a and c gives

b− ca− c

f(a) +a− ba− c

f(c) ≥ f(b),

which multiplied by (a− b)(b− c) gives

f(a)(b− c)2(a− b)

a− c+ f(c)

(a− b)2(b− c)a− c

≥ f(b)(b− a)(b− c).

To conclude the proof, it suffices to notice that

(b− c)2(a− b)a− c

≤ (a− c)(a− b)

and(a− b)2(b− c)

a− c≤ (a− c)(b− c).

This inequality is particularly used for function f of the form at, for examplefor f(x) = x2 we get a4+b4+c4+abc(a+b+c) ≥ a3(b+c)+b3(a+c)+c3(a+b).In this form, the inequality has already been encountered and used severaltimes in the book, mostly for t = 1 but also for some other values.

148

Page 150: Олимпиадад бэлтгэгчдэд тусламж (Тэнцэтгэл биш 1)

1.25 The generalized Means

We have met the Arithmetic Mean, the Quadratic Mean and the HarmonicMean. All these means have the form

M(t) =

n∑i=1

ati

n

1t

Moreover, the inequalities between them for the case of two variables are

M(−1) ≤M(1) ≤M(2).

So, we might propose that M(x) is an increasing function in x for any fixedsystem of numbers a1, a2, . . . , an.The aim of this paragraph is to prove this inequality. The function is definedfor all x 6= 0, so to prove our assertion we need to prove that M ′(x) > 0 forx 6= 0 and to prove that M(x) ≥ M(y) for x > 0 > y. The last inequalityfollows from AM-GM, as AM-GM for ati gives M(t) ≥ G for t > 0 and M(t) ≤G for t < 0, where G is the geometric mean of ai.

Now, let’s compute M ′(x). Note F =

n∑i=1

ati

n, for simplicity. Then

M ′(x) =

(elnFx

)′= M(x)

F ′x− F lnF

x2F.

So, we must prove thatF ′x− F lnF ≥ 0.

It rewrites as

1

n

n∑i=1

axi ln aix− ln

n∑i=1

axi

n

n∑i=1

axi

,and is clearly equivalent to

1

n

n∑i=1

axi ln

naxin∑i=1

axi

≥ 0.

Multiplying this byn2∑ni=1 a

xi

we have to prove

n∑i=1

ti ln ti ≥ 0, where

ti =naxi

ax1 + ax2 + . . .+ axn

149

Page 151: Олимпиадад бэлтгэгчдэд тусламж (Тэнцэтгэл биш 1)

thusn∑i=1

ti = n.

This follows from the convexity of the function x lnx, whose second derivative

is1

x> 0. Moreover, this function is strictly convex hence if not all ai are equal

then M(x) is strictly increasing, as we wish.We have remarked that f is increasing and continuous in any point exceptx = 0, where it is not defined. However we know that for x > 0 f(x) > Gand for x < 0 f(x) < G. So, let’s prove that lim

x→0M(x) = G and then we

can put M(0) = G. Dividing by G, we have to prove that limx→0

(n∑i=1

axi

) 1x

=

1, if a1a2 . . . an = 1. Now we use Taylor expansion for the function axk as

1 + ln akx+∞∑i=2

lni akxi

i!.

If we denote by A the maximum number among | ln ai|, then we see that

|axk − (1− ln akx)| < nx2A2∞∑i=0

(Ax)i

(i+ 2)!< x2A2 < xε

for any ε if x is sufficiently small.

Thereforen∑i=1

axi −

(n∑i=1

1 + ln akx

)< nxε.

Dividing by n and using the fact thatn∑i=1

ln ai = 0 we get 1 ≤n∑i=1

axi ≤ 1 + εx.

Finally using the fact that (1 + εx)1x < eε we deduce that 1 < M(x) < eε for

x sufficiently small and this means that M(x) tends to 0 as desired.Finally, it’s easy to prove that

M(+∞) = limx→+∞

M(x) = max{ai} and M(−∞) = limx→−∞

M(x) = min{ai}.

Concluding, M(x) is a non-decreasing (actually strictly increasing unless allvariables are equal in which case it is constant) function that spans all valuesbetween min{ai} and max{ai} i.e.

min{ai} = M(−∞) < M(0) = n√a1a2 . . . an < M(+∞) = max{ai}.

1.26 Inequalities between the symmetric sums

All polynomial expressions that are symmetric in n variables can be expressedin terms of their symmetric sums - this is a content of a well-known theoremin algebra. Therefore, in solving symmetric inequalities one would like tocompare the symmetric sums among each other.More precisely, take x1, x2, . . . , xn be some real numbers and consider thepolynomial

P (X) = (X − x1)(X − x2) . . . (X − xn).

150

Page 152: Олимпиадад бэлтгэгчдэд тусламж (Тэнцэтгэл биш 1)

Its coefficient at xn−k is clearly

(−1)kσk = (−1)k∑

1≤i1<...<ik≤nxi1xi2 . . . xik .

The numbers σi are called the symmetric sums of x1, . . . , xn. Clearly σk hasdegree k and has

(nk

)monomials.

Therefore if xi > 0 the numbers mk = k

√σk(nk

) are a kind of mean of the xi

(when all of them equal a, mk also equals a).The polynomial P (X) has all roots real and distinct hence so does P ′(X) byRolle’s Theorem. Repeating this procedure k times we obtain P (k)(X) hasn − k real and distinct roots. Then the polynomial Q(X) = xn−kP (k)(X)

also has n − k real roots. Then, Q(n−k−2)X is a quadratic with two real roots,

hence its discriminant must be positive. Now, let’s compute its coefficients.All coefficients (−1)iσi vanish for i < k. Moreover, all coefficients (−1)iσi fori > k+2 vanish also as after the ”reversal” (the passing from P (k) to Q) thereare not among the three leading coefficients and vanish. Thus, the coefficientsthat are left are (−1)kσk, (−1)k+1σk+1, (−1)k+2σk+2. Now one can computethat the remaining quadratic is actually

k!(n− k)!

2(−1)kσkx

2+(k+1)!(n−k−1)!(−1)k+1σk+1x+(k + 2)!(n− k − 2)!

2σk+2.

This is written simpler by passing to mi:

(−1)kn!

(1

2mkkx

2 −mk+1k+1x+

1

2mk+2k+2

).

It’s discriminant is m2(k+1)k+1 −mk

kmk+2k+2 ≥ 0.

This can be also rewritten as(mk

mk+1

)k≤(mk+1

mk+2

)k+2

.

Raising this to k+ 1th power and multiplying them for k = 0, 1, . . . , i+ 1 and

cancelling common terms we deduce 1 ≤(

mi

mi+1

)i(i+1)

so mi ≥ mi+1.

The inequality mi ≥ mi+1 is a pretty strong inequality. For example, theinequality 3(x2 + y2 + z2) ≥ (x+ y+ z)2 is equivalent to 3(σ21 − 2σ2) ≥ σ21 i.e.σ21 ≥ 3σ2 which follows directly from the inequality m1 ≥ m2 for n = 3.We invite the reader to play with these relations and find more inequalities.Note: other symmetric inequalities, like Schur’s inequality, can also be writtenin terms of symmetric sums. In the following chapter, we will concentrate onthe case n = 3, which gives most applications.

1.27 The pqr technique

This inequality is devoted to a special kind of substitution. It is a particularcase of the symmetric sums from the previous chapter, for three variables.

151

Page 153: Олимпиадад бэлтгэгчдэд тусламж (Тэнцэтгэл биш 1)

Recall from the previous chapter that if x1, x2, . . . , xn are variables, there arespecial expressions called the symmetric sums: the coefficients of the polyno-mial

(t− x1)(t− x2) . . . (t− xn)

Explicitly, they are

σ1 =

n∑i=1

xi, σ2 =∑

1≤i<j≤nxixj , . . . , σn = x1x2 . . . xn

It is a well-known result in elementary algebra that every polynomial whichis symmetric in x1, . . . , xn can be expressed as an expression in σ1, σ2, . . . , σn(more precisely as a polynomial in the variables σ1, . . . , σn). It is thereforesometimes convenient to rewrite a symmetric inequality in terms of the sym-metric sums.

This idea is most often applied in the case n = 3, when there are three sym-metric sums, that are usually denoted as p = x1 + x2 + x3, q = x1x2 + x2x3 +x3x1, q = x1x2x3, hence the name ”the pqr technique”.

There are two important ideas that facilitate the application of this method.

First, we should think about the relationship between p, q, r when we use theabove substitution. Some simple inequalities, proven before (as inequalities inx1, x2, x3) are

p2 ≥ 3q, q2 ≥ 3pr. (1.1)

These inequalities are just obtained by making the substitution in the wellknown inequalities (x + y + z)2 ≥ 3(xy + yz + zx) and (X + Y + Z)2 ≥3(XY + Y Z + ZX) where X = yz, Y = xz, Z = xy.

Another useful inequality that tells information about p, q, r is Schur’s inequal-ity

ak(a− b)(a− c) + bk(b− c)(b− a) + ck(c− a)(c− b) ≥ 0

for any a, b, c ≥ 0 if k ∈ R and any a, b, c ∈ R if k = 2n (n ∈ N). The proof ofthis inequality has been presented before. Here we just consider its applicationin proving inequalities. We usually use the case k = 1 (for a, b, c ≥ 0) andk = 2 since they are easily transformed into pqr form. When k = 1 anda, b, c ≥ 0, one gets a(a− b)(a− c) + b(b− c)(b− a) + c(c− a)(c− b) ≥ 0.Wehave

a(a− b)(a− c) + b(b− c)(b− a) + c(c− a)(c− b) =

= a(a2 − ab− ac+ bc) + b(b2 − bc− ba+ ca) + c(c2 − ca− cb+ ab)

= a [a(a+ b+ c)− 2(ab+ bc+ ca) + 3bc] + b [b(a+ b+ c)− 2(ab+ bc+ ca) + 3ca]

+c [c(a+ b+ c)− 2(ab+ bc+ ca) + 3ab]

= (a2 + b2 + c2)(a+ b+ c)− 2(a+ b+ c)(ab+ bc+ ca) + 9abc

=[(a+ b+ c)2 − 4(ab+ bc+ ca)

](a+ b+ c) + 9abc

= 9r + p(p2 − 4q).

And thus, we get

r ≥ p(4q − p2)9

. (1.2)

152

Page 154: Олимпиадад бэлтгэгчдэд тусламж (Тэнцэтгэл биш 1)

When k = 2, we have a2(a− b)(a− c) + b2(b− c)(b− a) + c2(c− a)(c− b) ≥ 0.And after proceeding as above, we will get

6pr ≥ (p2 − q)(4q − p2). (1.3)

These are some relations we usually use in proving inequalities. But some-times, these inequalities are not strong enough to help us, so we will needanother estimate. We will present it at the end of this section.Secondly, we should establish for ourselves some identities so that we can easilychange the inequality into pqr form without having to to waste time to do thisthing every time we try this method. Here are some easy identities.

a2 + b2 + c2 = p2 − 2qa3 + b3 + c3 = p3 − 3pq + 3ra4 + b4 + c4 = p4 − 4p2q + 2q2 + 4pra5 + b5 + c5 = p5 − 5p3q + 5pq2 + 5(p2 − q)ra2b2 + b2c2 + c2a2 = q2 − 2pra3b3 + b3c3 + c3a3 = q3 − 3pqr + 3r2

(a+ b)(b+ c)(c+ a) = pq − rab(a+ b) + bc(b+ c) + ca(c+ a) = pq − 3ra3(b+ c) + b3(c+ a) + c3(a+ b) = p2q − 2q2 − pra4(b+ c) + b4(c+ a) + c4(a+ b) = qp3 − 3pq2 + (5q − p2)ra3(b2 + c2) + b3(c2 + a2) + c3(a2 + b2) = pq2 − (2p2 + q)r(a+ b)(a+ c) + (b+ c)(b+ a) + (c+ a)(c+ b) = p2 + q

Finally, let us apply this technique in practice.

Example 1. Let a, b, c > 0 such that a+ b+ c = 1. Show that4

ab+ bc+ ca+

81abc ≥ 15.

Solution: Our inequality is equivalent to4

q+ 81r ≥ 15 .Now, since a, b, c

are positive real numbers, the Schur’s inequality in degree three (i.e. k = 1)gives us 9r ≥ p(4q − p2) = 4q − 1. Using this inequality, it suffices to showthat 4

q + 9(4q − 1) ≥ 15, or 1q + 9q ≥ 6 which is obviously true by AM-GM

inequality.

Example 2. If a, b, c are positive reals such that abc = 1, thena+ b+ c

3≥

5

√a2 + b2 + c2

3

Solution: Our inequality is equivalent to p5 ≥ 81(p2−2q). Since q2 ≥ 3pr = 3p,

we have 3 ≤ q2

p . Therefore 81(p2 − 2q) ≤ 27q2(p2−2q)p . It suffices to prove that

p6 ≥ 27q2(p2 − 2q), or equivalently p6 + 54q3 ≥ 27p2q2. But this inequality istrue since by AM-GM inequality, we have

p6 + 54q3 = p6 + 27q3 + 27q3 ≥ 3 3√p6 · 27q3 · 27q3 = 27p2q2.

Example 3. Let a, b, c be real numbers such that a2 + b2 + c2 = 2. Prove thata+ b+ c ≤ 2 + abc.

153

Page 155: Олимпиадад бэлтгэгчдэд тусламж (Тэнцэтгэл биш 1)

Solution: From the given condition, we get q = p2−22 and we may write our

inequality as r + 2 − p ≥ 0. If p ≤ 1, then the inequality is trivial sinceby AM-GM inequality, we have 2 = a2 + b2 + c2 ≥ 3

3√a2b2c2 ≥ 2

3√a2b2c2,

then |r| ≤ 1. If p > 1, then by Schur’s inequality for fourth degree, we get

6pr ≥ (p2 − q)(4q − p2) =

(p2 − p2 − 2

2

)[2(p2 − 2)− p2

]=

(p2 + 2)(p2 − 4)

2,

or r ≥ (p2 + 2)(p2 − 4)

12p. It suffices to show that (p2+2)(p2−4)

12p + 2 − p ≥ 0, or

(4p+ p2 − 2)(p− 2)2 ≥ 0 which is obviously true since p ≥ 1.

Example 4. Let a, b, c > 0 such that abc = 1. Show that (a+ b)(b+ c)(c+ a) ≥4(a+ b+ c− 1).

Solution: We may write our inequality as pq−r ≥ 4(p−1), or p(q−4)+3 ≥ 0.If q ≥ 4, then the inequality is trivial. If q ≤ 4, since q2 ≥ 3pr = 3p. We have

p(q−4)+3 ≥ q2

3·(q−4)+3 =

1

3(q−3)(q2−q−3). On the other hand, we have

q = ab+bc+ca ≥ 33√a2b2c2 = 3. Therefore q2−q−3 ≥ 3q−q−3 = 2q−3 > 0.

Our proof is complete.As we said above, sometimes the above estimations are not always enough, forexample with the following inequality

1

a+

1

b+

1

c+ 48(ab+ bc+ ca) ≥ 25.

for any a, b, c > 0 such that a+ b+ c = 1.We cannot apply the above estimations to prove this inequality since it hasa ”strange” equality case, that is

(12 ,

14 ,

14

), but all above estimations are just

used for the case a = b = c or a = b, c = 0 or b = c = 0. That is the mainreason.Here is a stronger estimate:

Denote a+ b+ c = p, ab+ bc+ ca = p2−t23 , abc = r, where t ≥ 0, then we have

(p+ t)2(p− 2t)

27≤ r ≤ (p− t)2(p+ 2t)

27.

This estimate attains equality when (a− b)(b− c)(c− a) = 0.

Exercises

1. Let a, b, c > 0 such that a+ b+ c = 1. Show that a3 + b3 + c3 + 4abc ≥ 1

4.

Solution: After transforming the inequality into pqr form, we need toprove 1 − 3q + 7r ≥ 1

4 . By Schur’s inequality for third degree, we have

4q ≤ 1 + 9r, hence 1− 3q + 7r ≥ 1− 3

4(1 + 9r) + 7r =

1

4+

1

4r ≥ 1

4.

2. Let a, b, c be real numbers such that a2 + b2 + c2 = 9. Prove that 2(a+b+ c)− abc ≤ 10.

Solution: With this problem, we will get confused since the originalinequality attains equality for (a, b, c) = (2, 2,−1). With this equality

154

Page 156: Олимпиадад бэлтгэгчдэд тусламж (Тэнцэтгэл биш 1)

point, we cannot apply the Schur’s inequality to prove it since the Schur’sInequality attains equality for (t, t, 0). But there is a key for us to applyit, that is to change the equality case from (2, 2,−1) to (t, t, 0). To dothis, we will use the substitution a = x+ 1, b = y+ 1, c = z+ 1, then theoriginal condition can be rewritten as (x−1)2+(y−1)2+(z−1)2 = 9, orp2 − 2p− 2q = 6.And we need to prove 2 [(x− 1) + (y − 1) + (z − 1)]−

(x−1)(y−1)(z−1) ≤ 10, or p+ q− r ≤ 15, or p+p2 − 2p− 6

2− r ≤ 15,

or 2r ≥ p2 − 36. From (x− 1)2 + (y − 1)2 + (z − 1)2 = 9, we have

|(x− 1)(y − 1)(z − 1)| ≤

√[(x− 1)2 + (y − 1)2 + (z − 1)2

3

]3= 3√

3.

Hence

2 [(x− 1) + (y − 1) + (z − 1)]− (x− 1)(y − 1)(z − 1) ≤≤ 2 [(x− 1) + (y − 1) + (z − 1)] + |(x− 1)(y − 1)(z − 1)|= 2p+ |(x− 1)(y − 1)(z − 1)| − 6 ≤ 2p+ 3

√3− 6.

Thus, if 2p ≤ 16− 3√

3, then our inequality is proved. If 2p ≥ 16− 3√

3,then by Schur’s inequality for fourth degree, we have

6pr ≥ (p2 − q)(4q − p2) =

(p2 − p2 − 2p− 6

2

)(4 · p

2 − 2p− 6

2− p2

)=

1

2(p+ 2)(p− 6)(p2 + 2p+ 6),

or

2r ≥ (p+ 2)(p− 6)(p2 + 2p+ 6)

6p.

It suffices to show that(p+ 2)(p− 6)(p2 + 2p+ 6)

6p≥ p2 − 36, or (p −

6)2(p2 + 4p− 2) ≥ 0 which is obviously true since p ≥ 16−3√3

2 > 1.

3. Let a, b, c ≥ 0 such that a+b+c = 2. Prove that a2b2+b2c2+c2a2+abc ≤1.

Solution: We have p = 2 and our inequality is equivalent to q2− 3r ≤ 1.By Schur’s inequality for third degree, we have 9r ≥ p(4q−p2) = 8(q−1),

or q ≤ 8 + 9r

8. Hence q2−3r ≤

(8 + 9r

8

)2

−3r = 1+3

64r(27r−16) ≤ 1 as

by AM-GM inequality, we have r ≤(a+ b+ c

3

)3

=8

27<

16

27.

4. Let a, b, c be positive real numbers such that abc = 1. Prove that

1

1 + a+ b+

1

1 + b+ c+

1

1 + c+ a≤ 1

a+ 2+

1

b+ 2+

1

c+ 2.

Solution: After some computations (with notice that r = 1), we can findthat

1

1 + a+ b+

1

1 + b+ c+

1

1 + c+ a=

p2 + 4p+ 3 + q

p2 + 2p+ pq + q,

155

Page 157: Олимпиадад бэлтгэгчдэд тусламж (Тэнцэтгэл биш 1)

and1

a+ 2+

1

b+ 2+

1

c+ 2=

4p+ 12 + q

4p+ 9 + 2p.

Hence, our inequality is equivalent to4p+ 12 + q

4p+ 9 + 2p≥ p2 + 4p+ 3 + q

p2 + 2p+ pq + q,

or (3q−5)p2 + (q2 + 6q−24)p− q2−3q−27 ≥ 0. By AM-GM inequality,we have p, q ≥ 3, therefore q2 + 6q − 24 > 0. Hence

(3q − 5)p2 + (q2 + 6q − 24)p− q2 − 3q − 27 ≥≥ (3q − 5) · 32 + (q2 + 6q − 24) · 3− q2 − 3q − 27

= 2(q − 3)(q + 24) ≥ 0.

5. If a, b, c > 0 thena

b+ c+

b

c+ a+

c

a+ b+

abc

2(a3 + b3 + c3)≥ 5

3.

Solution: We normalize for p = 1, then we have

a

b+ c+

b

c+ a+

c

a+ b= (a+ b+ c)

(1

b+ c+

1

c+ a+

1

a+ b

)− 3

=q + 1

q − r− 3,

andabc

a3 + b3 + c3=

r

1− 3q + 3r.

Hence, our inequality is equivalent toq + 1

q − r+

r

2(1− 3q + 3r)≥ 14

3. By

Schur’s inequality for third degree, we have r ≥ 4q − 1

9. Hence

q + 1

q − r≥

q + 1

q − 4q − 1

9

=9(q + 1)

5q + 1, and

r

2(1− 3q + 3r)≥

4q−19

2

(1− 3q + 3 · 4q − 1

9

) =

4q − 1

6(2− 5q).We need to prove

9(q + 1)

5q + 1+

4q − 1

6(2− 5q)≥ 14

3, or

(17− 50q)(1− 3q)

2(5q + 1)(2− 5q)≥

0 which is true since17

50>

1

3≥ q.

6. Let a, b, c ≥ 0 such that ab + bc + ca + abc = 4. Prove that a + b + c ≥ab+ bc+ ca.

Solution: From the given condition, we have q + r = 4. And we need toprove p ≥ q. If p ≥ 4, then it is trivial since p ≥ 4 ≥ q. If 4 ≥ p, by AM-GM inequality, we can easily check that p ≥ 3. And the Schur’s inequality

for third degree gives us r ≥ p(4q − p2)9

. Then q +p(4q − p2)

9≤ 4, or

q ≤ p3 + 36

4p+ 9. Hence p− q ≥ p− p3 + 36

4p+ 9=

(4− p)(p2 − 9)

4p+ 9≥ 0.

7. Let a, b, c ≥ 0. Prove thata2 + b2 + c2

ab+ bc+ ca+

8abc

(a+ b)(b+ c)(c+ a)≥ 2.

156

Page 158: Олимпиадад бэлтгэгчдэд тусламж (Тэнцэтгэл биш 1)

Solution: Also, we normalize that p = 1, then our inequality becomes1− 2q

q+

8r

q − r≥ 2, or

8r

q − r≥ 4q − 1

q.By Schur’s inequality for third

degree and fourth degree, we have r ≥ max

{(4q − 1)(1− q)

6, 4q−19

}.

Then8r

q − r≥ 8r

q − 4q−19

=72r

1 + 5q≥ 12(4q − 1)(1− q)

1 + 5q. It suffices to

show that12(4q − 1)(1− q)

1 + 5q≥ 4q − 1

q, or

(1− 3q)(1− 4q)2

q(1 + 5q)≥ 0 which

is true.

8. Let a, b, c ≥ 0 such that ab+bc+ca+6abc = 9. Prove that a+b+c+3abc ≥6.

Solution: From the given condition, we have q + 6r = 9 and we needto prove p + 3r ≥ 6, or 2p − q − 3 ≥ 0. If p ≥ 6, then it is trivial since2p−q−3 ≥ 2·6−9−3 = 0. If 6 ≥ p, by AM-GM inequality, we get p ≥ 3.

And the Schur’s inequality for third degree gives us r ≥ p(4q − p2)9

.

Hence q +2p(4q − p2)

3≤ 9, or q ≤ 2p3 + 27

8p+ 3. Thus 2p − q − 3 ≥ 2p −

2p3 + 27

8p+ 3− 3 =

2(6− p)(p− 3)(p+ 1)

8p+ 3≥ 0.

9. Let a, b, c ≥ 0 such that a2 + b2 + c2 = 3. Prove that 12 + 9abc ≥7(ab+ bc+ ca).

Solution: The given condition can be rewritten as p2 − 2q = 3, and weneed to prove 12 + 9r ≥ 7q. By Schur’s inequality for third degree, wehave 9r ≥ p(4q − p2) = p

[2(p2 − 3)− p2

]= p(p2 − 6). Hence

12 + 9r − 7q ≥ 12 + p(p2 − 6)− 7q = 12 + p(p2 − 6)− 7 · p2 − 3

2

=1

2(2p+ 5)(p− 3)2 ≥ 0.

10. If a, b, c are positive reals such that abc = 1. Then 1 +3

a+ b+ c≥

6

ab+ bc+ ca.

Solution: The inequality can be rewritten as 1 +3

p− 6

q≥ 0. By AM-GM

inequality, we have q2 ≥ 3pr = 3p, hence3

p≥ 9

q2. Thus 1 +

3

p− 6

q≥

9

q2− 6

q+ 1 =

(3

q− 1

)2

≥ 0.

11. If a, b, c > 0, abc = 1. Then 2(a2+b2+c2)+12 ≥ 3(a+b+c)+3(ab+bc+ca).

Solution: Rewrite the original inequality in the pqr form 2p2− 3p− 7q+12 ≥ 0. By Schur’s inequality for third degree, we have p(4q−p2) ≤ 9r =

157

Page 159: Олимпиадад бэлтгэгчдэд тусламж (Тэнцэтгэл биш 1)

9, hence q ≤ p3+94p , thus

2p2 − 3p− 7q + 12 ≥ 2p2 − 3p− 7 · p3 + 9

4p+ 12 =

(p− 3)(p2 − 9p+ 21)

4p.

By AM-GM inequality, we have p ≥ 3 and p2−9p+21 ≥(2√

21− 9)p >

0. Our proof is complete.

12. If a, b, c are nonnegative real numbers, no two of which are zero. Deter-

mine the least constant a such that

(x+ y + z

3

)a(xy + yz + zx

3

)3−a2≥

(x+ y)(y + z)(z + x)

8.

Solution: Let a = b = 1, c → 0, then we get a ≥ 3 ln 3− 4 ln 2

2 ln 2− ln 3= a0 ≈

1.81884 . . . We will show that it is the value which we find, that is

(x+ y + z

3

)a0 (xy + yz + zx

3

)3−a02≥ (x+ y)(y + z)(z + x)

8.

Since this inequality is homogeneous, we can normalize that p = 1, then

it becomes r +8q

3−a02

33+a02

− q ≥ 0. If 1 ≥ 4q, then we have8q

3−a02

33+a02

− q =

q3−a02

(8

33+a02

− qa0−12

)≥ q

3−a02

8

33+a02

−(

1

4

)a0−12

= 0. If 13 ≥ q ≥

14 , then by Schur’s inequality for third degree, we have r ≥ 4q − 1

9. It

suffices to show that4q − 1

9+

8q3−a02

33+a02

−q ≥ 0, or f(q) = −5

9q+

8q3−a02

33+a02

1

9≥ 0. We have f ′′(q) = −2(3− a0)(a0 − 1)

33+a02 · q

1+a02

< 0. Hence f(q) is concave,

thus f(q) ≥ min

{f

(1

3

), f

(1

4

)}= 0. Our proof is complete.

13. Let a, b, c ≥ 0 such that a+b+c = 1. Prove that1

a+ b+

1

b+ c+

1

c+ a+

2abc ≥ 247

54.

Solution: The inequality is equivalent to 1+qq−r + 2r ≥ 247

54 , or 1+rq−r + 2r −

19354 ≥ 0. By Schur’s inequality for third degree, we have q ≤ 1+9r

4 . Hence

1 + r

q − r+ 2r − 193

54≥ 1 + r

1+9r4 − r

+ 2r − 193

54=

4(r + 1)

5r + 1+ 2r − 193

54

=(23− 20r)(1− 27r)

54(5r + 1)≥ 0

aince by AM-GM inequality, we have 27r ≤ 1.

158

Page 160: Олимпиадад бэлтгэгчдэд тусламж (Тэнцэтгэл биш 1)

14. If a, b, c ≥ 0, prove that a4(b+ c) + b4(c+a) + c4(a+ b) ≤ 1

12(a+ b+ c)5.

Solution: We normalize that p = 1, then the inequality becomes (5q −1)r+ q−3q2 ≤ 1

12, or 12(5q−1)r ≤ (1−6q)2. If q ≤ 1

5 , then it is trivial.

If1

3≥ q ≥ 1

5, then from 3r ≤ q2, we have

12(5q − 1)r − (1− 6q)2 ≤ 4q2(5q − 1)− (1− 6q)2

= 20q3 − 40q2 + 12q − 1

=

(20q3 − 36q2 + 12q − 28

25

)− 4q2 +

3

25

=4

25(5q − 1)2(5q − 7)− 4q2 +

3

25

≤ −4q2 +3

25≤ 3

25− 4 ·

(1

5

)2

= − 1

25< 0.

15. If a, b, c are nonnegative real numbers, no two of which are zero. Provethat(

a

b+ c

)2

+

(b

c+ a

)2

+

(c

a+ b

)2

+10abc

(a+ b)(b+ c)(c+ a)≥ 2.

Solution: Denote x =2a

b+ c, y =

2b

c+ a, z =

2c

a+ b, then we have 1

x+2 +

1y+2 + 1

z+2 = 1, or xy + yz + zx+ xyz = 4. Now, put p = x+ y + z, q =xy+ yz+ zx and r = xyz, then we have q+ r = 4, and we need to provex2 + y2 + z2 + 5xyz ≥ 8, or p2 − 2q + 5(4− q) ≥ 8, or p2 − 7q + 12 ≥ 0.If p ≥ 4, then it is trivial since p2 − 7q + 12 ≥ 42 − 7 · 4 + 12 = 0.If 4 ≥ p, then we can easily check by AM-GM inequality that p ≥ 3.

Using Schur’s inequality for third degree, we have r ≥ p(4q − p2)9

, hence

q +p(4q − p2)

9≤ 4, or q ≤ p3 + 36

4p+ 9. It follows that p2 − 7q + 12 ≥

p2 − 7 · p3 + 36

4p+ 9+ 12 =

3(p− 3)(16− p2)4p+ 9

≥ 0.

1.28 The tangent line technique and its extensions

The technique of using tangent line is an elegant method that provides niceand simple solutions for many inequalities we solve. But as we see, the originaltechnique can only help us solve the inequalities with only one equality casewhen all variables are equal. This is really a big disadvantage of this technique.In the latter part of this section, we will expand the technique to a morecomplicated version which can sometimes solve inequalities that the originaltangent line technique cannot tackle.

159

Page 161: Олимпиадад бэлтгэгчдэд тусламж (Тэнцэтгэл биш 1)

Starting with some examples

Example 1. If a, b, c are positive real numbers, then

a

b+ c+

b

c+ a+

c

a+ b≥ 3

2.

Solution: Without loss of generality, we may assume that a+ b+ c = 3, thenour inequality becomes

a

3− a+

b

3− b+

c

3− c≥ 3

2.

For any x ≤ 3, we have4x

3− x− 3x+ 1 =

3(x− 1)2

3− x≥ 0. Hence

4a

3− a+

4b

3− b+

4c

3− c≥ (3a− 1) + (3b− 1) + (3c− 1) = 6.

Example 2. Let a, b, c be positive real numbers, then prove that

(2a+ b+ c)2

2a2 + (b+ c)2+

(2b+ c+ a)2

2b2 + (c+ a)2+

(2c+ a+ b)2

2c2 + (a+ b)2≤ 8.

Solution: Suppose that a+ b+ c = 3, then our inequality becomes

(3 + a)2

2a2 + (3− a)2+

(3 + b)2

2b2 + (3− b)2+

(3 + c)2

2c2 + (3− c)2≤ 8,

or equivalently

a2 + 6a+ 9

a2 − 2a+ 3+b2 + 6b+ 9

b2 − 2b+ 3+c2 + 6c+ 9

c2 − 2c+ 3≤ 24.

For any x > 0, we havex2 + 6x+ 9

x2 − 2x+ 3− (4x + 4) = −(4x+ 3)(x− 1)2

x2 − 2x+ 3≤ 0.

Hence

a2 + 6a+ 9

a2 − 2a+ 3+b2 + 6b+ 9

b2 − 2b+ 3+c2 + 6c+ 9

c2 − 2c+ 3≤ (4a+ 4) + (4b+ 4) + (4c+ 4) = 24.

Example 3. Let a, b, c be positive real numbers such that abc = 1. Prove that√a2 + 1 +

√b2 + 1 +

√c2 + 1 ≤

√2(a+ b+ c).

Solution: Consider the function f(x) =√x2 + 1−

√2x+

1√2

lnx with x > 0,

we have

f ′(x) =x√

x2 + 1+

1√2x−√

2 =x√

x2 + 1+

1− 2x√2x

.

It is easy to check that f ′(x) > 0 if x ≤ 1

2. And in the case x >

1

2, we have

f ′(x) =x√

x2 + 1+

1− 2x√2x

=

x2

x2 + 1− (1− 2x)2

2x2

x√x2 + 1

+2x− 1√

2x

=(1− x)(2x3 − 2x2 + 3x− 1)

2x2(x2 + 1)

(x√

x2 + 1+

2x− 1√2x

) .160

Page 162: Олимпиадад бэлтгэгчдэд тусламж (Тэнцэтгэл биш 1)

And since 2x3−2x2 +3x−1 = x3 +x(x−1)2 +(2x−1) > 0 (for any x > 1/2),we obtain f ′(x) = 0 if and only if x = 1. By writing the variation board, wecan easily check that f(x) ≤ f(1) = 0 ∀x > 0.Hence

√a2 + 1 +

√b2 + 1 +

√c2 + 1 ≤

√2(a+ b+ c)− 1√

2(ln a+ ln b+ ln c)

=√

2(a+ b+ c).

Establishing the basic technique

While considering the above examples the reader may ask wonder about theorigin of the intermediate inequalities used. For example, with Example 1, the

fact is4x

3− x≥ 3x−1, and with Example 3, the fact is

√x2 + 1 ≤

√2x− 1√

2lnx.

The method applies when we have to prove an inequality of the form f(x1) +f(x2)+ . . .+f(xn) ≥ 0 but working directly with f(x) is troublesome . In thatcase we try to find a function g(x) such that f(x) ≥ g(x) and it is easier towork with g(x), then we will only need to prove the inequality g(x1) + g(x2) +. . .+g(xn) ≥ 0. As a matter of fact, g(x) is usually created from what are givenin the hypothesis. For example, if the hypothesis is x1+x2+ . . .+xn = n, thenwe may predict g(x) = k(x− 1). If the hypothesis is x1x2 . . . xn = 1, then wemay predict g(x) = k lnx. If the hypothesis is xk1 + xk2 + . . .+ xkn = n, then wemay predict g(x) = k(xk − 1). In general, we will consider inequalities whereequality is attained when all variables are equal. Notice that these predictionscannot be mechanical, they must depend on the case of equality (for example,the above predictions of g(x) count on the case x1 = x2 = . . . = xn = 1). Howdo we find the valid value of k? In the case when we want to compare f withx− 1, and f(x) is differentiable, then we must choose k = f ′(1) since we needto choose k such that the sign of h(x) = f(x)− g(x) does not change when xgets through the point x = 1. Similarly, if we are subject to x1 . . . xn = 1 sowe compare f to lnx we must have k = −f ′(1) because lnx has derivative −1at 1.

In the above examples, the estimate 4x3−x ≥ 3x− 1 is obtained in exactly this

way: we want to prove∑ 4x

3−x ≥ 6 i.e.∑

( 4x3−x − 2) ≥ 0. As 4x

3−x − 2 has

derivative equal to 3 when evaluated at 1, we propose the inequality 4x3−x−2 ≥

3(x− 1) i.e. 4x3−x ≥ 3x− 1, which is true.

This way is very neat and powerful with simple inequalities. But with sharperones, it is more difficult to apply useless since the inequality f(x) ≥ g(x) doesnot always holds. For example, consider with the following

Example 4. Let a, b, c be real numbers such that a+ b+ c = 1. Prove that

a

a2 + 1+

b

b2 + 1+

c

c2 + 1≤ 9

10.

(Poland 1997)

Solution: Using the above way, we can establish the following inequalityx

x2 + 1≤ 36x+ 3

50. But unfortunately, this inequality can only holds when

161

Page 163: Олимпиадад бэлтгэгчдэд тусламж (Тэнцэтгэл биш 1)

x ≥ −3

4since

36x+ 3

50− x

x2 + 1=

(4x+ 3)(3x− 1)2

50(x2 + 1)while the original prob-

lem is asking us to prove it in the case a, b, c are arbitrary real numbers. Hence,we need to examine the small cases in order to prove it.

Case 1. If min {a, b, c} ≥ −3

4, then we have

a

a2 + 1+

b

b2 + 1+

c

c2 + 1≤ 36

50(a+ b+ c) +

9

50=

9

10.

Case 2. Assume that one of a, b, c is less than −3

4, for example c < −3

4. Then,

by AM-GM inequality, we haveb

b2 + 1≤ 1

2.Hence, our inequality is proved if

a

a2 + 1≤ 2

5, or a ≤ 1

2∨a ≥ 2. It suffices for us to consider the case 2 ≥ a ≥ 1

2.

And by the same manner, we only need to prove the inequality in the case

2 ≥ a, b ≥ 12 , then we have −3

4> c = 1 − a − b ≥ −3, hence

c

c2 + 1≤ − 3

10.

Thusa

a2 + 1+

b

b2 + 1+

c

c2 + 1≤ 1

2+

1

2− 3

10=

7

10<

9

10.

In this example, the tangent line technique still works, but we need somecasework for when it fails. This is one way to extend the method, but itmakes the solutions longer and less elegant. In the next subsection, we willpresent another extension of the technique.

An extension of tangent line technique

Example 5. Let a, b, c be nonnegative real numbers such that a + b + c = 3.Prove that

1

2a2 + 3+

1

2b2 + 3+

1

2c2 + 3≥ 3

5.

Solution: The original technique cannot be applied directly since this inequal-ity attains equality not only when a = b = c but also for a = b, c = 0.Without loss of generality, assume that a ≥ b ≥ c. We will consider 2 cases

Case 1. If c ≥ 1

4, then we have

1

2x2 + 3+

4

25x− 9

25=

2(4x− 1)(x− 1)2

25(2x2 + 3)≥ 0

∀x ≥ 1

4, hence

1

2a2 + 3+

1

2b2 + 3+

1

2c2 + 3≥

(9

25− 4

25a

)+

(9

25− 4

25b

)+

(9

25− 4

25c

)=

3

5.

Case 2. If c ≤ 1

4, then

1

2x2 + 3+

8

75x− 22

75=

(4x+ 1)(2x− 3)2

75(2x2 + 3)≥ 0 ∀x ≥ 0,

hence

1

2a2 + 3+

1

2b2 + 3≥ 44

75− 8

75(a+ b) =

44

75− 8

75(3− c)

=8

75c+

4

15.

162

Page 164: Олимпиадад бэлтгэгчдэд тусламж (Тэнцэтгэл биш 1)

It suffices to show that1

2c2 + 3+

8

75c ≥ 1

3which is true since

1

2c2 + 3+

8

75c− 1

3−

2c[8c2 + 23c+ 12(1− 4c)

]75(2c2 + 3)

≥ 0.

The idea of this solution is quite similar to the original technique, but is onelevel higher in complexity. In order to prove the inequality f(x1)+f(x2)+. . .+f(xn) ≥ 0, we try establish the inequality f(x) ≥ g(x) and find when it holdsfor all x1, x2, . . . , xn. In the other case, there exist xk, xk+1, . . . , xn such thatf(xj) ≤ g(xj) for all j = k, . . . , n then will find another function h(x) suchthat f(x) ≥ h(x) for any x1, x2, . . . , xk−1, then we may reduce our inequalityinto h(x1) + h(x2) + . . .+ h(xk−1) + f(xk) + . . .+ f(xn) ≥ 0. And from now,we will prove this inequality based on the relationship between x1, x2, . . . , xn.There is many ways to establish the function h(x). In the case we considered,after consider the first case (which yields the first equality case a = b = c),in the second case, we have established the function h(x) based on the second

equality case a = b =3

2, c = 0, since it is better to choose h(x) as a linear

function: h(x) = kx + m and we need to have f(x) ≥ h(x) for all x ∈ [0, 3] ,

and it has equality when x =3

2, we can choose k = f ′

(3

2

), then we can

easily choose m (notice that this way is quite similar to the first case).

Example 6. Let a, b, c, d be nonnegative real numbers such that a+b+c+d = 2.Prove that

1

3a2 + 1+

1

3b2 + 1+

1

3c2 + 1+

1

3d2 + 1≥ 16

7.

Solution: Notice that this inequality attains equality when a = b = c = d = 1

and a = b = c =2

3, d = 0, we will use the idea above to prove it. Suppose

that a ≥ b ≥ c ≥ d, then we have 2 cases

Case 1. If d ≥ 1

12, then we have

1

3x2 + 1+

48

49x−52

49=

3(12x− 1)(2x− 1)2

39(3x2 + 1)≥ 0

∀x ≥ 1

12, hence

1

3a2 + 1+

1

3b2 + 1+

1

3c2 + 1+

1

3d2 + 1≥

≥(

52

49− 48

49a

)+

(52

49− 48

49b

)+

(52

49− 48

49c

)+

(52

49− 48

49d

)= 2.

Case 2. If d ≤ 1

12, then we have

1

3x2 + 1+

36

49x− 45

49=

(12x+ 1)(3x− 2)2

49(3x2 + 1)≥ 0

∀x ≥ 0, hence

1

3a2 + 1+

1

3b2 + 1+

1

3c2 + 1≥ 135

49− 36

49(a+ b+ c)

=135

49− 36

49(2− d) =

36

49d+

9

7.

163

Page 165: Олимпиадад бэлтгэгчдэд тусламж (Тэнцэтгэл биш 1)

It suffices to prove that1

3d2 + 1+

36

49d ≥ 1, which is true since

1

3d2 + 1+

36

49d− 1 =

3d[36d2 + 95d+ 12(1− 12d)]

49(3d2 + 1)≥ 0.

Example 7. Let a, b, c be nonnegative real numbers, not all are zero. Showthat

a2

2a2 + (b+ c)2+

b2

2b2 + (c+ a)2+

c2

2c2 + (a+ b)2≤ 2

3.

Solution: This inequality attains equality only for a = b, c = 0. But we stillcan use the above idea to prove it. Indeed, without loss of generality, assumethat a+ b+ c = 1 and a ≥ b ≥ c, then our inequality becomes

a2

3a2 − 2a+ 1+

b2

3b2 − 2b+ 1+

c2

3c2 − 2c+ 1≤ 2

3.

There are 2 cases

Case 1. If b ≥ 1

6, then we have

x2

3x2 − 2x+ 1+ 1

9−8

9x = −(6x− 1)(2x− 1)2

9(3x2 − 2x+ 1)≤

0 ∀x ≥ 1

6, hence

a2

3a2 − 2a+ 1+

b2

3b2 − 2b+ 1≤ 8

9(a+ b)− 2

9=

8

9(1− c)− 2

9=

2

3− 8

9c.

Butc2

3c2 − 2c+ 1− 8

9c = −c[(1− 3c)(17− 24c) + 7]

27(3c2 − 2c+ 1)≤ 0. Therefore, our in-

equality is proved.

Case 2. If1

6≥ b ≥ c, then a = 1− b− c ≥ 2

3, we have

x2

3x2 − 2x+ 1− 2

9x =

x(6x− 1)(2− x)

9(3x2 − 2x+ 1)≤ 0 ∀x ≤ 1

6,

Henceb2

3b2 − 2b+ 1+

c2

3c2 − 2c+ 1≤ 2

9(b+ c) =

2

9− 2

9a.

It suffices to show thata2

3a2 − 2a+ 1− 2

9a− 4

9≤ 0, which is true since

a2

3a2 − 2a+ 1− 2

9a− 4

9= −(3a− 2)(6a2 + 3a− 4) + 4

27(3a2 − 2a+ 1)≤ 0.

Another extension

Example 8. Let a, b, c be nonnegative real numbers such that a + b + c = 3.Prove that

1

9− ab+

1

9− bc+

1

9− ca≤ 3

8.

Solution: This inequality has the form of f(x1) + f(x2) + f(x3) ≥ 0 which issimilar to the above inequalities, that we have considered but the difference is

164

Page 166: Олимпиадад бэлтгэгчдэд тусламж (Тэнцэтгэл биш 1)

that x1, x2, x3 are equal to ab, bc, ca and not the original variables. hypothesisis related to a, b, c not ab, bc, ca. Even more, when we try to establish theinequality f(x) ≤ g(x) where g(x) has the form k(x − 1) then we actuallyobtain a reversed inequality. Indeed, by this way, we would want to establish

the inequality1

9− x≤ x+ 7

64. But

1

9− x− x+ 7

64=

(x− 1)2

64(9− x)≥ 0 which is

not what we want.This can still be remedied. Taking notice that ab, bc, ca < 3, it suffices to

consider x < 3, then we have1

9− x=x+ 7

64+

(x− 1)2

64(9− x)≤ x+ 7

64+

(x− 1)2

64(9− 3)=

x2 + 4x+ 43

384. Then we get

1

9− ab+

1

9− bc+

1

9− ca≤ 1

384(a2b2 + b2c2 + c2a2 + 4ab+ 4bc+ 4ca+ 43).

It suffices to show that a2b2 + b2c2 + c2a2 + 4(ab + bc + ca) ≤ 15. Put x =ab + bc + ca, then x ≤ 3 and by Schur’s inequality degree a(a − b)(a − c) +b(b− c)(b− a) + c(c− a)(c− b) ≥ 0, we have abc ≥ max

{0, 4x−93

}.

If 4x ≤ 9, then

a2b2 + b2c2 + c2a2 + 4(ab+ bc+ ca) = x2 + 4x− 6abc

≤ x2 + 4x ≤ 225

16< 15.

If 4x ≥ 9, then

a2b2 + b2c2 + c2a2 + 4(ab+ bc+ ca) = x2 + 4x− 6abc

≤ x2 + 4x− 2(4x− 9)

= (x− 1)(x− 3) + 15 ≤ 15.

Example 9. Let a, b, c be positive real number such that a4+b4+c4 = 3. Provethat

1

4− ab+

1

4− bc+

1

4− ca≤ 1.

Solution: Notice that ab, bc, ca <3

2, and for all x <

3

2, we have

1

4− x=x+ 2

9+

(x− 1)2

9(4− x)≤ x+ 2

9+

(x− 1)2

9

(4− 3

2

) =2x2 + x+ 12

45.

We have

1

4− ab+

1

4− bc+

1

4− ca≤ 1

45

[2(a2b2 + b2c2 + c2a2) + ab+ bc+ ca+ 12

].

On the other hand, by AM-GM inequality, we have a2b2 + b2c2 + c2a2 ≤

a4 + b4 + c4 = 3 and ab+ bc+ ca ≤ a2b2 + b2c2 + c2a2 + 3

2≤ 3. From this, we

get the result.

165

Page 167: Олимпиадад бэлтгэгчдэд тусламж (Тэнцэтгэл биш 1)

Section for self study

As we saw above, the simple and intuitive tangent line technique can be ex-panded in many complicated, creative or lucrative ways to solve harder in-equalities. Unfortunately, not all these expansions can be categorized. Inpractice, one needs to start with the original idea and modify it accordinglyto suit the needs of every particular problem. In this section, we present someexamples that can be solved in this way.

Example 10. Let a, b, c be positive real numbers such that abc = 1. Show that

a

a2 + 3+

b

b2 + 3+

c

c2 + 3≤ 3

4.

Solution: Notice thatx

x2 + 3− 3x(x+ 1)

8(x2 + x+ 1)= − x(3x+ 1)(x− 1)2

8(x2 + 3)(x2 + x+ 1)≤ 0

∀x ≥ 0. Hence

a

a2 + 3+

b

b2 + 3+

c

c2 + 3≤ 3

8

(a2 + a

a2 + a+ 1+

b2 + b

b2 + b+ 1+

c2 + c

c2 + c+ 1

)=

3

8

(3− 1

a2 + a+ 1− 1

b2 + b+ 1− 1

c2 + c+ 1

).

It suffices to prove that

1

a2 + a+ 1+

1

b2 + b+ 1+

1

c2 + c+ 1≥ 1.

Put a =yz

x2, b =

zx

y2, c =

xy

z2where x, y, z > 0, our inequality becomes

x4

x4 + x2yz + y2z2+

y4

y4 + y2zx+ z2x2+

z4

z4 + z2xy + x2y2≥ 1.

which is true since by Cauchy Schwartz inequality and AM-GM inequality, wehave

x4

x4 + x2yz + y2z2+

y4

y4 + y2zx+ z2x2+

z4

z4 + z2xy + x2y2≥

≥ (x2 + y2 + z2)2

x4 + y4 + z4 + x2y2 + y2z2 + z2x2 + xyz(x+ y + z)

≥ (x2 + y2 + z2)2

x4 + y4 + z4 + 2(x2y2 + y2z2 + z2x2)= 1.

Example 11. Let a, b, c be positive real number such that a+ b+ c = 2. Provethat

bc

a2 + 1+

ca

b2 + 1+

ab

c2 + 1≤ 1.

Solution: Our inequality is equivalent to

abc

(a

a2 + 1+

b

b2 + 1+

c

c2 + 1

)+ 1− (ab+ bc+ ca) ≥ 0.

166

Page 168: Олимпиадад бэлтгэгчдэд тусламж (Тэнцэтгэл биш 1)

Note that for all x ≥ 0, we have 1x2+1

− 1 +1

2x =

x(x− 1)2

x2 + 1≥ 0. Hence

a

a2 + 1+

b

b2 + 1+

c

c2 + 1≥ a

(1− 1

2a

)+ b

(1− 1

2b

)+ c

(1− 1

2c

)= 2− 1

2(a2 + b2 + c2) = ab+ bc+ ca.

It suffices to show that abc(ab + bc + ca) + 1 − (ab + bc + ca) ≥ 0. Put x =

ab+ bc+ ca ≤ 1

3(a+ b+ c)2 =

4

3, then by Schur’s inequality for fourth degree

a2(a−b)(a−c)+b2(b−c)(b−a)+c2(c−a)(c−b) ≥ 0, we get r ≥ (x− 1)(4− x)

3.

Therefore

abc(ab+ bc+ ca) + 1− (ab+ bc+ ca) = xabc+ 1− x

≥ x(x− 1)(4− x)

3+ 1− x

=(3− x)(x− 1)2

3≥ 0.

Example 12. Let a, b, c be nonnegative real numbers such that a + b + c = 1.Prove that

a2 + bc

a2 + 1+b2 + ca

b2 + 1+c2 + ab

c2 + 1≤ 13

20.

Solution: Notice that for all 0 ≤ x ≤ 1, we have

x

x2 + 1− 12

25x− 4

25= −(3x+ 4)(2x− 1)2

25(x2 + 1)≤ 0,

and1

x2 + 1− 1 +

1

2x2 =

x2(x2 − 1)

2(x2 + 1)≤ 0.

We have

a2

a2 + 1+

b2

b2 + 1+

c2

c2 + 1≤ a

(12

25a+

4

25

)+ b

(12

25b+

4

25

)+ c

(12

25c+

4

25

)=

12

25(a2 + b2 + c2) +

4

25,

and

bc

a2 + 1+

ca

b2 + 1+

ab

c2 + 1≤ bc

(1− 1

2a2)

+ ca

(1− 1

2b2)

+ ab

(1− 1

2c2)

= ab+ bc+ ca− 1

2abc.

We need to prove

12

25(a2 + b2 + c2) +

4

25+ ab+ bc+ ca− 1

2abc ≤ 13

20,

Put q = ab + bc + ca, r = abc then the inequality becomes 1225(1 − 2q) + 4

25 +q − 1

2r ≤1320 , or (1 − 4q + 9r) + 41r ≥ 0 which is obviously true by Schur’s

inequality for third degree.

Exercises

167

Page 169: Олимпиадад бэлтгэгчдэд тусламж (Тэнцэтгэл биш 1)

1. Show that if a+ b+ c = 6 then a4 + b4 + c4 ≥ 2(a3 + b3 + c3).

Solution: We have a4 − 2a3 − 8(a − 2) =[(a+ 1)2 + 3

](a − 2)2 ≥ 0,

hence

a4 + b4 + c4 − 2(a3 + b3 + c3) ≥ 8(a− 2) + 8(b− 2) + 8(c− 2) = 0.

2. Let a, b, c > 0, a+ b+ c = 1. Prove thata

1 + bc+

b

1 + ca+

c

1 + ab≥ 9

10.

Solution: We have1

1 + x+

81

100x− 99

100=

(9x− 1)2

100(x+ 1)≥ 0 ∀x ≥ 0.Hence

a

1 + bc+

b

1 + ca+

c

1 + ab≥

≥ a

(99

100− 81

100bc

)+ b

(99

100− 81

100ca

)+ c

(99

100− 81

100ab

)=

99

100− 243

100abc ≥ 99

100− 243

100·(a+ b+ c

3

)3

=9

10.

3. Show that if a, b, c > 0 and abc = 1 then a2 + b2 + c2 + 9(ab+ bc+ ca) ≥10(a+ b+ c).

Solution: The inequality is equivalent to

a2 + b2 + c2 + 9

(1

a+

1

b+

1

c

)− 10(a+ b+ c) ≥ 0.

Consider the function f(x) = x2 +9

x− 10x+ 17 lnx with x > 0, we have

f ′(x) = 2x− 9

x2− 10 +

17

x=

(x− 1)[2(x− 2)2 + 1

]x2

,

Hence f ′(x) = 0 if and only if x = 1. Now, by making variation board,we get f(x) ≥ f(1) = 0 ∀x > 0. Therefore

a2 +b2 +c2 +9

(1

a+

1

b+

1

c

)−10(a+b+c) ≥ −17(ln a+ln b+ln c) = 0.

4. Show that if a, b, c > 0 then

(b+ c− a)2

a2 + (b+ c)2+

(c+ a− b)2

b2 + (c+ a)2+

(a+ b− c)2

c2 + (a+ b)2≥ 3

5.

Solution: Assume that a+ b+ c = 1, then our inequality becomes

(1− 2a)2

2a2 − 2a+ 1+

(1− 2b)2

2b2 − 2b+ 1+

(1− 2c)2

2c2 − 2c+ 1≥ 3

5.

We have(1− 2a)2

2a2 − 2a+ 1− 23

25+

54

25a =

2(6a+ 1)(1− 3a)2

25(2a2 − 2a+ 1)≥ 0. Hence

(1− 2a)2

2a2 − 2a+ 1+

(1− 2b)2

2b2 − 2b+ 1+

(1− 2c)2

2c2 − 2c+ 1≥ 69

25− 54

25(a+ b+ c) =

3

5.

168

Page 170: Олимпиадад бэлтгэгчдэд тусламж (Тэнцэтгэл биш 1)

5. Let a, b, c > 0, a2 + b2 + c2 = 3. Prove that1

a+

1

b+

1

c+

4

3(a+ b+ c) ≥ 7.

Solution: We have1

a+

4

3a− a2 + 13

6=

(6− a)(a− 1)2

6a≥ 0. Hence

1

a+

1

b+

1

c+

4

3(a+ b+ c) ≥ a2 + 13

6+b2 + 13

6+c2 + 13

6= 7.

6. For any positive real numbers a, b, c such that a+b+c = 1, then 10(a3 +b3 + c3)− 9(a5 + b5 + c5) ≥ 1.

Solution: We have 10a3−9a5+16

27−25

9a =

(16 + 21a− 18a2 − 27a3)(1− 3a)2

27.

Hence, assume that a ≥ b ≥ c, and we have 2 cases

Case 1. If 16 + 21a− 18a2 − 27a3 ≥ 0, then we observe that

16 + 21b− 18b2 − 27b3 ≥ 0, 16 + 21c− 18c2 − 27c3 ≥ 0.

We have

10(a3 + b3 + c3)− 9(a5 + b5 + c5) ≥ 25

9(a+ b+ c)− 16

9= 1.

Case 2. If 16 + 21a − 18a2 − 27a3 ≤ 0, then we have 27a3 + 18a2 ≥

21a + 16 ≥ 37a, or a ≥ 2√

30− 3

9>

2

3, it follows that b, c <

1

3. In this

case, we have

10b3 − 9b5 ≥ 9b3, 10c3 − 9c5 ≥ 9c3.

Hence

10b3 − 9b5 + 10c3 − 9c5 ≥ 9(b3 + c3) ≥ 9

4(b+ c)3 =

9

4(1− a)3.

It suffices to show that 10a3 − 9a5 +9

4(1− a)3 ≥ 1 which is true since

10a3 − 9a5 +9

4(1− a)3 − 1 =

(1− a)(3a− 1)(12a3 + 16a2 + 7a− 5)

4

≥ 0.

(since a >

2

3

)This completes our proof.

7. Let a, b, c be positive real numbers, then

a√b+ c

+b√c+ a

+c√a+ b

≥√

3

2(a+ b+ c).

Solution: Assume that a+ b+ c = 1, then our inequality becomes

a√1− a

+b√

1− b+

c√1− c

≥√

3

2.

169

Page 171: Олимпиадад бэлтгэгчдэд тусламж (Тэнцэтгэл биш 1)

We will show thata√

1− a≥ 5√

6

8

(a− 1

15

). Indeed, put 1 − a = 6t2

with t ≥ 0, then we have a = 1− 6t2, therefore

a√1− a

− 5√

6

8

(a− 1

15

)=

1− 6t2√6t− 5√

6

8

(1− 6t2 − 1

15

)=

√6(5t+ 2)(3t− 1)2

12t≥ 0.

Hence

a√1− a

+b√

1− b+

c√1− c

≥ 5√

6

8

(a+ b+ c− 1

5

)=

√3

2.

8. Let a, b, c be real numbers such that a2 + b2 + c2 = 1. Prove that

1

1− ab+

1

1− bc+

1

1− ca≤ 9

2.

Solution: Since

1

1− ab+

1

1− bc+

1

1− ca≤ 1

1− |ab|+

1

1− |bc|+

1

1− |ca|,

and|a|2 + |b|2 + |c|2 = a2 + b2 + c2 = 1.

It suffices to consider the case a, b, c ≥ 0. Now, for all x ≤ 1

2, we have

1

1− x=

9x+ 3

4+

(1− 3x)2

4(1− x)≤ 9x+ 3

4+

(1− 3x)2

4

(1− 1

2

) =18x2 − 3x+ 5

4.

Now, since ab, bc, ca ≤ 1

2, we get

1

1− ab+

1

1− bc+

1

1− ca≤ 3

4

[6(a2b2 + b2c2 + c2a2)− (ab+ bc+ ca) + 5

].

It suffices to show that

6(a2b2 + b2c2 + c2a2)− (ab+ bc+ ca) ≤ 1,

or

6(a2b2 + b2c2 + c2a2)− (ab+ bc+ ca)(a2 + b2 + c2) ≤ (a2 + b2 + c2)2,

or ∑cyc

a2(a− b)(a− c) + 2ab(a− b)2 + 2bc(b− c)2 + 2ca(c− a)2 ≥ 0.

which is obviously true by Schur’s inequality for fourth degree.

170

Page 172: Олимпиадад бэлтгэгчдэд тусламж (Тэнцэтгэл биш 1)

9. Let a, b, c be real numbers such that a2 + b2 + c2 + d2 = 1. Prove that

1

1− ab+

1

1− bc+

1

1− cd+

1

1− da≤ 16

3.

Solution: For all x ≤ 1

2, we have

1

1− x=

16x+ 8

9+

(1− 4x)2

9(1− x)≤ 16x+ 8

9+

(1− 4x)2

9

(1− 1

2

)=

32x2 + 10

9.

Now, since ab, bc, cd, da ≤ 1

2, we have

1

1− ab+

1

1− bc+

1

1− cd+

1

1− da≤ 32(a2b2 + b2c2 + c2d2 + d2a2) + 40

9

=32(a2 + c2)(b2 + d2) + 40

9

≤ 8(a2 + b2 + c2 + d2)2 + 40

9

=16

3.

10. Suppose a, b, c are real numbers such that a2 + b2 + c2 = 1. Show that

1

3 + a2 − 2bc+

1

3 + b2 − 2ca+

1

3 + c2 − 2ab≤ 9

8.

Solution: Since

1

3 + a2 − 2bc+

1

3 + b2 − 2ca+

1

3 + c2 − 2ab

≤ 1

3 + |a|2 − 2 |bc|+

1

3 + |b|2 − 2 |ca|+

1

3 + |c|2 − 2 |ab|,

and

|a|2 + |b|2 + |c|2 = a2 + b2 + c2 = 1.

It suffices for us to consider the case a, b, c ≥ 0. Now, for all x ∈ [−1, 1] ,we have

1

3 + x=

21− 9x

64+

(1 + 3x)2

64(3 + x)≤ 21− 9x

64+

(1 + 3x)2

64(3− 1)

=9x2 − 12x+ 43

128.

171

Page 173: Олимпиадад бэлтгэгчдэд тусламж (Тэнцэтгэл биш 1)

Since a2 − 2bc, b2 − 2ca, c2 − 2ab ∈ [−1, 1] (it is easy to check), we have

1

3 + a2 − 2bc+

1

3 + b2 − 2ca+

1

3 + c2 − 2ab

≤ 3

128

[3∑cyc

(a2 − 2bc)2 − 4∑cyc

(a2 − 2bc) + 43

]

=3

128

[3∑cyc

(a2 − 2bc)2 + 8∑cyc

ab+ 39

].

It suffices to show that

3∑cyc

(a2 − 2bc)2 + 8∑cyc

ab ≤ 9,

or

3∑cyc

(a2 − 2bc)2 + 8(ab+ bc+ ca)(a2 + b2 + c2) ≤ 9(a2 + b2 + c2)2,

or6∑cyc

a4 + 6∑cyc

a2b2 + 4abc∑cyc

a ≥ 8∑cyc

ab(a2 + b2).

By Schur’s inequality for fourth degree, we have

4∑cyc

a4 + 4abc∑cyc

a ≥ 4∑cyc

ab(a2 + b2).

We have to prove

2∑cyc

a4 + 6∑cyc

a2b2 ≥ 4∑cyc

ab(a2 + b2).

which is true since

a4 + b4 + 6a2b2 − 4ab(a2 + b2) = (a− b)4 ≥ 0.

11. Let a, b, c be positive real numbers such that abc = 1. Prove that

1

3a2 + (a− 1)2+

1

3b2 + (b− 1)2+

1

3c2 + (c− 1)2≥ 1.

Solution: Assume that a ≥ b ≥ c, then if c ≤ 1

2, we have

1

3c2 + (c− 1)2=

1

4c2 − 2c+ 1=

1

2c(2c− 1) + 1≥ 1.

In the case a ≥ b ≥ c ≥ 1

2, then consider the function f(x) =

1

3x2 + (x− 1)2−

1

3+

2

3lnx with x ≥ 1

2, we have f ′(x) =

2(x− 1)(16x3 − 1)

3x(4x2 − 2x+ 1)2, hence

172

Page 174: Олимпиадад бэлтгэгчдэд тусламж (Тэнцэтгэл биш 1)

f ′(x) = 0 if and only if x = 1 (since x ≥ 1

2). Now, by writing the

variation board, we have f(x) ≥ f(1) = 0 ∀x ≥ 1

2. Therefore

1

3a2 + (a− 1)2+

1

3b2 + (b− 1)2+

1

3c2 + (c− 1)2≥ 1−2

3(ln a+ln b+ln c) = 1.

Remark. There is another approach for this inequality: we have

1

3a2 + (a− 1)2− 1

a4 + a2 + 1=

a(a+ 2)(a− 1)2

(4a2 − 2a+ 1)(a4 + a2 + 1)≥ 0.

Hence, it suffices to show that

1

a4 + a2 + 1+

1

b4 + b2 + 1+

1

c4 + c2 + 1≥ 1.

This inequality has been proved in Example 10.

12. Let a, b, c be nonnegative real numbers, no two of which are zero. Provethat

a2

5a2 + (b+ c)2+

b2

5b2 + (c+ a)2+

c2

5c2 + (a+ b)2≤ 1

3.

Solution: Normalize by declaring a + b + c = 1 and assume a ≥ b ≥ c,then our inequality becomes

a2

6a2 − 2a+ 1+

b2

6b2 − 2b+ 1+

c2

6c2 − 2c+ 1≤ 1

3.

Consider 2 cases

Case 1. If c ≥ 1

8, then

x2

6x2 − 2x+ 1− 12x− 1

27= −(8x− 1)(3x− 1)2

27(6x2 − 2x+ 1)≤

0 ∀x ≥ 1

8. Hence

a2

6a2 − 2a+ 1+

b2

6b2 − 2b+ 1+

c2

6c2 − 2c+ 1≤ 12a− 1

27+

12b− 1

27+

12c− 1

27

=1

3.

Case 2. If c ≤ 1

8, then

x2

6x2 − 2x+ 1− 4x+ 1

18= −(6x+ 1)(2x− 1)2

18(6x2 − 2x+ 1)≤

0 ∀x ≥ 0. Hence

a2

6a2 − 2a+ 1+

b2

6b2 − 2b+ 1≤ 4(a+ b) + 2

18=

4(1− c) + 2

18=

1

3− 2

9c.

It suffices to show that c2

6c2−2c+1− 2

9c ≤ 0 which is true since

c2

6c2 − 2c+ 1− 2

9c = −c[12c2 + 3c+ 2(1− 8c)]

9(6c2 − 2c+ 1)≤ 0.

173

Page 175: Олимпиадад бэлтгэгчдэд тусламж (Тэнцэтгэл биш 1)

13. Let a, b, c, d be positive real numbers such that a+ b+ c+ d = 4. Provethat

(a2 + 1)(b2 + 1)(c2 + 1)(d2 + 1) ≥ (a+ 1)(b+ 1)(c+ 1)(d+ 1).

Solution: Assume that a ≥ b ≥ c ≥ d. Then our inequality is equivalentto ∑

cyc

[ln(a2 + 1)− ln(a+ 1)] ≥ 0.

We have 2 cases

Case 1. If a ≤ 2, consider the function f(x) = ln(x2 + 1)− ln(x+ 1)−1

2(x− 1) with x ≤ 2, we have

f ′(x) =(x− 1)(3− x2)2(x2 + 1)(x+ 1)

From this, we get f ′(x) = 0 if and only if x = 1 or x =√

3 and by writingthe variation board, we have f(x) ≥ min {f(1), f(2)} = 0 ∀x ≤ 2. Hence∑

cyc

[ln(a2 + 1)− ln(a+ 1)] ≥ 1

2

∑cyc

(a− 1) = 0.

Case 2. If a ≥ 2, then 2 ≥ b ≥ c ≥ d, consider the function g(x) =

ln(x2 + 1)− ln(x+ 1)− 7

65(3x− 2)− ln

13

15with x ≤ 2, we have

g′(x) =(3x− 2)(43 + 10x− 7x2)

65(x2 + 1)(x+ 1).

Since x ≤ 2, we have g′(x) = 0 if and only if x =2

3and by writing the

variation board, we get

g(x) ≥ g(

2

3

)= 0 ∀x ≤ 2.

Hence

[ln(b2 + 1)− ln(b+ 1)] + [ln(c2 + 1)− ln(c+ 1)] + [ln(d2 + 1)− ln(d+ 1)]

≥ 7

65(3b+ 3c+ 3d− 6) + 3 ln

13

15=

21

65(2− a) + 3 ln

13

15.

We need to prove

h(a) = ln(a2 + 1)− ln(a+ 1) +21

65(2− a) + 3 ln

13

15≥ 0.

We have

h′(a) =(3a− 2)(43 + 10a− 7a2)

65(a2 + 1)(a+ 1),

and since a ≥ 2, we get h′(a) = 0 if and only if a = 5+√326

7 . Writing thevariation board again, we have h(a) ≥ min {h(2), h(4)} > 0.

174

Page 176: Олимпиадад бэлтгэгчдэд тусламж (Тэнцэтгэл биш 1)

14. Let a, b, c be nonnegative real numbers, no two of which are zero. Provethat √

1 +48a

b+ c+

√1 +

48b

c+ a+

√1 +

48c

a+ b≥ 15.

Solution: Suppose that a+ b+ c = 1 and a ≥ b ≥ c, then our inequalitybecomes √

1 + 47a

1− a+

√1 + 47b

1− b+

√1 + 47c

1− c≥ 15.

We have 2 cases

Case 1. If c ≥ 2

27, then

1 + 47x

1− x−(

54

5x+

7

5

)2

=12(27x− 2)(3x− 1)2

25(1− x)≥ 0 ∀1 ≥ x ≥ 2

27.

Hence√1 + 47a

1− a+

√1 + 47b

1− b+

√1 + 47c

1− c≥ 54

5(a+ b+ c) +

21

5= 15.

Case 2. If c ≤ 2

27, then

1 + 47x

1− x−(

96

7x+

1

7

)2

=48(48x+ 1)(2x− 1)2

49(1− x)≥ 0 ∀1 ≥ x ≥ 0.

Hence √1 + 47a

1− a+

√1 + 47b

1− b≥ 96

7(a+ b) +

2

7= 14− 96

7c.

We need to prove√

1+47c1−c ≥ 1 + 96

7 c. Put 1+47c1−c = t2 (t ≥ 0) , then we

have c = t2−1t2+47

and since 0 ≤ c ≤ 227 , we get 11

5 ≥ t ≥ 1, our inequality

becomes t ≥ 1 + 96(t2−1)7(t2+47)

, or (t − 1)(7t2 − 96t + 233) ≥ 0 which is true

since 115 ≥ t ≥ 1.

15. Let a, b, c be nonnegative real numbers such that a + b + c = 1. Provethat

1− 4a2

1 + 3a− 3a2+

1− 4b2

1 + 3b− 3b2+

1− 4c2

1 + 3c− 3c2≤ 1.

Solution: Without loss of generality, assume that a ≥ b ≥ c. We have 2cases

Case 1. If c ≥ 1

9, then

1− 4x2

1 + 3x− 3x2+

27x− 14

15=

(1− 9x)(1− 3x)2

15(1 + 3x− 3x2)≤ 0 ∀1 ≥ x ≥ 1

9.

Hence

1− 4a2

1 + 3a− 3a2+

1− 4b2

1 + 3b− 3b2+

1− 4c2

1 + 3c− 3c2≤ 42− 27(a+ b+ c)

15= 1.

175

Page 177: Олимпиадад бэлтгэгчдэд тусламж (Тэнцэтгэл биш 1)

Case 2. If c ≤ 1

9, then

1− 4x2

1 + 3x− 3x2− 8(1− 2x)

7= −(12x+ 1)(2x− 1)2

7(1 + 3x− 3x2)≤ 0 ∀1 ≥ x ≥ 0.

Hence

1− 4a2

1 + 3a− 3a2+

1− 4b2

1 + 3b− 3b2≤ 8

7(2− 2a− 2b) =

16

7c.

We need to prove 167 c+ 1−4c2

1+3c−3c2 ≤ 1 which is true since

16

7c+

1− 4c2

1 + 3c− 3c2− 1 = −c(48c2 − 41c+ 5)

7(1 + 3c− 3c2)≤ 0

(since c ≤ 1

9

).

16. Let a, b, c, d be nonnegative real numbers such that a + b + c + d = 2.Prove that

a2

(a2 + 1)2+

b2

(b2 + 1)2+

c2

(c2 + 1)2+

d2

(d2 + 1)2≤ 16

25.

Solution: Without loss of generality, assume that a ≥ b ≥ c ≥ d. Wehave 2 cases

Case 1. If 12d3 + 11d2 + 32d ≥ 4, then

x2

(x2 + 1)2−48x− 4

125= −(12x3 + 11x2 + 32x− 4)(2x− 1)2

125(x2 + 1)2≤ 0

for all x such that 12x3 + 11x2 + 32x ≥ 4. Hence

a2

(a2 + 1)2+

b2

(b2 + 1)2+

c2

(c2 + 1)2+

d2

(d2 + 1)2≤

≤ 48a− 4

125+

48b− 4

125+

48c− 4

125+

48d− 4

125=

16

25.

Case 2. If 4 ≥ 12d3 + 11d2 + 32d ≥ 32d, it follows that d ≤ 1

8, we have

x2

(x2 + 1)2−108(5x+ 1)

2197= −(60x3 + 92x2 + 216x+ 27)(3x− 2)2

2197(x2 + 1)2≤ 0 ∀x ≥ 0.

Hence

a2

(a2 + 1)2+

b2

(b2 + 1)2+

c2

(c2 + 1)2≤ 108 [5(a+ b+ c) + 3]

2197

=108 (5(2− d) + 3)

2197

=108

169− 540

2197d.

It suffices to prove that

d2

(d2 + 1)2− 540

2197a+

108

169≤ 16

25,

176

Page 178: Олимпиадад бэлтгэгчдэд тусламж (Тэнцэтгэл биш 1)

or169d2

(d2 + 1)2− 540

13d ≤ 4

25.

We have

169d2

(d2 + 1)2− 540

13d− 4

25≤ 169d2

(d2 + 1)2− 540

15a− 4

25

=169d2

(d2 + 1)2− 36a− 4

25

= −4 + 900d− 4217a2 + 1800d3 + 4d4 + 900d5

25(d2 + 1)2

≤ −4 + 8 · 900d2 − 4217d2 + 1800d3 + 4d4 + 900d5

25(d2 + 1)2

= −4 + 2983d2 + 1800d3 + 4d4 + 900d5

25(d2 + 1)2< 0.

17. Let a1, a2, . . . , an (n ≥ 2) be nonnegative real numbers such that a1 +a2 + . . .+ an = n. Prove that

(n− 1)(a31 + a32 + . . .+ a3n) + n2 ≥ (2n− 1)(a21 + a22 + . . .+ a2n).

Solution: If n = 2, then the inequality becomes equality. If n = 3,put q = a1a2 + a2a3 + a3a1, r = a1a2a3, then the inequality becomes2(27 − 9q + 3r) + 9 ≥ 5(9 − 2q), or 3r + 9 − 4q ≥ 0 which is justSchur’s inequality for third degree. Consider the case n ≥ 4, assumethat a1 ≥ a2 ≥ . . . ≥ an. We have 2 cases

Case 1. If an ≥1

n− 1, then we have

(n−1)x3−(2n−1)x2+(n+1)x−1 = (x−1)2[(n−1)x−1] ≥ 0 ∀x ≥ 1

n− 1.

Hence

(n− 1)

n∑i=1

a3i ≥n∑i=1

[(2n− 1)a2i − (n+ 1)ai + 1],

or

(n− 1)n∑i=1

a3i + n2 ≥ (2n− 1)n∑i=1

a2i .

Case 2. If an ≤1

n− 1, then we have

(n− 1)x3 − (2n− 1)x2 +n(n− 1)(n− 2)x+ n2

(n− 1)2=

=[(n− 1)x− n]2[(n− 1)x+ 1]

(n− 1)2≥ 0 ∀x ≥ 0.

Hence

n−1∑i=1

[(n− 1)a3i − (2n− 1)a2i +

n(n− 1)(n− 2)ai + n2

(n− 1)2

]≥ 0,

177

Page 179: Олимпиадад бэлтгэгчдэд тусламж (Тэнцэтгэл биш 1)

or

(n− 1)n−1∑i=1

a3i − (2n− 1)n−1∑i=1

a2i + n2 ≥ n(n− 2)

n− 1an.

We need to prove

(n− 1)a3n − (2n− 1)a2n +n(n− 2)

n− 1an ≥ 0,

oran[(n− 1)(2− an)[1− (n− 1)an] + n2 − 4n+ 2]

n− 1≥ 0

which is true since n ≥ 4 and an ≤ 1n−1 .

1.29 Using identities to prove inequalities

This is another collection of loosely related methods which provide short so-lutions to hard problems. The idea is to find special identities which help ussolve the problem. Let us consider some examples:

Example 1. Let x, y be real numbers such that x 6= −y, show that

x2 + y2 +

(1 + xy

x+ y

)2

≥ 2.

Solution: Put z = −1+xyx+y then we have xy + yz + zx = −1 and our original

inequality is equivalent to x2 +y2 +z2 ≥ 2, or x2 +y2 +z2 ≥ −2(xy+yz+zx),which is just simply to (x+ y + z)2 ≥ 0.

Example 2. If a, b, c are distinct real numbers, then

1 + a2b2

(a− b)2+

1 + b2c2

(b− c)2+

1 + c2a2

(c− a)2≥ 3

2.

Solution: Notice that(1− aba− b

− 1

)(1− bcb− c

− 1

)(1− cac− a

− 1

)=

=(1− a)(1 + b)(1− b)(1 + c)(1− c)(1 + a)

(a− b)(b− c)(c− a)

=(1 + a)(1− b)

a− b· (1 + b)(1− c)

b− c· (1 + c)(1− a)

c− a

=

(1− aba− b

+ 1

)(1− bcb− c

+ 1

)(1− cac− a

+ 1

),

and (1 + ab

a− b− i)(

1 + bc

b− c− i)(

1 + ca

c− a− i)

=

=(i+ a)(b− i)(i+ b)(c− i)(i+ c)(a− i)

(a− b)(b− c)(c− a)

=(a− i)(i+ b)

a− b· (b− i)(i+ c)

b− c· (c− i)(i+ a)

c− a

=

(1 + ab

a− b+ i

)(1 + bc

b− c+ i

)(1 + ca

c− a+ i

),

178

Page 180: Олимпиадад бэлтгэгчдэд тусламж (Тэнцэтгэл биш 1)

Hence

1− aba− b

· 1− bcb− c

+1− bcb− c

· 1− cac− a

+1− cac− a

· 1− aba− b

= −1,

1 + ab

a− b· 1 + bc

b− c+

1 + bc

b− c· 1 + ca

c− a+

1 + ca

c− a· 1 + ab

a− b= 1.

Furthermore, for any x, y, z ∈ R, we always have (x−y)2+(y−z)2+(z−x)2 ≥ 0,and (x+ y + z)2 ≥ 0. Hence x2 + y2 + z2 ≥ xy + yz + zx, and x2 + y2 + z2 ≥−2(xy + yz + zx). Therefore(

1 + ab

a− b

)2

+

(1 + bc

b− c

)2

+

(1 + ca

c− a

)2

≥∑cyc

1 + ab

a− b· 1 + bc

b− c= 1.

and (1− aba− b

)2

+

(1− bcb− c

)2

+

(1− cac− a

)2

≥ −2∑cyc

1− aba− b

· 1− bcb− c

= 2.

Thus

2(1 + a2b2)

(a− b)2+

2(1 + b2c2)

(b− c)2+

2(1 + c2a2)

(c− a)2=

∑cyc

(1 + ab)2 + (1− ab)2

(a− b)2

=∑cyc

(1 + ab

a− b

)2

+∑cyc

(1− aba− b

)2

≥ 3.

Example 3. Let a, b, c be positive real numbers such that abc = 1. Prove that

4

(a

(a+ 1)2+

b

(b+ 1)2+

c

(c+ 1)2

)≤ 1 +

16

(a+ 1)(b+ 1)(c+ 1).

Solution: Put x =1− a1 + a

, y =1− b1 + b

, z =1− c1 + c

, then we have

−1 ≤ x, y, z ≤ 1, a =1− x1 + x

, b =1− y1 + y

, c =1− z1 + z

.

It is easy to verify that (1 − x)(1 − y)(1 − z) = (1 + x)(1 + y)(1 + z), hencex+ y + z + xyz = 0. But

4a

(a+ 1)2=

4 · 1−x1+x(1 +

1− x1 + x

)2 = 1− x2, 2

a+ 1=

2

1 +1− x1 + x

= 1 + x.

Therefore, our inequality is equivalent to

1− x2 + 1− y2 + 1− z2 ≤ 1 + 2(1 + x)(1 + y)(1 + z),

orx2 + y2 + z2 + 2(xy + yz + zx) + 2(x+ y + z + xyz) ≥ 0.

179

Page 181: Олимпиадад бэлтгэгчдэд тусламж (Тэнцэтгэл биш 1)

or

(x+ y + z)2 ≥ 0.

Example 4. Let a, b, c be positive real numbers, then show that

a

b+ c+

b

c+ a+

c

a+ b+

4abc

(a+ b)(b+ c)(c+ a)≥ 2.

Solution: Put x =a

b+ c, y =

b

c+ a, z =

c

a+ b, then we have

1

x+ 1+

1

y + 1+

1

z + 1= 2, or xy+yz+zx+2xyz = 1. And we need to prove x+y+z+4xyz ≥ 2,

or x+y+z ≥ 2(xy+yz+zx). Put p = x+y+z, q = xy+yz+zx, r = xyz then wehave q+2r = 1 and we need to prove p ≥ 2q. If p ≥ 2, then it is trivial since p ≥2 ≥ 2q. If p ≤ 2, then it is easy to verify that 2 ≥ p ≥ 3

2. By Schur’s inequality

for third degree, we have x(x−y)(x−z)+y(y−z)(y−x)+z(z−x)(z−y) ≥ 0,

or r ≥ p(4q − p2)9

. Hence q +2p(4q − p2)

9≤ 1, which implies us q ≤ 2p3 + 9

8p+ 9.

Therefore p− 2q ≥ p− 2(2p3 + 9)

8p+ 9=

(4p2 − 9)(2− p)8p+ 9

≥ 0.

Remark. Since (a+b)(b+c)(c+a) ≥ 8abc, this inequality implies us a stronger

version of the famous Nesbitt inequality

a

b+ c+

b

c+ a+

c

a+ b≥ 3

2.

Exercises

1. Let a, b, c be positive real numbers such that abc = 1. Prove that

1

(1 + a)2+

1

(1 + b)2+

1

(1 + c)2+

2

(1 + a)(1 + b)(1 + c)≥ 1.

Solution: Using the substitution as Example 3, we need to prove

(1 + x)2 + (1 + y)2 + (1 + z)2 + (1 + x)(1 + y)(1 + z) ≥ 4,

or

x2 + y2 + z2 + x2y2z2 ≥ 4xyz.

By AM-GM inequality, we have

x2 + y2 + z2 + x2y2z2 ≥ 4 4√x4y4z4 = 4 |xyz| ≥ 4xyz.

2. If a, b, c are distinct real numbers, then

a2 + ab+ b2

(a− b)2+b2 + bc+ c2

(b− c)2+c2 + ca+ a2

(c− a)2≥ 9

4.

180

Page 182: Олимпиадад бэлтгэгчдэд тусламж (Тэнцэтгэл биш 1)

Solution: We have a2 + ab+ b2 =3

4(a+ b)2 +

1

4(a− b)2, hence

a2 + ab+ b2

(a− b)2+b2 + bc+ c2

(b− c)2+c2 + ca+ a2

(c− a)2=

3

4

[∑cyc

(a+ b

a− b

)2

+ 1

].

Moreover, it is easy to verify that(a+ b

a− b+ 1

)(b+ c

b− c+ 1

)(c+ a

c− a+ 1

)=

=

(a+ b

a− b− 1

)(b+ c

b− c− 1

)(c+ a

c− a− 1

),

Hencea+ b

a− b· b+ c

b− c+b+ c

b− c· c+ a

c− a+c+ a

c− a· a+ b

a− b= −1

Therefore(a+ b

a− b

)2

+

(b+ c

b− c

)2

+

(c+ a

c− a

)2

≥ −2∑cyc

a+ b

a− b· b+ c

b− c= 2.

This inequality implies

a2 + ab+ b2

(a− b)2+b2 + bc+ c2

(b− c)2+c2 + ca+ a2

(c− a)2≥ 3

4(2 + 1) =

9

4.

3. If a, b, c are distinct real numbers, then(a2 + b2 + c2

)( 1

(a− b)2+

1

(b− c)2+

1

(c− a)2

)≥ 9

2.

Solution: According to the proof above, we have(a+ b

a− b

)2

+

(b+ c

b− c

)2

+

(c+ a

c− a

)2

≥ 2.

Hence

2(a2 + b2)

(a− b)2+

2(b2 + c2)

(b− c)2+

2(c2 + a2)

(c− a)2=

∑cyc

(a+ b)2 + (a− b)2

(a− b)2

=∑cyc

(a+ b

a− b

)2

+ 3 ≥ 5.

Also, we have (a

b− c+ 1

)(b

c− a+ 1

)(c

a− b+ 1

)=

=

(a

b− c− 1

)(b

c− a− 1

)(c

a− b− 1

),

Hencea

b− c· b

c− a+

b

c− a· c

a− b+

c

a− b· a

b− c= −1.

181

Page 183: Олимпиадад бэлтгэгчдэд тусламж (Тэнцэтгэл биш 1)

Thus(a

b− c

)2

+

(b

c− a

)2

+

(c

a− b

)2

≥ −2∑cyc

a

b− c· b

c− a= 2.

Hence (a2 + b2 + c2

)( 1

(a− b)2+

1

(b− c)2+

1

(c− a)2

)=

=∑cyc

a2 + b2

(a− b)2+∑cyc

(a

b− c

)2

≥ 5

2+ 2 =

9

2.

4. Let a, b, c be positive real numbers, show that(a

b+ c

)2

+

(b

c+ a

)2

+

(c

a+ b

)2

+10abc

(a+ b)(b+ c)(c+ a)≥ 2.

Solution: By the same substitution as Example 4, we need to prove

x2 + y2 + z2 + 10xyz ≥ 2.

Put p = x + y + z, q = xy + yz + zx, r = xyz then we have q + 2r = 1and we need to prove p2 − 2q + 10r ≥ 2, or p2 − 7q + 3 ≥ 0. If p ≥ 2,then it is trivial since p2 + 3 ≥ 7 ≥ 7q. If 2 ≥ p, then we can easily check

that 2 ≥ p ≥ 3

2. And from the proof of Example 4, we get q ≤ 2p3 + 9

8p+ 9.

Hence

p2 − 7q + 3 ≥ p2 + 3− 7(2p3 + 9)

8p+ 9=

3(2p− 3)(4− p2)8p+ 9

≥ 0.

5. Let a, b, c be distinct real numbers. Prove that

(a− 2b)2 + (a− 2c)2

(b− c)2+

(b− 2c)2 + (b− 2a)2

(c− a)2+

(c− 2a)2 + (c− 2b)2

(a− b)2≥ 22.

Solution: We have

(a− 2b)2 + (a− 2c)2

(b− c)2− 2 = 2

(b+ c− ab− c

)2

,

Hence, our inequality is equivalent to(b+ c− ab− c

)2

+

(c+ a− bc− a

)2

+

(a+ b− ca− b

)2

≥ 8.

We have (b+ c− ab− c

− 2

)(c+ a− bc− a

− 2

)(a+ b− ca− b

− 2

)=

=(3c− a− b)(3a− b− c)(3b− c− a)

(a− b)(b− c)(c− a)

=3b− c− ab− c

· 3c− a− bc− a

· 3a− b− ca− b

=

(b+ c− ab− c

+ 2

)(c+ a− bc− a

+ 2

)(a+ b− ca− b

+ 2

).

182

Page 184: Олимпиадад бэлтгэгчдэд тусламж (Тэнцэтгэл биш 1)

Hence

b+ c− ab− c

· c+ a− bc− a

+c+ a− bc− a

· a+ b− ca− b

+a+ b− ca− b

· b+ c− ab− c

= −4.

Thus, our inequality is equivalent to x2 + y2 + z2 ≥ 8, or x2 + y2 + z2 ≥−2(xy + yz + zx), which is just simply to (x+ y + z)2 ≥ 0.

6. Let a, b, c be distinct real numbers. Prove that

(1− a2)(1− b2)(a− b)2

+(1− b2)(1− c2)

(b− c)2+

(1− c2)(1− a2)(c− a)2

≥ −1.

Solution: Since

(1− a2)(1− b2)(a− b)2

+ 1 =

(1− aba− b

)2

,

Our inequality is equivalent to(1− aba− b

)2

+

(1− bcb− c

)2

+

(1− cac− a

)2

≥ 2.

This inequality has been already proved in Example 2.

7. Let a, b, c be positive real numbers such that abc = 1. Prove that

a+ 3

(a+ 1)2+

b+ 3

(b+ 1)2+

c+ 3

(c+ 1)2≥ 3.

Solution: Using the substitution as Example 3, we have

a+ 3

(a+ 1)2=

1− x1 + x

+ 3(1− x1 + x

+ 1

)2 =1

2(x2 + 3x+ 2).

Hence, our inequality is equivalent to x2 + y2 + z2 + 3(x+ y+ z) ≥ 0, orx2 + y2 + z2 ≥ 3xyz. By AM-GM inequality, we have

x2 + y2 + z2 ≥ 3 3√x2y2z2 ≥ 3 |xyz| (since |xyz| ≤ 1)

≥ 3xyz.

8. Let a, b, c be real numbers such that a, b, c 6= 1 and abc = 1. Prove that

a2

(a− 1)2+

b2

(b− 1)2+

c2

(c− 1)2≥ 1.

Solution: By the same subsitution as Example 3, put

a

a− 1=

1

1− 1

a

=1

1− 1 + x

1− x

=1

2

(1− 1

x

),

183

Page 185: Олимпиадад бэлтгэгчдэд тусламж (Тэнцэтгэл биш 1)

Hence, our inequality is equivalent to(1− 1

x

)2

+

(1− 1

y

)2

+

(1− 1

z

)2

≥ 4,

or1

x2+

1

y2+

1

z2− 2

(1

x+

1

y+

1

z

)− 1 ≥ 0.

Since x + y + z + xyz = 0 and x, y, z 6= 0 (since a, b, c 6= 1), we have1

xy+

1

yz+

1

zx= −1. Hence

1

x2+

1

y2+

1

z2=

(1

x+

1

y+

1

z

)2

− 2

(1

xy+

1

yz+

1

zx

)=

(1

x+

1

y+

1

z

)2

+ 2.

It suffices to show that(1

x+

1

y+

1

z

)2

− 2

(1

x+

1

y+

1

z

)+ 1 ≥ 0.

which is just simply to (1

x+

1

y+

1

z− 1

)2

≥ 0.

184

Page 186: Олимпиадад бэлтгэгчдэд тусламж (Тэнцэтгэл биш 1)

Chapter 2

Problems

83.1. Let a, b, c be the sidelengths of a triangle. Prove that

a2b(a− b) + b2c(b− c) + c2a(c− a) ≥ 0.

(IMO 1983)83.2. Show that for any positive reals a, b, c, d, e, f , we have

ab

a+ b+

cd

c+ d+

ef

e+ f≤ (a+ c+ e)(b+ d+ f)

a+ b+ c+ d+ e+ f.

(United Kingdom 1983)84.1. Let a1, a2, . . . , an > 0, n ≥ 2. Prove that

a21a2

+a22a3

+ · · ·+a2n−1an

+a2na1≥ a1 + a2 + · · ·+ an.

(China 1984)84.2. Let x, y, z be nonnegative real numbers such that x+ y + z = 1. Provethat

0 ≤ xy + yz + zx− 2xyz ≤ 7

27.

(IMO 1984)84.3. Prove that for any a, b > 0, we have that

(a+ b)2

2+a+ b

4≥ a√b+ b

√a.

(Russia 1984)85.1. Let x1, x2, . . . , xn be real numbers from the interval [0, 2]. Prove that

n∑i,j=1

|xi − xj | ≤ n2.

When do we have equality?(United Kingdom 1985)

86.1. Find the maximum value of the constant c such that for any x1, x2, . . . , xn >0 for which xk+1 ≥ x1 + x2 + · · ·+ xk for any k, the inequality

√x1 +

√x2 + · · ·+

√xn ≤ c

√x1 + x2 + · · ·+ xn

185

Page 187: Олимпиадад бэлтгэгчдэд тусламж (Тэнцэтгэл биш 1)

also holds for any n.(IMO Shorlist 1986)

86.2. Let n be a positive integer. Prove that

|sin 1|+ |sin 2|+ · · ·+ |sin 3n| > 8

5n.

(Russia 1986)86.3. Show that for all positive real numbers a1, a2, . . . , an, we have

1

a1+

2

a1 + a2+ · · ·+ n

a1 + a2 + · · ·+ an≤ 4

(1

a1+

1

a2+ · · ·+ 1

an

).

(Russia 1986)86.4. Find the maximum value of

x2y + y2z + z2x

for reals x, y, z with x+ y + z = 0 and x2 + y2 + z2 = 6.(United Kingdom 1986)

87.1. Prove that if x, y, z are real numbers such that x2 + y2 + z2 = 2, then

x+ y + z ≤ xyz + 2.

(IMO Shorlist 1987)88.1. Show that

(1 + x)n ≥ (1− x)n + 2nx(1− x2)n−12

for all 0 ≤ x ≤ 1 and all positive integer n.(Ireland 1988)

88.2. Given a triangle ABC. Prove that

2

(sinA

A+

sinB

B+

sinC

C

)≤(

1

B+

1

C

)sinA+

(1

C+

1

A

)sinB+

(1

A+

1

B

)sinC.

(Russia 1988)89.1. Let x1, x2, . . . , xn be positive real numbers, and let S = x1+x2+· · ·+xn.Prove that

(1 + x1)(1 + x2) · · · (1 + xn) ≤ 1 + S +S2

2!+ · · ·+ Sn

n!.

(APMO 1989)

89.2. Let x, y, z be real numbers such that 0 < x < y < z <π

2. Prove that

π

2+ 2 sinx cos y + 2 sin y cos z > sin 2x+ sin 2y + sin 2z.

(Iberoamerica 1989)89.3. Let a, b, c be the sidelengths of a triangle. Prove that∣∣∣∣a− ba+ b

+b− cb+ c

+c− ac+ a

∣∣∣∣ < 1

16.

186

Page 188: Олимпиадад бэлтгэгчдэд тусламж (Тэнцэтгэл биш 1)

(Iberoamerica 1989)89.4. Find the least possible value of

(x+ y)(y + z)

for positive reals x, y, z satisfying xyz(x+ y + z) = 1.(Russia 1989)

90.1. Let a, b, c, d be positive real numbers such that ab + bc + cd + da = 1.Prove that

a3

b+ c+ d+

b3

c+ d+ a+

c3

d+ a+ b+

d3

a+ b+ c≥ 1

3.

(IMO Shortlist 1990)90.2. Show that

x4 > x− 1

2

for all real x.(Russia 1990)

90.3. Let x1, x2, . . . , xn be positive reals with sum 1. Show that

x21x1 + x2

+x22

x2 + x3+ · · ·+ x2n

xn + x1≥ 1

2.

(Russia 1990)90.4. Show that√

x2 − xy + y2 +√y2 − yz + z2 ≥

√z2 + zx+ x2

for any positive real numbers x, y, z.(United Kingdom 1990)

91.1. Let a1, a2, . . . , an and b1, b2, . . . , bn be positive real numbers such thata1 + a2 + · · ·+ an = b1 + b2 + · · ·+ bn. Prove that

a21a1 + b1

+a22

a2 + b2+ · · ·+ a2n

an + bn≥ a1 + a2 + · · ·+ an

2.

(APMO 1991)91.2. Given positive real numbers a, b, c satisfying a+ b+ c = 1, show that

(1 + a) (1 + b) (1 + c) ≥ 8 (1− a) (1− b) (1− c) .

(Russia 1991)91.3. Show that

(x+ y + z)2

3≥ x√yz + y

√zx+ z

√xy

for all nonnegative reals x, y, z.(Russia 1991)

91.4. The real numbers x1, x2, . . . , x1991 satisfy

|x1 − x2|+ |x2 − x3|+ · · ·+ |x1990 − x1991| = 1991.

187

Page 189: Олимпиадад бэлтгэгчдэд тусламж (Тэнцэтгэл биш 1)

Denote sn =x1 + x2 + . . .+ xn

n. What is the maximum possible value of

|s1 − s2|+ |s2 − s3|+ . . .+ |s1990 − s1991|?

(Russia 1991)91.5. A triangle has sides a, b, c with sum 2. Show that

a2 + b2 + c2 + 2abc < 2.

(United Kingdom 1991)91.6. Prove the inequality

x2y

z+y2z

x+z2x

y≥ x2 + y2 + z2

for any positive real numbers x, y, z with x ≥ y ≥ z.(Vietnam 1991)

92.1. For every integer n ≥ 2, find the smallest positive number λ = λ(n)

such that if 0 ≤ a1, a2, . . . , an ≤1

2, b1, b2, . . . , bn > 0, and a1 + a2 + · · ·+ an =

b1 + b2 + · · ·+ bn = 1, then

b1b2 · · · bn ≤ λ(a1b1 + a2b2 + · · ·+ anbn).

(China 1992)92.2. Show that

x4 + y4 + z2 ≥ 2√

2xyz

for all positive reals x, y, z.(Russia 1992)

92.3. Show that for any real numbers x, y > 1, we have

x2

y − 1+

y2

x− 1≥ 8.

(Russia 1992)92.4. Positive real numbers a, b, c satisfy a ≥ b ≥ c. Prove that

a2 − b2

c+c2 − b2

a+a2 − c2

b≥ 3a− 4b+ c.

(Ukraine 1992)92.5. Let x, y, z, w be positive real numbers. Prove that

12

x+ y + z + w≤∑sym

1

x+ y≤ 3

4

(1

x+

1

y+

1

z+

1

w

).

(United Kingdom 1992)93.1. Let a, b, c, d be positive real numbers. Prove that

a

b+ 2c+ 3d+

b

c+ 2d+ 3a+

c

d+ a+ 3b+

d

a+ 2b+ 3c≥ 2

3.

(IMO Shortlist 1993)

188

Page 190: Олимпиадад бэлтгэгчдэд тусламж (Тэнцэтгэл биш 1)

93.2. Let a, b, c ∈ [0, 1]. Prove that

a2 + b2 + c2 ≤ a2b+ b2c+ c2a+ 1.

(Italia 1993)93.3. Let x, y, u, v be positive real numbers. Prove that

xy + xv + yu+ uv

x+ y + u+ v≥ xy

x+ y+

uv

u+ v.

(Poland 1993)93.4. If the equation x4 + ax3 + 2x2 + bx + 1 = 0 has at least one real root,then a2 + b2 ≥ 8.

(Tournament of the Towns 1993)93.5. Let x1, x2, x3, x4 be real numbers such that

1

2≤ x21 + x22 + x23 + x24 ≤ 1.

Find the largest and the smallest values of the expression

A = (x1 − 2x2 + x3)2 + (x2 − 2x3 + x4)

2 + (x2 − 2x1)2 + (x3 − 2x4)

2.

(Vietnam 1993)94.1. Let a1, a2, . . . , an be a sequence of positive real numbers satisfyinga1 + · · ·+ ak ≥

√k for all k = 1, 2, . . . , n. Prove that

a21 + a22 + · · ·+ a2n >1

4

(1 +

1

2+ · · ·+ 1

n

).

(USA 1994)95.1. Prove that for any positive real numbers x, y and any positive integersm,n,

(m−1)(n−1)(xm+n + ym+n

)+(m+ n− 1) (xmyn + xnym) ≥ mn

(xm+n−1y + xym+n−1) .

(Austrian-Polish Competition 1995)95.2. Let a, b, c, d be positive real numbers. Prove that

a+ c

a+ b+b+ d

b+ c+c+ a

c+ d+d+ b

d+ a≥ 4.

(Baltic Way 1995)95.3. Let x, y, z be positive real numbers. Prove that

xxyyzz ≥ (xyz)x+y+z

3 .

(Canada 1995)95.4. If a, b, c are positive real numbers such that abc = 1, then

1

a3 (b+ c)+

1

b3 (c+ a)+

1

c3 (a+ b)≥ 3

2.

(IMO 1995)

189

Page 191: Олимпиадад бэлтгэгчдэд тусламж (Тэнцэтгэл биш 1)

95.5. Suppose that x1, x2, . . . , xn are real numbers satisfying |xi − xi+1| < 1and xi ≥ 1 for i = 1, 2, . . . , n (xn+1 = x1). Prove the following inequality

x1x2

+x2x3

+ · · ·+ xnx1

< 2n− 1.

(India 1995)95.6. (a) Find the maximum value of the expression x2y−y2x when 0 ≤ x ≤ 1and 0 ≤ y ≤ 1.(b) Find the maximum value of the expression x2y+y2z+z2x−y2x−z2y−x2zwhen 0 ≤ x ≤ 1, 0 ≤ y ≤ 1 and 0 ≤ z ≤ 1.

(United Kingdom 1995)95.7. Let a, b, c be real numbers satisfying a < b < c, a + b + c = 6 andab+ bc+ ca = 9. Prove that

0 < a < 1 < b < 3 < c < 4.

(United Kingdom 1995)95.8. Find the greatest constant k such that for any positive integer n whichis not a square, ∣∣(1 +

√n)

sin(π√n)∣∣ > k.

(Vietnam (IMO training camp) 1995)96.1. Let m and n be positive integers such that n ≤ m. Prove that

2n · n! ≤ (m+ n)!

(m− n)!≤ (m2 +m)n.

(APMO 1996)96.2. Let a, b, c be the lengths of the sides of a triangle. Prove that

√a+ b− c+

√b+ c− a+

√c+ a− b ≤

√a+√b+√c,

and determine when equality occurs.(APMO 1996)

96.3. The real numbers x, y, z, t satisfy the equalities x + y + z + t = 0 andx2 + y2 + z2 + t2 = 1. Prove that

−1 ≤ xy + yz + zt+ tx ≤ 0.

(Austrian-Polish Competition 1996)96.4. Let a, b, c be positive real numbers, such that a+ b+ c =

√abc. Prove

thatab+ bc+ ca ≥ 9(a+ b+ c).

(Belarus 1996)96.5. Suppose n ∈ N, x0 = 0, xi > 0 for i = 1, 2, . . . , n, and x1+x2+· · ·+xn =1. Prove that

1 ≤n∑i=1

xi√1 + x0 + · · ·+ xi−1 ·

√xi + · · ·+ xn

2.

(China 1996)

190

Page 192: Олимпиадад бэлтгэгчдэд тусламж (Тэнцэтгэл биш 1)

96.6. (a) Find the minimum value of xx for x a positive real number.(b) If x and y are positive real numbers, show that xy + yx > 1.

(France 1996)96.7. Let a, b, c be positive real numbers such that abc = 1. Prove that

ab

a5 + b5 + ab+

bc

b5 + c5 + bc+

ca

c5 + a5 + ca≤ 1.

(IMO Shortlist 1996)96.8. Let a and b be positive real numbers with a+ b = 1. Prove that

a2

a+ 1+

b2

b+ 1≥ 1

3.

(Hungary 1996)96.9. Prove the following inequality for positive real numbers x, y, z,

(xy + yz + zx)

[1

(y + z)2+

1

(z + x)2+

1

(x+ y)2

]≥ 9

4.

(Iran 1996)96.10. Let n ≥ 2 be a fixed natural number and let a1, a2, . . . , an be positivenumbers whose sum is 1. Prove that for any positive numbers x1, x2, . . . , xnwhose sum is 1,

2∑

1≤i<j≤nxixj ≤

n− 2

n− 1+

n∑i=1

aix2i

1− ai,

and determine when equality holds.(Poland 1996)

96.11. Let a, b, c be real numbers such that a+ b+ c = 1. Prove that

a

a2 + 1+

b

b2 + 1+

c

c2 + 1≤ 9

10.

(Poland 1996)96.12. Let x1, x2, . . . , xn, xn+1 be positive reals such that x1 +x2 + · · ·+xn =xn+1. Prove that

n∑i=1

√xi(xn+1 − xi) ≤

√√√√ n∑i=1

xn+1(xn+1 − xi).

(Romania 1996)96.13. Let x, y, z be real numbers. Prove that the following conditions areequivalent

(i) x > 0, y > 0, z > 0 and1

x+

1

y+

1

z≤ 1;

(ii) for every quadrilateral with sides a, b, c, d, a2x+ b2y + c2z > d2.(Romania 1996)

96.14. Let a, b, c be positive real numbers.(a) Prove that 4(a3 + b3) ≥ (a+ b)3.(b) Prove that 9(a3 + b3 + c3) ≥ (a+ b+ c)3.

191

Page 193: Олимпиадад бэлтгэгчдэд тусламж (Тэнцэтгэл биш 1)

(United Kingdom 1996)96.15. Let a, b, c, d be positive real numbers such that

2(ab+ ac+ ad+ bc+ bd+ cd) + abc+ bcd+ cda+ dab = 16.

Prove that

a+ b+ c+ d ≥ 2

3(ab+ ac+ ad+ bc+ bd+ cd).

(Vietnam 1996)96.16. Prove that for any real numbers a, b, c,

(a+ b)4 + (b+ c)4 + (c+ a)4 ≥ 4

7(a4 + b4 + c4).

(Vietnam 1996)97.1. a, b, c be positive numbers such that abc = 1. Prove that

1

1 + a+ b+

1

1 + b+ c+

1

1 + c+ a≤ 1

a+ 2+

1

b+ 2+

1

c+ 2.

(Bulgaria 1997)97.2. Prove that

1

1999<

1

2· 3

4· · · 1997

1998<

1

44.

(Canada 1997)97.3. Let x1, x2, . . . , x1997 be real numbers satisfying the following conditions

(a) − 1√3≤ xi ≤

√3 for i = 1, 2, . . . , 1997;

(b) x1 + x2 + · · ·+ x1997 = −318√

3.Determine the maximum value of x121 + x122 + · · ·+ x121997.

(China 1997)97.4. For each natural number n ≥ 2, determine the largest possible value ofthe expression

Vn = sinx1 cosx2 + sinx2 cosx3 + · · ·+ sinxn cosx1,

where x1, x2, . . . , xn are arbitrary real numbers.(Czech-Slovak 1997)

97.5. Let x, y, z be positive real numbers. Prove that

xyz(x+ y + z +

√x2 + y2 + z2

)(x2 + y2 + z2) (xy + yz + zx)

≤ 3 +√

3

9.

(Hong Kong 1997)97.6. Given x1, x2, x3, x4 are positive real numbers such that x1x2x3x4 = 1.Prove that

x31 + x32 + x33 + x34 ≥ max

{x1 + x2 + x3 + x4,

1

x1+

1

x2+

1

x3+

1

x4

}.

(Iran 1997)

192

Page 194: Олимпиадад бэлтгэгчдэд тусламж (Тэнцэтгэл биш 1)

97.7. Let a, b, c be nonnegative real numbers such that a+ b+ c ≥ abc. Provethat

a2 + b2 + c2 ≥ abc.

(Ireland 1997)97.8. Let a, b, c be positive real numbers. Prove that

(b+ c− a)2

(b+ c)2 + a2+

(c+ a− b)2

(c+ a)2 + b2+

(a+ b− c)2

(a+ b)2 + c2≥ 3

5.

(Japan 1997)97.9. Let a1, . . . , an be positive numbers, and define

A =a1 + · · ·+ an

n, G = n

√a1 · · · an, H =

n

a−11 + · · ·+ a−1n.

(a) If n is even, show thatA

H≤ −1 + 2

(A

G

)n.

(b) If n is odd, show thatA

H≤ −n− 2

n+

2(n− 1)

n

(A

G

)n.

(Korea 1997)97.10. For any positive real numbers x, y, z such that xyz = 1, prove theinequality

x9 + y9

x6 + x3y3 + y6+

y9 + z9

y6 + y3z3 + z6+

z9 + x9

z6 + z3z3 + x6≥ 2.

(Romania 1997)97.11. Let a, b, c be positive real numbers. Prove that

a2

a2 + 2bc+

b2

b2 + 2ca+

c2

c2 + 2ab≥ 1 ≥ bc

a2 + 2bc+

ca

b2 + 2ca+

ab

c2 + 2ab.

(Romania 1997)97.12. Show that if 1 < a < b < c, then

loga(loga b) + logb(logb c) + logc(logc a) > 0.

(Russia 1997)97.13. Prove that for x, y, z ≥ 2,

(y3 + x)(z3 + y)(x3 + z) ≥ 125xyz.

(Saint Petersburg 1997)97.14. Given an integer n ≥ 2, find the minimal value of

x51x2 + x3 + · · ·+ xn

+x52

x3 + · · ·+ xn + x1+ · · ·+ x5n

x1 + x2 + · · ·+ xn−1,

for positive real numbers x1, . . . , xn subject to the condition x21 + · · ·+x2n = 1.(Turkey 1997)

97.15. Let x, y and z be positive real numbers.

193

Page 195: Олимпиадад бэлтгэгчдэд тусламж (Тэнцэтгэл биш 1)

(a) If x+ y + z ≥ 3, is it necessarily true that1

x+

1

y+

1

z≤ 3?

(b) If x+ y + z ≤ 3, is it necessarily true that1

x+

1

y+

1

z≥ 3?

(United Kingdom 1997)

97.16. Prove that, for all positive real numbers a, b, c,

1

a3 + b3 + abc+

1

b3 + c3 + abc+

1

c3 + a3 + abc≤ 1

abc.

(USA 1997)

98.1. Let a, b, c be positive real numbers. Prove that(1 +

a

b

)(1 +

b

c

)(1 +

c

a

)≥ 2

(1 +

a+ b+ c3√abc

).

(APMO 1998)

98.2. Let x1, x2, y1, y2 be real numbers such that x21 + x22 ≤ 1. Prove theinequality

(x1y1 + x2y2 − 1)2 ≥ (x21 + x22 − 1)(y21 + y22 − 1).

(Austrian-Polish Competition 1998)

98.3. If n ≥ 2 is an integer and 0 < a1 < a2 < . . . < a2n+1 are real numbers,prove the inequality

n√a1− n√a2+ n√a3−· · ·− n

√a2n+ n

√a2n+1 <

n√a1 − a2 + a3 − . . .− a2n + a2n+1.

(Balkan 1998)

98.4. Let a, b, c be positive real numbers. Prove that

a

b+b

c+c

a≥ a+ b

b+ c+b+ c

a+ b+ 1.

(Belarus 1998)

98.5. Let n be a natural number such that n ≥ 2. Prove that

1

n+ 1

(1 +

1

3+ . . .+

1

2n− 1

)>

1

n

(1

2+

1

4+ . . .+

1

2n

).

(Canada 1998)

98.6. Let n ≥ 2 be a positive integer and let x1, x2, . . . , xn be real numberssuch that

n∑i=1

x2i +n−1∑i=1

xixi+1 = 1.

For every positive integer k, 1 ≤ k ≤ n, determine the maximum value of |xk| .(China 1998)

98.7. Let a, b, c ≥ 1. Prove that

√a− 1 +

√b− 1 +

√c− 1 ≤

√c (ab+ 1).

(Hong Kong 1998)

194

Page 196: Олимпиадад бэлтгэгчдэд тусламж (Тэнцэтгэл биш 1)

98.8. Let a1, a2, . . . , an > 0 such that a1 + a2 + · · ·+ an < 1. Prove that

a1a2 · · · an (1− a1 − a2 − · · · − an)

(a1 + a2 + · · ·+ an) (1− a1) (1− a2) · · · (1− an)≤ 1

nn+1.

(IMO Shortlist 1998)98.9. Let a, b, c be positive real numbers such that abc = 1. Prove that

a3

(1 + b) (1 + c)+

b3

(1 + a) (1 + c)+

c3

(1 + a) (1 + b)≥ 3

4.

(IMO Shortlist 1998)

98.10. Let x, y, z > 1 such that1

x+

1

y+

1

z= 2. Prove that

√x+ y + z ≥

√x− 1 +

√y − 1 +

√z − 1.

(Iran 1998)98.11. Suppose that a1 < a2 < · · · < an are real numbers. Prove that

a1a42 + a2a

43 + · · ·+ an−1a

4n + ana

41 ≥ a2a41 + a3a

42 + · · ·+ ana

4n−1 + a1a

4n.

(Iran 1998)98.12. Show that if x is a nonzero real number, then

x8 − x5 − 1

x+

1

x4≥ 0.

(Ireland 1998)98.13. Prove that if a, b, c are positive real numbers, then

9

a+ b+ c≤ 2

(1

a+ b+

1

b+ c+

1

c+ a

),

and1

a+ b+

1

b+ c+

1

c+ a≤ 1

2

(1

a+

1

b+

1

c

).

(Ireland 1998)98.14. If x, y, z are positive real numbers such that x+ y + z = xyz, then

1√1 + x2

+1√

1 + y2+

1√1 + z2

≤ 3

2.

(Korea 1998)98.15. Let a, b, c, d, e, f be positive real numbers such that

a+ b+ c+ d+ e+ f = 1, ace+ bdf ≥ 1

108.

Prove that

abc+ bcd+ cde+ def + efa+ fab ≤ 1

36.

(Poland 1998)

195

Page 197: Олимпиадад бэлтгэгчдэд тусламж (Тэнцэтгэл биш 1)

98.16. Let a be a positive real numbers and let x1, x2, . . . , xn be positive realnumbers such that x1 + x2 + · · ·+ xn = 1. Prove that

ax1−x2

x1 + x2+

ax2−x3

x2 + x3+ · · ·+ axn−x1

xn + x1≥ n2

2.

(Serbia 1998)98.17. Find the minimum of the expression

F (x, y) =√

(x+ 1)2 + (y − 1)2+√

(x− 1)2 + (y + 1)2+√

(x+ 2)2 + (y + 2)2,

where x, y are real numbers.(Vietnam 1998)

98.18. Let n ≥ 2 and x1, x2, . . . , xn be positive real numbers satisfying

1

x1 + 1998+

1

x2 + 1998+ · · ·+ 1

xn + 1998=

1

1998.

Prove thatn√x1x2 · · ·xnn− 1

≥ 1998.

(Vietnam 1998)99.1. Let {an} be a sequence of real numbers such that ai+j ≤ ai + aj for alli, j. Prove that the following inequality holds

a1 +a22

+ · · ·+ ann≥ an.

(APMO 1999)99.2. Let a, b, c be positive real numbers such that a2 + b2 + c2 = 3. Provethat

1

1 + ab+

1

1 + bc+

1

1 + ca≥ 3

2.

(Belarus 1999)99.3. For any nonnegative real numbers x, y, z such that x+ y+ z = 1, provethe following inequality

x2y + y2z + z2x ≤ 4

27.

(Canada 1999)99.4. Let a, b, c be positive real numbers. Prove that

a

b+ 2c+

b

c+ 2a+

c

a+ 2b≥ 1.

(Czech-Slovak 1999)99.5. Let n ≥ 2 be a fixed integer. Find the least constant C such that theinequality ∑

1≤i<j≤nxixj(x

2i + x2j ) ≤ C(x1 + x2 + · · ·+ xn)4

holds for every x1, . . . , xn ≥ 0. For this constant C, characterize the instancesof equality.

196

Page 198: Олимпиадад бэлтгэгчдэд тусламж (Тэнцэтгэл биш 1)

(IMO 1999)99.6. For real numbers x1, x2, . . . , x6 ∈ [0, 1], prove the inequality

x31x52 + x53 + x54 + x55 + x56 + 5

+ · · ·+ x36x51 + x52 + x53 + x54 + x55 + 5

≤ 3

5.

(Ukraine 1999)99.7. Nonnegative real numbers p, q and r satisfy p+ q + r = 1. Prove that

7(pq + qr + rp) ≤ 2 + 9pqr.

(United Kingdom 1999)99.8. Let n > 3 and a1, a2, . . . , an be real numbers such that a1+a2+· · ·+an ≥n and a21 + a22 + · · ·+ a2n ≥ n2. Prove that max {a1, a2, . . . , an} ≥ 2.

(USA 1999)

99.9. Let a0, a1, . . . , an be numbers from the interval(

0,π

2

)such that

tan(a0 −

π

4

)+ tan

(a1 −

π

4

)+ · · ·+ tan

(an −

π

4

)≥ n− 1.

Prove thattan a0 tan a1 · · · tan an ≥ nn+1.

(USA 1999)99.10. Let x, y, z > 1. Prove that

xx2+2yzyy

2+2zxzz2+2xy ≥ (xyz)xy+yz+zx .

(USA (Shortlist) 1999)00.1. Let a, b be real numbers and a 6= 0. Prove that

a2 + b2 +1

a2+b

a≥√

3.

(Austria 2000)00.2. For all real numbers a, b, c ≥ 0 such that a+ b+ c = 1, prove that

2 ≤ (1− a2)2 + (1− b2)2 + (1− c2)2 ≤ (1 + a)(1 + b)(1 + c).

(Austrian-Polish Competition 2000)00.3. Let a, b, c, x, y, z be positive real numbers. Prove that

a3

x+b3

y+c3

z≥ (a+ b+ c)3

3 (x+ y + z).

(Belarus 2000)00.4. Suppose that the real numbers a1, a2, . . . , a100 satisfy(i) a1 ≥ a2 ≥ · · · ≥ a100 ≥ 0;(ii) a1 + a2 ≤ 100;(iii) a3 + a4 + · · ·+ a100 ≤ 100.Determine the maximum possible value of a21 + a22 + · · · + a2100, and find allpossible sequences a1, a2, . . . , a100 which achieve this maximum.

(Canada 2000)

197

Page 199: Олимпиадад бэлтгэгчдэд тусламж (Тэнцэтгэл биш 1)

00.5. Show that

3

√2 (a+ b)

(1

a+

1

b

)≥ 3

√a

b+

3

√b

a

for all positive real numbers a and b, and determine when the equality occurs.

(Czech-Slovak 2000)

00.6. Let a, b, c be positive real numbers such that abc = 1. Prove that

1 + ab2

c3+

1 + bc2

a3+

1 + ca2

b3≥ 18

a3 + b3 + c3.

(Hong Kong 2000)

00.7. Let x, y, and z denote positive real numbers, each less than 4. Prove

that at least one of the numbers1

x+

1

4− y,

1

y+

1

4− z, and

1

z+

1

4− xis greater

than or equal to 1.

(Hungary 2000)

00.8. Let a, b, c be positive real numbers such that abc = 1. Prove the in-equality (

a+1

b− 1

)(b+

1

c− 1

)(c+

1

a− 1

)≤ 1.

(IMO 2000)

00.9. Let x, y ≥ 0 with x+ y = 2. Prove that

x2y2(x2 + y2) ≤ 2.

(Ireland 2000)

00.10. The real numbers a, b, c, x, y, z satisfy a ≥ b ≥ c > 0 and x ≥ y ≥ z >0. Prove that

a2x2

(by + cz)(bz + cy)+

b2y2

(cz + ax)(cx+ az)+

c2z2

(ax+ by)(ay + bx)≥ 3

4.

(Korea 2000)

00.11. Let x, y, z be real numbers. Prove that

x2 + y2 + z2 ≥√

2 (xy + yz) .

(Macedonia 2000)

00.12. Let a, b, x, y, z be positive real numbers. Prove that

x

ay + bz+

y

az + bx+

z

ax+ by≥ 3

a+ b.

(MOSP 2000)

00.13. Let ABC be an acute triangle. Prove that(cosA

cosB

)2

+

(cosB

cosC

)2

+

(cosC

cosA

)2

+ 8 cosA cosB cosC ≥ 4.

(MOSP 2000)

198

Page 200: Олимпиадад бэлтгэгчдэд тусламж (Тэнцэтгэл биш 1)

00.14. Let a, b, c be positive real numbers such that min {a, b} ≥ c. Provethat √

c(a− c) +√c(b− c) ≤

√ab.

(MOSP 2000)00.15. Let a1, a2, . . . , an be nonnegative real numbers. Prove that

a1 + a2 + · · ·+ ann

− n√a1a2 · · · an ≥

≥ 1

2min

{(√a1 −

√a2)

2 , (√a2 −

√a3)

2 , . . . , (√an −

√a1)

2}.

(MOSP 2000)00.16. Let (an) be an infinite sequence of positive numbers such that

a11

+a42

+ · · ·+ ak2

k≤ 1

for all k. Prove thata11

+a22

+ · · ·+ ann< 2,

for all n.(MOSP 2000)

00.17. Let a, b, c be nonnegative real numbers such that ab + bc + ca = 1.Prove that

1

b+ c+

1

c+ a+

1

a+ b≥ 5

2.

(MOSP 2000)00.18. Let a1, a2, . . . , an be positive real numbers such that

1

a1+

1

a2+ · · ·+ 1

an≤ 1.

Prove that for any positive integer k,

(ak1 − 1)(ak2 − 1) · · · (akn − 1) ≥ (nk − 1)n.

(MOSP 2000)00.19. Let a, b, c be positive real numbers. Prove that

1

a+ b+

1

b+ c+

1

c+ a+

1

2 3√abc≥

(a+ b+ c+ 3

√abc)2

(a+ b)(b+ c)(c+ a).

(MOSP 2000)00.20. For any integer n ≥ 2, consider n−1 positive real numbers a1, a2, . . . , an−1having sum 1, and n real numbers b1, b2, . . . , bn. Prove that

b21 +b22a1

+b23a2

+ . . .+b2nan−1

≥ 2b1(b2 + b3 + . . .+ bn).

(Romania 2000)00.21. Positive real numbers x, y, z satisfy xyz = 1. Prove that the followinginequality holds

x2 + y2 + z2 + x+ y + z ≥ 2 (xy + yz + zx) .

199

Page 201: Олимпиадад бэлтгэгчдэд тусламж (Тэнцэтгэл биш 1)

(Russia 2000)00.22. Let x1, x2, . . . , xn be real numbers (n ≥ 2), satisfying the conditions−1 < x1 < x2 < · · · < xn < 1 and

x131 + x132 + · · ·+ x13n = x1 + x2 + · · ·+ xn.

Prove that

x131 y1 + x132 y2 + · · ·+ x13n yn < x1y1 + x2y2 + · · ·+ xnyn

for any real numbers y1 < y2 < · · · < yn.(Russia 2000)

00.23. Show that for all n ∈ N and x ∈ R,

sinn 2x+ (sinn x− cosn x)2 ≤ 1.

(Russia 2000)00.24. Let n ≥ 3 be a positive integer. Prove that for all positive real numbersa1, a2, . . . , an, we have

a1 + a22

· a2 + a32

· · · an + a12

≤ a1 + a2 + a3

2√

2· · · an + a1 + a2

2√

2.

(Saint Petersburg 2000)00.25. Let n ≥ 3 be an integer. Prove that for positive numbers x1 ≤ x2 ≤· · · ≤ xn,

xnx1x2

+x1x2x3

+ · · ·+ xn−1xnx1

≥ x1 + x2 + · · ·+ xn.

(Saint Petersburg 2000)

00.26. Let a, b, c, d be positive real numbers such that c2 + d2 =(a2 + b2

)3.

Prove thata3

c+b3

d≥ 1.

(Singapore 2000)00.27. Given that x, y, z are positive real numbers satisfying xyz = 32, findthe minimum value of

x2 + 4xy + 4y2 + 2z2.

(United Kingdom 2000)00.28. Prove that for any nonnegative real numbers a, b, c, the followinginequality holds

a+ b+ c

3− 3√abc ≤ max

{(√a−√b)2,(√

b−√c)2,(√c−√a)2}

.

(USA 2000)01.1. Let a, b, c ≥ 0 such that a + b + c ≥ abc. Prove that the followinginequality holds

a2 + b2 + c2 ≥√

3abc.

(Balkan 2001)

200

Page 202: Олимпиадад бэлтгэгчдэд тусламж (Тэнцэтгэл биш 1)

01.2. Let x1, x2, x3 be real numbers in [−1, 1], and let y1, y2, y3 be real num-bers in [0, 1). Find the maximum possible value of the expression

1− x11− x2y3

· 1− x21− x3y1

· 1− x31− x1y2

.

(Belarus 2001)01.3. Let x and y be any two real numbers. Prove that

3(x+ y + 1)2 + 1 ≥ 3xy.

Under what conditions does equality hold?(Colombia 2001)

01.4. Let n (n ≥ 2) be an integer and let a1, a2, . . . , an be positive realnumbers. Prove the inequality

(a31 + 1)(a32 + 1) · · · (a3n + 1) ≥ (a21a2 + 1) · · · (a2na1 + 1).

(Czech-Slovak-Polish 2001)01.5. Prove that for any positive real numbers a, b, c, the following inequalityholds

a√a2 + 8bc

+b√

b2 + 8ca+

c√c2 + 8ab

≥ 1.

(IMO 2001)01.6. Let a, b, c be positive real numbers such that abc = 1. Prove that

ab+cbc+aca+b ≤ 1.

(India 2001)01.7. Let x, y, z be positive real numbers such that xyz ≥ xy+yz+zx. Provethat

xyz ≥ 3 (x+ y + z) .

(India 2001)01.8. Prove that for any real numbers a, b, c the following inequality holds

(b+ c− a)2(c+ a− b)2(a+ b− c)2 ≥ (b2 + c2 − a2)(c2 + a2 − b2)(a2 + b2 − c2).

(Japan 2001)01.9. Let a, b, c be the sidelengths of an acute-angled triangle. Prove that

(a+ b+ c)(a2 + b2 + c2)(a3 + b3 + c3) ≥ 4(a6 + b6 + c6).

(Japan 2001)01.10. Prove that for any real numbers x1, x2, . . . , xn, y1, y2, . . . , yn such thatx21 + x22 + · · ·+ x2n = y21 + y22 + · · ·+ y2n = 1,

(x1y2 − x2y1)2 ≤ 2

(1−

n∑k=1

xkyk

).

(Korea 2001)

201

Page 203: Олимпиадад бэлтгэгчдэд тусламж (Тэнцэтгэл биш 1)

01.11. Prove that if a, b, c are positive real numbers, then√a4 + b4 + c4 +

√a2b2 + b2c2 + c2a2 ≥

√a3b+ b3c+ c3a+

√ab3 + bc3 + ca3.

(Korea 2001)01.12. Prove that for all a, b, c > 0,√

(a2b+ b2c+ c2a) (ab2 + bc2 + ca2) ≥ abc+ 3√

(a3 + abc) (b3 + abc) (c3 + abc).

(Korea 2001)01.13. Prove that if a, b, c > 0 have product 1, then

(a+ b)(b+ c)(c+ a) ≥ 4(a+ b+ c− 1).

(MOSP 2001)01.14. Show that the inequality

n∑i=1

ixi ≤(n

2

)+

n∑i=1

xii

holds for every integer n ≥ 2 and all real numbers x1, x2, . . . , xN ≥ 0.(Poland 2001)

01.15. Let a and b be positive real numbers in the interval (0, 1]. Prove that

1√1 + a2

+1√

1 + b2≤ 2√

1 + ab.

(Russia 2001)01.16. Let a, b, c, x, y, z be positive real numbers such that x + y + z = 1.Prove that

ax+ by + cz + 2√

(xy + yz + zx)(ab+ bc+ ca) ≤ a+ b+ c.

(Ukraine 2001)01.17. Let a, b, c be positive real numbers such that a + b + c ≥ abc. Provethat at least two of the inequalities

2

a+

3

b+

6

c≥ 6,

2

b+

3

c+

6

a≥ 6,

2

c+

3

a+

6

b≥ 6

are true.(USA 2001)

01.18. Prove that for any nonnegative real numbers a, b, c such that a2 + b2 +c2 + abc = 4, we have

0 ≤ ab+ bc+ ca− abc ≤ 2.

(USA 2001)01.19. Let x, y, z be positive real numbers satisfying

(i)1√2≤ z ≤ 1

2min

{x√

2, y√

3}

;

(ii) x+ z√

3 ≥√

6;(iii) y

√3 + z

√10 ≥ 2

√5.

202

Page 204: Олимпиадад бэлтгэгчдэд тусламж (Тэнцэтгэл биш 1)

Find the maximum of P (x, y, z) =1

x2+

2

y2+

3

z2.

(Vietnam 2001)

01.20. Let x, y, z be positive real numbers such that

(i)2

5≤ z ≤ min {x, y} ;

(ii) xz ≥ 4

15;

(iii) yz ≥ 1

5.

Determine the maximum possible value of

P (x, y, z) =1

x+

2

y+

3

z.

(Vietnam 2001)

01.21. Find the minimum value of the expression1

a+

2

b+

3

cwhere a, b, c are

positive real numbers such that 21ab+ 2bc+ 8ca ≤ 12.

(Vietnam 2001)

02.1. Let x, y, z be positive real numbers such that

1

x+

1

y+

1

z= 1.

Prove that

√x+ yz +

√y + zx+

√z + xy ≥ √xyz +

√x+√y +√z.

(APMO 2002)

02.2. Let a, b, c be positive real numbers. Prove that

a3

b2+b3

c2+c3

a2≥ a2

b+b2

c+c2

a.

(Balkan (Shortslit) 2002)

02.3. If a, b, c are positive real numbers such that abc = 2, then

a3 + b3 + c3 ≥ a√b+ c+ b

√c+ a+ c

√a+ b.

(Balkan (Shortlist) 2002)

02.4. Let a, b, c be real numbers such that a2+b2+c2 = 1. Prove the inequality

a2

1 + 2bc+

b2

1 + 2ca+

c2

1 + 2ab≥ 3

5.

(Bosnia and Herzegovina 2002)

02.5. Show that for any positive real numbers a, b, c, we have

a3

bc+b3

ca+c3

ab≥ a+ b+ c.

(Canada 2002)

203

Page 205: Олимпиадад бэлтгэгчдэд тусламж (Тэнцэтгэл биш 1)

02.6. Assume (P1, P2, . . . , Pn) (n ≥ 2) is an arbitrary permutation of (1, 2, . . . , n).Prove that

1

P1 + P2+

1

P2 + P3+ . . .+

1

Pn−1 + Pn>n− 1

n+ 2.

(China 2002)02.7. Let x, y be positive real numbers such that x+ y = 2. Prove that

x3y3(x3 + y3) ≤ 2.

(India 2002)02.8. For any positive real numbers a, b, c, show that the following inequalityholds

a

b+b

c+c

a≥ c+ a

c+ b+a+ b

a+ c+b+ c

b+ a.

(India 2002)02.9. Let x1, x2, . . . , xn be positive real numbers. Prove that

x11 + x21

+x2

1 + x21 + x22+ · · ·+ xn

1 + x21 + x22 + · · ·+ x2n<√n.

(India 2002)02.10. Let a, b, c, d be the positive real numbers such that

1

1 + a4+

1

1 + b4+

1

1 + c4+

1

1 + d4= 1.

Prove that abcd ≥ 3.(Latvia 2002)

02.11. Prove that for any positive real numbers a, b, c, we have

a

2a+ b+

b

2b+ c+

c

2c+ a≤ 1.

(Moldova 2002)02.12. Positive numbers α, β, x1, x2, . . . , xn (n ≥ 1) satisfy the conditionx1 + x2 + · · ·+ xn = 1. Prove that

x31αx1 + βx2

+x32

αx2 + βx3+ · · ·+ x3n

αxn + βx1≥ 1

n(α+ β).

(Moldova 2002)02.13. Let a, b, c be positive real numbers. Prove that(

2a

b+ c

) 23

+

(2b

c+ a

) 23

+

(2c

a+ b

) 23

≥ 3.

(MOSP 2002)02.14. If a, b, c ∈ (0, 1), prove that

√abc+

√(1− a)(1− b)(1− c) < 1.

(Romania 2002)

204

Page 206: Олимпиадад бэлтгэгчдэд тусламж (Тэнцэтгэл биш 1)

02.15. Given positive real numbers a, b, c and x, y, z, for which a+x = b+y =c+ z = 1. Prove that

(abc+ xyz)

(1

ay+

1

bz+

1

cx

)≥ 3.

(Russia 2002)02.16. Let x, y, z be positive real numbers with sum 3. Prove that

√x+√y +√z ≥ xy + yz + zx.

(Russia 2002)02.17. Let a1, a2, . . . , an and b1, b2, . . . , bn be real numbers between 1001 and2002 inclusive. Suppose a21 + · · ·+ a2n = b21 + · · ·+ b2n. Prove that

n∑i=1

a3ibi≤ 17

10

n∑i=1

a2i .

Determine when equality holds.(Singapore 2002)

02.18. Let a, b, c, d be real numbers contained in the interval

(0,

1

2

). Prove

that

a4 + b4 + c4 + d4

abcd≥ (1− a)4 + (1− b)4 + (1− c)4 + (1− d)4

(1− a)(1− b)(1− c)(1− d).

(Taiwan 2002)02.19. Let x, y, z be positive real numbers such that x2 + y2 + z2 = 1. Provethat

x2yz + y2zx+ z2xy ≤ 1

3.

(United Kingdom 2002)02.20. Let a, b, c be real numbers satisfying a2 + b2 + c2 = 9. Prove theinequality

2(a+ b+ c)− abc ≤ 10.

(Vietnam 2002)03.1. If a, b, c > −1, then

1 + a2

1 + b+ c2+

1 + b2

1 + c+ a2+

1 + c2

1 + a+ b2≥ 2.

(Laurentiu Panaitopol, Balkan 2003)03.2. Prove that if a, b and c are positive real numbers with sum 3, then

a

b2 + 1+

b

c2 + 1+

c

a2 + 1≥ 3

2.

(Bulgaria 2003)03.3. Let a, b, c, d be positive real numbers such that ab + cd = 1 and letx1, x2, x3, x4, y1, y2, y3, y4 be real numbers such that x21 + y21 = x22 + y22 =x23 + y23 = x24 + y24 = 1. Prove that the following inequality holds

(ay1 + by2 + cy3 + dy4)2 + (ax4 + bx3 + cx2 + dx1)

2 ≤ 2

(a2 + b2

ab+c2 + d2

cd

).

205

Page 207: Олимпиадад бэлтгэгчдэд тусламж (Тэнцэтгэл биш 1)

(China 2003)03.4. Let a1, a2, . . . , a2n be real numbers such that

2n−1∑i=1

(ai − ai+1)2 = 1.

Determine the maximum value of

(an+1 + an+2 + · · ·+ a2n)− (a1 + a2 + . . .+ an).

(China 2003)

03.5. Suppose x be a real number in the interval

[3

2, 5

]. Prove that

2√x+ 1 +

√2x− 3 +

√15− 3x < 2

√19.

(China 2003)03.6. Let x1, x2, . . . , x5 be nonnegative real numbers such that

5∑i=1

1

1 + xi= 1.

Prove that5∑i=1

xix2i + 4

≤ 1.

(China 2003)03.7. Find the greatest real number k such that, for any positive a, b, c witha2 > bc,

(a2 − bc)2 > k(b2 − ca)(c2 − ab).

(Japan 2003)03.8. Prove that in any acute triangle ABC,

cot3A+ cot3B + cot3C + 6 cotA cotB cotC ≥ cotA+ cotB + cotC.

(MOSP 2003)03.9. Let ai be positive real numbers, for all i = 1, 2, . . . , n, satisfying

a1 + a2 + · · ·+ an =1

a1+

1

a2+ · · ·+ 1

an.

Prove that

1

n− 1 + a1+

1

n− 1 + a2+ · · ·+ 1

n− 1 + an≤ 1.

(Vasile Cirtoaje, MOSP 2003)03.10. Let a, b, c be nonnegative real numbers satisfying a2 + b2 + c2 = 1.Prove that

1 ≤ a

1 + bc+

b

1 + ca+

c

1 + ab≤√

2.

(Faruk Zejnulahi, MOSP 2003)

206

Page 208: Олимпиадад бэлтгэгчдэд тусламж (Тэнцэтгэл биш 1)

03.11. Let a, b, c be positive real numbers so that abc = 1. Prove that

1 +3

a+ b+ c≥ 6

ab+ bc+ ca.

(Romania 2003)03.12. Let a, b, c be positive real numbers. Prove that

(2a+ b+ c)2

2a2 + (b+ c)2+

(2b+ c+ a)2

2b2 + (c+ a)2+

(2c+ a+ b)2

2c2 + (a+ b)2≤ 8.

(USA 2003)

03.13. Prove that for any a, b, c ∈(

0,π

2

), the following inequality holds

∑ sin a · sin(a− b) · sin(a− c)sin(b+ c)

≥ 0.

(USA 2003)04.1. Prove that

(a2 + 2)(b2 + 2)(c2 + 2) ≥ 9(ab+ bc+ ca)

for any positive real numbers a, b, c.(APMO 2004)

04.2. Let x, y, z, t be positive real numbers such that xyzt = 1. Prove that

1

(1 + x)2+

1

(1 + y)2+

1

(1 + z)2+

1

(1 + t)2≥ 1.

(China 2004)04.3. If a, b, c are positive real numbers, then

1 <a√

a2 + b2+

b√b2 + c2

+c√

c2 + a2≤ 3√

2

2.

(China 2004)04.4. Let n be a positive integer with n greater than one, and let a1, a2, . . . , anbe positive integers such that a1 < a2 < · · · < an and

1

a1+

1

a2+ · · ·+ 1

an≤ 1.

Prove that, for any real number x, the following inequality holds(1

a21 + x2+

1

a22 + x2+ · · ·+ 1

a2n + x2

)2

≤ 1

2· 1

a1(a1 − 1) + x2.

(China 2004)04.5. Find all real numbers k such that the following inequality

a3 + b3 + c3 + d3 + 1 ≥ k(a+ b+ c+ d)

holds for all real numbers a, b, c, d ≥ −1.

207

Page 209: Олимпиадад бэлтгэгчдэд тусламж (Тэнцэтгэл биш 1)

(China 2004)04.6. Determine the maximum constant λ such that

x+ y + z ≥ λ,

where x, y, z are positive real numbers with x√yz + y

√zx+ z

√xy ≥ 1.(China 2004)

04.7. Let a, b, c be positive real numbers. Determine the minimal value of thefollowing expression

a+ 3c

a+ 2b+ c+

4b

a+ b+ 2c− 8c

a+ b+ 3c.

(China 2004)04.8. Determine the largest constant M such that the following inequalityholds for all real numbers x, y, z,

x4 + y4 + z4 + xyz (x+ y + z) ≥M (xy + yz + zx)2 .

(Hellenic 2004)04.9. Let n ≥ 3 be an integer and let t1, t2, . . . , tn be positive real numberssuch that

n2 + 1 > (t1 + t2 + · · ·+ tn)

(1

t1+

1

t2+ · · ·+ 1

tn

).

Show that ti, tj , tk are the side lengths of a triangle for all i, j, k with 1 ≤ i <j < k ≤ n.

(IMO 2004)04.10. For a, b, c positive reals, find the minimum value of

b2 + c2

a2 + bc+c2 + a2

b2 + ca+a2 + b2

c2 + ab.

(India 2004)

04.11. Let x1, x2, . . . , xn be real numbers in the interval

(0,

1

2

). Prove that

n∏i=1

xi(n∑i=1

xi

)n ≤n∏i=1

(1− xi)[n∑i=1

(1− xi)

]n .

(India 2004)04.12. Prove that for all positive real numbers a, b, the following inequalityholds √

2a(a+ b)3 + b√

2(a2 + b2) ≤ 3(a2 + b2).

(Ireland 2004)04.13. If a, b, c are positive reals such that a+ b+ c = 1, prove that

1 + a

1− a+

1 + b

1− b+

1 + c

1− c≤ 2

(b

a+c

b+a

c

).

208

Page 210: Олимпиадад бэлтгэгчдэд тусламж (Тэнцэтгэл биш 1)

(Japan 2004)04.14. Let a, b be real numbers in the interval [0, 1]. Prove that

a√2b2 + 5

+b√

2a2 + 5≤ 2√

7.

(Lithuania 2004)04.15. Prove that for any positive real numbers a, b, c,∣∣∣∣a3 − b3a+ b

+b3 − c3

b+ c+c3 − a3

c+ a

∣∣∣∣ ≤ (a− b)2 + (b− c)2 + (c− a)2

4.

(Moldova 2004)04.16. Prove that if n > 3 and x1, x2, . . . , xn > 0 have product 1, then

1

1 + x1 + x1x2+

1

1 + x2 + x2x3+ · · ·+ 1

1 + xn + xnx1> 1.

(Russia 2004) 04.17. (a) Given real numbers a, b, c with a+ b+ c = 0, provethat

a3 + b3 + c3 > 0 if and only if a5 + b5 + c5 > 0.

(b) Given real numbers a, b, c, d with a+ b+ c+ d = 0, prove that

a3 + b3 + c3 + d3 > 0 if and only if a5 + b5 + c5 + d5 > 0.

(United Kingdom 2004)04.18. Let a, b, c be positive real numbers. Prove that

(a5 − a2 + 3)(b5 − b2 + 3)(c5 − c2 + 3) ≥ (a+ b+ c)3.

(USA 2004)04.19. Let x, y, z > 0 such that (x+ y+ z)3 = 32xyz. Find the minimum and

maximum ofx4 + y4 + z4

(x+ y + z)4.

(Vietnam 2004)05.1. For any positive real numbers a, b, c such that abc = 8, then

a2√(a3 + 1) (b3 + 1)

+b2√

(b3 + 1) (c3 + 1)+

c2√(c3 + 1) (a3 + 1)

≥ 4

3.

(APMO 2005)05.2. Let a, b, c, d be positive real numbers. Show that

1

a3+

1

b3+

1

c3+

1

d3≥ a+ b+ c+ d

abcd.

(Austria 2005)05.3. Let a, b, c be positive real numbers. Prove that

a2

b+b2

c+c2

a≥ a+ b+ c+

4(a− b)2

a+ b+ c.

(Balkan 2005)

209

Page 211: Олимпиадад бэлтгэгчдэд тусламж (Тэнцэтгэл биш 1)

05.4. If a, b, c are positive real numbers such that abc = 1, then the inequalityholds

a

a2 + 2+

b

b2 + 2+

c

c2 + 2≤ 1.

(Baltic Way 2005)05.5. Let a, b, c be positive real numbers. Prove that(

a2 + b+3

4

)(b2 + a+

3

4

)≥(

2a+1

2

)(2b+

1

2

).

(Belarus 2005)05.6. Given positive real numbers a, b, c such that a+ b+ c = 1. Prove that

a√b+ b

√c+ c

√a ≤ 1√

3.

(Bosnia and Hercegovina 2005)05.7. Let x, y be positive real numbers such that x3 + y3 = x− y. Prove that

x2 + 4y2 < 1.

(China 2005)

05.8. Let a, b, c be positive real numbers such that ab + bc + ca =1

3. Prove

that1

a2 − bc+ 1+

1

b2 − ca+ 1+

1

c2 − ab+ 1≤ 3.

(China 2005)05.9. Given positive real numbers a, b, c such that a + b + c = 1. Prove thatthe following inequality holds

10(a3 + b3 + c3)− 9(a5 + b5 + c5) ≥ 1.

(China 2005)05.10. Let ABC be an acute triangle. Determine the least value of thefollowing expression

P =cos2A

cosA+ 1+

cos2B

cosB + 1+

cos2C

cosC + 1.

(China 2005)

05.11. Let a, b, c be positive real numbers such that1

a+

1

b+

1

c= 1. Prove

that

(a− 1) (b− 1) (c− 1) ≥ 8.

(Croatia 2005)05.12. Let a, b, c > 0 such that abc = 1. Prove that

a

(a+ 1) (b+ 1)+

a

(b+ 1) (c+ 1)+

a

(c+ 1) (a+ 1)≥ 3

4.

(Czech-Slovak 2005)

210

Page 212: Олимпиадад бэлтгэгчдэд тусламж (Тэнцэтгэл биш 1)

05.13. If a, b, c are three positive real numbers such that ab + bc + ca = 1,prove that

3

√1

a+ 6b+

3

√1

b+ 6c+

3

√1

c+ 6a ≤ 1

abc.

(Germany 2005)05.14. Given positive real numbers x, y, z such that xyz ≥ 1. Prove that

x5 − x2

x5 + y2 + z2+

y5 − y2

y5 + z2 + x2+

z5 − z2

z5 + x2 + y2≥ 0.

(IMO 2005)05.15. Let a1 ≤ a2 ≤ · · · ≤ an be positive real numbers such that

a21 + a22 + · · ·+ a2nn

= 1,a1 + a2 + · · ·+ an

n= m,

where 1 ≥ m > 0. Prove that for all i satisfying ai ≤ m, we have

n− i ≥ n(m− ai)2.

(Iran 2005)05.16. If three nonnegative real numbers a, b, c satisfy the condition

1

a2 + 1+

1

b2 + 1+

1

c2 + 1= 2,

prove that

ab+ bc+ ca ≤ 3

2.

(Iran 2005)05.17. If x, y, z are real numbers satisfying xyz = −1, prove that

x4 + y4 + z4 + 3(x+ y + z) ≥ y2 + z2

x+z2 + x2

y+x2 + y2

z.

(Iran 2005)05.18. Let a, b, c be positive real numbers such that a+ b+ c = 1. Prove that

a3√

1 + b− c+ b 3√

1 + c− a+ c3√

1 + a− b ≤ 1.

(Japan 2005)05.19. For any real numbers x1, x2, . . . , xn with x21 + x22 + · · ·+ x2n = 1, provethat

x11 + x21

+x2

1 + x21 + x22+ · · ·+ xn

1 + x21 + x22 + · · ·+ x2n<

√n

2.

(Korea 2005)05.20. Given positive real numbers a, b, c such that a4 + b4 + c4 = 3. Provethat

1

4− ab+

1

4− bc+

1

4− ca≤ 1.

(Moldova 2005)

211

Page 213: Олимпиадад бэлтгэгчдэд тусламж (Тэнцэтгэл биш 1)

05.21. Let a, b, c ∈ [0, 1]. Prove that

a

bc+ 1+

b

ca+ 1+

c

ab+ 1≤ 2.

(Poland 2005)05.22. Let a, b, c be positive real numbers such that a+ b+ c = 1. Prove that

√ab (1− c) +

√bc (1− a) +

√ca (1− b) ≤

√2

3.

(Republic of Srpska 2005)05.23. Let a, b, c be positive real numbers such that abc ≥ 1. Prove that

1

1 + a+ b+

1

1 + b+ c+

1

1 + c+ a≤ 1.

(Romania 2005)05.24. Let n is a positive integer. Prove that if x is a positive real numbers,then

1 + xn+1 ≥ (2x)n

(1 + x)n−1.

(Russia 2005)05.25. Given positive real numbers x, y, z such that x2 + y2 + z2 = 1. Provethe following inequality

x

x3 + yz+

y

y3 + zx+

z

z3 + xy> 3.

(Russia 2005)05.26. Let a, b, c be positive real numbers. Prove that

a√b+ c

+b√c+ a

+c√a+ b

≥√

3

2(a+ b+ c).

(Serbia and Montenegro 2005)05.27. If a, b, c are positive real numbers such that ab + bc + ca = 1. Provethat

33

√1

abc+ 6 (a+ b+ c) ≤

3√

3

abc.

(Slovenia 2005)05.28. Let a1, a2, . . . , a95 be positive real numbers. Prove that

95∑k=1

ak ≤ 94 +

95∏k=1

max {1, ak} .

(Taiwan 2005)05.29. Let a, b, c be positive real numbers. Prove that(

a

b+b

c+c

a

)2

≥ (a+ b+ c)

(1

a+

1

b+

1

c

).

212

Page 214: Олимпиадад бэлтгэгчдэд тусламж (Тэнцэтгэл биш 1)

(United Kingdom 2005)05.30. Let a, b, c be positive real numbers. Prove that(

a

a+ b

)3

+

(b

b+ c

)3

+

(c

c+ a

)3

≥ 3

8.

(Vietnam 2005)06.1. Let x1, x2, . . . , xn be positive real numbers such that x1+x2+ · · ·+xn =1. Prove that (

n∑i=1

√xi

)(n∑i=1

1√1 + xi

)≤ n2√

n+ 1.

(China 2006)06.2. Let a, b, c be positive real numbers satisfying a+ b+ c = 1. Prove that

ab√ab+ bc

+bc√

bc+ ca+

ca√ca+ ab

≤√

2

2.

(China 2006)06.3. Suppose that a1, a2, . . . , an are real numbers with sum 0. Prove that thefollowing inequality holds

max1≤i≤n

a2i ≤n

3

n−1∑i=1

(ai − ai+1)2.

(China 2006)06.4. Let a, b, c be the sidelengths of a triangle. Prove that

√b+ c− a√

b+√c−√a

+

√c+ a− b

√c+√a−√b

+

√a+ b− c

√a+√b−√c≤ 3.

(IMO Shortlist 2006)06.5. Determine the least real number M such that the inequality∣∣ab(a2 − b2) + bc(b2 − c2) + ca(c2 − a2)

∣∣ ≤M(a2 + b2 + c2)2

holds for all real numbers a, b, c.(IMO 2006)

06.6. Let x1, x2, x3, y1, y2, y3, z1, z2, z3 be positive real numbers. Find themaximum value of real number A if

M = (x31 + x32 + x33 + 1)(y31 + y32 + y33 + 1)(z31 + z32 + z33 + 1),

and

N = A(x1 + y1 + z1)(x2 + y2 + z2)(x3 + y3 + z3),

then M ≥ N always holds, and find the condition that the equality holds.(Japan 2006)

06.7. Let a, b, c, d be real numbers with sum 0. Prove the inequality

(ab+ ac+ ad+ bc+ bd+ cd)2 + 12 ≥ 6(abc+ abd+ acd+ bcd).

213

Page 215: Олимпиадад бэлтгэгчдэд тусламж (Тэнцэтгэл биш 1)

(Kazakhstan 2006)06.8. Let a, b, c be sides of the triangle. Prove that

a2(b

c− 1

)+ b2

( ca− 1)

+ c2(ab− 1)≥ 0.

(Moldova 2006)06.9. Let a, b, c be positive real numbers with ab+ bc+ ca = abc. Prove that

a4 + b4

ab(a3 + b3)+

b4 + c4

bc(b3 + c3)+

c4 + a4

ca(c3 + a3)≥ 1.

(Poland 2006)06.10. Find the maximum value of

(x3 + 1)(y3 + 1),

for all real numbers x, y, satisfying the condition that x+ y = 1.(Romania 2006)

06.11. Let a, b, c be three positive real numbers with sum 3. Prove that

1

a2+

1

b2+

1

c2≥ a2 + b2 + c2.

(Romania 2006)

06.12. Consider real numbers a, b, c contained in the interval

[1

2, 1

]. Prove

that

2 ≤ a+ b

1 + c+b+ c

1 + a+c+ a

1 + b≤ 3.

(Romania 2006)06.13. Let a, b be positive real numbers. Determine the largest constant Msuch that for all k ∈ [0, π] , we have

1

ka+ b+

1

kb+ a≥ M

a+ b.

(Thailand 2006)06.14. If x, y, z are positive numbers satisfying the condition xy+yz+zx = 1,show that

27

4(x+ y)(y + z)(z + x) ≥

(√x+ y +

√y + z +

√z + x

)2 ≥ 6√

3.

(Turkey 2006)06.15. Let a, b, c be real numbers. Prove that the following inequality holds∑√

(a2 − ab+ b2)(b2 − bc+ c2) ≥ a2 + b2 + c2.

(VMEO 2006)07.1. Let x, y, z be positive real numbers such that

√x+√y+√z = 1. Prove

that the following inequality holds

x2 + yz√2x2(y + z)

+y2 + zx√2y2(z + x)

+z2 + xy√2z2(x+ y)

≥ 1.

214

Page 216: Олимпиадад бэлтгэгчдэд тусламж (Тэнцэтгэл биш 1)

(APMO 2007)

07.2. If a, b, c ∈ R such that abc = 1, then

a2+b2+c2+1

a2+

1

b2+

1

c2+2

(a+ b+ c+

1

a+

1

b+

1

c

)≥ 6+2

(b+ c

a+c+ a

b+a+ b

c

).

(Brazil 2007) 07.3. Given an integer n ≥ 2, find the largest constant C(n)for which the inequality

n∑i=1

xi ≥ C(n)∑

1≤j<i≤n

(2xixj +

√xixj

)

holds for all real numbers xi ∈ (0, 1) satisfying (1 − xi)(1 − xj) ≥1

4for

1 ≤ j < i ≤ n.

(Bulgaria 2007)

07.4. Let a, b, c be the sidelengths of a triangle such that a + b + c = 3.Determine the minimum value of

a2 + b2 + c2 +4abc

3.

(China 2007)

07.5. Let α, β be acute angles. Determine the maximal value of(1−√

tanα tanβ)2

cotα+ cotβ.

(China 2007)

07.6. Let a, b, c be positive real numbers such that abc = 1. Prove that for allk ≥ 2, we have

ak

a+ b+

bk

b+ c+

ck

c+ a≥ 3

2.

(China 2007)

07.7. Let a, b, c > 0 such that a+ b+ c = 1. Prove that

a2

b+b2

c+c2

a≥ 3(a2 + b2 + c2).

(Croatia 2007)

07.8. Let a, b, c, d be positive real numbers such that a+ b+ c+ d = 1. Provethat

6(a3 + b3 + c3 + d3) ≥ a2 + b2 + c2 + d2 +1

8.

(France 2007)

07.9. Let a, b, c be sides of a triangle, show that

(c+ a− b)4

a(a+ b− c)+

(a+ b− c)4

b(b+ c− a)+b(b+ c− a)

c(c+ a− b)≥ ab+ bc+ ca.

(Greece 2007)

215

Page 217: Олимпиадад бэлтгэгчдэд тусламж (Тэнцэтгэл биш 1)

07.10. Let a, b, c, d be real numbers such that

a2 ≤ 1, a2 + b2 ≤ 5, a2 + b2 + c2 ≤ 14, a2 + b2 + c2 + d2 ≤ 30.

Prove thata+ b+ c+ d ≤ 10.

(Hungary-Isarel 2007)07.11. Let a1, a2, . . . , a100 be nonnegative eral numbers such that a21 + a22 +· · ·+ a2100 = 1. Prove that

a21a2 + a22a3 + · · ·+ a2100a1 <12

25.

(IMO Shortlist 2007)07.12. Let n be a positive integer, and let x and y be positive real numberssuch that xn + yn = 1. Prove that(

n∑k=1

1 + x2k

1 + x4k

)(n∑k=1

1 + y2k

1 + y4k

)<

1

(1− x)(1− y).

(IMO Shortlist 2007)07.13. Real numbers a1, a2, . . . , an are given. For each i (1 ≤ i ≤ n) define

di = max {aj : 1 ≤ j ≤ i} −min {aj : i ≤ j ≤ n},

and letd = max {di : 1 ≤ i ≤ n}.

(a) Prove that for any real numbers x1 ≤ x2 ≤ · · · ≤ xn, we have

max {|xi − ai| : 1 ≤ i ≤ n} ≥ d

2.

(b) Show that there are real numbers x1 ≤ x2 ≤ · · · ≤ xn such that we haveequality in (a).

(IMO 2007)07.14. If x, y, z are positive real numbers, prove that the following inequalityholds

(x+ y + z)2(yz + zx+ xy)2 ≤ 3(y2 + yz + z2)(z2 + zx+ x2)(x2 + xy + y2).

(India 2007)07.15. Let a, b, c be distinct positive real numbers. Show that∣∣∣∣a+ b

a− b+b+ c

b− c+c+ a

c− a

∣∣∣∣ > 1.

(Iran 2007)07.16. Find the largest constant T such that for all nonnegative real numbersa, b, c, d, e satisfying a+ b = c+ d+ e, we have√

a2 + b2 + c2 + d2 + e2 ≥ T(√

a+√b+√c+√d+√e)2.

216

Page 218: Олимпиадад бэлтгэгчдэд тусламж (Тэнцэтгэл биш 1)

(Iran 2007)

07.17. Prove that for any positive real numbers a, b, c, we have

a+ b+ c

3≤√a2 + b2 + c2

3≤ 1

3

(bc

a+ca

b+ab

c

).

(Ireland 2007)

07.18. Let n ≥ 2 be a given integer. Determine

(a) the largest real cn such that

1

1 + a1+

1

1 + a2+ · · ·+ 1

1 + an≥ cn

holds for any positive numbers a1, a2, . . . , an with a1a2 · · · an = 1.

(b) the largest real dn such that

1

1 + 2a1+

1

1 + 2a2+ · · ·+ 1

1 + 2an≥ dn

holds for any positive numbers a1, a2, . . . , an with a1a2 · · · an = 1.

(Italy 2007)

07.19. If a, b are positive real numbers such that ab ≥ 1, then

1

(2a+ 3)2+

1

(2b+ 3)2≥ 2

5(2ab+ 3).

(Kiev 2007)

07.20. For all positive real numbers a, b, c, find all values of positive numberk such that the following inequality holds

a

c+ kb+

b

a+ kc+

c

b+ ka≥ 1

2007.

(Korea 2007)

07.21. Let a, b, c be positive real numbers. Prove that

1 +3

ab+ bc+ ca≥ 6

a+ b+ c.

(Macedonia 2007)

07.22. Let a, b, c, d be nonnegative real numbers such that a+ b+ c+ d = 4.Prove that

a2bc+ b2cd+ c2da+ d2ab ≤ 4.

(Middle Europe 2007)

07.23. Let a, b, c, d be positive real numbers in the interval

[1

2, 2

]and abcd =

1. Find the maximum value of(a+

1

b

)(b+

1

c

)(c+

1

d

)(d+

1

a

).

(Middle Europe 2007)

217

Page 219: Олимпиадад бэлтгэгчдэд тусламж (Тэнцэтгэл биш 1)

07.24. Let a1, a2, . . . , an be positive real numbers such that ai ≥1

ifor all

i = 1, 2, . . . , n. Prove the inequality

(a1 + 1)

(a2 +

1

2

)· · ·(an +

1

n

)≥ 2n

(n+ 1)!(1 + a1 + 2a2 + · · ·+ nan).

(Moldova 2007)07.25. Let a1, a2, . . . , an ∈ [0, 1]. If S = a31 + a32 + · · ·+ a3n, then prove that

a12n+ 1 + S − a31

+a2

2n+ 1 + S − a32+ · · ·+ an

2n+ 1 + S − a3n≤ 1

3.

(Moldova 2007)07.26. Let a, b, c be positive real numbers such that

a+ b+ c ≥ 1

a+

1

b+

1

c.

Prove that

a+ b+ c ≥ 3

a+ b+ c+

2

abc.

(Peru 2007)07.27. a, b, c, d are positive real numbers satisfying the following condition

1

a+

1

b+

1

c+

1

d= 4.

Prove that

3

√a3 + b3

2+

3

√b3 + c3

2+

3

√c3 + d3

2+

3

√d3 + a3

2≤ 2(a+ b+ c+ d)− 4.

(Poland 2007)07.28. Let x, y, z be nonnegative real numbers. Prove that the followinginequality holds

x3 + y3 + z3

3≥ xyz +

3

4|(x− y)(y − z)(z − x)| .

(Romania 2007)07.29. For n ∈ N, n ≥ 2, determine

maxn∏i=1

(1− xi), for xi ∈ R+, 1 ≤ i ≤ n,n∑i=1

x2i = 1.

(Romania 2007)07.30. Let a, b, c be positive real numbers such that

1

a+ b+ 1+

1

b+ c+ 1+

1

c+ a+ 1≥ 1.

Show that

a+ b+ c ≥ ab+ bc+ ca.

218

Page 220: Олимпиадад бэлтгэгчдэд тусламж (Тэнцэтгэл биш 1)

(Romania 2007)07.31. For n ∈ N, n ≥ 2, ai, bi ∈ R, 1 ≤ i ≤ n, such that

n∑i=1

a2i = 1,n∑i=1

b2i = 1, andn∑i=1

aibi = 0,

prove that (n∑i=1

ai

)2

+

(n∑i=1

bi

)2

≤ n.

(C. Lupu and T. Lupu, Romania 2007)07.32. Positive real numbers a, b, c satisfy a+ b+ c = 1. Show that

1

ab+ 2c2 + 2c+

1

bc+ 2a2 + 2a+

1

ca+ 2b2 + 2b≥ 1

ab+ bc+ ca.

(Turkey 2007)07.33. Let a, b, c be positive real numbers such that abc ≥ 1. Prove that(

a+1

a+ 1

)(b+

1

b+ 1

)(c+

1

c+ 1

)≥ 27

8,

and

27(a3+a2+a+1)(b3+b2+b+1)(c3+c2+c+1) ≥ 64(a2+a+1)(b2+b+1)(c2+c+1).

(Ukraine 2007)07.34. Show that for any real numbers a, b, c, then

(a2 + b2)2 ≥ (a+ b+ c)(b+ c− a)(c+ a− b)(a+ b− c).

(United Kingdom 2007)07.35. Given a triangle ABC. Determine the minimum value of

cos2A

2cos2

B

2

cos2C

2

+cos2

B

2cos2

C

2

cos2A

2

+cos2

C

2cos2

A

2

cos2B

2

.

(Vietnam 2007)08.1. Let a, b, c be real numbers such that a2 + b2 + c2 = 3. Prove that

a2

2 + b+ c2+

b2

2 + c+ a2+

c2

2 + a+ b2≥ (a+ b+ c)2

12.

(Baltic Way 2008)08.2. Suppose that a, b, c are positive real numbers with a2 + b2 + c2 = 1.Prove that

a5 + b5

ab(a+ b)+

b5 + c5

bc(b+ c)+

c5 + a5

ca(a+ b)≥ 3(ab+ bc+ ca)− 2.

(Bosnia 2008)

219

Page 221: Олимпиадад бэлтгэгчдэд тусламж (Тэнцэтгэл биш 1)

08.3. Let x, y, z be real numbers. Show that the following inequality holds

x2 + y2 + z2 − xy − yz − zx ≥ max

{3(x− y)2

4,3(y − z)2

4,3(z − x)2

4

}.

(Bosnia 2008)

08.4. Let a, b, c be positive real numbers. Prove that(1 +

4a

b+ c

)(1 +

4b

a+ c

)(1 +

4c

a+ b

)> 25.

(Bosnia 2008)

08.5. Let x, y, z be real numbers such that x+y+z = xy+yz+zx. Determinethe least value of the following expression

P =x

x2 + 1+

y

y2 + 1+

z

z2 + 1.

(Brazil 2008)

08.6. Let a, b, c be positive real numbers such that a+ b+ c = 1. Prove that

a− bca+ bc

+b− cab+ ca

+c− abc+ ab

≤ 3

2.

(Canada 2008)

08.7. Let a, b, c be nonnegative real numbers such that a + b + c = 1. Provethat √

a+(b− c)2

4+√b+√c ≤√

3.

(China 2008)

08.8. Let x, y, z be positive numbers. Find the minimal value of

(a)x2 + y2 + z2

xy + yz;

(b)x2 + y2 + 2z2

xy + yz.

(Croatia 2008)

08.9. Determine the smallest constant C such that the following inequality

1 + (x+ y)2 ≤ C(1 + x2)(1 + y2)

holds for any real numbers x, y.

(Germany 2008)

08.10. Prove that if a1, a2, . . . , an are positive integers, then the followinginequality (

a21 + a22 + · · ·+ a2na1 + a2 + · · ·+ an

) knt

≥ a1a2 · · · an

holds for k = max {a1, a2, . . . , an} and t = min {a1, a2, . . . , an} . When do wehave the equality?

(Greece 2008)

220

Page 222: Олимпиадад бэлтгэгчдэд тусламж (Тэнцэтгэл биш 1)

08.11. Let x, y, z be positive real numbers such that x2 + y2 + z2 = 3. Provethe inequality

3

2<

1 + y2

2 + x+

1 + z2

2 + y+

1 + x2

2 + z< 3.

(Greece 2008)08.12. Prove that for any positive real numbers a, b, c, d, we have the followinginequality

(a− b)(a− c)a+ b+ c

+(b− c)(b− d)

b+ c+ d+

(c− d)(c− a)

c+ d+ a+

(d− a)(d− b)d+ a+ b

≥ 0.

(Darij Grinberg, IMO Shortlist 2008)08.13. (i) If x, y, z are three real numbers, all different from 1, such thatxyz = 1, then prove that

x2

(x− 1)2+

y2

(y − 1)2+

z2

(z − 1)2≥ 1.

(ii) Prove that equality is achieved for infinitely many triples of rational num-bers x, y, z.

(IMO 2008)08.14. Let n ≥ 3 is an integer and let x1, x2, . . . , xn be real numbers suchthat xi > 1 for all i. Prove the following inequality

x1x2x3 − 1

+ · · ·+ xn−1xnx1 − 1

+xnx1x2 − 1

≥ 4n.

(Indonesia 2008)08.15. Let a, b, c be nonnegative real numbers, from which at least two arenonzero and satisfying the condition ab+ bc+ ca = 1. Prove that√

a3 + a+√b3 + b+

√c3 + c ≥ 2

√a+ b+ c.

(Iran 2008)08.16. Let x, y, z be positive real numbers such that x+y+z = 3. Prove that

x3

y3 + 8+

y3

z3 + 8+

z3

x3 + 8≥ 1

9+

2

27(xy + xz + yz).

(Iran 2008)08.17. Find the smallest real k such that for each x, y, z > 0, we have theinequality

x√y + y

√z + z

√x ≤ k

√(x+ y)(y + z)(z + x).

(Iran 2008)08.18. For any positive real numbers a, b, c, d such that a2 + b2 + c2 + d2 = 1,prove the inequality

a2b2cd+ b2c2da+ c2d2ab+ d2a2bc+ c2a2db+ d2b2ac ≤ 3

32.

(Ireland 2008)

221

Page 223: Олимпиадад бэлтгэгчдэд тусламж (Тэнцэтгэл биш 1)

08.19. Let a, b, c be positive real numbers satisfying abc = 1. Prove that

1

b(a+ b)+

1

c(b+ c)+

1

a(c+ a)≥ 3

2.

(Kazakhstan 2008)08.20. Let a, b, c be positive real numbers such that (a+ b) (b+ c) (c+ a) = 8.Prove the inequality

a+ b+ c

3≥ 27

√a3 + b3 + c3

3.

(Macedonia 2008)08.21. Let a, b, c, d be positive real numbers such that a ≥ b ≥ c ≥ d andabcd = 1. Prove that

1

a3 + 1+

1

b3 + 1+

1

c3 + 1≥ 3

abc+ 1.

(MathLinks Contest 2008)08.22. Let a, b, c be nonnegative real numbers satisfying ab + bc + ca = 3.Prove that

1

1 + a2(b+ c)+

1

1 + b2(c+ a)+

1

1 + c2(a+ b)≤ 3

1 + 2abc.

(MathLinks Contest 2008)08.23. Determine the least value of the expression

P = abc+1

abc

where a, b, c are positive real numbers satisfying a+ b+ c ≤ 3

2.

(Moldova 2008)08.24. Let a1, a2, . . . , an be positive real numbers such that a1+a2+· · ·+an ≤n

2. Determine the smallest value of the following expression

A =

√a21 +

1

a22+

√a22 +

1

a23+ · · ·+

√a2n +

1

a21.

(Moldova 2008)08.25. Find the maximum value of constant C such that the inequality

x3 + y3 + z3 + C(xy2 + yz2 + zx2) ≥ (C + 1)(x2y + y2z + z2x)

holds for any nonnegative real numbers x, y, z.(Mongolia 2008)

08.26. If a, b, c are nonnegative real numbers, then the inequality holds

4(√

a3b3 +√b3c3 +

√c3a3

)≤ 4c3 + (a+ b)3.

(Poland 2008)

222

Page 224: Олимпиадад бэлтгэгчдэд тусламж (Тэнцэтгэл биш 1)

08.27. For real numbers xi > 1, 1 ≤ i ≤ n, n ≥ 2, such that

x2ixi − 1

≥ S =

n∑j=1

xj , for all i = 1, 2, . . . , n,

find, with proof, supS.

(Romania 2008)

08.28. Show that for all integers n ≥ 1, we have

n

(1 +

1

2+ · · ·+ 1

n

)≥ (n+ 1)

(1

2+

1

3+ · · ·+ 1

n+ 1

).

(Romania 2008)

08.29. Let a, b, c be positive real numbers with ab+ bc+ ca = 3. Prove that

1

1 + a2(b+ c)+

1

1 + b2(c+ a)+

1

1 + c2(a+ b)≤ 1

abc.

(Romania 2008)

08.30. Determine the maximum value of real number k such that

(a+ b+ c)

(1

a+ b+

1

b+ c+

1

c+ a− k)≥ k

for all real numbers a, b, c ≥ 0 with a+ b+ c = ab+ bc+ ca.

(Romania 2008)

08.31. Let a, b ∈ [0, 1]. Prove the inequality

1

1 + a+ b≤ 1− a+ b

2+ab

3.

(Romania 2008)

08.32. Let n ≥ 3 is an odd integer. Determine the maximum value of thecyclic sum, for 0 ≤ xi ≤ 1, i = 1, 2, . . . , n,

E =√|x1 − x2|+

√|x2 − x3|+ · · ·+

√|xn − x1|.

(Romania 2008)

08.33. Let a, b, c be three positive real numbers satisfying abc = 8. Provethat

a− 2

a+ 1+b− 2

b+ 1+c− 2

c+ 1≤ 0.

(Romania 2008)

08.34. Let a1, a2, . . . , an be positive real numbers satisfying the conditionthat a1 + a2 + · · ·+ an = 1. Prove that

n∑j=1

aj1 + a1 + · · ·+ aj

<1√2.

(Romania 2008)

223

Page 225: Олимпиадад бэлтгэгчдэд тусламж (Тэнцэтгэл биш 1)

08.35. Let x, y, z be positive real numbers such that x+ y+ z = 1. Prove theinequality

1

yz + x+1

x

+1

xz + y +1

y

+1

xy + z +1

z

≤ 27

31.

(Serbia 2008)08.36. Determine the least value of the expression x2 + y2 + z2, where x, y, zare real numbers such that x3 + y3 + z3 − 3xyz = 1.

(United Kingdom 2008)08.37. If a, b, c, d are positive real numbers, then

(a+ b)(b+ c)(c+ d)(d+ a)(

1 +4√abcd

)4≥ 16abcd(1 + a)(1 + b)(1 + c)(1 + d).

(Ukraine 2008)08.38. Let a, b, c be positive real numbers such that a2 + b2 + c2 = 3. Provethe inequality√

a2

a2 + b+ c+

√b2

b2 + c+ a+

√c2

c2 + a+ b≤√

3.

(Ukraine 2008)08.39. Let x, y, z be distinct nonnegative real numbers. Prove that

1

(x− y)2+

1

(y − z)2+

1

(z − x)2≥ 4

xy + yz + zx.

(Vietnam 2008)

224

Page 226: Олимпиадад бэлтгэгчдэд тусламж (Тэнцэтгэл биш 1)

Chapter 3

Solutions

83.1. Let a, b, c be the sidelengths of a triangle. Prove that

a2b(a− b) + b2c(b− c) + c2a(c− a) ≥ 0.

(IMO 1983)

First solution: After setting a = y+ z, b = z + x, c = x+ y for x, y, z > 0, itbecomes

x3z + y3x+ z3y ≥ x2yz + xy2z + xyz2 orx2

y+y2

z+z2

x≥ x+ y + z.

However, an application of the Cauchy Schwarz Inequality gives

(y + z + x)

(x2

y+y2

z+z2

x

)≥ (x+ y + z)2.

Note that the equality holds if and only if a = b = c.

Second solution: Without loss of generality, we may assume that b is thesmallest number between a and c. Then we have two cases to consider: a ≥c ≥ b and c ≥ a ≥ b.For the former case, we have

a2b(a− b) = b(a− b)(a− c)2 + bc(2a− c)(a− b) ≥ bc(2a− c)(a− b).

Therefore, it suffices to prove that

b(2a− c)(a− b) + b2(b− c) + ca(c− a) ≥ 0,

or(2b− c)a2 − (2b− c)(b+ c)a+ b3 ≥ 0.

If c ≥ 2b then we are done because

(2b− c)a2 − (2b− c)(b+ c)a+ b3 = a(c− 2b)(b+ c− a) + b3 ≥ 0.

Conversely, if 2b ≥ c, then we have

(2b− c)a2 = (2b− c)(a− c)2 + c(2b− c)(2a− c) ≥ c(2b− c)(2a− c).

225

Page 227: Олимпиадад бэлтгэгчдэд тусламж (Тэнцэтгэл биш 1)

It now remains to check that

c(2b− c)(2a− c)− (2b− c)(b+ c)a+ b3 ≥ 0.

However, this is true because

c(2b− c)(2a− c)− (2b− c)(b+ c)a+ b3 = (2b− c)(c− b)a+ c3 − 2bc2 + b3

≥ (2b− c)(c− b)c+ c3 − 2bc2 + b3

= b(b− c)2 ≥ 0.

For the latter case, we have

a3b+ b3c+ c3a− (ab3 + bc3 + ca3) = (a− b)(c− b)(c− a)(a+ b+ c) ≥ 0,

and hence

2(a3b+b3c+c3a) ≥ (a3b+ab3)+(b3c+bc3)+(c3a+ca3) ≥ 2(a2b2+b2c2+c2a2),

from which we deduce that

a2b(a− b) + b2c(b− c) + c2a(c− a) ≥ 0,

as desired.?F?

83.2. Show that for any positive reals a, b, c, d, e, f , we have

ab

a+ b+

cd

c+ d+

ef

e+ f≤ (a+ c+ e)(b+ d+ f)

a+ b+ c+ d+ e+ f.

(United Kingdom 1983)

First solution: The inequality is equivalent to(a+ b

4− ab

a+ b

)+

(c+ d

4− cd

c+ d

)+

(e+ f

4− ef

e+ f

)≥

≥ a+ c+ e+ b+ d+ f

4− (a+ c+ e)(b+ d+ f)

a+ b+ c+ d+ e+ f,

or(a− b)2

a+ b+

(c− d)2

c+ d+

(e− f)2

e+ f≥ (a+ c+ e− b− d− f)2

a+ b+ c+ d+ e+ f.

Now, using the Cauchy Schwarz Inequality, we get

(a− b)2

a+ b+

(c− d)2

c+ d+

(e− f)2

e+ f≥ [(a− b) + (c− d) + (e− f)]2

(a+ b) + (c+ d) + (e+ f)

=(a+ c+ e− b− d− f)2

a+ b+ c+ d+ e+ f.

Therefore, the last inequality is valid and so, the problem is solved.

226

Page 228: Олимпиадад бэлтгэгчдэд тусламж (Тэнцэтгэл биш 1)

Second solution: According to the Cauchy Schwarz Inequality, we have[(b+ d+ f)2a+ (a+ c+ e)2b

](b+ a) ≥

[(b+ d+ f)

√ab+ (a+ c+ e)

√ba]2

= ab(a+ b+ c+ d+ e+ f)2.

This implies that

ab

a+ b≤ (b+ d+ f)2a+ (a+ c+ e)2b

(a+ b+ c+ d+ e+ f)2.

Similarly, we have

cd

c+ d≤ (b+ d+ f)2c+ (a+ c+ e)2d

(a+ b+ c+ d+ e+ f)2,

andef

e+ f≤ (b+ d+ f)2e+ (a+ c+ e)2f

(a+ b+ c+ d+ e+ f)2.

Adding up these three inequalities, we get

ab

a+ b+

cd

c+ d+

ef

e+ f≤ (b+ d+ f)2(a+ c+ e) + (a+ c+ e)2(b+ d+ e)

(a+ b+ c+ d+ e+ f)2

=(a+ c+ e)(b+ d+ f)

a+ b+ c+ d+ e+ f,

as desired.?F?

84.1. Let a1, a2, . . . , an > 0, n ≥ 2. Prove that

a21a2

+a22a3

+ · · ·+a2n−1an

+a2na1≥ a1 + a2 + · · ·+ an.

(China 1984)

Solution: From the AM-GM Inequality, for each i (an+1 = a1), we get

a2iai+1

+ ai+1 ≥ 2

√a2iai+1

· ai+1 = 2ai.

Setting i = 1, 2, . . . , n, we get the n similar inequalities and summing themup, we can get the desired result. Note that the equality holds if and only ifa1 = a2 = · · · = an.

?F?

84.2. Let x, y, z be nonnegative real numbers such that x+ y + z = 1. Provethat

0 ≤ xy + yz + zx− 2xyz ≤ 7

27.

(IMO 1984)

227

Page 229: Олимпиадад бэлтгэгчдэд тусламж (Тэнцэтгэл биш 1)

Solution: Let f(x, y, z) = xy + yz + zx − 2xyz. Without loss of generality,we may assume that 0 ≤ x ≤ y ≤ z ≤ 1. Since x + y + z = 1, this implies

that x ≤ 1

3. It follows that f(x, y, z) = (1− 3x)yz+ xyz+ zx+ xy ≥ 0, which

proves the left inequality. For the right part, applying the AM-GM Inequality,

we obtain yz ≤(y + z

2

)2

=

(1− x

2

)2

. Since 1− 2x ≥ 0, this implies that

f(x, y, z) = x(y+z)+yz(1−2x) ≤ x(1−x)+

(1− x

2

)2

(1−2x) =−2x3 + x2 + 1

4.

Our job is now to maximize a one-variable function F (x) =1

4(−2x3 +x2 + 1),

where x ∈[0,

1

3

]. Since F ′(x) =

3

2x

(1

3− x)≥ 0 on

[0,

1

3

], we conclude that

F (x) ≤ F(

1

3

)=

7

27for all x ∈

[0,

1

3

].

?F?

84.3. Prove that for any a, b > 0, we have that

(a+ b)2

2+a+ b

4≥ a√b+ b

√a.

(Russia 1984)

Solution: The AM-GM Inequality gives us(2a2 +

b

2

)+(

2b2 +a

2

)≥ 2a

√b+ 2b

√a,

and (2ab+

b

2

)+(

2ab+a

2

)≥ 2b√a+ 2a

√b.

By summing up these two inequalities, we get

2(a+ b)2 + (a+ b) ≥ 4a√b+ 4b

√a,

and then, we deduce that

(a+ b)2

2+a+ b

4≥ a√b+ b

√a,

as desired. Note that the equality holds if and only if a = b =1

4.

?F?

85.1. Let x1, x2, . . . , xn be real numbers from the interval [0, 2]. Prove that

n∑i,j=1

|xi − xj | ≤ n2.

When do we have equality?

228

Page 230: Олимпиадад бэлтгэгчдэд тусламж (Тэнцэтгэл биш 1)

(United Kingdom 1985)

Solution: Since the inequality is symmetric, we can assume that 0 ≤ x1 ≤x2 ≤ · · · ≤ xn ≤ 2. Accordingly, we find that

n∑i,j=1

|xi − xj | = 2

n∑1≤i<j≤n

|xi − xj | = 2

n∑1≤i<j≤n

(xj − xi) = 2

n∑i=1

(2i− 1− n)xi.

And so, we are left to prove

n∑i=1

(2i− 1− n)xi ≤n2

2.

If n = 2k (k ∈ N, k ≥ 1) , then we have

n∑i=1

(2i− 1− n)xi =

2k∑i=1

(i− 1− 2k)xi

=2k∑

i=k+1

(2i− 1− 2k)xi +k∑i=1

(2i− 1− 2k)xi

≤2k∑

i=k+1

(2i− 1− 2k)xi ≤ 22k∑

i=k+1

(2i− 1− 2k) =n2

2,

If n = 2k + 1 (k ∈ N) , we proceed the same way as the preceding case andobtain

n∑i=1

(2i− 1− n)xi = 2

2k+1∑i=1

(i− 1− k)xi

= 22k+1∑i=k+2

(i− 1− k)xi + 2k+1∑i=1

(i− 1− k)xi

≤ 22k+1∑i=k+2

(i− 1− k)xi ≤ 42k+1∑i=k+2

(i− 1− k)

= 2k(k + 1) <(2k + 1)2

2=n2

2.

Since the inequality holds for both cases n even and n odd, we conclude thatit holds for any value of n. In addition, from the two cases above, one can findthat the inequality can become equality only for n = 2k, and in this case, theequality is attained for x1 = x2 = · · · = xk = 0 and xk+1 = xk+2 = · · · =x2k = 2.

?F?

86.1. Find the maximum value of the constant c such that for any x1, x2, . . . , xn >0 for which xk+1 ≥ x1 + x2 + · · ·+ xk for any k, the inequality

√x1 +

√x2 + · · ·+

√xn ≤ c

√x1 + x2 + · · ·+ xn

229

Page 231: Олимпиадад бэлтгэгчдэд тусламж (Тэнцэтгэл биш 1)

also holds for any n.

(IMO Shorlist 1986)

Solution: First, let us see what happens if xk+1 and x1 + x2 + · · · + xk areequal for any k. For example, we can let n ≥ 2 and take x1 = 1, xk = 2k−2 forany k ≥ 2. In this case, we have xk+1 = x1 + x2 + · · · + xk for all k ≥ 1, andthus, we find that

c ≥

1 +

n∑k=2

(√2)k−2

√√√√1 +n∑k=2

2k−2

for any n ≥ 2. Taking the limit, we find that c ≥ 1 +√

2. Now, let us provethat 1 +

√2 works. We will prove the inequality

√x1 +

√x2 + · · ·+

√xn ≤

(1 +√

2)√

x1 + x2 + · · ·+ xn

by induction. For n = 1 and n = 2, it is clear. Suppose that it is true for nand we will prove that

√x1 +

√x2 + · · ·+

√xn +

√xn+1 ≤

(1 +√

2)√

x1 + x2 + · · ·+ xn + xn+1.

Of course, it is enough to prove that

√xn+1 ≤

(1 +√

2) (√

x1 + x2 + · · ·+ xn + xn+1 −√x1 + x2 + · · ·+ xn

),

which is equivalent to

√x1 + x2 + · · ·+ xn + xn+1 +

√x1 + x2 + · · ·+ xn ≤

(1 +√

2)√

xn+1.

But this one is obvious because x1 + x2 + · · ·+ xn ≤ xn+1.?F?

86.2. Let n be a positive integer. Prove that

|sin 1|+ |sin 2|+ · · ·+ |sin 3n| > 8

5n.

(Russia 1986)

Solution: We will prove first that the following inequality holds for any x ≥ 0

f(x) = |sinx|+ |sin(x+ 1)|+ |sin(x+ 2)| − 8

5> 0.

Since f(x) = f(x + π), it suffices to prove it for x ∈ [0, π], then for x > πwe can put x = kπ + r where k ≥ 1, k ∈ N and 0 < r < π, and thusf(x) = f(kπ + r) = f(r) > 0. Now, from the assumption that x ∈ [0, π], wewill consider two cases

230

Page 232: Олимпиадад бэлтгэгчдэд тусламж (Тэнцэтгэл биш 1)

The first case is when 0 ≤ x ≤ π−2. In this case, we have sinx ≥ 0, sin(x+2) ≥0 and sin(x+ 1) ≥ sin 1 > 0. Therefore

|sinx|+ |sin(x+ 1)|+ |sin(x+ 2)| = sinx+ sin(x+ 1) + sin(x+ 2)

= (1 + 2 cos 1) sin(x+ 1) ≥ (1 + 2 cos 1) sin 1 >8

5.

The second one is when π−2 ≤ x ≤ π. In this case, sinx ≥ 0 and sin(x+2) ≤ 0,our inequality is equivalent to

sinx+ |sin(x+ 1)| − sin(x+ 2) >8

5.

Denote t = cos(x+ 1) then −1 ≤ t ≤ − cos 1. We have

sinx+ |sin(x+ 1)| − sin(x+ 2) = |sin(x+ 1)| − 2 sin 1 cos(x+ 1)

=√

1− t2 − 2t sin 1 = g(t)

Since g(t) is concave, we have

g(t) ≥ min {g(−1), g (− cos 1)} = min {2 sin 1, sin 1 + 2 sin 1 cos 1} > 8

5.

Now, we will prove the original inequality by induction on n. For n = 1, it isclear. Suppose that the inequality holds for n and let us prove it for n + 1,that is to prove

|sin 1|+|sin 2|+· · ·+|sin 3n|+| sin(3n+1)|+| sin(3n+2)|+| sin(3n+3)| > 8

5(n+1).

However, this is trivial since from the inductive hypothesis, we have

|sin 1|+ |sin 2|+ · · ·+ |sin 3n| > 8

5n,

and from what we have shown above,

| sin(3n+ 1)|+ | sin[(3n+ 1) + 1]|+ | sin[(3n+ 1) + 2]| ≥ 8

5.

This completes our proof.?F?

86.3. Show that for all positive real numbers a1, a2, . . . , an, we have

1

a1+

2

a1 + a2+ · · ·+ n

a1 + a2 + · · ·+ an≤ 4

(1

a1+

1

a2+ · · ·+ 1

an

).

(Russia 1986)

Solution: We will show that the stronger inequality holds

n∑k=1

kk∑j=1

aj

≤ 2n∑k=1

1

ak.

231

Page 233: Олимпиадад бэлтгэгчдэд тусламж (Тэнцэтгэл биш 1)

Indeed, by the Cauchy Schwarz Inequality, we have

(a1 + · · ·+ ak)

(b21a1

+ · · ·+b2kak

)≥ (b1 + · · ·+ bk)

2,

where bi are arbitrary positive real numbers for all i = 1, 2, . . . , n. Therefore

kk∑j=1

aj

≤ k

(b1 + · · ·+ bk)2

(b21a1

+ · · ·+b2kak

),

from which we deduce that

n∑k=1

kk∑j=1

aj

≤n∑i=1

ciai,

where

ck =kb2k

(b1 + · · ·+ bk)2+

(k + 1)b2k(b1 + · · ·+ bk+1)2

+ · · ·+nb2k

(b1 + · · ·+ bn)2,

for all k = 1, 2, . . . , n. Choosing now bk = k, we get

ck =k3(k∑i=1

i

)2 +k2(k + 1)(k+1∑i=1

i

)2 + · · ·+ k2n(n∑i=1

i

)2

= 4k2n∑j=k

1

j(j + 1)2= 4k2

n∑j=k

[1

j(j + 1)− 1

(j + 1)2

]

≤ 4k2n∑j=k

[1

2j2+

1

2(j + 1)2− 1

(j + 1)2

]

= 2k2n∑j=k

[1

j2− 1

(j + 1)2

]= 2k2

[1

k2− 1

(n+ 1)2

]< 2,

for all k = 1, 2, . . . , n. In this case, it follows that

n∑k=1

kk∑j=1

aj

≤ 2

n∑i=1

1

ai,

and so our proof is completed.?F?

86.4. Find the maximum value of

x2y + y2z + z2x

232

Page 234: Олимпиадад бэлтгэгчдэд тусламж (Тэнцэтгэл биш 1)

for reals x, y, z with x+ y + z = 0 and x2 + y2 + z2 = 6.(United Kingdom 1986)

First solution: From the given condition, we have xy + yz + zx = −3, andthen, it follows that

x2y2 + y2z2 + z2x2 = (xy + yz + zx)2 − 2xyz(x+ y + z) = 9.

Now, let us expand the following nonnegative expression (x− xy − 1)2 + (y −yz − 1)2 + (z − zx− 1)2. The work is very simple, we find that it is equal to

3 +∑

x2 +∑

x2y2 + 2∑

xy − 2∑

x− 2∑

x2y.

Replacing∑

x2 by 6,∑

x2y2 by 9,∑

xy by −3 and∑

x by 0, respectively,

we conclude that

(x− xy − 1)2 + (y − yz − 1)2 + (z − zx− 1)2 = 12− 2(x2y + y2z + z2x).

Accordingly, we infer that

x2y + y2z + z2x ≤ 6.

Note that the equality can occur, for example when x = 2 cos2π

9,y = 2 cos

9,z =

2 cos8π

9. This allows us to conclude that the maximum of x2y + y2z + z2x is

6.

Second solution: We will give another way to prove that x2y+y2z+z2x ≤ 6.Denote with p, q, r the values of x+ y+ z, xy+ yz + zx and xyz, respectively.By this substitution, one can easily check that

(x− y)2(y − z)2(z − x)2 = p2q2 − 4q3 + 2(9q − 2p2)pr − 27r2,

and(x2y + y2z + z2x) + (xy2 + yz2 + zx2) = pq − 3r.

On the other hand, from the given hypothesis, we have p = 0 and q = −3.Therefore, the above identities imply

(x− y)2(y − z)2(z − x)2 = −4 · (−3)3 − 27r2 = 27(4− r2),

and(x2y + y2z + z2x) + (xy2 + yz2 + zx2) = −3r.

Accordingly, we have

2∑

x2y =∑

x2y +∑

xy2 +(∑

x2y −∑

xy2)

= −3r + (x− y)(x− z)(y − z) ≤ −3r +√

(x− y)2(y − z)2(z − x)2

= −3r + 3√

3(4− r2) ≤ 3

√[12 +

(√3)2] [

(−r)2 +(√

4− r2)2]

= 12,

233

Page 235: Олимпиадад бэлтгэгчдэд тусламж (Тэнцэтгэл биш 1)

from which it deduces that x2y + y2z + z2x ≤ 6, as desired.

Remark: In addition, we can prove that the following inequality holds (inthe same manner with the two solutions above)

−2

9

(3n2 −m2

2

)3/2

+m3

9≤ x2y + y2z + z2x ≤ 2

9

(3n2 −m2

2

)3/2

+m3

9

for all real numbers x, y, z satisfying x + y + z = m and x2 + y2 + z2 = n2

(3n2 > m2).?F?

87.1. Prove that if x, y, z are real numbers such that x2 + y2 + z2 = 2, then

x+ y + z ≤ xyz + 2.

(IMO Shorlist 1987)

First solution: If one of x, y, z is negative, for example z < 0. Then, werewrite the inequality in the form

−z(1− xy) + 2− x− y ≥ 0,

which is true because

2− x− y ≥ 2−√

2(x2 + y2) ≥ 2−√

2(x2 + y2 + z2) = 0,

and

−z(1− xy) ≥ −z(

1− x2 + y2

2

)= −z

3

2≥ 0.

Let us consider the now the case x, y, z ≥ 0. Without loss of generality, we

may assume that z = max {x, y, z} . By this assumption, we have z >1√3.

Denote t = x+ y, we find that

xy =t2 − x2 − y2

2=t2 + z2 − 2

2.

Therefore, our inequality is equivalent to

2 + z · t2 + z2 − 2

2≥ t+ z,

orf(t) = zt2 − 2t+ z3 − 4z + 4 ≥ 0.

We see that f(t) is a quadratic function of t with the largest coefficient isz > 0. In addtion, its discrimimant is

∆′f = 1−z(z3−4z+4) = −(z2+2z−1)(1−z)2 < −(

1

3+

2√3− 1

)(1−z)2 ≤ 0.

Therefore f(t) ≥ 0, and this ends our proof. Note that the equality holds ifand only if x = y = 1, z = 0 or y = z = 1, x = 0 or z = x = 1, y = 0.

234

Page 236: Олимпиадад бэлтгэгчдэд тусламж (Тэнцэтгэл биш 1)

Second solution: Using the Cauchy Schwarz Inequality, we find that

x+ y + z − xyz = x(1− yz) + (y + z) · 1 ≤√

[x2 + (y + z)2][(1− yz)2 + 1].

So, it is enough to prove that this last quantity is at most 2, which is equivalentto the inequality (2 + 2yz)(2 − 2yz + y2z2) ≤ 4, or 2y3z3 ≤ 2y2z2, which isclearly true, because 2 ≥ y2 + z2 ≥ 2yz.

Third solution: By the same arguments with the first solution, we see thatit suffices to prove the inequality for x, y, z ≥ 0. Because of symmetry, we mayassume that 0 ≤ x ≤ y ≤ z. If z ≤ 1, then

2 + xyz − x− y − z = (1− z)(1− xy) + (1− x)(1− y) ≥ 0.

Now, if z > 1, we have

z + (x+ y) ≤√

2[z2 + (x+ y)2] = 2√

1 + xy ≤ 1 + (1 + xy) ≤ 2 + xyz.

This ends the proof.?F?

88.1. Show that

(1 + x)n ≥ (1− x)n + 2nx(1− x2)n−12

for all 0 ≤ x ≤ 1 and all positive integer n.(Ireland 1988)

Solution: For n = 1, the inequality becomes equality. Suppose that n ≥ 2,

then since (1− x2)n−12 ≤ 1, it suffices to prove that

f(x) = (1 + x)n − (1− x)n − 2nx ≥ 0.

We have f ′(x) = n[(1 + x)n−1 + (1− x)n−1 − 2

]. Since n−1 ≥ 1, the Bernoulli’s

Inequality implies that

(1 + x)n−1 + (1− x)n−1 ≥ 1 + (n− 1)x+ 1 + (n− 1)(−x) = 2.

This means that f ′(x) ≥ 0. Therefore f(x) is increasing on [0, 1], and wededuce that f(x) ≥ f(0) = 0, as claimed. Note that the equality holds if andonly if n = 1 or x = 0.

?F?

88.2. Given a triangle ABC. Prove that

2

(sinA

A+

sinB

B+

sinC

C

)≤(

1

B+

1

C

)sinA+

(1

C+

1

A

)sinB+

(1

A+

1

B

)sinC.

(Russia 1988)

Solution: Rewrite the inequality as(1

B+

1

C− 2

A

)sinA+

(1

C+

1

A− 2

B

)sinB +

(1

A+

1

B− 2

C

)sinC ≥ 0.

235

Page 237: Олимпиадад бэлтгэгчдэд тусламж (Тэнцэтгэл биш 1)

Now, since this inequality is symmetric, we can assume that A ≥ B ≥ C. Bythis assumption, we have a ≥ b ≥ c, and since a = 2R sinA, we deduce thatsinA ≥ sinB ≥ sinC. Also, it is clear from the assumption that

1

B+

1

C− 2

A≥ 1

C+

1

A− 2

B≥ 1

A+

1

B− 2

C.

Thefore, by applying the Chebyshev’s Inequality, we deduce that(1

B+

1

C− 2

A

)sinA+

(1

C+

1

A− 2

B

)sinB +

(1

A+

1

B− 2

C

)sinC ≥

≥ 1

3

(1

B+

1

C− 2

A+

1

C+

1

A− 2

B+

1

A+

1

B− 2

C

)(sinA+sinB+sinC) = 0.

Note that the equality holds if and only if ABC is a equilateral triangle.?F?

89.1. Let x1, x2, . . . , xn be positive real numbers, and let S = x1+x2+· · ·+xn.Prove that

(1 + x1)(1 + x2) · · · (1 + xn) ≤ 1 + S +S2

2!+ · · ·+ Sn

n!.

(APMO 1989)

Solution: By the AM-GM Inequality, we have

(1 + x1)(1 + x2) . . . (1 + xn) ≤[

(1 + x1) + (1 + x2) + · · ·+ (1 + xn)

n

]n=

(1 +

S

n

)n.

Therefore, it suffices to prove that(1 +

S

n

)n≤ 1 + S +

S2

2!+ . . .+

Sn

n!,

and since

(1 +

S

n

)n= 1 +

n∑k=1

(n

k

)nk

Sk, we can rewrite this inequality as

n∑k=1

1

k!−

(n

k

)nk

Sk ≥ 0.

This is true since for any k = 1, . . . , n, we have

nk − k!

(n

k

)= nk − k! · n!

(n− k)!k!= nk − n!

(n− k)!

= nk − (n− k + 1) · · ·n ≥ nk − n · · ·n︸ ︷︷ ︸k numbers

= 0.

236

Page 238: Олимпиадад бэлтгэгчдэд тусламж (Тэнцэтгэл биш 1)

?F?

89.2. Let x, y, z be real numbers such that 0 < x < y < z <π

2. Prove that

π

2+ 2 sinx cos y + 2 sin y cos z > sin 2x+ sin 2y + sin 2z.

(Iberoamerica 1989)

Solution: Put a = sinx, b = sin y, c = sin z, then 1 > c > b > a > 0. Ourinequality becomes

π

4+ a√

1− b2 + b√

1− c2 > a√

1− a2 + b√

1− b2 + c√

1− c2,

orπ

4> a

√1− a2 + (b− a)

√1− b2 + (c− b)

√1− c2.

By the AM-GM Inequality, we get

a√

1− a2 + (b− a)√

1− b2 + (c− b)√

1− c2 ≤

≤ a(

1− a2 +1

4

)+ (b− a)

(1− b2 +

1

4

)+ (c− b)

(1− c2 +

1

4

)= −1

4(4a3 + 4b3 + 4c3 − 4ab2 − 4bc2 − 5c).

Therefore, it suffices to prove that

4a3 + 4b3 + 4c3 − 4ab2 − 4bc2 − 5c+ π > 0.

Using the AM-GM Inequality again, we haveπ

2+π

2+

500

27π2c3 ≥ 5c, and hence

π − 5c ≥ − 500

27π2c3 > −2c3. Consequently, it is enough to check that

2a3 + 2b3 + c3 − 2ab2 − 2bc2 ≥ 0,

or (c3 − 2bc2 +

32

27b3)

+

(2a3 − 2ab2 +

22

27b3)≥ 0.

This is true since by the AM-GM Inequality, we have

c3 − 2bc2 +32

27b3 =

c3

2+c3

2+

32

27b3 − 2bc2

≥ 33

√c3

2· c

3

2· 32

27b3 − 2bc2 = 0,

and

2a3 − 2ab2 +22

27b3 = 2a3 +

11

27b3 +

11

27b3 − 2ab2

≥ 33

√2a3 · 11

27b3 · 11

27b3 − 2ab2

=

(3√

242

3− 2

)ab2 ≥ 0.

237

Page 239: Олимпиадад бэлтгэгчдэд тусламж (Тэнцэтгэл биш 1)

?F?

89.3. Let a, b, c be the sidelengths of a triangle. Prove that∣∣∣∣a− ba+ b+b− cb+ c

+c− ac+ a

∣∣∣∣ < 1

16.

(Iberoamerica 1989)

Solution: Using the identity

a− ba+ b

+b− cb+ c

+c− ac+ a

=(a− b)(b− c)(a− c)(a+ b)(b+ c)(c+ a)

,

we can rewrite the original inequality as

(a+ b)(b+ c)(c+ a) > 16 |(a− b)(b− c)(a− c)| .

Now, since a, b, c are the sidelengths of a triangle, we may put a = y + z, b =z + x, c = x+ y where x, y, z > 0, and hence, our inequality becomes

(2x+ y + z)(2y + z + x)(2z + x+ y) > 16 |(x− y)(y − z)(z − x)| .

Without loss of generality, we may assume that x ≥ y ≥ z, then we have

(2x+ y + z)(2y + z + x)(2z + x+ y) > (2x+ y)(x+ 2y)(x+ y),

and|(x− y)(y − z)(z − x)| = (x− y)(x− z)(y − z) < xy(x− y).

Therefore, it suffices to prove that

(2x+ y)(x+ 2y)(x+ y) > 16xy(x− y).

Now, we apply the AM-GM Inequality to get

(2x+ y)(x+ 2y) = 2(x− y)2 + 9xy ≥ 2√

2(x− y)2 · 9xy = 6√

2xy(x− y),

andx+ y ≥ 2

√xy.

Multiplying these two inequalities, we deduce that

(2x+ y)(x+ 2y)(x+ y) ≥ 12√

2xy(x− y) ≥ 16xy(x− y).

However, it is clear that the equality cannot occur, so we must have

(2x+ y)(x+ 2y)(x+ y) > 16xy(x− y).

The proof is completed.?F?

89.4. Find the least possible value of

(x+ y)(y + z)

238

Page 240: Олимпиадад бэлтгэгчдэд тусламж (Тэнцэтгэл биш 1)

for positive reals x, y, z satisfying xyz(x+ y + z) = 1.(Russia 1989)

Solution: From the AM-GM Inequality, we have

(x+ y)(y + z) = y(x+ y + z) + xz ≥ 2√xyz(x+ y + z) = 2.

The equality holds iff y(x + y + z) = xz and xyz(x + y + z) = 1, which canbe attained for x = z = 1, y =

√2 − 1. This allows us to conclude that the

minimum value of (x+ y)(y + z) is 2.?F?

90.1. Let a, b, c, d be positive real numbers such that ab + bc + cd + da = 1.Prove that

a3

b+ c+ d+

b3

c+ d+ a+

c3

d+ a+ b+

d3

a+ b+ c≥ 1

3.

(IMO Shortlist 1990)

Solution: From the AM-GM Inequality, we get

2 (ab+ ac+ ad+ bc+ bd+ cd) ≤ 3(a2 + b2 + c2 + d2

).

Furthermore, the Cauchy Schwarz Inequality gives us

a2 + b2 + c2 + d2 =√a2 + b2 + c2 + d2

√b2 + c2 + d2 + a2

≥ ab+ bc+ cd+ da = 1.

Therefore, from these two inequalities, we deduce that

(a2 + b2 + c2 + d2)2 ≥ 2

3(ab+ ac+ ad+ bc+ bd+ cd).

Now, using the Cauchy Schwarz Inequality and this inequality, we obtain

a3

b+ c+ d+

b3

c+ d+ a+

c3

d+ a+ b+

d3

a+ b+ c≥

≥(a2 + b2 + c2 + d2

)22 (ab+ ac+ ad+ bc+ bc+ cd)

≥ 1

3,

as desired. Note that the equality holds if and only if a = b = c = d =1

2.

?F?

90.2. Show that

x4 > x− 1

2

for all real x.(Russia 1990)

Solution: According to the AM-GM Inequality, we have

x4 +1

2= x4 +

1

6+

1

6+

1

6≥ 4

4

√x4

63=

44√

63|x| ≥ |x| ≥ x.

239

Page 241: Олимпиадад бэлтгэгчдэд тусламж (Тэнцэтгэл биш 1)

It is easy to see that the equality cannot be attained. Therefore

x4 > x− 1

2,

and our proof is completed.?F?

90.3. Let x1, x2, . . . , xn be positive reals with sum 1. Show that

x21x1 + x2

+x22

x2 + x3+ · · ·+ x2n

xn + x1≥ 1

2.

(Russia 1990)

Solution: According to the Cauchy Schwarz Inequality, we have

x21x1 + x2

+x22

x2 + x3+ · · ·+ x2n

xn + x1≥ (x1 + x2 + · · ·+ xn)2

(x1 + x2) + (x2 + x3) + · · ·+ (xn + x1)

=x1 + x2 + · · ·+ xn

2=

1

2,

as desired. Note that the equality holds if and only if x1 = x2 = · = xn =1

n.

?F?

90.4. Show that√x2 − xy + y2 +

√y2 − yz + z2 ≥

√z2 + zx+ x2

for any positive real numbers x, y, z.(United Kingdom 1990)

Solution: By applying the Minkowsky’s Inequality, we have√x2 − xy + y2 +

√y2 − yz + z2 ≥

=

√√√√(√3

2x

)2

+(x

2− y)2

+

√√√√(√3

2z

)2

+(y − z

2

)2

√√√√(√3

2x+

√3

2z

)2

+(x

2− y + y − z

2

)2=√x2 + xz + z2,

as desired.?F?

91.1. Let a1, a2, . . . , an and b1, b2, . . . , bn be positive real numbers such thata1 + a2 + · · ·+ an = b1 + b2 + · · ·+ bn. Prove that

a21a1 + b1

+a22

a2 + b2+ · · ·+ a2n

an + bn≥ a1 + a2 + · · ·+ an

2.

(APMO 1991)

240

Page 242: Олимпиадад бэлтгэгчдэд тусламж (Тэнцэтгэл биш 1)

First solution: By the Cauchy Schwarz Inequality, we have

a21a1 + b1

+a22

a2 + b2+ · · ·+ a2n

an + bn≥ (a1 + a2 + · · ·+ an)2

(a1 + b1) + (a2 + b2) + · · ·+ (an + bn)

=(a1 + a2 + · · ·+ an)2

(a1 + a2 + · · ·+ an) + (b1 + b2 + · · ·+ bn)

=(a1 + a2 + · · ·+ an)2

(a1 + a2 + · · ·+ an) + (a1 + a2 + · · ·+ an)

=a1 + a2 + · · ·+ an

2,

as desired. Note that the equality holds if and only if ai = bi for all i =1, 2, . . . , n.

Second solution: From the AM-GM Inequality, we see that for all i =1, 2, . . . , n

a2iai + bi

= ai −aibiai + bi

≥ ai −(ai + bi)

2

4(ai + bi)=

3ai − bi4

.

Therefore

a21a1 + b1

+a22

a2 + b2+ · · ·+ a2n

an + bn≥ 3a1 − b1

4+

3a2 − b24

+ · · ·+ 3an − bn4

=3(a1 + a2 + · · ·+ an)− (b1 + b2 + · · ·+ bn)

4

=a1 + a2 + · · ·+ an

2,

as desired.?F?

91.2. Given positive real numbers a, b, c satisfying a+ b+ c = 1, show that

(1 + a) (1 + b) (1 + c) ≥ 8 (1− a) (1− b) (1− c) .

(Russia 1991)

Solution: By applying the AM-GM Inequality, we have that

1 + a = (1− b) + (1− c) ≥ 2√

(1− b) (1− c).

Similarly, we also have

1 + b ≥ 2√

(1− c) (1− a), 1 + c ≥ 2√

(1− a) (1− b).

Multiplying these three inequalities, we get the desired result. Note that the

equality holds if and only if a = b = c =1

3.

?F?

91.3. Show that

(x+ y + z)2

3≥ x√yz + y

√zx+ z

√xy

241

Page 243: Олимпиадад бэлтгэгчдэд тусламж (Тэнцэтгэл биш 1)

for all nonnegative reals x, y, z.(Russia 1991)

Solution: By the AM-GM Inequality, we get

x√yz + y

√zx+ z

√xy ≤ x · y + z

2+ y · z + x

2+ z · x+ y

2

= xy + yz + zx ≤ (x+ y + z)2

3,

as desired. The equality occurs iff x = y = z.?F?

91.4. The real numbers x1, x2, . . . , x1991 satisfy

|x1 − x2|+ |x2 − x3|+ · · ·+ |x1990 − x1991| = 1991.

Denote sn =x1 + x2 + . . .+ xn

n. What is the maximum possible value of

|s1 − s2|+ |s2 − s3|+ . . .+ |s1990 − s1991|?

(Russia 1991)

Solution: We denote with E the desired expression. For all 1 ≤ i ≤ 1990, wehave

si+1 − si =

i+1∑k=1

xk

i+ 1−

i∑k=1

xk

i=

ixi+1 −i∑

k=1

xk

i(i+ 1)=

i∑k=1

k(xk+1 − xk)

i(i+ 1).

This implies that

|si+1 − si| ≤

i∑k=1

k |xk+1 − xk|

i(i+ 1).

According to this inequality, we deduce that

E =1990∑i=1

|si+1 − si| ≤1990∑i=1

i∑k=1

k |xk+1 − xk|

i(i+ 1)=

1990∑i=1

i

(1990∑k=i

1

k(k + 1)

)|xi+1 − xi|

=

1990∑i=1

(1− i

1991

)|xi+1 − xi| ≤

1990∑i=1

(1− 1

1991

)|xi+1 − xi|

=1990

1991

1990∑i=1

|xi+1 − xi| = 1990.

On ther other hand, let x1 = 1991, x2 = x3 = · · · = x1991 = 0, we have|x1 − x2| + |x2 − x3| + · · · + |x1990 − x1991| = 1991 and E = 1990. Therefore,the maximum value of E is 1990.

242

Page 244: Олимпиадад бэлтгэгчдэд тусламж (Тэнцэтгэл биш 1)

?F?

91.5. A triangle has sides a, b, c with sum 2. Show that

a2 + b2 + c2 + 2abc < 2.

(United Kingdom 1991)

Solution: Since a, b, c are the sidelengths of a triangle and a+ b+ c = 2, wesee that 1 > a, b, c > 0. Hence (1 − a)(1 − b)(1 − c) > 0, which implies thatabc < 1− (a+ b+ c) + ab+ bc+ ca = ab+ bc+ ca− 1. And thus,

a2 + b2 + c2 + 2abc < a2 + b2 + c2 + 2(ab+ bc+ ca− 1)

= (a+ b+ c)2 − 2 = 2,

as desired.?F?

91.6. Prove the inequality

x2y

z+y2z

x+z2x

y≥ x2 + y2 + z2

for any positive real numbers x, y, z with x ≥ y ≥ z.(Vietnam 1991)

First solution: By expanding, we see that the original inequality is equivalentto

x3y2 + y3z2 + z3x2 ≥ x3yz + y3zx+ z3xy,

or

x3y(y − z) + y2z2(y − z) + z3(y2 − 2yx+ x2)− xyz(y2 − z2) ≥ 0,

that is(y − z)(x− z)[x2y + yz(x− y)] + z3(x− y)2 ≥ 0.

The last inequality is obviously true since x ≥ y ≥ z. So, our problem is solved.Note that the equality holds if and only if x = y = z.

Second solution: From the Cauchy Schwarz Inequality, we have√x2y

z+y2z

x+z2x

√x2z

y+y2x

z+z2y

x≥ x2 + y2 + z2.

Hence, it suffices to prove that

x2y

z+y2z

x+z2x

y≥ x2z

y+y2x

z+z2y

x.

However, we find that(x2y

z+y2z

x+z2x

y

)−(x2z

y+y2x

z+z2y

x

)=

(x− y)(y − z)(x− z)(xy + yz + zx)

xyz,

243

Page 245: Олимпиадад бэлтгэгчдэд тусламж (Тэнцэтгэл биш 1)

which is obviously nonnegative because x ≥ y ≥ z > 0. Thus, the lastinequality is true and our proof is completed.

?F?

92.1. For every integer n ≥ 2, find the smallest positive number λ = λ(n)

such that if 0 ≤ a1, a2, . . . , an ≤1

2, b1, b2, . . . , bn > 0, and a1 + a2 + · · ·+ an =

b1 + b2 + · · ·+ bn = 1, then

b1b2 · · · bn ≤ λ(a1b1 + a2b2 + · · ·+ anbn).

(China 1992)

Solution: If n = 2, then from 0 ≤ a1, a2 ≤ 12 and a1 +a2 = 1, we deduce that

a1 = a2 =1

2. Therefore

a1b1 + a2b2b1b2

=b1 + b22b1b2

=1

2

(1

b1+

1

b2

)≥ 2

b1 + b2= 2,

and the equality holds for b1 = b2 =1

2. This allows us to conclude that

λ(2) =1

2=

1

2(2− 1)2−1. Now, suppose that n ≥ 3, and let b1 = b2 =

1

2(n− 1),

b3 = · · · = bn =1

n− 1, a1 = a2 =

1

2and a3 = · · · = an = 0, we get

λ(n) ≥ 1

2(n− 1)n−1.

We will show that it is the value we need to find, i.e.

a1b1 + a2b2 + · · ·+ anbn ≥ 2(n− 1)n−1b1b2 · · · bn,

ora1b1 + a2b2 + · · ·+ anbn

a1 + a2 + · · ·+ an≥ 2(n− 1)n−1b1b2 · · · bn.

Without loss of generality, we may assume that b1 ≤ b2 ≤ · · · ≤ bn. By thisassumption, we have

a1b1 + a2b2 + · · ·+ anbna1 + a2 + · · ·+ an

− (a2 + · · ·+ an)b1 + a2b2 + · · ·+ anbn2(a2 + · · ·+ an)

=

=(a2 + · · ·+ an − a1) [a2b2 + · · ·+ anbn − (a2 + · · ·+ an)b1]

2(a2 + · · ·+ an)(a1 + a2 + · · ·+ an)

=(a2 + · · ·+ an − a1) [a2(b2 − b1) + · · ·+ an(bn − b1)]

2(a2 + · · ·+ an)(a1 + a2 + · · ·+ an).

The last quantity is nonnegative since a2(b2− b1) + · · ·+ an(bn− b1) ≥ 0 fromthe assumption and a2 + · · ·+ an− a1 = 1− 2a1 ≥ 0. Therefore, from this, we

244

Page 246: Олимпиадад бэлтгэгчдэд тусламж (Тэнцэтгэл биш 1)

deduce that

a1b1 + a2b2 + · · ·+ anbna1 + a2 + · · ·+ an

≥ (a2 + · · ·+ an)b1 + a2b2 + · · ·+ anbn2(a2 + · · ·+ an)

=(b2 + b1)a2 + · · ·+ (bn + b1)an

2(a2 + · · ·+ an)

≥ (b2 + b1)a2 + · · ·+ (b2 + b1)an2(a2 + · · ·+ an)

=(b2 + b1)(a2 + · · ·+ an)

2(a2 + · · ·+ an)

=b1 + b2

2≥√b1b2.

The inequality is reduced to proving that√b1b2 ≥ 2(n− 1)n−1b1b2 · · · bn,

or equivalently,

b1b2b23 · · · b2n ≤

1

4(n− 1)2(n−1).

This is true since by the AM-GM Inequality, we have

b1b2b23 · · · b2n =

1

4· (2b1) · (2b2) · b3 · b3 · · · bn · bn

≤ 1

4

(2b1 + 2b2 + b3 + b3 + · · ·+ bn + bn

2(n− 1)

)2(n−1)

=1

4

(b1 + b2 + · · ·+ bn

n− 1

)2(n−1)=

1

4(n− 1)2(n−1).

Our statement is proved. And thus, we conclude that λ(n) =1

2(n− 1)n−1for

every integer n.?F?

92.2. Show thatx4 + y4 + z2 ≥ 2

√2xyz

for all positive reals x, y, z.(Russia 1992)

Solution: Using the AM-GM Inequality, we get

x4 + y4 + z2 ≥ 2x2y2 + z2 ≥ 2√

2x2y2 · z2 = 2√

2xyz,

as desired. Note that the equality holds if and only if x = y and z =√

2x2.?F?

92.3. Show that for any real numbers x, y > 1, we have

x2

y − 1+

y2

x− 1≥ 8.

245

Page 247: Олимпиадад бэлтгэгчдэд тусламж (Тэнцэтгэл биш 1)

(Russia 1992)

Solution: From the AM-GM Inequality, we have

x2

y − 1+

y2

x− 1≥ 4x2

[1 + (y − 1)]2+

4y2

[1 + (x− 1)]2=

4x2

y2+

4y2

x2≥ 8,

as desired. Note that the equality holds if and only if x = y = 2.?F?

92.4. Positive real numbers a, b, c satisfy a ≥ b ≥ c. Prove that

a2 − b2

c+c2 − b2

a+a2 − c2

b≥ 3a− 4b+ c.

(Ukraine 1992)

Solution: From a ≥ b ≥ c > 0, we havea+ b

c≥ 2,

c+ b

a≤ 2, and

a+ c

b≥ 1.

Now, we get (with noting that a− b ≥ 0, c− b ≤ 0 and a− c ≥ 0)

a2 − b2

c≥ 2(a− b), c2 − b2

a≥ 2(c− b), a2 − c2

b≥ a− c.

After addition of these inequalities, we have

a2 − b2

c+c2 − b2

a+a2 − c2

b≥ 2(a− b) + 2(c− b) + (a− c) = 3a− 4b+ c,

as claimed. Note that the equality holds if and only if a = b = c.?F?

92.5. Let x, y, z, w be positive real numbers. Prove that

12

x+ y + z + w≤∑sym

1

x+ y≤ 3

4

(1

x+

1

y+

1

z+

1

w

).

(United Kingdom 1992)

Solution: By the Cauchy Schwarz Inequality, we have∑sym

1

x+ y≥ 62∑

sym

(x+ y)=

12

x+ y + z + w,

and ∑sym

1

x+ y≤∑sym

(1

4x+

1

4y

)=

3

4

(1

x+

1

y+

1

z+

1

w

).

From these two inequalities, we infer that

12

x+ y + z + w≤∑sym

1

x+ y≤ 3

4

(1

x+

1

y+

1

z+

1

w

),

246

Page 248: Олимпиадад бэлтгэгчдэд тусламж (Тэнцэтгэл биш 1)

as desired. Note that the equality holds (in both parts) if and only if x = y =z = w.

?F?

93.1. Let a, b, c, d be positive real numbers. Prove that

a

b+ 2c+ 3d+

b

c+ 2d+ 3a+

c

d+ a+ 3b+

d

a+ 2b+ 3c≥ 2

3.

(IMO Shortlist 1993)

Solution: From the AM-GM Inequality, we have

2 (ab+ ac+ ad+ bc+ bd+ cd) ≤ 3(a2 + b2 + c2 + d2

),

and hence, it follows that

(a+ b+ c+ d)2 = a2 + b2 + c2 + d2 + 2 (ab+ ac+ ad+ bc+ bd+ cd)

≥ 8

3(ab+ ac+ ad+ bc+ bd+ cd) .

Now, using the Cauchy Schwarz Inequality in combination with this inequality,we get

a

b+ 2c+ 3d+

b

c+ 2d+ 3a+

c

d+ a+ 3b+

d

a+ 2b+ 3c≥

≥ (a+ b+ c+ d)2

4 (ab+ ac+ ad+ bc+ bd+ cd)≥ 2

3,

as desired. The equality holds if and only if a = b = c = d.?F?

93.2. Let a, b, c ∈ [0, 1]. Prove that

a2 + b2 + c2 ≤ a2b+ b2c+ c2a+ 1.

(Italia 1993)

Solution: The desired inequality is equivalent to

a2 (1− b) + b2 (1− c) + c2 (1− a) ≤ 1.

Since a, b, c ∈ [0, 1], we have

a2 (1− b) + b2 (1− c) + c2 (1− a) ≤ a (1− b) + b (1− c) + c (1− a)

= (a− 1)(b− 1)(c− 1) + 1− abc ≤ 1.

?F?

93.3. Let x, y, u, v be positive real numbers. Prove that

xy + xv + yu+ uv

x+ y + u+ v≥ xy

x+ y+

uv

u+ v.

247

Page 249: Олимпиадад бэлтгэгчдэд тусламж (Тэнцэтгэл биш 1)

(Poland 1993)

Solution: Put a = x+ y, b = u+ v. Then we have to prove that

xy + xv + uy + uv

a+ b≥ xy

a+uv

b,

or equivalently,

ab (xy + xv + uy + uv)− (a+ b) (bxy + auv) ≥ 0.

The left hand side of the last inequality is equal to (au− bx)(by − av) whichis clearly nonnegative because

(au− bx) (by − av) = [(x+ y)u− (u+ v)x] [(u+ v) y − (x+ y) v]

= (yu− xv)2 ≥ 0.

The proof is completed.?F?

93.4. If the equation x4 + ax3 + 2x2 + bx + 1 = 0 has at least one real root,then a2 + b2 ≥ 8.

(Tournament of the Towns 1993)

First solution: Let x be the real root of the equation. Using the CauchySchwarz Inequality, we infer that

a2 + b2 ≥ (x4 + 2x2 + 1)2

x6 + x2≥ 8,

because the last inequality is equivalent to (x2 − 1)2 ≥ 0.

Second solution: Assume that x0 is the root of the desired equation, then

x40 + ax30 + 2x20 + bx0 + 1 = 0.

According to the two trivial inequalities x20(a+ 2x0)2 ≥ 0 and (bx0 + 2)2 ≥ 0,

we find that

x40 + ax30 ≥ −1

4a20x

20, and b0x0 + 1 ≥ −1

4b20x

20.

Therefore

0 = x40 + ax30 + 2x20 + bx0 + 1 ≥ −1

4a20x

20 + 2x20 −

1

4b20x

20 =

1

4x20(8− a2 − b2).

On the other hand, it is easy to see that x0 6= 0. Thus, from the aboveinequality, we can deduce that

a2 + b2 ≥ 8,

as desired.?F?

248

Page 250: Олимпиадад бэлтгэгчдэд тусламж (Тэнцэтгэл биш 1)

93.5. Let x1, x2, x3, x4 be real numbers such that

1

2≤ x21 + x22 + x23 + x24 ≤ 1.

Find the largest and the smallest values of the expression

A = (x1 − 2x2 + x3)2 + (x2 − 2x3 + x4)

2 + (x2 − 2x1)2 + (x3 − 2x4)

2.

(Vietnam 1993)

Solution: (1) To find the maxA, we rewrite

A = 5(x21 + x24) + 6(x22 + x2 − 3)− 8(x1x2 + x2x3 + x3x4) + 2x1x3 + 2x2x4.

Now, we try to replace xixj by a sum of squares, so that at the end all squaresx2i appear with the same coefficient. This can be done using the followingremark: For any positive number r, the AM-GM Inequality implies that

• −8x1x2 ≤ rx21 +16

rx22 with equality iff rx1 = −4x2;

• −8x2x3 ≤ 4(x22 + x23) with equality iff x2 = −x3;• −8x3x4 ≤ rx24 +

16

rx23 with equality iff rx4 = −4x3;

• 2x1x3 ≤r

4x21 +

4

rx23 with equality iff rx1 = 4x3;

• 2x2x4 ≤4

rx22 +

r

4x24 with equality iff rx4 = 4x2.

Therefore

A ≤(

5 +5r

4

)(x21 + x24) +

(10 +

20

r

)(x22 + x23).

Now, by choosing r > 0 such that 5 +5r

4= 10 +

20

r, or equivalently r =

2(1 +√

5), we get

A ≤5(3 +√

5)

2.

The equality holds if and only if the above conditions on xi can be simulta-neously satisfied and x21 + x22 + x23 + x24 = 1. One can verify that A reaches itsmaximum at two points

x1 = ±1

2

√5−√

5

5, x2 = −1 +

√5

2x1, x3 =

1 +√

5

2x1, x4 = −x1.

(2) For finding the minimum, we follow the same idea (but more complicated).Fix a number r and rewrites

A = (x1 − rx2 + x3)2 + (x2 − rx3 + x4)

2 − 2(4− r)x1x2 − 4(2− r)x2x3− 2(4− r)x3x4 + 4(x21 + x24) + (5− r2)(x22 + x23).

By the freedom, we may assume r < 2. Then for any s > 0, we have

• −2(4− r)x1x2 ≥ −(4− r)(sx21 +

x22s

)with equality iff sx1 = x2;

249

Page 251: Олимпиадад бэлтгэгчдэд тусламж (Тэнцэтгэл биш 1)

• −2(4− r)x3x4 ≥ −(4− r)(sx24 +

x23s

)with equality iff sx4 = x3;

• −4(2− r)x2x3 ≥ −2(2− r)(x22 + x23) with equality iff x2 = x3.Hence

A ≥ −2(4− r)x1x2 − 4(2− r)x2x3 − 2(4− r)x3x4 + 4(x21 + x24) + (5− r2)(x22 + x23)

≥ [4− (4− r)s](x21 + x24) +

(1 + 2r − 4− r

s− r2

)(x22 + x23).

Choosing s such that

4− (4− r)s = 1 + 2r − 4− rs− r2, (∗)

we get

A ≥ 4− (4− r)s2

.

The equality holds if and only if

sx1 = x2

sx4 = x3x2 = x3x1 − rx2 + x3 = 0x2 − rx3 + x4 = 0

x21 + x22 + x23 + x24 =1

2

x2 = x3x1 = x4x1 = (r − 1)x2

x1 =e

2√

1 + s2(e = ±1)

r − 1 =1

s

.

From (∗) and r − 1 =1

s, we get

4 +

(1

s− 3

)s = 2− 1

s2+

1

s

(1

s− 3

),

which is equivalent to s2 − s− 1 = 0, i.e. s =1 +√

5

2. Then

x1 =e√

10 + 2√

5, x2 =

1 +√

5

2x1.

Thus minA =7− 3

√5

4and it is attained at

x1 = x4 = ± 1√10 + 2

√5, x2 = x3 = ± 1 +

√5

2√

10 + 2√

5.

?F?

94.1. Let a1, a2, . . . , an be a sequence of positive real numbers satisfyinga1 + · · ·+ ak ≥

√k for all k = 1, 2, . . . , n. Prove that

a21 + a22 + · · ·+ a2n >1

4

(1 +

1

2+ · · ·+ 1

n

).

250

Page 252: Олимпиадад бэлтгэгчдэд тусламж (Тэнцэтгэл биш 1)

(USA 1994)

Solution: Denote bi =√i−√i− 1 for i = 1, 2, . . . , n, then we have a1 + · · ·+

ak ≥ b1 + · · ·+ bk for every k = 1, 2, . . . , n. We claim that

a21 + a22 + · · ·+ a2n ≥ b21 + b22 + · · ·+ b2n.

Indeed, from the Cauchy Schwarz Inequality, we have(n∑i=1

a2i

)(n∑i=1

b2i

)≥

(n∑i=1

aibi

)2

.

Therefore, in order to prove our claim, it suffices to prove that

n∑i=1

aibi ≥n∑i=1

b2i .

Now, using the Abel’s Summation formula, we have

n∑i=1

aibi−n∑i=1

b2i =n∑i=1

bi(ai− bi) =n−1∑i=1

(bi− bi+1)i∑

k=1

(ak− bk) + bn

n∑i=1

(ai− bi).

Since

i∑k=1

(ak − bk) ≥ 0 for all i = 1, 2, . . . , n, and

bi − bi+1 = 2√i−(√

i+ 1 +√i− 1

)≥ 2√i−√

(1 + 1)(i+ 1 + i− 1) = 0,

we see that each element of the above sum is nonnegative and hence, it is clearthat

n∑i=1

aibi ≥n∑i=1

b2i .

This proves the claim that stated

n∑i=1

a2i ≥n∑i=1

b2i .

On the other hand, we have

n∑i=1

b2i =n∑i=1

(√i−√i− 1

)2=

n∑i=1

1(√i+√i− 1

)2>

n∑i=1

1(√i+√i)2 =

1

4

(1 +

1

2+ · · ·+ 1

n

).

Therefore, from this and the above inequality, we deduce that

a21 + a22 + · · ·+ a2n >1

4

(1 +

1

2+ · · ·+ 1

n

),

251

Page 253: Олимпиадад бэлтгэгчдэд тусламж (Тэнцэтгэл биш 1)

as desired.?F?

95.1. Prove that for any positive real numbers x, y and any positive integersm,n,

(m−1)(n−1)(xm+n + ym+n

)+(m+ n− 1) (xmyn + xnym) ≥ mn

(xm+n−1y + xym+n−1) .

(Austrian-Polish Competition 1995)

First solution: The inequality is equivalent to

(m− 1)(n− 1)(xm+n + ym+n − xmyn − xnym

)≥

≥ mn(xm+n−1y + xym+n−1 − xmyn − xnym

),

or

(m− 1)(n− 1)(xm − ym)(xn − yn) ≥ mnxy(xm−1 − ym−1)(xn−1 − yn−1).

Due to symmetry, we may assume that x ≥ y > 0. Then, we find that theabove inequality follows from multiplying the following inequalities

(m− 1)(xm − ym) ≥ m√xy(xm−1 − ym−1),

and(n− 1)(xn − yn) ≥ n√xy(xn−1 − yn−1).

So, it suffices to prove that these two inequalities holds. Let us first provethe former inequality. Denote x = t2y where t ≥ 1, then we can rewrite thisinequality as

f(t) = (m− 1)(t2m − 1)−mt(t2m−2 − 1) ≥ 0.

We have

f ′(t) = m[(2m− 2)t2m−1 − (2m− 1)t2m−2 + 1

]≥ 0,

since by the AM-GM Inequality, it follows that

(2m− 2)t2m−1 + 1 = (2m− 2) · t2m−1 + 1 · 1 ≥ (2m− 1)(t2m−1

) 2m−22m−1 · 1

12m−1

= (2m− 1)t2m−2.

This means that f(t) is increasing for all t ≥ 1 and thus, it is clear thatf(t) ≥ f(1) = 0 for any t ≥ 1. This proves the first inequality. Similarly, wecan prove that the second inequality holds, and it ends our proof. Note thatthe equality holds if and only if x = y or m = 1 or n = 1.

Second solution: If x = y, then it is trivial. Conversely, in case x 6= y, wetransform the inequality as follows

mn(x− y)(xm+n−1 − ym+n−1) ≥ (m+ n− 1)(xm − ym)(xn − yn),

252

Page 254: Олимпиадад бэлтгэгчдэд тусламж (Тэнцэтгэл биш 1)

or equivalently,

xm+n−1 − ym+n−1

(m+ n− 1)(x− y)≥ xm − ym

m(x− y)· x

n − yn

n(x− y).

(we have assumed that x > y). The last relation can also be written

(x− y)

x∫y

tm+n−2dt ≥x∫y

tm−1dt ·x∫y

tn−1dt,

and this follows from the Chebyshev’s Inequality for integrals.?F?

95.2. Let a, b, c, d be positive real numbers. Prove that

a+ c

a+ b+b+ d

b+ c+c+ a

c+ d+d+ b

d+ a≥ 4.

(Baltic Way 1995)

Solution: According to the AM-GM Inequality, we have

a+ c

a+ b+b+ d

b+ c+c+ a

c+ d+d+ b

d+ a=

=(a+ c) (a+ b+ c+ d)

(a+ b) (c+ d)+

(b+ d) (a+ b+ c+ d)

(b+ c) (d+ a)

≥ (a+ b+ c+ d)

[4 (a+ c)

[(a+ b) + (c+ d)]2+

4 (b+ d)

[(b+ c) + (d+ a)]2

]= 4 · a+ b+ c+ d

a+ b+ c+ d= 4,

as desired. Note that the equality holds if and only if a = c and b = d.?F?

95.3. Let x, y, z be positive real numbers. Prove that

xxyyzz ≥ (xyz)x+y+z

3 .

(Canada 1995)

Solution: The original inequality can be written as

x lnx+ y ln y + z ln z ≥ 1

3(x+ y + z)(lnx+ ln y + ln z).

Due to symmetry, we may assume that x ≥ y ≥ z. Since the function f(x) =lnx is increasing for any positive reals x, this assumption also implies thatlnx ≥ ln y ≥ ln z. Therefore, we can see that the above inequality is a directconsequence of the Chebyshev’s Inequality. Note that the equality holds ifand ony if x = y = z.

?F?

253

Page 255: Олимпиадад бэлтгэгчдэд тусламж (Тэнцэтгэл биш 1)

95.4. If a, b, c are positive real numbers such that abc = 1, then

1

a3 (b+ c)+

1

b3 (c+ a)+

1

c3 (a+ b)≥ 3

2.

(IMO 1995)

Solution: Applying the AM-GM Inequality, we have

1

a3 (b+ c)+b+ c

4bc≥ 2

√b+ c

4a3bc (b+ c)=

1

a.

Adding this to the two analogous inequalities, we get

1

a3 (b+ c)+

1

b3 (c+ a)+

1

c3 (a+ b)+

1

4

(b+ c

bc+c+ a

ca+a+ b

ab

)≥ 1

a+

1

b+

1

c.

Therefore

1

a3 (b+ c)+

1

b3 (c+ a)+

1

c3 (a+ b)≥ 1

2

(1

a+

1

b+

1

c

)≥ 3

23

√1

abc=

3

2.

The proof is completed. Note that the equality holds if and only if a = b =c = 1.

?F?

95.5. Suppose that x1, x2, . . . , xn are real numbers satisfying |xi − xi+1| < 1and xi ≥ 1 for i = 1, 2, . . . , n (xn+1 = x1). Prove the following inequality

x1x2

+x2x3

+ · · ·+ xnx1

< 2n− 1.

(India 1995)

First solution: Sincex1x2· x2x3· · · xn

x1= 1, there exists an index k such that

xkxk+1

< 1. On the other hand, since |xi−xi+1| < 1 and xi ≥ 1 for all i, we find

that xi < xi+1+1 ≤ 2xi+1, and thusxixi+1

< 2 for all i. From these arguments,

we get

x1x2

+x2x3

+ · · ·+ xnx1

=∑i 6=k

xixi+1

+xkxk+1

< 2(n− 1) + 1 = 2n− 1,

as desired.

Second solution: Similar to the preceding solution, one may prove thatxi < 2xi+1 for all i. Now, we rewrite the inequality as(

2− x1x2

)+

(2− x2

x3

)+ · · ·+

(2− xn

x1

)> 1,

orn∑i=1

2xi+1 − xixi+1

> 1.

254

Page 256: Олимпиадад бэлтгэгчдэд тусламж (Тэнцэтгэл биш 1)

Since 2xi+1 − xi > 0, the Cauchy Schwarz Inequality implies

n∑i=1

2xi+1 − xixi+1

(n∑i=1

xi

)2

n∑i=1

xi+1(2xi+1 − xi)=

(n∑i=1

xi

)2

2

n∑i=1

x2i −n∑i=1

xixi+1

>

n∑i=1

x2i +

n∑i=1

xixi+1

2n∑i=1

x2i −n∑i=1

xixi+1

>

n∑i=1

x2i +

n∑i=1

xi ·xi2

2n∑i=1

x2i −n∑i=1

xi ·xi2

= 1.

This proves our inequality.?F?

95.6. (a) Find the maximum value of the expression x2y−y2x when 0 ≤ x ≤ 1and 0 ≤ y ≤ 1.

(b) Find the maximum value of the expression x2y+y2z+z2x−y2x−z2y−x2zwhen 0 ≤ x ≤ 1, 0 ≤ y ≤ 1 and 0 ≤ z ≤ 1.

(United Kingdom 1995)

Solution: (a) By the AM-GM Inequality, we have

x2y − y2x = xy(x− y) ≤ xy(1− y) ≤ y(1− y) ≤(y + 1− y

2

)2

=1

4,

and the equality holds when x = 1 and y =1

2. Therefore, the maximum value

of the desired expression is1

4.

(b) Due to the cyclicity, we may assume that x = max {x, y, z} . Then, applyingthe AM-GM Inequality, we find that

x2y + y2z + z2x− y2x− z2y − x2z = (x− y)(x− z)(y − z)≤ (x− y)(x− z)y ≤ xy(x− y)

≤ x ·(y + x− y

2

)2

=x3

4≤ 1

4.

The equality holds for example when x = 1, y =1

2and z = 0. Therefore, the

maximum value of the desired expression is1

4.

?F?

95.7. Let a, b, c be real numbers satisfying a < b < c, a + b + c = 6 andab+ bc+ ca = 9. Prove that

0 < a < 1 < b < 3 < c < 4.

255

Page 257: Олимпиадад бэлтгэгчдэд тусламж (Тэнцэтгэл биш 1)

(United Kingdom 1995)

Solution: From the given condition, we have a < 2 < c. Now, let us provethe following chain of identities

3(a− 1)(a− 3)− (a− b)(a− c) = 3(b− 1)(b− 3)− (b− c)(b− a)

= 3(c− 1)(c− 3)− (c− a)(c− b) = 0.

Indeed, we have

0 = 9− ab− bc− ca = 9 + a2 − 2a(b+ c)− (a− b)(a− c)= 9 + a2 − 2a(6− a)− (a− b)(a− c)= 3(a− 1)(a− 3)− (a− b)(a− c).

Since a < b < c, it is clear that

(a− b)(a− c) > 0, (b− c)(b− a) < 0, (c− a)(c− b) > 0.

And hence, the above identities imply that (a−1)(a−3) > 0, (b−1)(b−3) < 0and (c − 1)(c − 3) > 0, from which we deduce that (with noting a < 2 < c)a < 1 < b < 3 < c. This yields that (1− a)(1− b) < 0 and (3− b)(3− c) < 0.Therefore, according to the identities

0 = 9− ab− bc− ca = 10− (c+ 1)(a+ b)− (1− a)(1− b)= 10− (c+ 1)(6− c)− (1− a)(1− b)= (c− 1)(c− 4)− (1− a)(1− b),

and

0 = 9− ab− bc− ca = 18− (a+ 3)(b+ c)− (3− b)(3− c)= 18− (a+ 3)(6− a)− (3− b)(3− c)= a(a− 3)− (3− b)(3− c),

we infer that (c− 1)(c− 4) < 0 and a(a− 3) < 0, from which we obtain c < 4and a > 0. Thus, from these arguments, we conclude that

0 < a < 1 < b < 3 < c < 4,

as desired.?F?

95.8. Find the greatest constant k such that for any positive integer n whichis not a square, ∣∣(1 +

√n)

sin(π√n)∣∣ > k.

(Vietnam (IMO training camp) 1995)

Solution: We will prove thatπ

2is the best constant. For any positive integer

i, we must have

k <(

1 +√i2 + 1

) ∣∣∣sin(π√i2 + 1)∣∣∣ .

256

Page 258: Олимпиадад бэлтгэгчдэд тусламж (Тэнцэтгэл биш 1)

Because∣∣∣sin(π√i2 + 1

)∣∣∣ = sinπ√

i2 + 1 + i, we deduce that

π√i2 + 1 + i

≥∣∣∣sin(π√i2 + 1

)∣∣∣ > k√i2 + 1 + 1

,

from which it follows that k ≤ π

2. Now, let us prove that the constant

π

2is

good. Clearly, the inequality can be rewritten as

sin(π{√

n})

2 (√n+ 1)

.

We have two cases

Case 1. {n} ≤ 1

2. Of course,

{√n}≥√n−√n− 1 =

1√n+√n− 1

,

and because sinx ≥ x− x3

6, we find that

sin(π{√

n})≥ sin

π√n+√n− 1

≥ π√n− 1 +

√n− 1

6

(π√

n− 1 +√n

)3

.

Let us prove that the last quantity is at leastπ

2 (1 +√n). This comes down to

2 +√n−√n− 1

1 +√n

>π2

3(√n+√n− 1

)2 ,or

6(√n+√n− 1

)2+ 3

(√n+√n− 1

)> π2

(1 +√n),

and it is clear.

Case 2. {n} > 1

2. Let x = 1 − {

√n} < 1

2and let n = k2 + p, 1 ≤ p ≤ 2k.

Because {n} > 1

2, we have p ≥ k + 1. Then it is easy to see that x ≥

1

k + 1 +√k2 + 2k

and so, it suffices to prove that

sinx

k + 1 +√k2 + 2k

≥ π

2(

1 +√k2 + k

) .Using again the inequality sinx ≥ x− x3

6, we infer that

2√k2 + k −

√k2 + 2k − k + 1

1 +√k2 + k

>π2

3(

1 + k +√k2 + 2k

)2 .But from the Cauchy Schwarz Inequality, we have 2

√k2 + k−

√k2 + 2k−k ≥ 0.

And because the inequality(

1 + k +√k2 + 2k

)2>π2

3

(1 +√k2 + k

)holds,

this case is also solved.

257

Page 259: Олимпиадад бэлтгэгчдэд тусламж (Тэнцэтгэл биш 1)

?F?

96.1. Let m and n be positive integers such that n ≤ m. Prove that

2n · n! ≤ (m+ n)!

(m− n)!≤ (m2 +m)n.

(APMO 1996)

Solution: The quantity in the middle is (m+n)(m+n−1) · · · (m−n+ 1). Ifwe pair of terms of the form (m+ x) and (m+ 1− x), we get products whichdo not exceed m(m + 1), since the function f(x) = (m + x)(m + 1 − x) is a

concave parabola with maximum at x =1

2. From this, the right inequality

follows. For the left, we need only show (m + x)(m + 1 − x) ≥ 2x for x ≤ n,this rearranges to (m− x)(m+ 1 + x) ≥ 0, which holds because m ≥ n ≥ x.

?F?

96.2. Let a, b, c be the lengths of the sides of a triangle. Prove that

√a+ b− c+

√b+ c− a+

√c+ a− b ≤

√a+√b+√c,

and determine when equality occurs.(APMO 1996)

Solution: By the triangle inequality, b+ c− a and c+ a− b are positive. Forany positive x, y, we have

2(x+ y) ≥ x+ y + 2√xy =

(√x+√y)2

by the AM-GM Inequality, with equality for x = y. Substituting x = a+ b−c, y = b+ c− a, we get

√a+ b− c+

√b+ c− a ≤ 2

√a,

which added to the two analogous inequalities yields the desired result. Theequality holds for a+ b− c = b+ c− a = c+ a− b, i.e. a = b = c.

?F?

96.3. The real numbers x, y, z, t satisfy the equalities x + y + z + t = 0 andx2 + y2 + z2 + t2 = 1. Prove that

−1 ≤ xy + yz + zt+ tx ≤ 0.

(Austrian-Polish Competition 1996)

Solution: The inner expression is (x + z)(y + t) = −(x + z)2, so the secondinequality is obvious. As for the first, note that

1 = (x2 + z2) + (y2 + t2) ≥ 1

2[(x+ z)2 + (y + t)2] ≥ |(x+ z)(y + t)|

by two applications of the power mean inequality.

258

Page 260: Олимпиадад бэлтгэгчдэд тусламж (Тэнцэтгэл биш 1)

?F?

96.4. Let a, b, c be positive real numbers, such that a+ b+ c =√abc. Prove

thatab+ bc+ ca ≥ 9(a+ b+ c).

(Belarus 1996)

Solution: Applying the well-known inequality (x+ y+ z)2 ≥ 3(xy+ yz+ zx)with x = ab, y = bc, z = ca, we get

(ab+ bc+ ca)2 ≥ 3abc(a+ b+ c) = 3(a+ b+ c)3,

and thusab+ bc+ ca ≥ (a+ b+ c)

√3(a+ b+ c).

On the other hand, the AM-GM Inequality implies

a+ b+ c =√abc ≤

√(a+ b+ c

3

)3

,

from which we deduce that√

3(a+ b+ c) ≥ 9. Therefore

ab+ bc+ ca ≥ (a+ b+ c)√

3(a+ b+ c) ≥ 9(a+ b+ c),

as claimed. Note that the equality holds if and only if a = b = c =√

3.?F?

96.5. Suppose n ∈ N, x0 = 0, xi > 0 for i = 1, 2, . . . , n, and x1+x2+· · ·+xn =1. Prove that

1 ≤n∑i=1

xi√1 + x0 + · · ·+ xi−1 ·

√xi + · · ·+ xn

2.

(China 1996)

Solution: The left inequality follows from the fact that√1 + x0 + · · ·+ xi−1 ·

√xi + · · ·+ xn ≤

1

2(1 + x0 + · · ·+ xn) = 1,

so that the middle quantity is at least x1+· · ·+xn = 1. For the right inequality,let θi = arcsin(x0 + · · ·+ xi) for i = 0, . . . , n, so that√

1 + x0 + · · ·+ xi−1 ·√xi + · · ·+ xn = cos θi−1,

and the desired inequality is

n∑i=1

sin θi − sin θi−1cos θi−1

2.

Now note that

sin θi − sin θi−1 = 2 cosθi + θi−1

2sin

θi − θi−12

< (θi − θi−1) cos θi−1,

259

Page 261: Олимпиадад бэлтгэгчдэд тусламж (Тэнцэтгэл биш 1)

using the facts that θi−1 < θi and that sinx < x for x > 0, so that

n∑i=1

sin θi − sin θi−1cos θi−1

<

n∑i=1

(θi − θi−1) = θn − θ0 <π

2,

as claimed.?F?

96.6. (a) Find the minimum value of xx for x a positive real number.

(b) If x and y are positive real numbers, show that xy + yx > 1.

(France 1996)

First solution: (a) Since xx = ex log x and ex is an increasing function of x, itsuffices to determine the minimum of x log x. This is easily done by setting its

derivative 1+log x to zero, yielding x =1

e. The second derivative

1

xis positive

for x > 0, so the function is everywhere convex, and the unique extremum isindeed a global minimum. Hence xx has minimum value e−

1e .

(b) If x ≥ 1, then xy ≥ 1 for y > 0, so we may assume 0 < x, y < 1. Withoutloss of generality, assume x ≤ y, now note that the function f(x) = xy + yx

has derivative f ′(x) = xy log x + yx−1. Since yx ≥ xx ≥ xy for x ≤ y and1

x≥ − log x, we see that f ′(x) > 0 for 0 ≤ x ≤ y and so the minimum of

f occurs with x = 0, in which case f(x) = 1, since x > 0, we have strictinequality.

Second solution: We present another way to prove (b). Firstly, we will

prove that ab ≥ a

a+ b− abfor any a, b ∈ (0, 1). Indeed, from the Bernoull’s

Inequality, it follows that a1−b = [1+(a−1)]1−b ≤ 1+(a−1)(1−b) = a+b−ab,and thus the conclusion follows. Now, if x or y is at least 1, we are done.Otherwise, let 0 < x, y < 1. In this case, we apply the above observation andfind that

xy + yx ≥ x

x+ y − xy+

y

x+ y − xy>

x

x+ y+

y

x+ y= 1.

?F?

96.7. Let a, b, c be positive real numbers such that abc = 1. Prove that

ab

a5 + b5 + ab+

bc

b5 + c5 + bc+

ca

c5 + a5 + ca≤ 1.

(IMO Shortlist 1996)

Solution: For any positive numbers x, y, we have (x3− y2)(x2− y2) ≥ 0, andhence x5 + y5 ≥ x2y2(x+ y). According to this inequality, we have

ab

a5 + b5 + ab=

a2b2c

a5 + b5 + a2b2c≤ a2b2c

a2b2 (a+ b) + a2b2c=

c

a+ b+ c.

260

Page 262: Олимпиадад бэлтгэгчдэд тусламж (Тэнцэтгэл биш 1)

Adding this to the two analogous inequalities, we get the desired result. Notethat the equality holds if and ony if a = b = c = 1.

?F?

96.8. Let a and b be positive real numbers with a+ b = 1. Prove that

a2

a+ 1+

b2

b+ 1≥ 1

3.

(Hungary 1996)

First solution: Using the condition a + b = 1, we can reduce the giveninequality to homogeneous one

1

3≤ a2

(a+ b)[a+ (a+ b)]+

b2

(a+ b)[b+ (a+ b)],

ora2b+ ab2 ≤ a3 + b3,

which follows from

(a3 + b3)− (a2b+ ab2) = (a− b)2(a+ b) ≥ 0.

The equality holds if and only if a = b =1

2.

Second solution: By the Cauchy Schwarz Inequality, we have that

a2

a+ 1+

b2

b+ 1≥ (a+ b)2

a+ 1 + b+ 1=

1

3,

as desired.?F?

96.9. Prove the following inequality for positive real numbers x, y, z,

(xy + yz + zx)

[1

(y + z)2+

1

(z + x)2+

1

(x+ y)2

]≥ 9

4.

(Iran 1996)

First solution: Without loss of generality, we may assume that x ≥ y ≥ z.Denote

P (x, y, z) =1

(y + z)2+

1

(z + x)2+

1

(x+ y)2− 9

4(xy + yz + zx).

We have to show that P (x, y, z) ≥ 0. Note that

P (t, t, z) =z(z − t)2

2t2(2z + t)(z + t)2≥ 0

for any positive reals t ≥ z > 0. Therefore, a key to solve this problem is tofind a suitable number t ≥ z such that

P (x, y, z) ≥ P (t, t, z).

261

Page 263: Олимпиадад бэлтгэгчдэд тусламж (Тэнцэтгэл биш 1)

We will prove that the number t =x+ y

2satisfies this property, i.e.

1

(x+ z)2+

1

(x+ z)2− 2

(t+ z)2≥ 9

4(xy + yz + zx)− 9

4(t2 + 2zt).

With noting that

1

(x+ z)2+

1

(x+ z)2− 2

(t+ z)2=

(1

x+ z− 1

y + z

)2

+2

(x+ z)(y + z)− 2

(t+ z)2

=(x− y)2

(x+ z)2(y + z)2+

(x− y)2

2(x+ z)(y + z)(t+ z)2,

and

9

4(xy + yz + zx)− 9

4(t2 + 2zt)=

9(x− y)2

16(t2 + 2tz)(xy + yz + zx),

we can rewrite the above inequality as

(x− y)2

(x+ z)2(y + z)2+

(x− y)2

2(x+ z)(y + z)(t+ z)2≥ 9(x− y)2

16(t2 + 2tz)(xy + yz + zx),

or equivalently,

1

(x+ z)2(y + z)2+

1

2(x+ z)(y + z)(t+ z)2≥ 9

16(xy + yz + zx)(t2 + 2tz).

We have 4(xy + yz + zx)− 3(x+ z)(y + z) = xy + yz + zx− 3z2 ≥ 0, then

(x+ z)(y + z) ≤ 4

3(xy + yz + zx),

and hence, it follows that

1

(x+ z)2(y + z)2≥ 9

16(xy + yz + zx)2≥ 9

16(xy + yz + zx)(t2 + 2zt).

Therefore, the last inequality is valid and our proof is completed. Note thatthe equality holds if and only if x = y = z.

Second solution: Due to symmetry, we can assume that x ≥ y ≥ z > 0.

Now, sincexy + yz + zx

(y + z)2=

x

y + z+

yz

(y + z)2, the original inequality can be

written as

x

y + z+

y

z + x+

z

x+ y+

xy

(x+ y)2+

yz

(y + z)2+

zx

(z + x)2≥ 9

4.

Sincex

y + z+

y

z + x= (x + y + z)

(1

x+ z+

1

y + z

)− 1, this inequality is

equivalent to

(x+ y+ z)

(1

x+ z+

1

y + z

)+

z

x+ y+

xy

(x+ y)2+

yz

(y + z)2+

zx

(z + x)2≥ 17

4,

262

Page 264: Олимпиадад бэлтгэгчдэд тусламж (Тэнцэтгэл биш 1)

or

(x+ y + z)

(1

x+ z+

1

y + z− 4

x+ y + 2z

)+

4(x+ y + z)

x+ y + 2z+

z

x+ y+

+xy

(x+ y)2+

yz

(y + z)2+

zx

(z + x)2≥ 17

4.

From this, it is not hard for us to write the inequality in the form

M(x− y)2 +Nz ≥ 0,

where

M =x+ y + z

(x+ z)(y + z)(x+ y + 2z)− 1

4(x+ y)2

≥ x+ y + z

(x+ y)(y + x)(x+ y + 2z)− 1

4(x+ y)2

=3x+ 3y + 2z

4(x+ y)2(x+ y + 2z)≥ 0,

and

N =1

x+ y+

x

(x+ z)2+

y

(y + z)2− 4

x+ y + 2z

=

(1

x+ z+

1

y + z− 4

x+ y + 2z

)+

1

x+ y− z

(x+ z)2− z

(y + z)2

=(x− y)2

(x+ z)(y + z)(x+ y + 2z)+

1

x+ y− z

(x+ z)2− z

(y + z)2.

Sincez

(x+ z)2− y

(x+ y)2= −(y − z)(x2 − yz)

(x+ y)2(x+ z)2≤ 0,

we deduce that

N ≥ (x− y)2

(x+ y)(y + y)(x+ y + 2y)+

1

x+ y− y

(x+ y)2− z

4yz

=(x− y)2

2y(x+ y)(x+ 3y)− (x− y)2

4y(x+ y)2=

(x− y)3

4y(x+ y)2(x+ 3y)≥ 0.

Therefore, in the last inequality, each element of it is nonnegative, and hence,the result follows.

?F?

96.10. Let n ≥ 2 be a fixed natural number and let a1, a2, . . . , an be positivenumbers whose sum is 1. Prove that for any positive numbers x1, x2, . . . , xnwhose sum is 1,

2∑

1≤i<j≤nxixj ≤

n− 2

n− 1+

n∑i=1

aix2i

1− ai,

and determine when equality holds.(Poland 1996)

263

Page 265: Олимпиадад бэлтгэгчдэд тусламж (Тэнцэтгэл биш 1)

Solution: The left side is 1−n∑i=1

x2i , so we can rewrite the desired result as

1

n− 1≤

n∑i=1

x2i1− ai

.

By the Cauchy Schwarz Inequality,(n∑i=1

x2i1− ai

)[n∑i=1

(1− ai)

]≥

(n∑i=1

xi

)2

= 1.

Sincen∑i=1

(1− ai) = n− 1, we have the desired result.

?F?

96.11. Let a, b, c be real numbers such that a+ b+ c = 1. Prove that

a

a2 + 1+

b

b2 + 1+

c

c2 + 1≤ 9

10.

(Poland 1996)

First solution: In order to prove this inequality, we shall consider two cases:

Case 1. If min {a, b, c} ≥ −3

4, then we have

36a+ 3

50− a

a2 + 1=

(4a+ 3)(3a− 1)2

50(a2 + 1)≥ 0,

from which it follows thata

a2 + 1≤ 36a+ 3

50, and so

a

a2 + 1+

b

b2 + 1+

c

c2 + 1≤ 36

50(a+ b+ c) +

9

50=

36

50+

9

50=

9

10.

Case 2. If min {a, b, c} < −3

4, without loss of generality, we can assume that

c < −3

4. From the AM-GM Inequality, we have

b

b2 + 1≤ 1

2, and thus, the

initial inequality is proved if we can show that

a

a2 + 1≤ 2

5.

If a ≤ 1

2or a ≥ 2, we are done. So, it suffices to consider the case 2 ≥ a ≥ 1

2.

Similarly, we can show separately that it is enough to show the case when

2 ≥ b ≥ 1

2. Therefore, we can combine these two remarks and can resume

to proving the problem under the condition 2 ≥ a, b ≥ 1

2. In this case,

−3

4> c = 1− a− b ≥ −3, which means that

c

c2 + 1≤ − 3

10. Hence

a

a2 + 1+

b

b2 + 1+

c

c2 + 1≤ 1

2+

1

2− 3

10=

7

10<

9

10.

264

Page 266: Олимпиадад бэлтгэгчдэд тусламж (Тэнцэтгэл биш 1)

This completes our proof. The equality holds if and only if a = b = c =1

3.

Second solution: Firstly, we note that at least two of the three numbers a, b,

and c are both greater than or equal to1

3or less than or equal to

1

3. Without

loss of generality, we assume that the numbers with this property are a and b.Then we have (3a− 1)(3b− 1) ≥ 0. According to this, we infer that

a2 + b2 = (a+ b)2 − 2ab = (a+ b)2 − 2

3

(a+ b− 1

3

)− 2

9(3a− 1) (3b− 1)

≤ (1− c)2 − 2

3

(1− c− 1

3

)=

9c2 − 12c+ 5

9.

Now, we rewrite our inequality as(1− 2a

a2 + 1

)+

(1− 2b

b2 + 1

)≥ 1

5+

2c

c2 + 1,

or(a− 1)2

a2 + 1+

(b− 1)2

b2 + 1≥ c2 + 10c+ 1

5(c2 + 1).

Applying the Cauchy Schwarz Inequality and the above note, we obtain

(a− 1)2

a2 + 1+

(b− 1)2

b2 + 1≥ (a+ b− 2)2

a2 + b2 + 2=

(c+ 1)2

9c2 − 12c+ 5

9+ 2

=9(c+ 1)2

9c2 − 12c+ 23.

Therefore, it suffices to prove that

9(c+ 1)2

9c2 − 12c+ 23≥ c2 + 10c+ 1

5(c2 + 1).

By some easy computations, we find that the last inequality can be simplifiedinto

2(2c2 + 2c+ 11)(3c− 1)2

5(9c2 − 12c+ 23)(c2 + 1)≥ 0,

which is true since 2c2+2c+11 > 0 and 9c2−12c+23 > 0 for any real numberc.

?F?

96.12. Let x1, x2, . . . , xn, xn+1 be positive reals such that x1 +x2 + · · ·+xn =xn+1. Prove that

n∑i=1

√xi(xn+1 − xi) ≤

√√√√ n∑i=1

xn+1(xn+1 − xi).

(Romania 1996)

First solution: First, note that

n∑i=1

xn+1(xn+1 − xi) = nx2n+1 − xn+1

n∑i=1

xi = (n− 1)x2n+1.

265

Page 267: Олимпиадад бэлтгэгчдэд тусламж (Тэнцэтгэл биш 1)

Hence, the given inequality may be rewritten as

n∑i=1

√√√√ xixn+1

·1− xi

xn+1

n− 1≤ 1.

On the other hand, by the AM-GM Inequality, the left side is at most

n∑i=1

xi2xn+1

+

1− xixn+1

2(n− 1)

=1

2+

n− 1

2(n− 1)= 1.

Second solution: From the Cauchy Schwarz Inequality, we see that

n∑i=1

√xi(xn+1 − xi) ≤

√√√√n

n∑i=1

xi(xn+1 − xi) =

√√√√nxn+1

n∑i=1

xi − nn∑i=1

x2i

√√√√nxn+1

n∑i=1

xi −

(n∑i=1

xi

)2

=√

(n− 1)x2n+1.

On the other hand, we have√√√√ n∑i=1

xn+1(xn+1 − xi) =

√√√√nx2n+1 − xn+1

n∑i=1

xi =√

(n− 1)x2n+1.

Therefore, we conclude that

n∑i=1

√xi(xn+1 − xi) ≤

√√√√ n∑i=1

xn+1(xn+1 − xi),

as desired.?F?

96.13. Let x, y, z be real numbers. Prove that the following conditions areequivalent

(i) x > 0, y > 0, z > 0 and1

x+

1

y+

1

z≤ 1;

(ii) for every quadrilateral with sides a, b, c, d, a2x+ b2y + c2z > d2.(Romania 1996)

Solution: (i)⇒ (ii). Applying the Cauhy Schwarz Inequality, we have

a2x+ b2y + c2z ≥ (a2x+ b2y + c2z)

(1

x+

1

y+

1

z

)≥ (a+ b+ c)2 > d2.

(ii) ⇒ (i). If x ≤ 0, then by taking a quadrilateral with sides a = n, b =1, c = 1, d = n, we get y + z > n2(1 − x), which is impossible for large n.

266

Page 268: Олимпиадад бэлтгэгчдэд тусламж (Тэнцэтгэл биш 1)

Therefore x > 0, and in the same way y, z > 0. Using now a quadrilateral

with sides a =1

x, b =

1

y, c =

1

z, d =

1

x+

1

y+

1

z− 1

n(where n is sufficiently

large), one has1

x2· x+

1

y2· y+

1

z2· z >

(1

x+

1

y+

1

z− 1

n

)2

, that is1

x+

1

y+

1

z>

(1

x+

1

y+

1

z− 1

n

)2

for every sufficiently large n, whence1

x+

1

y+

1

z≥(

1

x+

1

y+

1

z

)2

, and therefore1

x+

1

y+

1

z≤ 1.

?F?

96.14. Let a, b, c be positive real numbers.(a) Prove that 4(a3 + b3) ≥ (a+ b)3.(b) Prove that 9(a3 + b3 + c3) ≥ (a+ b+ c)3.

(United Kingdom 1996)

Solution: Both parts follow from the Power Mean Inequality: for r > 1 andx1, . . . , xn positive, (

xr1 + · · ·+ xrnn

) 1r

≥ x1 + · · ·+ xnn

,

which in turn follows from the Jensen’s Inequality applied to the convex func-tion xr.

?F?

96.15. Let a, b, c, d be positive real numbers such that

2(ab+ ac+ ad+ bc+ bd+ cd) + abc+ bcd+ cda+ dab = 16.

Prove that

a+ b+ c+ d ≥ 2

3(ab+ ac+ ad+ bc+ bd+ cd).

(Vietnam 1996)

Solution: By applying the Rolle’s Theorem, we see that exist positive realnumbers x, y, z such that

a+ b+ c+ d =4

3(x+ y + z)

ab+ ac+ ad+ bc+ bd+ cd = 2(xy + yz + zx)abc+ bcd+ cda+ dab = 4xyz

.

Therefore, from the given hypothesis, we have xy + yz + zx + xyz = 4, andwe need to prove that

x+ y + z ≥ xy + yz + zx.

According to the Schur’s Inequality (applied for third degree), we have

(x+ y + z)3 + 9xyz ≥ 4(x+ y + z)(xy + yz + zx).

267

Page 269: Олимпиадад бэлтгэгчдэд тусламж (Тэнцэтгэл биш 1)

Multiplying both sides of this inequality by(xy + yz + zx)2

x+ y + z, we get

(x+ y + z)2(xy + yz + zx)2 +9xyz(xy + yz + zx)2

x+ y + z≥ 4(xy + yz + zx)3.

On the other hand, the AM-GM Inequality implies that

9xyz(xy + yz + zx)2

x+ y + z≤ xyz(x+ y + z)3.

Therefore, from the above inequality, we deduce that

(x+ y + z)2(xy + yz + zx)2 + xyz(x+ y + z)3 ≥ 4(xy + yz + zx)3.

Setting p = x+ y + z, q = xy + yz + zx, this inequality is equivalent to

p2q2 + (4− q)p3 ≥ 4q3,

or

(p− q)[(4− q)p2 + 4pq + 4q2

]≥ 0.

Sine 4 ≥ q, it is clear that (4−q)p2+4pq+4q2 > 0, and hence, we conclude thatp ≥ q, as desired. Note that the equality holds if and only if a = b = c = d = 1.

?F?

96.16. Prove that for any real numbers a, b, c,

(a+ b)4 + (b+ c)4 + (c+ a)4 ≥ 4

7(a4 + b4 + c4).

(Vietnam 1996)

First solution: Let us make the substitution a+b = 2z, c+a = 2y, b+c = 2x.

The inequality becomes P =∑

(y + z − x)4 ≤ 28∑

x4. Now, we have the

following chain of identities

P =∑(∑

x2 + 2yz − 2xy − 2xz)2

= 3(∑

x2)2

+ 4(∑

x2) [∑

(yz − xy − xz)]

+ 4∑

(xy + xz − yz)2

= 3(∑

x2)2− 4

(∑x2)(∑

xy)

+ 16∑

x2y2 − 4(∑

xy)2

= 4(∑

x2)2

+ 16∑

x2y2 −(∑

x)4≤ 28

∑x4,

because(∑

x2)2≤ 3

∑x4,∑

x2y2 ≤∑

x4.

?F?

97.1. a, b, c be positive numbers such that abc = 1. Prove that

1

1 + a+ b+

1

1 + b+ c+

1

1 + c+ a≤ 1

a+ 2+

1

b+ 2+

1

c+ 2.

268

Page 270: Олимпиадад бэлтгэгчдэд тусламж (Тэнцэтгэл биш 1)

(Bulgaria 1997)

Solution: Put x = a+ b+ c and y = ab+ bc+ ca. Then the given inequalitycan be rewritten

3 + 4x+ y + x2

2x+ y + x2 + xy≤ 12 + 4x+ y

9 + 4x+ 2y,

or3x2y + xy2 + 6xy − 5x2 − y2 − 24x− 3y − 27 ≥ 0.

This inequality is equivalent to

(3x2y − 5x2 − 12x) + (xy2 − y2 − 3x− 3y) + (6xy − 9x− 27) ≥ 0,

which is true because x, y ≥ 3. Note that the equality holds if and only ifa = b = c = 1.

?F?

97.2. Prove that1

1999<

1

2· 3

4· · · 1997

1998<

1

44.

(Canada 1997)

Solution: Let us first note that for every integer n ≥ 1,

n2

(n+ 1)2=

n2

n(n+ 2) + 1<

n2

n(n+ 2)=

n

n+ 2, (1)

andn2

(n+ 1)2>

n2 − 1

(n+ 1)2=n− 1

n+ 1. (2)

From (1), we deduce that(1

2· 3

4· · · 1997

1998

)2

<1

3· 3

5· · · 1997

1999=

1

1999,

and hence1

2· 3

4· · · 1997

1998<

1√1999

<1

44.

This proves the right inequality. On the hand, from (2), we infer that(1

2· 3

4· · · 1997

1998

)2

>1

4· 2

4· 4

6· · · 1996

1998=

1

4· 2

1998=

1

3996>

1

19992.

Therefore1

2· 3

4· · · 1997

1998>

1

1999.

The left inequality is also proved and so, our proof is completed.?F?

97.3. Let x1, x2, . . . , x1997 be real numbers satisfying the following conditions

(a) − 1√3≤ xi ≤

√3 for i = 1, 2, . . . , 1997;

269

Page 271: Олимпиадад бэлтгэгчдэд тусламж (Тэнцэтгэл биш 1)

(b) x1 + x2 + · · ·+ x1997 = −318√

3.Determine the maximum value of x121 + x122 + · · ·+ x121997.

(China 1997)

Solution: Since x12 is a convex function of x, the sum of the twelfth powersof the xi is maximized by having all but perhaps one of the xi at the endpoints

of the prescribed interval. Suppose n of the xi equal − 1√3, 1996−n equal

√3

and the last one equals

−318√

3 +n√3− (1996− n)

√3.

This number must be in the range as well, so

−1 ≤ −318 · 3 + n− 3(1996− n) ≤ 3.

Equivalently −1 ≤ 4n− 6942 ≤ 3. The only such integer is n = 1736, the last

value is2√3

, and the maximum is 1736 · 3−6 + 260 · 36 +

(4

3

)6

.

?F?

97.4. For each natural number n ≥ 2, determine the largest possible value ofthe expression

Vn = sinx1 cosx2 + sinx2 cosx3 + · · ·+ sinxn cosx1,

where x1, x2, . . . , xn are arbitrary real numbers.(Czech-Slovak 1997)

Solution: By the inequality 2ab ≤ a2 + b2, we get

Vn ≤sin2 x1 + cos2 x2

2+ · · ·+ sin2 xn + cos2 x1

2=n

2,

with equality for x1 = · · · = xn =π

4.

?F?

97.5. Let x, y, z be positive real numbers. Prove that

xyz(x+ y + z +

√x2 + y2 + z2

)(x2 + y2 + z2) (xy + yz + zx)

≤ 3 +√

3

9.

(Hong Kong 1997)

Solution: We apply first the Cauchy Schwarz Inequality and then the AM-GM Inequality, and get

xyz(x+ y + z +

√x2 + y2 + z2

)(x2 + y2 + z2) (xy + yz + zx)

≤xyz

(√3 (x2 + y2 + z2) +

√x2 + y2 + z2

)(x2 + y2 + z2) (xy + yz + zx)

=

(√3 + 1

)xyz√

x2 + y2 + z2 (xy + yz + zx)

≤(√

3 + 1)xyz√

3 3√x2y2z23 3

√x2y2z2

=3 +√

3

9,

270

Page 272: Олимпиадад бэлтгэгчдэд тусламж (Тэнцэтгэл биш 1)

as desired. Note that the equality holds if and only if x = y = z.?F?

97.6. Given x1, x2, x3, x4 are positive real numbers such that x1x2x3x4 = 1.Prove that

x31 + x32 + x33 + x34 ≥ max

{x1 + x2 + x3 + x4,

1

x1+

1

x2+

1

x3+

1

x4

}.

(Iran 1997)

Solution: The problem requires us to prove that

x31 + x32 + x33 + x34 ≥ x1 + x2 + x3 + x4,

and

x31 + x32 + x33 + x34 ≥1

x1+

1

x2+

1

x3+

1

x4.

The proof of these two inequalities are very easy. Indeed, for the first inequal-ity, applying the AM-GM Inequality, for each i, we have

x3i + 1 + 1 ≥ 3xi,

and we deduce that

x31 + x32 + x33 + x34 + 8 ≥ (x1 + x2 + x3 + x4) + 2 (x1 + x2 + x3 + x4)

≥ x1 + x2 + x3 + x4 + 2 · 4 4√x1x2x3x4

= x1 + x2 + x3 + x4 + 8.

This inequality implies that

x31 + x32 + x33 + x34 ≥ x1 + x2 + x3 + x4,

which is the first one. For the second one, we apply the AM-GM Inequalityagain and get

x31 + x32 + x33 + x34 =

=1

3

[(x31 + x32 + x33

)+(x32 + x33 + x34

)+(x33 + x34 + x31

)+(x34 + x31 + x32

)]≥ 1

3

(3 3

√x31x

32x

33 + 3 3

√x32x

33x

34 + 3 3

√x33x

34x

31 + 3 3

√x34x

31x

32

)=

1

x4+

1

x1+

1

x2+

1

x3.

The proof is completed. Note that the equality holds if and only if x1 = x2 =x3 = x4 = 1.

?F?

97.7. Let a, b, c be nonnegative real numbers such that a+ b+ c ≥ abc. Provethat

a2 + b2 + c2 ≥ abc.

271

Page 273: Олимпиадад бэлтгэгчдэд тусламж (Тэнцэтгэл биш 1)

(Ireland 1997)

Solution: We may assume a, b, c > 0. Suppose by way of contradiction thata2 + b2 + c2 < abc, then abc > a2 and so a < bc, and likewise b < ca, c < ab.Then

abc ≥ a2 + b2 + c2 ≥ ab+ bc+ ca,

by the AM-GM Inequality, and the right side exceeds a+ b+ c, contradiction.?F?

97.8. Let a, b, c be positive real numbers. Prove that

(b+ c− a)2

(b+ c)2 + a2+

(c+ a− b)2

(c+ a)2 + b2+

(a+ b− c)2

(a+ b)2 + c2≥ 3

5.

(Japan 1997)

Solution: Without loss of generality, we may assume that c ≥ b ≥ a. Then,by applying the Cauchy Schwarz Inequality, we deduce that[∑ (b+ c− a)2

(b+ c)2 + a2

] [∑b2[(b+ c)2 + a2

]]≥[∑

b(b+ c− a)]2

=(∑

a2)2.

The inequality is reduced to proving that

5(∑

a2)2≥ 3

∑b2[(b+ c)2 + a2

].

After some simple computations, we find that it is equivalent to

2(a4 + b4 + c4) + 4(a2b2 + b2c2 + c2a2) ≥ 6(a3b+ b3c+ c3a).

Now, since c ≥ b ≥ a > 0, we have

a3b+ b3c+ c3a− (ab3 + bc3 + ca3) = −(b− a)(c− b)(c− a)(a+ b+ c) ≤ 0,

and hence

6(a3b+ b3c+ c3a) ≤ 3(a3b+ b3c+ c3a+ ab3 + bc3 + ca3

).

From this, we can see that the above inequality is deduced from

2(a4 + b4 + c4) + 4(a2b2 + b2c2 + c2a2) ≥ 3(a3b+ b3c+ c3a+ ab3 + bc3 + ca3

).

Now, note that

a4 + b4 + 4a2b2 − 3a3b− 3ab3 = (a2 − ab+ b2)(a− b)2 ≥ 0.

Adding this to the analogous inequalities, the result follows immediately. Theequality holds if and only if a = b = c.

?F?

97.9. Let a1, . . . , an be positive numbers, and define

A =a1 + · · ·+ an

n, G = n

√a1 · · · an, H =

n

a−11 + · · ·+ a−1n.

272

Page 274: Олимпиадад бэлтгэгчдэд тусламж (Тэнцэтгэл биш 1)

(a) If n is even, show thatA

H≤ −1 + 2

(A

G

)n.

(b) If n is odd, show thatA

H≤ −n− 2

n+

2(n− 1)

n

(A

G

)n.

(Korea 1997)

Solution: Note that

Gn

H=a1 · · · an(a−11 + · · ·+ a−1n )

n

=1

n

n∑i=1

a1 · · · anai

(1

n

n∑i=1

ai

)n−1= An−1.

by the Maclaurin’s Inequality, soA

H≤(A

G

)n. Since A ≥ G,

(A

G

)n≥ 1, so

A

H≤ −1 + 2

(A

G

)n, proving part (a). For part (b),

A

H≤(A

G

)n≤(A

G

)n+n− 2

n

[(A

G

)n− 1

]= −n− 2

n+

2(n− 1)

n

(A

G

)n.

?F?

97.10. For any positive real numbers x, y, z such that xyz = 1, prove theinequality

x9 + y9

x6 + x3y3 + y6+

y9 + z9

y6 + y3z3 + z6+

z9 + x9

z6 + z3z3 + x6≥ 2.

(Romania 1997)

Solution: Setting a = x3, b = y3, c = z3. Then our inequality becomes

a3 + b3

a2 + ab+ b2+

b3 + c3

b2 + bc+ c2+

c3 + a3

c2 + ca+ a2≥ 2.

Now, notice that for any u, v > 0, we have

u3 + v3

u2 + uv + v2− u+ v

3=

2(u+ v)(u− v)2

3(u2 + uv + v2)≥ 0.

Applying this into the left hand side of the above inequality, we deduce that

a3 + b3

a2 + ab+ b2+

b3 + c3

b2 + bc+ c2+

c3 + a3

c2 + ca+ a2≥ a+ b

3+b+ c

3+c+ a

3

=2

3· 3 3√abc = 2.

The equality holds if and only if x = y = z = 1.?F?

97.11. Let a, b, c be positive real numbers. Prove that

a2

a2 + 2bc+

b2

b2 + 2ca+

c2

c2 + 2ab≥ 1 ≥ bc

a2 + 2bc+

ca

b2 + 2ca+

ab

c2 + 2ab.

273

Page 275: Олимпиадад бэлтгэгчдэд тусламж (Тэнцэтгэл биш 1)

(Romania 1997)

Solution: Since

bc

a2 + 2bc+

ca

b2 + 2ca+

ab

c2 + 2ab=

3

2− 1

2

(a2

a2 + 2bc+

b2

b2 + 2ca+

c2

c2 + 2ab

),

we can see that the right hand side inequality follows immediately from theleft hand side inequality, so it suffices to prove the left inequality. However,this inequality is obviously true since by the Cauchy Schwarz Inequality,

a2

a2 + 2bc+

b2

b2 + 2ca+

c2

c2 + 2ab≥ (a+ b+ c)2

a2 + 2bc+ b2 + 2ca+ c2 + 2ab= 1.

The proof is completed. Note that the equality holds if and only if a = b = c.?F?

97.12. Show that if 1 < a < b < c, then

loga(loga b) + logb(logb c) + logc(logc a) > 0.

(Russia 1997)

Solution: Since loga b > 1, we have loga(loga b) > logb(loga b). Also, sincelogc a < 1, we find that logc(logc a) > logb(logc a). Thus the left side of thegiven inequality exceeds

logb(loga b logb c logc a) = 0.

?F?

97.13. Prove that for x, y, z ≥ 2,

(y3 + x)(z3 + y)(x3 + z) ≥ 125xyz.

(Saint Petersburg 1997)

First solution: The left side is at least (4y + x)(4z + y)(4x + y). By theWeighted AM-GM Inequality, 4y+x ≥ 5y4/5x1/5. This and the two analogousinequalities imply the claim. Note that the equality holds if and only if x =y = z = 2.

Second solution: Dividing both sides by xyz > 0, we can rewrite the originalinequality as (

x2 +y

x

)(y2 +

z

y

)(z3 +

x

z

)≥ 125.

Now, applying the AM-GM Inequality, we have

x2 +y

x=x2

4+x2

4+x2

4+x2

4+y

x≥ 5

5

√x7y

44.

274

Page 276: Олимпиадад бэлтгэгчдэд тусламж (Тэнцэтгэл биш 1)

Multiplying this to the two analogous inequalities, we obtain(x2 +

y

x

)(y2 +

z

y

)(z3 +

x

z

)≥ 125

5

√x8y8z8

412≥ 125

5

√224

224= 125,

as desired.

Third solution: We have to prove that(x3

y+ 1

)(y3

z+ 1

)(z3

x+ 1

)≥ 125.

This is true, since by the Holder’s Inequality, we have(x3

y+ 1

)(y3

z+ 1

)(z3

x+ 1

)≥

(1 + 3

√x3

y· y

3

z· z

3

x

)3

≥ 125.

?F?

97.14. Given an integer n ≥ 2, find the minimal value of

x51x2 + x3 + · · ·+ xn

+x52

x3 + · · ·+ xn + x1+ · · ·+ x5n

x1 + x2 + · · ·+ xn−1,

for positive real numbers x1, . . . , xn subject to the condition x21 + · · ·+x2n = 1.(Turkey 1997)

First solution: Let S = x1 + · · · + xn. By the Chebyshev’s Inequality, the

average ofx5i

S − xiis at least the average of x4i times the average of

xiS − xi

(since both are increasing functions of xi). The latter function is convex, so

its average is at least1

n− 1. We apply the Power Mean Inequality to the

former, which gives

n∑i=1

x4i

n

1/2

n∑i=1

x2i

n=

1

n. We conclude

n∑i=1

x5iS − xi

≥ n · 1

n2· 1

n− 1=

1

n(n− 1),

with equality iff x1 = · · · = xn =1√n.

Second solution: By the Cauchy Schwarz Inequality, we have

P =

n∑i=1

x5in∑i=1

xi − xi≥

(n∑i=1

x3i

)2

n∑i=1

xi

(n∑i=1

xi − xi

) =

(n∑i=1

x3i

)2

(n∑i=1

xi

)2

− 1

.

275

Page 277: Олимпиадад бэлтгэгчдэд тусламж (Тэнцэтгэл биш 1)

In addition, the Chebyshev’s Inequality implies that

n∑i=1

x3i ≥1

n

(n∑i=1

xi

)(n∑i=1

x2i

)=

1

n

(n∑i=1

xi

).

From this and the above inequality, we deduce that

P ≥ 1

n2·

(n∑i=1

xi

)2

(n∑i=1

xi

)2

− 1

=1

n2

1 +1(

n∑i=1

xi

)2

− 1

≥ 1

n2

1 +1

n

n∑i=1

x2i − 1

=1

n2

(1 +

1

n− 1

)=

1

n(n− 1).

Note that the equality holds iff x1 = x2 = · · · = xn =1√n.

?F?

97.15. Let x, y and z be positive real numbers.

(a) If x+ y + z ≥ 3, is it necessarily true that1

x+

1

y+

1

z≤ 3?

(b) If x+ y + z ≤ 3, is it necessarily true that1

x+

1

y+

1

z≥ 3?

(United Kingdom 1997)

Solution: (a) The answer is ”No”. For example, take x = 2, y = 1, z =1

3.

(b) The answer is ”Yes”, since by the Cauchy Schwarz Inequality, we have

1

x+

1

y+

1

z≥ 9

x+ y + z≥ 9

3= 3.

?F?

97.16. Prove that, for all positive real numbers a, b, c,

1

a3 + b3 + abc+

1

b3 + c3 + abc+

1

c3 + a3 + abc≤ 1

abc.

(USA 1997)

Solution: The inequality (a− b)(a2 − b2) ≥ 0 implies a3 + b3 ≥ ab(a+ b), so

1

a3 + b3 + abc≤ 1

ab(a+ b) + abc=

c

abc(a+ b+ c).

Similarly, we have

1

b3 + c3 + abc≤ a

abc(a+ b+ c), and

1

c3 + a3 + abc≤ b

abc(a+ b+ c).

276

Page 278: Олимпиадад бэлтгэгчдэд тусламж (Тэнцэтгэл биш 1)

Thus

1

a3 + b3 + abc+

1

b3 + c3 + abc+

1

c3 + a3 + abc≤ a+ b+ c

abc(a+ b+ c)=

1

abc.

?F?

98.1. Let a, b, c be positive real numbers. Prove that(1 +

a

b

)(1 +

b

c

)(1 +

c

a

)≥ 2

(1 +

a+ b+ c3√abc

).

(APMO 1998)

First solution: After expanding, we find that the original inequality is equiv-alent to (

a

b+b

c+c

a

)+

(a

c+b

a+c

b

)≥ 2 · a+ b+ c

3√abc

.

Now, we apply the AM-GM Inequality to get

3

(a

b+b

c+c

a

)=

(a

b+a

b+b

c

)+

(b

c+b

c+c

a

)+( ca

+c

a+a

b

)≥ 3

3

√a

b· ab· bc

+ 33

√b

c· bc· ca

+ 3 3

√c

a· ca· ab

=3 (a+ b+ c)

3√abc

,

and

3

(a

c+b

a+c

b

)=(ac

+a

c+c

b

)+

(b

a+b

a+a

c

)+

(c

b+c

b+b

a

)≥ 3 3

√a

c· ac· cb

+ 33

√b

a· ba· ac

+ 33

√c

b· cb· ba

=3(a+ b+ c)

3√abc

.

These two inequalities imply that

a

b+b

c+c

a≥ a+ b+ c

3√abc

, andb

a+c

b+a

c≥ a+ b+ c

3√abc

,

and hence the result follows. Note that the equality holds if and only if a =b = c = 1.

Second solution: According to the well-known inequalities (a+ b)(b+ c)(c+

a) ≥ 8

9(a+ b+ c)(ab+ bc+ ca) and ab+ bc+ ca ≥

√3abc(a+ b+ c), we infer

that (1 +

a

b

)(1 +

b

c

)(1 +

c

a

)=

(a+ b)(b+ c)(c+ a)

abc

≥ 8(a+ b+ c)(ab+ bc+ ca)

9abc

≥8(a+ b+ c)

√3(a+ b+ c)

9√abc

.

277

Page 279: Олимпиадад бэлтгэгчдэд тусламж (Тэнцэтгэл биш 1)

On the other hand, it is clear that

2

(1 +

a+ b+ c3√abc

)≤ 8(a+ b+ c)

3 3√abc

.

Therefore, it suffices to prove that

8(a+ b+ c)√

3(a+ b+ c)

9√abc

≥ 8(a+ b+ c)

3 3√abc

,

which is equivalently to √a+ b+ c

3≥ 6√abc.

Of course, this is clearly true from the AM-GM Inequality and hence, ourproof is completed.

?F?

98.2. Let x1, x2, y1, y2 be real numbers such that x21 + x22 ≤ 1. Prove theinequality

(x1y1 + x2y2 − 1)2 ≥ (x21 + x22 − 1)(y21 + y22 − 1).

(Austrian-Polish Competition 1998)

Solution: Since (x1y1 +x2y2− 1)2 = (x1y1 +x2y2)2− 2(x1y1 +x2y2) + 1, and

(x21 + x22 − 1)(y21 + y22 − 1) = (x21 + x22)(y21 + y22)− (x21 + x22 + y21 + y22) + 1,

the original inequality can be rewritten as

x21 + x22 + y21 + y22 − 2(x1y1 + x2y2) ≥ (x21 + x22)(y21 + y22)− (x1y1 + x2y2)

2,

or equivalently,

(x1 − y1)2 + (x2 − y2)2 ≥ (x1y2 − x2y1)2.

Now, since x21 + x22 ≤ 1, it suffices to prove that

(x21 + x22)[(x1 − y1)2 + (x2 − y2)2

]≥ (x1y2 − x2y1)2.

Of course, this is true since by the Cauchy Schwarz Inequality, we have

(x21 + x22)[(x1 − y1)2 + (x2 − y2)2

]≥ [x1(x2 − y2) + x2(y1 − x1)]2

= (x1y2 − x2y1)2.

?F?

98.3. If n ≥ 2 is an integer and 0 < a1 < a2 < . . . < a2n+1 are real numbers,prove the inequality

n√a1− n√a2+ n√a3−· · ·− n

√a2n+ n

√a2n+1 <

n√a1 − a2 + a3 − . . .− a2n + a2n+1.

(Balkan 1998)

278

Page 280: Олимпиадад бэлтгэгчдэд тусламж (Тэнцэтгэл биш 1)

Solution: Denote bi = n√ai, then 0 < b1 < b2 < · · · < b2n+1 and our

inequality becomes

n

√bn1 − bn2 + bn3 − · · · − bn2n + bn2n+1 > b1 − b2 + b3 − · · · − b2n + b2n+1.

Consider the function

f(t) = n

√tn − bn2 + bn3 − · · · − bn2n + bn2n+1 − (t− b2 + b3 − · · · − b2n + b2n+1),

where 0 < t < b2. We have

f ′(t) =tn−1(

tn − bn2 + bn3 − · · · − bn2n + bn2n+1

)n−1n

− 1

=bn−11[

bn1 + (bn2n+1 − bn2n) + · · ·+ (bn3 − bn2 )]n−1

n

− 1

<bn−11

(bn1 + 0 + · · ·+ 0)n−1n

− 1 = 0.

This meanst that f(t) is strictly decreasing, and hence

f(t) > f(b2) = n

√bn3 − · · · − bn2n + bn2n+1 − (b3 − · · · − b2n + b2n+1),

for any 0 < t < b2. Since 0 < b1 < b2, we infer that f(b1) > f(b2), or

n

√bn1 − bn2 + bn3 − · · · − bn2n + bn2n+1 − (b1 − b2 + b3 − · · · − b2n + b2n+1) >

> n

√bn3 − · · · − bn2n + bn2n+1 − (b3 − · · · − b2n + b2n+1).

In the same manner, we can establish the chain inequalities

n

√bn1 − bn2 + bn3 − · · · − bn2n + bn2n+1 − (b1 − b2 + b3 − · · · − b2n + b2n+1) >

> n

√bn3 − · · · − bn2n + bn2n+1 − (b3 − · · · − b2n + b2n+1)

> n

√bn5 − · · · − bn2n + bn2n+1 − (b5 − · · · − b2n + b2n+1)

> · · · > n

√bn2n−1 − bn2n + bn2n+1 − (b2n−1 − b2n + b2n+1)

> n

√bn2n − bn2n + bn2n+1 − (b2n − b2n + b2n+1) = 0,

and so, our proof is completed.?F?

98.4. Let a, b, c be positive real numbers. Prove that

a

b+b

c+c

a≥ a+ b

b+ c+b+ c

a+ b+ 1.

(Belarus 1998)

279

Page 281: Олимпиадад бэлтгэгчдэд тусламж (Тэнцэтгэл биш 1)

First solution: Note that the inequality is equivalent to

(a+ b+ c)

(a

b+b

c+c

a− 3

)≥ (a+ b+ c)

(a+ b

b+ c+b+ c

a+ b− 2

),

and thus it can be rewritten as

a2

b+b2

c+c2

a+ab

c+bc

a+ca

b− 2(a+ b+ c) ≥ (a+ b+ c)(a− c)2

(a+ b)(b+ c).

Now, since

a2

b+b2

c+c2

a− (a+ b+ c) =

(a− b)2

b+

(b− c)2

c+

(c− a)2

a,

and

ab

c+bc

a+ca

b− (a+ b+ c) =

a2(b− c)2 + b2(c− a)2 + c2(a− b)2

2abc≥ 0,

it suffices to prove that

(a− b)2

b+

(b− c)2

c+

(c− a)2

a≥ (a+ b+ c)(a− c)2

(a+ b)(b+ c).

By the Cauchy Schwarz Inequality, we get

(a− b)2

b+

(b− c)2

c≥ (a− c)2

b+ c,

and thus, it remains to show that

1

b+ c+

1

a≥ a+ b+ c

(a+ b)(b+ c).

Of course, this is obvious since it is equivalent with

b(a+ b+ c)

a(a+ b)(b+ c)≥ 0.

The equality holds if and only if a = b = c.

Second solution: After writing x =a

band y =

c

b, we get

c

a=y

x,

a+ b

b+ c=x+ 1

1 + y,

b+ c

b+ a=

1 + y

x+ 1.

One may rewrite the inequality as

x3y2 + x2 + x+ y3 + y2 ≥ x2y + 2xy + 2xy2.

Now, we apply the AM-GM Inequality to obtain

x3y2 + x

2≥ x2y, x3y2 + x+ y3 + y3

2≥ 2xy2, x2 + y2 ≥ 2xy.

280

Page 282: Олимпиадад бэлтгэгчдэд тусламж (Тэнцэтгэл биш 1)

Adding up these three inequalities, we get the result.?F?

98.5. Let n be a natural number such that n ≥ 2. Prove that

1

n+ 1

(1 +

1

3+ . . .+

1

2n− 1

)>

1

n

(1

2+

1

4+ . . .+

1

2n

).

(Canada 1998)

Solution: The inequality is equivalent to

1

n+ 1

n∑k=1

1

2k − 1>

1

n

n∑k=1

1

2k, or

n∑k=1

n+ 1− 2k

2k(2k − 1)> 0.

Now, we see that

n+ 1− 2 · 1 > n+ 1− 2 · 2 > · · · > n+ 1− 2 · n,

and1

1 · 2>

1

3 · 4> · · · > 1

2n(2n− 1).

Therefore, applying the Chebyshev’s Inequality, we have

n∑k=1

n+ 1− 2k

2k(2k − 1)≥ 1

n

[n∑k=1

(n+ 1− 2k)

][n∑k=1

1

2k(2k − 1)

]

=1

n

[n(n+ 1)− 2

n∑k=1

k

][n∑k=1

1

2k(2k − 1)

]= 0.

On the other hand, it is clear that the equality cannot occur, so we must have

n∑k=1

n+ 1− 2k

2k(2k − 1)> 0,

and the proof is completed.?F?

98.6. Let n ≥ 2 be a positive integer and let x1, x2, . . . , xn be real numberssuch that

n∑i=1

x2i +

n−1∑i=1

xixi+1 = 1.

For every positive integer k, 1 ≤ k ≤ n, determine the maximum value of |xk| .(China 1998)

Solution: For convenience, let us denote x0 = xn+1 = 0. Now, we will provefirst that the following inequality holds for every n ≥ 1,

x20 + x21 + · · ·+ x2n + x0x1 + · · ·+ xn−1xn + xnx0 ≥n+ 1

2nx2m,

281

Page 283: Олимпиадад бэлтгэгчдэд тусламж (Тэнцэтгэл биш 1)

wherem = 1 orm = n, and the equality holds when x1 = − n

n− 1x2, . . . , xn−1 =

−2xn for n ≥ 2,m = 1 and xn = − n

n− 1xn−1, . . . , x2 = −2x1 for n ≥ 2,m =

n. We will prove this statement by induction on n. And with noting that theproofs for m = 1 and m = n are quite similar, it suffices to prove the state-ment for m = 1 (by induction, of course). For n = 1 and n = 2, it is clear.Suppose that the statement holds for n = k ≥ 2 and let us prove that it holdsfor n = k + 1. Indeed, from the inductive hypothesis, we see that for anyy0, . . . , yk,

y20 + y21 + · · ·+ y2k + y0y1 + · · ·+ yk−1yk + yky0 ≥k + 1

2ky21,

Choosing (y0, y1, . . . , yk) = (0, x2, . . . , xk+1), we get

x22 + · · ·+ x2k+1 + x2x3 + · · ·+ xkxk+1 ≥k + 1

2kx22.

On the other hand, we have

x21 +k + 1

2kx22 + x1x2 −

k + 2

2(k + 1)x21 =

[kx1 + (k + 1)x2]2

2k (k + 1)≥ 0.

Using this in combination with the above inequality, it is easy to deduce that

x20 + x21 + · · ·+ x2k+1 + x0x1 + · · ·+ xkxk+1 + xk+1x0 ≥k + 2

2(k + 1)x21.

This proves our statement. Now, turning back to our problem, applying theabove inequality, we can see that

1 =n∑i=1

x2i +n−1∑i=1

xixi+1 =n∑i=0

x2i +n∑i=0

xixi+1 ≥n+ 1

2nx21.

Therefore max |x1| =√

2n

n+ 1, with equality if x1 = ±

√2n

n+ 1, x2 = −n− 1

nx1,

. . . , xn = −1

2xn−1. Similarly, we can find that max |xn| =

√2n

n+ 1. These are

also all values we need to find for the case n = 2. In case n ≥ 3, for all2 ≤ k ≤ n− 1, we have

1 =

n∑i=1

x2i +

n−1∑i=1

xixi+1

=(x20 + x21 + · · ·+ x2k−1 + x0x1 + · · ·+ xk−2xk−1 + xk−1x0

)+ (x20 + x2k+1+

+ · · ·+ x2n + x0xk+1 + · · ·+ xn−1xn + xnx0) + x2k + xk−1xk + xkxk+1

≥ k

2(k − 1)x2k−1 +

n− k + 1

2(n− k)x2k+1 + x2k + xk−1xk + xkxk+1

=[kxk−1 + (k − 1)xk]

2

2k(k − 1)+

[(n− k)xk + (n− k + 1)xk+1]2

2(n− k)(n− k + 1)+

n+ 1

2k(n+ 1− k)x2k

≥ n+ 1

2k(n+ 1− k)x2k.

282

Page 284: Олимпиадад бэлтгэгчдэд тусламж (Тэнцэтгэл биш 1)

Hence |xk| ≤√

2k(n+ 1− k)

n+ 1, and it is easy to check that the equality can be

attained (according to the equality of the statement we have proved above),thus

max |xk| =√

2k(n+ 1− k)

n+ 1.

?F?

98.7. Let a, b, c ≥ 1. Prove that

√a− 1 +

√b− 1 +

√c− 1 ≤

√c (ab+ 1).

(Hong Kong 1998)

Solution: The given inequality is a consequence of the Cauchy Schwarz In-equality. Indeed, from the Cauchy Schwarz Inequality, we get

√a− 1 +

√b− 1 +

√c− 1 ≤

√(1 + b− 1) (a− 1 + 1) +

√c− 1 =

√ab+

√c− 1

≤√

(1 + c− 1) (ab+ 1) =√c (ab+ 1),

as desired.?F?

98.8. Let a1, a2, . . . , an > 0 such that a1 + a2 + · · ·+ an < 1. Prove that

a1a2 · · · an (1− a1 − a2 − · · · − an)

(a1 + a2 + · · ·+ an) (1− a1) (1− a2) · · · (1− an)≤ 1

nn+1.

(IMO Shortlist 1998)

Solution: Put an+1 = 1−a1−a2−· · ·−an. The original inequality becomes

a1a2 · · · anan+1

(1− a1) (1− a2) · · · (1− an) (1− an+1)≤ 1

nn+1.

Now, for each i (1 ≤ i ≤ n+ 1), we apply the AM-GM Inequality as follows

1− ai =∑j 6=i

aj ≥ n n

√∏j 6=i

aj .

Setting i = 1, 2, . . . , n+1, we get the n+1 similar inequalities and multiplyingthem up, we obtain

(1− a2) (1− a2) · · · (1− an) (1− an+1) ≥ nn+1 n

√an1a

n2 · · · annann+1

= nn+1a1a2 · · · anan+1.

From this, it follows that

a1a2 · · · anan+1

(1− a1) (1− a2) · · · (1− an) (1− an+1)≤ 1

nn+1,

283

Page 285: Олимпиадад бэлтгэгчдэд тусламж (Тэнцэтгэл биш 1)

as desired. The equality holds if and only if a1 = a2 = · · · = an =1

n+ 1.

?F?

98.9. Let a, b, c be positive real numbers such that abc = 1. Prove that

a3

(1 + b) (1 + c)+

b3

(1 + a) (1 + c)+

c3

(1 + a) (1 + b)≥ 3

4.

(IMO Shortlist 1998)

Solution: Applying the AM-GM Inequality, we have

a3

(1 + b) (1 + c)+

1 + b

8+

1 + c

8≥ 3 3

√a3 (1 + b) (1 + c)

82 (1 + b) (1 + c)=

3a

4.

Adding this to the two analogous inequalities, we deduce that∑ a3

(1 + b)(1 + c)+a+ b+ c+ 3

4≥ 3(a+ b+ c)

4,

and hence, it follows that

∑ a3

(1 + b)(1 + c)≥ a+ b+ c

2− 3

4≥ 3 3√abc

2− 3

4,

as desired. The equality holds if and only if a = b = c = 1.?F?

98.10. Let x, y, z > 1 such that1

x+

1

y+

1

z= 2. Prove that

√x+ y + z ≥

√x− 1 +

√y − 1 +

√z − 1.

(Iran 1998)

First solution: By applying the Cauchy Schwarz Inequality, we have that

(√x− 1 +

√y − 1 +

√z − 1

)2=

(√x− 1

x·√x+

√y − 1

y· √y +

√z − 1

z·√z

)2

≤ (x+ y + z)

(x− 1

x+y − 1

y+z − 1

z

)= (x+ y + z)

(3− 1

x− 1

y− 1

z

)= (x+ y + z) (3− 2) = x+ y + z,

and then, we deduce that

√x− 1 +

√y − 1 +

√z − 1 ≤

√x+ y + z,

as desired. Note that the equality holds if and only if x = y = z =3

2.

284

Page 286: Олимпиадад бэлтгэгчдэд тусламж (Тэнцэтгэл биш 1)

Second solution: After the algebraic substitution a =1

x, b =

1

y, c =

1

z, we

are required to prove that√1

a+

1

b+

1

c≥√

1− aa

+

√1− bb

+

√1− cc

,

where a, b, c ∈ (0, 1) and a+ b+ c = 2. Using the constraint a+ b+ c = 2, weobtain a homogeneous inequality

√1

2(a+ b+ c)

(1

a+

1

b+

1

c

)≥∑√√√√ a+ b+ c

2− a

a,

or √(a+ b+ c)

(1

a+

1

b+

1

c

)≥√b+ c− a

a+

√c+ a− b

b+

√a+ b− c

c,

which immediately follows from the Cauchy Schwarz Inequality√[(b+ c− a) + (c+ a− b) + (a+ b− c)]

(1

a+

1

b+

1

c

)≥

≥√b+ c− a

a+

√c+ a− b

b+

√a+ b− c

c.

?F?

98.11. Suppose that a1 < a2 < · · · < an are real numbers. Prove that

a1a42 + a2a

43 + · · ·+ an−1a

4n + ana

41 ≥ a2a41 + a3a

42 + · · ·+ ana

4n−1 + a1a

4n.

(Iran 1998)

Solution: We prove the result by induction on n. For n = 2, we have equality.The case n = 3 will be needed below. For n = 3, we have to show that

a1a42 + a2a

43 + a3a

41 ≥ a2a41 + a3a

42 + a1a

43.

This is true, since

a1a42 + a2a

43 + a3a

41 − (a2a

41 + a3a

42 + a1a

43) =

=1

2(a2 − a1)(a3 − a2)(a3 − a1)[(a1 + a2)

2 + (a2 + a3)2 + (a3 + a1)

2] ≥ 0.

Assume that the claim is true for n− 1, and let us prove it for n. By applyingthe induction hypothesis, we find that it is sufficient to prove that

an−1a4n + ana

41 − an−1a41 ≥ ana4n−1 + a1a

4n − a1a4n−1,

which is the case n = 3.?F?

285

Page 287: Олимпиадад бэлтгэгчдэд тусламж (Тэнцэтгэл биш 1)

98.12. Show that if x is a nonzero real number, then

x8 − x5 − 1

x+

1

x4≥ 0.

(Ireland 1998)

Solution: We have

x8 − x5 − 1

x+

1

x4=

(x8 − 1

x

)−(x5 − 1

x4

)=x9 − 1

x− x9 − 1

x4

=(x9 − 1)(x3 − 1)

x4=

(x3 − 1)2(x6 + x3 + 1)

x4≥ 0.

The equality holds if and only if x = 1.?F?

98.13. Prove that if a, b, c are positive real numbers, then

9

a+ b+ c≤ 2

(1

a+ b+

1

b+ c+

1

c+ a

),

and1

a+ b+

1

b+ c+

1

c+ a≤ 1

2

(1

a+

1

b+

1

c

).

(Ireland 1998)

Solution: Both inequalities are the consequences of the Cauchy Schwarz In-equality. Indeed, from the Cauchy Schwarz Inequality, we have that

2

(1

a+ b+

1

b+ c+

1

c+ a

)≥ 2 · 9

(a+ b) + (b+ c) + (c+ a)=

9

a+ b+ c,

and

1

2

(1

a+

1

b+

1

c

)=

(1

4a+

1

4b

)+

(1

4b+

1

4c

)+

(1

4c+

1

4a

)≥ 4

4a+ 4b+

4

4b+ 4c+

4

4c+ 4a

=1

a+ b+

1

b+ c+

1

c+ a.

The equality (in both inequalities) occurs if and only if a = b = c.?F?

98.14. If x, y, z are positive real numbers such that x+ y + z = xyz, then

1√1 + x2

+1√

1 + y2+

1√1 + z2

≤ 3

2.

(Korea 1998)

286

Page 288: Олимпиадад бэлтгэгчдэд тусламж (Тэнцэтгэл биш 1)

Solution: Applying the AM-GM Inequality, we find that

1√x2 + 1

=1√

x2 + xyzx+y+z

=

√x+ y + z

x(x+ y)(x+ z)

=

√yz

(x+ y)(x+ z)≤ y

2(x+ y)+

z

2(x+ z).

Adding this to the two analogous inequalities, we conclude that

1√1 + x2

+1√

1 + y2+

1√1 + z2

≤ x+ y

2(x+ y)+

y + z

2(y + z)+

z + x

2(z + x)=

3

2,

as desired. Note that the equality holds if and only if x = y = z =√

3.?F?

98.15. Let a, b, c, d, e, f be positive real numbers such that

a+ b+ c+ d+ e+ f = 1, ace+ bdf ≥ 1

108.

Prove that

abc+ bcd+ cde+ def + efa+ fab ≤ 1

36.

(Poland 1998)

Solution: Put M = ace + bdf and N = abc + bcd + cde + def + efa + fab.Applying the AM-GM Inequality, we find that

M +N = (a+ d) (b+ e) (c+ f) ≤(a+ d+ b+ e+ c+ f

3

)3

=1

27,

and hence, we deduce that

N ≤ 1

27−M ≤ 1

27− 1

108=

1

36,

as desired.?F?

98.16. Let a be a positive real numbers and let x1, x2, . . . , xn be positive realnumbers such that x1 + x2 + · · ·+ xn = 1. Prove that

ax1−x2

x1 + x2+

ax2−x3

x2 + x3+ · · ·+ axn−x1

xn + x1≥ n2

2.

(Serbia 1998)

Solution: The original inequality is a direct consequence of the AM-GMInequality. Indeed, denote with P the left hand side of the original inequality,

287

Page 289: Олимпиадад бэлтгэгчдэд тусламж (Тэнцэтгэл биш 1)

then by applying the AM-GM Inequality, we have that

P ≥ n n

√ax1−x2

x1 + x2· a

x2−x3

x2 + x3· · · a

xn−x1

xn + x1

= n n

√a(x1−x2)+(x2−x3)+···+(xn−x1)

(x1 + x2) (x2 + x3) · · · (xn + x1)

≥ n · n

(x1 + x2) + (x2 + x3) + · · ·+ (xn + x1)=n2

2.

The equality holds if and only if x1 = x2 = · · · = xn =1

n.

?F?

98.17. Find the minimum of the expression

F (x, y) =√

(x+ 1)2 + (y − 1)2+√

(x− 1)2 + (y + 1)2+√

(x+ 2)2 + (y + 2)2,

where x, y are real numbers.(Vietnam 1998)

First solution: According to the Cauchy Schwarz Inequality, we have

√(x+ 1)2 + (y − 1)2 =

√[(√3 + 1

)2+(√

3− 1)2]

[(1− y)2 + (x+ 1)2]

2√

2

≥(1− y)

(√3 + 1

)+ (1 + x)

(√3− 1

)2√

2.

Similarly, we find that

√(x− 1)2 + (y + 1)2 ≥

(1− x)(√

3 + 1)

+ (1 + y)(√

3− 1)

2√

2.

On the other hand, the Cauchy Schwarz Inequality also implies√(x+ 2)2 + (y + 2)2 ≥ (x+ 2) + (y + 2)√

2.

Adding up these three inequalities, we get F (x, y) ≥√

6+2√

2. As we can see,

this upper bound is attained for x = y = − 1√3, and so the minimum value we

are looking for is√

6 + 2√

2.

Second solution: In the coordinate plane, consider four points A(−1, 1),B(1,−1), C(−2,−2) and M(x, y). Then we can reformulate the problem in ageometric ones as follows: Find a point M such that the sum of its distancesto the vertices of the triangle ABC is minimum. Note that this is an acutetriangle. The well-known solution is that all angles ∠AMB = ∠BMC =∠CMA = 120◦. From this, one can easily deduce that the minimum of F (x, y)is√

6 + 2√

2.

288

Page 290: Олимпиадад бэлтгэгчдэд тусламж (Тэнцэтгэл биш 1)

?F?

98.18. Let n ≥ 2 and x1, x2, . . . , xn be positive real numbers satisfying

1

x1 + 1998+

1

x2 + 1998+ · · ·+ 1

xn + 1998=

1

1998.

Prove thatn√x1x2 · · ·xnn− 1

≥ 1998.

(Vietnam 1998)

Solution: For each i (1 ≤ i ≤ n), the AM-GM Inequality gives us

xi1998(xi + 1998)

=1

1998− 1

xi + 1998=∑j 6=i

1

xj + 1998≥ n− 1

n−1

√∏j 6=i

(xj + 1998)

.

From this, it follows that

xi ≥ 1998(n− 1)n−1

√√√√√(xi + 1998)n−1∏j 6=i

(xj + 1998).

Setting i = 1, 2, . . . , n, we get the n similar inequalities and multiplying themup, the result follows immediately. Note that the equality holds if and only ifx1 = x2 = · · · = xn = 1998(n− 1).

?F?

99.1. Let {an} be a sequence of real numbers such that ai+j ≤ ai + aj for alli, j. Prove that the following inequality holds

a1 +a22

+ · · ·+ ann≥ an.

(APMO 1999)

Solution: We will prove the required inequality by induction on n. For n = 1and n = 2, it is clear. Suppose that the inequality holds for n = k ≥ 2 and letus prove it for n = k + 1. Indeed, from the inductive hypothesis, we have

a1 ≥ a1, a1 +a22≥ a2, . . . , a1 +

a22

+ · · ·+ akk≥ ak.

Adding up these k inequalities, we get

ka1 +(k − 1)a2

2+ · · ·+ ak

k≥ a1 + a2 + · · ·+ ak.

This inequality is equivalent to

(k + 1)(a1 +

a22

+ · · ·+ akk

)≥ 2(a1 + a2 + · · ·+ ak).

289

Page 291: Олимпиадад бэлтгэгчдэд тусламж (Тэнцэтгэл биш 1)

On the other hand, from the given condition, we have

2k∑i=1

ai =k∑i=1

ai +k∑i=1

ai =k∑i=1

ai +k∑i=1

ak+1−i

=

k∑i=1

(ai + ak+1−i) ≥k∑i=1

ai+k+1−i = kak+1.

From this and the above inequality, we deduce that

a1 +a22

+ · · ·+ akk≥ k

k + 1ak+1,

or

a1 +a22

+ · · ·+ akk

+ak+1

k + 1≥ ak+1,

as desired.?F?

99.2. Let a, b, c be positive real numbers such that a2 + b2 + c2 = 3. Provethat

1

1 + ab+

1

1 + bc+

1

1 + ca≥ 3

2.

(Belarus 1999)

Solution: From the Cauchy Schwarz Inequality and the AM-GM Inequality,we deduce that

1

1 + ab+

1

1 + bc+

1

1 + ca≥ (1 + 1 + 1)2

1 + ab+ 1 + bc+ 1 + ca

≥ 9

3 + a2 + b2 + c2=

3

2,

as desired. The equality holds if and only if a = b = c = 1.?F?

99.3. For any nonnegative real numbers x, y, z such that x+ y+ z = 1, provethe following inequality

x2y + y2z + z2x ≤ 4

27.

(Canada 1999)

Solution: Assume without loss of generality that x = max{x, y, z}. We havetwo cases

+ If x ≥ y ≥ z, then

x2y + y2z + z2x ≤ x2y + y2z + z2x+ z[xy + (x− y)(y − z)]

= (x+ z)2y = 4

(1

2− y

2

)(1

2− y

2

)y ≤ 4

27,

290

Page 292: Олимпиадад бэлтгэгчдэд тусламж (Тэнцэтгэл биш 1)

where the last inequality follows from the AM-GM Inequality. The equality

occurs if and only if z = 0 (from the first inequality) and y =1

3, in which case

(x, y, z) =

(2

3,1

3, 0

).

+ If x ≥ z ≥ y, then

x2y + y2z + z2x = x2z + z2y + y2x− (x− z)(z − y)(x− y)

≤ x2z + z2y + y2x ≤ 4

27,

where the second inequality is true from the result we proved for x ≥ y ≥z (except with y and z reversed). The quality holds in the first inequalityonly when two of x, y, z are equal, and in the second only when (x, z, y) =(

2

3,1

3, 0

). Since these conditions cant both be true, the inequality is actually

strict in this case.

Therefore the inequality is indeed true, and the equality holds when (x, y, z)

equals

(2

3,1

3, 0

), or

(1

3, 0,

2

3

), or

(0,

2

3,1

3

).

?F?

99.4. Let a, b, c be positive real numbers. Prove that

a

b+ 2c+

b

c+ 2a+

c

a+ 2b≥ 1.

(Czech-Slovak 1999)

Solution: By applying Cauchy Schwarz Inequality in combination with thewell-known inequality (a+ b+ c)2 ≥ 3(ab+ bc+ ca), we get

a

b+ 2c+

b

c+ 2a+

c

a+ 2b≥ (a+ b+ c)2

a (b+ 2c) + b (c+ 2a) + c (a+ 2b)

≥ 3 (ab+ bc+ ca)

3 (ab+ bc+ ca)= 1,

as desired. The equality holds if and only if a = b = c.?F?

99.5. Let n ≥ 2 be a fixed integer. Find the least constant C such that theinequality ∑

1≤i<j≤nxixj(x

2i + x2j ) ≤ C(x1 + x2 + · · ·+ xn)4

holds for every x1, . . . , xn ≥ 0. For this constant C, characterize the instancesof equality.

(IMO 1999)

291

Page 293: Олимпиадад бэлтгэгчдэд тусламж (Тэнцэтгэл биш 1)

First solution: Applying the AM-GM Inequality, we have

∑1≤i<j≤n

xixj(x2i + x2j ) ≤

∑1≤i<j≤n

xixj

x2i + x2j +∑

k 6=i,k 6=jx2k

=

∑1≤i<j≤n

xixj

( n∑i=1

x2i

)

=1

2∑

1≤i<j≤nxixj

·( n∑i=1

x2i

)

≤ 1

2

2∑

1≤i<j≤nxixj +

n∑i=1

x2i

2

2

=1

8

(n∑i=1

xi

)4

.

The equality holds if and only if xixj(x2i + x2j ) = xixj(x

21 + x22 + · · ·+ x2n) for

any i < j and x21 + x22 + · · · + x2n = 2∑

1≤i<j≤nxixj , which holds if and only

if n − 2 of the xi are zero and the remaining two are equal. And since the

equality can occur, it follows that1

8is the smallest possible value of C.

Second solution: For x1 = x2 = · · · = xn = 0, it holds for any C ≥ 0.Hence, we consider the case when x1 + x2 + · · ·+ xn > 0. Since the inequalityis homogeneous, we may normalize to x1 + x2 + · · ·+ xn = 1. We denote

F (x1, x2, . . . , xn) =∑

1≤i<j≤nxixj(x

2i + x2j ).

From the assumption x1 + x2 + · · ·+ xn = 1, we have

F (x1, x2, . . . , xn) =∑

1≤i<j≤nx3ixj +

∑1≤i<j≤n

xix3j =

∑1≤i≤n

x3i∑j 6=i

xi

=

n∑i=1

x3i (1− xi) =

n∑i=1

xi(x2i − x3i ).

We claim that C =1

8. It suffices to show that

F (x1, x2, . . . , xn) ≤ 1

8= F

(1

2,1

2, 0, . . . , 0

).

Now, note that for any 0 ≤ x ≤ y ≤ 1

2, we have

x2 − x3 ≤ y2 − y3.

Indeed, since x+y ≤ 1, we get x+y ≥ (x+y)2 ≥ x2+xy+y2. Since y−x ≥ 0,this implies that y2 − x2 ≥ y3 − x3 or y2 − y3 ≥ x2 − x3, as desired.

292

Page 294: Олимпиадад бэлтгэгчдэд тусламж (Тэнцэтгэл биш 1)

To prove our claim, we need to consider two cases

The first one is when1

2≥ x1 ≥ x2 ≥ · · · ≥ xn. In this case, we apply the

above note and get

n∑i=1

xi(x2i − x3i ) ≤

n∑i=1

xi

[(1

2

)2

−(

1

2

)3]

=1

8

n∑i=1

xi =1

8.

The second case is when x1 ≥1

2≥ x2 ≥ . . . ≥ xn. Let x1 = x and y = 1−x =

x2 + . . . + xn. Since1

2≥ y ≥ x2, . . . , xn, we can apply the above note again

to get

F (x1, . . . , xn) = x3y+n∑i=2

xi(x2i−x3i ) ≤ x3y+

n∑i=2

xi(y2−y3) = x3y+y(y2−y3).

Since x3y + y(y2 − y3) = x3y + y3(1 − y) = xy(x2 + y2), it remains to showthat

xy(x2 + y2) ≤ 1

8.

Using x+ y = 1, we homogenize the above inequality as following

xy(x2 + y2) ≤ 1

8(x+ y)4.

However, we immediately find that (x+ y)4 − 8xy(x2 + y2) = (x− y)4 ≥ 0.?F?

99.6. For real numbers x1, x2, . . . , x6 ∈ [0, 1], prove the inequality

x31x52 + x53 + x54 + x55 + x56 + 5

+ · · ·+ x36x51 + x52 + x53 + x54 + x55 + 5

≤ 3

5.

(Ukraine 1999)

Solution: Since x1, . . . , x6 are in the interval [0, 1],

x52 + x53 + x54 + x55 + x56 + 5 ≥ x51 + x52 + x53 + x54 + x55 + x56 + 4.

By permuting the subscripts, we see that the left side of the inequality in theproblem is at most

6∑i=1

x3ix51 + x52 + · · ·+ x56 + 4

=

6∑i=1

x3i

6∑i=1

x5i + 4

.

For y ≥ 0, the AM-GM Inequality gives us 3y5 + 2 ≥ 5y3. Thus

56∑i=1

x3i ≤6∑i=1

(3x5i + 2) = 3

(6∑i=1

x5i + 4

),

293

Page 295: Олимпиадад бэлтгэгчдэд тусламж (Тэнцэтгэл биш 1)

that is6∑i=1

x3i

6∑i=1

x5i + 4

≤ 3

5.

This, together with our initial observations, completes the proof.?F?

99.7. Nonnegative real numbers p, q and r satisfy p+ q + r = 1. Prove that

7(pq + qr + rp) ≤ 2 + 9pqr.

(United Kingdom 1999)

Solution: According to the Schur’s Inequality (applied for third degree), wehave

4(p+ q + r)(pq + qr + rp) ≤ (p+ q + r)3 + 9pqr.

Using the constraint p+ q + r = 1, we deduce that

pq + qr + rp ≤ 1 + 9pqr

4.

Therefore, it suffices to prove that

7 · 1 + 9pqr

4≤ 2 + 9pqr, or equivalently, 27pqr ≤ 1.

Applying the AM-GM Inequality, we see that

27pqr ≤ (p+ q + r)3 = 1.

Therefore, the last inequality is obviously true and our proof is completed.

The equality holds if and only if p = q = r =1

3.

?F?

99.8. Let n > 3 and a1, a2, . . . , an be real numbers such that a1+a2+· · ·+an ≥n and a21 + a22 + · · ·+ a2n ≥ n2. Prove that max {a1, a2, . . . , an} ≥ 2.

(USA 1999)

First solution: The most natural idea is to suppose that ai < 2 for all i and

to substitute xi = 2 − ai > 0. Then we have

n∑i=1

(2 − xi) ≥ n or

n∑i=1

xi ≤ n,

and also

n2 ≤n∑i=1

a2i =

n∑i=1

(2− xi)2 = 4n− 4

n∑i=1

xi +

n∑i=1

x2i .

Now, using the fact that xi > 0, we obtain

n∑i=1

x2i <

(n∑i=1

xi

)2

, which com-

bined with the above inequality yields

n2 < 4n− 4n∑i=1

xi +

(n∑i=1

xi

)2

≤ 4n+ (n− 4)

n∑i=1

xi.

294

Page 296: Олимпиадад бэлтгэгчдэд тусламж (Тэнцэтгэл биш 1)

(we have used the fact thatn∑i=1

xi ≤ n). Thus, we have (n−4)

(n∑i=1

xi − n

)>

0, which is clearly impossible since n ≥ 4 and

n∑i=1

xi ≤ n. So, our assumption

was wrong and consequently max{a1, a2, . . . , an} ≥ 2.

Second solution: Assume that ai < 2 for all i = 1, 2, . . . , n, then we have

a1 + a2 + · · ·+ ai − 2(i− 1) < 2 ∀i = 1, 2, . . . , n.

Now, notice that for any m,n < 2, we have

(m+ n− 2)2 + 22 −m2 − n2 = 2(2−m)(2− n) > 0.

Using this inequality in combination with the above observation, we have

(a1 + a2 − 2)2 + 22 > a21 + a22,

[a1 + a2 + a3 − 2(3− 1)]2 + 22 > (a1 + a2 − 2)2 + a23,· · · · · · · · · · · · · · · · · · · · ·

[a1 + a2 + . . .+ an − 2(n− 1)]2 + 22 > [a1 + a2 + . . .+ an−1 − 2(n− 2)]2 + a2n.

Summing up these inequalities, we get

[a1 + a2 + · · ·+ an − 2(n− 1)]2 + 4(n− 1) > a21 + a22 + · · ·+ a2n.

Since ai < 2, n ≥ 4 and a1 + a2 + . . .+ an ≥ n, we have

n− 2 ≥ 2 > a1 + a2 + · · ·+ an − 2(n− 1) ≥ 2− n.

From here, we deduce that

[a1 + a2 + · · ·+ an − 2(n− 1)]2 ≤ (n− 2)2,

and hencea21 + a22 + · · ·+ a2n < (n− 2)2 + 4(n− 1) = n2,

which contradicts with the given hypothesis. So, our assumption was wrong,or in the other words, we must have max{a1, a2, . . . , an} ≥ 2.

?F?

99.9. Let a0, a1, . . . , an be numbers from the interval(

0,π

2

)such that

tan(a0 −

π

4

)+ tan

(a1 −

π

4

)+ · · ·+ tan

(an −

π

4

)≥ n− 1.

Prove thattan a0 tan a1 · · · tan an ≥ nn+1.

(USA 1999)

Solution: Let tan ai = xi for all i = 0, 1, . . . , n. It follows from the hypothesisthat for each i, xi > 0, and

x0 − 1

x0 + 1+x1 − 1

x1 + 1+ · · ·+ xn − 1

xn + 1≥ n− 1,

295

Page 297: Олимпиадад бэлтгэгчдэд тусламж (Тэнцэтгэл биш 1)

or equivalently,

1 ≥ 1

x0 + 1+

1

x1 + 1+ · · ·+ 1

xn + 1.

Now, for each i (0 ≤ i ≤ n), the AM-GM Inequality gives us

xixi + 1

= 1− 1

xi + 1≥∑j 6=i

1

xj + 1≥ n

n

√∏j 6=i

(xj + 1)

,

and hence, it follows that

xi ≥ nn

√√√√√ (xi + 1)n∏j 6=i

(xj + 1).

Setting i = 0, 1, . . . , n, we get the n + 1 similar inequalities and multiplyingthem up, we deduce that

x0x1 · · ·xn ≥ nn+1, or equivalently, tan a0 tan a1 · · · tan an ≥ nn+1,

as desired. The equality holds if and only if a0 = a1 = · · · = an = arctann.?F?

99.10. Let x, y, z > 1. Prove that

xx2+2yzyy

2+2zxzz2+2xy ≥ (xyz)xy+yz+zx .

(USA (Shortlist) 1999)

Solution: The original inequality is equivalent to(x2 + 2yz

)lnx+

(y2 + 2zx

)ln y +

(z2 + 2xy

)ln z ≥ (xy + yz + zx) ln (xyz) ,

or

(x− y) (x− z) lnx+ (y − x) (y − z) ln y + (z − x) (z − y) ln z ≥ 0.

Without loss of generality, we may assume that x ≥ y ≥ z. By this assumption,we have (x−y)(x−z) ≥ 0, (z−x)(z−y) ≥ 0. Therefore, with noting x, y, z > 1,we infer that

(x− y) (x− z) lnx ≥ (x− y)(x− z) ln y, and (z − x)(z − y) ln z ≥ 0.

From this, we can see that the left hand side of the above inequality is notless than

(x− y)(x− z) ln y + (y − z)(y − x) ln y = (x− y)2 ln y ≥ 0.

Our proof is completed. Note that the equality holds if and only if x = y = z.?F?

296

Page 298: Олимпиадад бэлтгэгчдэд тусламж (Тэнцэтгэл биш 1)

00.1. Let a, b be real numbers and a 6= 0. Prove that

a2 + b2 +1

a2+b

a≥√

3.

(Austria 2000)

Solution: Applying the AM-GM Inequality, we get

a2 + b2 +1

a2+b

a=

(b+

1

2a

)2

+ a2 +3

4a2≥ a2 +

3

4a2≥ 2

√a2 · 3

4a2=√

3,

as desired. The equality holds if and only if b = − 1

2aand a2 =

3

4a2, i.e. when

(a, b) equals

(4

√3

4,−1

24

√4

3

), or

(− 4

√3

4,1

24

√4

3

).

?F?

00.2. For all real numbers a, b, c ≥ 0 such that a+ b+ c = 1, prove that

2 ≤ (1− a2)2 + (1− b2)2 + (1− c2)2 ≤ (1 + a)(1 + b)(1 + c).

(Austrian-Polish Competition 2000)

Solution: Let us first prove the left inequality. Without loss of generality,

assume that a = max {a, b, c} . By this assumption, we have a ≥ 1

3. Now, with

noting that b2 + c2 ≤ (b + c)2 = (1 − a)2 ≤ 1, we apply the Cauchy SchwarzInequality and get

(1− b2)2 + (1− c2)2 ≥ (2− b2 − c2)2

2≥[2− (1− a)2

]22

=(1 + 2a− a2)2

2.

Therefore, it suffices to prove that

(1− a2)2 +(1 + 2a− a2)2

2≥ 2.

After some simple computations, we see that

(1− a2)2 +(1 + 2a− a2)2

2− 2 =

(1− a)2(1 + a)(3a− 1)

2,

which is clearly nonnegative since a ≥ 1

3. This ends the proof for the left

inequality. Note that the equality holds if and only if (a, b, c) equals (1, 0, 0),or (0, 1, 0), or (0, 0, 1).

For the right inequality, to prove it, let us notice that for every nonnegativenumbers x,

9(1− x)(1 + x)2 + 15x2 − 10x− 9 = −x(1− 3x)2 ≤ 0.

297

Page 299: Олимпиадад бэлтгэгчдэд тусламж (Тэнцэтгэл биш 1)

This implies that

(1− x)(1 + x)2 ≤ 9 + 10x− 15x2

9.

Using this inequality, we deduce that∑(1− a2)2 =

∑(1− a)(1− a)(1 + a)2 ≤ 1

9

∑(1− a)(9 + 10a− 15a2),

and hence, it suffices to prove that∑(1− a)(9 + 10a− 15a2) ≤ 9(1 + a)(1 + b)(1 + c).

Now, expanding with noting that a+ b+ c = 1, we can rewrite this inequalityas

15∑

a3 − 25∑

a2 + 10 ≤ 9∑

ab+ 9abc,

or

15∑

a3 − 25(∑

a2)(∑

a)

+ 10(∑

a)3≤ 9

(∑a)(∑

ab)

+ 9abc.

The last one can be simplified to

4∑

ab(a+ b) ≥ 24abc,

which is obviously true by the AM-GM Inequality. Therefore, the right handside inequality is proved. It is easy to see that the equality holds if and only

if (a, b, c) is one of the triples (1, 0, 0), (0, 1, 0), (0, 0, 1),

(1

3,1

3,1

3

).

?F?

00.3. Let a, b, c, x, y, z be positive real numbers. Prove that

a3

x+b3

y+c3

z≥ (a+ b+ c)3

3 (x+ y + z).

(Belarus 2000)

First solution: From the Cauchy Schwarz Inequality, we have

a3

x+b3

y+c3

z≥

(a√a+ b

√b+ c

√c)2

x+ y + z.

Using this inequality, it suffices to prove that

3(a√a+ b

√b+ c

√c)2≥ (a+ b+ c)3 .

Because of the homogeneity of this inequality, we may homogenize a+b+c = 3,and hence, it becomes

a√a+ b

√b+ c

√c ≥ 3.

298

Page 300: Олимпиадад бэлтгэгчдэд тусламж (Тэнцэтгэл биш 1)

Now, applying the Bernoulli’s Inequality, we have

a√a = [1 + (a− 1)]

32 ≥ 1 +

3

2(a− 1).

Adding this to the two analogous inequalities, we conclude that

a√a+ b

√b+ c

√c ≥ 3 +

3

2(a+ b+ c− 3) = 3,

as desired.

Second solution: By the Holder’s Inequality,(a3

x+b3

y+c3

z

)1/3

(1 + 1 + 1)1/3(x+ y + z)1/3 ≥ a+ b+ c.

Cubing both sides and then dividing both sides by 3(x+y+z) gives the desiredresult.

?F?

00.4. Suppose that the real numbers a1, a2, . . . , a100 satisfy(i) a1 ≥ a2 ≥ · · · ≥ a100 ≥ 0;(ii) a1 + a2 ≤ 100;(iii) a3 + a4 + · · ·+ a100 ≤ 100.Determine the maximum possible value of a21 + a22 + · · · + a2100, and find allpossible sequences a1, a2, . . . , a100 which achieve this maximum.

(Canada 2000)

Solution: We have

a21 + a22 − a23 − (a1 + a2 − a3)2 = −2(a1 − a3)(a2 − a3) ≤ 0,

and hence

a21 + a22 ≤ a23 + (a1 + a2 − a3)2 ≤ a23 + (100− a3)2.

Note that the equality in this inequality holds iff a2 = a3 and a1 + a2 = 100.Now, since a3 ≥ a4 ≥ · · · ≥ a100, we have

a24 + · · ·+ a2100 ≤ a3a4 + · · ·+ a3a100 = a3(a4 + · · ·+ a100) ≤ a3(100− a3),

with equality iff a24 = a3a4, . . . , a2100 = a3a100 and a3(a3+a4+· · ·+a100−100) =

0. In addition, since a1 ≥ a2 ≥ a3 and a1 + a2 ≤ 100, we deduce that a3 ≤ 50,and hence a3(a3 − 50) ≤ 0 with equality iff a3(a3 − 50) = 0. From theseinequalities, we infer that

a21 + a22 + · · ·+ a2100 ≤ a23 + (100− a3)2 + a23 + a3(100− a3)= 10000 + 2a3(a3 − 50) ≤ 10000.

The equality holds if and only if the equality must hold in each inequalityfound above - that is, we must have:(a) a1 + a2 = 100, a2 = a3,

299

Page 301: Олимпиадад бэлтгэгчдэд тусламж (Тэнцэтгэл биш 1)

(b) a24 = a3a4, . . . , a2100 = a3a100, a3(a3 + a4 + · · ·+ a100 − 100) = 0,

(c) a3(a3 − 50) = 0.Solving the system of equations (a), (b) and (c) (with noting at the givenhypothesis), we can find that the equality holds only when the sequencea1, a2, . . . , a100 equals

100, 0, 0, . . . , 0, or 50, 50, 50, 50, 0, 0, . . . , 0.

?F?

00.5. Show that

3

√2 (a+ b)

(1

a+

1

b

)≥ 3

√a

b+

3

√b

a

for all positive real numbers a and b, and determine when the equality occurs.(Czech-Slovak 2000)

First solution: Multiplying both sides of the desired inequality by 3√ab gives

the equivalent inequality

3√a2 +

3√b2 ≤ 3

√2 (a+ b)2.

Setting 3√a = x and 3

√b = y, we see that it suffices to prove that

x2 + y2 ≤ 3

√2 (x3 + y3)2, or equivalently,

(x2 + y2

)3 ≤ 2(x3 + y3

)2.

Now, by applying the Cauchy Schwarz Inequality and the Chebyshev’s In-equality, we get(x2 + y2

)3=(x2 + y2

) (√x ·√x3 +

√y ·√y3)2≤(x2 + y2

)(x+ y)

(x3 + y3

)≤ 2

(x3 + y3

) (x3 + y3

)= 2

(x3 + y3

)2,

as desired. It is easy to see that the equality holds if and only if a = b.

Second solution: By the Power Mean Inequality, we have3

√a

b+ 3

√b

a

2

3

√a

b+

√b

a

2

2

,

with equality if and only ifa

b=b

a, or equivalently a = b. It is easy to see that

the desired result follows from this inequality and the identity(√a

b+

√b

a

)2

= (a+ b)

(1

a+

1

b

).

?F?

300

Page 302: Олимпиадад бэлтгэгчдэд тусламж (Тэнцэтгэл биш 1)

00.6. Let a, b, c be positive real numbers such that abc = 1. Prove that

1 + ab2

c3+

1 + bc2

a3+

1 + ca2

b3≥ 18

a3 + b3 + c3.

(Hong Kong 2000)

Solution: Multiplying both sides of the desired inequality by a3 + b3 + c3 > 0gives the equivalent inequality

(a3 + b3 + c3

)( 1

a3+

1

b3+

1

c3

)+(a3 + b3 + c3

)(ab2c3

+bc2

a3+ca2

b3

)≥ 18.

Now, applying the AM-GM Inequality, we have

(a3 + b3 + c3

)( 1

a3+

1

b3+

1

c3

)≥ 3

3√a3b3c3 · 3 3

√1

a3b3c3= 9,

and

(a3 + b3 + c3

)(ab2c3

+bc2

a3+ca2

b3

)≥ 3

3√a3b3c3 · 3 3

√a3b3c3

a3b3c3= 9.

Adding up these two inequalities, we get the result. It is easy to see that theequality holds if and only if a = b = c = 1.

?F?

00.7. Let x, y, and z denote positive real numbers, each less than 4. Prove

that at least one of the numbers1

x+

1

4− y,

1

y+

1

4− z, and

1

z+

1

4− xis greater

than or equal to 1.(Hungary 2000)

Solution: From the Cauchy Schwarz Inequality, we have

1

x+

1

4− x≥ 4

x+ (4− x)= 1.

Now, if the three numbers1

x+

1

4− y,

1

y+

1

4− z, and

1

z+

1

4− xwere all less

than 1, their sum S would be less than 3. However,

S =

(1

x+

1

4− x

)+

(1

y+

1

4− y

)+

(1

z+

1

4− z

)≥ 3.

This contradiction proves the requested result.?F?

00.8. Let a, b, c be positive real numbers such that abc = 1. Prove the in-equality (

a+1

b− 1

)(b+

1

c− 1

)(c+

1

a− 1

)≤ 1.

(IMO 2000)

301

Page 303: Олимпиадад бэлтгэгчдэд тусламж (Тэнцэтгэл биш 1)

First solution: Without loss of generality, we can suppose that b = min{a, b, c},then it is clear that b ≤ 1. Now, replacing c =

1

ab, the inequality becomes

(a+

1

b− 1

)(b+ ab− 1)

(1

ab+

1

a− 1

)≤ 1,

or

(ab+ 1− b)(ab+ b− 1)(b+ 1− ab) ≤ ab2.

Because (ab+b−1)+(b+1−ab) = 2b > 0, we can see that max{ab+b−1, b+1 − ab} > 0. On the other hand, it is obvious that ab + 1 − b > 0, thereforeif min{ab + b − 1, b + 1 − ab} ≤ 0, the left hand side of the above inequalityis not greater than 0 while the right hand side is, so the inequality is trivial.Otherwise, one can see that

0 < (ab+ 1− b)(ab+ b− 1) = a2b2 − (b− 1)2 ≤ a2b2,

0 < (ab+ b− 1)(b+ 1− ab) = b2 − (ab− 1)2 ≤ b2,

0 < (b+ 1− ab)(ab+ 1− b) = 1− b2(a− 1)2 ≤ 1.

Multiplying these three inequalities and taking the square root, we can getthe desired result. The equality holds if and only if a = b = c = 1.

Second solution: Using the condition abc = 1, it is straightforward to verifythe equalities

2 =1

a

(a− 1 +

1

b

)+ c

(b− 1 +

1

c

),

2 =1

b

(b− 1 +

1

c

)+ a

(c− 1 +

1

a

),

2 =1

c

(c− 1 +

1

a

)+ b

(a− 1 +

1

c

).

In particular, they show that at most one of the numbers u = a − 1 +1

b,

v = b− 1 +1

c, w = c− 1 +

1

ais negative. If there is such a number, we have

(a− 1 +

1

b

)(b− 1 +

1

c

)(c− 1 +

1

a

)= uvw < 0 < 1.

And if u, v, w ≥ 0, the AM-GM Inequality yields

2 =1

au+ cv ≥ 2

√c

auv, 2 =

1

bv+aw ≥ 2

√a

bvw, 2 =

1

cw+aw ≥ 2

√b

cwu.

Thus, uv ≤ a

c, vw ≤ b

a, wu ≤ c

b, so (uvw)2 ≤ a

c· ba· cb

= 1. Since u, v, w ≥ 0,

this completes the proof.

302

Page 304: Олимпиадад бэлтгэгчдэд тусламж (Тэнцэтгэл биш 1)

Third solution: From the given hypothesis, we may substitute a =x

y, b =

y

z,

c =z

x, where x, y, z > 0. By this substitution, our inequality becomes(

x

y− 1 +

z

y

)(yz− 1 +

x

z

)( zx− 1 +

y

x

)≤ 1,

orxyz ≥ (y + z − x) (z + x− y) (x+ y − z) .

By expanding, we find that the last inequality is equivalent to

x3 + y3 + z3 + 3xyz ≥ xy(x+ y) + yz(y + z) + zx(z + x),

which is just the Schur’s Inequality (in the special case third degree).?F?

00.9. Let x, y ≥ 0 with x+ y = 2. Prove that

x2y2(x2 + y2) ≤ 2.

(Ireland 2000)

First solution: After homogenizing it, we need to prove

2

(x+ y

2

)6

≥ x2y2(x2 + y2), or (x+ y)6 ≥ 32x2y2(x2 + y2).

(Now, forget the constraint x + y = 2!) In case xy = 0, it clearly holds. Wenow assume that xy 6= 0. Because of the homogeneity of the inequality, thismeans that we may normalize to xy = 1. Then, it becomes(

x+1

x

)6

≥ 32

(x2 +

1

x2

), or p3 ≥ 32(p− 2),

where p =

(x+

1

x

)2

≥ 4. Our job is now to minimize F (p) = p3 − 32(p− 2)

on [4,∞). Since F ′(p) = 3p2 − 32 ≥ 0, where p ≥√

32

3, F is (monotone)

increasing on [4,∞). So, F (p) ≥ F (4) = 0 for all p ≥ 4.

Second solution: As in the first solution, we prove that (x+ y)6 ≥ 32(x2 +y2)(xy)2 for all x, y ≥ 0. In case x = y = 0, it’s clear. Now, if x2+y2 > 0, then

we may normalize to x2+y2 = 2. Setting p = xy, we have 0 ≤ p ≤ x2 + y2

2= 1

and (x+ y)2 = x2 + y2 + 2xy = 2 + 2p. It now becomes

(2 + 2p)3 ≥ 64p2, or p3 − 5p2 + 3p+ 1 ≥ 0.

We want to minimize F (p) = p3− 5p2 + 3p+ 1 on [0, 1]. We compute F ′(p) =

3

(p− 1

3

)(p − 3). We find that F is monotone increasing on

[0,

1

3

]and

303

Page 305: Олимпиадад бэлтгэгчдэд тусламж (Тэнцэтгэл биш 1)

monotone decreasing on

[1

3, 1

]. Since F (0) = 1 and F (1) = 0, we conclude

that F (p) ≥ F (1) = 0 for all p ∈ [0, 1].

Third solution: We show that (x+ y)6 ≥ 32(x2 + y2)(xy)2 where x ≥ y ≥ 0.We make the substitution u = x+ y and v = x− y. Then, we have u ≥ v ≥ 0.It becomes

u6 ≥ 32

(u2 + v2

2

)(u2 − v2

4

)2

, or u6 ≥ (u2 + v2)(u2 − v2)2.

Note that u4 ≥ u4 − v4 ≥ 0 and that u2 ≥ u2 − v2 ≥ 0. So,

u6 ≥ (u4 − v4)(u2 − v2) = (u2 + v2)(u2 − v2)2.

Fourth solution: According to the AM-GM Inequality, we find that

xy ≤(x+ y

2

)2

= 1,

and

xy(x2 + y2) =1

2· 2xy · (x2 + y2) ≤ 1

2

(2xy + x2 + y2

2

)2

= 2.

Therefore, by combining these two results, we conclude that

x2y2(x2 + y2) = xy · xy(x2 + y2) ≤ 1 · 2 = 2,

as desired.?F?

00.10. The real numbers a, b, c, x, y, z satisfy a ≥ b ≥ c > 0 and x ≥ y ≥ z >0. Prove that

a2x2

(by + cz)(bz + cy)+

b2y2

(cz + ax)(cx+ az)+

c2z2

(ax+ by)(ay + bx)≥ 3

4.

(Korea 2000)

First solution: Denote the left hand side of the given inequality by S. Be-cause a ≥ b ≥ c and x ≥ y ≥ z, by the Rearrangement Inequality, we havebz+ cy ≤ by+ cz so (by+ cz)(bz+ cy) ≤ (by+ cz)2 ≤ 2[(by)2 + (cz)2]. Settingm = (ax)2, n = (by)2, p = (cz)2, we obtain

a2x2

(by + cz)(bz + cy)≥ a2x2

2[(by)2 + (cz)2]=

m

2(n+ p).

Adding this to the two analogous inequalities, we find that

S ≥ 1

2

(m

n+ p+

n

p+m+

p

m+ n

).

304

Page 306: Олимпиадад бэлтгэгчдэд тусламж (Тэнцэтгэл биш 1)

On the other hand, according to the Nesbitt’s Inequality, it is easy to see that

m

n+ p+

n

p+m+

p

m+ n≥ 3

2,

and therefore S ≥ 3

4, as desired.

Second solution: According to the Cauchy Schwarz Inequality, we haveby+ cz ≤

√(b2 + c2)(y2 + z2) and bz+ cy ≤

√(b2 + c2)(z2 + y2). Multiplying

these inequalities, we get (by + cz)(bz + cy) ≤ (b2 + c2)(y2 + z2), and hence

a2x2

(by + cz)(bz + cy)≥ a2

b2 + c2· x2

y2 + z2.

Adding this to the two analogous inequalities, we find that

a2x2

(by + cz)(bz + cy)+

b2y2

(cz + ax)(cx+ az)+

c2z2

(ax+ by)(ay + bx)≥

≥ a2

b2 + c2· x2

y2 + z2+

b2

c2 + a2· y2

z2 + x2+

c2

a2 + b2· z2

x2 + y2.

On the other hand, from the given hypothesis, it is easy to verify that

a2

b2 + c2≥ b2

c2 + a2≥ c2

a2 + b2,

x2

y2 + z2≥ y2

z2 + x2≥ z2

x2 + y2.

Therefore, by the Chebyshev’s Inequality, we have

a2

b2 + c2· x2

y2 + z2+

b2

c2 + a2· y2

z2 + x2+

c2

a2 + b2· z2

x2 + y2≥

≥ 1

3

(a2

b2 + c2+

b2

c2 + a2+

c2

a2 + b2

)(x2

y2 + z2+

y2

z2 + x2+

z2

x2 + y2

)≥ 3

4,

where the last inequality holds according to the Nesbitt’s Inequalitym

n+ p+

n

p+m+

p

m+ n≥ 3

2. From this and the above estimation, we can get the

result.?F?

00.11. Let x, y, z be real numbers. Prove that

x2 + y2 + z2 ≥√

2 (xy + yz) .

(Macedonia 2000)

Solution: By the AM-GM Inequality, we find that

x2 + y2 + z2 =

(x2 +

y2

2

)+

(y2

2+ z2

)≥ 2

√x2 · y

2

2+ 2

√y2

2· z2

=√

2 (|xy|+ |yz|) ≥√

2(xy + yz),

305

Page 307: Олимпиадад бэлтгэгчдэд тусламж (Тэнцэтгэл биш 1)

as desired. It is easy to see that the equality holds if and only if x = z =y

2.

?F?

00.12. Let a, b, x, y, z be positive real numbers. Prove that

x

ay + bz+

y

az + bx+

z

ax+ by≥ 3

a+ b.

(MOSP 2000)

Solution: Using the Cauchy Schwarz Inequality in combination with the well-known inequality (x+ y + z)2 ≥ 3(xy + yz + zx), we deduce that

x

ay + bz+

y

az + bx+

z

ax+ by≥ (x+ y + z)2

x(ay + bz) + y(az + bx) + z(ax+ by)

=1

a+ b· (x+ y + z)2

xy + yz + zx≥ 3

a+ b,

as desired. It is easy to see that the equality holds if and only if x = y = z = 1.?F?

00.13. Let ABC be an acute triangle. Prove that(cosA

cosB

)2

+

(cosB

cosC

)2

+

(cosC

cosA

)2

+ 8 cosA cosB cosC ≥ 4.

(MOSP 2000)

Solution: Since 4−8 cosA cosB cosC = 4(cos2A+cos2B+cos2C), it sufficesto prove that(

cosA

cosB

)2

+

(cosB

cosC

)2

+

(cosC

cosA

)2

≥ 4(cos2A+ cos2B + cos2C).

Now, applying the AM-GM Inequality in combination with the well-known

inequality cosA cosB cosC ≤ 1

8, we have

(cosA

cosB

)2

+

(cosA

cosB

)2

+

(cosB

cosC

)2

≥ 33

√cos4A

cos2B cos2C

=3 cos2A

3√

cos2A cos2 b cos2C

≥ 3 cos2A

3

√1

82

= 12 cos2A.

Adding this inequality with its analogous forms and dividing both sides ofthe resulting inequality by 3, we obtain desired result. It is easy to see that

equality holds if and only if A = B = C =π

3.

?F?

306

Page 308: Олимпиадад бэлтгэгчдэд тусламж (Тэнцэтгэл биш 1)

00.14. Let a, b, c be positive real numbers such that min {a, b} ≥ c. Provethat √

c(a− c) +√c(b− c) ≤

√ab.

(MOSP 2000)

Solution: By the Cauchy Schwarz Inequality, we have[√c(a− c) +

√c(b− c)

]2≤ [c+ (b− c)] [(a− c) + c] = ab.

Taking square root of each side of this inequality yields the desired result.?F?

00.15. Let a1, a2, . . . , an be nonnegative real numbers. Prove that

a1 + a2 + · · ·+ ann

− n√a1a2 · · · an ≥

≥ 1

2min

{(√a1 −

√a2)

2 , (√a2 −

√a3)

2 , . . . , (√an −

√a1)

2}.

(MOSP 2000)

Solution: Denote an+1 = a1, because

nmin{

(√a1 −

√a2)

2 , (√a2 −

√a3)

2 , . . . , (√an −

√a1)

2}≤

n∑i=1

(√ai −

√ai+1)

2 ,

one can see that the original inequality is deduced from the following inequality

a1 + a2 + · · ·+ an − n n√a1a2 · · · an ≥

1

2

n∑i=1

(√ai −

√ai+1)

2 .

This is equivalent to

a1+a2+ · · ·+an−n n√a1a2 · · · an ≥ a1+a2+ . . .+an−(

√a1a2 + · · ·+

√ana1) ,

or √a1a2 + · · ·+

√ana1 ≥ n n

√a1a2 · · · an.

The last one is clearly true by the AM-GM Inequality, so our proof is com-pleted. Note that the equality holds if and only if a1 = a2 = · · · = an.

?F?

00.16. Let (an) be an infinite sequence of positive numbers such that

a11

+a42

+ · · ·+ ak2

k≤ 1

for all k. Prove thata11

+a22

+ · · ·+ ann< 2,

for all n.(MOSP 2000)

307

Page 309: Олимпиадад бэлтгэгчдэд тусламж (Тэнцэтгэл биш 1)

Solution: One can see that we can put n = k2+p where k ≥ 1 and 0 ≤ p ≤ 2k.Then

a11

+a22

+ · · ·+ ann

=a11

+a22

+ . . .+ak2+pk2 + p

≤ a11

+a22

+ · · ·+ak2+pk2 + p

+ · · ·+ ak2+2k

k2 + 2k

=k∑i=1

(ai2

i2+ · · ·+ ai2+2i

i2 + 2i

).

Since (an) is an decreasing sequence, it is clear that

ai2

i2+ · · ·+ ai2+2i

i2 + 2i≤(

1

i2+

1

i2 + 1+ · · ·+ 1

i2 + 2i

)ai2 .

On the other hand, we have

1

i2+ · · ·+ 1

i2 + 2i=

(1

i2+ · · ·+ 1

i2 + i− 1

)+

(1

i2 + i+ . . .+

1

i2 + 2i

)<

i

i2+i+ 1

i2 + i=

2

i.

Therefore, for any i = 1, 2, . . . , k, we have

ai2

i2+ · · ·+ ai2+2i

i2 + 2i≤ 2ai2

i,

and from this, it follows that

a11

+a22

+ · · ·+ ann< 2

k∑i=1

ai2

i≤ 2,

as desired.?F?

00.17. Let a, b, c be nonnegative real numbers such that ab + bc + ca = 1.Prove that

1

b+ c+

1

c+ a+

1

a+ b≥ 5

2.

(MOSP 2000)

First solution: If a+ b+ c ≥ 2, then we have

1

b+ c+

1

c+ a+

1

a+ b=ab+ bc+ ca

b+ c+ab+ bc+ ca

c+ a+ab+ bc+ ca

a+ b

= a+ b+ c+bc

b+ c+

ca

c+ a+

ab

a+ b

≥ a+ b+ c+bc

b+ c+ a+

ca

c+ a+ b+

ab

a+ b+ c

= a+ b+ c+1

a+ b+ c≥ 2 +

1

2=

5

2.

308

Page 310: Олимпиадад бэлтгэгчдэд тусламж (Тэнцэтгэл биш 1)

So, let us consider now the case a + b + c ≤ 2. Without loss of generality,

we can assume that a = max {a, b, c} , then a ≥ 1√3>

1

2. Accordingly, this

assumption allows us to proceed the following estimation

1

b+ c+

1

c+ a+

1

a+ b=ab+ bc+ ca

a+ b+ab+ bc+ ca

a+ c+

1

b+ c

=

(b+ c+

1

b+ c

)+

(ab

a+ b+

ac

a+ c

)≥ 2 +

ab

a+ b+

ac

a+ c= 2 +

a(1 + bc)

a(a+ b+ c) + bc

≥ 2 +a(1 + bc)

2a+ bc≥ 2 +

a(1 + bc)

2a+ 2abc

= 2 +1

2=

5

2.

Therefore, in any cases, we always have

1

b+ c+

1

c+ a+

1

a+ b≥ 5

2.

The equality holds if and only if (a, b, c) is a permutation of (1, 1, 0).?F?

00.18. Let a1, a2, . . . , an be positive real numbers such that

1

a1+

1

a2+ · · ·+ 1

an≤ 1.

Prove that for any positive integer k,

(ak1 − 1)(ak2 − 1) · · · (akn − 1) ≥ (nk − 1)n.

(MOSP 2000)

Solution: In order to prove the given inequality, we put bi =1

ai, and then

b1 + b2 + · · ·+ bn ≤ 1. By this substitution, we need to prove that(1

bk1− 1

)(1

bk2− 1

)· · ·(

1

bkn− 1

)≥ (nk − 1)n.

Firstly, we will show that(1

b1− 1

)(1

b2− 1

). . .

(1

bn− 1

)≥ (n− 1)n.

Indeed, by the AM-GM Inequality, we have

1

bi− 1 ≥ b1 + b2 + · · ·+ bn

bi− 1 =

∑j 6=i

bj

bi≥

(n− 1) n−1

√∏j 6=ibj

bi.

309

Page 311: Олимпиадад бэлтгэгчдэд тусламж (Тэнцэтгэл биш 1)

Setting i = 1, 2, . . . , n, we get the n similar inequalities and multiplying themup, we can get the above inequality. Now, since k is an positive integer, theHolder’s Inequality implies

n∏i=1

(1

bki− 1

)=

n∏i=1

(1

bi− 1

)·n∏i=1

(1 +

1

bi+ · · ·+ 1

bk−1i

)

≥ (n− 1)nn∏i=1

(1 +

1

bi+ · · ·+ 1

bk−1i

)

≥ (n− 1)n

[1 +

1n√b1b2 . . . bn

+ · · ·+ 1(n√b1b2 . . . bn

)k−1]n.

On the other hand, from the condition b1+b2+· · ·+bn ≤ 1, we can easily deduce

that n√b1b2 · · · bn ≤

1

n. Using this in combination with the above inequality,

we get

n∏i=1

(1

bki− 1

)≥ (n− 1)n(1 + n+ · · ·+ nk−1)n = (nk − 1)n,

as desired. Note that the inequality becomes equality if and only if a1 = a2 =· · · = an = n.

?F?

00.19. Let a, b, c be positive real numbers. Prove that

1

a+ b+

1

b+ c+

1

c+ a+

1

2 3√abc≥

(a+ b+ c+ 3

√abc)2

(a+ b)(b+ c)(c+ a).

(MOSP 2000)

First solution: Applying the Cauchy Schwarz Inequality, we have

1

a+ b+

1

b+ c+

1

c+ a+

1

2 3√abc

=a2

a2(b+ c)+

b2

b2(c+ a)+

c2

c2(a+ b)+

(3√abc)2

2abc

(a+ b+ c+ 3

√abc)2

a2(b+ c) + b2(c+ a) + c2(a+ b) + 2abc.

On the other hand, it is easy to verify that a2(b+c)+b2(c+a)+c2(a+b)+2abc =(a+ b)(b+ c)(c+ a). Therefore, the above inequality implies

1

a+ b+

1

b+ c+

1

c+ a+

1

2 3√abc≥

(a+ b+ c+ 3

√abc)2

(a+ b)(b+ c)(c+ a).

The equality holds if and only if a = b = c = 1.

Second solution: The original inequality is equivalent to∑(a+ b)(a+ c) +

(a+ b)(b+ c)(c+ a)

2 3√abc

≥(a+ b+ c+

3√abc)2,

310

Page 312: Олимпиадад бэлтгэгчдэд тусламж (Тэнцэтгэл биш 1)

or

ab+ bc+ ca+(a+ b)(b+ c)(c+ a)

2 3√abc

≥ 23√abc(a+ b+ c) +

3√a2b2c2.

Since ab+ bc+ ca ≥ 33√a2b2c2, the above inequality follows from

(a+ b)(b+ c)(c+ a)

2 3√abc

+ 23√a2b2c2 ≥ 2

3√abc(a+ b+ c).

Multiplying both sides for 2 3√abc > 0, we can rewrite this inequality as

(a+ b)(b+ c)(c+ a) + 4abc ≥ 43√a2b2c2(a+ b+ c).

Without loss of generality, we can assume that a ≥ b ≥ c. Now, we write theinequality in form

(b+ c)[(a+ b)(a+ c)− 4

3√a2b2c2

]≥ 4

3√a2b2c2

(a− 3√abc).

Because (a+ b)(a+ c) = a2 + ab+ ac+ bc, it is equivalent to

(b+ c)(a2 + ab+ bc+ ca− 4

3√a2b2c2

)≥ 4

3√a2b2c2

(a− 3√abc).

Using again the estimation ab+ bc+ ca ≥ 33√a2b2c2, it suffices to prove that

(b+ c)(a2 − 3

√a2b2c2

)≥ 4

3√a2b2c2

(a− 3√abc),

or equivalently,

(b+ c)(a+

3√abc)≥ 4

3√a2b2c2.

Of course, this is obvious since by the AM-GM Inequality, we have

(b+ c)(a+

3√abc)≥ 2√bc · 2

√a

3√abc = 4

3√a2b2c2.

?F?

00.20. For any integer n ≥ 2, consider n−1 positive real numbers a1, a2, . . . , an−1having sum 1, and n real numbers b1, b2, . . . , bn. Prove that

b21 +b22a1

+b23a2

+ . . .+b2nan−1

≥ 2b1(b2 + b3 + . . .+ bn).

(Romania 2000)

Solution: By the Cauchy Schwarz Inequality, we have

b22a1

+b23a2

+ · · ·+ b2nan−1

≥ (b2 + b3 + · · ·+ bn)2

a1 + a2 + · · ·+ an−1= (b2 + b3 + · · ·+ bn)2.

From this inequality, we deduce that

b21 +b22a1

+b23a2

+ · · ·+ b2nan−1

≥ b21 + (b2 + b3 + · · ·+ bn)2

≥ 2 |b1(b2 + b3 + · · ·+ bn)|≥ 2b1(b2 + b3 + · · ·+ bn),

311

Page 313: Олимпиадад бэлтгэгчдэд тусламж (Тэнцэтгэл биш 1)

as desired.?F?

00.21. Positive real numbers x, y, z satisfy xyz = 1. Prove that the followinginequality holds

x2 + y2 + z2 + x+ y + z ≥ 2 (xy + yz + zx) .

(Russia 2000)

Solution: From the AM-GM Inequality and the given hypothesis, we havex+ y + z ≥ 3 3

√xyz = 3. Therefore, it suffices to prove that

x2 + y2 + z2 + 3 ≥ 2 (xy + yz + zx) .

Multiplying both sides of this inequality by x+ y+ z > 0, we can rewrite it as

(x2 + y2 + z2)(x+ y + z) + 3(x+ y + z) ≥ 2(x+ y + z)(xy + yz + zx).

Because(∑

x2)(∑

x)

=∑

x3 +∑

xy(x + y) and(∑

x)(∑

xy)

=∑xy(x+ y) + 3xyz =

∑xy(x+ y) + 3, the latter inequality is equivalent to

x3 + y3 + z3 + 3 (x+ y + z) ≥ xy (x+ y) + yz (y + z) + zx (z + x) + 6.

Now, we use the AM-GM Inequality and the Schur’s Inequality (applied forthird degree), and get

x3 + y3 + z3 + 3 (x+ y + z) =[x3 + y3 + z3 + (x+ y + z)

]+ 2 (x+ y + z)

≥(x3 + y3 + z3 + 3 3

√xyz

)+ 2 · 3 3

√xyz

=(x3 + y3 + z3 + 3xyz

)+ 2 · 3 3

√xyz

≥ xy (x+ y) + yz (y + z) + zx (z + x) + 6.

Therefore, the last inequality is valid and so, our proof is completed. Notethat the equality holds if and only if x = y = z = 1.

?F?

00.22. Let x1, x2, . . . , xn be real numbers (n ≥ 2), satisfying the conditions−1 < x1 < x2 < · · · < xn < 1 and

x131 + x132 + · · ·+ x13n = x1 + x2 + · · ·+ xn.

Prove that

x131 y1 + x132 y2 + · · ·+ x13n yn < x1y1 + x2y2 + · · ·+ xnyn

for any real numbers y1 < y2 < · · · < yn.

(Russia 2000)

312

Page 314: Олимпиадад бэлтгэгчдэд тусламж (Тэнцэтгэл биш 1)

Solution: Using the Abel’s summation formula, we have

n∑i=1

xiyi −n∑i=1

x13i yi =n∑i=1

yi(xi − x13i )

= y1

n∑i=1

(xi − x13i ) +n∑i=2

(yi − yi−1)n∑j=i

(xj − x13j )

=n∑i=2

(yi − yi−1)n∑j=i

(xj − x13j ).

Therefore, in order to prove the desired inequality, it suffices to prove that eachterm of the above sum is positive, and since yi− yi−1 > 0 for any i = 2, . . . , n,it suffices to prove that

n∑j=i

(xj − x13j ) > 0

for any i = 2, . . . , n. Indeed, if xi > 0 then we have 1 > xn > · · · > xi >0, from which it follows that xj > x13j for any j = i, . . . , n, and thereforen∑j=i

(xj−x13j ) > 0.Alternatively, if xi ≤ 0 then we have−1 < x1 < . . . < xi ≤ 0,

from which it follows that xj < x13j for any j = 1, . . . , i− 1, so it is clear that

i−1∑j=1

(xj − x13j ) < 0,

and since

n∑j=i

(xj − x13j ) = −i−1∑j=1

(xj − x13j ), we conclude that

n∑j=i

(xj − x13j ) > 0.

This completes our proof.?F?

00.23. Show that for all n ∈ N and x ∈ R,

sinn 2x+ (sinn x− cosn x)2 ≤ 1.

(Russia 2000)

Solution: For n = 0 and n = 1, the inequality becomes equality. In casen ≥ 2, denote a = sinx, b = cosx, then we have a2 + b2 = 1. The left handside of the desired inequality equals

(2ab)n + (an − bn)2 = a2n + b2n + (2n − 2)anbn,

while the right hand side equals

1 = (a2 + b2)n = a2n + b2n +

n−1∑j=1

(n

j

)a2(n−j)b2j .

313

Page 315: Олимпиадад бэлтгэгчдэд тусламж (Тэнцэтгэл биш 1)

It thus suffices to prove that

n−1∑j=1

(n

j

)a2(n−j)b2j ≥ (2n − 2)anbn. We can do so

by viewing

n−1∑j=1

(n

j

)a2(n−j)b2j as a sum of

n−1∑j=1

(n

j

)= 2n−2 terms of the form

a2(n−j)b2j , and then applying the AM-GM Inequality to these terms.?F?

00.24. Let n ≥ 3 be a positive integer. Prove that for all positive real numbersa1, a2, . . . , an, we have

a1 + a22

· a2 + a32

· · · an + a12

≤ a1 + a2 + a3

2√

2· · · an + a1 + a2

2√

2.

(Saint Petersburg 2000)

Solution: We take the indices of the ai modulo n. Observe that

4(ai−1 + ai + ai+1)2 = [(2ai−1 + ai) + (ai + 2ai+1)]

2

≥ 4(2ai−1 + ai)(ai + 2ai+1)

by the AM-GM Inequality. Equivalently,

(ai−1 + ai + ai+1)2 ≥ (2ai−1 + ai)(ai + 2ai+1).

In addition to this inequality, note that

(2ai−1 + ai)(ai + 2ai−1) ≥ 2(ai−1 + ai)2

since ai is positive number for each i. Multiplying these two inequalitiestogether for i = 1, 2, . . . , n, and then taking the square root of each side of theresulting inequality, gives the desired result.

?F?

00.25. Let n ≥ 3 be an integer. Prove that for positive numbers x1 ≤ x2 ≤· · · ≤ xn,

xnx1x2

+x1x2x3

+ · · ·+ xn−1xnx1

≥ x1 + x2 + · · ·+ xn.

(Saint Petersburg 2000)

First solution: Suppose that 0 ≤ x ≤ y and 0 < a ≤ 1. We have 1 ≥ a andy ≥ x ≥ ax, implying that (1− a)(y− ax) ≥ 0 or ax+ ay ≤ a2x+ y. Dividingboth sides of this final inequality by a, we find that

x+ y ≤ ax+y

a. (1)

We may set (x, y, a) =

(xn, xn ·

xn−1x2

,x1x2

)in (1) to find that

xn +xn−1xnx2

≤ xnx1x2

+xn−1xnx1

. (2)

314

Page 316: Олимпиадад бэлтгэгчдэд тусламж (Тэнцэтгэл биш 1)

Furthermore, for i = 1, 2, . . . , n−2, we may set (x, y, a) =

(x1, xn−1 ·

xi+1

x2,xi+1

xi+2

)in (1) to find that

xi + xn−1 ·xi+1

x2≤ xi ·

xi+1

xi+2+ xn−1 ·

xi+1

x2· xi+2

xi+1= xi ·

xi+1

xi+2+ xn−1 ·

xi+2

xi.

Summing this inequality for i = 1, 2, . . . , n− 2 yields

(x1 + · · ·+ xn−2) + xn−1 ≤(x1x2x3

+ · · ·+ xn−2xn−1xn

)+xn−1xnx2

.

Summing this last inequality with (2) yields the desired inequality.

Second solution: We will prove the inequality by induction on n. For n = 3,it becomes

x1x2x3

+x3x1x2

+x2x3x1≥ x1 + x2 + x3,

which is true since by the AM-GM Inequality, we have

2

(x1x2x3

+x3x1x2

+x2x3x1

)=∑(

x1x2x3

+x3x1x2

)≥ 2(x1 + x2 + x3).

Now, suppose that the equality holds for n ≥ 3 and let us prove it for n + 1,that is to prove

x1x2x3

+· · ·+xn−2xn−1xn

+xn−1xnxn+1

+xnxn+1

x1+xn+1x1x2

≥ x1+x2+· · ·+xn+xn+1.

Since x1 ≤ x2 ≤ · · · ≤ xn, we can apply the inductive hypothesis to get

x1x2x3

+ · · ·+ xn−2xn−1xn

+xn−1xnx1

+xnx1x2≥ x1 + x2 + · · ·+ xn.

Therefore, in order to prove that the inequality holds for n + 1, it suffices toprove that

xn−1xnxn+1

+xnxn+1

x1+xn+1x1x2

− xn+1 ≥xn−1xnx1

+xnx1x2

.

We have

xn−1xnxn+1

+xnxn+1

x1+xn+1x1x2

− xn−1xnx1

− xnx1x2− xn+1 =

=xn(xn+1 − xn−1)

x1+x1(xn+1 − xn)

x2+xn−1xn − x2n+1

xn+1

=xn(xn+1 − xn + xn − xn−1)

x1+x1(xn+1 − xn)

x2+xn−1xn − x2n + x2n − x2n+1

xn+1

= (xn+1 − xn)

(xnx1

+x1x2− xnxn+1

− 1

)+ (xn − xn−1)

(xnx1− xnxn+1

).

Because xn+1 ≥ xn ≥ xn−1, we find thatxnx1≥ 1 ≥ xn

xn+1, and

xnx1

+x1x2≥ 2

√xnx2≥ 2 ≥ xn

xn+1+ 1.

315

Page 317: Олимпиадад бэлтгэгчдэд тусламж (Тэнцэтгэл биш 1)

Therefore, the last quantity is obviously nonnegative and it implies that

xn−1xnxn+1

+xnxn+1

x1+xn+1x1x2

− xn+1 ≥xn−1xnx1

+xnx1x2

,

as desired. Our proof is completed.?F?

00.26. Let a, b, c, d be positive real numbers such that c2 + d2 =(a2 + b2

)3.

Prove thata3

c+b3

d≥ 1.

(Singapore 2000)

Solution: From the given hypothesis and the Cauchy Schwarz Inequality, wehave(

a3

c+b3

d

)(ac+ bd) =

(√a3

c

)2

+

(√b3

d

)2[(√ac)2 +

(√bd)2]

(√a3

c·√ac+

√b3

d·√bd

)2

=(a2 + b2

)2=√

(a2 + b2) (c2 + d2) ≥ ac+ bd.

This implies thata3

c+b3

d≥ 1,

as desired.?F?

00.27. Given that x, y, z are positive real numbers satisfying xyz = 32, findthe minimum value of

x2 + 4xy + 4y2 + 2z2.

(United Kingdom 2000)

Solution: By the AM-GM Inequality, we have that

x2 + 4xy + 4y2 + 2z2 = (x− 2y)2 + 4xy + 4xy + 2z2 ≥ 4xy + 4xy + 2z2

≥ 3 3√

4xy · 4xy · 2z2 = 3 3√

32(xyz)2 = 96.

The equality holds when x = 2y and 4xy = 2z2, i.e. when x = 4, y = 2, z = 4.?F?

00.28. Prove that for any nonnegative real numbers a, b, c, the followinginequality holds

a+ b+ c

3− 3√abc ≤ max

{(√a−√b)2,(√

b−√c)2,(√c−√a)2}

.

(USA 2000)

316

Page 318: Олимпиадад бэлтгэгчдэд тусламж (Тэнцэтгэл биш 1)

First solution: We prove the stronger inequality

a+ b+ c− 33√abc ≤

(√a−√b)2

+(√

b−√c)2

+(√c−√a)2. (1)

The conclusion is immediate if abc = 0, so we assume that a, b, c > 0. Bymultiplying a, b, c by a suitable factor, we may reduce to the case abc = 1.Without loss of generality, assume that a and b are both greater than or equalto 1, or both less than or equal to 1. The desired inequality now becomes

a+ b+ c− 2√ab− 2

√bc− 2

√ca+ 3 ≥ 0.

By replacing c =1

ab,√bc =

1√a, and

√ca =

1√b, we find that it is equivalent

to

P = a+ b+1

ab− 2√ab− 2√

a− 2√

b+ 3 ≥ 0.

We have

P =(√

a−√b)2

+1

ab− 2√

a− 2√

b+ 3

=(√

a−√b)2

+

(1√a− 1

)2

+

(1√b− 1

)2

+1

ab− 1

a− 1

b+ 1

=(√

a−√b)2

+

(1√a− 1

)2

+

(1√b− 1

)2

+

(1

a− 1

)(1

b− 1

)≥ 0.

Therefore the inequality (1) is proved and from it, the result follows immedi-ately. Note that the equality holds if and only if a = b = c.

Second solution: We again prove the stronger inequality (1), which can bewritten ∑

sym

[a− 2(ab)1/2 + (abc)1/3] ≥ 0.

But this inequality follows from adding the two inequalities∑sym

[a− 2a2/3b1/3 + (abc)1/3] ≥ 0,

and ∑sym

(a2/3b1/3 + a1/3b2/3 − 2a1/2b1/2) ≥ 0.

The first of these is the Schur’s Inequality with x = a1/3, y = b1/3, z = c1/3,while the second follows from the AM-GM Inequality.

Third solution: Without loss of generality, assume that b is between a andc. The desired inequality reads

a+ b+ c− 33√abc ≤ 3

(a+ c− 2

√ac).

As a function of b, the right side minus the left side is concave (its second

derivative is −2(ac)1/3

3b5/3), so its minimum value in the range [a, c] occurs at

317

Page 319: Олимпиадад бэлтгэгчдэд тусламж (Тэнцэтгэл биш 1)

one of the endpoints. Thus, without loss of generality, we may assume a = b.Moreover, we may rescale the variables to get a = b = 1. Now the claim reads

2c+ 3c1/3 + 1

6≥ c1/2.

This is an instance of weighted AM-GM Inequality.?F?

01.1. Let a, b, c ≥ 0 such that a + b + c ≥ abc. Prove that the followinginequality holds

a2 + b2 + c2 ≥√

3abc.

(Balkan 2001)

Solution: From the AM-GM Inequality, we

(a+ b+ c)3 ≥ 27abc.

Multiplying both sides of this inequality by a+ b+ c > 0 and using the givenhypothesis, we deduce that

(a+ b+ c)4 = (a+ b+ c)3 (a+ b+ c) ≥ 27a2b2c2.

Taking the square root of each side of this inequality, we get

(a+ b+ c)2 ≥ 3√

3abc.

On the other hand, it is clear that 3(a2 + b2 + c2) ≥ (a + b + c)2 from theCauchy Schwarz Inequality. Combining this and the above inequality, we getthe result. It is easy to that the equality holds if and only if a = b = c =

√3.

?F?

01.2. Let x1, x2, x3 be real numbers in [−1, 1], and let y1, y2, y3 be real num-bers in [0, 1). Find the maximum possible value of the expression

1− x11− x2y3

· 1− x21− x3y1

· 1− x31− x1y2

.

(Belarus 2001)

Solution: Since 1−x1y2 = (1−x1)y2+(1−y2) > 0, 1+x1 ≥ 0 and 1−y2 > 0,we have

1− x11− x1y2

− 2

1 + y2= − (1 + x1)(1− y2)

(1 + y2)(1− x1y2)≤ 0.

From this, we deduce that

1− x11− x1y2

≤ 2

1 + y2≤ 2.

Similarly, we have1− x2

1− x2y3≤ 2,

1− x31− x3y1

≤ 2.

318

Page 320: Олимпиадад бэлтгэгчдэд тусламж (Тэнцэтгэл биш 1)

Now, note that from the given hypothesis, three expressions1− x1

1− x1y2,

1− x21− x2y3

,

1− x31− x3y1

must be nonnegative. From this note and the three inequalities

above, we get

1− x11− x2y3

· 1− x21− x3y1

· 1− x31− x1y2

=1− x1

1− x1y2· 1− x2

1− x2y3· 1− x3

1− x3y1≤ 8.

On the other hand, it is easy to see that the equality can be attained (forexample, take x1 = x2 = x3 = −1, y1 = y2 = y3 = 0), therefore the searchedmaximum is 8.

?F?

01.3. Let x and y be any two real numbers. Prove that

3(x+ y + 1)2 + 1 ≥ 3xy.

Under what conditions does equality hold?(Colombia 2001)

First solution: Denote t =x+ y

2, then we have xy ≤ t2 (according to the

AM-GM Inequality). And hence, it suffices to prove that

3(2t+ 1)2 + 1 ≥ 3t2.

We have 3(2t+1)2+1−3t2 = (3t+2)2 which is clearly nonnegative. Therefore,the above inequality holds and our proof is completed. Note that the equality

holds if and only if x = y = −2

3.

Second solution: For any real numbers X and Y , we have

X2 + Y 2 +XY =

(X +

Y

2

)2

+3Y 2

4≥ 0,

with equality if and only if X = Y = 0. Let x and y be any two real numbers.

Letting X = x+2

3and Y = y +

2

3, we obtain

(x+

2

3

)2

+

(y +

2

3

)2

+

(x+

2

3

)(y +

2

3

)≥ 0.

Expanding and multiplying by 3 gives

3x2 + 3y2 + 3xy + 6x+ 6y + 4 ≥ 0.

This may be written as

3(x+ y + 1)2 − 3xy + 1 ≥ 0,

from which we arrive at the desired inequality.?F?

319

Page 321: Олимпиадад бэлтгэгчдэд тусламж (Тэнцэтгэл биш 1)

01.4. Let n (n ≥ 2) be an integer and let a1, a2, . . . , an be positive realnumbers. Prove the inequality

(a31 + 1)(a32 + 1) · · · (a3n + 1) ≥ (a21a2 + 1) · · · (a2na1 + 1).

(Czech-Slovak-Polish 2001)

First solution: We try to apply the Cauchy Schwarz Inequality for eachfactor of the product in the right hand side. It is natural to write (1+a21a2)

2 ≤(1 + a31)(1 + a1a

22), since we need 1 + a31, which appears in the left hand side.

Similarly, we can write

(1 + a22a3)2 ≤ (1 + a32)(1 + a2a

23), . . . , (1 + a2na1)

2 ≤ (1 + a3n)(1 + ana21).

Multiplying we obtain

[(a21a2+1) · · · (a2na1+1)]2 ≤ [(a31+1) · · · (a3n+1)][(1+a1a22) · · · (1+ana

21)]. (∗)

Now, we use again the same argument to find that

[(1+a1a22) · · · (1+ana

21)]

2 ≤ [(a31+1) · · · (a3n+1)][(a21a2+1) · · · (a2na1+1)]. (∗∗)

Thus, if (a21a2 + 1) · · · (a2na1 + 1) ≥ (1 + a1a22) · · · (1 + ana

21), then (∗) will give

the answer, otherwise (∗∗) will. It is easy to see that the equality holds if andonly if a1 = a2 = · · · = an.

Second solution: From the Holder’s Inequality,

(a31 + 1)1/3(a31 + 1)1/3(a32 + 1)1/3 ≥ a21a2 + 1,

we get

(a31 + 1)2(a32 + 1) ≥ (a21a2 + 1)3.

In the same manner, we can establish the folllowing inequalities(a31 + 1

)2 (a32 + 1

)≥(a21a2 + 1

)3, . . . ,

(a3n + 1

)2 (a31 + 1

)≥(a2na1 + 1

)3.

Multiplying them up and taking the cube root of each side of the resultinginequality, we get the desired result.

?F?

01.5. Prove that for any positive real numbers a, b, c, the following inequalityholds

a√a2 + 8bc

+b√

b2 + 8ca+

c√c2 + 8ab

≥ 1.

(IMO 2001)

First solution: Applying the AM-GM Inequality, we have

a√a2 + 8bc

=2a(a+ b+ c)

2(a+ b+ c)√a2 + 8bc

≥ 2a(a+ b+ c)

(a+ b+ c)2 + a2 + 8bc.

320

Page 322: Олимпиадад бэлтгэгчдэд тусламж (Тэнцэтгэл биш 1)

By establishing the similar inequalities for the two other expressions, we get∑ a√a2 + 8bc

≥ 2(a+ b+ c)∑ a

(a+ b+ c)2 + a2 + 8bc.

On the other hand, the Cauchy Schwarz Inequality implies that

∑ a

(a+ b+ c)2 + a2 + 8bc=∑ a2

a(a+ b+ c)2 + a3 + 8abc

≥ (a+ b+ c)2∑[a(a+ b+ c)2 + a3 + 8abc]

=(a+ b+ c)2

(a+ b+ c)3 + a3 + b3 + c3 + 24abc.

It coud be said that∑ a√a2 + 8bc

≥ 2(a+ b+ c)3

(a+ b+ c)3 + a3 + b3 + c3 + 24abc.

Therefore, in order to prove the original inequality, it suffices to prove that

(a+ b+ c)3 ≥ a3 + b3 + c3 + 24abc.

Of course, this is trivial since by the AM-GM Inequality, we have

(a+ b+ c)3 = a3 + b3 + c3 + 3(a+ b)(b+ c)(c+ a) ≥ a3 + b3 + c3 + 24abc.

The equality occurs iff a = b = c.

Second solution: First we shall prove that

a√a2 + 8bc

≥ a43

a43 + b

43 + c

43

,

or equivalently, that (a

43 + b

43 + c

43

)2≥ a

23 (a2 + 8bc).

The AM-GM inequality yields(a

43 + b

43 + c

43

)2− a

83 =

(b43 + c

43

)(a

43 + a

43 + b

43 + c

43

)≥ 2b

23 c

23 · 8a

23 b

13 c

13 = 8a

23 bc.

Thus (a

43 + b

43 + c

43

)2≥ a

83 + 8a

23 bc = a

23 (a2 + 8bc),

so our statement is proved. Similarly, we have

b√b2 + 8ca

≥ b43

a43 + b

43 + c

43

, andc√

c2 + 8ab≥ c

43

a43 + b

43 + c

43

.

321

Page 323: Олимпиадад бэлтгэгчдэд тусламж (Тэнцэтгэл биш 1)

Adding these three inequalities yields

a√a2 + 8bc

+b√

b2 + 8ca+

c√c2 + 8ab

≥ 1.

Third solution: To remove the square roots, we make the following substi-tution

x =a√

a2 + 8bc, y =

b√b2 + 8ca

, z =c√

c2 + 8ab.

Clearly, x, y, z ∈ (0, 1). Our aim is to show that x+ y+ z ≥ 1. We notice that

a2

8bc=

x2

1− x2,

b2

8ac=

y2

1− y2,

c2

8ab=

z2

1− z2.

Hence1

512=

(x2

1− x2

)(y2

1− y2

)(z2

1− z2

).

Thus, we need to show that x+y+z ≥ 1 where 0 < x, y, z < 1 and (1−x2)(1−y2)(1 − z2) = 512(xyz)2. We use contradiction method. Assume that thereexist x, y, z satisfying both conditions 0 < x, y, z < 1, (1−x2)(1−y2)(1−z2) =512(xyz)2 and also x+y+z < 1. We will prove that this is impossible. Indeed,from 1 > x+ y + z, it follows that

(1− x2)(1− y2)(1− z2) >> [(x+ y + z)2 − x2][(x+ y + z)2 − y2][(x+ y + z)2 − z2]= (x+ x+ y + z)(y + z)(x+ y + y + z)(z + x)(x+ y + z + z)(x+ y)

≥ 4(x2yz)14 · 2(yz)

12 · 4(y2zx)

14 · 2(zx)

12 · 4(z2xy)

14 · 2(xy)

12

= 512(xyz)2.

This is a contradiction.?F?

01.6. Let a, b, c be positive real numbers such that abc = 1. Prove that

ab+cbc+aca+b ≤ 1.

(India 2001)

Solution: The desired inequality is equivalent to

(b+ c) ln a+ (c+ a) ln b+ (a+ b) ln c ≤ 0.

Without loss of generality, we may assume that a ≥ b ≥ c. By this assumption,we have b + c ≤ c + a ≤ a + b, and ln a ≥ ln b ≥ ln c. Therefore, by theChebyshev’s Inequality, we get∑

(b+ c) ln a ≤ 1

3[(b+ c) + (c+ a) + (a+ b)] (ln a+ ln b+ ln c)

=2

3(a+ b+ c) ln(abc) = 0,

322

Page 324: Олимпиадад бэлтгэгчдэд тусламж (Тэнцэтгэл биш 1)

as claimed. It is easy to see that the equality holds if and only if a = b = c = 1.?F?

01.7. Let x, y, z be positive real numbers such that xyz ≥ xy+yz+zx. Provethat

xyz ≥ 3 (x+ y + z) .

(India 2001)

Solution: Applying the well-known inequality (a+ b+ c)2 ≥ 3(ab+ bc+ ca)for the triple (a, b, c) = (xy, yz, zx), we deduce that

(xy + yz + zx)2 ≥ 3xyz(x+ y + z).

On the other hand, from the given hypothesis, we have x2y2z2 ≥ (xy + yz +zx)2. It follows that

x2y2z2 ≥ 3xyz(x+ y + z).

Dividing both sides of the last inequality by xyz > 0, we get the desired result.Note that the equality holds if and only if x = y = z = 3.

?F?

01.8. Prove that for any real numbers a, b, c the following inequality holds

(b+ c− a)2(c+ a− b)2(a+ b− c)2 ≥ (b2 + c2 − a2)(c2 + a2 − b2)(a2 + b2 − c2).

(Japan 2001)

First solution: The inequality is equivalent to

3(a2 + b2 + c2)(b+ c− a)2(c+ a− b)2(a+ b− c)2 ≥≥ 3(a2 + b2 + c2)(b2 + c2 − a2)(c2 + a2 − b2)(a2 + b2 − c2).

By the Cauchy Schwarz Inequality, we have

3(a2 + b2 + c2) ≥ (a+ b+ c)2,

and thus it is suffice to prove that

(a+ b+ c)2(b+ c− a)2(c+ a− b)2(a+ b− c)2 ≥≥ 3(a2 + b2 + c2)(b2 + c2 − a2)(c2 + a2 − b2)(a2 + b2 − c2).

Since

(a+ b+ c)2(b+ c− a)2(c+ a− b)2(a+ b− c)2 =(

2∑

a2b2 −∑

a4)2

and

3(a2+b2+c2)(b2+c2−a2)(c2+a2−b2)(a2+b2−c2) = 3(

2∑

a4b4 −∑

a8),

the last inequality is equivalent to(2∑

xy −∑

x2)2≥ 3

(2∑

x2y2 −∑

x4),

323

Page 325: Олимпиадад бэлтгэгчдэд тусламж (Тэнцэтгэл биш 1)

where x = a2, y = b2, z = c2. This last one rewrites as

4∑

x2(x− y)(x− z) ≥ 0,

which is a particular case of the Schur’s Inequality. Note that the equalityholds iff a = b = c, or c = 0 and a = b, or b = 0 and c = a, or a = 0 and b = c.

Second solution: It is clear that we only need to consider the case a2, b2, c2

are the sidelengths of a triangle (because in the conversed case, the giveninequality is trivial). Then, we will prove that

[a2 − (b− c)2]2 ≥ (a2 + b2 − c2)(a2 + c2 − b2).

Indeed, it is equivalent to

a4 − 2a2(b− c)2 + (b− c)4 ≥ a4 − (b− c)2(b+ c)2 ≥ 0,

or

(b− c)2(b2 + c2 − a2) ≥ 0.

Of course, this is true and so the claim is proved. Proceeding by the sameway, we can establish the similar inequalities as follows

[b2 − (c− a)2]2 ≥ (b2 + c2 − a2)(b2 + a2 − c2),

[c2 − (a− b)2]2 ≥ (c2 + a2 − b2)(c2 + b2 − a2).

Multiplying these three inequalities, we get

[a2−(b−c)2]2[b2−(c−a)2]2[c2−(a−b)2]2 ≥ (a2+b2−c2)2(b2+c2−a2)2(c2+a2−b2)2,

which is equivalent to

(a+b−c)4(b+c−a)4(c+a−b)4 ≥≥ (a2+b2−c2)2(b2+c2−a2)2(c2+a2−b2)2.

Now, taking square root for both sides, we can get the result.?F?

01.9. Let a, b, c be the sidelengths of an acute-angled triangle. Prove that

(a+ b+ c)(a2 + b2 + c2)(a3 + b3 + c3) ≥ 4(a6 + b6 + c6).

(Japan 2001)

Solution: By the Cauchy Schwarz Inequality, we have

(a+ b+ c)(a3 + b3 + c3) ≥ (a2 + b2 + c2)2,

and therefore it is enough to prove that

(a2 + b2 + c2)3 ≥ 4(a6 + b6 + c6).

324

Page 326: Олимпиадад бэлтгэгчдэд тусламж (Тэнцэтгэл биш 1)

Set x = a2, y = b2, z = c2. Since a, b, c are the sidelengths of an acute-angledtriangle, (it is well-known and easy to prove that) the numbers x, y, z aretheirselves the sidelengths of a triangle. The inequality to prove now becomes

(x+ y + z)3 ≥ 4(x3 + y3 + z3).

According to the triangle Inequality, we proceed as follows

(x+ y + z)3 = x3 + y3 + z3 + 3x2(y + z) + 3y2(z + x) + 3z2(x+ y) + 6xyz

≥ x3 + y3 + z3 + 3x2(y + z) + 3y2(z + x) + 3z2(x+ y)

≥ x3 + y3 + z3 + 3x2 · x+ 3y2 · y + 3z2 · z= 4(x3 + y3 + z3).

Note that the equality holds if and only if the triangle is degenerated, havingsidelengths of the type (0, t, t), where t is a positive real number.

?F?

01.10. Prove that for any real numbers x1, x2, . . . , xn, y1, y2, . . . , yn such thatx21 + x22 + · · ·+ x2n = y21 + y22 + · · ·+ y2n = 1,

(x1y2 − x2y1)2 ≤ 2

(1−

n∑k=1

xkyk

).

(Korea 2001)

First solution: We clearly have the inequality

(x1y2 − x2y1)2 ≤∑

1≤i<j≤n(xiyj − xjyi)2 =

(n∑i=1

x2i

)(n∑i=1

y2i

)−

(n∑i=1

xiyi

)2

=

(1−

n∑i=1

xiyi

)(1 +

n∑i=1

xiyi

).

Because we also have

∣∣∣∣∣n∑i=1

xiyi

∣∣∣∣∣ ≤ 1, we find immediately that

(1−

n∑i=1

xiyi

)(1 +

n∑i=1

xiyi

)≤ 2

(1−

n∑i=1

xiyi

),

and the problem is solved.

Second solution: Observe that

2

(1−

n∑k=1

xkyk

)=

n∑k=1

x2k +n∑k=1

y2k − 2n∑k=1

xkyk =n∑k=1

(xk − yk)2

≥ (x1 − y1)2 + (x2 − y2)2,

therefore, it suffices to prove that

(x1 − y1)2 + (x2 − y2)2 ≥ (x1y2 − x2y1)2.

325

Page 327: Олимпиадад бэлтгэгчдэд тусламж (Тэнцэтгэл биш 1)

However, this is true since by the Cauchy Schwarz Inequality, we have

(x1 − y1)2 + (x2 − y2)2 ≥ (x22 + x21)[(x1 − y1)2 + (x2 − y2)2]≥ [x2(x1 − y1) + x1(y2 − x2)]2 = (x1y2 − x2y1)2.

The proof is completed.?F?

01.11. Prove that if a, b, c are positive real numbers, then√a4 + b4 + c4 +

√a2b2 + b2c2 + c2a2 ≥

√a3b+ b3c+ c3a+

√ab3 + bc3 + ca3.

(Korea 2001)

Solution: Without loss of generality, we may assume that

max{a3b+ b3c+ c3a, ab3 + bc3 + ca3

}= a3b+ b3c+ c3a.

Then, we have√a3b+ b3c+ c3a+

√ab3 + bc3 + ca3 ≤ 2

√a3b+ b3c+ c3a.

On the other hand, applying the AM-GM Inequality in combination with theCauchy Schwarz Inequality, we find that√a4 + b4 + c4 +

√a2b2 + b2c2 + c2a2 ≥ 2 4

√(a4 + b4 + c4)(a2b2 + b2c2 + c2a2)

≥ 24

√(a2 · ab+ b2 · bc+ c2 · ca)2

= 2√a3b+ b3c+ c3a.

From this and the above inequality, it follows immediately that√a4 + b4 + c4 +

√a2b2 + b2c2 + c2a2 ≥

√a3b+ b3c+ c3a+

√ab3 + bc3 + ca3.

Note that the equality holds if and only if a = b = c.?F?

01.12. Prove that for all a, b, c > 0,√(a2b+ b2c+ c2a) (ab2 + bc2 + ca2) ≥ abc+ 3

√(a3 + abc) (b3 + abc) (c3 + abc).

(Korea 2001)

Solution: Dividing by abc, it becomes√(a

c+b

a+c

b

)(c

a+a

b+b

c

)≥ abc+ 3

√(a2

bc+ 1

)(b2

ca+ 1

)(c2

ab+ 1

).

After the substitution x =a

b, y =

b

c, z =

c

a, we obtain the constraint xyz = 1.

It takes the form

√(x+ y + z) (xy + yz + zx) ≥ 1 + 3

√(xz

+ 1)(y

x+ 1)(z

y+ 1

).

326

Page 328: Олимпиадад бэлтгэгчдэд тусламж (Тэнцэтгэл биш 1)

From the constraint xyz = 1, we find two identities(xz

+ 1)(y

x+ 1)(z

y+ 1

)= (z + x)(x+ y)(y + z),

and

(x+ y + z) (xy + yz + zx) = (x+y)(y+z)(z+x)+xyz = (x+y)(y+z)(z+x)+1.

Letting p = 3√

(x+ y)(y + z)(z + x), the inequality now becomes√p3 + 1 ≥

1+p. Applying the AM-GM Inequality, we have p ≥ 3

√2√xy · 2√yz · 2

√zx =

2. It follows that

(p3 + 1)− (1 + p)2 = p(p+ 1)(p− 2) ≥ 0.

The proof is completed. Note that the equality holds if and only if a = b = c.?F?

01.13. Prove that if a, b, c > 0 have product 1, then

(a+ b)(b+ c)(c+ a) ≥ 4(a+ b+ c− 1).

(MOSP 2001)

First solution: Using the identity (a+ b)(b+ c)(c+a) = (a+ b+ c)(ab+ bc+ca)− 1, we reduce the problem to the following one

ab+ bc+ ca+3

a+ b+ c≥ 4.

Now, we can apply the AM-GM Inequality in the following form

ab+ bc+ ca+3

a+ b+ c= 3 · ab+ bc+ ca

3+

3

a+ b+ c≥ 4 4

√(ab+ bc+ ca)3

9(a+ b+ c).

And so, it is enough to prove that

(ab+ bc+ ca)3 ≥ 9(a+ b+ c).

But this is easy, because we clearly have ab+ bc+ ca ≥ 3 and (ab+ bc+ ca)2 ≥3abc(a+ b+ c) = 3(a+ b+ c). It is easy to see that the equality holds if andonly if a = b = c = 1.

Second solution: We will use the fact that (a+ b)(b+ c)(c+ a) ≥ 8

9(a+ b+

c)(ab+ bc+ ca). So, it is enough to prove that

2

9(ab+ bc+ ca) +

1

a+ b+ c≥ 1.

Using the AM-GM Inequality, we can write

2

9(ab+ bc+ ca) +

1

a+ b+ c≥ 3 3

√(ab+ bc+ ca)2

81(a+ b+ c)≥ 1,

327

Page 329: Олимпиадад бэлтгэгчдэд тусламж (Тэнцэтгэл биш 1)

because (ab+ bc+ ca)2 ≥ 3abc(a+ b+ c) = 3(a+ b+ c).

Third solution: By homogenizing, we can write the inequality as

(a+ b)(b+ c)(c+ a) + 4abc ≥ 43√a2b2c2(a+ b+ c).

Without loss of generality, we can assume that a ≥ b ≥ c. Now, we write theinequality in form

(b+ c)[(a+ b)(a+ c)− 4

3√a2b2c2

]≥ 4

3√a2b2c2

(a− 3√abc).

Because (a+ b)(a+ c) = a2 + ab+ ac+ bc, it is equivalent to

(b+ c)(a2 + ab+ bc+ ca− 4

3√a2b2c2

)≥ 4

3√a2b2c2

(a− 3√abc).

Using again the estimation ab+ bc+ ca ≥ 33√a2b2c2, it suffices to prove that

(b+ c)(a2 − 3

√a2b2c2

)≥ 4

3√a2b2c2

(a− 3√abc),

or equivalently,

(b+ c)(a+

3√abc)≥ 4

3√a2b2c2.

Of course, this is obvious since by the AM-GM Inequality, we have

(b+ c)(a+

3√abc)≥ 2√bc · 2

√a

3√abc = 4

3√a2b2c2.

?F?

01.14. Show that the inequality

n∑i=1

ixi ≤(n

2

)+

n∑i=1

xii

holds for every integer n ≥ 2 and all real numbers x1, x2, . . . , xN ≥ 0.(Poland 2001)

Solution: From the Bernoulli’s Inequality, for each nonnegative real numberx and for each positive integer k, we have xk = [1 + (x− 1)]k ≥ 1 + k(x− 1),and from this, we deduce that kx ≤ xk +k− 1, with equality, for k ≥ 2, if andonly if x = 1. Using this for each xi and summing leads to

n∑i=1

ixi ≤n∑i=1

[xii + (i− 1)] =

(n

2

)+

n∑i=1

xii,

as desired. Note that the equality holds if and only if x2 = · · · = xn = 1 (andx1 ≥ 0 is arbitrary).

?F?

01.15. Let a and b be positive real numbers in the interval (0, 1]. Prove that

1√1 + a2

+1√

1 + b2≤ 2√

1 + ab.

328

Page 330: Олимпиадад бэлтгэгчдэд тусламж (Тэнцэтгэл биш 1)

(Russia 2001)

Solution: Because a and b are positive real numbers, there are angles x and y,with 0◦ < x, y < 90◦, such that tanx = a and tan y = b. The desired inequalityis clearly true when a = b. Hence we assume that a 6= b, or equivalently, x 6= y.

Then 1 + a2 = sec2 x and1√

1 + a2= cosx. Note that

1 + ab =cosx cos y + sinx sin y

cosx cos y=

cos(x− y)

cosx cos y

by the addition and subtraction formulas. The desired inequality reduces to

cosx+ cos y ≤ 2

√cosx cos y

cos(x− y).

Squaring both sides, we can rewrite it as

cos2 x+ cos2 y + 2 cosx cos y ≤ 4 cosx cos y

cos(x− y).

Because 0◦ < |x − y| < 90◦, it follows that 0 < cos(x − y) < 1. Hence

2 cosx cos y ≤ 2 cosx cos y

cos(x− y). It suffices to show that

cos(x− y)(cos2 x+ cos2 y) ≤ 2 cosx cos y,

orcos(x− y)(cos 2x+ cos 2y + 2) ≤ 4 cosx cos y

by the double-angle formulas. By the sum-to-product formulas, the last in-equality is equivalent to

cos(x− y)[2 cos(x− y) cos(x+ y) + 2] ≤ 2[cos(x− y) + cos(x+ y)],

or cos 2(x − y) cos(x + y) ≤ cos(x + y), which is clearly true, because for0 < a, b ≤ 1, we have 0◦ < x, y ≤ 45◦, and so 0◦ < x + y ≤ 90◦ andcos(x+ y) > 0. This completes our proof.

?F?

01.16. Let a, b, c, x, y, z be positive real numbers such that x + y + z = 1.Prove that

ax+ by + cz + 2√

(xy + yz + zx)(ab+ bc+ ca) ≤ a+ b+ c.

(Ukraine 2001)

First solution: We will use the Cauchy Schwarz Inequality twice. First, wecan write ax + by + cz ≤

√a2 + b2 + c2 ·

√x2 + y2 + z2 and then we apply

again the Cauchy Schwarz Inequality to obtain

ax+ by + cz + 2√

(xy + yz + zx)(ab+ bc+ ca) ≤

≤√∑

a2 ·√∑

x2 +

√2∑cyc

xy ·√

2∑

ab

≤√∑

a2 + 2∑

ab ·√∑

x2 + 2∑

xy =∑

a.

329

Page 331: Олимпиадад бэлтгэгчдэд тусламж (Тэнцэтгэл биш 1)

Second solution: The inequality being homogeneous in a, b, c, so we canassume that a+ b+ c = 1. We apply this time the AM-GM Inequality and wefind that the left hand side of the original inequality is not greater than

ax+ by + cz + xy + yz + zx+ ab+ bc+ ca.

Consequently,

xy + yz + zx+ ab+ bc+ ca =1− x2 − y2 − z2

2+

1− a2 − b2 − c2

2≤ 1− ax− by − cz,

the last one being equivalent to (x− a)2 + (x− b)2 + (x− c)2 ≥ 0.

Third solution: Applying the AM-GM Inequality, we have

2√

(ab+ bc+ ca)(xy + yz + zx) =

= 2

√[a(y + z) +

bc(y + z)

b+ c

] [x(b+ c) +

yz(b+ c)

y + z

]≤ a(y + z) +

bc(y + z)

b+ c+ x(b+ c) +

yz(b+ c)

y + z.

Therefore, if we denote with P the left hand side of the original inequality,then we find that

P ≤ ax+ by + cz + a(y + z) +bc(y + z)

b+ c+ x(b+ c) +

yz(b+ c)

y + z.

And so, it suffices to prove that

ax+by+cz+a(y+z)+bc(y + z)

b+ c+x(b+c)+

yz(b+ c)

y + z≤ (a+b+c)(x+y+z),

or

bz + cy ≥ bc(y + z)

b+ c+yz(b+ c)

y + z.

The last one is true since by the Cauchy Schwarz Inequality, we have

bz + cy − bc(y + z)

b+ c=b2z + c2y

b+ c=

(b2z + c2y)(y + z)

(b+ c)(y + z)

≥(b√zy + c

√yz)2

(b+ c)(y + z)=yz(b+ c)

y + z.

?F?

01.17. Let a, b, c be positive real numbers such that a + b + c ≥ abc. Provethat at least two of the inequalities

2

a+

3

b+

6

c≥ 6,

2

b+

3

c+

6

a≥ 6,

2

c+

3

a+

6

b≥ 6

330

Page 332: Олимпиадад бэлтгэгчдэд тусламж (Тэнцэтгэл биш 1)

are true.(USA 2001)

Solution: Denote x =1

a, y =

1

b, z =

1

c, then we have xy + yz + zx ≥ 1, and

we are required to prove that: There are at least two of the inequalities

2x+ 3y + 6z ≥ 6, 2y + 3z + 6x ≥ 6, 2z + 3x+ 6y ≥ 6

are true. We have that

(2x+ 3y + 6z) + (2y + 3z + 6x) = 8x+ 5y + 9z,

and

(8x+ 5y + 9z)2 − 144(xy + yz + zx) = 64x2 − 64xy + 25y2 − 54yz + 81z2

= 16(2x− y)2 + 9(y − 3z)2 ≥ 0,

therefore

(2x+ 3y + 6z) + (2y + 3z + 6x) ≥ 12√xy + yz + zx ≥ 12,

and it follows that

max {2x+ 3y + 6z, 2y + 3z + 6x} ≥ 6.

Without loss of generality, we may assume that max {2x+ 3y + 6z, 2y + 3z + 6x} =2x+ 3y + 6z, then from the above inequality, we have 2x+ 3y + 6z ≥ 6.On the other hand, proceeding by the same way as above, we can also findthat

(2y + 3z + 6x) + (2z + 3x+ 6y) ≥ 12,

from which it follows

max {2y + 3z + 6x, 2z + 3x+ 6y} ≥ 6.

These arguments show that there are at least 2 inequalities among the giveninequalities holds.

?F?

01.18. Prove that for any nonnegative real numbers a, b, c such that a2 + b2 +c2 + abc = 4, we have

0 ≤ ab+ bc+ ca− abc ≤ 2.

(USA 2001)

First solution: From the condition, at least one of a, b and c does not exceed1, say a ≤ 1. Then

ab+ bc+ ca− abc = a(b+ c) + bc(1− a) ≥ 0.

To obtain the equality, we have a(b+c) = bc(1−a) = 0. If a = 1, then b+c = 0or b = c = 0, which contradicts the given condition a2 + b2 + c2 + abc = 4.

331

Page 333: Олимпиадад бэлтгэгчдэд тусламж (Тэнцэтгэл биш 1)

Hence 1− a 6= 0 and only one of b and c is 0. Without loss of generality, sayb = 0. Therefore b+ c > 0 and a = 0. Plugging a = b = 0 back into the givencondition gives c = 2. By permutation, the lower bound holds if and only if(a, b, c) is one of the triples (2, 0, 0), (0, 2, 0), and (0, 0, 2).

Now we prove the upper bound. Let us note that at least two of the threenumbers a, b, and c are both greater than or equal to 1 or less than or equal to1. Without loss of generality, we assume that the numbers with this propertyare b and c. Then we have

(1− b)(1− c) ≥ 0. (1)

The given equality a2 + b2 + c2 + abc = 4 and the inequality b2 + c2 ≥ 2bcimply

a2 + 2bc+ abc ≤ 4, or bc(2 + a) ≤ 4− a2.

Dividing both sides of the last inequality by 2 + a yields

bc ≤ 2− a. (2)

Combining (1) and (2) gives

ab+ bc+ ca− abc ≤ a(b+ c) + (2− a)− abc= 2− a(1 + bc− b− c)= 2− a(1− b)(1− c) ≤ 2,

as desired. The last equality holds if and only if b = c and a(1− b)(1− c) = 0.Hence, the equality for the upper bound holds if and only if (a, b, c) is one ofthe triples (1, 1, 1),

(0,√

2,√

2),(√

2, 0,√

2), and

(√2,√

2, 0).

Second solution: We won’t prove again the lower part, since this is an easyproblem. Let us concentrate on the upper bound. Let a ≥ b ≥ c and leta = x+ y, b = x− y (x ≥ y ≥ 0). The hypothesis becomes x2(2 + c) + y2(2−c) = 4 − c2, and we have to prove that (x2 − y2)(1 − c) ≤ 2(1 − xc). Since

y2 = 2 + c− 2 + c

2− cx2, the problem asks to prove the inequality

4x2 − (4− c2)2− c

(1− c) ≤ 2(1− xc).

Of course, we have c ≤ 1 and 0 ≤ y2 = 2 + c − 2 + c

2− cx2, therefore x2 ≤ 2 − c

and hence x ≤√

2− c. Now, consider the function f :[0,√

2− c]→ R,

f(x) = 2(1− cx)− 4x2 − (4− c2)2− c

(1− c).

We have f ′(x) = −2c − 8x(1− c)2− c

≤ 0 and thus f is decreasing and f(x) ≥

f(√

2− c). So we have to prove that f

(√2− c

)≥ 0, or equivalently

2(1− c

√2− c

)≥ (2− c)(1− c).

332

Page 334: Олимпиадад бэлтгэгчдэд тусламж (Тэнцэтгэл биш 1)

By some simple calculations, we find that it is equivalent to c(1−√

2− c)2 ≥

0, clearly true. Thus, the problem is solved.

Third solution: We give another way to prove the upper part

ab+ bc+ ca− abc ≤ 2.

Letting a = 2p, b = 2q, c = 2r, we get p2 + q2 + r2 + 2pqr = 1. Therefore, we

may put a = 2 cosA, b = 2 cosB, c = 2 cosC for some A,B,C ∈[0,π

2

]with

A+B + C = π. We are required to prove

cosA cosB + cosB cosC + cosC cosA− 2 cosA cosB cosC ≤ 1

2.

Assume that A ≥ π

3or 1− 2 cosA ≥ 0. Note that

cosA cosB + cosB cosC + cosC cosA− 2 cosA cosB cosC =

= cosA(cosB + cosC) + cosB cosC(1− 2 cosA).

We apply the Jensen’s Inequality to deduce cosB + cosC ≤ 32 − cosA. Note

that 2 cosB cosC = cos(B − C) + cos(B + C) ≤ 1− cosA. These imply that

cosA(cosB + cosC) + cosB cosC(1− 2 cosA) ≤

≤ cosA

(3

2− cosA

)+

(1− cosA

2

)(1− 2 cosA).

However, it’s easy to verify that cosA

(3

2− cosA

)+

(1− cosA

2

)(1−2 cosA) =

1

2.

?F?

01.19. Let x, y, z be positive real numbers satisfying

(i)1√2≤ z ≤ 1

2min

{x√

2, y√

3}

;

(ii) x+ z√

3 ≥√

6;(iii) y

√3 + z

√10 ≥ 2

√5.

Find the maximum of P (x, y, z) =1

x2+

2

y2+

3

z2.

(Vietnam 2001)

First solution: From the first condition, we have√

2 ≥ 1

z≥√

2

x, and it

follows that1

z2≤ 2,

z2

x2≤ 1

2. The second condition leads us to x2 + 3z2 ≥ 3,

and from it, we deduce that2

3+

2z2

x2≥ 2

x2. Thus

1

x2+

1

z2=

2

x2+

1

z2− 1

x2≤ 2

3+

2z2

x2+

1

z2

(1− z2

x2

)≤ 2

3+

2z2

x2+ 2

(1− z2

x2

)=

8

3.

333

Page 335: Олимпиадад бэлтгэгчдэд тусламж (Тэнцэтгэл биш 1)

Similarly, using the first and third conditions, we get

1

y2+

1

z2≤ 13

5.

From these two inequalities, we obtain

P =1

x2+

1

z2+ 2

(1

y2+

1

z2

)≤ 8

3+

26

3=

118

15.

In addition, from above proofs, it is easy to see that P =118

15if and only if

x =

√3

2, y =

√5

3, z =

1√2. Obviously, these values of x, y, z satisfy (i), (ii)

and (iii). So maxP =118

15.

Second solution: From the given conditions, we have x, y > z and

x ≥√

3(√

2− z), y ≥

√10

3

(√2− z

).

To find the maximum of P, we need to consider two cases:

Case 1.√

3(√

2− z)≥ z. In this case, we find that

1√2≤ z ≤

√6√

3 + 1, and

1

x2+

2

y2+

3

z2≤ 1

3(√

2− z)2 +

210

3

(√2− z

)2 +3

z2

=14

15(√

2− z)2 +

3

z2= f(z).

Now, we find that f ′′(z) =18

z4+

28

5(√

2− z)4 > 0, therefore f(z) is convex,

and since1√2≤ z ≤

√6√

3 + 1, it follows that

f(z) ≤ max

{f

(1√2

), f

( √6√

3 + 1

)}= max

{118

15,58 + 29

√3

15

}=

118

15.

Case 2.√

3(√

2− z)≤ z. In this case, we find that z ≥

√6√

3 + 1. Therefore,

by using the inequalities x > z and y > z, we get

1

x2+

2

y2+

3

z2≤ 6

z2≤ 6( √

6√3 + 1

)2 = 4 + 2√

3 <118

15.

Actually, we have proved in both cases the following inequality

P =1

x2+

2

y2+

3

z2≤ 118

15.

334

Page 336: Олимпиадад бэлтгэгчдэд тусламж (Тэнцэтгэл биш 1)

Moreover, let x =

√3

2, y =

√5

3, z =

1√2, we can easily see that the conditions

(i), (ii) and (iii) are still satisfied and also P =118

15. Therefore maxP =

118

15.

?F?

01.20. Let x, y, z be positive real numbers such that

(i)2

5≤ z ≤ min {x, y} ;

(ii) xz ≥ 4

15;

(iii) yz ≥ 1

5.

Determine the maximum possible value of

P (x, y, z) =1

x+

2

y+

3

z.

(Vietnam 2001)

First solution: From the given condition, we have x ≥ 4

15z, y ≥ 1

5z. To find

the maximum value of P, we will consider two cases

The first case is when4

15z≥ z. In this case, we have

2

5≤ z ≤ 2√

15. Now,

observe that

1

x+

2

y+

3

z≤ 15z

4+ 2 · 5z +

3

z=

55

4z +

3

z= f(z).

It is easy to check that f(z) is convex, therefore

f(z) ≤ max

{f

(2

5

), f

(2√15

)}= max

{13,

10√

5

3

}= 13,

and hence, we deduce that P ≤ 13 with equality iff z =2

5, x =

4

15z=

2

3, y =

1

5z=

1

2.

The second case is when4

15z≤ z. In this case, we have z ≥ 2√

15, and hence

1

x+

2

y+

3

z≤ 6

z≤ 3√

15 < 13.

So, in both cases, we always have P ≤ 13 with equality iff x =2

3, y =

1

2, z =

2

5.

Therefore, the searched maximum of P is 13.

Second solution: Applying the AM-GM Inequality in combination with thegiven hypothesis, we deduce that

3x+ 5z ≥ 2√

15xz ≥ 4, 4y + 5z ≥ 4√

5yz ≥ 4.

335

Page 337: Олимпиадад бэлтгэгчдэд тусламж (Тэнцэтгэл биш 1)

From this, it follows that

2

x≤ 5z

2x+

3

2,

2

y≤ 5z

2y+ 2.

Using these two inequalities with noting that 1− z

x≥ 0, 1− z

y≥ 0 and z ≥ 2

5,

we have

1

x+

1

z=

2

x+

1

z

(1− z

x

)≤ 5z

2x+

3

2+

1

z

(1− z

x

)≤ 5z

2x+

3

2+

5

2

(1− z

x

)= 4,

and

1

y+

1

z=

2

y+

1

z

(1− z

y

)≤ 5z

2y+ 2 +

1

z

(1− z

y

)≤ 5z

2y+ 2 +

5

2

(1− z

y

)=

9

2.

Therefore, we conclude that

P =1

x+

2

y+

3

z=

(1

x+

1

z

)+ 2

(1

y+

1

z

)≤ 4 + 2 · 9

2= 13,

with equality if and only if 3x = 4y = 5z, xz =4

15, yz =

1

5, z =

2

5, i.e. when

x =2

3, y =

1

2, z =

2

5.

?F?

01.21. Find the minimum value of the expression1

a+

2

b+

3

cwhere a, b, c are

positive real numbers such that 21ab+ 2bc+ 8ca ≤ 12.(Vietnam 2001)

First solution: Let x =1

a, y =

2

b, z =

3

c. Then it is easy to check that

the condition of the problem becomes 2xyz ≥ 2x + 4y + 7z. And we need tominimize x+y+z. But from the hypothesis, we find that z(2xy−7) ≥ 2x+4y

and hence 2xy > 7, z ≥ 2x+ 4y

2xy − 7. Now, we transform the expression so that

after one application of the AM-GM Inequality, the numerator 2xy− 7 shouldvanish

x+y+z ≥ x+y+2x+ 4y

2xy − 7= x+

11

2x+

(y − 7

2x

)+

2x+14

x2xy − 7

≥ x+11

2x+2

√1 +

7

x2.

But, it is immediate to prove that 2

√1 +

7

x2≥

3 +7

x2

and so x + y + z ≥3

2+ x +

9

x≥ 15

2. We have equality for x = 3, y =

5

2, z = 2. Therefore, in the

initial problem, the answer is15

2, achieved for a =

1

3, b =

4

5, c =

3

2.

336

Page 338: Олимпиадад бэлтгэгчдэд тусламж (Тэнцэтгэл биш 1)

Second solution: We use the same substitution and reduce the problem tofinding the minimum value of x+ y + z when 2xyz ≥ 2x+ 4y + 7z. Applyingthe weighted AM-GM Inequality, we find that

x+ y + z ≥(

5x

2

) 25

(3y)13

(15z

4

) 415

.

And also 2x + 4y + 7z ≥ 1015 · 12

13 · 5

715 · x

15 · y

13 · z

715 . This means that

(x + y + z)2(2x + 4y + 7z) ≥ 225

2xyz. Because 2xyz ≥ 2x + 4y + 7z, we will

have

(x+ y + z)2 ≥ 225

4, or x+ y + z ≥ 15

2,

with equality for x = 3, y =5

2, z = 2.

?F?

02.1. Let x, y, z be positive real numbers such that

1

x+

1

y+

1

z= 1.

Prove that

√x+ yz +

√y + zx+

√z + xy ≥ √xyz +

√x+√y +√z.

(APMO 2002)

Solution: Let a =1

x, b =

1

y, c =

1

z. Then from the given hypothesis, we have

a+ b+ c = 1, and the original inequality becomes√1

a+

1

bc+

√1

b+

1

ca+

√1

c+

1

ab≥ 1√

abc+

1√a

+1√b

+1√c,

which is equivalent to

√a+ bc+

√b+ ca+

√c+ ab ≥

√ab+

√bc+

√ca+ 1,

or ∑√a(a+ b+ c) + bc ≥

√ab+

√bc+

√ca+ a+ b+ c.

By the Cauchy Schwarz Inequality, we have∑√a(a+ b+ c) + bc =

∑√(a+ b)(a+ c) ≥

∑(a+√bc)

=∑

a+∑√

ab.

Therefore, the last inequality is true and thus, our problem is solved. Notethat the equality holds if and only if x = y = z = 3.

?F?

02.2. Let a, b, c be positive real numbers. Prove that

a3

b2+b3

c2+c3

a2≥ a2

b+b2

c+c2

a.

337

Page 339: Олимпиадад бэлтгэгчдэд тусламж (Тэнцэтгэл биш 1)

(Balkan (Shortslit) 2002)

Solution: Using the AM-GM Inequality, we have

a3

b2+ a ≥ 2a2

b,

b3

c2+ b ≥ 2b2

c,

c3

a2+ c ≥ 2c2

a.

Adding up these three inequalities, we deduce that

a3

b2+b3

c2+c3

a2≥ 2

(a2

b+b2

c+c2

a

)− (a+ b+ c).

From this inequality, we see that it suffices to prove that

a2

b+b2

c+c2

a≥ a+ b+ c,

which is true since the AM-GM Inequality yields

a2

b+ b ≥ 2a,

b2

c+ c ≥ 2b,

c2

a+ a ≥ 2c.

The equality occurs if and only if a = b = c.?F?

02.3. If a, b, c are positive real numbers such that abc = 2, then

a3 + b3 + c3 ≥ a√b+ c+ b

√c+ a+ c

√a+ b.

(Balkan (Shortlist) 2002)

Solution: Because

2(a3 + b3 + c3)− a2(b+ c)− b2(c+ a)− c2(a+ b) =

=[a3 + b3 − ab(a+ b)

]+[b3 + c3 − bc(b+ c)

]+[c3 + a3 − ca(a+ b)

]= (a− b)2(a+ b) + (b− c)2(b+ c) + (c− a)2(c+ a) ≥ 0,

it suffices to prove that

a2(b+ c) + b2(c+ a) + c2(a+ b) ≥ 2(a√b+ c+ b

√c+ a+ c

√a+ b

).

On the other hand, the Cauchy Schwarz Inequality yields

a2(b+ c) + b2(c+ a) + c2(a+ b) ≥ 1

3

(a√b+ c+ b

√c+ a+ c

√a+ b

)2,

so it is enough to prove that

a√b+ c+ b

√c+ a+ c

√a+ b ≥ 6,

which is true since from the AM-GM Inequality, we have

a√b+ c+ b

√c+ a+ c

√a+ b ≥ 3

3

√abc√

(a+ b)(b+ c)(c+ a)

≥ 33

√abc√

8abc = 6.

338

Page 340: Олимпиадад бэлтгэгчдэд тусламж (Тэнцэтгэл биш 1)

The equality holds if and only if a = b = c = 3√

2.?F?

02.4. Let a, b, c be real numbers such that a2+b2+c2 = 1. Prove the inequality

a2

1 + 2bc+

b2

1 + 2ca+

c2

1 + 2ab≥ 3

5.

(Bosnia and Herzegovina 2002)

Solution: In view of the inequality 2xy ≤ x2 + y2 and the observation that1 + 2bc = a2 + (b+ c)2 > 0, etc., we see that

a2

1 + 2bc+

b2

1 + 2ca+

c2

1 + 2ab≥ a2

1 + b2 + c2+

b2

1 + c2 + a2+

c2

1 + a2 + b2

=a2

2− a2+

b2

2− b2+

c2

2− c2

= −3 + 2

(1

2− a2+

1

2− b2+

1

2− c2

).

On the other hand, according to the Cauchy Schwarz Inequality, one can findthat

1

2− a2+

1

2− b2+

1

2− c2≥ 9

6− a2 − b2 − c2=

9

5,

from which it follows

a2

1 + 2bc+

b2

1 + 2ca+

c2

1 + 2ab≥ −3 + 2 · 9

5=

3

5,

as desired. Note that the equality holds if and only if a = b = c = ± 1√3.

?F?

02.5. Show that for any positive real numbers a, b, c, we have

a3

bc+b3

ca+c3

ab≥ a+ b+ c.

(Canada 2002)

First solution: From the AM-GM Inequality, we get

a3

bc+ b+ c ≥ 3

3

√a3

bc· b · c = 3a.

Adding this to the two analogous inequalities, we get the desired result. Notethat the equality holds if and only if a = b = c.

Second solution: Multiplying both sides of the desired inequality by abc > 0,we can rewrite it as

a4 + b4 + c4 ≥ abc (a+ b+ c) .

339

Page 341: Олимпиадад бэлтгэгчдэд тусламж (Тэнцэтгэл биш 1)

Now, we apply the well-known inequality x2 + y2 + z2 ≥ xy + yz + zx with(x, y, z) = (a2, b2, c2) and (x, y, z) = (ab, bc, ca) to get

a4 + b4 + c4 ≥ a2b2 + b2c2 + c2a2, and a2b2 + b2c2 + c2a2 ≥ abc(a+ b+ c).

Combining these two inequalities, we get the desired result.?F?

02.6. Assume (P1, P2, . . . , Pn) (n ≥ 2) is an arbitrary permutation of (1, 2, . . . , n).Prove that

1

P1 + P2+

1

P2 + P3+ . . .+

1

Pn−1 + Pn>n− 1

n+ 2.

(China 2002)

Solution: Denote with A the left hand side of the desired inequality. By theCauchy Schwarz Inequality, we find that

A ≥ (n− 1)2

(P1 + P2) + (P2 + P3) + · · ·+ (Pn−1 + Pn)

=(n− 1)2

2(P1 + P2 + · · ·+ Pn)− P1 − Pn=

(n− 1)2

2(1 + 2 + · · ·+ n)− P1 − Pn

=(n− 1)2

n(n+ 1)− P1 − Pn≥ (n− 1)2

n(n+ 1)− 1− 2=

(n− 1)2

(n− 1)(n+ 2)− 1

>(n− 1)2

(n− 1)(n+ 2)=n− 1

n+ 2,

as desired.?F?

02.7. Let x, y be positive real numbers such that x+ y = 2. Prove that

x3y3(x3 + y3) ≤ 2.

(India 2002)

Solution: According to the AM-GM Inequality and the given hypothesis, wefind that

x3y3(x3 + y3) = 2x3y3(x2 − xy + y2)

= 2 · xy · xy · xy · (x2 − xy + y2)

≤ 2

(xy + xy + xy + x2 − xy + y2

4

)4

= 2

[(x+ y)2

4

]4= 2,

as desired. The equality holds if and only if x = y = 1.?F?

340

Page 342: Олимпиадад бэлтгэгчдэд тусламж (Тэнцэтгэл биш 1)

02.8. For any positive real numbers a, b, c, show that the following inequalityholds

a

b+b

c+c

a≥ c+ a

c+ b+a+ b

a+ c+b+ c

b+ a.

(India 2002)

First solution: Rewrite the inequality in the form(a

b− a+ c

b+ c+ 1

)+

(b

c− a+ b

a+ c+ 1

)+

(c

a− b+ c

b+ a+ 1

)≥ 3,

orb2 + ca

b(b+ c)+c2 + ab

c(c+ a)+

a2 + bc

a(a+ b)≥ 3.

From this, using the AM-GM Inequality, we find that this inequality is deducedfrom

(a2 + bc)(b2 + ca)(c2 + ab) ≥ abc(a+ b)(b+ c)(c+ a).

Since (a2 + bc)(b2 + ca)− ab(a+ c)(b+ c) = c(a+ b)(a− b)2, we get

(a2 + bc)(b2 + ca) ≥ ab(a+ c)(b+ c),

and similarly, (b2 + ca)(c2 + ab) ≥ bc(a + b)(c + a) and (a2 + bc)(c2 + ab) ≥ac(a+ b)(b+ c). Thus

(a2 + bc)2(b2 + ca)2(c2 + ab)2 ≥ a2b2c2(a+ b)2(b+ c)2(c+ a)2,

and so(a2 + bc)(b2 + ca)(c2 + ab) ≥ abc(a+ b)(b+ c)(c+ a).

It is easy to see that the equality holds if and only if a = b = c.

Second solution: Let us take x =a

b, y =

b

c, z =

c

a. Observe that

a+ c

b+ c=

1 + xy

1 + y= x+

1− x1 + y

.

Using similar relations, the problem reduces to proving that if xyz = 1, then

x− 1

y + 1+y − 1

z + 1+z − 1

x+ 1≥ 0.

By expanding, we find that it is equivalent to

(x2 − 1)(z + 1) + (y2 − 1)(x+ 1) + (z2 − 1)(y + 1) ≥ 0,

or(xy2 + yz2 + zx2) + (x2 + y2 + z2) ≥ (x+ y + z) + 3.

But this inequality is very easy. Indeed, using the AM-GM Inequality, we havexy2 + yz2 + zx2 ≥ 3 and so it remains to prove that x2 + y2 + z2 ≥ x+ y + z,which follows from the inequalities

x2 + y2 + z2 ≥ (x+ y + z)2

3≥ x+ y + z.

341

Page 343: Олимпиадад бэлтгэгчдэд тусламж (Тэнцэтгэл биш 1)

?F?

02.9. Let x1, x2, . . . , xn be positive real numbers. Prove that

x11 + x21

+x2

1 + x21 + x22+ · · ·+ xn

1 + x21 + x22 + · · ·+ x2n<√n.

(India 2002)

Solution: Applying the Cauchy Schwarz Inequality, we can see that the lefthand side of the original inequality is not greater than

√n

√x21

(1 + x21)2

+x22

(1 + x21 + x22)2

+ · · ·+ x2n(1 + x21 + x22 + · · ·+ x2n)2

.

Now, we havex21

(1 + x21)2≤ x21

1 + x21= 1− 1

1 + x21,

and for all 2 ≤ i ≤ n, we can assert that

x2i(1 + x21 + · · ·+ x2i )

2≤ x2i

(1 + x21 + · · ·+ x2i−1)(1 + x21 + · · ·+ x2i )

=1

1 + x21 + · · ·+ x2i−1− 1

1 + x21 + · · ·+ x2i.

Adding the preceding expressions, we obtain

n∑i=1

x2i(1 + x21 + · · ·+ x2i )

2≤ 1− 1

1 + x21 + · · ·+ x2n< 1.

From this, it follows that the left hand side of the original inequality is lessthan

√n. This is what we want to prove.

?F?

02.10. Let a, b, c, d be the positive real numbers such that

1

1 + a4+

1

1 + b4+

1

1 + c4+

1

1 + d4= 1.

Prove that abcd ≥ 3.(Latvia 2002)

First solution: We need to prove the inequality a4b4c4d4 ≥ 81. After makingthe substitution

A =1

1 + a4, B =

1

1 + b4, C =

1

1 + c4, D =

1

1 + d4,

we obtain

a4 =1−AA

, b4 =1−BB

, c4 =1− CC

, d4 =1−DD

.

342

Page 344: Олимпиадад бэлтгэгчдэд тусламж (Тэнцэтгэл биш 1)

The constraint becomes A+B+C+D = 1 and the inequality can be writtenas

1−AA· 1−B

B· 1− C

C· 1−D

D≥ 81,

orB + C +D

A· C +D +A

B· D +A+B

C· A+B + C

D≥ 81,

or

(B + C +D)(C +D +A)(D +A+B)(A+B + C) ≥ 81ABCD.

However, this is an immediate consequence of the AM-GM Inequality

(B + C +D)(C +D +A)(D +A+B)(A+B + C) ≥

≥ 3 (BCD)13 · 3 (CDA)

13 · 3 (DAB)

13 · 3 (ABC)

13 .

Note that the equality holds if and only if a = b = c = d = 4√

3.

Second solution: We can write a2 = tanA, b2 = tanB, c2 = tanC, d2 =

tanD, where A,B,C,D ∈(

0,π

2

). Then, the algebraic identity becomes the

following trigonometric identity

cos2A+ cos2B + cos2C + cos2D = 1.

Applying the AM-GM Inequality, we obtain

sin2A = 1− cos2A = cos2B + cos2C + cos2D ≥ 3 (cosB cosC cosD)23 .

Similarly, we obtain

sin2B ≥ 3 (cosC cosD cosA)23 ,

sin2C ≥ 3 (cosD cosA cosB)23 ,

sin2D ≥ 3 (cosA cosB cosC)23 .

Multiplying these four inequalities, we get the result.?F?

02.11. Prove that for any positive real numbers a, b, c, we have

a

2a+ b+

b

2b+ c+

c

2c+ a≤ 1.

(Moldova 2002)

Solution: The original inequality is equivalent to(1− 2a

2a+ b

)+

(1− 2b

2b+ c

)+

(1− 2c

2c+ a

)≥ 1,

orb

2a+ b+

c

2b+ c+

a

2c+ a≥ 1,

343

Page 345: Олимпиадад бэлтгэгчдэд тусламж (Тэнцэтгэл биш 1)

which is true since by Cauchy Schwarz Inequality, we have

b

2a+ b+

c

2b+ c+

a

2c+ a≥ (b+ c+ a)2

b (2a+ b) + c (2b+ c) + a (2c+ a)= 1.

It is easy to see that the equality holds if and only if a = b = c.?F?

02.12. Positive numbers α, β, x1, x2, . . . , xn (n ≥ 1) satisfy the conditionx1 + x2 + · · ·+ xn = 1. Prove that

x31αx1 + βx2

+x32

αx2 + βx3+ · · ·+ x3n

αxn + βx1≥ 1

n(α+ β).

(Moldova 2002)

First solution: By the AM-GM Inequality, we have

x3iαxi + βxi+1

+αxi + βxi+1

n(α+ β)2+

1

n2(α+ β)≥ 3xin(α+ β)

,

for all i = 1, 2, . . . , n. Therefore

n∑i=1

x3iαxi + βxi+1

+n∑i=1

αxi + βxi+1

n(α+ β)2+

1

n(α+ β)≥

n∑i=1

3xin(α+ β)

.

Sincen∑i=1

αxi + βxi+1

n(α+ β)2=

1

n(α+ β)and

n∑i=1

3xin(α+ β)

=3

n(α+ β), we thus

have thatn∑i=1

x3iαxi + βxi+1

+2

n(α+ β)≥ 3

n(α+ β),

which yieldsn∑i=1

x3iαxi + βxi+1

≥ 1

n(α+ β).

The equality holds if and only if xi =1

nfor all i = 1, 2, . . . , n.

Second solution: By the Holder’s Inequality, we have that

n

(n∑i=1

x3iαxi + βxi+1

)[n∑i=1

(αxi + βxi+1)

]≥

(n∑i=1

xi

)3

= 1,

and sincen∑i=1

(αxi + βxi+1) = α+ β, we conclude that

n∑i=1

x3iαxi + βxi+1

≥ 1

n(α+ β).

?F?

344

Page 346: Олимпиадад бэлтгэгчдэд тусламж (Тэнцэтгэл биш 1)

02.13. Let a, b, c be positive real numbers. Prove that(2a

b+ c

) 23

+

(2b

c+ a

) 23

+

(2c

a+ b

) 23

≥ 3.

(MOSP 2002)

Solution: By the AM-GM Inequality, we have that(2a

b+ c

) 23

=2a

3√

2a · (b+ c) · (b+ c)≥ 6a

2a+ (b+ c) + (b+ c)=

3a

a+ b+ c.

Similarly, we have(2b

c+ a

) 23

≥ 3b

a+ b+ c,

(2c

a+ b

) 23

≥ 3c

a+ b+ c.

Adding up these three inequalities, we get the result. It is easy to see that theequality holds if and only if a = b = c.

?F?

02.14. If a, b, c ∈ (0, 1), prove that

√abc+

√(1− a)(1− b)(1− c) < 1.

(Romania 2002)

First solution: Observe that√x < 3

√x for x ∈ (0, 1). Thus

√abc < 3

√abc

and√

(1− a)(1− b)(1− c) < 3√

(1− a)(1− b)(1− c). On the other hand, theAM-GM Inequality implies

3√abc ≤ a+ b+ c

3, and 3

√(1− a)(1− b)(1− c) ≤ (1− a) + (1− b) + (1− c)

3.

Therefore

√abc+

√(1− a)(1− b)(1− c) < a+ b+ c

3+

(1− a) + (1− b) + (1− c)3

= 1,

as desired.

Second solution: We have

√abc+

√(1− a)(1− b)(1− c) <

√b ·√c+√

1− b ·√

1− c < 1,

by the Cauchy Schwarz Inequality.

Third solution: Let a = sin2 x, b = sin2 y, c = sin2 z, where x, y, z ∈(

0,π

2

).

The inequality becomes

sinx · sin y · sin z + cosx · cos y · cos z < 1,

345

Page 347: Олимпиадад бэлтгэгчдэд тусламж (Тэнцэтгэл биш 1)

and it follows from the inequalities

sinx ·sin y ·sin z+cosx ·cos y ·cos z < sinx ·sin y+cosx ·cos y < cos(x−y) ≤ 1.

?F?

02.15. Given positive real numbers a, b, c and x, y, z, for which a+x = b+y =c+ z = 1. Prove that

(abc+ xyz)

(1

ay+

1

bz+

1

cx

)≥ 3.

(Russia 2002)

First solution: Our inequality is equivalent to

[abc+ (1− a)(1− b)(1− c)][

1

a(1− b)+

1

b(1− c)+

1

c(1− a)

]≥ 3.

Since a + x = b + y = c + z = 1 and x, y, z are positive numbers, we have0 < a, b, c < 1. And thus, there exist positive numbers m,n, p such that

a =1

m+ 1, b =

1

n+ 1, c =

1

p+ 1. By this substitution, we have

abc+ (1− a)(1− b)(1− c) =mnp+ 1

(m+ 1)(n+ 1)(p+ 1),

and

1

a(1− b)+

1

b(1− c)+

1

c(1− a)=

(m+ 1)(n+ 1)

n+

(n+ 1)(p+ 1)

p+

(p+ 1)(m+ 1)

m.

Therefore, the above inequality can be written as

mnp+ 1

m(n+ 1)+mnp+ 1

n(p+ 1)+mnp+ 1

p(m+ 1)≥ 3.

By the AM-GM Inequality, we have

1 +mnp

m(1 + n)+

1 +mnp

n(1 + p)+

1 +mnp

p(1 +m)+ 3 =

=1 +m+mn+mnp

m(1 + n)+

1 + n+ np+mnp

n(1 + p)+

1 + p+ pm+mnp

p(1 +m)

=(1 +m) +mn(1 + p)

m(1 + n)+

(1 + n) + np(1 +m)

n(1 + p)+

(1 + p) + pm(1 + n)

p(1 +m)

=

[1 +m

m(1 + n)+

1 + n

n(1 + p)+

1 + p

p(1 +m)

]+

[n(1 + p)

1 + n+p(1 +m)

1 + p+m(1 + n)

1 +m

]≥ 3

3√mnp

+ 3 3√mnp ≥ 6.

From this, we deduce that

mnp+ 1

m(n+ 1)+mnp+ 1

n(p+ 1)+mnp+ 1

p(m+ 1)≥ 3,

346

Page 348: Олимпиадад бэлтгэгчдэд тусламж (Тэнцэтгэл биш 1)

as desired. It is easy to see that the equality holds if and only if m = n = p = 1,

i.e. if and only if a = b = c = x = y = z =1

2.

Second solution: Proceeding similar as above, we see that it suffices to provethat the inequality

(1 + abc)

[1

a(1 + b)+

1

b(1 + c)+

1

c(1 + a)

]≥ 3

holds for any positive real numbers a, b, c. Note that in general we have abc 6= 1,

so we cannot apply the substitutions a =v

u, b =

w

v, c =

u

wwith positive

reals u, v, w. However, we can substitute a = k · vu, b = k · w

v, c = k · u

w,

where k, u, v, w are positive real numbers. Then abc = k3, and our inequalitysimplifies to(

1 + k3) [ u

k (v + kw)+

v

k (w + ku)+

w

k (u+ kv)

]≥ 3.

This can be rewritten as

1 + k3

k·(

u

v + kw+

v

w + ku+

w

u+ kv

)≥ 3.

Now, using the Cauchy Schwarz Inequality and the well-known (u+v+w)2 ≥3(uv + vw + wu), we have

u

v + kw+

v

w + ku+

w

u+ kv≥ (u+ v + v)2

(k + 1)(uv + vw + wu)≥ 3

k + 1.

So, it is sufficient to prove thatk3 + 1

k· 3

k + 1≥ 3, which is true.

Third solution: We will give another proof for the inequality

(1 + abc)

[1

a(1 + b)+

1

b(1 + c)+

1

c(1 + a)

]≥ 3.

We have∑ 1 + abc

a(1 + b)− 3 =

∑ 1− a− ab+ abc

a(1 + b)=∑ (1− ab)− a(1− bc)

a(1 + b)

=∑ 1− ab

a(1 + b)−∑ 1− bc

1 + b=∑ 1− ab

a(1 + b)−∑ 1− ab

1 + a

=∑

(1− ab)[

1

a(1 + b)− 1

1 + a

]=∑ (1− ab)2

a(1 + a)(1 + b).

The last quantity is obviously nonnegative, so we must have

(1 + abc)

[1

a(1 + b)+

1

b(1 + c)+

1

c(1 + a)

]≥ 3,

as desired.

347

Page 349: Олимпиадад бэлтгэгчдэд тусламж (Тэнцэтгэл биш 1)

Fourth solution: We present another approach to the inequality

(1 + abc)

[1

a(1 + b)+

1

b(1 + c)+

1

c(1 + a)

]≥ 3.

Multiplying each side of this inequality by(1 + a)(1 + b)(1 + c)

1 + abc> 0, we can

rewrite it as ∑ (1 + a)(1 + c)

a≥ 3(1 + a)(1 + b)(1 + c)

1 + abc,

or ∑ 1

a+∑

a+∑ a

b+ 3 ≥ 3(1 + a)(1 + b)(1 + c)

1 + abc.

Because (1 + a)(1 + b)(1 + c) =∑

a+∑

ab+ abc+ 1, it is equivalent to

∑a+

∑ 1

a+∑ a

b≥

3∑

a+ 3∑

ab

1 + abc,

or

abc∑

a+∑ 1

a+∑

a2c+∑ a

b≥ 2

∑a+ 2

∑ab.

But this follows from the inequalities

a2bc+b

c≥ 2ab, b2ca+

c

a≥ 2bc, c2ab+

a

b≥ 2ca,

and

a2c+1

c≥ 2a, b2a+

1

a≥ 2b, c2b+

1

b≥ 2c.

Fifth solution: In the same manner with all above solutions, we shall provethat

(abc+ 1)

[1

a(1 + b)+

1

b(1 + c)+

1

c(1 + a)

]≥ 3

for positive real numbers a, b, c. Squaring both sides and using the well-knowninequality (x+ y + z)2 ≥ 3(xy + yz + zx), we see that it suffices to prove

(abc+ 1)2 · 3∑ 1

ab(1 + b)(1 + c)≥ 9,

or equivalently,

(abc+ 1)2 ·

∑a+

∑ab

abc(1 + a)(1 + b)(1 + c)≥ 3.

Now, using the AM-GM Inequality, we have∑a+

∑ab

(1 + a)(1 + b)(1 + c)=

1

1 + abc∑a+

∑ab

+ 1

≥ 1

1 + abc

3 3√abc+ 3

3√a2b2c2

+ 1

=3 3√abc+ 3

3√a2b2c2(

3√abc+ 1

)3 =3 3√abc(

3√abc+ 1

)2 .348

Page 350: Олимпиадад бэлтгэгчдэд тусламж (Тэнцэтгэл биш 1)

So, it is enough to prove that

(abc+ 1)2 · 3 3√abc

abc(

3√abc+ 1

)2 ≥ 3,

which is not hard to prove.?F?

02.16. Let x, y, z be positive real numbers with sum 3. Prove that

√x+√y +√z ≥ xy + yz + zx.

(Russia 2002)

First solution: By applying the Holder’s Inequality, we have(√x+√y +√z)2 (

x2 + y2 + z2)≥ (x+ y + z)3 = 27,

and hence, it suffices to prove that(x2 + y2 + z2

)(xy + yz + zx)2 ≤ 27.

This is true since by the AM-GM Inequality, we have

(x2 + y2 + z2

)(xy + yz + zx)2 ≤

[x2 + y2 + z2 + 2(xy + yz + zx)

3

]3=

[(x+ y + z)2

3

]3= 27.

Note that the equality holds if and only if x = y = z = 1.

Second solution: Rewrite the inequality in the form

x2 + 2√x+ y2 + 2

√y + z2 + 2

√z ≥ x2 + y2 + z2 + 2(xy + yz + zx).

Since x2 + y2 + z2 + 2(xy + yz + zx) = (x+ y + z)2 = 9, it is equivalent to

x2 + 2√x+ y2 + 2

√y + z2 + 2

√z ≥ 9.

Now, from the AM-GM Inequality, we have

x2 + 2√x = x2 +

√x+√x ≥ 3

3

√x2 ·√x ·√x = 3x.

Therefore

x2 + 2√x+ y2 + 2

√y + z2 + 2

√z ≥ 3(x+ y + z) = 9,

as desired.?F?

349

Page 351: Олимпиадад бэлтгэгчдэд тусламж (Тэнцэтгэл биш 1)

02.17. Let a1, a2, . . . , an and b1, b2, . . . , bn be real numbers between 1001 and2002 inclusive. Suppose a21 + · · ·+ a2n = b21 + · · ·+ b2n. Prove that

n∑i=1

a3ibi≤ 17

10

n∑i=1

a2i .

Determine when equality holds.(Singapore 2002)

Solution: For each i, we have1

2=

1001

2002≤ ai

bi≤ 2002

1001= 2, and therefore

(2ai − bi)(2bi − ai) ≥ 0, that is

5aibi ≥ 2a2i + 2b2i . (1)

Multiplying this inequality byaibi> 0, we get

5a2i ≥ 2a3ibi

+ 2aibi. (2)

From (1), we have 2aibi ≥4

5(a2i + b2i ). Using this inequality in (2), we obtain

5a2i ≥ 2a3ibi

+4

5(a2i + b2i ),

which may be rewritten as

a3ibi≤ 21

10a2i −

2

5b2i . (3)

Note that the equality in (3) holds if and only if bi = 2ai or ai = 2bi, that is,if and ony if (ai, bi) = (1001, 2002) or (ai, bi) = (2002, 1001). Now, summing

over i in (3) and recallingn∑i=1

b2i =n∑i=1

a2i , we get

n∑i=1

a3ibi≤ 21

10

n∑i=1

a2i −2

5

n∑i=1

b2i =17

10

n∑i=1

a2i ,

as desired. The equality holds if and only if, for each i, either (ai, bi) =

(1001, 2002) or (ai, bi) = (2002, 1001). The condition

n∑i=1

b2i =

n∑i=1

a2i can be

rewritten as 10012p+ (n− p)20022 = 20022p+ (n− p)10012, which is p =n

2.

Thus, the equality holds if and only if n is even and (ai, bi) = (1001, 2002) forhalf of the subscripts i while (ai, bi) = (2002, 1001) for the other half.

?F?

02.18. Let a, b, c, d be real numbers contained in the interval

(0,

1

2

). Prove

that

a4 + b4 + c4 + d4

abcd≥ (1− a)4 + (1− b)4 + (1− c)4 + (1− d)4

(1− a)(1− b)(1− c)(1− d).

350

Page 352: Олимпиадад бэлтгэгчдэд тусламж (Тэнцэтгэл биш 1)

(Taiwan 2002)

First solution: Without loss of generality, we may assume that a ≥ b ≥ c ≥d. Then, by noting at the identity x4 + y4 + z4 + t4 = (x2− y2)2 + (z2− t2)2 +2(x2y2 + z2t2), we can see that the original inequality follows from adding thefollowing three inequalities

(a2 − b2)2

abcd≥ (a− b)2(2− a− b)2

(1− a)(1− b)(1− c)(1− d),

(c2 − d2)2

abcd≥ (c− d)2(2− c− d)2

(1− a)(1− b)(1− c)(1− d),

and2(a2b2 + c2d2)

abcd≥ 2[(1− a)2(1− b)2 + (1− c)2(1− d)2]

(1− a)(1− b)(1− c)(1− d).

Hence, in order to prove it, it suffices to show that these three inequalitiesholds. Indeed, the first one is equivalent to

(1− c)(1− d)

cd

(1

a− 1

)(1

b− 1

)≥(

2

a+ b− 1

)2

,

and since(1− c)(1− d)

cd≥ 1, it suffices to prove that

(1

a− 1

)(1

b− 1

)≥(

2

a+ b− 1

)2

.

This is true, because by the AM-GM Inequality, we have(1

a− 1

)(1

b− 1

)=

1− a− bab

+ 1 ≥ 4(1− a− b)(a+ b)2

+ 1 =

(2

a+ b− 1

)2

.

Since the proof for

(c2 − d2)2

abcd≥ (c− d)2(2− c− d)2

(1− a)(1− b)(1− c)(1− d)

is similar to the previous one, we are left to show that

a2b2 + c2d2

abcd≥ (1− a)2(1− b)2 + (1− c)2(1− d)2

(1− a)(1− b)(1− c)(1− d),

or equivalently,

f

(ab

cd

)≥ f

((1− c)(1− d)

(1− a)(1− b)

),

where f is the real-valued function such that f(x) = x+1

x. Now, since f(x)

is monotonically increasing for all x ≥ 1, and moreover since

ab

cd≥ 1,

(1− c)(1− d)

(1− a)(1− b)≥ 1,

351

Page 353: Олимпиадад бэлтгэгчдэд тусламж (Тэнцэтгэл биш 1)

we only need to show that

ab

cd≥ (1− c)(1− d)

(1− a)(1− b), or

a(1− a)

c(1− c)· b(1− b)d(1− d)

≥ 1.

But this is obviously true, because

a(1− a)

c(1− c)− 1 =

(a− c)(1− a− c)c(1− c)

≥ 0,

andb(1− b)d(1− d)

− 1 =(b− d)(1− b− d)

d(1− d)≥ 0.

Note that the equality holds iff a = b = c = d.

Second solution: First, we claim that for any x, y, z, t ∈(

0,1

2

), the in-

equality holds

(1− x)4 + (1− y)4

(1− x)(1− y)+

(x2 − y2)2[zt(1− x)(1− y)− xy(1− z)(1− t)]x2y2zt

≤ (1− x)(1− y)(x4 + y4)

x2y2.

Indeed, since z, t ∈(

0,1

2

), we have

zt(1− x)(1− y)− xy(1− z)(1− t)zt

= (1− x)(1− y)− xy · (1− z)(1− t)zt

< (1− x)(1− y)− xy = 1− x− y,

and thus, it suffices to prove that

(1− x)(1− y)(x4 + y4)

x2y2− (1− x)4 + (1− y)4

(1− x)(1− y)≥ (x2 − y2)2(1− x− y)

x2y2,

which is equivalent to (note that mn(u2+v2)−uv(m2+n2) = (um−vn)(un−vm))

(x− y)2(1− x− y)(x+ y − x2 − y2)(x+ y − 2xy)

x2y2(1− x)(1− y)≥ (x2 − y2)2(1− x− y)

x2y2.

This can be simplified into

(x+ y − x2 − y2)(x+ y − 2xy) ≥ (x+ y)2(1− x)(1− y).

Applying the Cauchy Schwarz Inequality, we have

(x+ y − x2 − y2)(x+ y − 2xy) = [x(1− x) + y(1− y)][x(1− y) + y(1− x)]

≥[x√

(1− x)(1− y) + y√

(1− y)(1− x)]2

= (x+ y)2(1− x)(1− y).

352

Page 354: Олимпиадад бэлтгэгчдэд тусламж (Тэнцэтгэл биш 1)

This proves our claim. Now, turning back to the main problem. Becausethe inequality is symmetric, we may assume without loss of generality thatd ≥ c ≥ b ≥ a. Then, applying the above claim by replacing (x, y, z, t) with(a, b, c, d) and (c, d, a, b), we get

(1− a)4 + (1− b)4

(1− a)(1− b)(1− c)(1− d)≤ (1− a)(1− b)(a4 + b4)

a2b2(1− c)(1− d)− M(a2 − b2)2

a2b2cd(1− c)(1− d),

(1− c)4 + (1− d)4

(1− a)(1− b)(1− c)(1− d)≤ (1− c)(1− d)(c4 + d4)

c2d2(1− a)(1− b)+

M(c2 − d2)2

abc2d2(1− a)(1− b),

where M = cd(1−a)(1−b)−ab(1−c)(1−d) ≥ 0. From these two inequalities,we can see that the original inequality is deduced from

cd(1− a)(1− b)(a4 + b4)

ab(1− c)(1− d)+ab(1− c)(1− d)(c4 + d4)

cd(1− a)(1− b)+

+M(c2 − d2)2

cd(1− a)(1− b)− M(a2 − b2)2

ab(1− c)(1− d)≤ a4 + b4 + c4 + d4.

Since

a4 + b4 − cd(1− a)(1− b)(a4 + b4)

ab(1− c)(1− d)= − M(a4 + b4)

ab(1− c)(1− d),

and

c4 + d4 − ab(1− c)(1− d)(c4 + d4)

cd(1− a)(1− b)=

M(c4 + d4)

cd(1− a)(1− b),

one can see that the last inequality is equivalent to

M(a4 + b4)

ab(1− c)(1− d)− M(a2 − b2)2

ab(1− c)(1− d)≤ M(c4 + d4)

cd(1− a)(1− b)− M(c2 − d2)2

cd(1− a)(1− b),

that is2abM

(1− c)(1− d)≤ 2cdM

(1− a)(1− b).

Of couse, this inequality is true because of M ≥ 0 and the fact x(1 − x) ≥y(1− y) for any

1

2> x ≥ y > 0. The proof is now completed.

?F?

02.19. Let x, y, z be positive real numbers such that x2 + y2 + z2 = 1. Provethat

x2yz + y2zx+ z2xy ≤ 1

3.

(United Kingdom 2002)

First solution: From the Cauchy Schwarz Inequality and the hypothesis, wehave

x2yz + y2zx+ z2xy = xyz(x+ y + z) ≤ xyz√

3(x2 + y2 + z2) =√

3xyz.

353

Page 355: Олимпиадад бэлтгэгчдэд тусламж (Тэнцэтгэл биш 1)

On the other hand,1

3=x2 + y2 + z2

3≥ 3√x2y2z2, using the AM-GM Inequal-

ity, hence xyz ≤ 1

3√

3. The desired inequality follows immediately by combin-

ing the two results. Note that the equality holds if and only if x = y = z =1√3.

Second solution: From the identity (a+b+c)2 = (a2+b2+c2)+2(ab+bc+ca),and the well-known inequality a2 + b2 + c2 ≥ ab+ bc+ ca, it follows that

(a+ b+ c)2 ≥ 3(ab+ bc+ ca).

Setting a = xy, b = yz, c = zx in this inequality yields

3(x2yz + y2zx+ z2xy)2 ≤ (xy + yz + zx)2 ≤ (x2 + y2 + z2) = 1,

from which we get

x2yz + y2zx+ z2xy ≤ 1

3,

as claimed.?F?

02.20. Let a, b, c be real numbers satisfying a2 + b2 + c2 = 9. Prove theinequality

2(a+ b+ c)− abc ≤ 10.

(Vietnam 2002)

First solution: If there is a negative number among a, b, c (for exampleb < 0). Then, from the AM-GM Inequality, we have

2(a+ b+ c)− abc ≤ 2(a+ b+ c)− b · a2 + c2

2

≤ a2 + 4

2+c2 + 4

2+ 2b− b · a

2 + c2

2

= 4 +9− b2

2+ 2b− b · 9− b2

2

= 10 +(b− 3)(b+ 1)2

2≤ 10.

Let us consider now the case a, b, c ≥ 0. Without loss of generality, we mayassume that a ≥ b ≥ c, then 3 ≥ a ≥

√3. There are two cases to consider

If ab ≥ 1, we have

2(a+ b+ c)− abc ≤ 2(a+ b+ c)− c = 2a+ 2b+ c

≤√

(22 + 22 + 12)(a2 + b2 + c2) = 9 < 10.

If ab < 1, then it follows that a2b2 + a2c2 ≤ 2a2b2 < 2, or b2 + c2 < 2a2. This

yields

2(a+ b+ c)− abc ≤ 2(a+ b+ c) ≤ 2a+ b2 + c2 + 2

< 2a+2

a2+ 2 < 2 · 3 +

2(√3)2 + 2 =

26

3< 10.

354

Page 356: Олимпиадад бэлтгэгчдэд тусламж (Тэнцэтгэл биш 1)

Therefore, in any cases, we always have

2(a+ b+ c)− abc ≤ 10.

The equality holds if and only if (a, b, c) is a permutation of (2, 2,−1).

Second solution: Without loss of generality, we may assume that a2 ≤ b2 ≤c2. And since a2 + b2 + c2 = 9, this assumption gives us c2 ≥ 3 and 2ab ≤ 6.From this, applying the Cauchy Schwarz Inequality, we have

[2(a+ b+ c)− abc]2 = [2(a+ b) + (2− ab)c]2 ≤ [4 + (2− ab)2][(a+ b)2 + c2]

= (8− 4ab+ a2b2)(9 + 2ab) = 2a3b3 + a2b2 − 20ab+ 72

= (ab+ 2)2(2ab− 7) + 100 ≤ 100.

According to this inequality, it follows that

2(a+ b+ c)− abc ≤ |2(a+ b+ c)− abc| ≤ 10,

as desired.

Third solution: Because max{a, b, c} ≤ 3 and |abc| ≤ 10, it is enough toconsider only the cases when a, b, c ≥ 0 or exactly one of the three numbers isnegative. First, we will suppose that a, b, c are nonnegative. If abc ≥ 1, thenwe are done because

2(a+ b+ c)− abc ≤ 2√

3(a2 + b2 + c2)− 1 < 10.

Otherwise, we may assume that a < 1. In this case, we have

2(a+ b+ c)− abc ≤ 2[a+

√2(b2 + c2)

]= 2a+ 2

√18− 2a2 ≤ 10.

Now, assume that not all three numbers are nonnegative and let c < 0. Thus,the problem reduces to proving that for any nonnegative x, y, z whose sum ofsquares is 9, we have 4(x+y−z)+2xyz ≤ 20. But we can write this inequalityas (x − 2)2 + (y − 2)2 + (z − 1)2 ≥ 2xyz − 6z − 2. Because 2xyz − 6z − 2 ≤z(x2 + y2) − 6z − 2 = −z3 + 3z − 2 = −(z − 1)2(z + 2) ≤ 0, the inequalityfollows.

Fourth solution: Of course, we have |a|, |b|, |c| ≤ 3 and |a+b+c|, |abc| ≤ 3√

3.Also, we may assume of course that a, b, c are non-zero and that a ≤ b ≤ c.If c < 0, then we have 2(a + b + c) − abc < −abc ≤ 3

√3 < 10. Also, if

a ≤ b < 0 < c, then we have 2(a + b + c) < 2c ≤ 6 < 10 + abc becauseabc > 0. If a < 0 < b ≤ c, using the Cauchy Schwarz Inequality, we find that2b+ 2c− a ≤ 9. Thus 2(a+ b+ c) = (2b+ 2c− a) + 3a ≤ 9 + 3a and it remainsto prove that 3a− 1 ≤ abc. But a < 0 and 2bc ≤ 9− a2, so that it remains to

show that9a− a3

2≥ 3a− 1 or (a+ 1)2(a− 2) ≤ 0, which follows. So, we just

have to threat the case 0 < a ≤ b ≤ c. In this case we have 2b + 2c + a ≤ 9and hence 2(a+ b+ c) ≤ 9 + a. So, we need to prove that a ≤ 1 + abc. This isclear if a < 1 and if a ≥ 1, we have b, c ≥ 1 and the inequality is again. Thus,the problem is solved.

355

Page 357: Олимпиадад бэлтгэгчдэд тусламж (Тэнцэтгэл биш 1)

?F?

03.1. If a, b, c > −1, then

1 + a2

1 + b+ c2+

1 + b2

1 + c+ a2+

1 + c2

1 + a+ b2≥ 2.

(Laurentiu Panaitopol, Balkan 2003)

Solution: We have 1 + b + c2 ≥ 1 + b > 0 and 1 + b + c2 ≤ 1 + b2

2+ 1 + c2,

hence1 + a2

1 + b+ c2≥ 2(1 + a2)

1 + b2 + 2(1 + c2).

Setting x = 1 + a2, y = 1 + b2, z = 1 + c2, it suffices to show that

x

y + 2z+

y

z + 2x+

z

x+ 2y≥ 1.

Using the Cauchy Schwarz Inequality, we have

x

y + 2z+

y

z + 2x+

z

x+ 2y≥ (x+ y + z)2

x(y + 2z) + y(z + 2x) + z(x+ 2y)

=(x+ y + z)2

3(xy + yz + zx)≥ 1.

Note that the equality holds if and only if a = b = c = 1.?F?

03.2. Prove that if a, b and c are positive real numbers with sum 3, then

a

b2 + 1+

b

c2 + 1+

c

a2 + 1≥ 3

2.

(Bulgaria 2003)

Solution: By the AM-GM Inequality, we have

a

b2 + 1= a− ab2

b2 + 1≥ a− ab2

2b= a− ab

2.

Adding this to the two analogous inequalities, we get

a

b2 + 1+

b

c2 + 1+

c

a2 + 1≥ a+b+c−ab+ bc+ ca

2≥ a+b+c− (a+ b+ c)2

6=

3

2,

as desired. Note that the equality holds if and only if a = b = c = 1.?F?

03.3. Let a, b, c, d be positive real numbers such that ab + cd = 1 and letx1, x2, x3, x4, y1, y2, y3, y4 be real numbers such that x21 + y21 = x22 + y22 =x23 + y23 = x24 + y24 = 1. Prove that the following inequality holds

(ay1 + by2 + cy3 + dy4)2 + (ax4 + bx3 + cx2 + dx1)

2 ≤ 2

(a2 + b2

ab+c2 + d2

cd

).

356

Page 358: Олимпиадад бэлтгэгчдэд тусламж (Тэнцэтгэл биш 1)

(China 2003)

First solution: Fix a number k > 0. Using the Cauchy Schwarz Inequality,we have

(ay1 + by2 + cy3 + dy4)2 ≤

(a2k + b2 + c2 + d2k

)(y21k

+ y22 + y23 +y24k

),

(ax4 + bx3 + cx2 + dx1)2 ≤

(a2k + b2 + c2 + d2k

)(x24k

+ x23 + x22 +x21k

),

and then, we deduce that

(ay1 + by2 + cy3 + dy4)2 + (ax4 + bx3 + cx2 + dx1)

2 ≤

≤ (a2k + b2 + c2 + d2k)

(x21 + y21

k+ x22 + y22 + x23 + y23 +

x24 + y24k

)= 2

(1

k+ 1

)[k(a2 + d2) + (b2 + c2)

]= 2(a2 + b2 + c2 + d2) + 2k(a2 + d2) +

2(b2 + c2)

k.

Now, we choose k > 0 such that

k(a2 + d2) +b2 + c2

k=cd(a2 + b2)

ab+ab(c2 + d2)

cd,

or

f(k) = (a2 + d2)k2 −[cd(a2 + b2)

ab+ab(c2 + d2)

cd

]k + b2 + c2 = 0.

We have

∆f =

[cd(a2 + b2)

ab+ab(c2 + d2)

cd

]2− 4(a2 + d2)(b2 + c2)

=

[a2d2(b2 + c2) + b2c2(a2 + d2)

]2a2b2c2d2

− 4(a2 + d2)(b2 + c2)

=

[a2d2(b2 + c2)− b2c2(a2 + d2)

]2a2b2c2d2

≥ 0.

Therefore, the number k > 0 satisfies the above equation is

k =

cd(a2 + b2)

ab+ab(c2 + d2)

cd+

∣∣a2d2(b2 + c2)− b2c2(a2 + d2)∣∣

abcd2(a2 + d2)

.

From this value of k and the above arguments, we have that

(ay1 + by2 + cy3 + dy4)2 + (ax4 + bx3 + cx2 + dx1)

2 ≤

≤ 2(a2 + b2 + c2 + d2) +2cd(a2 + b2)

ab+

2ab(c2 + d2)

cd

= 2(ab+ cd)

(a2 + b2

ab+c2 + d2

cd

)= 2

(a2 + b2

ab+c2 + d2

cd

),

357

Page 359: Олимпиадад бэлтгэгчдэд тусламж (Тэнцэтгэл биш 1)

as desired.

Second solution: Set u = ay1 + by2, v = cy3 + dy4, u1 = ax4 + bx3 andv1 = cx2 + dx1. Then

u2 ≤ (ay1 + by2)2 + (ax1 − bx2)2 = a2 + b2 + 2ab(y1y2 − x1x2),

that is

x1x2 − y1y2 ≤a2 + b2 − u2

2ab.

Similarly, we have

v21 ≤ (cx2 + dx1)2 + (dy1 − cy2)2 = c2 + d2 − 2cd(y1y2 − x1x2),

and then

y1y2 − x1x2 ≤c2 + d2 − v21

2cd.

From this, we deduce that

a2 + b2 − u2

2ab+c2 + d2 − v21

2cd≥ (x1x2 − y1y2) + (y1y2 − x1x2) = 0,

that isu2

ab+v21cd≤ a2 + b2

ab+c2 + d2

cd.

Similarly, we also have

v2

cd+u21ab≤ a2 + b2

ab+c2 + d2

cd.

Now, applying the Cauchy Schwarz Inequality, we get

(u+ v)2 + (u1 + v1)2 ≤ (ab+ cd)

(u2

ab+v2

cd

)+ (ab+ cd)

(u21ab

+v21cd

)=u2

ab+v21cd

+v2

cd+u21ab≤ 2

(a2 + b2

ab+c2 + d2

cd

),

as desired.

Third solution: By the Cauchy Schwarz Inequality, we get

(ay1 + by2 + cy3 + dy4)2 ≤ (ab+ cd)

[(ay1 + by2)

2

ab+

(cy3 + dy4)2

cd

]=

(ay1 + by2)2

ab+

(cy3 + dy4)2

cd

=a

by21 +

b

ay22 +

c

dy23 +

d

cy24 + 2y1y2 + 2y3y4,

and

(ax4 + bx3 + cx2 + dx1)2 ≤ (ab+ cd)

[(ax4 + bx3)

2

ab+

(cx2 + dx1)2

cd

]=

(ax4 + bx3)2

ab+

(cx2 + dx1)2

cd

=a

bx24 +

b

ax23 +

c

dx22 +

d

cx21 + 2x1x2 + 2x3x4.

358

Page 360: Олимпиадад бэлтгэгчдэд тусламж (Тэнцэтгэл биш 1)

Denote

P = (ay1+by2+cy3+dy4)2+(ax4+bx3+cx2+dx1)

2−2

(a2 + b2

ab+c2 + d2

cd

).

Then from the above inequalities, we find that

P ≤ a

by21 +

b

ay22 +

c

dy23 +

d

cy24 + 2y1y2 + 2y3y4 +

a

bx24 +

b

ax23 +

c

dx22+

+d

cx21 + 2x1x2 + 2x3x4 − 2

(a

b+b

a+c

d+d

c

)= −

(a

bx21 +

b

ax22

)−(c

dx23 +

d

cx24

)−(a

by24 +

b

ay23

)−(c

dy22 +

d

cy21

)+

+ 2x1x2 + 2x3x4 + 2y1y2 + 2y3y4

≤ −2x1x2 − 2x3x4 − 2y4y3 − 2y2y1 + 2x1x2 + 2x3x4 + 2y1y2 + 2y3y4 = 0,

as desired.?F?

03.4. Let a1, a2, . . . , a2n be real numbers such that

2n−1∑i=1

(ai − ai+1)2 = 1.

Determine the maximum value of

(an+1 + an+2 + · · ·+ a2n)− (a1 + a2 + . . .+ an).

(China 2003)

Solution: For n = 1, we have (a1 − a2)2 = 1, implying that a2 − a1 = ±1.Therefore, in this case, max {a2 − a1} = 1. Suppose that n ≥ 2, let x1 =a1, xi+1 = ai+1 − ai for i = 1, 2, . . . , 2n − 1. By this substitution, we haveai = x1 + · · ·+ xi for i = 1, 2, . . . , 2n and x22 + x23 + · · ·+ x22n = 1. Now, usingthe Cauchy Schwarz Inequality, we have

(an+1 + an+2 + · · ·+ a2n)− (a1 + a2 + · · ·+ an) =

= n(x1 + x2 + · · ·+ xn + xn+1) + (n− 1)xn+2 + · · ·+ x2n −n∑i=1

(n+ 1− i)xi

= x2 + 2x3 + · · ·+ (n− 1)xn + nxn+1 + (n− 1)xn+2 + · · ·+ x2n

≤√

[12 + 22 + · · ·+ (n− 1)2 + n2 + (n− 1)2 + · · ·+ 12](x22 + x23 + · · ·+ x22n

)=√

12 + 22 + · · ·+ (n− 1)2 + n2 + (n− 1)2 + · · ·+ 12 =

√n(2n2 + 1)

3.

The equality holds when

ai =

√3i(i− 1)

2√n(2n2 + 1)

, an+i =

√3[2n2 − (n− i)(n− i+ 1)

]2√n(2n2 + 1)

359

Page 361: Олимпиадад бэлтгэгчдэд тусламж (Тэнцэтгэл биш 1)

for all i = 1, 2, . . . , n. So, the maximum value of the desired expression is√n(2n2 + 1)

3.

?F?

03.5. Suppose x be a real number in the interval

[3

2, 5

]. Prove that

2√x+ 1 +

√2x− 3 +

√15− 3x < 2

√19.

(China 2003)

Solution: From the Cauchy Schwarz Inequality and the given hypothesis, wehave

2√x+ 1 +

√2x− 3 +

√15− 3x =

=√x+ 1 +

√x+ 1 +

√2x− 3 +

√15− 3x

≤√

[(x+ 1) + (x+ 1) + (2x− 3) + (15− 3x)] (12 + 12 + 12 + 12)

= 2√x+ 14 ≤ 2

√19.

We have equality if and only if√x+ 1 =

√2x− 3 =

√15− 3x and x = 5, but

this is impossible. Therefore, we conclude that

2√x+ 1 +

√2x− 3 +

√15− 3x < 2

√19,

as desired.?F?

03.6. Let x1, x2, . . . , x5 be nonnegative real numbers such that

5∑i=1

1

1 + xi= 1.

Prove that5∑i=1

xix2i + 4

≤ 1.

(China 2003)

First solution: The original inequality is equivalent to

5∑i=1

(1− 5xi

x2i + 4

)≥ 0,

or5∑i=1

(xi − 1)(xi − 4)

x2i + 4≥ 0.

This last inequality can be written as

5∑i=1

x2i − 1

x2i + 4· xi − 4

xi + 1≥ 0.

360

Page 362: Олимпиадад бэлтгэгчдэд тусламж (Тэнцэтгэл биш 1)

Now, due to symmetry, we may assume without loss of generality that x1 ≥x2 ≥ x3 ≥ x4 ≥ x5. By this assumption, it is easy to check that

x21 − 1

x22 + 4≥ x22 − 1

x22 + 4≥ x23 − 1

x23 + 4≥ x24 − 1

x24 + 4≥ x25 − 1

x25 + 4,

andx1 − 4

x1 + 1≥ x2 − 4

x2 + 1≥ x3 − 4

x3 + 1≥ x4 − 4

x4 + 1≥ x5 − 4

x5 + 1.

Therefore, by the Chebyshev’s Inequality, we have

5∑i=1

x2i − 1

x2i + 4· xi − 4

xi + 1≥ 1

5

(5∑i=1

x2i − 1

x2i + 4

)(5∑i=1

xi − 4

xi + 1

)

=

(5∑i=1

x2i − 1

x2i + 4

)(1−

5∑i=1

1

xi + 1

)= 0.

The equality holds if and only if x1 = x2 = x3 = x4 = x5 = 4.

Second solution: For each i (1 ≤ i ≤ 5), let yi =1

xi + 1, then xi =

1− yiyi

>

0 and y1 + y2 + y3 + y4 + y5 = 1. Replacing xi by1− yiyi

into the desired

inequality, we see that it is equivalent to

5∑i=1

yi − y2i5y2i − 2yi + 1

≤ 1, or5∑i=1

5yi − 5y2i5y2i − 2yi + 1

≤ 5,

which is true because

5∑i=1

5yi − 5y2i5y2i − 2yi + 1

=5∑i=1

(−1 +

3yi + 1

5y2i − 2yi + 1

)=

5∑i=1

3yi + 1

5y2i − 2yi + 1− 5

=

5∑i=1

3yi + 1

5

(yi −

1

5

)2

+4

5

− 5 ≤ 5

4

5∑i=1

(3yi + 1)− 5 = 5.

?F?

03.7. Find the greatest real number k such that, for any positive a, b, c witha2 > bc,

(a2 − bc)2 > k(b2 − ca)(c2 − ab).

(Japan 2003)

Solution: We will prove that the greatest k is 4. First suppose that (a2−bc)2 >k(b2−ca)(c2−ab) whenever a, b, c > 0 and a2 > bc. Let t ∈ (0, 1), Since 12 > t·t,we have

(1− t2)2 > k(t2 − t)(t2 − t),

from which we deduce that (1 + t

t

)2

> k.

361

Page 363: Олимпиадад бэлтгэгчдэд тусламж (Тэнцэтгэл биш 1)

It follows that

k ≤ limt→1

(1 + t

t

)2

= 4.

Now, we will show that (a2 − bc)2 > 4(b2 − ca)(c2 − ab) whenever a, b, c > 0and a2 > bc. Assume on the contrary that

(a2 − bc)2 ≤ 4(b2 − ca)(c2 − ab) (1)

for some positive a, b, c such that a2 > bc, and define

f(x) = (b2 − ca)x2 + (a2 − bc)x+ (c2 − ab).

From (1), either f(x) ≥ 0 for all real x or f(x) ≤ 0 for all real x. Actually, theformer holds since f(x) = a2 + b2 + c2 − ab− bc− ca > 0 (note that a = b = cis excluded by a2 > bc, and so a2 + b2 + c2 > ab + bc + ca). It follows thatb2 − ca is positive. Now, we write

f(x) = (bx− c)2 − ag(x)− x(a2 − bc),

where g(x) = cx2 − 2ax+ b. Since a2 − bc > 0 and

g(cb

)=c(c2 − ab) + b(b2 − ac)

b2> 0

(since c2 − ab has the same sign as b2 − ac by (1)), we have f(cb

)< 0, a

contradiction. This completes the proof.?F?

03.8. Prove that in any acute triangle ABC,

cot3A+ cot3B + cot3C + 6 cotA cotB cotC ≥ cotA+ cotB + cotC.

(MOSP 2003)

Solution: Let x = cotA, y = cotB, z = cotC. Because xy + yz + zx = 1, itsuffices to prove the homogeneous inequality

x3 + y3 + z3 + 6xyz ≥ (x+ y + z)(xy + yz + zx).

But it is equivalent to

x(x− y)(x− z) + y(y − z)(y − x) + z(z − x)(z − y) ≥ 0,

which is the Schur’s Inequality (in the special case third degree). Note that

the equality holds if and only if A = B = C =π

3.

?F?

03.9. Let ai be positive real numbers, for all i = 1, 2, . . . , n, satisfying

a1 + a2 + · · ·+ an =1

a1+

1

a2+ · · ·+ 1

an.

362

Page 364: Олимпиадад бэлтгэгчдэд тусламж (Тэнцэтгэл биш 1)

Prove that

1

n− 1 + a1+

1

n− 1 + a2+ · · ·+ 1

n− 1 + an≤ 1.

(Vasile Cirtoaje, MOSP 2003)

Solution: Notice that the inequality is equivalent to

n∑i=1

(1− n− 1

n− 1 + ai

)≥ 1, or

n∑i=1

ain− 1 + ai

≥ 1.

Set bi =1

ai, then the condition from the hypothesis remains invariant, but in

terms of bi’s rewrites as

b1 + b2 + · · ·+ bn =1

b1+

1

b2+ · · ·+ 1

bn.

On the other hand, the inequality to prove becomes now

n∑i=1

1

(n− 1)bi + 1≥ 1.

We proceed now by making use of the contradiction method. In this case,assume that there exist some numbers b1, b2, . . . , bn satisfying the above con-dition, but such that

n∑i=1

1

(n− 1)bi + 1< 1.

Then there also exists some k < 1 such that

n∑i=1

1

(n− 1)kbi + 1= 1.

Furthermore, setting xi =1

(n− 1)kbi + 1< 1, i.e. bi =

1− xi(n− 1)kxi

, we have

that x1 + x2 + · · ·+ xn = 1, and the given condition becomes

n∑i=1

1− xi(n− 1)kxi

=

n∑i=1

(n− 1)kxi1− xi

,

and since k < 1, we also have that

n∑i=1

1− xi(n− 1)xi

<n∑i=1

(n− 1)xi1− xi

.

On the other hand, from the Cauchy Schwarz Inequality, we get

n∑i=1

∑j 6=i

xj

xi=

n∑i=1

xi∑j 6=i

1

xj

≥ n∑i=1

xi · (n− 1)2∑j 6=i

xj

= (n− 1)2n∑i=1

xi∑j 6=i

xj,

363

Page 365: Олимпиадад бэлтгэгчдэд тусламж (Тэнцэтгэл биш 1)

and therefore

n∑i=1

∑j 6=i

xj

xi≥ (n−1)2

n∑i=1

xi∑j 6=i

xj, or equivalently,

n∑i=1

1− xi(n− 1)xi

≥n∑i=1

(n− 1)xi1− xi

.

This obviously comes in contradiction with

n∑i=1

1− xi(n− 1)xi

<

n∑i=1

(n− 1)xi1− xi

,

completing then our proof. Note that the equality holds if and only if a1 =a2 = · · · = an = 1.

?F?

03.10. Let a, b, c be nonnegative real numbers satisfying a2 + b2 + c2 = 1.Prove that

1 ≤ a

1 + bc+

b

1 + ca+

c

1 + ab≤√

2.

(Faruk Zejnulahi, MOSP 2003)

First solution: For all real numbers x contained in the interval

[0,

1

2

], we

have 1− 2

3x− 1

1 + x=x(1− 2x)

3(1 + x)≥ 0, and therefore

1

1 + x≤ 1− 2

3x.

Now, notice that max{ab, bc, ca} ≤ 1

2. In this case, we get

a

1 + bc+

b

1 + ca+

c

1 + ab≤ a

(1− 2

3bc

)+ b

(1− 2

3ca

)+ c

(1− 2

3ab

)= a+ b+ c− 2abc = a(1− 2bc) + b+ c

≤√

[a2 + (b+ c)2][(1− 2bc)2 + 1]

=√

2(2b2c2 − 2bc+ 1)(1 + 2bc)

=√

2[1− 2b2c2(1− 2bc)] ≤√

2.

For the right hand side of the inequality, note that1

1 + x−1 +x =

x2

1 + x≥ 0,

and therefore1

1 + x≥ 1− x,

for all positive real numbers x. Hence

a

1 + bc+

b

1 + ca+

c

1 + ab≥ a(1− bc) + b(1− ca) + c(1− ab)

= a+ b+ c− 3abc

=√a2 + b2 + c2 + 2(ab+ bc+ ca)− 3abc

=√

1 + 2(ab+ bc+ ca)− 3abc,

364

Page 366: Олимпиадад бэлтгэгчдэд тусламж (Тэнцэтгэл биш 1)

and since

√1 + 2u− 1 =

2u

1 +√

2u+ 1≥ 2u

1 +√

3=(√

3− 1)u

for any 0 ≤ u ≤ 1, we obtain√1 + 2(ab+ bc+ ca)− 3abc ≥ 1 +

(√3− 1

)(ab+ bc+ ca)− 3abc

≥ 1 +(√

3− 1)

(ab+ bc+ ca)3/2 − 3abc

≥ 1 +(√

3− 1)(

33√a2b2c2

)3/2− 3abc

= 1 + 3(

2−√

3)abc ≥ 1.

This yieldsa

1 + bc+

b

1 + ca+

1

1 + ab≥ 1,

and the proof is completed. Note that the equality in the left inequality occurs

iff (a, b, c) is a permutation of

(1√2,

1√2, 0

), while the equality in the right

inequality occurs iff (a, b, c) is a permutation of (1, 0, 0).

Second solution: First, we will show that

(a+ b+ c)2 ≤ 2(1 + bc)2,

or equivalently,2a(b+ c) ≤ 1 + 2bc+ 2b2c2.

This can be rewritten as

2a(b+ c) ≤ a2 + b2 + c2 + 2bc+ 2b2c2,

which is obviously true since a2 + b2 + c2 + 2bc− 2a(b+ c) = (b+ c− a)2 ≥ 0.Thus, we now have that

a

1 + bc+

b

1 + ca+

c

1 + ab≤

√2a

a+ b+ c+

√2b

a+ b+ c+

√2c

a+ b+ c=√

2,

and since the AM-GM Inequality gives us

a+ abc ≤ a+a(b2 + c2)

2= a+

a(1− a2)2

= 1− (a− 1)2(a+ 2)

2≤ 1

it follows thata

1 + bc− a2 =

a[1− (a+ abc)]

1 + bc≥ 0,

and therefore

a

1 + bc+

b

1 + ca+

c

1 + ab≥ a2 + b2 + c2 = 1.

?F?

365

Page 367: Олимпиадад бэлтгэгчдэд тусламж (Тэнцэтгэл биш 1)

03.11. Let a, b, c be positive real numbers so that abc = 1. Prove that

1 +3

a+ b+ c≥ 6

ab+ bc+ ca.

(Romania 2003)

Solution: We set x =1

a, y =

1

b, z =

1

cand observe that xyz = 1. The

inequality is equivalent to

1 +3

xy + yz + zx≥ 6

x+ y + z.

From (x+ y + z)2 ≥ 3(xy + yz + zx), we get

1 +3

xy + yz + zx≥ 1 +

9

(x+ y + z)2,

so it suffices to prove that

1 +9

(x+ y + z)2≥ 6

x+ y + z.

The last inequality is equivalent to

(1− 3

x+ y + z

)2

≥ 0 and this ends the

proof. Note that the equality holds if and only if a = b = c = 1.?F?

03.12. Let a, b, c be positive real numbers. Prove that

(2a+ b+ c)2

2a2 + (b+ c)2+

(2b+ c+ a)2

2b2 + (c+ a)2+

(2c+ a+ b)2

2c2 + (a+ b)2≤ 8.

(USA 2003)

First solution: Because the inequality is homogeneous, we can assume thata+ b+ c = 3. Then

(2a+ b+ c)2

2a2 + (b+ c)2=

a2 + 6a+ 9

3a2 − 6a+ 9=

1

3

[1 +

2(4a+ 3)

2 + (a− 1)2

]≤ 1

3

[1 +

2(4a+ 3)

2

]=

4a+ 4

3.

Thus ∑ (2a+ b+ c)2

2a2 + (b+ c)2≤ 1

3

∑(4a+ 4) = 8,

as desired. Note that the equality holds if and only if a = b = c.

Second solution: Observe that

3− (2a+ b+ c)2

2a2 + (b+ c)2=

2(b+ c− a)2

2a2 + (b+ c)2,

366

Page 368: Олимпиадад бэлтгэгчдэд тусламж (Тэнцэтгэл биш 1)

the desired inequality can be rewritten as

(b+ c− a)2

2a2 + (b+ c)2+

(c+ a− b)2

2b2 + (c+ a)2+

(a+ b− c)2

2c2 + (a+ b)2≥ 1

2.

By the Cauchy Schwarz Inequality, we find that[∑ (b+ c− a)2

2a2 + (b+ c)2

] [∑b2[2a2 + (b+ c)2

]]≥[∑

b(b+ c− a)]2

=(∑

a2)2.

Therefore, the above inequality is deduced from

2(∑

a2)2≥∑

b2[2a2 + (b+ c)2

].

By expanding, we see that it is equivalent to

a4 + b4 + c4 + a2b2 + b2c2 + c2a2 ≥ 2(a3b+ b3c+ c3a),

which is true since a4 + a2b2 ≥ 2a3b, b4 + b2c2 ≥ 2b3c, c4 + c2a2 ≥ 2c3a.

Third solution: Denote x =b+ c

a, y =

c+ a

b, z =

a+ b

c. We have to prove

that ∑ (x+ 2)2

x2 + 2≤ 8, or equivalently,

∑ (x− 1)2

x2 + 2≥ 1

2.

But, from the Cauchy Schwarz Inequality, we have∑ (x− 1)2

x2 + 2≥ (x+ y + z − 3)2

x2 + y2 + z2 + 6.

It remains to prove that

2(x2 + y2 + z2 + 2xy + 2yz + 2zx− 6x− 6y − 6z + 9) ≥ x2 + y2 + z2 + 6,

or(x+ y + z)2 + 2(xy + yz + zx)− 12(x+ y + z) + 12 ≥ 0.

Now xy + yz + zx ≥ 3 3√x2y2z2 ≥ 12 (because xyz ≥ 8), so we still have to

prove that(x+ y + z)2 + 24− 12(x+ y + z) + 12 ≥ 0,

which is equivalent to (x+ y + z − 6)2 ≥ 0, clearly true.?F?

03.13. Prove that for any a, b, c ∈(

0,π

2

), the following inequality holds

∑ sin a · sin(a− b) · sin(a− c)sin(b+ c)

≥ 0.

(USA 2003)

Solution: Let x = sin a, y = sin b, z = sin c. Then we have x, y, z > 0. It iseasy to see that the following relations are true

sin a · sin(a− b) · sin(a− c) · sin(a+ b) · sin(a+ c) = x(x2 − y2)(x2 − z2).

367

Page 369: Олимпиадад бэлтгэгчдэд тусламж (Тэнцэтгэл биш 1)

Using similar relations for the other terms, we have to prove that

x(x2 − y2)(x2 − z2) + y(y2 − z2)(y2 − x2) + z(z2 − x2)(z2 − y2) ≥ 0.

With the substitution x =√u, y =

√v, z =

√w, the inequality becomes

√u(u− v)(u− w) +

√v(v − w)(v − u) +

√w(w − u)(w − v) ≥ 0.

But this follows from the Schur’s Inequality and hence, our proof is completed.Note that the equality holds if and only if a = b = c.

?F?

04.1. Prove that

(a2 + 2)(b2 + 2)(c2 + 2) ≥ 9(ab+ bc+ ca)

for any positive real numbers a, b, c.

(APMO 2004)

First solution: We denote x = a2 + 2, y = b2 + 2, z = c2 + 2, and apply theAM-GM Inequality to obtain

ab ≤ 1

2

[a2(b2 + 2)

a2 + 2+b2(a2 + 2)

b2 + 2

]=

1

2

[(1− 2

a2 + 2

)(b2 + 2) +

(1− 2

b2 + 2

)(a2 + 2)

]=

1

2

[(1− 2

x

)y +

(1− 2

y

)x

]=

1

2

(x+ y − 2x

y− 2y

x

).

Proceeding the same way as this, we can get two other similar estimations forbc and ca. Adding up these three inequalities, one can see that

ab+ bc+ ca ≤ x+ y + z − x

y− y

x− y

z− z

y− z

x− x

z.

And thus, we can reduce the original inequality into proving that

xyz ≥ 9

(x+ y + z − x

y− y

x− y

z− z

y− z

x− x

z

),

or equivalently,

9x

y+

9x

z+

9y

z+

9y

x+

9z

x+

9z

y+ xyz ≥ 9x+ 9y + 9z.

The rest is simple as we have (by the AM-GM Inequality)

9x

y+

9x

z+xyz

3≥ 3 3

√9x

y· 9x

z· xyz

3= 9x.

Note that the equality holds if and only if a = b = c = 1.

368

Page 370: Олимпиадад бэлтгэгчдэд тусламж (Тэнцэтгэл биш 1)

Second solution: Choose A,B,C ∈(0, π2

)with a =

√2 tanA, b =

√2 tanB,

and c =√

2 tanC. Using the well-known trigonometric identity 1 + tan2 θ =1

cos2 θ, we may rewrite our inequality as

4

9≥ cosA cosB cosC (cosA sinB sinC + sinA cosB sinC + sinA sinB cosC) .

We can easily check the following trigonometric identity

cos(A+B + C) = cosA cosB cosC −∑

cosA sinB sinC.

Then, the above trigonometric inequality takes the form

4

9≥ cosA cosB cosC [cosA cosB cosC − cos(A+B + C)] .

Let θ =A+B + C

3. Applying the AM-GM Inequality and Jensen’s Inequal-

ity, we have

cosA cosB cosC ≤(

cosA+ cosB + cosC

3

)3

≤ cos3 θ.

We now need to show that

4

9≥ cos3 θ(cos3 θ − cos 3θ).

Using the trigonometric identity

cos 3θ = 4 cos3 θ − 3 cos θ or cos3 θ − cos 3θ = 3 cos θ − 3 cos3 θ,

it becomes4

27≥ cos4 θ

(1− cos2 θ

),

which follows from the AM-GM Inequality[cos2 θ

2· cos2 θ

2·(1− cos2 θ

)] 13

≤ 1

3

[cos2 θ

2+

cos2 θ

2+(1− cos2 θ

)]=

1

3.

Third solution: Observe that 9(ab + bc + ca) ≤ 3(a + b + c)2, and so, itsuffices to prove that

(a2 + 2)(b2 + 2)(c2 + 2) ≥ 3(a+ b+ c)2.

On the other hand, one has

[(a2−1)(b2−1)]·[(b2−1)(c2−1)]·[(c2−1)(a2−1)] = (a2−1)2(b2−1)2(c2−1)2 ≥ 0.

Therefore, at least one of three expressions (a2 − 1)(b2 − 1), (b2 − 1)(c2 − 1),(c2 − 1)(a2 − 1) must be nonnegative. And since the inequality is symmetric,

369

Page 371: Олимпиадад бэлтгэгчдэд тусламж (Тэнцэтгэл биш 1)

we can suppose that (a2 − 1)(b2 − 1) ≥ 0. Now, applying the Cauchy SchwarzInequality, we have

(a+ b+ c)2 = (a · 1 + b · 1 + 1 · c)2 ≤ (a2 + b2 + 1)(12 + 12 + c2).

According to this inequality, one can see that the above inequality is deducedfrom

(a2 + 2)(b2 + 2) ≥ 3(a2 + b2 + 1).

However, this is obviously true since it is equivalent to (a2 − 1)(b2 − 1) ≥ 0.?F?

04.2. Let x, y, z, t be positive real numbers such that xyzt = 1. Prove that

1

(1 + x)2+

1

(1 + y)2+

1

(1 + z)2+

1

(1 + t)2≥ 1.

(China 2004)

First solution: From x, y, z, t > 0 and xyzt = 1, we can find that exist

positive numbers a, b, c, d satisfying x =b

a, y =

c

b, z =

d

c, t =

a

d. And by this

substitution, the desired inequality becomes

a2

(a+ b)2+

b2

(b+ c)2+

c2

(c+ d)2+

d2

(d+ a)2≥ 1.

Now, applying the Cauchy Schwarz Inequality, we have[∑ a2

(a+ b)2

] [∑(a+ b)2(a+ d)2

]≥[∑

a(a+ b)]2.

According to this estimation, it suffices to prove that[∑a(a+ b)

]2≥∑

(a+ b)2(a+ d)2.

Since∑

a(a + b) =1

2[(a + b)2 + (b + c)2 + (c + d)2 + (d + a)2], we find that

the last inequality is equivalent to

(m+ n+ p+ w)2 ≥ 4(mn+ np+ pw + wm),

where m = (a + b)2, n = (b + c)2, p = (c + d)2, w = (d + a)2. However, thisinequality is clearly true (according to the AM-GM Inequality), so our proofis completed. Note that the equality holds if and only if x = y = z = t = 1.

Second solution: According to the Cauchy Schwarz Inequality, we have

1

(1 + x)2+

1

(1 + y)2≥ 1

(1 + xy)

(1 +

x

y

) +1

(1 + xy)(

1 +y

x

) =1

1 + xy.

And in the same way, we also have

1

(1 + z)2+

1

(1 + t)2≥ 1

1 + zt=

xy

1 + xy.

370

Page 372: Олимпиадад бэлтгэгчдэд тусламж (Тэнцэтгэл биш 1)

Adding up these two inequalities, the result follows immediately.?F?

04.3. If a, b, c are positive real numbers, then

1 <a√

a2 + b2+

b√b2 + c2

+c√

c2 + a2≤ 3√

2

2.

(China 2004)

First solution: By applying the Cauchy Schwarz Inequality, we find that

(∑ a√a2 + b2

)2

=

[∑√a2

(a2 + b2)(a2 + c2)· (a2 + c2)

]2

≤[∑ a2

(a2 + b2)(a2 + c2)

] [∑(a2 + c2)

]=

4(a2 + b2 + c2)(a2b2 + b2c2 + c2a2)

(a2 + b2)(b2 + c2)(c2 + a2).

On the other hand, the AM-GM Inequality implies

(a2 + b2 + c2)(a2b2 + b2c2 + c2a2) =

= (a2 + b2)(b2 + c2)(c2 + a2) + a2b2c2

≤ (a2 + b2)(b2 + c2)(c2 + a2) +(a2 + b2)(b2 + c2)(c2 + a2)

8

=9

8(a2 + b2)(b2 + c2)(c2 + a2).

Using this in combination with the above inequality, we get(∑ a√a2 + b2

)2

≤ 9

2,

from which it follows that

a√a2 + b2

+b√

b2 + c2+

c√c2 + a2

≤ 3√

2

2.

This is the right hand side of the desired inequality.

We still have to prove the left hand side. However, this is actual easy and wecan prove it as follows

a√a2 + b2

+b√

b2 + c2+

c√c2 + a2

>a

a+ b+

b

b+ c+

c

c+ a

>a

a+ b+ c+

b

b+ c+ a+

c

c+ a+ b= 1.

Note that the equality in the right hand side occurs iff a = b = c.

371

Page 373: Олимпиадад бэлтгэгчдэд тусламж (Тэнцэтгэл биш 1)

Second solution: We provide another solution for the right hand side in-

equality. With the notations x =b

a, y =

c

b, z =

a

c, the inequality reduces to

proving that xyz = 1 implies√2

1 + x2+

√2

1 + y2+

√2

1 + z2≤ 3.

We presume that x ≤ y ≤ z, which implies xy ≤ 1 and z ≥ 1. We have(√2

1 + x2+

√2

1 + y2

)2

≤ 2

(2

1 + x2+

2

1 + y2

)= 4

[1 +

1− x2y2

(1 + x2)(1 + y2)

]≤ 4

[1 +

1− x2y2

(1 + xy)2

]=

8

1 + xy=

8z

z + 1,

so √2

1 + x2+

√2

1 + y2≤ 2

√2z

z + 1,

and we need to prove that

2

√2z

z + 1+

√2

1 + z2≤ 3.

Because

√2

1 + z2≤ 2

z + 1, we only need to prove that

2

√2z

z + 1+

2

z + 1≤ 3.

This inequality is equivalent to

1 + 3z − 2√

2z(1 + z) ≥ 0, or(√

2z −√

1 + z)2≥ 0,

which is obvious and we are done.

Third solution: We give another way to prove the right hand side inequality.Clearly, the inequality asks to prove that if xyz = 1, then√

2

1 + x+

√2

1 + y+

√2

1 + z≤ 3.

We have two cases. The first and easy one is when xy + yz + zx ≥ x+ y + z.In this case, we can apply the Cauchy Schwarz Inequality to get

∑√2

1 + x≤√

3∑ 2

1 + x.

But∑ 2

1 + x≤ 3, since it is equivalent to

∑2(xy + x+ y + 1) ≤ 3(2 + x+ y + z + xy + yz + zx),

372

Page 374: Олимпиадад бэлтгэгчдэд тусламж (Тэнцэтгэл биш 1)

orx+ y + z ≤ xy + yz + zx,

and so in this case, the inequality is proved. The second case is when xy +yz + zx < x+ y + z. Thus,

(x− 1)(y − 1)(z − 1) = x+ y + z − (xy + yz + zx) > 0,

and so exactly two of the numbers x, y, z are smaller than 1, let them be xand y. So, we must prove that if x and y are smaller than 1, then√

2

x+ 1+

√2

y + 1+

√2xy

xy + 1≤ 3.

Using the Cauchy Schwarz Inequality, we get√2

x+ 1+

√2

y + 1+

√2xy

xy + 1≤ 2

√1

x+ 1+

1

y + 1+

√2xy

xy + 1,

and so it is enough to prove that this last inequality is at most 3. But thiscomes down to

2 ·

1

x+ 1+

1

y + 1− 1

1 +

√1

x+ 1+

1

y + 1

≤1− 2xy

xy + 1

1 +

√2xy

xy + 1

.

Because1

x+ 1+

1

y + 1≥ 1, the left hand side is at most

1

x+ 1+

1

y + 1− 1 =

1− xy(x+ 1)(y + 1)

,

and so we are left with the inequality

1− xy(x+ 1)(y + 1)

≤ 1− xy

(1 + xy)

(1 +

√2xy

xy + 1

) ,or equivalently,

xy + 1 + (xy + 1)

√2xy

xy + 1≤ xy + 1 + x+ y,

that is,x+ y ≥

√2xy(xy + 1).

This last one follows from√

2xy(xy + 1) ≤ 2√xy ≤ x + y, and thus, our

inequality is proved.?F?

04.4. Let n be a positive integer with n greater than one, and let a1, a2, . . . , anbe positive integers such that a1 < a2 < · · · < an and

1

a1+

1

a2+ · · ·+ 1

an≤ 1.

373

Page 375: Олимпиадад бэлтгэгчдэд тусламж (Тэнцэтгэл биш 1)

Prove that, for any real number x, the following inequality holds(1

a21 + x2+

1

a22 + x2+ · · ·+ 1

a2n + x2

)2

≤ 1

2· 1

a1(a1 − 1) + x2.

(China 2004)

Solution: Applying the Cauchy Schwarz Inequality, we have(n∑i=1

1

a2i + x2

)2

(n∑i=1

1

ai

)[n∑i=1

ai(a2i + x2)2

]

≤n∑i=1

ai(a2i + x2)2

≤n∑i=1

ai(a2i + x2)2 − a2i

=

n∑i=1

ai(a2i + x2 + ai)(a2i + x2 − ai)

=1

2

n∑i=1

(1

a2i − ai + x2− 1

a2i + ai + x2

).

Denoting an+1 = an + 1, we have that a1 < a2 < · · · < an < an+1 and becauseai is a positive integer, we can see that ai ≤ ai+1 − 1 for any i = 1, 2, . . . , n.By this argument, it follows that

1

a2i − ai + x2− 1

a2i + ai + x2≤ 1

a2i − ai + x2− 1

(ai+1 − 1)2 + (ai+1 − 1) + x2

=1

a2i − ai + x2− 1

a2i+1 − ai+1 + x2

for any i = 1, 2, . . . , n. Therefore, from the above estimation, we get(n∑i=1

1

a2i + x2

)2

≤ 1

2

n∑i=1

(1

a2i − ai + x2− 1

a2i+1 − ai+1 + x2

)=

1

2

(1

a21 − a1 + x2− 1

a2n+1 − an+1 + x2

)<

1

2· 1

a21 − a1 + x2=

1

2· 1

a1(a1 − 1) + x2,

which is just the desired result.?F?

04.5. Find all real numbers k such that the following inequality

a3 + b3 + c3 + d3 + 1 ≥ k(a+ b+ c+ d)

holds for all real numbers a, b, c, d ≥ −1.

(China 2004)

374

Page 376: Олимпиадад бэлтгэгчдэд тусламж (Тэнцэтгэл биш 1)

Solution: If a = b = c = d = −1, then −3 ≥ k · (−4), and hence k ≥ 3

4. If

a = b = c = d =1

2, then 4 · 1

8+ 1 ≥ k · 2. Thus k ≤ 3

4, and so k =

3

4. Now, we

want to prove that the inequality

a3 + b3 + c3 + d3 + 1 ≥ 3

4(a+ b+ c+ d)

holds for any a, b, c, d ∈ [−1,∞). At first, we prove that 4x3 + 1 ≥ 3x forx ∈ [−1,∞). In fact, from (x+ 1)(2x− 1)2 ≥ 0, we have 4x3 + 1 ≥ 3x for allx ∈ [−1,∞). Therefore

4a3 + 1 ≥ 3a, 4b3 + 1 ≥ 3b, 4c3 + 1 ≥ 3c, 4c3 + 1 ≥ 3d.

By adding theses four inequalities and dividing each side of the resulting in-equality by 4, we get

a3 + b3 + c3 + d3 + 1 ≥ 3

4(a+ b+ c+ d),

as claimed. Thus, the real number we want to find is k =3

4.

?F?

04.6. Determine the maximum constant λ such that

x+ y + z ≥ λ,

where x, y, z are positive real numbers with x√yz + y

√zx+ z

√xy ≥ 1.

(China 2004)

Solution: The maximum value of λ is√

3. It is not difficult to see that for

x = y = z =

√3

3, x√yz + y

√zx + z

√xy = 1 and x + y + z =

√3. Hence,

the maximum value of λ is less than or equal to√

3. It suffices to show thatx+ y+ z ≥

√3. Indeed, applying the AM-GM Inequality in combination with

the well-known inequalities xy + yz + zx ≤ (x+ y + z)2

3, we have

1 ≤ x√yz + y√zx+ z

√xy ≤ x · y + z

2+ y · z + x

2+ z · x+ y

2

= xy + yz + zx ≤ (x+ y + z)2

3.

From this, we deduce that

x+ y + z ≥√

3,

as desired.?F?

04.7. Let a, b, c be positive real numbers. Determine the minimal value of thefollowing expression

a+ 3c

a+ 2b+ c+

4b

a+ b+ 2c− 8c

a+ b+ 3c.

375

Page 377: Олимпиадад бэлтгэгчдэд тусламж (Тэнцэтгэл биш 1)

(China 2004)

Solution: Setting x = a+ 2b+ c, y = a+ b+ 2c and z = a+ b+ 3c. It is easyto see that c = z − y, b = z + x − 2y, and a = 5y − x − 3z. Now, using theAM-GM Inequality, we have

a+ 3c

a+ 2b+ c+

4b

a+ b+ 2c− 8c

a+ b+ 3c=

=(5y − x− 3z) + 3(z − y)

x+

4(z + x− 2y)

y− 8(z − y)

z

= −17 + 2

(2x

y+y

x

)+ 4

(2y

z+z

y

)≥ 4√

2 + 8√

2− 17 = 12√

2− 17.

The equality holds if and only if2x

y=

y

xand

2y

z=

z

y, or 2x =

√2y = z.

Hence the equality holds if and only if a + b + 2c =√

2(a + 2b + c) anda+ b+3c = 2(a+2b+ c). Solving this system of equations for b and c in termsof a gives b =

(1 +√

2)a and c =

(4 + 3

√2)a. From now, we conclude that

the expressiona+ 3c

a+ 2b+ c+

4b

a+ b+ 2c− 8c

a+ b+ 3c

has minimum value 12√

2−17 if and only if (a, b, c) =(a,(1 +√

2)a,(4 + 3

√2)a).

?F?

04.8. Determine the largest constant M such that the following inequalityholds for all real numbers x, y, z,

x4 + y4 + z4 + xyz (x+ y + z) ≥M (xy + yz + zx)2 .

(Hellenic 2004)

Solution: We will show that2

3is the largest value of M. Indeed, take x =

y = z = 1, we get M ≤ 2

3. It suffices to prove that

x4 + y4 + z4 + xyz (x+ y + z) ≥ 2

3(xy + yz + zx)2 ,

oor equivalently,

3(x4 + y4 + z4

)≥ 2

(x2y2 + y2z2 + z2x2

)+ xyz (x+ y + z) .

Applying the well-known inequalities a2 + b2 + c2 ≥ ab + bc + ca when thetriple (a, b, c) equals (x2, y2, z2) and (xy, yz, zx), we get

3(x4 + y4 + z4) ≥ 3(x2y2 + y2z2 + z2x2),

andx2y2 + y2z2 + z2z2 ≥ xyz(x+ y + z).

376

Page 378: Олимпиадад бэлтгэгчдэд тусламж (Тэнцэтгэл биш 1)

Adding up these two inequalities, we get the result.?F?

04.9. Let n ≥ 3 be an integer and let t1, t2, . . . , tn be positive real numberssuch that

n2 + 1 > (t1 + t2 + · · ·+ tn)

(1

t1+

1

t2+ · · ·+ 1

tn

).

Show that ti, tj , tk are the side lengths of a triangle for all i, j, k with 1 ≤ i <j < k ≤ n.

(IMO 2004)

Solution: By symmetry, it suffices to show that t1 < t2 + t3. If n > 3, apply-ing the Cauchy Schwarz Inequality, we have

n2 + 1 >

(n∑i=1

ti

)(n∑i=1

1

ti

)=

(t1 + t2 + t3 +

n∑i=4

ti

)(1

t1+

1

t2+

1

t3+

n∑i=4

1

ti

)

√(t1 + t2 + t3)

(1

t1+

1

t2+

1

t3

)+

√√√√( n∑i=4

ti

)(n∑i=4

1

ti

)2

[√(t1 + t2 + t3)

(1

t1+

1

t2+

1

t3

)+ n− 3

]2.

Therefore, it follows that√(t1 + t2 + t3)

(1

t1+

1

t2+

1

t3

)<√n2 + 1− n+ 3.

If n = 3, this inequality holds as well (according to the hypothesis). Since√n2 + 1 − n =

1

n+√n2 + 1

≤ 1

3 +√

10=√

10 − 3, the above inequality

implies

(t1 + t2 + t3)

(1

t1+

1

t2+

1

t3

)< 10.

In order to prove t1 < t2 + t3, it suffices to prove that if t1 ≥ t2 + t3, then thisinequality cannot hold. Indeed, if t1 ≥ t2 + t3, we have

(t1 + t2 + t3)

(1

t1+

1

t2+

1

t3

)= 1 + t1

(1

t2+

1

t3

)+t2 + t3t1

+ (t2 + t3)

(1

t2+

1

t3

)≥ 1 +

4t1t2 + t3

+t2 + t3t1

+ 4

= 5 +3t1

t2 + t3+

(t1

t2 + t3+t2 + t3t1

)≥ 5 + 3 + 2 = 10.

This completes our proof.

377

Page 379: Олимпиадад бэлтгэгчдэд тусламж (Тэнцэтгэл биш 1)

Remark: It is not hard to determine the greatest number f(n) such that, forpositive t1, . . . , tn, the inequality(

n∑i=1

ti

)(n∑i=1

1

ti

)< f(n)

implies that each three of the ti’s are the side lengths of a triangle. The answer

is f(n) =(n+√

10− 3)2

, and the proof is quite similar to the above solution.?F?

04.10. For a, b, c positive reals, find the minimum value of

b2 + c2

a2 + bc+c2 + a2

b2 + ca+a2 + b2

c2 + ab.

(India 2004)

Solution: We will show that

b2 + c2

a2 + bc+c2 + a2

b2 + ca+a2 + b2

c2 + ab≥ 3,

with equality iff a = b = c. Indeed, after using the AM-GM Inequality, itsuffices to prove that

(a2 + b2)(b2 + c2)(c2 + a2) ≥ (a2 + bc)(b2 + ca)(c2 + ab),

which is true since by the Cauchy Schwarz Inequality, we have

(a2 + b2)(b2 + c2)(c2 + a2) =

=√

(a2 + b2)(a2 + c2) ·√

(b2 + c2)(b2 + a2) ·√

(c2 + a2)(c2 + b2)

≥ (a2 + bc)(b2 + ca)(c2 + ab).

This completes our proof, and so, the searched minimum is 3, which is attainedfor a = b = c.

?F?

04.11. Let x1, x2, . . . , xn be real numbers in the interval

(0,

1

2

). Prove that

n∏i=1

xi(n∑i=1

xi

)n ≤n∏i=1

(1− xi)[n∑i=1

(1− xi)

]n .

(India 2004)

Solution: The desired inequality is equivalent to

n∑i=1

lnxi − n ln

(n∑i=1

xi

)≤

n∑i=1

ln(1− xi)− n ln

[n∑i=1

(1− xi)

],

378

Page 380: Олимпиадад бэлтгэгчдэд тусламж (Тэнцэтгэл биш 1)

orn∑i=1

[ln(1− xi)− lnxi] ≥ n ln

[n∑i=1

(1− xi)

]− n ln

(n∑i=1

xi

).

Consider the function f(x) = ln(1− x)− lnx with x ∈(

0,1

2

). We have

f ′(x) = − 1

1− x− 1

x, f ′′(x) =

1

x2− 1

(1− x)2=

1− 2x

x2(1− x)2> 0.

Therefore f(x) is (strictly) convex on

(0,

1

2

). And thus, we are allowed to

apply the Jensen’s Inequality and get

f(x1) + f(x2) + · · ·+ f(xn) ≥ nf(x1 + x2 + · · ·+ xn

n

),

which is equivalent to

n∑i=1

[ln(1− xi)− lnxi] ≥ n ln

[n∑i=1

(1− xi)

]− n ln

(n∑i=1

xi

).

Our proof is completed. Note that the equality holds if and only if x1 = x2 =· · · = xn.

?F?

04.12. Prove that for all positive real numbers a, b, the following inequalityholds √

2a(a+ b)3 + b√

2(a2 + b2) ≤ 3(a2 + b2).

(Ireland 2004)

First solution: By applying the AM-GM Inequality, we get√2a(a+ b)3 ≤ 2a(a+ b) + (a+ b)2

2, and b

√2(a2 + b2) ≤ 2b2 + a2 + b2

2.

Therefore√2a(a+ b)3 + b

√2(a2 + b2) ≤ 2a2 + 2b2 + 2ab ≤ 3(a2 + b2).

The equality holds if and only if a = b.

Second solution: Denoting P as the left hand side of the original inequality.According to the Cauchy Schwarz Inequality, we have that

P 2 =[√

2a(a+ b) · (a+ b) +√

2b2 ·√a2 + b2

]2≤[2a(a+ b) + 2b2

] [(a+ b)2 + a2 + b2

]= 4(a2 + ab+ b2)2,

and thus

P =√

2a(a+ b)3 + b√

2(a2 + b2) ≤ 2a2 + 2b2 + 2ab ≤ 3(a2 + b2).

379

Page 381: Олимпиадад бэлтгэгчдэд тусламж (Тэнцэтгэл биш 1)

?F?

04.13. If a, b, c are positive reals such that a+ b+ c = 1, prove that

1 + a

1− a+

1 + b

1− b+

1 + c

1− c≤ 2

(b

a+c

b+a

c

).

(Japan 2004)

First solution: By the Cauchy Schwarz Inequality, it is easy to see that

b

a+c

b+a

c=b2

ab+c2

bc+a2

ca≥ (a+ b+ c)2

ab+ bc+ ca.

So, the original inequality is deduced from

2(a+ b+ c)2

ab+ bc+ ca≥ 1 + a

1− a+

1 + b

1− b+

1 + c

1− c.

Noting that1 + a

1− a=

(a+ b+ c) + a

(a+ b+ c)− a=

2a

b+ c+ 1, the last inequality is equiv-

alent to2(a+ b+ c)2

ab+ bc+ ca≥ 2a

b+ c+

2b

c+ a+

2c

a+ b+ 3.

Multplying both sides for ab+ bc+ ca > 0 and using the fact that

2a(ab+ bc+ ca)

b+ c= 2a2 +

2abc

b+ c≤ 2a2 +

a(b+ c)

2,

we can see that the above inequality follows from

2(a+b+c)2 ≥ 2a2+a(b+ c)

2+2b2+

b(c+ a)

2+2c2+

c(a+ b)

2+3(ab+bc+ca).

which is trivial since it leads to an identity. It is easy to see that the equality

occurs if and only if a = b = c =1

3.

Second solution: Similar to the above solution, we see that1 + a

1− a=

2a

b+ c+

1, hence the desired inequality can be written as(2b

a− 2b

c+ a

)+

(2c

b− 2c

a+ b

)+

(2a

c− 2a

b+ c

)≥ 3,

orbc

a(c+ a)+

ca

b(a+ b)+

ab

c(b+ c)≥ 3

2.

Now, applying the Cauchy Schwarz Inequality, we have

bc

a(c+ a)+

ca

b(a+ b)+

ab

c(b+ c)=

b2c2

abc(c+ a)+

c2a2

abc(a+ b)+

a2b2

abc(b+ c)

≥ (ab+ bc+ ca)2

2abc(a+ b+ c).

380

Page 382: Олимпиадад бэлтгэгчдэд тусламж (Тэнцэтгэл биш 1)

Therefore, in order to prove the original inequality, it suffices to prove that

(ab+ bc+ ca)2

2abc(a+ b+ c)≥ 3

2,

which can be written as (ab + bc + ca)2 ≥ 3abc(a + b + c). We have alreadyproved this nice inequality in the previous problem.

?F?

04.14. Let a, b be real numbers in the interval [0, 1]. Prove that

a√2b2 + 5

+b√

2a2 + 5≤ 2√

7.

(Lithuania 2004)

Solution: For a = b = 0, the inequality is trivial. Let us consider now thecase a2 + b2 > 0. Using the Cauchy Schwarz Inequality in combination withthe given hypothesis, we have(

a√2b2 + 5

+b√

2a2 + 5

)2

≤ 2

(a2

2b2 + 5+

b2

2a2 + 5

)≤ 2

(a2

2b2 + 5a2+

b2

2a2 + 5b2

).

So it is enough to prove that

a2

2b2 + 5a2+

b2

2a2 + 5b2≤ 2

7.

Becausea2

2b2 + 5a2=

1

5− b2

5(2b2 + 5a2), the last inequality can be written as

a2

2a2 + 5b2+

b2

2b2 + 5a2≥ 2

7.

Now, we apply first the Cauchy Schwarz Inequality and then the AM-GMInequality to get

a2

2a2 + 5b2+

b2

2b2 + 5a2≥ (a2 + b2)2

a2(2a2 + 5b2) + b2(2b2 + 5a2)=

(a2 + b2)2

2(a2 + b2)2 + 6a2b2

≥ (a2 + b2)2

2(a2 + b2)2 + 6 · (a2 + b2)2

4

=2

7,

as desired. It is easy to see that the equality holds if and only if a = b = 1.?F?

04.15. Prove that for any positive real numbers a, b, c,∣∣∣∣a3 − b3a+ b+b3 − c3

b+ c+c3 − a3

c+ a

∣∣∣∣ ≤ (a− b)2 + (b− c)2 + (c− a)2

4.

381

Page 383: Олимпиадад бэлтгэгчдэд тусламж (Тэнцэтгэл биш 1)

(Moldova 2004)

Solution: With notice that∣∣∣∣a3 − b3a+ b+b3 − c3

b+ c+c3 − a3

c+ a

∣∣∣∣ =(ab+ bc+ ca) |(a− b)(b− c)(c− a)|

(a+ b)(b+ c)(c+ a),

one can write the original inequality as

(a+ b)(b+ c)(c+ a)

ab+ bc+ ca[(a− b)2 + (b− c)2 + (c− a)2] ≥ 4 |(a− b)(b− c)(c− a)| .

We denote with P the left hand side of this inequality, then by applying theAM-GM Inequality, we find that

P ≥ 3(a+ b)(b+ c)(c+ a)

ab+ bc+ ca3√

(a− b)2(b− c)2(c− a)2

≥3 3√

(a+ b)2(b+ c)2(c+ a)2

ab+ bc+ ca|(a− b)(b− c)(c− a)| .

Therefore, in order to prove the original inequality, it suffices to show that

(a+ b)2(b+ c)2(c+ a)2 ≥ 64

27(ab+ bc+ ca)3.

This is obviously true because

(a+ b)2(b+ c)2(c+ a)2 ≥ 64

81(a+ b+ c)2(ab+ bc+ ca)2 ≥ 64

27(ab+ bc+ ca)3.

Note that the inequality becomes equality if and only if a = b = c.

Second solution: It is easy to see that the inequality is not only cyclic, butsymmetric. That is why we may assume that a ≥ b ≥ c > 0. The idea is touse the inequality

x+y

2≥ x2 + xy + y2

x+ y≥ y +

x

2,

which is true if x ≥ y > 0. The proof of this inequality is easy and we won’tinsist. Now, because a ≥ b ≥ c > 0, we have the three inequalities

a+b

2≥ a2 + ab+ b2

a+ b≥ b+

a

2, b+

c

2≥ b2 + bc+ c2

b+ c≥ c+

b

2,

and of course

a+c

2≥ a2 + ac+ c2

a+ c≥ c+

a

2.

That is why we can write∑ a3 − b3

a+ b= (a− b)a

2 + ab+ b2

a+ b+ (b− c)b

2 + bc+ c2

b+ c− (a− c)a

2 + ac+ c2

a+ c

≥ (a− b)(b+

a

2

)+ (b− c)

(c+

b

2

)− (a− c)

(a+

c

2

)= −

∑(a− b)2

4.

382

Page 384: Олимпиадад бэлтгэгчдэд тусламж (Тэнцэтгэл биш 1)

In the same manner, we can prove that

∑ a3 − b3

a+ b≤

∑(a− b)2

4,

and the conclusion follows.?F?

04.16. Prove that if n > 3 and x1, x2, . . . , xn > 0 have product 1, then

1

1 + x1 + x1x2+

1

1 + x2 + x2x3+ · · ·+ 1

1 + xn + xnx1> 1.

(Russia 2004)

Solution: We use a similar form of the classical substitution x1 =a2a1, x2 =

a3a2, . . . , x3 =

a1an. In this case, the inequality becomes

a1a1 + a2 + a3

+a2

a2 + a3 + a4+ · · ·+ an

an + a1 + a2> 1,

and it is clear, because n > 3 and ai + ai+1 + ai+2 < a1 + a2 + · · ·+ an for alli.

?F?

04.17. (a) Given real numbers a, b, c with a+ b+ c = 0, prove that

a3 + b3 + c3 > 0 if and only if a5 + b5 + c5 > 0.

(b) Given real numbers a, b, c, d with a+ b+ c+ d = 0, prove that

a3 + b3 + c3 + d3 > 0 if and only if a5 + b5 + c5 + d5 > 0.

(United Kingdom 2004)

Solution: It is easy to show that

a3 + b3 + c3 − (a+ b+ c)3 = −3(a+ b)(b+ c)(c+ a),

and

a5 + b5 + c5 − (a+ b+ c)5 = −5

2(a+ b)(b+ c)(c+ a)

∑(a+ b)2.

We will use these identities to solve our problem.

(a) Using the above identities with noting that a+ b+ c = 0, we have

a3 + b3 + c3 = −3(a+ b)(b+ c)(c+ a) = −3(−c)(−a)(−b) = 3abc,

and

a5 + b5 + c5 = −5

2(a+ b)(b+ c)(c+ a)

∑(a+ b)2

= −5

2(−c)(−a)(−b)

∑(−c)2 =

5

2abc∑

a2.

383

Page 385: Олимпиадад бэлтгэгчдэд тусламж (Тэнцэтгэл биш 1)

It follows that

(a3 + b3 + c3)(a5 + b5 + c5) =15

2a2b2c2

∑a2.

The last quantity is obvious nonnegative, from which we deduce that a5 +b5 +c5 ≥ 0 if and only if a3 + b3 + c3 ≥ 0. Since we need to prove a5 + b5 + c5 > 0if and only if a3 + b3 + c3 > 0, we have to disprove the following two casesThe first case is when it exists some numbers a, b, c such that a5 + b5 + c5 > 0and a3 + b3 + c3 = 0. Since a3 + b3 + c3 = 0 and a3 + b3 + c3 = 3abc, we get

abc = 0, and hence a5 + b5 + c5 =5

2abc∑

a2 = 0, a contradiction.

The second case is when it exists some numbers a, b, c such that a5+b5+c5 = 0

and a3 + b3 + c3 > 0. Since a5 + b5 + c5 = 0 and a5 + b5 + c5 =5

2abc∑

a2, we

get abc = 0 or∑

a2 = 0. If abc = 0, then we have a3 + b3 + c3 = 3abc = 0,

contradiction. If a2 + b2 + c2 = 0, then we have a = b = c = 0 and hencea3 + b3 + c3 = 0, contradiction.From these arguments, we conclude that

a3 + b3 + c3 > 0 if and only if a5 + b5 + c5 > 0,

as desired.

(b) We use the same arguments to prove this claim. Indeed, we have

a3 + b3 + c3 + d3 = a3 + b3 + c3 − (a+ b+ c)3 = −3(a+ b)(b+ c)(c+ a),

and

a5 + b5 + c5 + d5 = a5 + b5 + c5 − (a+ b+ c)5

= −5

2(a+ b)(b+ c)(c+ a)[(a+ b)2 + (b+ c)2 + (c+ a)2].

It follows that

(a3 + b3 + c3 + d3)(a5 + b5 + c5 + d5) =15

2(a+ b)2(b+ c)2(c+ a)2

∑(a+ b)2.

The last quantity is obvious nonnegative, from which we deduce that a5 +b5 + c5 + d5 ≥ 0 if and only if a3 + b3 + c3 + d3 ≥ 0. Since we need to provea5 + b5 + c5 + d5 > 0 if and only if a3 + b3 + c3 + d3 > 0, we have to disprovethe following two casesThe first case is when it exists some numbers a, b, c, d such that a5 + b5 +c5 + d5 > 0 and a3 + b3 + c3 + d5 = 0. Since a3 + b3 + c3 + d3 = 0 anda3 + b3 + c3 +d3 = −3(a+ b)(b+ c)(c+a), we get (a+ b)(b+ c)(c+a) = 0, and

hence a5+b5+c5+d3 = −5

2(a+b)(b+c)(c+a)

∑(a+b)2 = 0, a contradiction.

The second case is when it exists some numbers a, b, c, d such that a5 + b5 +c5 + d5 = 0 and a3 + b3 + c3 + d3 > 0. Since a5 + b5 + c5 + d5 = 0 and

a5+b5+c5+d5 = −5

2(a+b)(b+c)(c+a)

∑(a+b)2, we get (a+b)(b+c)(c+a) = 0

or∑

(a+ b)2 = 0. If (a+ b)(b+ c)(c+a) = 0, then we have a3 + b3 + c3 +d3 =

384

Page 386: Олимпиадад бэлтгэгчдэд тусламж (Тэнцэтгэл биш 1)

−3(a+b)(b+c)(c+a) = 0, contradiction. If (a+b)2+(b+c)2+(c+a)2 = 0, thenwe have a = b = c = 0 and hence a3+b3+c3+d3 = −3(a+b)(b+c)(c+a) = 0,contradiction.From these arguments, we conclude that

a3 + b3 + c3 + d3 > 0 if and only if a5 + b5 + c5 + d5 > 0,

as desired.?F?

04.18. Let a, b, c be positive real numbers. Prove that

(a5 − a2 + 3)(b5 − b2 + 3)(c5 − c2 + 3) ≥ (a+ b+ c)3.

(USA 2004)

Solution: For any positive number x, the quantities x2 − 1 and x3 − 1 havethe same sign. Thus, we have 0 ≤ (x3 − 1)(x2 − 1) = x5 − x3 − x2 + 1, or

x5 − x2 + 2 ≥ x3 + 2.

It follows that

(a5 − a2 + 3)(b5 − b2 + 3)(c5 − c2 + 3) ≥ (a3 + 2)(b3 + 2)(c3 + 2),

and hence it suffices to prove that

(a3 + 2)(b3 + 2)(c3 + 2) ≥ (a+ b+ c)3.

Of course, this is true since by the Holder’s Inequality, we have

(a3 + 1 + 1)(1 + b3 + 1)(1 + 1 + c3) ≥(

3√a3 · 1 · 1 +

3√

1 · b3 · 1 +3√

1 · 1 · c3)3

= (a+ b+ c)3.

Note that the equality holds if and only if a = b = c = 1.?F?

04.19. Let x, y, z > 0 such that (x+ y+ z)3 = 32xyz. Find the minimum and

maximum ofx4 + y4 + z4

(x+ y + z)4.

(Vietnam 2004)

First solution: We may of course assume that x + y + z = 4 and xyz = 2.

Thus, we have to find the extremal values ofx4 + y4 + z4

44. Now, we have

x4 + y4 + z4 = (x2 + y2 + z2)2 − 2(x2y2 + y2z2 + z2x2)

=(

16− 2∑

xy)2− 2

(∑xy)2

+ 4xyz(x+ y + z)

= 2a2 − 64a+ 288,

385

Page 387: Олимпиадад бэлтгэгчдэд тусламж (Тэнцэтгэл биш 1)

where a = xy + yz + zx. Because y + z = 4 − x and yz =2

x, we must have

(4− x)2 ≥ 8

x, which implies that 3−

√5 ≤ x ≤ 2. Due to symmetry, we have

x, y, z ∈[3−√

5, 2]. This means that (x− 2)(y − 2)(z − 2) ≤ 0, and also(

x− 3 +√

5)(

y − 3 +√

5)(

z − 3 +√

5)≥ 0.

Clearing parenthesis, we deduce that

a ∈

[5,

5√

5− 1

2

].

But becausex4 + y4 + z4

44=

(a− 16)2 − 112

128, we find that the extremal values

of the expression are9

128,

383− 165√

5

256, attained for the triples (2, 1, 1) and(

3−√

5,1 +√

5

2,1 +√

5

2

), respectively.

Second solution: As in the above solution, we must find the extremal values

of x2 + y2 + z2 when x+ y+ z = 1, xyz =1

32, because after that the extremal

values of the expression x4 + y4 + z4 can be immediately found. Let us make

substitution x =a

4, y =

b

4, z =

c

2, where abc = 1, a + b + 2c = 4. Then

x2 + y2 + z2 =a2 + b2 + 4c2

16and so we must find the extremal values of

a2 + b2 +4c2. Now, a2 + b2 +4c2 = (4−2c)2− 2

c+4c2 and the problem reduces

to find the maximum and minimum of 4c2 − 8c − 1

cwhere there are positive

real numbers a, b, c such that abc = 1, a + b + 2c = 4. Of course, this comes

down to (4− 2c)2 ≥ 4

c, or to c ∈

[3−√

5

2, 1

]. But this reduces to the study

of the function f(x) = 4x2−8x− 1

xdefined for

[3−√

5

2, 1

], which is an easy

task.?F?

05.1. For any positive real numbers a, b, c such that abc = 8, then

a2√(a3 + 1) (b3 + 1)

+b2√

(b3 + 1) (c3 + 1)+

c2√(c3 + 1) (a3 + 1)

≥ 4

3.

(APMO 2005)

Solution: For any x > 0, the AM-GM Inequality implies

√x3 + 1 =

√(x+ 1) (x2 − x+ 1) ≤

(x+ 1) +(x2 − x+ 1

)2

=x2 + 2

2.

386

Page 388: Олимпиадад бэлтгэгчдэд тусламж (Тэнцэтгэл биш 1)

Applying this inequality, we get

a2√(a3 + 1) (b3 + 1)

+b2√

(b3 + 1) (c3 + 1)+

c2√(c3 + 1) (a3 + 1)

≥ 4

[a2

(a2 + 2) (b2 + 2)+

b2

(b2 + 2) (c2 + 2)+

c2

(c2 + 2) (a2 + 2)

].

Therefore, in order to prove the desired inequality, it suffices to show that

a2

(a2 + 2) (b2 + 2)+

b2

(b2 + 2) (c2 + 2)+

c2

(c2 + 2) (a2 + 2)≥ 1

3.

By some easy computations (with noting that abc = 8), we can write thisinequality as

a2b2 + b2c2 + c2a2 + 2(a2 + b2 + c2) ≥ 72.

This last inequality is obviously true since from the AM-GM Inequality, wehave

a2b2 + b2c2 + c2a2 ≥ 33√a4b4c4 = 48,

and

a2 + b2 + c2 ≥ 33√a2b2c2 = 12.

The equality holds if and only if a = b = c = 2.?F?

05.2. Let a, b, c, d be positive real numbers. Show that

1

a3+

1

b3+

1

c3+

1

d3≥ a+ b+ c+ d

abcd.

(Austria 2005)

Solution: Setting x =1

a, y =

1

b, z =

1

c, t =

1

d, then we can rewrite the

inequality as

x3 + y3 + z3 + t3 ≥ xyz + yzt+ ztx+ txy.

By the AM-GM Inequality, we have that

x3+y3+z3 ≥ 3xyz, y3+z3+t3 ≥ 3yzt, z3+t3+x3 ≥ 3ztx, t3+x3+y3 ≥ 3txy.

Adding up these four inequalities and dividing each side of the resulting in-equality by 3, we get the result. It is easy to see that the equality holds if andonly if a = b = c = d.

?F?

05.3. Let a, b, c be positive real numbers. Prove that

a2

b+b2

c+c2

a≥ a+ b+ c+

4(a− b)2

a+ b+ c.

(Balkan 2005)

387

Page 389: Олимпиадад бэлтгэгчдэд тусламж (Тэнцэтгэл биш 1)

Solution: The inequality in question is equivalent to(a2

b+ b− 2a

)+

(b2

c+ c− 2b

)+

(c2

a+ a− 2c

)≥ 4(a− b)2

a+ b+ c,

or(a− b)2

b+

(c− b)2

c+

(a− c)2

a≥ 4(a− b)2

a+ b+ c.

Now, applying the Cauchy Schwarz Inequality, we have

(a− b)2

b+

(c− b)2

c+

(a− c)2

a≥ [(a− b) + (c− b) + (a− c)]2

b+ c+ a=

4(a− b)2

a+ b+ c,

which is just the desired inequality. The equality holds if and only if a = b = c.?F?

05.4. If a, b, c are positive real numbers such that abc = 1, then the inequalityholds

a

a2 + 2+

b

b2 + 2+

c

c2 + 2≤ 1.

(Baltic Way 2005)

Solution: Let x =1

a, y =

1

b, z =

1

c, then xyz = 1. Now, applying the

AM-GM Inequality, we have

a

a2 + 2+

b

b2 + 2+

c

c2 + 2≤ a

2a+ 1+

b

2b+ 1+

c

2c+ 1

=1

2 + x+

1

2 + y+

1

2 + z.

Thus, it suffices to show that

1

2 + x+

1

2 + y+

1

2 + z≤ 1,

which is equivalent to

(2 + x) (2 + y) + (2 + y) (2 + z) + (2 + z) (2 + x) ≤ (2 + x) (2 + y) (2 + z) ,

or3 ≤ xy + yz + zx.

However, this is true since xy + yz + zx ≥ 3 3√x2y2z2 = 3 by the AM-GM

Inequality. Therefore, our proof is completed. Note that the equality holds ifand only if a = b = c = 1.

?F?

05.5. Let a, b, c be positive real numbers. Prove that(a2 + b+

3

4

)(b2 + a+

3

4

)≥(

2a+1

2

)(2b+

1

2

).

(Belarus 2005)

388

Page 390: Олимпиадад бэлтгэгчдэд тусламж (Тэнцэтгэл биш 1)

Solution: Applying the AM-GM Inequality, we get(a2 + b+

3

4

)(b2 + a+

3

4

)≥

(2

√a2 · 1

4+ b+

1

2

)(2

√b2 · 1

4+ a+

1

2

)= (a+ b)2 + a+ b+

1

4≥ 4ab+ a+ b+

1

4

=

(2a+

1

2

)(2b+

1

2

),

as desired. The equality holds if and only if a = b =1

2.

?F?

05.6. Given positive real numbers a, b, c such that a+ b+ c = 1. Prove that

a√b+ b

√c+ c

√a ≤ 1√

3.

(Bosnia and Hercegovina 2005)

Solution: Using the well-known inequality (a+ b+ c)2 ≥ 3(ab+ bc+ ca) and

the hypothesise a+ b+ c = 1, we deduce that ab+ bc+ ca ≤ 1

3. Thus, by the

Cauchy Schwarz Inequality, we get(a√b+ b

√c+ c

√a)2

=(√

a√ab+

√b√bc+

√c√ca)2

≤ (a+ b+ c)(ab+ bc+ ca) ≤ 1 · 1

3=

1

3.

This implies that

a√b+ b

√c+ c

√a ≤ 1√

3,

which is just the desired inequality. It is easy to see that the equality holds iff

a = b = c =1

3.

?F?

05.7. Let x, y be positive real numbers such that x3 + y3 = x− y. Prove that

x2 + 4y2 < 1.

(China 2005)

Solution: From the given hypothesis, we have

(x3 + y3)(1− x2 − 4y2) = x3 + y3 − (x3 + y3)(x2 + 4y2)

= x3 + y3 − (x− y)(x2 + 4y2)

= y(x2 − 4xy + 5y2) = y[(x− 2y)2 + y2

]> 0.

Since x3 + y3 > 0, this inequality implies that

x2 + 4y2 < 1,

389

Page 391: Олимпиадад бэлтгэгчдэд тусламж (Тэнцэтгэл биш 1)

as desired.?F?

05.8. Let a, b, c be positive real numbers such that ab + bc + ca =1

3. Prove

that1

a2 − bc+ 1+

1

b2 − ca+ 1+

1

c2 − ab+ 1≤ 3.

(China 2005)

Solution: From 3 (ab+ bc+ ca) = 1, we have

a2 − bc+ 1 = a2 + 3 (ab+ ac) + 2bc = a (a+ b+ c) + 2 (ab+ bc+ ca) ,

and so, we can rewrite our inequality as∑ 2 (ab+ bc+ ca)

a (a+ b+ c) + 2 (ab+ bc+ ca)≤ 2.

This is equivalent to∑[1− 2 (ab+ bc+ ca)

a (a+ b+ c) + 2 (ab+ bc+ ca)

]≥ 1,

or(a+ b+ c)

∑ a

a (a+ b+ c) + 2 (ab+ bc+ ca)≥ 1.

Now, denoting with P the left hand side of the above inequality and applyingthe Cauchy Schwarz Inequality, we have

P = (a+ b+ c)∑ a2

a2 (a+ b+ c) + 2a (ab+ bc+ ca)

≥ (a+ b+ c) · (a+ b+ c)2∑[a2 (a+ b+ c) + 2a (ab+ bc+ ca)]

.

On the other hand, it is easy to verify that∑[a2(a+ b+ c) + 2a(ab+ bc+ ca)] = (a+ b+ c)3.

Therefore, from the above Cauchy Schwarz step, the result follows immedi-

ately. Note that the equality holds if and only if a = b = c =1

3.

?F?

05.9. Given positive real numbers a, b, c such that a + b + c = 1. Prove thatthe following inequality holds

10(a3 + b3 + c3)− 9(a5 + b5 + c5) ≥ 1.

(China 2005)

Solution: Replacing 1 by a+ b+ c, we can rewrite the original inequality as

(10a3 − 9a5 − a) + (10b3 − 9b5 − b) + (10c3 − 9c5 − c) ≥ 0,

390

Page 392: Олимпиадад бэлтгэгчдэд тусламж (Тэнцэтгэл биш 1)

or equivalently,

a(1− a2)(9a2 − 1) + b(1− b2)(9b2 − 1) + c(1− c2)(9c2 − 1) ≥ 0.

Now, with noting that for any nonnegative number x,

(1 + x)(9x2 − 1)− 8

3(3x− 1) =

1

3(3x+ 5)(3x− 1)2 ≥ 0,

we can find that the above inequality is deduced from

a(1− a)(3a− 1) + b(1− b)(3b− 1) + c(1− c)(3c− 1) ≥ 0.

Expanding (with noting that a+ b+ c = 1), one may write it into

4(a2 + b2 + c2)− 3(a3 + b3 + c3) ≥ 1,

or

4(a2 + b2 + c2)(a+ b+ c)− 3(a3 + b3 + c3) ≥ (a+ b+ c)3.

Expanding and simplifying once more time, it can be written in the form

ab(a+ b) + bc(b+ c) + ca(c+ a) ≥ 6abc.

Of course, this last inequality is obviously true by the AM-GM Inequality and

so, our proof is completed. The equality holds if and only if a = b = c =1

3.

?F?

05.10. Let ABC be an acute triangle. Determine the least value of thefollowing expression

P =cos2A

cosA+ 1+

cos2B

cosB + 1+

cos2C

cosC + 1.

(China 2005)

First solution: Put x = cotA, y = cotB, z = cotC, then x, y, z > 0 andxy + yz + zx = 1. Now, we see that

cos2A

cosA+ 1=

x2

x2 + 1

1 +x√

x2 + 1

=x2

√x2 + 1

(√x2 + 1 + x

)=x2(√

x2 + 1− x)

√x2 + 1

= x2 − x3√x2 + 1

= x2 − x3√x2 + xy + yz + zx

= x2 − x3√(x+ y)(x+ z)

≥ x2 − x3

2(x+ y)− x3

2(x+ z),

391

Page 393: Олимпиадад бэлтгэгчдэд тусламж (Тэнцэтгэл биш 1)

Similarly, we also have

cos2B

cosB + 1≥ y2 − y3

2(y + z)− y3

2(y + x),

cos2C

cosC + 1≥ z2 − z3

2(z + x)− z3

2(z + y).

Hence

P =cos2A

cosA+ 1+

cos2B

cosB + 1+

cos2C

cosC + 1

≥ x2 + y2 + z2 − x3 + y3

2(x+ y)− y3 + z3

2(y + z)− z3 + x3

2(z + x)

= x2 + y2 + z2 − 1

2(x2 − xy + y2)− 1

2(y2 − yz + z2)− 1

2(z2 − zx+ x2)

=1

2(xy + yz + zx) =

1

2.

Moreover, we can see that the equality holds for an equilateral triangle. There-

fore, the minimum value of P is1

2.

Second solution: Similarly, we need to prove the inequality

cos2A

cosA+ 1+

cos2B

cosB + 1+

cos2C

cosC + 1≥ 1

2.

Put x = cosA, y = cosB, z = cosC then x, y, z > 0 and x2+y2+z2+2xyz = 1.Our inequality becomes

x2

x+ 1+

y2

y + 1+

z2

z + 1≥ 1

2,

which is equivalent to(2x2

x+ 1− x2

)+

(2y2

y + 1− y2

)+

(2z2

z + 1− z2

)≥ 1− x2 − y2 − z2,

orx2(1− x)

1 + x+y2(1− y)

1 + y+z2(1− z)

1 + z≥ 2xyz.

By the AM-GM Inequality, we get

x2(1− x)

1 + x+y2(1− y)

1 + y+z2(1− z)

1 + z≥ 3 3

√x2y2z2(1− x)(1− y)(1− z)

(1 + x)(1 + y)(1 + z).

Hence, it suffices to prove that

(1− x)(1− y)(1− z) ≥ 8

27xyz(1 + x)(1 + y)(1 + z),

or (1

x− 1

)(1

y− 1

)(1

z− 1

)≥ 8

27(1 + x)(1 + y)(1 + z).

392

Page 394: Олимпиадад бэлтгэгчдэд тусламж (Тэнцэтгэл биш 1)

Now, we can see that this inequality follows from combining the two inequal-ities

8

27(1 + x)(1 + y)(1 + z) ≤ 1,

and (1

x− 1

)(1

y− 1

)(1

z− 1

)≥ 1.

In addition, it is easy to see that the first inequality follows immediately fromthe AM-GM Inequality and the well-known x+y+z = cosA+cosB+cosC ≤3

2. So, it suffices to prove the second. Without loss of generality, we may

assume that x ≥ y ≥ z. From this assumption and the inequality x+y+z ≤ 3

2,

we get y + z ≤ 1, and hence it follows that(1

y− 1

)(1

z− 1

)−(

2

y + z− 1

)2

=(1− y − z)(y − z)2

yz(y + z)2≥ 0.

On the other hand, we have

(y + z)2 − 2(1− x) = y2 + z2 + 2yz + 2x− 2(x2 + y2 + z2 + 2xyz)

= 2x(1− x− 2yz)− (y − z)2

=2x(1 + x)(1− x− 2yz)

1 + x− (y − z)2

=2x(1− x2 − 2yz − 2xyz)

1 + x− (y − z)2

=2x(y − z)2

1 + x− (y − z)2 =

(x− 1)(y − z)2

x+ 1≤ 0.

This implies that

2

y + z− 1 ≥ 2√

2(1− x)− 1 =

√2

1− x− 1 ≥ 0.

Combining this with the above inequality, we deduce that(1

x− 1

)(1

y− 1

)(1

z− 1

)≥(

1

x− 1

)(√2

1− x− 1

)2

=

(√2−√

1− x)2

x.

Furthermore, note that the inequality

(√2−√

1− x)2

x≥ 1 is equivalent to(

1−√

2− 2x)2 ≥ 0, which is true. So, from the above inequality, can conclude

that (1

x− 1

)(1

y− 1

)(1

z− 1

)≥ 1,

as desired.

Third solution: We will now give another proof for the inequality

cos2A

cosA+ 1+

cos2B

cosB + 1+

cos2C

cosC + 1≥ 1

2.

393

Page 395: Олимпиадад бэлтгэгчдэд тусламж (Тэнцэтгэл биш 1)

Since cosA =b2 + c2 − a2

2bc, we get

cos2A

cosA+ 1=

(b2 + c2 − a2)2

4b2c2+ 1

b2 + c2 − a2

2bc+ 1

=(b2 + c2 − a2)2

2[bc(b+ c)2 − a2bc].

Therefore, our inequality is equivalent to

(b2 + c2 − a2)2

bc(b+ c)2 − a2bc+

(c2 + a2 − b2)2

ca(c+ a)2 − b2ca+

(a2 + b2 − c2)2

ab(a+ b)2 − c2ab≥ 1.

By the Cauchy Schwarz Inequality, we have

(b2 + c2 − a2)2

bc(b+ c)2 − a2bc+

(c2 + a2 − b2)2

ca(c+ a)2 − b2ca+

(a2 + b2 − c2)2

ab(a+ b)2 − c2ab≥

≥ (a2 + b2 + c2)2

ab(a+ b)2 + bc(b+ c)2 + ca(c+ a)2 − abc(a+ b+ c).

So, it is enough to prove that

(a2 + b2 + c2)2 ≥ ab(a+ b)2 + bc(b+ c)2 + ca(c+ a)2 − abc(a+ b+ c),

which is equivalent to

a2(a− b)(a− c) + b2(b− c)(b− a) + c2(c− a)(c− b) ≥ 0,

It is the Schur’s Inequality (in the special case fourth degree).?F?

05.11. Let a, b, c be positive real numbers such that1

a+

1

b+

1

c= 1. Prove

that(a− 1) (b− 1) (c− 1) ≥ 8.

(Croatia 2005)

Solution: From the given hypothesis, we have ab+ bc+ ca = abc, and

1 =1

a+

1

b+

1

c≥ 3

3

√1

abc,

using the AM-GM Inequality. This implies that abc ≥ 27, and so we get

(a− 1) (b− 1) (c− 1) = abc− (ab+ bc+ ca) + a+ b+ c− 1

= abc− abc+ a+ b+ c− 1 ≥ 33√abc− 1 ≥ 8,

as desired. It is easy to see that the equality holds if and only if a = b = c = 3.?F?

05.12. Let a, b, c > 0 such that abc = 1. Prove that

a

(a+ 1) (b+ 1)+

a

(b+ 1) (c+ 1)+

a

(c+ 1) (a+ 1)≥ 3

4.

394

Page 396: Олимпиадад бэлтгэгчдэд тусламж (Тэнцэтгэл биш 1)

(Czech-Slovak 2005)

Solution: The desired inequality is equivalent to

4a (c+ 1) + 4b (a+ 1) + 4c (b+ 1) ≥ 3 (a+ 1) (b+ 1) (c+ 1) ,

or

ab+ bc+ ca+ a+ b+ c ≥ 6,

which is true since

ab+ bc+ ca+ a+ b+ c ≥ 66√a3b3c3 = 6,

from the AM-GM Inequality. The equality holds if and only if a = b = c = 1.?F?

05.13. If a, b, c are three positive real numbers such that ab + bc + ca = 1,prove that

3

√1

a+ 6b+

3

√1

b+ 6c+

3

√1

c+ 6a ≤ 1

abc.

(Germany 2005)

First solution: According to the AM-GM Inequality, we have 1 = ab+ bc+

ca ≥ 33√a2b2c2, from which it follows that abc ≤ 1

3√

3. On the other hand, for

any real numbers x, y, z, we have

(x+ y + z)2 − 3(xy + yz + zx) =1

2[(x− y)2 + (y − z)2 + (z − x)2] ≥ 0.

Setting x = ab, y = bc, z = ca in this inequality, we get 1 = (ab+ bc+ ca)2 ≥3(ab · bc+ bc · ca+ ca ·ab) = 3abc(a+ b+ c), or a+ b+ c ≤ 1

3abc. Now, applying

the AM-GM Inequality again, we have

3

√1

a+ 6b =

1

3·√

3 ·√

3 · 3

√1

a+ 6b ≤ 1

3√

3 + 3√

3 +

(1

a+ 6b

)3

=1

9

(6√

3 +1

a+ 6b

).

Therefore

∑3

√1

a+ 6b ≤ 1

9

[18√

3 +

(1

a+

1

b+

1

c

)+ 6(a+ b+ c)

]=

1

9

[18√

3 +ab+ bc+ ca

abc+ 6(a+ b+ c)

]=

1

9

[18√

3 +1

abc+ 6(a+ b+ c)

].

395

Page 397: Олимпиадад бэлтгэгчдэд тусламж (Тэнцэтгэл биш 1)

Now, from what we have shown above, it is easy to see that 18√

3 ≤ 6

abcand

6(a+ b+ c) ≤ 2

abc. Thus, from the last inequality, we get

3

√1

a+ 6b+

3

√1

b+ 6c+

3

√1

c+ 6a ≤ 1

9

(6

abc+

1

abc+

2

abc

)=

1

abc,

which is just the desired inequality. The equality holds if and only if three

numbers a, b, c are equal and they are equal to1√3.

Second solution: By the Power Mean Inequality

u+ v + w

3≤ 3

√u3 + v3 + w3

3,

the left hand side of the original inequality does not exceed

33√

3

3

√1

a+ 6b+

1

b+ 6c+

1

c+ 6a =

33√

3

3

√ab+ bc+ ca

abc+ 6(a+ b+ c). (1)

The condition ab+ bc+ ca = 1 enables us to write

a+ b =1− abc

=ab− (ab)2

abc, b+ c =

bc− (bc)2

abc, c+ a =

ca− (ca)2

abc.

Hence

ab+ bc+ ca

abc+ 6(a+ b+ c) =

1

abc+ 3[(a+ b) + (b+ c) + (c+ a)]

=4− 3[(ab)2 + (bc)2 + (ca)2]

abc.

Now, we have 3[(ab)2 + (bc)2 + (ca)2] ≥ (ab+ bc+ ca)2 = 1 by the well-knowninequality 3(u2 + v2 + w2) ≥ (u + v + w)2. Hence an upper bound for the

right hand side of (1) is3

3√abc

. So it suffices to check3

3√abc≤ 1

abc, which is

equivalent to (abc)2 ≤ 1

27. This follows from the AM-GM Inequality, in view

of ab+ bc+ ca = 1 again

(abc)2 = (ab)(bc)(ca) ≤(ab+ bc+ ca

3

)3

=

(1

3

)3

=1

27.

?F?

05.14. Given positive real numbers x, y, z such that xyz ≥ 1. Prove that

x5 − x2

x5 + y2 + z2+

y5 − y2

y5 + z2 + x2+

z5 − z2

z5 + x2 + y2≥ 0.

(IMO 2005)

396

Page 398: Олимпиадад бэлтгэгчдэд тусламж (Тэнцэтгэл биш 1)

First solution: Note that

x5 − x2

x5 + y2 + z2≥ x5 − x2

x3 (x2 + y2 + z2)

is equivalent to (x3 − 1

)2x2(y2 + z2

)x3 (x2 + y2 + z2) (x5 + y2 + z2)

≥ 0,

which is true for all positive x, y, z. Hence

x5 − x2

x5 + y2 + z2≥

x2 − 1

xx2 + y2 + z2

.

Summing the above inequality with its analogous cyclic inequalities, we seethat the desired result follows from

x2 + y2 + z2 − 1

x− 1

y− 1

z≥ 0.

Since xyz ≥ 1,

x2 + y2 + z2 − 1

x− 1

y− 1

z≥ x2 + y2 + z2 − yz − zx− xy ≥ 0,

so we are done. It is easy to see that the equality holds if and only if x = y =z = 1.

Second solution: Note that

x5 − x2

x5 + y2 + z2= 1− x2 + y2 + z2

x5 + y2 + z2,

and its cyclic analogous forms. The given inequality is equivalent to

x2 + y2 + z2

x5 + y2 + z2+x2 + y2 + z2

y5 + z2 + x2+x2 + y2 + z2

z5 + x2 + y2≤ 3.

In view of the Cauchy Schwarz Inequality and the condition xyz ≥ 1, we have

(x5 + y2 + z2)(yz + y2 + z2) ≥(x2√xyz + y2 + z2

)2 ≥ (x2 + y2 + z2)2,

orx2 + y2 + z2

x5 + y2 + z2≤ yz + y2 + z2

x2 + y2 + z2.

Taking the cyclic sum of the above inequality and analogous forms gives

x2 + y2 + z2

x5 + y2 + z2+x2 + y2 + z2

y5 + z2 + x2+x2 + y2 + z2

z5 + x2 + y2≤ 2 +

xy + yz + zx

x2 + y2 + z2.

It suffices to show that xy + yz + zx ≤ x2 + y2 + z2, which is well-known.

Third solution: We may write our inequality as∑ x5

x5 + y2 + z2≥∑ x2

x5 + y2 + z2.

397

Page 399: Олимпиадад бэлтгэгчдэд тусламж (Тэнцэтгэл биш 1)

Notice that for all m,n ≥ 0 the function f(t) =t

mt+ nis always increasing.

Therefore since x ≥ 1

yz, we have

x

x5 + y2 + z2=

x

x4 · x+ y2 + z2≥

1

yz

x4 · 1

yz+ y2 + z2

=1

x4 + yz(y2 + z2).

It follows thatx5

x5 + y2 + z2≥ x4

x4 + yz(y2 + z2),

and hence∑ x5

x5 + y2 + z2≥∑ x4

x4 + yz(y2 + z2)≥∑ x4

x4 + y4 + z4= 1. (1)

Also, since 1 ≤ xyz, we have

x2

x5 + y2 + z2=

x2 · 1(y2 + z2) · 1 + x5

≤ x2 · xyz(y2 + z2) · xyz + x5

=x2yz

x4 + yz(y2 + z2)

≤ x2yz

x4 + 2y2z2=

x2yz

x4 + y2z2 + y2z2≤ x2yz

2x2yz + y2z2=

x2

2x2 + yz.

From this, we deduce that

∑ x2

x5 + y2 + z2≤∑ x2

2x2 + yz. (2)

Combining (1) and (2), we see that it is enough to check that

x2

2x2 + yz+

y2

2y2 + zx+

z2

2z2 + xy≤ 1,

or equivalently,

yz

yz + 2x2+

zx

zx+ 2y2+

xy

xy + 2z2≥ 1.

By the Cauchy Schwarz Inequality, we get

∑ yz

yz + 2x2≥ (xy + yz + zx)2

yz(yz + 2x2) + zx(zx+ 2y2) + xy(xy + 2z2)

=(xy + yz + zx)2

(xy + yz + zx)2= 1,

as desired.

Fourth solution: We have to prove that

x2 + y2 + z2

x5 + y2 + z2+x2 + y2 + z2

y5 + z2 + x2+x2 + y2 + z2

z5 + x2 + y2≤ 3.

398

Page 400: Олимпиадад бэлтгэгчдэд тусламж (Тэнцэтгэл биш 1)

Because y2 + z2 ≥ 2yz and xyz ≥ 1, we have

1

x5 + y2 + z2≤ 1

x4

yz+ y2 + z2

≤ 1

2x4

y2 + z2+ y2 + z2

,

and the cyclic analogous forms. Thus it suffices to show that

x2 + y2 + z2

2x4

y2 + z2+ y2 + z2

+x2 + y2 + z2

2y4

z2 + x2+ z2 + x2

+x2 + y2 + z2

2z4

x2 + y2+ x2 + y2

≤ 3.

However, since this is a homogeneous inequality, the condition xyz ≥ 1 is notrelevant anymore. Furthermore, we may assume that x2 + y2 + z2 = 3. Thenthe inequality reduces to

1

2x4

3− x2+ 3− x2

+1

2y4

3− y2+ 3− y2

+1

2z4

3− z2+ 3− z2

≤ 1,

or3− x2

3x4 − 6x2 + 9+

3− y2

3y4 − 6y2 + 9+

3− z2

3z4 − 6z2 + 9≤ 1,

where x, y, z are positive real numbers with x2 + y2 + z2 = 3. Because 3x4 −6x2 + 9 = 3(x2 − 1)2 + 6 ≥ 6 and 3− x2 = y2 + z2 ≥ 0, we obtain

3− x2

3x4 − 6x2 + 9≤ 3− x2

6.

Adding the above inequality and the cyclic analogous forms gives

3− x2

3x4 − 6x2 + 9+

3− y2

3y4 − 6y2 + 9+

3− z2

3z4 − 6z2 + 9≤ 9− (x2 + y2 + z2)

6= 1,

as desired.

Fifth solution: From the given condition, we have1

x≤ yz. Using it and the

AM-GM Inequality, we find that

4(x5 − x2)x5 + y2 + z2

+ 1 =5x5 − 4x2 + y2 + z2

x5 + y2 + z2=

3x5 + (2x5 + y2 + z2 − 4x2)

x5 + y2 + z2

≥ 3x5 + 4 4√x10y2z2 − 4x2

x5 + y2 + z2≥ 3x5

x5 + y2 + z2

=3x4

x4 +1

x· (y2 + z2)

≥ 3x4

x4 + yz(y2 + z2)≥ 3x4

x4 + y4 + z4.

From this, it follows that

x5 − x2

x5 + y2 + z2≥ 3

4· x4

x4 + y4 + z4− 1

4.

399

Page 401: Олимпиадад бэлтгэгчдэд тусламж (Тэнцэтгэл биш 1)

Adding the above inequality and the cyclic analogous forms gives

x5 − x2

x5 + y2 + z2+

y5 − y2

y5 + z2 + x2+

z5 − z2

z5 + x2 + y2≥ 3

4· x

4 + y4 + z4

x4 + y4 + z4− 3

4= 0,

as desired.?F?

05.15. Let a1 ≤ a2 ≤ · · · ≤ an be positive real numbers such that

a21 + a22 + · · ·+ a2nn

= 1,a1 + a2 + · · ·+ an

n= m,

where 1 ≥ m > 0. Prove that for all i satisfying ai ≤ m, we have

n− i ≥ n(m− ai)2.

(Iran 2005)

First solution: By the Cauchy Schwarz Inequality, we have

m =a1 + a2 + · · ·+ ai + ai+1 + ai+2 + · · ·+ an

n

=a1 + a2 + · · ·+ ai

n+ai+1 + ai+2 + · · ·+ an

n

≤ a1 + a2 + · · ·+ ain

+

√(n− i)(a2i+1 + a2i+2 + · · ·+ a2n)

n

≤ ai +

√(n− i)(a2i+1 + a2i+2 + · · ·+ a2n)

n

= ai +

√(n− i)(n− a21 − a22 − · · · − a2i )

n

≤ ai +

√n(n− i)n

,

and therefore

m− ai ≤√n− in

.

This means that n− i ≥ n(m− ai)2, which yields our conclusion.

Second solution: Let bk = m− ak for all k = 1, 2, . . . , n, we then have that

m ≥ b1 ≥ b2 ≥ · · · ≥ bn,n∑k=1

bk = 0,

n∑k=1

b2k = n(1−m2).

Since ai ≤ m, we can note that m ≥ b1 ≥ b2 ≥ · · · ≥ bi ≥ 0, and thus itremains to prove that

b2i ≤n− in

.

By the Cauchy Schwarz Inequality, we get

b21 + b22 + · · ·+ b2i ≥1

i(b1 + b2 + · · ·+ bi)

2,

400

Page 402: Олимпиадад бэлтгэгчдэд тусламж (Тэнцэтгэл биш 1)

and

b2i+1 + b2i+2 + · · ·+ b2n ≥1

n− i(bi+1 + bi+2 + · · ·+ bn)2

=1

n− i(−b1 − b2 − · · · − bi)2

=1

n− i(b1 + b2 + · · ·+ bi)

2.

Summing up these two inequalities, it follows that

n(1−m2) ≥(

1

i+

1

n− i

)(b1 + b2 + · · ·+ bi)

2 ≥(

1

i+

1

n− i

)· i2b2i =

nib2in− i

,

and therefore

b2i ≤(n− i)(1−m2)

i.

On the other hand, we also have b2i ≤ m2, and so we can write that

b2i ≤ min

{m2,

(n− i)(1−m2)

i

}.

Note that this proves our inequality, since (it is easy to check that)

min

{m2,

(n− i)(1−m2)

i

}≤ n− i

n.

?F?

05.16. If three nonnegative real numbers a, b, c satisfy the condition

1

a2 + 1+

1

b2 + 1+

1

c2 + 1= 2,

prove that

ab+ bc+ ca ≤ 3

2.

(Iran 2005)

First solution: Set the following substitutions x =1

a2 + 1, y =

1

b2 + 1, and

z =1

c2 + 1, then 1 > x, y, z > 0 and x+ y + z = 2. We furthermore get

a2 =1− xx

=y + z − x

2x≥ 0,

b2 =1− yy

=z + x− y

2y≥ 0,

c2 =1− zz

=x+ y − z

2z≥ 0,

and therefore x, y, z are the sidelengths of a triangle. Next, putm =y + z − x

2, n =

z + x− y2

, p =x+ y − z

2. We then have that m+ n+ p = 2 and

a =

√m

n+ p, b =

√n

p+m, c =

√p

m+ n.

401

Page 403: Олимпиадад бэлтгэгчдэд тусламж (Тэнцэтгэл биш 1)

Now, by using the AM-GM Inequality,

2ab = 2

√m

n+ p· n

p+m= 2

√m

m+ p· n

n+ p≤ m

m+ p+

n

n+ p,

and similarly, we get

2bc ≤ n

m+ n+

p

m+ p, 2ca ≤ m

m+ n+

p

n+ p.

Thus

2(ab+ bc+ ca) ≤ m

m+ p+

n

n+ p+

n

m+ n+

p

m+ p+

m

m+ n+

p

n+ p= 3.

The equality holds if and only if a = b = c = 1√2.

Second solution: From the given hypothesis, we get

1 =

(1− 1

a2 + 1

)+

(1− 1

b2 + 1

)+

(1− 1

c2 + 1

)=

a2

a2 + 1+

b2

b2 + 1+

c2

c2 + 1.

On the other hand, the Cauchy Schwarz Inequality yields

a2

a2 + 1+

b2

b2 + 1+

c2

c2 + 1≥ (a+ b+ c)2

a2 + b2 + c2 + 3.

Thus, we have a2 + b2 + c2 + 3 ≥ (a+ b+ c)2, which leads us to

ab+ bc+ ca ≤ 3

2.

?F?

05.17. If x, y, z are real numbers satisfying xyz = −1, prove that

x4 + y4 + z4 + 3(x+ y + z) ≥ y2 + z2

x+z2 + x2

y+x2 + y2

z.

(Iran 2005)

Solution: According to the condition from the hypothesis, we have 3(x+ y+z) = −3xyz(x+ y + z), and thus

y2 + z2

x+z2 + x2

y+x2 + y2

z= −yz(y2 + z2)− zx(z2 + x2)− xy(x2 + y2)

= −x3(y + z)− y3(z + x)− z3(x+ y).

The inequality in question is now equivalent to

x4 + y4 + z4 − 3xyz(x+ y + z) ≥ −x3(y + z)− y3(z + x)− z3(x+ y),

which can be rewritten as

[x4 + y4 + z4 + x3(y + z) + y3(z + x) + z3(x+ y)]− 3xyz(x+ y + z) ≥ 0,

402

Page 404: Олимпиадад бэлтгэгчдэд тусламж (Тэнцэтгэл биш 1)

or(x+ y + z)(x3 + y3 + z3 − 3xyz) ≥ 0,

which is obviously true according to

(x+ y + z)(x3 + y3 + z3 − 3xyz) = (x+ y + z)2(x2 + y2 + z2 − xy − yz − zx),

and to the well-known

x2 + y2 + z2 ≥ xy + yz + zx.

?F?

05.18. Let a, b, c be positive real numbers such that a+ b+ c = 1. Prove that

a3√

1 + b− c+ b 3√

1 + c− a+ c3√

1 + a− b ≤ 1.

(Japan 2005)

Solution: Because 1 + b − c > 0, we can apply the AM-GM Inequality andget

a3√

1 + b− c = a 3√

1 · 1 · (1 + b− c) ≤ a(

1 + 1 + 1 + b− c3

)= a+

ab− ac3

.

Adding this to the two analogous inequalities and using the fact that a+b+c =1, we get the result. It is easy to see that the equality holds if and only if

a = b = c =1

3.

?F?

05.19. For any real numbers x1, x2, . . . , xn with x21 + x22 + · · ·+ x2n = 1, provethat

x11 + x21

+x2

1 + x21 + x22+ · · ·+ xn

1 + x21 + x22 + · · ·+ x2n<

√n

2.

(Korea 2005)

Solution: For convenience, let us denote x0 = 0. Then, by applying theCauchy Schwarz Inequality, we can see that the left hand side of the originalinequality does not exceed√√√√n

n∑i=1

x2i(1 + x20 + · · ·+ x2i )

2≤

√√√√n

n∑i=1

x2i(1 + x20 + · · ·+ x2i )

2

√√√√n

n∑i=1

x2i(1 + x20 + · · ·+ x2i−1)(1 + x20 + · · ·+ x2i )

=

√√√√n

n∑i=1

(1

1 + x20 + · · ·+ x2i−1− 1

1 + x20 + · · ·+ x2i

)

=

√n

(1

1 + x20− 1

1 + x20 + · · ·+ x2n

)=

√n

2.

403

Page 405: Олимпиадад бэлтгэгчдэд тусламж (Тэнцэтгэл биш 1)

It is easy to see that the equality does not occur. So, we must have

n∑i=1

xi1 + x21 + · · ·+ x2i

<

√n

2.

The proof is completed.?F?

05.20. Given positive real numbers a, b, c such that a4 + b4 + c4 = 3. Provethat

1

4− ab+

1

4− bc+

1

4− ca≤ 1.

(Moldova 2005)

Solution: Since a4 + b4 + c4 = 3, it is easy to see that a2, b2, c2 < 2. Now,applying the Cauchy Schwarz Inequality and the AM-GM Inequality, we findthat

1

4− a2+

1

4− b2≥ 4

4− a2 + 4− b2≥ 4

8− 2ab=

2

4− ab.

Adding this to the two analogous inequalities and dividing each side of theresulting inequality by 2, we get

1

4− ab+

1

4− bc+

1

4− ca≤ 1

4− a2+

1

4− b2+

1

4− c2.

Therefore, it suffices to prove that

1

4− a2+

1

4− b2+

1

4− c2≤ 1.

To prove it, we observe that for each x2 < 2,

1

4− x2≤ x4 + 5

18,

since it is equivalent to (x2 − 1)2(2− x2) ≥ 0, true. So we have

1

4− a2+

1

4− b2+

1

4− c2≤ a4 + b4 + c4 + 15

18= 1,

as desired. The proof is completed. Note that the equality holds if and onlyif a = b = c = 1.

?F?

05.21. Let a, b, c ∈ [0, 1]. Prove that

a

bc+ 1+

b

ca+ 1+

c

ab+ 1≤ 2.

(Poland 2005)

404

Page 406: Олимпиадад бэлтгэгчдэд тусламж (Тэнцэтгэл биш 1)

Solution: Since a, b, c ∈ [0, 1], we have (1− a)(1− b) + (1− ab)(1− c) ≥ 0. Itfollows that 2 + abc ≥ a+ b+ c. Using this inequality, we get

a

bc+ 1+

b

ca+ 1+

c

ab+ 1≤ a

abc+ 1+

b

abc+ 1+

c

abc+ 1

=a+ b+ c

1 + abc≤ 2 + abc

1 + abc≤ 2 + 2abc

1 + abc= 2,

as desired. Note that the equality holds if and only if (a, b, c) equals (1, 1, 0),or (1, 0, 1), or (0, 1, 1).

?F?

05.22. Let a, b, c be positive real numbers such that a+ b+ c = 1. Prove that

√ab (1− c) +

√bc (1− a) +

√ca (1− b) ≤

√2

3.

(Republic of Srpska 2005)

Solution: Applying the Cauchy Schwarz Inequality in combination with thewell-known inequality 3(ab+ bc+ ca) ≤ (a+ b+ c)2, we get[∑√

ab (1− c)]2≤ (ab+ bc+ ca) (1− c+ 1− a+ 1− b)

≤ 2 · (a+ b+ c)2

3=

2

3,

as desired. The equality holds iff (a, b, c) =

(1

3,1

3,1

3

).

?F?

05.23. Let a, b, c be positive real numbers such that abc ≥ 1. Prove that

1

1 + a+ b+

1

1 + b+ c+

1

1 + c+ a≤ 1.

(Romania 2005)

Solution: By expanding, we can rewrite the inequality as∑(1 + b+ c) (1 + c+ a) ≤ (1 + a+ b) (1 + b+ c) (1 + c+ a) ,

or equivalently,

2 + 2 (a+ b+ c) ≤ 2abc+ a2 (b+ c) + b2 (c+ a) + c2 (a+ b) .

Since abc ≥ 1, it suffices to show that

a2 (b+ c) + b2 (c+ a) + c2 (a+ b) ≥ 2 (a+ b+ c) .

405

Page 407: Олимпиадад бэлтгэгчдэд тусламж (Тэнцэтгэл биш 1)

Now, applying the AM-GM Inequality and the Chebyshev’s Inequality, wehave

a2 (b+ c) + b2 (c+ a) + c2 (a+ b) ≥ 2a2√bc+ 2b2

√ca+ 2c2

√ab

≥ 2(a√a+ b

√b+ c

√c)

≥ 2

3(a+ b+ c)

(√a+√b+√c)

≥ 2

3· 3

3

√√abc (a+ b+ c) ≥ 2(a+ b+ c).

Therefore, the last inequality is obviously true. And so, our proof is completed.Note that the equality holds if and only if a = b = c = 1.

?F?

05.24. Let n is a positive integer. Prove that if x is a positive real numbers,then

1 + xn+1 ≥ (2x)n

(1 + x)n−1.

(Russia 2005)

Solution: By the AM-GM Inequality, we have

1 + xn+1 ≥ 2√xn+1 = 2x

n+12 , 1 + x ≥ 2

√x = 2x

12 .

Therefore (1 + xn+1

)(1 + x)n−1 ≥ 2x

n+12 ·

(2x

12

)n−1= (2x)n .

Dividing both sides of this inequality by (1 + x)n−1, we get the result. It iseasy to see that the equality holds if and only if x = 1.

?F?

05.25. Given positive real numbers x, y, z such that x2 + y2 + z2 = 1. Provethe following inequality

x

x3 + yz+

y

y3 + zx+

z

z3 + xy> 3.

(Russia 2005)

Solution: Setting a =yz

x, b =

zx

y, c =

xy

z, then we have ab+ bc+ ca = 1 and

x

x3 + yz=

1

a+ bc.

Therefore, the original inequality is equivalent to

1

a+ bc+

1

b+ ca+

1

c+ ab> 3.

406

Page 408: Олимпиадад бэлтгэгчдэд тусламж (Тэнцэтгэл биш 1)

Without loss of generality, we may assume that a = max{a, b, c}. If b+ c > 1,then we have a + b ≥ c + b > 1 and a + c ≥ b + c > 1, from which it followsthat

1

a+ bc>

1

a(b+ c) + bc= 1,

1

b+ ca>

1

b(c+ a) + ca= 1,

1

c+ ab>

1

c(a+ b) + ab= 1.

Therefore, it is clear that

1

a+ bc+

1

b+ ca+

1

c+ ab> 3.

Let us consider now the case b + c ≤ 1. In this case, we apply the CauchySchwarz Inequality and obtain

1

b+ ca+

1

c+ ab≥ 4

b+ ca+ c+ ab=

4

a(b+ c) + b+ c=

4

1− bc+ b+ c>

4

1 + b+ c.

On the other hand, we have

a+ bc =1− bcb+ c

+ bc =1

b+ c+bc(b+ c− 1)

b+ c≤ 1

b+ c,

which yields that1

a+ bc≥ b + c. Using this in combination with the above

inequality, it suffices to prove that

(b+ c) +4

b+ c+ 1≥ 3.

However, this inequality is obvious since from the AM-GM Inequality, we get

(b+ c) +4

b+ c+ 1= (b+ c+ 1) +

4

b+ c+ 1− 1 ≥ 4− 1 = 3.

?F?

05.26. Let a, b, c be positive real numbers. Prove that

a√b+ c

+b√c+ a

+c√a+ b

≥√

3

2(a+ b+ c).

(Serbia and Montenegro 2005)

Solution: Without loss of generality, we may assume that a ≥ b ≥ c. Fromthis assumption, we have

1√b+ c

≥ 1√c+ a

≥ 1√a+ b

.

407

Page 409: Олимпиадад бэлтгэгчдэд тусламж (Тэнцэтгэл биш 1)

Therefore, applying the Chebyshev’s Inequality and the AM-GM Inequality,we get

a√b+ c

+b√c+ a

+c√a+ b

≥ 1

3(a+ b+ c)

(1√b+ c

+1√c+ a

+1√a+ b

)≥ a+ b+ c

3· 3

3

√√(a+ b) (b+ c) (c+ a)

≥ a+ b+ c√(a+ b) + (b+ c) + (c+ a)

3

=

√3

2(a+ b+ c),

as desired. The equality holds if and only if a = b = c.?F?

05.27. If a, b, c are positive real numbers such that ab + bc + ca = 1. Provethat

33

√1

abc+ 6 (a+ b+ c) ≤

3√

3

abc.

(Slovenia 2005)

Solution: The original inequality can be written as

a2b2c2[1 + 6abc (a+ b+ c)] ≤ 1

9.

Now, using the AM-GM Inequality, we find that 1 = ab+ bc+ ca ≥ 33√a2b2c2,

and hence a2b2c2 ≤ 1

27. On the other hand, applying the well-known inequality

(x+ y + z)2 ≥ 3(xy + yz + zx) for the triple (x, y, z) = (ab, bc, ca), we get

1 = (ab+ bc+ ca)2 ≥ 3abc(a+ b+ c).

From these inequalities, we conclude that

a2b2c2[1 + 6abc (a+ b+ c)] ≤ 1

27(1 + 2) =

1

9,

as desired. Note that the equality holds if and only if a = b = c =1√3.

?F?

05.28. Let a1, a2, . . . , a95 be positive real numbers. Prove that

95∑k=1

ak ≤ 94 +

95∏k=1

max {1, ak} .

(Taiwan 2005)

Solution: Let bk = max{ak, 1}, then we have

95∏k=1

max {1, ak} =

95∏k=1

bk, and

95∑k=1

ak ≤95∑k=1

bk.

408

Page 410: Олимпиадад бэлтгэгчдэд тусламж (Тэнцэтгэл биш 1)

So it suffices to prove that

95∑k=1

bk ≤ 94 +

95∏k=1

bk.

This inequality can be written as

(1− b1)(1− b2) + (1− b1b2)(1− b3) + · · ·+ (1− b1b2 · · · b94)(1− b95) ≥ 0,

which is obviously true because bk ≥ 1 for all k = 1, 2 . . . , 95. Our proof iscompleted.

?F?

05.29. Let a, b, c be positive real numbers. Prove that(a

b+b

c+c

a

)2

≥ (a+ b+ c)

(1

a+

1

b+

1

c

).

(United Kingdom 2005)

First solution: Due to the cyclicity, we may assume that c is the smallestnumbers among a, b, c. Then, applying the AM-GM Inequality, we have

(a+ b+ c)

(1

a+

1

b+

1

c

)=a+ b+ c

b

(b

a+ 1 +

b

c

)≤ 1

4

(a+ b+ c

b+b

a+b

c+ 1

)2

.

Using this inequality, it suffices to prove that

2

(a

b+b

c+c

a

)≥ a+ b+ c

b+b

a+b

c+ 1,

which is equivalent to(ab− c

b

)+

(b

c− b

a

)+

(2c

a− 2

)≥ 0,

or

(a− c)(

1

b+

b

ac− 2

a

)≥ 0,

and this is true since a− c ≥ 0 and1

b+

b

ac≥ 2√

ac≥ 2

a. It is easy to see that

the equality holds if and only if a = b = c.

Second solution: Denote x =a

b, y =

b

c, z =

c

a, then we have xyz = 1. Now,

with noting that

(a+ b+ c)

(1

a+

1

b+

1

c

)=a

b+b

c+c

a+b

a+c

b+a

c+ 3

= x+ y + z +1

x+

1

y+

1

z+ 3,

409

Page 411: Олимпиадад бэлтгэгчдэд тусламж (Тэнцэтгэл биш 1)

we can rewrite the original inequality in the following form

(x+ y + z)2 ≥ 3 + x+ y + z +1

x+

1

y+

1

z.

Because1

x+

1

y+

1

z= xy + yz + zx, it is equivalent to

(x+ y + z)2 ≥ 3 + x+ y + z + xy + yz + zx,

orx2 + y2 + z2 + xy + yz + zx ≥ 3 + x+ y + z.

By the AM-GM Inequality, we have

xy + yz + zx ≥ 3 3√x2y2z2 = 3,

and

x2 + y2 + z2 = (x2 + 1) + (y2 + 1) + (z2 + 1)− 3 ≥ 2(x+ y + z)− 3

≥ x+ y + z + 3 3√xyz − 3 ≥ x+ y + z.

Therefore, the last inequality is obviously true and so, our proof is completed.?F?

05.30. Let a, b, c be positive real numbers. Prove that(a

a+ b

)3

+

(b

b+ c

)3

+

(c

c+ a

)3

≥ 3

8.

(Vietnam 2005)

First solution: By the Cauchy Schwarz Inequality, we get[∑ a3

(a+ b)3

] [∑a(a+ b)

]≥(∑ a2

a+ b

)2

,

and since∑

a(a+ b) =∑

a2 +∑

ab ≤ 2∑

a2, it suffices to prove that(∑ a2

a+ b

)2

≥ 3

4

∑a2,

or equivalently, ∑ 2a2

a+ b≥√

3∑

a2.

Since∑ 2a2

a+ b=∑ a2 + b2

a+ b+∑ a2 − b2

a+ b=∑ a2 + b2

a+ b, the above inequality

can be rewritten as ∑ a2 + b2

a+ b≥√

3∑

a2,

or equivalently, ∑(a2 + b2

a+ b− a+ b

2

)≥√

3∑

a2 −∑

a,

410

Page 412: Олимпиадад бэлтгэгчдэд тусламж (Тэнцэтгэл биш 1)

i.e. ∑ (a− b)2

2(a+ b)≥

∑(a− b)2√

3∑

a2 +∑

a.

From the Cauchy Schwarz Inequality, we have√

3∑

a2 ≥∑

a, hence it is

enough to show that

∑ (a− b)2

2(a+ b)≥

∑(a− b)2

2∑

a,

which is obviously valid since it can be written as∑ c(a− b)2

(a+ b)(a+ b+ c)≥ 0.

Note that the equality holds if and only if a = b = c.

Second solution: By the Cauchy Schwarz Inequality, we get[∑ a3

(a+ b)3

] [∑c3(a+ b)3

]≥(∑

c3/2a3/2)2,

and therefore we are left to prove that

8(∑

c3/2a3/2)2≥ 3

∑c3(a+ b)3.

We continue by setting the substitutions x =√ab, y =

√bc, z =

√ca. In this

case, the inequality to prove becomes

8(x3 + y3 + z3)2 ≥ 3(x2 + y2)3 + 3(y2 + z2)3 + 3(z2 + x2)3,

or equivalently,∑(x6 + y6)− 9

∑x2y2(x2 + y2) + 16

∑x3y3 ≥ 0,

which is valid because

x6 + y6 − 9x2y2(x2 + y2) + 16x3y3 = (x2 + 4xy + y2)(x− y)4 ≥ 0.

Third solution: Using the Power Mean Inequality, we get[1

3

∑ a3

(a+ b)3

]1/3≥[

1

3

∑ a2

(a+ b)2

]1/2,

hence we must show that

a2

(a+ b)2+

b2

(b+ c)2+

c2

(c+ a)2≥ 3

4.

411

Page 413: Олимпиадад бэлтгэгчдэд тусламж (Тэнцэтгэл биш 1)

Consider the substitutions x =b

a, y =

c

b, z =

a

c. Then xyz = 1, and thus the

previous inequality becomes

1

(1 + x)2+

1

(1 + y)2+

1

(1 + z)2≥ 3

4.

Since

1

(1 + x)2+

1

(1 + y)2− 1

1 + xy=

xy(x− y)2 + (xy − 1)2

(1 + xy)(1 + x)2(1 + y)2≥ 0,

we have that1

(1 + x)2+

1

(1 + y)2≥ 1

1 + xy=

z

z + 1,

and therefore

1

(1 + x)2+

1

(1 + y)2+

1

(1 + z)2≥ z

z + 1+

1

(1 + z)2=

(z − 1)2

4(z + 1)2+

3

4≥ 3

4.

Fourth solution: Like in the third solution, we reach the point where wemust show that

a2

(a+ b)2+

b2

(b+ c)2+

c2

(c+ a)2≥ 3

4.

This time we proceed as follows: from the Cauchy Schwarz Inequality, we have[∑ a2

(a+ b)2

] [∑(a+ b)2(a+ c)2

]≥[∑

a(a+ b)]2

=1

4

[∑(a+ b)2

]2,

and therefore, we are left to prove that[∑(a+ b)2

]2≥ 3

∑(a+ b)2(a+ c)2,

which is immediate, according to the AM-GM Inequality.?F?

06.1. Let x1, x2, . . . , xn be positive real numbers such that x1+x2+ · · ·+xn =1. Prove that (

n∑i=1

√xi

)(n∑i=1

1√1 + xi

)≤ n2√

n+ 1.

(China 2006)

Firs solution: From the AM-GM Inequality, we get(n∑i=1

√xi

)(n∑i=1

1√1 + xi

)≤√n+ 1

4n

(n√n+ 1

n∑i=1

√xi +

n∑i=1

1√1 + xi

)2

,

and thus it is enough to show that

n∑i=1

(n√n+ 1

√xi +

1√1 + xi

)≤ 2n

√n

n+ 1.

412

Page 414: Олимпиадад бэлтгэгчдэд тусламж (Тэнцэтгэл биш 1)

We shall now make use of the following auxiliary resultLemma. For any positive real number x, the following inequality holds

n√n+ 1

√x+

1√1 + x

≤√n(n+ 1)

2(n+ 1)2(n2x+ 3n+ 4

).

Proof. This rewrites as(n√n+ 1

√x+

1√1 + x

)2

≤ n

4(n+ 1)3(n2x+ 3n+ 4)2,

and since from the Cauchy Schwarz Inequality, we have(n√n+ 1

√x+

1√1 + x

)2

≤ (nx+ 1)

(n

n+ 1+

1

x+ 1

)=

(nx+ 1)(nx+ 2n+ 1)

(n+ 1)(x+ 1),

the problem reduces to show that

4(n+ 1)2(nx+ 1)(nx+ 2n+ 1) ≤ n(x+ 1)(n2x+ 3n+ 4)2.

On the other hand, the AM-GM Inequality gives us that

(n2x+ 3n+ 4)2 ≥ 4(n2x+ n+ 2)(2n+ 2) = 8(n+ 1)(n2x+ n+ 2),

and therefore, we are left to prove that

(n+ 1)(nx+ 1)(nx+ 2n+ 1) ≤ 2n(x+ 1)(n2x+ n+ 2),

which is valid, since it reduces to (1− n)(t− 2)2 ≤ 0.Returning to our inequality, we now conclude that

n∑i=1

(n√n+ 1

√xi +

1√1 + xi

)≤√n(n+ 1)

2(n+ 1)2

n∑i=1

(n2xi + 3n+ 4

)= 2n

√n

n+ 1.

The equality holds if and only if x1 = x2 = · · · = xn =1

n.

Second solution: Put yi = xi + 1, for all i = 1, 2, . . . , n. In this case, wehave yi ≥ 1 and y1 + y2 + · · · + yn = n + 1, and therefore the inequality inquestion becomes (

n∑i=1

√yi − 1

)(n∑i=1

1√yi

)≤ n2√

n+ 1,

or equivalently,

n∑i=1

n∑j=1

√yj − 1

√yi

≤ n2√n+ 1

.

413

Page 415: Олимпиадад бэлтгэгчдэд тусламж (Тэнцэтгэл биш 1)

From the Cauchy Schwarz Inequality, we get

n∑i=1

√yi − 1

√y1

=

=

√n

(1√n·√y1 − 1 +

√y2 − 1 · 1√

n+ · · ·+

√yn − 1 · 1√

n

)√y1

√n

√[1

n+ (y2 − 1) + · · ·+ (yn − 1)

] [(y1 − 1) +

1

n+ · · ·+ 1

n

]√y1

=

√−ny1 + 2(n+ 1)− 2n+ 1

ny1,

and similarly, one can prove that

n∑j=1

√yj − 1

√yi

≤√−nyi + 2(n+ 1)− 2n+ 1

nyi

for any i = 1, 2, . . . , n. This yields

n∑i=1

n∑j=1

√yj − 1

√yi

≤n∑i=1

√−nyi + 2(n+ 1)− 2n+ 1

nyi

√√√√nn∑i=1

[−nyi + 2(n+ 1)− 2n+ 1

nyi

]

=

√√√√n

[n(n+ 1)− 2n+ 1

n

n∑i=1

1

yi

]

√√√√√√√√√n

n(n+ 1)− 2n+ 1

n· n2

n∑i=1

yi

=

√n

[n(n+ 1)− 2n+ 1

n· n2

n+ 1

]=

n2√n+ 1

.

?F?

06.2. Let a, b, c be positive real numbers satisfying a+ b+ c = 1. Prove that

ab√ab+ bc

+bc√

bc+ ca+

ca√ca+ ab

≤√

2

2.

414

Page 416: Олимпиадад бэлтгэгчдэд тусламж (Тэнцэтгэл биш 1)

(China 2006)

First solution: From the Cauchy Schwarz Inequality, we get

∑ ab√ab+ bc

=∑ a

√b√

c+ a≤∑ a

√2b√

c+√a,

hence it suffices to prove that

2a√b√

c+√a

+2b√c

√a+√b

+2c√a√

b+√c≤ 1.

Furthermore, set√a = x,

√b = y,

√c = z. We thus have to prove that

2x2y

z + x+

2y2z

x+ y+

2z2x

y + z≤ x2 + y2 + z2,

or equivalently,

x2 + y2 + z2 +∑(

2xy − 2x2y

z + x

)≥ 2(xy + yz + zx).

In other words,

x2 + y2 + z2 + 2xyz

(1

x+ y+

1

y + z+

1

z + x

)≥ 2(xy + yz + zx).

Now since the Cauchy Schwarz Inequality gives us that

2xyz

(1

x+ y+

1

y + z+

1

z + x

)≥ 9xyz

x+ y + z,

the problem reduces to proving that

x2 + y2 + z2 +9xyz

x+ y + z≥ 2(xy + yz + zx),

which is simply a particular case of the Schur’s Inequality written for the third

degree. Note that the equality occurs iff a = b = c =1

3.

Second solution: By the Cauchy Schwarz Inequality, we have(∑ ab√ab+ bc

)2

=

[∑√ab

ab+ bc+ ca·√a(ab+ bc+ ca)

c+ a

]2≤(∑ ab

ab+ bc+ ca

)[∑ a(ab+ bc+ ca)

c+ a

]=∑ a(ab+ bc+ ca)

c+ a,

and thus, it is enough to prove that∑ a(ab+ bc+ ca)

c+ a≤ 1

2(a+ b+ c)2,

415

Page 417: Олимпиадад бэлтгэгчдэд тусламж (Тэнцэтгэл биш 1)

or equivalently,2ab2

a+ b+

2bc2

b+ c+

2ca2

c+ a≤ a2 + b2 + c2.

This can be rewritten into(2b2 − 2ab2

a+ b

)+

(2c2 − 2bc2

b+ c

)+

(2a2 − 2ca2

c+ a

)≥ a2 + b2 + c2,

or in other words,

2

(b3

a+ b+

c3

b+ c+

c3

c+ a

)≥ a2 + b2 + c2.

This last inequality is valid, since from the Cauchy Schwarz Inequality, we get

2

(b3

a+ b+

c3

b+ c+

c3

c+ a

)≥ 2(a2 + b2 + c2)2

a2 + b2 + c2 + ab+ bc+ ca

≥ 2(a2 + b2 + c2)2

2(a2 + b2 + c2)= a2 + b2 + c2.

Third solution: By same Cauchy Schwarz Inequality, we have

(∑ ab√ab+ bc

)2

=

[∑√a+ b ·

√a2b

(a+ b)(a+ c)

]2

≤[∑

(a+ b)] [∑ a2b

(a+ b)(a+ c)

]= 2

∑ a2b

(a+ b)(a+ c),

hence it suffices to prove that

4∑ a2b

(a+ b)(a+ c)≤ a+ b+ c,

which is equivalent to

4a2b(b+ c) + 4b2c(c+ a) + 4c2a(a+ b) ≤ (a+ b)(b+ c)(c+ a)(a+ b+ c),

this last one being true, because it can be written as

ab(a− b)2 + bc(b− c)2 + ca(c− a)2 ≥ 0.

?F?

06.3. Suppose that a1, a2, . . . , an are real numbers with sum 0. Prove that thefollowing inequality holds

max1≤i≤n

a2i ≤n

3

n−1∑i=1

(ai − ai+1)2.

416

Page 418: Олимпиадад бэлтгэгчдэд тусламж (Тэнцэтгэл биш 1)

(China 2006)

Solution: It is sufficient to prove that for every 1 ≤ i ≤ n, we have

(a1 − a2)2 + (a2 − a3)2 + · · ·+ (an−1 − an)2 ≥ 3

na2i .

We will the Cauchy Schwarz Inequality to prove it. Indeed, from the CauchySchwarz Inequality, we have that for all m0,m1,m2, . . . ,mn−1,mn ∈ R (m0 =mn = 0),[

n−1∑k=1

(ak − ak+1)2

](n∑k=0

m2k

)=

[n−1∑k=1

(ak − ak+1)2

](n−1∑k=1

m2k

)

[n−1∑k=1

mk(ak − ak+1)

]2

=

[n∑k=1

(mk −mk−1)ak

]2.

Now, choosing mk = k for k = 0, . . . , i − 1 and mk = k − n for k = i, . . . , n,we have

n∑k=0

m2k =

i−1∑k=0

k2 +n∑k=i

(k − n)2,

and

n∑k=1

(mk −mk−1)ai =i−1∑k=1

(mk −mk−1)ak + (mi −mi−1)ai +n∑

k=i+1

(mk −mk−1)ak

=

i−1∑k=1

ak + (1− n)ai +

n∑k=i+1

ak = −nai.

So, from the above inequality, we get[n−1∑k=1

(ak − ak+1)2

][i−1∑k=0

k2 +

n∑k=i

(k − n)2

]≥ n2a2i .

On the other hand, it is clear that

i−1∑k=0

k2+n∑k=i

(k−n)2 =i(i− 1)(2i− 1) + (n− i)(n− i+ 1)(2n− 2i+ 1)

6≤ n3

3.

Combining this with the above inequality, we get the result.?F?

06.4. Let a, b, c be the sidelengths of a triangle. Prove that

√b+ c− a√

b+√c−√a

+

√c+ a− b

√c+√a−√b

+

√a+ b− c

√a+√b−√c≤ 3.

417

Page 419: Олимпиадад бэлтгэгчдэд тусламж (Тэнцэтгэл биш 1)

(IMO Shortlist 2006)

First solution: Since a, b, c are the sidelengths of a triangle, the numbersx =

√a, y =

√b, z =

√c are also the sidelengths of a triangle (note that the

numbers −x+y+z, x−y+z and x+y−z are positive). In this case, accordingto the Cauchy Schwarz Inequality,(∑√

−x2 + y2 + z2

−x+ y + z

)2

≤ 3∑ −x2 + y2 + z2

(−x+ y + z)2,

and thus, it is sufficient to prove that∑ −x2 + y2 + z2

(−x+ y + z)2≤ 3.

Moving all into one side and distributing the 3 to each of the fractions equally,the last inequality can be rewritten as∑[

1− y2 + z2 − x2

(y + z − x)2

]≥ 0,

or equivalently,

2∑ 1

(y + z − x)2(x− y)(x− z) ≥ 0.

Without loss of generality, we can assume that x ≥ y ≥ z, then 0 < y+z−x <z + x− y, from which it follows that

1

(y + z − x)2≥ 1

(z + x− y)2.

Therefore, with notice that (x− y)(x− z) ≥ 0, we get

1

(y + z − x)2(x− y)(x− z) +

1

(z + x− y)2(y − z)(y − x) ≥

≥ 1

(z + x− y)2(x− y)(x− z) +

1

(z + x− y)2(y − z)(y − x)

=(x− y)2

(z + x− y)2≥ 0.

On the other hand, it is clear that1

(x+ y − z)2(z − x)(z − y) ≥ 0, so

∑ 1

(y + z − x)2(x− y)(x− z) ≥ 0,

and it completes our proof. Note that the equality holds if and only if a =b = c.

Second solution: Since the inequality is symmetric in the three variables,we may assume that a ≥ b ≥ c. We claim that

√a+ b− c

√a+√b−√c≤ 1,

418

Page 420: Олимпиадад бэлтгэгчдэд тусламж (Тэнцэтгэл биш 1)

and √b+ c− a√

b+√c−√a

+

√c+ a− b

√c+√a−√b≤ 2.

It is clear that the denominators are positive. So, the first inequality is equiv-alent to √

a+√b ≥√a+ b− c+

√c.

Squaring both sides, we can rewrite it as(√a+√b)2≥(√

a+ b− c+√c)2,

or equivalently, √ab ≥

√c(a+ b− c).

This inequality follows immediately from the inequality (a − c)(b − c) ≥ 0.Now, we prove the second inequality. Setting p =

√a+√b and q =

√a−√b,

we obtain a− b = pq and p ≥ 2√c. It now becomes

√c− pq√c− q

+

√c+ pq√c+ q

≤ 2.

We now apply the Cauchy Schwarz Inequality to deduce(√c− pq√c− q

+

√c+ pq√c+ q

)2

≤(c− pq√c− q

+c+ pq√c+ q

)(1√c− q

+1√c+ q

)=

2(c√c− pq2

)c− q2

· 2√c

c− q2=

4(c2 −

√cpq2

)(c− q2)2

≤4(c2 − 2cq2

)(c− q2)2

≤4(c2 − 2cq2 + q4

)(c− q2)2

= 4.

?F?

06.5. Determine the least real number M such that the inequality∣∣ab(a2 − b2) + bc(b2 − c2) + ca(c2 − a2)∣∣ ≤M(a2 + b2 + c2)2

holds for all real numbers a, b, c.(IMO 2006)

Solution: The given inequality can be rewritten as

|(a− b)(b− c)(c− a)(a+ b+ c)| ≤M(a2 + b2 + c2)2.

Now, let a = 1− 3√2, b = 1, c = 1 +

3√2

, we get M ≥ 9√

2

32. We will show that

the constant M =9√

2

32works. Indeed, put x = a− b, y = b− c, z = c− a and

s = a+ b+ c, then x+ y + z = 0, and it becomes

|xyzs| ≤ M

9(x2 + y2 + z2 + s2)2.

419

Page 421: Олимпиадад бэлтгэгчдэд тусламж (Тэнцэтгэл биш 1)

Since x + y + z = 0, there exist two numbers, supposed x and y, having thesame sign. Now, denote x + y = 2m, then z = −2m. Applying the AM-GMInequality with noting that xy ≥ 0, we have

|xyzs| = xy|zs| ≤(x+ y

2

)2

|zs| =∣∣2m3s

∣∣ .On the other hand, from the Cauchy Schwarz Inequality and the AM-GMInequality, we deduce that

(x2 + y2 + z2 + s2)2 ≥[

(x+ y)2

2+ z2 + s2

]2= (2m2 + 2m2 + 2m2 + s2)2

≥(

44√

8m6s2)2

= 32√

2∣∣m3s

∣∣ .Combining these two inequalities, we get

|xyzs| ≤ 1

16√

2(x2 + y2 + z2 + s2)2 =

M

9(x2 + y2 + z2 + s2)2.

We have equality if and only if x = y and 2m2 = s2, i.e. when 2b = a + cand (c − a)2 = 18b2. There are many triples (a, b, c) satisfying this system of

equations (for example, we can take a = 1− 3√2, b = 1, c = 1 +

3√2

as above).

From now, we conclude that the minimum value of M is9√

2

32.

?F?

06.6. Let x1, x2, x3, y1, y2, y3, z1, z2, z3 be positive real numbers. Find themaximum value of real number A if

M = (x31 + x32 + x33 + 1)(y31 + y32 + y33 + 1)(z31 + z32 + z33 + 1),

and

N = A(x1 + y1 + z1)(x2 + y2 + z2)(x3 + y3 + z3),

then M ≥ N always holds, and find the condition that the equality holds.

(Japan 2006)

Solution: Let x1 = x2 = x3 = y1 = y2 = y3 = z1 = z2 = z3 =13√

6, we get

A ≤ 3

4. From this, if we can show that the constant A =

3

4works, then we can

also conclude that it is value we need to find. However, this constant indeedworks. It suffices to prove

(x31 + x32 + x33 + 1)(y31 + y32 + y33 + 1)(z31 + z32 + z33 + 1)

(x1 + y1 + z1)(x2 + y2 + z2)(x3 + y3 + z3)≥ 3

4.

Put a =x1 + x2 + x3

3, b =

y1 + y2 + y33

and c =z1 + z2 + z3

3. Using the

420

Page 422: Олимпиадад бэлтгэгчдэд тусламж (Тэнцэтгэл биш 1)

Holder’s Inequality, we have

(x31 + x32 + x33 + 1)(y31 + y32 + y33 + 1)(z31 + z32 + z33 + 1) ≥≥ (3a3 + 1)(3b3 + 1)(3c3 + 1)

=

(3a3 +

1

2+

1

2

)(1

2+ 3b3 +

1

2

)(1

2+

1

2+ 3c3

)≥

(3

√3a3 · 1

2· 1

2+

3

√1

2· 3b3 · 1

2+

3

√1

2· 1

2· 3c3

)3

=3

4(a+ b+ c)3.

On the other hand, from the AM-GM Inequality, we get

(x1 + y1 + z1)(x2 + y2 + z2)(x3 + y3 + z3) ≤

≤ [(x1 + y1 + z1) + (x2 + y2 + z2) + (x3 + y3 + z3)]3

27

=[(x1 + x2 + x3) + (y1 + y2 + y3) + (z1 + z2 + z3)]

3

27= (a+ b+ c)3.

Combining these two inequalities, we deduce that

(x31 + x32 + x33 + 1)(y31 + y32 + y33 + 1)(z31 + z32 + z33 + 1)

(x1 + y1 + z1)(x2 + y2 + z2)(x3 + y3 + z3)≥ 3

4,

as desired. And so, we conclude that the maximum value of A is3

4.

?F?

06.7. Let a, b, c, d be real numbers with sum 0. Prove the inequality

(ab+ ac+ ad+ bc+ bd+ cd)2 + 12 ≥ 6(abc+ abd+ acd+ bcd).

(Kazakhstan 2006)

First solution: Replacing d = −a− b− c, our inequality becomes

(a2 + b2 + c2 + ab+ bc+ ca)2 + 12 + 6(a+ b+ c)(ab+ bc+ ca) ≥ 6abc,

or equivalently,

(a2 + b2 + c2 + ab+ bc+ ca)2 + 12 + 6(a+ b)(b+ c)(c+ a) ≥ 0.

This rewrites as

1

4

[(a+ b)2 + (b+ c)2 + (c+ a)2

]2+ 12 + 6(a+ b)(b+ c)(c+ a) ≥ 0.

Setting x =b+ c

2y =

c+ a

2, z =

a+ b

2, then the previous inequality goes into

(x2 + y2 + z2)2 + 48 + 24xyz ≥ 0.

421

Page 423: Олимпиадад бэлтгэгчдэд тусламж (Тэнцэтгэл биш 1)

Now, by the AM-GM Inequality, we have

(x2 + y2 + z2)2 ≥ 9 |xyz|4/3 .

and because 24xyz ≥ −24 |xyz|, it suffices to prove that

9t4 + 48− 24t3 ≥ 0,

where t = |xyz|1/3. This is true, since

9t4 + 48− 24t3 = 3(3t2 + 4t+ 4)(t− 2)2 ≥ 0.

Secdon solution: From the Rolle’s theorem, we know that there exist realnumbers x, y, z such that

x+ y + z =3

4(a+ b+ c+ d) = 0

ab+ ac+ ad+ bc+ bd+ cd = 2(xy + yz + zx)abc+ abd+ acd+ bcd = 4xyz.

Therefore, the inequality in question is equivalent to

(xy + yz + zx)2 + 3 ≥ 6xyz.

Without loss of generality, assume now that z = min {x, y, z}; then sincex+ y + z = 0, we have z = −x− y ≤ 0, and so our inequality becomes

(x2 + xy + y2)2 + 3 + 6xy(x+ y) ≥ 0.

Let us now put s = x + y ≥ 0, p = xy. In this case, the previous inequalityrewrites as

(s2 − p)2 + 3 + 6sp ≥ 0,

orf(p) = p2 + 2(3s− s2)p+ s4 + 3 ≥ 0.

We have now that

∆f = (3s− s2)2 − s4 − 3 = −3(2s+ 1)(s− 1)2 ≤ 0,

and we thus conclude that f(p) ≥ 0.?F?

06.8. Let a, b, c be sides of the triangle. Prove that

a2(b

c− 1

)+ b2

( ca− 1)

+ c2(ab− 1)≥ 0.

(Moldova 2006)

First solution: The original inequality is equivalent to

a2b

c+b2c

a+c2a

b≥ a2 + b2 + c2.

422

Page 424: Олимпиадад бэлтгэгчдэд тусламж (Тэнцэтгэл биш 1)

By the Cauchy Schwarz Inequality, we have

a2b

c+b2c

a+c2a

b=a4b2

a2bc+b4c2

b2ca+c4a2

c2ab≥ (a2b+ b2c+ c2a)2

abc(a+ b+ c).

Therefore, it suffices to prove that

(a2b+ b2c+ c2a)2 ≥ abc(a+ b+ c)(a2 + b2 + c2).

Due to the cyclicity, we can suppose without loss of generality that b is thenumber between a and c, hence (a− b)(b− c) ≥ 0. Now, applying the AM-GMInequality, we obtain

4abc(a+ b+ c)(a2 + b2 + c2) ≤ [ac(a+ b+ c) + b(a2 + b2 + c2)]2.

On the other hand,

2(a2b+ b2c+ c2a)− ac(a+ b+ c)− b(a2 + b2 + c2) = (a− b)(b− c)(a+ b− c),

which is obviously nonnegative. Therefore the inequality

(a2b+ b2c+ c2a)2 ≥ abc(a+ b+ c)(a2 + b2 + c2)

is valid and it completes our proof. The equality holds if and only if a = b = c.

Second solution: Using the substitutions a =1

x, b =

1

yand c =

1

z, the

inequality becomes

1

x2

(z

y− 1

)+

1

y2

(xz− 1)

+1

z2

(yx− 1)≥ 0,

oryz2(z − y) + zx2(x− z) + xy2(y − x) ≥ 0.

Without loss of generality, we can assume that a = min{a, b, c}, and hencex = max{a, b, c}. Denoting the left hand side of the last inequality as E(x, y, z),we will show that

E(x, y, z) ≥ E(y, y, z) ≥ 0.

We have

E(x, y, z)− E(y, y, z) = z(x3 − y3)− z2(x2 − y2) + y3(x− y)− y2(x2 − y2)= (x− y)(x− z)(xz + yz − y2).

Since (x− y)(x− z) ≥ 0 and

xz + yz − y2 ≥ y(2z − y) =2b− cb2c

=(b− a) + (a+ b− c)

b2c> 0,

it follows that E(x, y, z)− E(y, y, z) ≥ 0. On the other hand, we have

E(y, y, z) = yz(y − z)2 ≥ 0.

423

Page 425: Олимпиадад бэлтгэгчдэд тусламж (Тэнцэтгэл биш 1)

So our statement is proved.

Third solution: Writing the inequality as E(a, b, c) ≥ 0, where

E(a, b, c) = a3b2 + b3c2 + c3a2 − abc(a2 + b2 + c2).

Since 2E(a, b, c) =∑

a3(b−c)2−∑

a2(b3−c3) and∑

a2(b3−c3) =∑

a2(b−c)3, we have directly

2E(a, b, c) =∑

a2(b− c)2(a− b+ c) ≥ 0.

?F?

06.9. Let a, b, c be positive real numbers with ab+ bc+ ca = abc. Prove that

a4 + b4

ab(a3 + b3)+

b4 + c4

bc(b3 + c3)+

c4 + a4

ca(c3 + a3)≥ 1.

(Poland 2006)

Solution: We first notice that the constraint can be written as

1

a+

1

b+

1

c= 1.

It is now enough to establish the auxiliary inequality

x4 + y4

xy(x3 + y3)≥ 1

2

(1

x+

1

y

),

or2(x4 + y4

)≥(x3 + y3

)(x+ y) ,

where x, y > 0. However, we obtain

2(x4 + y4

)−(x3 + y3

)(x+ y) = x4 + y4−x3y−xy3 =

(x3 − y3

)(x− y) ≥ 0.

Therefore, we have established the inequalityx4 + y4

xy(x3 + y3)≥ 1

2

(1

x+

1

y

).

And using it, we can get the result. Note that the equality holds if and onlyif a = b = c = 3.

?F?

06.10. Find the maximum value of

(x3 + 1)(y3 + 1),

for all real numbers x, y, satisfying the condition that x+ y = 1.(Romania 2006)

Solution: Put xy = t, as x + y = 1 we get (x3 + 1)(y3 + 1) = t3 − 3t + 2.

From x + y = 1, we obtain t = xy ≤(x+ y

2

)2

=1

4. It is easy to prove that

424

Page 426: Олимпиадад бэлтгэгчдэд тусламж (Тэнцэтгэл биш 1)

t3 − 3t + 2 ≤ 4 for t ≤ 1

4, with equality if and only if t = −1. We infer that

(x3 + 1)(y3 + 1) ≤ 4 for x, y ∈ R with x+ y = 1 and (ψ3 + 1)(−1/ψ3 + 1) = 4,where ψ is one of the roots of z2 − z − 1 = 0.

?F?

06.11. Let a, b, c be three positive real numbers with sum 3. Prove that

1

a2+

1

b2+

1

c2≥ a2 + b2 + c2.

(Romania 2006)

First solution: From the AM-GM Inequality, we get

1

a2+

1

b2+

1

c2≥ 1

ab+

1

bc+

1

ca=

3

abc,

hence it suffices to prove that

3 ≥ abc(a2 + b2 + c2).

Again, the AM-GM Inequality gives us that

(ab+ bc+ ca)2 ≥ 3abc(a+ b+ c) = 9abc,

and therefore, we are left to show that

(a2 + b2 + c2)(ab+ bc+ ca)2 ≤ 27,

which is obviously valid, since

(a2 + b2 + c2)(ab+ bc+ ca)2 ≤[a2 + b2 + c2 + 2(ab+ bc+ ca)

3

]3=

(a+ b+ c)6

27= 27.

Note that the equality holds iff a = b = c = 1.

Second solution: Write the inequality in the form∑(1

a2− a2 + 4a− 4

)≥ 0,

which is equivalent to ∑ (a− 1)2(1 + 2a− a2)a2

≥ 0.

Without loss of generality, we may assume that a ≥ b ≥ c. We have to considertwo casesCase 1. a ≤ 1 +

√2. Since c ≤ b ≤ a ≤ 1 +

√2, we have 1 + 2a − a2 ≥

0, 1 + 2b − b2 ≥ 0 and 1 + 2c − c2 ≥ 0. Therefore, the inequality is obviouslytrue.

425

Page 427: Олимпиадад бэлтгэгчдэд тусламж (Тэнцэтгэл биш 1)

Case 2. a > 1 +√

2. Since b+ c = 3− a < 2−√

2 <2

3, we have

bc ≤ (b+ c)2

4<

1

9,

and hence

1

a2+

1

b2+

1

c2>

1

b2+

1

c2≥ 2

bc> 18 > (a+ b+ c)2 > a2 + b2 + c2.

Remark: Actually, we can show more:For any two positive integers n, p satisfying, n ≥ 4 and p ≥ 4, the propositionP(n, p) is false:

n∑i=1

1

xpi≥

n∑i=1

xpi for xi ∈ R, xi > 0, i = 1, . . . , n ,n∑i=1

xi = n.

This variation was considered as a problem in the Romanian IMO Team Se-lection Tests from 2007. We continue with its solutionSolution: Notice first that it is enough to find a set of values xi for n = 4 suchthat

E =n∑i=1

1

xpi−

n∑i=1

xpi < 0,

as then for any n > 4 we can extend this set of values by taking the extran− 4 ones to be equal to 1.Now that we have reduced it to the case n = 4, it makes sense to look for”simple” cases:• some xi very small - it yields E > 0, no good;• all xi equal - it yields common value 1, for which E = 0, no good;• let’s then try taking the smallest three xi equal to some value 0 < x < 1,the fourth one, denoted by y, will be 1 < y < 4, y = 4− 3x.Then,

E =3

xp+

1

yp− 3xp − yp =

1

xp

[3 +

(x

y

)p− 3x2p − (xy)p

]=

1

xp[3− (xy)p] +

1

yp[1− 3(xy)p].

It seems natural now to look for the maximum possible value for xy, it is not

difficult to see that xy =1

3· (3x) · (4 − 3x) ≤ 1

3· (2)2 =

4

3(by the AM-GM

Inequality), with equality for 3x = 4 − 3x i.e. x =2

3and y = 2. Then, as

4

3> 1 and p ≥ 4, we have

(4

3

)p≥(

4

3

)4

=256

81> 3, hence E < 0 for the set

of values (2

3,2

3,2

3, 2 and 1 repeated n− 4 times).

You might now wonder what can we say about the propositions P(4, 3) andP(3, 4). As a matter of fact, they are true. However, we will omit their proofhere.

426

Page 428: Олимпиадад бэлтгэгчдэд тусламж (Тэнцэтгэл биш 1)

?F?

06.12. Consider real numbers a, b, c contained in the interval

[1

2, 1

]. Prove

that

2 ≤ a+ b

1 + c+b+ c

1 + a+c+ a

1 + b≤ 3.

(Romania 2006)

Solution: We begin by proving the left hand side of the inequality. Since

a, b ≥ 1

2, we have a+ b ≥ 1, and thus

a+ b

1 + c≥ a+ b

a+ b+ c.

By adding the other two similar relations to the inequality from above, weobtain

2 =(a+ b) + (b+ c) + (c+ a)

a+ b+ c≤ a+ b

1 + c+b+ c

1 + a+c+ a

1 + b.

For the second inequality, note that the considered expression can be writtenas ∑(

a

1 + c+

c

1 + a

).

As a, c ≤ 1, we have

a

1 + c≤ a

a+ c, and

c

1 + a≤ c

c+ a,

and soa

1 + c+

c

1 + a≤ a

a+ c+

c

c+ a= 1.

The other two cyclic relations occur (again) similarly. Summing up the allthree, we get the desired result.

?F?

06.13. Let a, b be positive real numbers. Determine the largest constant Msuch that for all k ∈ [0, π] , we have

1

ka+ b+

1

kb+ a≥ M

a+ b.

(Thailand 2006)

Solution: Let a = b, k = π, we get M ≤ 4

π + 1. We claim that

4

π + 1is our

answer. Indeed, from the Cauchy Schwarz Inequality, we have

1

ka+ b+

1

kb+ a≥ 4

ka+ b+ kb+ a=

4

k + 1· 1

a+ b≥ 4

π + 1· 1

a+ b.

This proves that the inequality holds for M =4

π + 1. And since the equality

can occur, we conclude that the maximum value of M is4

π + 1.

427

Page 429: Олимпиадад бэлтгэгчдэд тусламж (Тэнцэтгэл биш 1)

?F?

06.14. If x, y, z are positive numbers satisfying the condition xy+yz+zx = 1,show that

27

4(x+ y)(y + z)(z + x) ≥

(√x+ y +

√y + z +

√z + x

)2 ≥ 6√

3.

(Turkey 2006)

Solution: By the Cauchy Schwarz Inequality,(√x+ y +

√y + z +

√z + x

)2 ≤ 3 (x+ y + y + z + z + x) = 6(x+ y + z),

and hence, in order to prove the left inequality, it suffices to show that

(x+ y)(y + z)(z + x) ≥ 8

9(x+ y + z).

For this, we proceed as follows

(x+ y)(y + z)(z + x) =

= (x+ y + z)(xy + yz + zx)− xyz

≥ (x+ y + z)(xy + yz + zx)− 1

9(x+ y + z)(xy + yz + zx)

=8

9(x+ y + z)(xy + yz + zx) =

8

9(x+ y + z).

For the right hand side of the inequality, we can make use of the AM-GMInequality in combination with the Minkowsky’s Inequality(√

x+ y +√y + z +

√z + x

)2 ≥≥ 3

[√(x+ y)(x+ z) +

√(y + z)(y + x) +

√(z + x)(z + y)

]= 3

(√x2 + 1 +

√y2 + 1 +

√z2 + 1

)≥ 3√

(x+ y + z)2 + (1 + 1 + 1)2

≥ 3√

3(xy + yz + zx) + 9 = 6√

3.

?F?

06.15. Let a, b, c be real numbers. Prove that the following inequality holds∑√(a2 − ab+ b2)(b2 − bc+ c2) ≥ a2 + b2 + c2.

(VMEO 2006)

Solution: By the Cauchy Schwarz Inequality, we have

(a2 − ab+ b2)(b2 − bc+ c2) =

[(b− a

2

)2+

3a2

4

] [(b− c

2

)2+

3c2

4

]≥[(b− a

2

)(b− c

2

)+

3ac

4

]2.

428

Page 430: Олимпиадад бэлтгэгчдэд тусламж (Тэнцэтгэл биш 1)

It follows that√(a2 − ab+ b2)(b2 − bc+ c2) ≥

∣∣∣∣(b− a

2

)(b− c

2

)+

3ac

4

∣∣∣∣≥(b− a

2

)(b− c

2

)+

3ac

4

= b2 +1

2(2ca− ab− bc).

Therefore∑√(a2 − ab+ b2)(b2 − bc+ c2) ≥

∑[b2 +

1

2(2ca− ab− bc)

]= a2 + b2 + c2.

Our proof is completed. It is easy to see that the equality holds if and only if(a, b, c) equals (t, t, t), or (t, 0, 0), or (0, t, 0), or (0, 0, t), where t is an arbitraryreal numbers.

?F?

07.1. Let x, y, z be positive real numbers such that√x+√y+√z = 1. Prove

that the following inequality holds

x2 + yz√2x2(y + z)

+y2 + zx√2y2(z + x)

+z2 + xy√2z2(x+ y)

≥ 1.

(APMO 2007)

Solution: According to the Cauchy Schwarz Inequality and the well-knowninequality (a+ b+ c)2 ≥ 3(ab+ bc+ ca), we have∑ yz√

2x2(y + z)=

1√2xyz

∑ y2z2√y + z

≥ 1√2· (xy + yz + zx)2

xyz(√y + z +

√z + x+

√x+ y

)≥ 1

2√

3· (xy + yz + zx)2

xyz√x+ y + z

≥ 1

2√

3· 3xyz(x+ y + z)

xyz√x+ y + z

=1

2

√3(x+ y + z) ≥ 1

2

(√x+√y +√z)

=1

2,

and∑ x2√2x2(y + z)

=1√2

∑ x√y + z

≥ 1√2· (x+ y + z)2

x√y + z + y

√z + x+ z

√x+ y

≥ 1√2· (x+ y + z)2√

(x+ y + z) [x(y + z) + y(z + x) + z(x+ y)]

=1

2

√(x+ y + z)3

xy + yz + zx≥ 1

2

√(x+ y + z) · 3(xy + yz + zx)

xy + yz + zx

=1

2

√3(x+ y + z) ≥ 1

2

(√x+√y +√z)

=1

2.

Adding up these two inequalities, we get the desired result. It is easy to see

that the equality holds if and only if x = y = z =1

9.

429

Page 431: Олимпиадад бэлтгэгчдэд тусламж (Тэнцэтгэл биш 1)

?F?

07.2. If a, b, c ∈ R such that abc = 1, then

a2+b2+c2+1

a2+

1

b2+

1

c2+2

(a+ b+ c+

1

a+

1

b+

1

c

)≥ 6+2

(b+ c

a+c+ a

b+a+ b

c

).

(Brazil 2007)

Solution: We have

a2 + b2 + c2 + 2

(1

a+

1

b+

1

c

)= a2 + b2 + c2 + 2(ab+ bc+ ca)

= (a+ b+ c)2,

1

a2+

1

b2+

1

c2+ 2 (a+ b+ c) = a2b2 + b2c2 + c2a2 + 2abc(a+ b+ c)

= (ab+ bc+ ca)2,

and

6 + 2

(b+ c

a+c+ a

b+a+ b

c

)=

2(a+ b+ c)(ab+ bc+ ca)

abc

= 2(a+ b+ c)(ab+ bc+ ca).

Therefore, our inequality is equivalent to

(a+ b+ c)2 + (ab+ bc+ ca)2 ≥ 2(a+ b+ c)(ab+ bc+ ca),

which is obviously true by the AM-GM Inequality.?F?

07.3. Given an integer n ≥ 2, find the largest constant C(n) for which theinequality

n∑i=1

xi ≥ C(n)∑

1≤j<i≤n

(2xixj +

√xixj

)holds for all real numbers xi ∈ (0, 1) satisfying (1 − xi)(1 − xj) ≥

1

4for

1 ≤ j < i ≤ n.

(Bulgaria 2007)

Solution: Let us see what happens if x1 = x2 = · · · = xn =1

2. We have that

n

2≥ C(n)

(2 · 1

4· n(n− 1)

2+

1

2· n(n− 1)

2

),

and thus,

C(n) ≤ 1

n− 1.

430

Page 432: Олимпиадад бэлтгэгчдэд тусламж (Тэнцэтгэл биш 1)

We will now show that this is the value we are looking for. That is, we areleft to prove that

(n− 1)n∑i=1

xi ≥∑

1≤j<i≤n

(2xixj +

√xixj

).

By the AM-GM Inequality, we have

∑1≤j<i≤n

√xixj ≤

1

2

∑1≤j<i≤n

(xi + xj) =n− 1

2

n∑i=1

xi,

and furthermore,

2∑

1≤j<i≤nxixj=

(n∑i=1

xi

)2

−n∑i=1

x2i

(n∑i=1

xi

)2

− 1

n

(n∑i=1

xi

)2

=n− 1

n

(n∑i=1

xi

)2

.

Hence, the problem reduces to

(n− 1)

n∑i=1

xi ≥n− 1

n

(n∑i=1

xi

)2

+n− 1

2

n∑i=1

xi,

or equivalently,n∑i=1

xi ≤n

2.

For this, note that the AM-GM Inequality is giving us

2− xj − xi = (1− xj) + (1− xi) ≥ 2√

(1− xj)(1− xi) ≥ 1,

for all integers i, j satisfying 1 ≤ j < i ≤ n. This yields

1− xj − xi ≥ 0,

and therefore ∑1≤j<i≤n

(1− xi − xj) ≥ 0

from which we now can conclude that

n∑i=1

xi ≤n

2.

This proves our inequality, and therefore maxC(n) =1

n− 1.

?F?

431

Page 433: Олимпиадад бэлтгэгчдэд тусламж (Тэнцэтгэл биш 1)

07.4. Let a, b, c be the sidelengths of a triangle such that a + b + c = 3.Determine the minimum value of

a2 + b2 + c2 +4abc

3.

(China 2007)

Solution: Because

[(a− 1)(b− 1)][(b− 1)(c− 1)][(c− 1)(a− 1)] = (a− 1)2(b− 1)2(c− 1)2 ≥ 0,

we see that at least one of the numbers (a−1)(b−1), (b−1)(c−1), (c−1)(a−1)must be nonnegative. Due to symmetry, we may assume that (a−1)(b−1) ≥ 0,or ab ≥ a + b − 1 = 2 − c, and hence abc ≥ c(2 − c). Using this inequality incombination with the Cauchy Schwarz Inequality, we get

a2 + b2 + c2 +4abc

3≥ a2 + b2 + c2 +

4

3c(2− c) ≥ 1

2(a+ b)2 + c2 +

4

3c(2− c)

=1

2(3− c)2 + c2 +

4

3c(2− c) =

1

6

[(c− 1)2 + 26

]≥ 13

3.

We have equality when a = b = c = 1, therefore the searched minimum is13

3.

?F?

07.5. Let α, β be acute angles. Determine the maximal value of(1−√

tanα tanβ)2

cotα+ cotβ.

(China 2007)

Solution: Denote with P the desired expression. By the AM-GM Inequality,we have

P ≤(1−√

tanα tanβ)2

2√

cotα cotβ=

1

2

√tanα tanβ

(1−

√tanα tanβ

)2=

1

4· 2√

tanα tanβ ·(

1−√

tanα tanβ)·(

1−√

tanα tanβ)

≤ 1

4

[2√

tanα tanβ +(1−√

tanα tanβ)

+(1−√

tanα tanβ)

3

]3=

2

27.

We have equality when α = β = arctan1

3, therefore the searched maximum is

2

27.

?F?

07.6. Let a, b, c be positive real numbers such that abc = 1. Prove that for allk ≥ 2, we have

ak

a+ b+

bk

b+ c+

ck

c+ a≥ 3

2.

432

Page 434: Олимпиадад бэлтгэгчдэд тусламж (Тэнцэтгэл биш 1)

(China 2007)

Solution: By the Cauchy Schwarz Inequality, we have

ak

a+ b+

bk

b+ c+

ck

c+ a≥

(a

k+12 + b

k+12 + c

k+12

)2a(a+ b) + b(b+ c) + c(c+ a)

=

(a

k+12 + b

k+12 + c

k+12

)2a2 + b2 + c2 + ab+ bc+ ca

.

So, it suffices to prove that

2(a

k+12 + b

k+12 + c

k+12

)2≥ 3(a2 + b2 + c2 + ab+ bc+ ca),

or equivalently,

2(ak+1+bk+1+ck+1)+4

(1

ak+12

+1

bk+12

+1

ck+12

)≥ 3

(a2 + b2 + c2 +

1

a+

1

b+

1

c

).

Now, using the Chebyshev’s Inequality, we have

1

ak+12

+1

bk+12

+1

ck+12

≥ 1

3

(1

ak−12

+1

bk−12

+1

ck−12

)(1

a+

1

b+

1

c

)≥ 1

3· 3 3

√1

ak−12 b

k−12 c

k−12

·(

1

a+

1

b+

1

c

)=

1

a+

1

b+

1

c,

and

ak+1 + bk+1 + ck+1 ≥ 1

3(ak−2 + bk−2 + ck−2)(a3 + b3 + c3)

≥ 1

3· 3 3√ak−2bk−2ck−2 · (a3 + b3 + c3)

= a3 + b3 + c3.

According to these two inequalities, we see that it is enough to check that

2(a3 + b3 + c3) + 4

(1

a+

1

b+

1

c

)≥ 3

(a2 + b2 + c2 +

1

a+

1

b+

1

c

),

which is equivalent to ∑(2a3 +

1

a− 3a2

)≥ 0.

By the AM-GM Inequality, we have 2a3 − 3a2 + 1 ≥ 0. It follows that∑(2a3 +

1

a− 3a2

)≥∑(

1

a− 1

)=

1

a+

1

b+

1

c− 3 ≥ 3

3√abc− 3 = 0,

using the AM-GM Inequality again. The equality holds if and only if a = b =c = 1.

433

Page 435: Олимпиадад бэлтгэгчдэд тусламж (Тэнцэтгэл биш 1)

?F?

07.7. Let a, b, c > 0 such that a+ b+ c = 1. Prove that

a2

b+b2

c+c2

a≥ 3(a2 + b2 + c2).

(Croatia 2007)

First solution: Applying the AM-GM Inequality, we have1

b+ 9b ≥ 6. It

follows thata2

b≥ 3a2(2− 3b). According to this inequality, it suffices to prove

that

a2(2− 3b) + b2(2− 3c) + c2(2− 3a) ≥ a2 + b2 + c2,

which is equivalent to

a2 + b2 + c2 ≥ 3(a2b+ b2c+ c2a),

or

(a2 + b2 + c2)(a+ b+ c) ≥ 3(a2b+ b2c+ c2a).

After some simple computations, we can rewrite it as

a(a− b)2 + b(b− c)2 + c(c− a)2 ≥ 0,

which is obviously true. It is easy to see that the equality holds if and only if

a = b = c =1

3.

Second solution: From the Cauchy Schwarz Inequality, we have

a2

b+b2

c+c2

a≥ (a2 + b2 + c2)2

a2b+ b2c+ c2a.

It suffices to prove that

(a2 + b2 + c2)2

a2b+ b2c+ c2a≥ 3(a2 + b2 + c2),

or equivalently,

a2 + b2 + c2 ≥ 3(a2b+ b2c+ c2a).

This inequality has been proved in the above solution.?F?

07.8. Let a, b, c, d be positive real numbers such that a+ b+ c+ d = 1. Provethat

6(a3 + b3 + c3 + d3) ≥ a2 + b2 + c2 + d2 +1

8.

(France 2007)

434

Page 436: Олимпиадад бэлтгэгчдэд тусламж (Тэнцэтгэл биш 1)

Solution: According to the Chebyshev’s Inequality, we have

a3 + b3 + c3 + d3 ≥ 1

4(a+ b+ c+ d)(a2 + b2 + c2 + d2)

=1

4(a2 + b2 + c2 + d2).

It suffices to show that

3

2(a2 + b2 + c2 + d2) ≥ a2 + b2 + c2 + d2 +

1

8,

or equivalently,

a2 + b2 + c2 + d2 ≥ 1

4.

This is true since by the Cauchy Schwarz Inequality, we have

a2 + b2 + c2 + d2 ≥ 1

4(a+ b+ c+ d)2 =

1

4.

The equality holds if and only if a = b = c = d =1

4.

?F?

07.9. Let a, b, c be sides of a triangle, show that

(c+ a− b)4

a(a+ b− c)+

(a+ b− c)4

b(b+ c− a)+b(b+ c− a)

c(c+ a− b)≥ ab+ bc+ ca.

(Greece 2007)

First solution: Because a, b, c are the sidelengths of a triangle, we see thata+ b− c, b+ c− a, a+ b− c are positive real numbers. And thus, the CauchySchwarz Inequality yields

(c+ a− b)4

a(a+ b− c)+

(a+ b− c)4

b(b+ c− a)+

(b+ c− a)4

c(c+ a− b)≥

≥ [(c+ a− b)2 + (a+ b− c)2 + (b+ c− a)2]2

a(a+ b− c) + b(b+ c− a) + c(c+ a− b).

On the other hand, it is easy to verify that a(a+b−c)+b(b+c−a)+c(c+a−b) =a2 + b2 + c2, and

(c+ a− b)2+(a+ b− c)2 + (b+ c− a)2 =

= (a2 + b2 + c2) + 2(a2 + b2 + c2 − ab− bc− ca)

≥ a2 + b2 + c2.

Therefore, from the above Cauchy Schwarz step, we can get a stronger esti-mation than the original statement is

(c+ a− b)4

a(a+ b− c)+

(a+ b− c)4

b(b+ c− a)+

(b+ c− a)4

c(c+ a− b)≥ a2 + b2 + c2 ≥ ab+ bc+ ca.

435

Page 437: Олимпиадад бэлтгэгчдэд тусламж (Тэнцэтгэл биш 1)

The equality holds if and only if a = b = c.

Second solution: By the AM-GM Inequality, we have

(c+ a− b)4

a(a+ b− c)+ a(a+ b− c) ≥ 2(c+ a− b)2.

It follows that∑ (c+ a− b)4

a(a+ b− c)≥∑

[2(c+ a− b)2 − a(a+ b− c)].

On the other hand, it is easy to verify that∑

[2(c+ a− b)2 − a(a+ b− c)] ≥ab+ bc+ ca, (as it is equivalent to a2 + b2 + c2 ≥ ab+ bc+ ca after expanding),we can obtain ∑ (c+ a− b)4

a(a+ b− c)≥ ab+ bc+ ca.

?F?

07.10. Let a, b, c, d be real numbers such that

a2 ≤ 1, a2 + b2 ≤ 5, a2 + b2 + c2 ≤ 14, a2 + b2 + c2 + d2 ≤ 30.

Prove that

a+ b+ c+ d ≤ 10.

(Hungary-Isarel 2007)

Solution: Applying the Cauchy Schwarz Inequality, we have

(a+ b+ c+ d)2 =

(1 · a+

√2 · b√

2+√

3 · c√3

+ 2 · d2

)2

≤ (1 + 2 + 3 + 4)

(a2 +

b2

2+c2

3+d2

4

)=

5

6

(12a2 + 6b2 + 4c2 + 3d2

).

On the other hand, from the given hypothesis, we find that

12a2 + 6b2 + 4c2 + 3d2 =

= 6a2 + 2(a2 + b2) + (a2 + b2 + c2) + 3(a2 + b2 + c2 + d2)

≤ 6 · 1 + 2 · 5 + 14 + 3 · 30 = 120.

Using this in combination with the above inequality, we deduce that

(a+ b+ c+ d)2 ≤ 5

6· 120 = 100,

and hence, it follows that

a+ b+ c+ d ≤ |a+ b+ c+ d| ≤ 10,

436

Page 438: Олимпиадад бэлтгэгчдэд тусламж (Тэнцэтгэл биш 1)

as desired. It is easy to see that the equality holds if and only if (a, b, c, d) =(1, 2, 3, 4).

?F?

07.11. Let a1, a2, . . . , a100 be nonnegative eral numbers such that a21 + a22 +· · ·+ a2100 = 1. Prove that

a21a2 + a22a3 + · · ·+ a2100a1 <12

25.

(IMO Shortlist 2007)

Solution: According to the Cauchy Schwarz Inequality,

1

3[a1(a

2100 + 2a1a2)+a2(a

21 + 2a2a3) + · · ·+ a100(a

299 + 2a100a1)]

≤ 1

3

(100∑k=1

a2k

) 12

·

[100∑k=1

(a2k + 2ak+1ak+2)2

] 12

.

Thus, it is sufficient to show that

100∑k=1

(a2k + 2ak+1ak+2)2 ≤ 2.

Each term of this last sum can be seen as

a4k + 4a2k+1a2k+2 + 4a2k(ak+1 · ak+2) ≤ (a4k + 2a2ka

2k+1 + 2a2ka

2k+2) + 4a2k+1a

2k+2.

The required inequality now follows from

100∑k=1

(a4k + 2a2ka2k+1 + 2a2ka

2k+2) ≤

(100∑k=1

a2k

)2

= 1,

and

100∑k=1

a2ka2k+1 ≤ (a21 + a23 + · · ·+ a299) · (a22 + a24 + · · ·+ a2100)

≤ 1

4

(100∑k=1

a2k

)2

=1

4.

?F?

07.12. Let n be a positive integer, and let x and y be positive real numberssuch that xn + yn = 1. Prove that(

n∑k=1

1 + x2k

1 + x4k

)(n∑k=1

1 + y2k

1 + y4k

)<

1

(1− x)(1− y).

(IMO Shortlist 2007)

437

Page 439: Олимпиадад бэлтгэгчдэд тусламж (Тэнцэтгэл биш 1)

Solution: The inequality1 + t2

1 + t4<

1

tholds for all t ∈ (0, 1) because it is

equivalent to 0 < t4 − t3 − t + 1 = (1 − t)(1 − t3). Applying it to t = xk andsumming over k = 1, 2, . . . , n, we get

n∑k=1

1 + x2k

1 + x4k<

n∑k=1

1

xk=

xn − 1

xn(x− 1)=

yn

xn(1− x).

Similarly, we obtain that

n∑k=1

1 + x2k

1 + x4k<

xn

yn(1− y),

and therefore, by multiplying them up, yields(n∑k=1

1 + x2k

1 + x4k

)(n∑k=1

1 + y2k

1 + y4k

)<

1

(1− x)(1− y),

as desired.?F?

07.13. Real numbers a1, a2, . . . , an are given. For each i (1 ≤ i ≤ n) define

di = max {aj : 1 ≤ j ≤ i} −min {aj : i ≤ j ≤ n},

and letd = max {di : 1 ≤ i ≤ n}.

(a) Prove that for any real numbers x1 ≤ x2 ≤ · · · ≤ xn, we have

max {|xi − ai| : 1 ≤ i ≤ n} ≥ d

2.

(b) Show that there are real numbers x1 ≤ x2 ≤ · · · ≤ xn such that we haveequality in (a).

(IMO 2007)

Solution: (a) Let 1 ≤ p ≤ q ≤ r ≤ n be indices for which

d = dq, ap = max{aj : 1 ≤ j ≤ q}, ar = min{aj : q ≤ j ≤ n},

and thus d = ap − aq. (These indices are not necessarily unique.)For arbitrary real numbers x1 ≤ x2 ≤ · · · ≤ xn, consider just the two quantities|xp − ap| and |xr − ar|. Since

(ap − xp) + (xr − ar) = (ap − ar) + (xr − xp) ≥ ap − ar = d,

we have either ap − xp ≥d

2or xr − ar ≥

d

2. Hence,

max{|xi−ai| : 1 ≤ i ≤ n} ≥ max{|xp−ap|, |xr−ar|} ≥ max{xp−ap, xr−ar} ≥d

2.

438

Page 440: Олимпиадад бэлтгэгчдэд тусламж (Тэнцэтгэл биш 1)

(b) For each 1 ≤ i ≤ n, letMi = max {aj : 1 ≤ j ≤ i} andmi = min {aj : i ≤ j ≤ n}.

Set xi =mi +Mi

2. Clearly, mi ≤ ai ≤ Mi and both (mi) and (Mi) are non-

decreasing. Furthermore, from di = Mi −mi, we obtain that

−di2

=mi −Mi

2= xi −Mi ≤ xi − ai ≤ xi −mi =

Mi −mi

2=di2.

Therefore,

max {|xi − ai| : 1 ≤ i ≤ n} ≤ max

{di2

: 1 ≤ i ≤ n}

=d

2.

Since the opposite inequality has been proved in part (a), we must have equal-ity.

?F?

07.14. If x, y, z are positive real numbers, prove that the following inequalityholds

(x+ y + z)2(yz + zx+ xy)2 ≤ 3(y2 + yz + z2)(z2 + zx+ x2)(x2 + xy + y2).

(India 2007)

Solution: By the AM-GM Inequality, we have that

y2 + yz + z2 =3

4(y + z)2 +

1

4(y − z)2 ≥ 3

4(y + z)2,

hence it suffices to show that

81

64(y + z)2(z + x)2(x+ y)2 ≥ (x+ y + z)2(yz + zx+ xy)2.

This rewrites as

(x+ y)(y + z)(z + x) ≥ 8

9(x+ y + z)(xy + yz + zx),

which we have proved at the preceding problem.

Remark: Actually, we can show that the original inequality is valid for anyreal numbers x, y, z and they are not necessary to be nonnegative. The resultis followed from an interesting identity

3(x2 + xy+ y2)(y2 + yz + z2)(z2 + zx+ x2)− (x+ y+ z)2(xy+ yz + zx)2 =

=1

2(xy + yz + zx)2

∑(x− y)2 +

1

2(x+ y + z)2

∑(zx− yz)2.

?F?

07.15. Let a, b, c be distinct positive real numbers. Show that∣∣∣∣a+ b

a− b+b+ c

b− c+c+ a

c− a

∣∣∣∣ > 1.

439

Page 441: Олимпиадад бэлтгэгчдэд тусламж (Тэнцэтгэл биш 1)

(Iran 2007)

Solution: Observe that

a+ b

a− b· b+ c

b− c+b+ c

b− c· c+ a

c− a+c+ a

c− a· a+ b

a− b= −1.

From this, it follows that(a+ b

a− b+b+ c

b− c+c+ a

c− a

)2

=∑ (a+ b)2

(a− b)2+ 2

∑ a+ b

a− b· b+ c

b− c

=∑ (a+ b)2

(a− b)2− 2 =

∑[(a+ b)2

(a− b)2− 1

]+ 1

= 4∑ ab

(a− b)2+ 1 > 1.

Taking square root of each side of this inequality, we get the desired result.?F?

07.16. Find the largest constant T such that for all nonnegative real numbersa, b, c, d, e satisfying a+ b = c+ d+ e, we have√

a2 + b2 + c2 + d2 + e2 ≥ T(√

a+√b+√c+√d+√e)2.

(Iran 2007)

Solution: Let a = b =1

2, c = d = e =

1

3, then we get T ≤

√5

6

(√3−√

2)2.

Now, we will prove that the desired inequality holds for T =

√5

6

(√3−√

2)2

.

Indeed, by the Cauchy Schwarz Inequality, we have√a2 + b2 + c2 + d2 + e2 ≥

√(a+ b)2

2+

(c+ d+ e)2

3=

√5

6(a+ b),

and(√a+√b+√c+√d+√e)2

=

=

(14√

2· 4√

2a2 +14√

2· 4√

2b2 +14√

3· 4√

3c2 +14√

3· 4√

3d2 +14√

3· 4√

3e2)2

≤(

1√2

+1√2

+1√3

+1√3

+1√3

)(√2a+

√2b+

√3c+

√3d+

√3e)

=(√

2 +√

3)2

(a+ b).

It follows that

T(√

a+√b+√c+√d+√e)2≤ T

(√2 +√

3)2

(a+ b)

=

√5

6

(√3−√

2)2 (√

2 +√

3)2

(a+ b)

=

√5

6(a+ b) ≤

√a2 + b2 + c2 + d2 + e2,

440

Page 442: Олимпиадад бэлтгэгчдэд тусламж (Тэнцэтгэл биш 1)

as desired. So, for T =

√5

6

(√3−√

2)2

, the inequality holds and it attains

the equality for a = b =3

2c =

3

2d =

3

2e. This allows us to conclude that the

largest constant T satisfies the required question is T =

√5

6

(√3−√

2)2

.

?F?

07.17. Prove that for any positive real numbers a, b, c, we have

a+ b+ c

3≤√a2 + b2 + c2

3≤ 1

3

(bc

a+ca

b+ab

c

).

(Ireland 2007)

Solution: The left inequality is just the Cauchy Schwarz Inequality. So itis enough to prove the right inequality. Applying the well-known inequality

(x+ y + z)2 ≥ 3(xy + yz + zx) for the triple (x, y, z) =

(ab

c,bc

a,ca

b

), we get

(bc

a+ca

b+ab

c

)2

≥ 3

(bc

a· cab

+ca

b· abc

+ab

c· bca

)= 3(a2 + b2 + c2).

Taking square root of each side of this inequality, we get the desired inequality.It is easy to see that the equality (in both inequalities) holds if and only ifa = b = c.

?F?

07.18. Let n ≥ 2 be a given integer. Determine(a) the largest real cn such that

1

1 + a1+

1

1 + a2+ · · ·+ 1

1 + an≥ cn

holds for any positive numbers a1, a2, . . . , an with a1a2 · · · an = 1.(b) the largest real dn such that

1

1 + 2a1+

1

1 + 2a2+ · · ·+ 1

1 + 2an≥ dn

holds for any positive numbers a1, a2, . . . , an with a1a2 · · · an = 1.(Italy 2007)

Solution: (a) Let us see what happens if a1 = a2 = · · · = an−1 = x and

an =1

xn−1, where x is an arbitrary positive real number. In this case, the

inequality becomesn− 1

x+ 1+

xn−1

xn−1 + 1≥ cn,

and by setting x near to infinity, we get cn ≤ 1. Now, we are left to show thatindeed we have

1

1 + a1+

1

1 + a2+ · · ·+ 1

1 + an≥ 1.

441

Page 443: Олимпиадад бэлтгэгчдэд тусламж (Тэнцэтгэл биш 1)

For this, we can proceed by assuming (without loss of generality) that a1 ≤a2 ≤ · · · ≤ an, then a1a2 ≤ 1, and therefore

1

1 + a1+

1

1 + a2≥ 1

1 + a1+

1

1 + 1a1

=1

1 + a1+

a1a1 + 1

= 1.

This proves (a).

(b) If n = 2 then the inequality becomes

1

1 + 2a1+

1

1 + 2a2≥ d2, i.e.

1

1 + 2a1+

a1a1 + 2

≥ d2.

This can be rewritten as

2(a1 − 1)2

3(a1 + 2)(2a1 + 1)+

2

3≥ d2,

and it easily follows that d2 =2

3is the ”sharpest choice”.

If n ≥ 3, we set a1 = a2 = · · · = an−1 = x and an =1

xn−1, where x is an

arbitrary positive real number. In this case, the inequality becomes

n− 1

1 + 2x+

xn−1

xn−1 + 2≥ dn,

and thus, by letting (again) x near to infinity, we get get dn ≤ 1. We are nowleft to prove that

1

1 + 2a1+

1

1 + 2a2+ · · ·+ 1

1 + 2an≥ 1.

Without loss of generality, we can assume that a1 ≤ a2 ≤ · · · ≤ an, thena1a2a3 ≤ 1, and therefore there exists a positive number k such that k < 1

and satisfying a1a2a3 = k3. Now, by setting a1 =knp

m2, a2 =

kpm

n2, a3 =

kmn

p2,

we have

1

1 + 2a1+

1

1 + 2a2+

1

1 + 2a3=

m2

m2 + 2knp+

n2

n2 + 2kpm+

p2

p2 + 2kmn

≥ m2

m2 + 2np+

n2

n2 + 2pm+

p2

p2 + 2mn

≥ (m+ n+ p)2

m2 + 2np+ n2 + 2pm+ p2 + 2mn= 1.

This yields

dn =

{23 , if n = 21, if n > 2

.

?F?

07.19. If a, b are positive real numbers such that ab ≥ 1, then

1

(2a+ 3)2+

1

(2b+ 3)2≥ 2

5(2ab+ 3).

442

Page 444: Олимпиадад бэлтгэгчдэд тусламж (Тэнцэтгэл биш 1)

(Kiev 2007)

Solution: By the Cauchy Schwarz Inequality, we have

1

(2a+ 3)2+

1

(2b+ 3)2≥ (a+ b)2

b2(2a+ 3)2 + a2(2b+ 3)2

=(a+ b)2

9(a+ b)2 + 12ab(a+ b) + 8a2b2 − 18ab.

It suffices to prove that

(a+ b)2

9(a+ b)2 + 12ab(a+ b) + 8a2b2 − 18ab≥ 2

5(2ab+ 3),

or equivalently,

5

2(2ab+ 3) ≥ 9 +

12ab

a+ b+

8a2b2 − 18ab

(a+ b)2.

Now, we see that

5

2(2ab+ 3)−

(9 + 6

√ab+

8a2b2 − 18ab

4ab

)= 3

(√ab− 1

)2≥ 0,

so it is enough to check that

12ab

a+ b+

8a2b2 − 18ab

(a+ b)2≤ 6√ab+

8a2b2 − 18ab

4ab,

which is equivalent to

8a2b2 − 18ab

(a+ b)2− 8a2b2 − 18ab

4ab≤

6√ab(√

a−√b)2

a+ b,

or

(9− 4ab)(a− b)2

2(a+ b)2≤

6√ab(√

a−√b)2

a+ b.

Since

(√a−√b)2

a+ b≥ 0, we see that the above inequality follows from

(9− 4ab)(√

a+√b)2

2(a+ b)≤ 6√ab,

which is true because

(9− 4ab)(√

a+√b)2

2(a+ b)≤

5(√

a+√b)2

2(a+ b)≤ 5 < 6

√ab.

This completes our proof. It is easy to see that the equality holds iff a = b = 1.?F?

443

Page 445: Олимпиадад бэлтгэгчдэд тусламж (Тэнцэтгэл биш 1)

07.20. For all positive real numbers a, b, c, find all values of positive numberk such that the following inequality holds

a

c+ kb+

b

a+ kc+

c

b+ ka≥ 1

2007.

(Korea 2007)

Solution: Let a = b = c, we get k ≤ 6020. We claim that the answer is0 < k ≤ 6020. To prove this claim, it suffices to prove that for 0 < k ≤ 6020,the above inequality holds. Indeed, from the Cauchy Schwarz Inequality andthe well-known inequality (a+ b+ c)2 ≥ 3(ab+ bc+ ca), we have

a

c+ kb+

b

a+ kc+

c

b+ ka≥ (a+ b+ c)2

a(c+ kb) + b(a+ kc) + c(b+ ka)

=(a+ b+ c)2

(k + 1)(ab+ bc+ ca)≥ 3(ab+ bc+ ca)

(k + 1)(ab+ bc+ ca)

=3

k + 1≥ 1

2007,

as desired.?F?

07.21. Let a, b, c be positive real numbers. Prove that

1 +3

ab+ bc+ ca≥ 6

a+ b+ c.

(Macedonia 2007)

Solution: We use first the well-known inequality ab+ bc+ ca ≤ (a+ b+ c)2

3and then the AM-GM Inequality to get

1 +3

ab+ bc+ ca≥ 1 +

9

(a+ b+ c)2≥ 2

√9

(a+ b+ c)2=

6

a+ b+ c,

as desired. The equality holds if and only if a = b = c = 1.?F?

07.22. Let a, b, c, d be nonnegative real numbers such that a+ b+ c+ d = 4.Prove that

a2bc+ b2cd+ c2da+ d2ab ≤ 4.

(Middle Europe 2007)

Solution: We have

a2bc+ b2cd+ c2da+ d2ab− (ac+ bd)(ab+ cd) = −bd(a− c)(b− d),

and

a2bc+ b2cd+ c2da+ d2ab− (bc+ ad)(bd+ ac) = ac(a− c)(b− d).

444

Page 446: Олимпиадад бэлтгэгчдэд тусламж (Тэнцэтгэл биш 1)

Therefore

a2bc+ b2cd+ c2da+ d2ab ≤ max {(ac+ bd)(ab+ cd), (bc+ ad)(bd+ ac)} .

On the other hand, using the AM-GM Inequality, we see that

(ac+ bd)(ab+ cd) ≤ 1

4(ac+ bd+ ab+ cd)2 =

1

4(a+ d)2(b+ c)2

≤ 1

43(a+ d+ b+ c)4 = 4,

and

(bc+ ad)(bd+ ac) ≤ 1

4(bc+ ad+ bd+ ac)2 =

1

4(a+ b)2(c+ d)2

≤ 1

43(a+ b+ c+ d)4 = 4.

This means that

max{(ac+ bd)(ab+ cd), (bc+ ad)(bd+ ac)} ≤ 4,

and hence, we conclude that

a2bc+ b2cd+ c2da+ d2ab ≤ 4,

as desired. The equality holds if and only if (a, b, c, d) equals (1, 1, 1, 1), or(2, 1, 1, 0) or (1, 1, 0, 2), or (0, 2, 1, 1), or (1, 0, 2, 1).

?F?

07.23. Let a, b, c, d be positive real numbers in the interval

[1

2, 2

]and abcd =

1. Find the maximum value of(a+

1

b

)(b+

1

c

)(c+

1

d

)(d+

1

a

).

(Middle Europe 2007)

Solution: Since1

2≤ a, b, c, d ≤ 2, we get

1

4≤ a

c,b

d≤ 4. This implies

(4a−c)(a−4c) ≤ 0 and (4b−d)(b−4c) ≤ 0, from which deduce that (a+c)2 ≤25

4ac and (b+ d)2 ≤ 25

4bd. Using these inequalities, we obtain

(ab+ 1)(bc+ 1) = b2ac+ b(a+ c) + 1 ≤ b2ac+5

2b√ac+ 1

=1

2

(2b√ac+ 1

) (b√ac+ 2

).

Similarly, we have

(cd+ 1)(da+ 1) ≤ 1

2

(2d√ac+ 1

) (d√ac+ 2

).

445

Page 447: Олимпиадад бэлтгэгчдэд тусламж (Тэнцэтгэл биш 1)

Multilplying these two inequality and using the remark that(a+

1

b

)(b+

1

c

)(c+

1

d

)(d+

1

a

)= (ab+ 1)(bc+ 1)(cd+ 1)(da+ 1),

we obtain(a+

1

b

)(b+

1

c

)(c+

1

d

)(d+

1

a

)≤

≤ 1

4

(2b√ac+ 1

) (2d√ac+ 1

) (b√ac+ 2

) (d√ac+ 2

).

Now, proceeding the same way as above, we have(2b√ac+ 1

) (2d√ac+ 1

)= 4abcd+ 2

√ac(b+ d) + 1

≤ 4abcd+ 5√abcd+ 1 = 10,

and (b√ac+ 2

) (d√ac+ 2

)= abcd+ 2

√ac(b+ d) + 4

≤ abcd+ 5√abcd+ 4 = 10.

Therefore (a+

1

b

)(b+

1

c

)(c+

1

d

)(d+

1

a

)≤ 25.

On the other hand, we can see that the inequality holds for a = b = 2, c = d =1

2. Thus, we conclude that the searched maximum is 25.

?F?

07.24. Let a1, a2, . . . , an be positive real numbers such that ai ≥1

ifor all

i = 1, 2, . . . , n. Prove the inequality

(a1 + 1)

(a2 +

1

2

)· · ·(an +

1

n

)≥ 2n

(n+ 1)!(1 + a1 + 2a2 + · · ·+ nan).

(Moldova 2007)

Solution: Notice for all x1, x2, . . . , xn ≥ 0, we have

(1 + x1)(1 + x2) · · · (1 + xn) ≥ 1 + x1 + x2 + · · ·+ xn

≥ 1 +2

n+ 1(x1 + x2 + · · ·+ xn).

In this inequality, we replace xi byiai − 1

2≥ 0 to get

(1 +

a1 − 1

2

)(1 +

2a2 − 1

2

)· · ·(

1 +nan − 1

2

)≥

≥ 1 +2

n+ 1

(a1 − 1

2+

2a2 − 1

2+ · · ·+ nan − 1

2

),

446

Page 448: Олимпиадад бэлтгэгчдэд тусламж (Тэнцэтгэл биш 1)

or equivalently,

n!

2n(a1 + 1)

(a2 +

1

2

)· · ·(an +

1

n

)≥ 1

n+ 1(1 + a1 + 2a2 + · · ·+ nan) .

Dividing each side of the last inequality byn!

2n, we get the desired result. It is

easy to see that the equality holds if and only if ai =1

ifor all i = 1, 2, . . . , n.

?F?

07.25. Let a1, a2, . . . , an ∈ [0, 1]. If S = a31 + a32 + · · ·+ a3n, then prove that

a12n+ 1 + S − a31

+a2

2n+ 1 + S − a32+ · · ·+ an

2n+ 1 + S − a3n≤ 1

3.

(Moldova 2007)

Solution: For every 1 ≤ i ≤ n, the AM-GM Inequality implies

2n+ 1 + S − a3i =∑j 6=i

(a3j + 2) + 3 ≥ 3∑j 6=i

aj + 3 ≥ 3(a1 + a2 + · · ·+ an).

Thereforeai

2n+ 1 + S − a3i≤ 1

3· aia1 + a2 + · · ·+ an

.

Now, adding up the n inequalities obtained from taking i = 1, 2, . . . , n, we getthe result. The equality holds if and only if a1 = a2 = · · · = an = 1.

?F?

07.26. Let a, b, c be positive real numbers such that

a+ b+ c ≥ 1

a+

1

b+

1

c.

Prove that

a+ b+ c ≥ 3

a+ b+ c+

2

abc.

(Peru 2007)

Solution: By the Cauchy Schwarz Inequality, we have

a+ b+ c ≥ 1

a+

1

b+

1

c≥ 9

a+ b+ c,

and thus

a+ b+ c ≥ 3.

Returning to the inequality in question, we see that it can be rewritten as

(a+ b+ c)2 ≥ 3 + 2

(1

ab+

1

bc+

1

ca

).

447

Page 449: Олимпиадад бэлтгэгчдэд тусламж (Тэнцэтгэл биш 1)

By the AM-GM Inequality, we get

1

ab+

1

bc+

1

ca≤ 1

3

(1

a+

1

b+

1

c

)2

≤ 1

3(a+ b+ c)2,

and so it is sufficient to show that

(a+ b+ c)2 ≥ 3 +2

3(a+ b+ c)2,

which is obviously true according to the fact that a+ b+ c ≥ 3. The equalityholds if and only if a = b = c = 1.

?F?

07.27. a, b, c, d are positive real numbers satisfying the following condition

1

a+

1

b+

1

c+

1

d= 4.

Prove that

3

√a3 + b3

2+

3

√b3 + c3

2+

3

√c3 + d3

2+

3

√d3 + a3

2≤ 2(a+ b+ c+ d)− 4.

(Poland 2007)

Solution: The key to solve this problem is to cancel the cube root, and wecan proceed as follows: Applying the AM-GM Inequality, we have

3

√a3 + b3

2=

4

(a+ b)2· a+ b

2· a+ b

2· 3

√a3 + b3

2

≤ 4

(a+ b)2·

(a+ b)3

8+

(a+ b)3

8+a3 + b3

23

=4

a+ b·

(a+ b)2

4+a2 − ab+ b2

23

=a2 + b2

a+ b.

Therefore, it suffices to prove that

a2 + b2

a+ b+b2 + c2

b+ c+c2 + d2

c+ d+d2 + a2

d+ a≤ 2(a+ b+ c+ d)− 4.

Because 2(a+b+c+d) = (a+b)+(b+c)+(c+d)+(d+a) and a+b− a2 + b2

a+ b=

2ab

a+ b=

21

a+

1

b

, the last inequality can be written as

11

a+

1

b

+1

1

b+

1

c

+1

1

c+

1

d

+1

1

d+

1

a

≥ 2.

448

Page 450: Олимпиадад бэлтгэгчдэд тусламж (Тэнцэтгэл биш 1)

By the Cauchy Schwarz Inequality, we have

11

a+

1

b

+1

1

b+

1

c

+1

1

c+

1

d

+1

1

d+

1

a

≥ 16(1

a+

1

b

)+

(1

b+

1

c

)+

(1

c+

1

d

)+

(1

d+

1

a

)=

81

a+

1

b+

1

c+

1

d

= 2.

The equality attains if and only if a = b = c = d = 1.?F?

07.28. Let x, y, z be nonnegative real numbers. Prove that the followinginequality holds

x3 + y3 + z3

3≥ xyz +

3

4|(x− y)(y − z)(z − x)| .

(Romania 2007)

Solution: With noting that

x3 + y3 + z3 − 3xyz =(x+ y + z)[(x− y)2 + (y − z)2 + (z − x)2]

3,

we can rewrite the original inequality as

(x+ y + z)[(x− y)2 + (y − z)2 + (z − x)2]

6≥ 3

4|(x− y)(y − z)(z − x)| .

Now, by the AM-GM Inequality, we have

2(x+ y + z) = (x+ y) + (y + z) + (z + x) ≥ |x− y|+ |y − z|+ |z − x|

≥ 3 3√|(x− y)(y − z)(z − x)|,

and

(x− y)2 + (y − z)2 + (z − x)2 ≥ 3 3√|(x− y)2(y − z)2(z − x)2|.

Multiplying these two inequalities and dividing each side of the resulting in-equality by 12, we get the result. It is easy to see that the equality holds ifand only if x = y = z.

?F?

07.29. For n ∈ N, n ≥ 2, determine

maxn∏i=1

(1− xi), for xi ∈ R+, 1 ≤ i ≤ n,n∑i=1

x2i = 1.

(Romania 2007)

449

Page 451: Олимпиадад бэлтгэгчдэд тусламж (Тэнцэтгэл биш 1)

First solution: Let us analyze E(x, y) = (1 − x)(1 − y), x, y ≥ 0, x2 +

y2 = k2, 0 < k ≤ 1. Take x = k sin θ, y = k cos θ, θ ∈[0,π

2

]. Now,

x + y = k(sin θ + cos θ) = k√

2 sin(θ +

π

4

)= k√

2 cos(θ − π

4

), and xy =

k2 sin θ cos θ =k2

2sin 2θ =

k2

2cos(

2θ − π

2

)=k2

2cos 2

(θ − π

4

), and hence,

xy = k2 cos2(θ − π

4

)− k2

2.

Take u = cos(θ − π

4

), so u ∈

[1√2, 1

], and then E(x, y) = E(u) = k2z2 −

k√

2z + 1 − k2

2. Its minimum value is reached for u0 =

1

k√

2≥ 1√

2, and

therefore

• for 1 <1

k√

2+

(1

k√

2− 1√

2

)=

2− kk√

2, i.e. k < 2

(√2− 1

), the maximum

value for E(u) is reached for u =1√2

, i.e. when θ ∈ {0, π}, which means x or

y being zero;

• for k ≥ 2(√

2− 1), the maximum value for E(u) is reached for u = 1, i.e.

when θ =π

4, which means x = y.

Now, for x2 + y2 + z2 = 1, there will be two, for which we have that x2 + y2 ≤2

3<(2(√

2− 1))2

(we have assumed them to be x and y), so first case applies

for x2+y2 = k2, k < 2(√

2− 1)

(case k = 0 is trivial), therefore the maximumvalue is reached when one of the three variables is zero.

So, when

n∑i=1

x2i = 1, E =n∏i=1

(1− xi)

takes maximum value when all but two variables (be them x, y) are zero, thenE = E(x, y) = (1 − x)(1 − y), with x2 + y2 = 1, and second case applies,

yielding maxE =

(1− 1√

2

)2

, for x = y =1√2

.

Second solution: We will prove by induction that for any x1, x2, . . . , xn, x21+

x22 + · · ·+ x2n = 1,

(1− x1)(1− x2) · · · (1− xn) ≤(

1− 1√2

)2

,

with equality (for example) when x1 = x2 =1√2, x3 = · · · = xn = 0. Indeed,

for n = 2, the inequality becomes

(1− x1)(1− x2) ≤(

1− 1√2

)2

,

450

Page 452: Олимпиадад бэлтгэгчдэд тусламж (Тэнцэтгэл биш 1)

where x21 + x22 = 1. By the AM-GM Inequality, we have

(1− x1)(1− x2) =(1− x21)(1− x22)(1 + x1)(1 + x2)

=x21x

22

(1 + x1)(1 + x2)≤ x21x

22(

1 +√x1x2

)2=

x1x2(1

√x1x2

+ 1

)2 ≤

x21 + x222(√

2

x21 + x22+ 1

)2 =

(1− 1√

2

)2

.

This proves our claim for n = 2. Now, suppose that the inequality holds forn ≥ 2 and let us prove it for n + 1. Due to symmetry, we may assume that

x1 ≤ x2 ≤ · · · ≤ xn+1, then x21+x22 ≤2

n+ 1. Note that

√x21 + x22, x3, . . . , xn+1

are n nonnegative numbers and(√

x21 + x22

)2+ x23 + · · ·+ x2n+1 = 1, so from

the inductive hypothesis, we find that(1−

√x21 + x22

)(1− x3) · · · (1− xn+1) ≤

(1− 1√

2

)2

.

According to this inequality, we see that it is enough to prove that

(1− x1)(1− x2) ≤ 1−√x21 + x22,

or equivalently,

x1x2 ≤ x1 + x2 −√x21 + x22.

Because x1 + x2 −√x21 + x22 =

2x1x2

x1 + x2 +√x21 + x22

, we can write the above

inequality as

x1x2

(x1 + x2 +

√x21 + x22 − 2

)≤ 0,

which is true because x1x2 ≥ 0 and

x1 + x2 +√x21 + x22 ≤

(√2 + 1

)√x21 + x22 ≤

(√2 + 1

)√ 2

n+ 1

≤(√

2 + 1)√2

3< 2.

This completes our proof.?F?

07.30. Let a, b, c be positive real numbers such that

1

a+ b+ 1+

1

b+ c+ 1+

1

c+ a+ 1≥ 1.

Show thata+ b+ c ≥ ab+ bc+ ca.

(Romania 2007)

451

Page 453: Олимпиадад бэлтгэгчдэд тусламж (Тэнцэтгэл биш 1)

First solution: By applying the Cauchy Schwarz Inequality, we obtain

(a+ b+ 1)(a+ b+ c2) ≥ (a+ b+ c)2,

and thus1

a+ b+ 1≤ c2 + a+ b

(a+ b+ c)2.

Now by summing cyclically, we obtain

1

a+ b+ 1+

1

b+ c+ 1+

1

c+ a+ 1≤ a2 + b2 + c2 + 2(a+ b+ c)

(a+ b+ c)2.

But from the condition, we can see that

a2 + b2 + c2 + 2(a+ b+ c) ≥ (a+ b+ c)2,

and thereforea+ b+ c ≥ ab+ bc+ ca.

We see that the equality occurs if and only if a = b = c = 1.

Second solution: We first observe that

2 ≥∑(

1− 1

a+ b+ 1

)=∑ a+ b

a+ b+ 1=∑ (a+ b)2

(a+ b)2 + a+ b.

Apply the Cauchy Schwarz Inequality to get

2 ≥∑ (a+ b)2

(a+ b)2 + a+ b≥

[∑(a+ b)

]2∑

[(a+ b)2 + a+ b]

=4∑

a2 + 8∑

ab

2∑

a2 + 2∑

ab+ 2∑

a,

and thus

2(

2∑

a2 + 2∑

ab+ 2∑

a)≥ 4

∑a2 + 8

∑ab.

From this inequality, we deduce that

a+ b+ c ≥ ab+ bc+ ca,

as claimed.?F?

07.31. For n ∈ N, n ≥ 2, ai, bi ∈ R, 1 ≤ i ≤ n, such that

n∑i=1

a2i = 1,n∑i=1

b2i = 1, andn∑i=1

aibi = 0,

prove that (n∑i=1

ai

)2

+

(n∑i=1

bi

)2

≤ n.

452

Page 454: Олимпиадад бэлтгэгчдэд тусламж (Тэнцэтгэл биш 1)

(C. Lupu and T. Lupu, Romania 2007)

First solution: Denote A =n∑i=1

ai and B =n∑i=1

bi. Applying the Cauchy

Schwarz Inequality, we have

(A2 +B2)2 =

[n∑i=1

(aiA+ biB)

]2≤ n

n∑i=1

(aiA+ biB)2

= n

n∑i=1

(a2iA2 + b2iB

2 + 2aibiAB)

= n

(A2

n∑i=1

a2i +B2n∑i=1

b2i + 2ABn∑i=1

aibi

)= n(A2 +B2).

Hence (A2 + B2)2 ≤ n(A2 + B2), and it follows that A2 + B2 ≤ n, so theinequality is proved.

Second solution: With the notations of the preceding solution, we have

0 ≤n∑i=1

(1− aiA− biB)2 =n∑i=1

(1 + a2iA2 + b2iB

2 − 2aiA− 2biB + 2aibiAB)

= n+A2n∑i=1

a2i +B2n∑i=1

b2i − 2An∑i=1

ai − 2Bn∑i=1

bi + 2ABn∑i=1

aibi

= n+A2 +B2 − 2A2 − 2B2 + 0 = n− (A2 +B2).

and the inequality follows directly from this identity.?F?

07.32. Positive real numbers a, b, c satisfy a+ b+ c = 1. Show that

1

ab+ 2c2 + 2c+

1

bc+ 2a2 + 2a+

1

ca+ 2b2 + 2b≥ 1

ab+ bc+ ca.

(Turkey 2007)

First solution: Applying the AM-GM Inequality, we have

ab+ 2c2 + 2c = ab+ 2c2 + 2c(a+ b+ c) = (2c+ a)(2c+ b)

≤ [b(2c+ a) + a(2c+ b)]2

4ab=

(ab+ bc+ ca)2

ab.

Therefore1

ab+ 2c2 + 2c≥ ab

(ab+ bc+ ca)2,

and it follows that∑ 1

ab+ 2c2 + 2c≥ ab+ bc+ ca

(ab+ bc+ ca)2=

1

ab+ bc+ ca.

453

Page 455: Олимпиадад бэлтгэгчдэд тусламж (Тэнцэтгэл биш 1)

Note that the equality occurs iff a = b = c =1

3.

Second solution: Similar to the preceding solution, we see that ab+2c2+2c =(2c+ a)(2c+ b), so the original inequality is equivalent to

1

(2a+ b)(2a+ c)+

1

(2b+ c)(2b+ a)+

1

(2c+ a)(2c+ b)≥ 1

ab+ bc+ ca.

Now, setting a =1

x, b =

1

y, z =

1

z, then we have

1

(2a+ b)(2a+ c)=

x2yz

(x+ 2y)(x+ 2z), and

1

ab+ bc+ ca=

xyz

x+ y + z,

and thus, the above inequality is equivalent to

x

(x+ 2y)(x+ 2z)+

y

(y + 2z)(y + 2x)+

z

(z + 2x)(z + 2y)≥ 1

x+ y + z.

By the Cauchy Schwarz Inequality, we get∑ x

(x+ 2y)(x+ 2z)=∑ x2

x(x+ 2y)(x+ 2z)≥ (x+ y + z)2∑

x(x+ 2y)(x+ 2z).

Therefore, the last inequality is deduced from

(x+ y + z)3 ≥ x(x+ 2y)(x+ 2z) + y(y + 2z)(y + 2x) + z(z + 2x)(z + 2y).

After some small computations, one can see that it is equivalent to

xy(x+ y) + yz(y + z) + zx(z + x) ≥ 6xyz.

This is obviously true by the AM-GM Inequality, so the inequality is proved.?F?

07.33. Let a, b, c be positive real numbers such that abc ≥ 1. Prove that(a+

1

a+ 1

)(b+

1

b+ 1

)(c+

1

c+ 1

)≥ 27

8,

and

27(a3+a2+a+1)(b3+b2+b+1)(c3+c2+c+1) ≥ 64(a2+a+1)(b2+b+1)(c2+c+1).

(Ukraine 2007)

Solution: For any positive number x, we have

x+1

x+ 1− 3

4(x+ 1) =

(x− 1)2

4(x+ 1)≥ 0.

This implies that(a+

1

a+ 1

)(b+

1

b+ 1

)(c+

1

c+ 1

)≥ 27

64(a+ 1)(b+ 1)(c+ 1).

454

Page 456: Олимпиадад бэлтгэгчдэд тусламж (Тэнцэтгэл биш 1)

On the other hand, by the Holder’s Inequality, we have

(a+ 1)(b+ 1)(c+ 1) ≥(

3√abc+ 1

)3= 8.

Combining these two inequalities, we get(a+

1

a+ 1

)(b+

1

b+ 1

)(c+

1

c+ 1

)≥ 27

8.

This proves the first inequality in the required question. Now, let us prove thesecond. Notice that a3 + a2 + a + 1 = (a2 + 1)(a + 1), so we can rewrite theinequality as

(a2 + 1)(b2 + 1)(c2 + 1)

(a2 + a+ 1)(b2 + b+ 1)(c2 + c+ 1)· (a+ 1)(b+ 1)(c+ 1) ≥ 64

27.

Now, by the AM-GM Inequality, we have

(a2 + 1)(b2 + 1)(c2 + 1)

(a2 + a+ 1)(b2 + b+ 1)(c2 + c+ 1)≥

≥ (a2 + 1)(b2 + 1)(c2 + 1)(a2 +

a2 + 1

2+ 1

)(b2 +

b2 + 1

2+ 1

)(c2 +

c2 + 1

2+ 1

) =8

27.

So, it is enough to check that

(a+ 1)(b+ 1)(c+ 1) ≥ 8,

which we have been proved above. The proof is completed. It is easy to seethat the equality (in both inequalities) holds iff a = b = c = 1.

?F?

07.34. Show that for any real numbers a, b, c, then

(a2 + b2)2 ≥ (a+ b+ c)(b+ c− a)(c+ a− b)(a+ b− c).

(United Kingdom 2007)

Solution: From the AM-GM Inequality, we get

(a+ b+ c)(b+ c− a)(c+ a− b)(a+ b− c) =[(a+ b)2 − c2

] [c2 − (a− b)2

]≤ 1

4

[(a+ b)2 − c2 + c2 − (a− b)2

]2= 4a2b2 ≤ (a2 + b2)2,

as desired. The equality holds if and only if (a+ b)2 − c2 = c2 − (a− b)2 anda2 = b2, i.e. when 2a2 = 2b2 = c2.

?F?

07.35. Given a triangle ABC. Determine the minimum value of

cos2A

2cos2

B

2

cos2C

2

+cos2

B

2cos2

C

2

cos2A

2

+cos2

C

2cos2

A

2

cos2B

2

.

455

Page 457: Олимпиадад бэлтгэгчдэд тусламж (Тэнцэтгэл биш 1)

(Vietnam 2007)

Solution: Let x = tanA

2, y = tan

B

2, z = tan

C

2, then x, y, z > 0 and xy +

yz + zx = 1. By this substitution, we have

cos2A

2cos2

B

2

cos2C

2

=

1

x2 + 1· 1

y2 + 11

z2 + 1

=x2 + 1

(y2 + 1)(z2 + 1)

=(x+ y)(x+ z)

(y + z)(y + z)(z + x)(z + y)=

1

(y + z)2.

So, the desired expression is equal to

1

(y + z)2+

1

(z + x)2+

1

(x+ y)2.

On the other hand, it is known that (see the Iran 1996 problem)

1

(y + z)2+

1

(z + x)2+

1

(x+ y)2≥ 9

4(xy + yz + zx),

with equality iff x = y = z. Using this inequality with noting that xy + yz +zx = 1, we get

cos2A

2cos2

B

2

cos2C

2

+cos2

B

2cos2

C

2

cos2A

2

+cos2

C

2cos2

A

2

cos2B

2

≥ 9

4(xy + yz + zx)=

9

4,

with equality iff tanA

2= tan

B

2= tan

C

2, i.e. when ABC is an equilateral

triangle. From this, we conclude that the searched minimum is9

4.

?F?

08.1. Let a, b, c be real numbers such that a2 + b2 + c2 = 3. Prove that

a2

2 + b+ c2+

b2

2 + c+ a2+

c2

2 + a+ b2≥ (a+ b+ c)2

12.

(Baltic Way 2008)

Solution: From the given hypothesis a2+b2+c2 = 3, we find that |a| , |b| , |c| ≤√3 < 2, and hence 2 + b+ c2 > 0, 2 + c+a2 > 0, 2 +a+ b2 > 0. Now, applying

the Cauchy Schwarz Inequality, we have

a2

2 + b+ c2+

b2

2 + c+ a2+

c2

2 + a+ b2≥ (a+ b+ c)2

6 + (a+ b+ c) + (a2 + b2 + c2)

=(a+ b+ c)2

9 + a+ b+ c

≥ (a+ b+ c)2

9 +√

3(a2 + b2 + c2)

=(a+ b+ c)2

12,

456

Page 458: Олимпиадад бэлтгэгчдэд тусламж (Тэнцэтгэл биш 1)

as desired. Note that the equality holds if and only if a = b = c = 1.?F?

08.2. Suppose that a, b, c are positive real numbers with a2 + b2 + c2 = 1.Prove that

a5 + b5

ab(a+ b)+

b5 + c5

bc(b+ c)+

c5 + a5

ca(a+ b)≥ 3(ab+ bc+ ca)− 2.

(Bosnia 2008)

Solution: For any positive numbers x, y, we have (x3 − y3)(x2 − y2) ≥ 0,

implying x5 + y5 ≥ x2y2(x+ y), orx5 + y5

xy(x+ y)≥ xy. Accordingly, we find that

the left hand side of the original inequality is not smaller than ab+ bc+ ca. Itsuffices to prove that

ab+ bc+ ca ≥ 3(ab+ bc+ ca)− 2, or 2 ≥ 2(ab+ bc+ ca),

which is true according to the well-known inequality a2 +b2 +c2 ≥ ab+bc+caand the hypothesis x2 + y2 + z2 = 1. The equality holds if and only if a = b =

c =1√3.

?F?

08.3. Let x, y, z be real numbers. Show that the following inequality holds

x2 + y2 + z2 − xy − yz − zx ≥ max

{3(x− y)2

4,3(y − z)2

4,3(z − x)2

4

}.

(Bosnia 2008)

Solution: Without loss of generality, we may assume that (x−z)2 = max{(x−y)2, (y − z)2, (z − x)2}. From this assumption, applying the Cauchy SchwarzInequality, we get

2(x2 + y2 + z2 − xy − yz − zx) = (x− y)2 + (y − z)2 + (x− z)2

≥ 1

2[(x− y) + (y − z)]2 + (x− z)2

=3

2(x− z)2,

and hence, we deduce that

x2+y2+z2−xy−yz−zx ≥ 3

4(x−z)2 = max

{3(x− y)2

4,3(y − z)2

4,3(z − x)2

4

},

as desired.?F?

08.4. Let a, b, c be positive real numbers. Prove that(1 +

4a

b+ c

)(1 +

4b

a+ c

)(1 +

4c

a+ b

)> 25.

457

Page 459: Олимпиадад бэлтгэгчдэд тусламж (Тэнцэтгэл биш 1)

(Bosnia 2008)

Solution: By expanding, we can rewrite our inequality as

4(a3 + b3 + c3) + 28abc > 4a2(b+ c) + 4b2(c+ a) + 4c2(a+ b),

or

4 [a(a− b)(a− c) + b(b− c)(b− a) + c(c− a)(c− b)] + 16abc > 0.

This is true since 16abc > 0 and∑

a(a − b)(a − c) ≥ 0 from the Schur’s

Inequality (applied for third degree). Our proof is completed.?F?

08.5. Let x, y, z be real numbers such that x+y+z = xy+yz+zx. Determinethe least value of the following expression

P =x

x2 + 1+

y

y2 + 1+

z

z2 + 1.

(Brazil 2008)

Solution: We claim that the minimum value of P is −1

2which attains when

(x, y, z) is a permutation of (−1,−1, 1). We will prove this claim by provingthat

2x

x2 + 1+ 1 +

2y

y2 + 1+ 1 ≥ 1− 2z

z2 + 1,

or equivalently,(x+ 1)2

x2 + 1+

(y + 1)2

y2 + 1≥ (z − 1)2

z2 + 1.

Now, from the given hypothesis, we have z(x+y−1) = x+y−xy. If x+y = 1,

then it follows that xy = x+y = 1, which is impossible since xy ≤(x+ y

2

)2

=

1

4, so we must have x+ y 6= 1, and hence

z =x+ y − xyx+ y − 1

.

Replacing this into the above inequality, we can rewrite it as

(x+ 1)2

x2 + 1+

(y + 1)2

y2 + 1≥

(x+ y − xyx+ y − 1

− 1

)2

(x+ y − xyx+ y − 1

)2

+ 1

,

or(x+ 1)2

x2 + 1+

(y + 1)2

y2 + 1≥ (1− xy)2

(x+ y − xy)2 + (x+ y − 1)2.

458

Page 460: Олимпиадад бэлтгэгчдэд тусламж (Тэнцэтгэл биш 1)

If x = y = 1, this inequality is trivial. Alternatively, if (x− 1)2 + (y− 1)2 > 0,we make use of the Cauchy Schwarz Inequality and get

(x+ 1)2

x2 + 1+

(y + 1)2

y2 + 1≥ [(1 + x)(1− y) + (1− x)(1 + y)]2

(1− y)2(1 + x2) + (1− x)2(1 + y2)

=4(1− xy)2

(1− y)2(1 + x2) + (1− x)2(1 + y2).

Therefore, we can see that the above inequality is deduced from

4(x+ y − xy)2 + 4(x+ y − 1)2 ≥ (1− y)2(1 + x2) + (1− x)2(1 + y2).

By some easy computations, we can rewrite it as

f(x) = (y2 − 3y + 3)x2 − (3y2 − 8y + 3)x+ 3y2 − 3y + 1 ≥ 0.

Note that f(x) is a quadratic polynomial of x with the highest coefficient ispositive. On the other hand, the discrimimant of f(x) is

∆f = (3y2 − 8y + 3)2 − 4(y2 − 3y + 3)(3y2 − 3y + 1) = −3(y2 − 1)2 ≤ 0.

So, it is clear that f(x) ≥ 0, and our claim is proved. Or in the other words,

we have proved that the minimum value of the expression P is −1

2.

?F?

08.6. Let a, b, c be positive real numbers such that a+ b+ c = 1. Prove that

a− bca+ bc

+b− cab+ ca

+c− abc+ ab

≤ 3

2.

(Canada 2008)

Solution: By the Cauchy Schwarz Inequality, we have

a− bca+ bc

+b− cab+ ca

+c− abc+ ab

= 3− 2

(bc

a+ bc+

ca

b+ ca+

ab

c+ ab

)≤ 3− 2(ab+ bc+ ca)2

3abc+ b2c2 + c2a2 + a2b2

= 3− 2(ab+ bc+ ca)2

3abc(a+ b+ c) + b2c2 + c2a2 + a2b2

= 3− 2(ab+ bc+ ca)2

abc(a+ b+ c) + (ab+ bc+ ca)2

≤ 3− 2(ab+ bc+ ca)2

(ab+ bc+ ca)2

3+ (ab+ bc+ ca)2

=3

2.

Note that the equality holds if and only if a = b = c =1

3.

?F?

459

Page 461: Олимпиадад бэлтгэгчдэд тусламж (Тэнцэтгэл биш 1)

08.7. Let a, b, c be nonnegative real numbers such that a + b + c = 1. Provethat √

a+(b− c)2

4+√b+√c ≤√

3.

(China 2008)

Solution: Set x =√bc, then we have

√b+√c =

√b+ c+ 2

√bc =

√1− a+ 2

√bc =

√1− a+ 2x,

and √a+

(b− c)24

=

√a+

(b+ c)2 − 4bc

4=

1

2

√(1 + a)2 − 4x2.

Therefore, the inequality in question now rewrites as√(1 + a)2 − 4x2 + 2

√1− a+ 2x ≤ 2

√3.

By the Cauchy Schwarz Inequality, we have√(1 + a)2 − 4x2 + 2

√1− a+ 2x ≤

√3 [(1 + a)2 − 4x2 + 2(1− a+ 2x)],

and hence it suffices to prove that

(1 + a)2 − 4x2 + 2(1− a+ 2x) ≤ 4, or equivalently, a2 − 4x2 + 4x− 1 ≤ 0.

This can be also rewritten as

(1− 2x− a)(1 + a− 2x) ≥ 0,

which is obviously true since

1 + a− 2x ≥ 1− 2x− a = 1− 2√bc− a ≥ 1− (b+ c)− a = 0.

It is easy to see that the equality holds if and only if a = b = c =1

3.

?F?

08.8. Let x, y, z be positive numbers. Find the minimal value of

(a)x2 + y2 + z2

xy + yz;

(b)x2 + y2 + 2z2

xy + yz.

(Croatia 2008)

Solution: (a) Using the identity

x2 + y2 + z2

xy + yz−√

2 =

(x−√

2

2y

)2

+

(z −√

2

2y

)2

xy + yz≥ 0,

460

Page 462: Олимпиадад бэлтгэгчдэд тусламж (Тэнцэтгэл биш 1)

we conclude that the expressionx2 + y2 + z2

xy + yzhas minimal value

√2 when

x = z =

√2

2y.

(b) Similarly, using the identity

x2 + y2 + 2z2

xy + yz−√

8

3=

(x−

√2

3y

)2

+ 2

(z −

√1

6y

)2

xy + yz≥ 0,

we conclude that the expressionx2 + y2 + 2z2

xy + yzhas minimal value

√8

3when

x =

√2

3y and z =

√1

6y.

?F?

08.9. Determine the smallest constant C such that the following inequality

1 + (x+ y)2 ≤ C(1 + x2)(1 + y2)

holds for any real numbers x, y.(Germany 2008)

First solution: Let x = y =

√2

2, we get C ≥ 4

3. We will prove that the

constant k =4

3is our answer, i.e.

4

3(1 + x2)(1 + y2) ≥ (x+ y)2 + 1.

Expanding, we can rewrite this inequality as follows

f(x) = (4y2 + 1)x2 − 6xy + y2 + 1 ≥ 0.

We see that f(x) is a quadratic function of x with the highest coefficient bepositive. In addition, its discriminant is

∆′f = 9y2 − (y2 + 1)(4y2 + 1) = −(1− 2y2)2 ≤ 0.

This means that f(x) is nonnegative for any real number x. So, our proof iscompleted.

Second solution: Similarly, we need to prove the inequality

4

3(1 + x2)(1 + y2) ≥ (x+ y)2 + 1.

By the Cauchy Schwarz Inequality, we get

4(1 + x2)(1 + y2) = 3 + 2(x2 + y2) + (1 + 2x2)(2y2 + 1)

≥ 3 + (x+ y)2 +(√

2x+√

2y)2

= 3[(x+ y)2 + 1].

461

Page 463: Олимпиадад бэлтгэгчдэд тусламж (Тэнцэтгэл биш 1)

Dividing both sides of this inequality by 3, we get the desired inequality.?F?

08.10. Prove that if a1, a2, . . . , an are positive integers, then the followinginequality (

a21 + a22 + · · ·+ a2na1 + a2 + · · ·+ an

) knt

≥ a1a2 · · · an

holds for k = max {a1, a2, . . . , an} and t = min {a1, a2, . . . , an} . When do wehave the equality?

(Greece 2008)

Solution: By the Cauchy Schwarz Inequality, we have

n(a21 + a22 + · · ·+ a2n) ≥ (a1 + a2 + · · ·+ an)2,

and hencea21 + a22 + · · ·+ a2na1 + a2 + · · ·+ an

≥ a1 + a2 + · · ·+ ann

.

Since ai is positive integer, we have ai ≥ 1 for all i = 1, 2, . . . , n. Therefore

a1 + a2 + · · ·+ ann

≥ 1.

Also, since the function f(x) = ax is increasing for all a ≥ 1 andk

t≥ 1, we

have (a21 + a22 + · · ·+ a2na1 + a2 + · · ·+ an

) knt

≥(a1 + a2 + · · ·+ an

n

) knt

≥(a1 + a2 + · · ·+ an

n

)n≥ a1a2 · · · an,

as desired. Note that the equality holds if and only if a1 = a2 = · · · = an.?F?

08.11. Let x, y, z be positive real numbers such that x2 + y2 + z2 = 3. Provethe inequality

3

2<

1 + y2

2 + x+

1 + z2

2 + y+

1 + x2

2 + z< 3.

(Greece 2008)

Solution: From the given hypothesis, we have 0 < x, y, z <√

3. This yields

1 + y2

2 + x+

1 + z2

2 + y+

1 + x2

2 + z>

1 + y2

2 + 2+

1 + z2

2 + 2+

1 + x2

2 + 2

=x2 + y2 + z2 + 3

4=

3

2,

462

Page 464: Олимпиадад бэлтгэгчдэд тусламж (Тэнцэтгэл биш 1)

and

1 + y2

2 + x+

1 + z2

2 + y+

1 + x2

2 + z<

1 + y2

2+

1 + z2

2+

1 + x2

2

=x2 + y2 + z2 + 3

2= 3.

Therefore, we have

3

2<

1 + y2

2 + x+

1 + z2

2 + y+

1 + x2

2 + z< 3,

as claimed.

Remark: Actually, one can prove that the stronger inequality holds with thesame condition

2 ≤ 1 + y2

2 + x+

1 + z2

2 + y+

1 + x2

2 + z<

9

2−√

3.

Indeed, by the AM-GM Inequality and Cauchy Schwarz Inequality, we have

1 + y2

2 + x+

1 + z2

2 + y+

1 + x2

2 + z≥ 1 + y2

2 +x2 + 1

2

+1 + z2

2 + y2+12

+1 + x2

2 +z2 + 1

2

= 2

(y2 + 1

x2 + 5+z2 + 1

y2 + 5+x2 + 1

z2 + 5

)≥ 2(x2 + y2 + z2 + 3)2∑

(x2 + 5)(y2 + 1)

=72

15 + 6(x2 + y2 + z2) + (x2y2 + y2z2 + z2x2)

≥ 72

15 + 18 +(x2 + y2 + z2)2

3

= 2.

This proves the left inequality. For the right inequality, we will make use ofthe following inequality for all t ∈

(0,√

3),

1

t+ 2− 1

2+

2−√

3

2√

3t2 =

2−√

3

2√

3t2 − t

2(t+ 2)

=t

2(t+ 2)

[2−√

3√3

t(t+ 2)− 1

]

<t

2(t+ 2)

[2−√

3√3·√

3(√

3 + 2)− 1

]= 0,

that is1

t+ 2<

1

2− 2−

√3

2√

3t2.

463

Page 465: Олимпиадад бэлтгэгчдэд тусламж (Тэнцэтгэл биш 1)

According to this inequality, we have

1 + y2

2 + x+

1 + z2

2 + y+

1 + x2

2 + z<∑

(y2 + 1)

(1

2− 2−

√3

2√

3x2

)

=9

2−√

3− 2−√

3

2√

3(x2y2 + y2z2 + z2x2) <

9

2−√

3.

?F?

08.12. Prove that for any positive real numbers a, b, c, d, we have the followinginequality

(a− b)(a− c)a+ b+ c

+(b− c)(b− d)

b+ c+ d+

(c− d)(c− a)

c+ d+ a+

(d− a)(d− b)d+ a+ b

≥ 0.

(Darij Grinberg, IMO Shortlist 2008)

First solution: We denote with P (a, b, c, d) the left hand side of the originalinequality. The key to solve this inequality is to notice that, without loss ofgenerality, we can assume the following property (a− c)(b− d) ≥ 0. Indeed, if(a−c)(b−d) ≤ 0, then we can let a1 = b, b1 = c, c1 = d, d1 = a, and we will haveP (a1, b1, c1, d1) = P (a, b, c, d), and also (a1−c1)(b1−d1) = −(a−c)(b−d) ≥ 0.This is contradiction so what we have assumed is true. Now, we see that

(a− b)(a− c)a+ b+ c

+(c− d)(c− a)

c+ d+ a=

(a− c)2

a+ b+ c− (a+ 2c)(a− c)(b− d)

(a+ b+ c)(a+ c+ d),

and

(b− c)(b− d)

b+ c+ d+

(d− a)(d− b)d+ a+ b

=(b− d)2

b+ c+ d+

(b+ 2d)(a− c)(b− d)

(b+ c+ d)(d+ a+ b)

≥ (b− d)2

b+ c+ d.

Therefore, the original inequality is just a trivial corollary of

(a− c)2

a+ b+ c+

(b− d)2

b+ c+ d≥ (a+ 2c)(a− c)(b− d)

(a+ b+ c)(a+ c+ d).

According to the AM-GM Inequality, we have

(a− c)2

a+ b+ c+

(b− d)2

b+ c+ d≥ 2(a− c)(b− d)√

(a+ b+ c)(b+ c+ d).

Hence, it suffices to prove that

2(a+ c+ d)

√a+ b+ c

b+ c+ d≥ a+ 2c.

If a ≥ d then it is clear that

√a+ b+ c

b+ c+ d≥ 1 and 2(a+ c+ d) ≥ a+ 2c, so the

above inequality is trivial. Alternatively, if d ≥ a, then we can easily check

that

√a+ b+ c

b+ c+ d≥√a+ c

c+ d, so we only need to prove

2(a+ c+ d)√a+ c ≥ (a+ 2c)

√c+ d.

464

Page 466: Олимпиадад бэлтгэгчдэд тусламж (Тэнцэтгэл биш 1)

This can be proved as follows

2(a+ c+ d)√a+ c = 2

√a+ c+ d

√(a+ c+ d)(a+ c)

≥ 2(a+ c)√a+ c+ d ≥ 2(a+ c)

√c+ d

≥ (a+ 2c)√c+ d.

It is easy to see that the equality holds iff a = c and b = d.

Second solution: Denote

A =(a− b)(a− c)a+ b+ c

, B =(b− c)(b− d)

b+ c+ d, C =

(c− a)(c− d)

c+ d+ a, D =

(d− a)(d− b)d+ a+ b

.

It is easy to see that 2A = A′ +A′′, where

A′ =(a− c)2

a+ b+ cand A′′ =

(a− c)(a+ c− 2b)

a+ b+ c.

Similarly, we have 2B = B′ + B′′, 2C = C ′ + C ′′ and 2D = D′ + D′′. Byapplying the Cauchy Schwarz Inequality, we have

A′ +B′ + C ′ +D′ =(a− c)2

a+ b+ c+

(b− d)2

b+ c+ d+

(c− a)2

c+ d+ a+

(d− b)2

d+ a+ b

≥ (|a− c|+ |b− d|+ |c− a|+ |d− b|)2

(a+ b+ c) + (b+ c+ d) + (c+ d+ a) + (d+ a+ b)

=4(|a− c|+ |b− d|)2

3(a+ b+ c+ d).

On the other hand, according to the identities

A′′+C ′′ =3(a− c)(d− b)(a+ c)

(a+ b+ c+ d)(a+ c) + bd, B′′+C ′′ =

3(a− c)(b− d)(b+ d)

(a+ b+ c+ d)(b+ d) + ac,

we find that

A′′ +B′′ + C ′′ +D′′ =

= 3(a− c)(b− d)

[b+ d

(a+ b+ c+ d)(b+ d) + ac− a+ c

(a+ b+ c+ d)(a+ c) + bd

]=

3(a− c)(b− d)[bd(b+ d)− ac(a+ c)]

[(a+ b+ c+ d)(b+ d) + ac][(a+ b+ c+ d)(a+ c) + bd].

Furthermore,

[(a+ b+ c+ d)(b+ d) + ac][(a+ b+ c+ d)(a+ c) + bd] >

> [ac(a+ c) + bd(b+ d)](a+ b+ c+ d)

> |ac(a+ c)− bd(b+ d)|(a+ b+ c+ d).

And so, we deduce that

A′′ +B′′ + C ′′ +D′′ ≥ −3|a− c||b− d|a+ b+ c+ d

≥ −3(|a− c|+ |b− d|)2

4(a+ b+ c+ d).

465

Page 467: Олимпиадад бэлтгэгчдэд тусламж (Тэнцэтгэл биш 1)

From this, we obtain

2(A+B + C +D) ≥ 4(|a− c|+ |b− d|)2

3(a+ b+ c+ d)− 3(|a− c|+ |b− d|)2

4(a+ b+ c+ d)

=7(|a− c|+ |b− d|)2

12(a+ b+ c+ d)≥ 0,

and the conclusion follows.?F?

08.13. (i) If x, y, z are three real numbers, all different from 1, such thatxyz = 1, then prove that

x2

(x− 1)2+

y2

(y − 1)2+

z2

(z − 1)2≥ 1.

(ii) Prove that equality is achieved for infinitely many triples of rational num-bers x, y, z.

(IMO 2008)

First solution: (i) Since xyz = 1, there exist three nonnegative real numbers

a, b, c, such that x =a

b, y =

b

c, z =

c

a(clearly, a, b, c are distinct numbers since

x, y, z 6= 1), the inequality becomes

a2

(a− b)2+

b2

(b− c)2+

c2

(c− a)2≥ 1.

By the Cauchy Schwarz Inequality, we get[∑ a2

(a− b)2

] [∑(a− b)2(a− c)2

]≥[∑

a(a− c)]2

=(∑

a2 −∑

ab)2.

Moreover, we have∑(a− b)2(a− c)2 =

∑(a− b)2(a− c)2 + 2

∑(a− b)(a− c) · (b− c)(b− a)

=[∑

(a− b)(a− c)]2

=(∑

a2 −∑

ab)2.

This proves (i).

(ii) As we have seen in the proof of (i), the equality holds if and only if

a

a− b(a− b)(a− c)

=

b

b− c(b− c)(b− a)

=

c

c− a(c− a)(c− b)

.

This can be rewritten as

a

(a− b)2(a− c)=

b

(b− c)2(b− a)=

c

(c− a)2(c− b),

orb

a+c

b+a

c= 3.

466

Page 468: Олимпиадад бэлтгэгчдэд тусламж (Тэнцэтгэл биш 1)

Returning to the notations with x, y, z, this is equivalent with

1

x+

1

y+

1

z= 3.

Now, note that in the above equation we can choose

(x, y, z) =

(n

(n+ 1)2,−n(n+ 1),−n+ 1

n2

),

where n is an arbitrary rational number. This shows that the equality inthe inequality in question is achieved for infinitely many triples of rationalnumbers x, y, z.

Second solution: We provides the second way to prove (i). Denote a =x

x− 1, b =

y

y − 1, c =

z

z − 1, then we can find that x =

a

a− 1, y =

b

b− 1, z =

z

z − 1. And since xyz = 1, it follows that abc = (a − 1)(b − 1)(c − 1), or

a+ b+ c− ab− bc− ca = 1. According to the substitution, we need to provea2 + b2 + c2 ≥ 1. Observe that 1 = 2− 1 = 2(a+ b+ c− ab− bc− ca)− 1, thisinequality is equivalent to

a2 + b2 + c2 ≥ 2(a+ b+ c− ab− bc− ca)− 1.

The last one can be simplified into a complete square (a + b + c − 1)2 ≥ 1.This proves our inequality.

Third solution: We give the third proof for (i). Let a = 3√x, b = 3

√y, c = 3

√z,

then we havea2

bc=

3√x2

3√yz

=x

3√xyz

= x,

and similarly,b2

ca= y,

c2

ab= z. Therefore, using this substitution, we can write

our inequality in the form

a4

(a2 − bc)2+

b4

(b2 − ca)2+

c4

(c2 − ab)2≥ 1.

Applying the Cauchy Schwarz Inequality, we have

a4

(a2 − bc)2+

b4

(b2 − ca)2+

c4

(c2 − ab)2≥ (a2 + b2 + c2)2

(a2 − bc)2 + (b2 − ca)2 + (c2 − ab)2.

On the other hand,

(a2 + b2 + c2)2 − [(a2 − bc)2 + (b2 − ca)2 + (c2 − ab)2] = (ab+ bc+ ca)2 ≥ 0.

Therefore, from the above estimation, the result follows immediately.?F?

08.14. Let n ≥ 3 is an integer and let x1, x2, . . . , xn be real numbers suchthat xi > 1 for all i. Prove the following inequality

x1x2x3 − 1

+ · · ·+ xn−1xnx1 − 1

+xnx1x2 − 1

≥ 4n.

467

Page 469: Олимпиадад бэлтгэгчдэд тусламж (Тэнцэтгэл биш 1)

(Indonesia 2008)

Solution: From the AM-GM Inequality, we get

x1x2x3 − 1

+ · · ·+ xn−1xnx1 − 1

+xnx1x2 − 1

≥ x1x2

x23 + 4

4− 1

+ · · ·+ xn−1xn

x21 + 4

4− 1

+xnx1

x22 + 4

4− 1

= 4

(x1x2x23

+ · · ·+ xn−1xnx21

+xnx1x22

)≥ 4n n

√x1x2x23· · · xn−1xn

x21· xnx1x22

= 4n,

as claimed. Note that the equality holds if and only if x1 = x2 = · · · = xn = 2.?F?

08.15. Let a, b, c be nonnegative real numbers, from which at least two arenonzero and satisfying the condition ab+ bc+ ca = 1. Prove that√

a3 + a+√b3 + b+

√c3 + c ≥ 2

√a+ b+ c.

(Iran 2008)

First solution: From the Holder’s Inequality, we have(∑√a3 + a

)2(∑ a2

a2 + 1

)≥ (a+ b+ c)3,

hence it suffices to prove that

(a+ b+ c)2 ≥ 4

(a2

a2 + 1+

b2

b2 + 1+

c2

c2 + 1

),

or equivalently,

(a+ b+ c)2

ab+ bc+ ca≥ 4

[a2

(a+ b)(a+ c)+

b2

(b+ c)(b+ a)+

c2

(c+ a)(c+ b)

].

This can be rewritten into

(a+ b+ c)2

ab+ bc+ ca≥

4[a2(b+ c) + b2(c+ a) + c2(a+ b)

](a+ b)(b+ c)(c+ a)

,

or in other words,

a2 + b2 + c2

ab+ bc+ ca+

8abc

(a+ b)(b+ c)(c+ a)≥ 2.

Because the inequality is symmetric, we can suppose without loss of generality

that c = min{a, b, c}. Then, applying the evident inequalityA

B≥ A+ C

B + C∀C ≥ 0, A ≥ B > 0 with A = a2 + b2 + c2, B = ab + bc + ca and C = c2, weget

a2 + b2 + c2

ab+ bc+ ca≥ a2 + b2 + c2 + c2

ab+ bc+ ca+ c2=a2 + b2 + 2c2

(a+ c)(b+ c).

468

Page 470: Олимпиадад бэлтгэгчдэд тусламж (Тэнцэтгэл биш 1)

So, it suffices proving that

a2 + b2 + 2c2

(a+ c)(b+ c)+

8abc

(a+ b)(b+ c)(c+ a)≥ 2.

After some easy computations, we can see that this inequality is equivalent to

(a− b)2(a+ b− 2c)

(a+ b)(b+ c)(c+ a)≥ 0,

and of course, it is true since c = min{a, b, c}. Note that the equality holds if

and only if a = b = c =1√3

or (a, b, c) is a cyclic permutation of (1, 1, ).

Second solution: Since a3 + a = a3 + a(ab+ bc+ ca) = a2(a+ b+ c) + abc,the original inequality can be rewritten as∑√(

a√a+ b+ c

)2+(√

abc)2≥ 2√

(a+ b+ c)(ab+ bc+ ca).

By applying the Minkowsky’s Inequality, we have∑√(a√a+ b+ c

)2+(√

abc)2≥√(∑

a√a+ b+ c

)2+(∑√

abc)2

=√

(a+ b+ c)3 + 9abc.

Therefore, the inequality is deduced from

(a+ b+ c)3 + 9abc ≥ 4(a+ b+ c)(ab+ bc+ ca),

and of course, this is true since it is the Schur’s Inequality applied for thirddegree.

?F?

08.16. Let x, y, z be positive real numbers such that x+y+z = 3. Prove that

x3

y3 + 8+

y3

z3 + 8+

z3

x3 + 8≥ 1

9+

2

27(xy + xz + yz).

(Iran 2008)

First solution: Applying the Cauchy Schwarz Inequality, the well-knowninequality (a+b+c)2 ≥ 3(ab+bc+ca) and the AM-GM Inequality, respectively,we get

x3

y3 + 8+

y3

z3 + 8+

z3

x3 + 8≥ (x3 + y3 + z3)2

x3y3 + y3z3 + z3x3 + 8(x3 + y3 + z3)

≥ (x3 + y3 + z3)2

(x3 + y3 + z3)2

3+ 8(x3 + y3 + z3)

=3(x3 + y3 + z3)

x3 + y3 + z3 + 24= 3− 72

x3 + y3 + z3 + 24

≥ 3− 72

3x+ 3y + 3z + 18=

1

3.

469

Page 471: Олимпиадад бэлтгэгчдэд тусламж (Тэнцэтгэл биш 1)

On the other hand, from the inequality 3(xy + yz + zx) ≤ (x + y + z)2, wehave

1

9+

2

27(xy + xz + yz) ≤ 1

9+

2

81(x+ y + z)2 =

1

3.

From these two inequalities, we conclude that

x3

y3 + 8+

y3

z3 + 8+

z3

x3 + 8≥ 1

9+

2

27(xy + xz + yz),

as desired. The equality holds if and only if x = y = z = 1.

Second solution: From the AM-GM Inequality, we have

x3

y3 + 8+y + 2

27+y2 − 2y + 4

27≥ x

3,

y3

z3 + 8+z + 2

27+z2 − 2z + 4

27≥ y

3,

z3

x3 + 8+x+ 2

27+x2 − 2x+ 4

27≥ z

3.

Adding up these three inequalities, we deduce that

x3

y3 + 8+

y3

z3 + 8+

z3

x3 + 8≥

≥ x+ y + z

3− x+ y + z + 6

27− x2 + y2 + z2 − 2(x+ y + z) + 12

27

=12− x2 − y2 − z2

27=

12−[(x+ y + z)2 − 2(xy + yz + zx)

]27

=1

9+

2

27(xy + xz + yz),

as desired.?F?

08.17. Find the smallest real k such that for each x, y, z > 0, we have theinequality

x√y + y

√z + z

√x ≤ k

√(x+ y)(y + z)(z + x).

(Iran 2008)

Solution: The answer is3

2√

2. Let x = y = z = 1, we get k ≥ 3

2√

2, so the

number k must be at least3

2√

2. To show that it is our answer, it suffices to

prove that

x√y + y

√z + z

√x ≤ 3

2√

2

√(x+ y)(y + z)(z + x).

470

Page 472: Олимпиадад бэлтгэгчдэд тусламж (Тэнцэтгэл биш 1)

Indeed, from the Cauchy Schwarz Inequality and the AM-GM Inequality, wehave(

x√y + y

√z + z

√x)2 ≤ (xy + yz + zx)(x+ y + z)

= (x+ y)(y + z)(z + x) + xyz

≤ (x+ y)(y + z)(z + x) +(x+ y)(y + z)(z + x)

8

=9

8(x+ y)(y + z)(z + x).

From this, we conclude that

x√y + y

√z + z

√x ≤ 3

2√

2

√(x+ y)(y + z)(z + x),

as desired. Our proof is completed.?F?

08.18. For any positive real numbers a, b, c, d such that a2 + b2 + c2 + d2 = 1,prove the inequality

a2b2cd+ b2c2da+ c2d2ab+ d2a2bc+ c2a2db+ d2b2ac ≤ 3

32.

(Ireland 2008)

Solution: Firstly, we rewrite the inequality as

abcd(ab+ ac+ ad+ bc+ bd+ cd) ≤ 3

32.

According to the AM-GM Inequality, we have

ab+ ac+ ad+ bc+ bd+ cd =

≤ a2 + b2

2+a2 + c2

2+a2 + d2

2+b2 + c2

2+b2 + d2

2+c2 + d2

2

=3

2(a2 + b2 + c2 + d2) =

3

2,

and

abcd =√a2b2c2d2 ≤

√(a2 + b2 + c2 + d2

4

)4

=1

16.

Multiplying both two inequalities, we can get the result. Note that the equality

holds if and ony if a = b = c = d =1

2.

?F?

08.19. Let a, b, c be positive real numbers satisfying abc = 1. Prove that

1

b(a+ b)+

1

c(b+ c)+

1

a(c+ a)≥ 3

2.

(Kazakhstan 2008)

471

Page 473: Олимпиадад бэлтгэгчдэд тусламж (Тэнцэтгэл биш 1)

Solution: Firstly, we note that

1

b(a+ b)=

abc

b(a+ b)=

c

b

(1

a+

1

b

) .Therefore, the Cauchy Schwarz Inequality implies that[∑ 1

b(a+ b)

] [∑(1

a+

1

b

)]≥(∑√

c

b

)2

.

The inequality is reduced to(√b

a+

√c

b+

√a

c

)2

≥ 3

(1

a+

1

b+

1

c

),

orb

a+c

b+a

c+ 2

√a

b+ 2

√b

c+ 2

√c

a≥ 3

(1

a+

1

b+

1

c

).

The last inequality is trivial, since by the AM-GM Inequality, we have

b

a+

√c

a+

√c

a≥ 3

3

√b

a· ca

=3

a.

Remark: The inequality in question is sharpening a problem from the JuniorBalkan Mathemathetical Olympiads 2002, which stated that

1

b(a+ b)+

1

c(b+ c)+

1

a(c+ a)≥ 27

2(a+ b+ c)2.

This is a nice problem, and actually, we can show that the more general resultholds: If a, b, c are positive real numbers such that abc = 1, then for any k > 0,we have

1

b(a+ kb)+

1

c(b+ kc)+

1

a(c+ ka)≥ 3

k + 1.

?F?

08.20. Let a, b, c be positive real numbers such that (a+ b) (b+ c) (c+ a) = 8.Prove the inequality

a+ b+ c

3≥ 27

√a3 + b3 + c3

3.

(Macedonia 2008)

Solution: By the AM-GM Inequality, we have

(a+ b+ c)3 = a3 + b3 + c3 + 8(a+ b)(b+ c)(c+ a)

= a3 + b3 + c3 + 24 = 1 · (a3 + b3 + c3) + 8 · 3

≥ 9 9√

38(a3 + b3 + c3).

472

Page 474: Олимпиадад бэлтгэгчдэд тусламж (Тэнцэтгэл биш 1)

From this, we deduce that

a+ b+ c

3≥ 27

√a3 + b3 + c3

3,

as desired. The equality holds if and only if a = b = c = 1.?F?

08.21. Let a, b, c, d be positive real numbers such that a ≥ b ≥ c ≥ d andabcd = 1. Prove that

1

a3 + 1+

1

b3 + 1+

1

c3 + 1≥ 3

abc+ 1.

(MathLinks Contest 2008)

Solution: Firstly, note that if x, y are positive real numbers satisfying xy ≥ 1,then

1

x2 + 1+

1

y2 + 1− 2

xy + 1=

(x− y)2(xy − 1)

(xy + 1)(x2 + 1)(y2 + 1)≥ 0,

and thus1

x2 + 1+

1

y2 + 1≥ 2

xy + 1.

Now, using it for t2 = ab ≥√abcd = 1, we get

1

a3 + 1+

1

b3 + 1≥ 2

t3 + 1,

hence it suffices to prove that

2

t3 + 1+

1

c3 + 1≥ 3

t2c+ 1,

or in other words,

(t3c+ 2t2c2 − 2t− c)(t− c)2 ≥ 0.

For this, we proceed as follows

t3c+ 2t2c2 − 2t− c = t3c+ 2t2c2 − (2t+ c)(t2cd)3/4

≥ t3c+ 2t2c2 − (2t+ c)(t2c2)3/4

= t3c+ 2t2c2 − tc(2t+ c)√tc

≥ t3c+ 2t2c2 − 1

2tc(t+ c)(2t+ c)

=1

2tc2(t− c) ≥ 0.

Notice that the equality occurs if and only if a = b = c = d = 1.?F?

08.22. Let a, b, c be nonnegative real numbers satisfying ab + bc + ca = 3.Prove that

1

1 + a2(b+ c)+

1

1 + b2(c+ a)+

1

1 + c2(a+ b)≤ 3

1 + 2abc.

473

Page 475: Олимпиадад бэлтгэгчдэд тусламж (Тэнцэтгэл биш 1)

(MathLinks Contest 2008)

Solution: Note that the inequality is equivalent to∑ a2(b+ c)

1 + a2(b+ c)+

3

1 + 2abc≥ 3.

By the Cauchy Schwarz Inequality, we have∑ a2(b+ c)

1 + a2(b+ c)≥ 36∑

(b+ c)[1 + a2(b+ c)]=

18

9 +(∑

a)

(1− abc),

and hence it suffices to prove that

6

9 +(∑

a)

(1− abc)+

1

1 + 2abc≥ 1.

For this, we proceed by denoting with p, r the terms a + b + c, and abc,respectively. In this case, the last inequality becomes

6

9 + p(1− r)+

1

1 + 2r≥ 1,

which now rewrites as(1− r)(3− pr) ≥ 0.

This one is obviously valid, since by the AM-GM Inequality we have

r = abc ≤(ab+ bc+ ca

3

)3/2

= 1,

and

pr = abc(a+ b+ c) ≤ (ab+ bc+ ca)2

3= 3.

It is easy to see that the equality holds if and only if a = b = c = 1.?F?

08.23. Determine the least value of the expression

P = abc+1

abc

where a, b, c are positive real numbers satisfying a+ b+ c ≤ 3

2.

(Moldova 2008)

Solution: From the given hypothesis and the AM-GM Inequality, we find

that3

2≥ a + b + c ≥ 3 3

√abc, and hence abc ≤ 1

8. From now, we apply the

AM-GM Inequality again to obtain

P = abc+1

abc=

(abc+

1

64abc

)+

63

64abc≥ 1

4+

63

64 · 1

8

=65

8.

474

Page 476: Олимпиадад бэлтгэгчдэд тусламж (Тэнцэтгэл биш 1)

Note that the equality holds for a = b = c =1

2. This means that

65

8is the

minimum value of P. The problem is solved.?F?

08.24. Let a1, a2, . . . , an be positive real numbers such that a1+a2+· · ·+an ≤n

2. Determine the smallest value of the following expression

A =

√a21 +

1

a22+

√a22 +

1

a23+ · · ·+

√a2n +

1

a21.

(Moldova 2008)

First solution: The answer is

√17

2n, which can be obtained by setting a1 =

a2 = · · · = an =1

2. To prove this claim, we need to prove that A ≥

√17

2n.

Indeed, by the Cauchy Schwarz Inequality, for all x, y > 0, we have√(x2 +

1

y2

)(1

4+ 4

)≥ x

2+

2

y,

and thus √x2 +

1

y2≥ 1√

17

(x+

4

y

).

According to this inequality, we have

A ≥ 1√17

n∑i=1

(ai +

4

ai+1

)=

1√17

(n∑i=1

ai + 4n∑i=1

1

ai

)

≥ 1√17

n∑i=1

ai +4n2

n∑i=1

ai

=1√17

n∑i=1

ai +n2

4n∑i=1

ai

+15n2

4n∑i=1

ai

≥ 1√

17

n+15n2

4 · n2

=

√17

2n.

Second solution: We give another way to prove that A ≥√

17

2n. From the

Minkowsky’s Inequality and Cauchy Schwarz Inequality, we have

A ≥

√(a1 + a2 + · · ·+ an)2 +

(1

a1+

1

a2+ · · ·+ 1

an

)2

√(a1 + a2 + · · ·+ an)2 +

n4

(a1 + a2 + · · ·+ an)2.

475

Page 477: Олимпиадад бэлтгэгчдэд тусламж (Тэнцэтгэл биш 1)

On the other hand, the AM-GM Inequality implies that

(a1 + a2 + · · ·+ an)2 +n4

16(a1 + a2 + · · ·+ an)2≥ n2

2,

and from the given hypothesis, we have

15n4

16(a1 + a2 + · · ·+ an)2≥ 15n4

16 ·(n

2

)2 =15n2

4.

Adding up these two inequalities, we get

(a1 + a2 + · · ·+ an)2 +n4

(a1 + a2 + · · ·+ an)2≥ 17n2

4,

and thus, we deduce that

A ≥√

17

2n,

as claimed.?F?

08.25. Find the maximum value of constant C such that the inequality

x3 + y3 + z3 + C(xy2 + yz2 + zx2) ≥ (C + 1)(x2y + y2z + z2x)

holds for any nonnegative real numbers x, y, z.(Mongolia 2008)

Solution: Let z = 0, x = 1, y = t (0 < t < 1). Then from the hypothesis thatthe inequality is true, we must have

t3 + 1 + Ct2 ≥ Ct, or C ≤ t3 + 1

t− t2

for all t ∈ (0, 1). This means that the number C satisfies the required question

cannot be greater than the local minimum of the function f(t) =t3 + 1

t− t2on

(0, 1). By some simple computions, we find that

mint∈(0,1)

f(t) = C0 =1 +√

2 +√

2√

2− 1

2+

1√√2 +

√2√

2− 1

≈ 2.4844 . . .

So, we must have C ≤ C0. Now, we will prove that C0 is our answer, i.e.

x3 + y3 + z3 + C0(xy2 + yz2 + zx2) ≥ (C0 + 1)(x2y + y2z + z2x).

Due to the cyclicity, we may assume that z = min {x, y, z} and let x − z =p, y − z = q. By this assumption, we have p, q ≥ 0. Now, after some easycomputions, we can write that above inequality in the following equivalentform

2(p2 − pq + q2)z + p3 − (C0 + 1)p2q + C0pq2 + q3 ≥ 0.

476

Page 478: Олимпиадад бэлтгэгчдэд тусламж (Тэнцэтгэл биш 1)

Because 2(p2 − pq + q2)z ≥ 0, it suffices to show that

p3 − (C0 + 1)p2q + C0pq2 + q3 ≥ 0.

Let q = tp where t ≥ 0, then this inequality is equivalent to

p3 − (C0 + 1)tp3 + C0p3t2 + t3p3 ≥ 0,

or equivalently,1− (C0 + 1)t+ C0t

2 + t3 ≥ 0.

For t = 0, it is clear. For t ≥ 1, we have C0t2 + t3 ≥ C0t + t = (C0 + 1)t, so

the above inequality also holds. Suppose that 0 < t < 1, then we may writethe inequality as

t3 + 1 ≥ C0(t− t2) or C0 ≤t3 + 1

t− t2= f(t),

which is true because C0 = mint∈(0,1)

f(t). This completes our proof.

?F?

08.26. If a, b, c are nonnegative real numbers, then the inequality holds

4(√

a3b3 +√b3c3 +

√c3a3

)≤ 4c3 + (a+ b)3.

(Poland 2008)

Solution: Put x =√a, y =

√b, z =

√c, then our inequality becomes

(x2 + y2)3 + 4z6 ≥ 4x3y3 + 4z3(x3 + y3).

By the AM-GM Inequality, we have 4z3(x3 + y3) ≤ 4z6 + (x3 + y3)2. Conse-quently, we can see that the above inequality is deduced from

(x2 + y2)3 ≥ 4x3y3 + (x3 + y3)2.

We have (x2 + y2)3 − 4x3y3 − (x3 + y3)2 = 3x2y2(x− y)2, which is obviouslynonnegative. And this finishes our proof. Note that the equality holds if andonly if a = b = c.

?F?

08.27. For real numbers xi > 1, 1 ≤ i ≤ n, n ≥ 2, such that

x2ixi − 1

≥ S =

n∑j=1

xj , for all i = 1, 2, . . . , n,

find, with proof, supS.(Romania 2008)

Solution: For n = 2, S is unbounded to the right, since for the pairs(v,

v

v − 1

), where v > 1, we have S =

v2

v − 1, and then lim

v→1

v2

v − 1=∞.

477

Page 479: Олимпиадад бэлтгэгчдэд тусламж (Тэнцэтгэл биш 1)

For n > 2, we will prove the tight upper bound S ≤ n2

n− 1, with equality

if and only if xi =n

n− 1, for all i = 1, 2, . . . , n. Consider the function f :

(1, +∞)→ [4, +∞), defined by f(x) =x2

x− 1. It is straightforward to prove

that f is strictly decreasing on (1, 2]. Let now M = max{x1, x2, . . . , xn}, then

M2

M − 1≥ S > M + (n− 1),

and therefore M <n− 1

n− 2≤ 2, which implies that xi ∈ (1, 2) for all i. Now, for

m > 1, either xi ≥ m for some i, and then S ≤ x2ixi − 1

≤ m2

m−1 (according to

the monotonicity of f), or xi ≤ m for all i, and then clearly S ≤ nm. Solving

the equation m2

m−1 = nm, obtained by equaling the two possible upper bounds

for S, yields as unique solution m0 =n

n− 1, and therefore, in all cases,

S ≤ m20

m0 − 1= nm0 =

n2

n− 1,

with equality if and only if all xi’s are equal withn

n− 1.

Remark: Note that from this inequality, we can deduce the following problemof S. Berlov, which appeared in 2005 at the Russian MO:

Real numbers a1, a2, a3 > 1 satisfy a1 + a2 + a3 = S anda2i

ai − 1> S for

i = 1, 2, 3. Prove that

1

a1 + a2+

1

a2 + a3+

1

a3 + a3> 1.

?F?

08.28. Show that for all integers n ≥ 1, we have

n

(1 +

1

2+ · · ·+ 1

n

)≥ (n+ 1)

(1

2+

1

3+ · · ·+ 1

n+ 1

).

(Romania 2008)

Solution: The inequality is equivalent to

n∑k=1

(n

k− n+ 1

k + 1

)≥ 0,

orn∑k=1

n− kk(k + 1)

≥ 0,

which is obviously true since n ≥ k for all k = 1, 2, . . . , n.

478

Page 480: Олимпиадад бэлтгэгчдэд тусламж (Тэнцэтгэл биш 1)

?F?

08.29. Let a, b, c be positive real numbers with ab+ bc+ ca = 3. Prove that

1

1 + a2(b+ c)+

1

1 + b2(c+ a)+

1

1 + c2(a+ b)≤ 1

abc.

(Romania 2008)

Solution: Using the AM-GM Inequality, we deriveab+ bc+ ca

3≥ 3√a2b2c2.

As ab+ bc+ ca = 3, then abc ≤ 1. Now∑ 1

1 + a2(b+ c)=∑ 1

1 + a(ab+ ac)=∑ 1

1 + a(3− bc)

=∑ 1

3a+ (1− abc)≤∑ 1

3a=ab+ bc+ ca

3abc=

1

abc,

as required. Note that the equality holds if and only if a = b = c = 1.?F?

08.30. Determine the maximum value of real number k such that

(a+ b+ c)

(1

a+ b+

1

b+ c+

1

c+ a− k)≥ k

for all real numbers a, b, c ≥ 0 with a+ b+ c = ab+ bc+ ca.(Romania 2008)

First solution: Observe that the numbers a = b = 2, c = 0 fulfill the condi-tion a+ b+ c = ab+ bc+ ca. Plugging into the given inequality, we derive thatk ≤ 1. We claim that the inequality holds for k = 1, proving the maximumvalue of k is 1. To this end, rewrite the inequality as follows

(ab+ bc+ ca)

(1

a+ b+

1

b+ c+

1

c+ a− 1

)≥ 1,

or equivalently, ∑ ab+ bc+ ca

a+ b≥ ab+ bc+ ca+ 1.

Since∑ ab+ bc+ ca

a+ b=∑(

ab

a+ b+ c

)=∑ ab

a+ b+a+b+c and a+b+c =

ab+ bc+ ca, the last one can be rewritten as

ab

a+ b+

bc

b+ c+

ca

c+ a≥ 1.

Notice thatab

a+ b≥ ab

a+ b+ c, since a, b, c ≥ 0. Suming over a cyclic permu-

tation of a, b, c, we get∑ ab

a+ b≥∑ ab

a+ b+ c=ab+ bc+ ca

a+ b+ c= 1,

479

Page 481: Олимпиадад бэлтгэгчдэд тусламж (Тэнцэтгэл биш 1)

as needed.

Second solution: The inequality is equivalent to the following

S =a+ b+ c

a+ b+ c+ 1

(1

a+ b+

1

b+ c+

1

c+ a

)≥ k.

Under the given condition, we get

1

a+ b+

1

b+ c+

1

c+ a=a2 + b2 + c2 + 3(ab+ bc+ ca)

(a+ b)(b+ c)(c+ a)

=a2 + b2 + c2 + 2(ab+ bc+ ca) + (a+ b+ c)

(a+ b+ c)(ab+ bc+ ca)− abc

=(a+ b+ c)(a+ b+ c+ 1)

(a+ b+ c)2 − abc,

hence

S =(a+ b+ c)2

(a+ b+ c)2 − abc.

It is now clear that S ≥ 1, and the equality holds iff abc = 0. Consequently,k = 1 is the maximum value.

?F?

08.31. Let a, b ∈ [0, 1]. Prove the inequality

1

1 + a+ b≤ 1− a+ b

2+ab

3.

(Romania 2008)

First solution: By brute force, the given inequality is equivalent to

2a2b+ 2ab2 − 3a2 − 3b2 − 4ab+ 3a+ 3b ≥ 0.

Rearrange terms, we can rewrite it as

2(a+ b)(a− 1)(b− 1) + a(1− a) + b(1− b) ≥ 0,

which holds true since all terms are nonnegative by the given condition. Notethat the equality holds if and only if a = b = 0 or a = 1, b = 0 or a = 0, b = 1.

Second solution: We have two casesCase 1. a+ b ≥ 1. Since a, b ∈ [0, 1] , we have (a− 1)(b− 1) ≥ 0 from which itfollows that ab ≥ a+ b− 1. Therefore

1− a+ b

2+ab

3− 1

1 + a+ b≥ 1− a+ b

2+a+ b− 1

3− 1

1 + a+ b

=4− a− b

6− 1

1 + a+ b

=(a+ b− 1)(2− a− b)

6(a+ b+ 1)≥ 0.

480

Page 482: Олимпиадад бэлтгэгчдэд тусламж (Тэнцэтгэл биш 1)

Case 2. 1 > a+ b ≥ 0. In this case, we have

1− a+ b

2+ab

3− 1

1 + a+ b≥ 1− a+ b

2− 1

1 + a+ b

=(a+ b)(1− a− b)

2(a+ b+ 1)≥ 0.

These two cases allow us to conclude that the following inequality holds forany a, b ∈ [0, 1]

1

1 + a+ b≤ 1− a+ b

2+ab

3,

as desired.?F?

08.32. Let n ≥ 3 is an odd integer. Determine the maximum value of thecyclic sum, for 0 ≤ xi ≤ 1, i = 1, 2, . . . , n,

E =√|x1 − x2|+

√|x2 − x3|+ · · ·+

√|xn − x1|.

(Romania 2008)

Solution: The expression E is cyclic, so we may assume without loss ofgenerality that x1 is the maximum number among x1, x2, . . . , xn. Denotingxn+1 = x1, we will prove that exists i ∈ {1, 2, . . . , n} such that (xi+1 −xi)(xi+1 − xi+2) ≤ 0. Indeed, if it does not exist a number i satisfying thisproperty, then for all i = 1, 2, . . . , n, we have

(xi+1 − xi)(xi+1 − xi+2) > 0.

If i = 1, we get (x2 − x1)(x2 − x3) > 0. Because x1 = maxxi, we obtainx2 < x3. Similarly, if i = 2, we get (x3 − x2)(x3 − x4) > 0, and thereforex3 > x4. We can proceed this way, step by step, to get easily as conclusion

that x2k+1 > max{x2k, x2k+2} for all 1 ≤ k ≤ n− 1

2(note that n is an odd

number). From this conclusion, we have

x2·n−12

+1 > max{x2·n−1

2, x2·n−1

2+2

},

or xn > max{xn−1, xn+1} = max{xn−1, x1} = x1, which is contradiction aswe have x1 ≥ xi for all i = 1, 2, . . . , n. Therefore, it must exist a number isuch that (xi+1 − xi)(xi+1 − xi+2) ≤ 0. Now, the Cauchy Schwarz Inequalityimplies that√

|xi − xi+1|+√|xi+1 − xi+2| ≤

√2(|xi − xi+1|+ |xi+1 − xi+2|).

It is easy to verify this fact: If ab ≤ 0 then |a| + |b| = |a − b|. Applying thispropriety for a = xi+1 − xi, b = xi+1 − xi+2, we have

|xi − xi+1|+ |xi+1 − xi+2| = |(xi+1 − xi)− (xi+1 − xi+2)| = |xi − xi+2| ≤ 1.

481

Page 483: Олимпиадад бэлтгэгчдэд тусламж (Тэнцэтгэл биш 1)

Therefore√|xi − xi+1|+

√|xi+1 − xi+2| ≤

√2. Moreover, because xi ∈ [0, 1],

the following inequality is trivial

∑j 6=i,j 6=i+1

√|xj − xj+1| ≤ n− 2.

So we can easily obtain that E ≤ n−2 +√

2. This is the maximum value of E

because the equality can occur for x1 =1

2, x2 = 1, x3 = 0, . . . , xn−1 = 1, xn =

0.?F?

08.33. Let a, b, c be three positive real numbers satisfying abc = 8. Provethat

a− 2

a+ 1+b− 2

b+ 1+c− 2

c+ 1≤ 0.

(Romania 2008)

Solution: The inequality in question rewrites as

1 ≤ 1

a+ 1+

1

b+ 1+

1

c+ 1.

Consider the substitutions a =2x

y, b =

2y

z, c =

2z

x, we have

1

a+ 1+

1

b+ 1+

1

c+ 1=

y2

2xy + y2+

z2

2yz + z2+

x2

2zx+ x2

≥ (x+ y + z)2

x2 + y2 + z2 + 2xy + 2yz + 2zx= 1.

?F?

08.34. Let a1, a2, . . . , an be positive real numbers satisfying the conditionthat a1 + a2 + · · ·+ an = 1. Prove that

n∑j=1

aj1 + a1 + · · ·+ aj

<1√2.

(Romania 2008)

Solution: For convenience, we take a0 = 0 and denote P as the left hand sideof the orginal inequality. Then, applying the Cauchy Schwarz Inequality, we

482

Page 484: Олимпиадад бэлтгэгчдэд тусламж (Тэнцэтгэл биш 1)

have that

P =

n∑j=1

aj1 + a0 + · · ·+ aj

=

n∑j=1

( √aj

1 + a0 + · · ·+ aj· √aj

)

√√√√√ n∑j=1

aj(1 + a0 + · · ·+ aj)2

n∑j=1

aj

<

√√√√ n∑j=1

aj(1 + a0 + · · ·+ aj−1)(1 + a0 + · · ·+ aj)

=

√√√√ n∑j=1

(1

1 + a0 + · · ·+ aj−1− 1

1 + a0 + · · ·+ aj

)

=

√1− 1

1 + a0 + · · ·+ an=

1√2.

?F?

08.35. Let x, y, z be positive real numbers such that x+ y+ z = 1. Prove theinequality

1

yz + x+1

x

+1

xz + y +1

y

+1

xy + z +1

z

≤ 27

31.

(Serbia 2008)

First solution: Put a = 3x, b = 3y, c = 3z, then we have a + b + c = 3 andthe desired inequality becomes

a

3a2 + abc+ 27+

b

3b2 + abc+ 27+

c

3c2 + abc+ 27≤ 3

31.

According to the Schur’s Inequality (applied for third degree), we have

(a+ b+ c)3 + 9abc ≥ 4(a+ b+ c)(ab+ bc+ ca),

and thus abc ≥ 4(ab+ bc+ ca)− 9

3. Using this inequality, it suffices to prove

that ∑ a

3a2 +4(ab+ bc+ ca)− 9

3+ 27

≤ 3

31,

or equivalently, ∑ 3a

9a2 + 4(ab+ bc+ ca) + 72≤ 3

31.

We rewrite the latter inequality as

∑[1− 31a(a+ b+ c)

9a2 + 4(ab+ bc+ ca) + 72

]≥ 0,

483

Page 485: Олимпиадад бэлтгэгчдэд тусламж (Тэнцэтгэл биш 1)

that is ∑ 9a2 + 4(ab+ bc+ ca) + 8(a+ b+ c)2 − 31a(a+ b+ c)

a2 + s≥ 0,

where s =4(ab+ bc+ ca) + 72

9. Because 9a2 +4(ab+ bc+ ca)+8(a+ b+ c)2−

31a(a + b + c) = (7a + 8c + 10b)(c − a) − (7a + 8b + 10c)(a − b), the aboveinequality is equivalent to∑ (7a+ 8c+ 10b)(c− a)− (7a+ 8b+ 10c)(a− b)

a2 + s≥ 0,

that is ∑(a− b)

(8a+ 7b+ 10c

b2 + s− 7a+ 8b+ 10c

a2 + s

)≥ 0.

By some easy computations, we can rewrite it as∑(a− b)2 · 8a2 + 8b2 + 15ab+ 10c(a+ b) + s

(a2 + s)(b2 + s)≥ 0.

The last one is obviously true, therefore our proof is completed. Note that the

equality holds if and only if x = y = z =1

3.

?F?

08.36. Determine the least value of the expression x2 + y2 + z2, where x, y, zare real numbers such that x3 + y3 + z3 − 3xyz = 1.

(United Kingdom 2008)

Solution: Using the well-known identity x3+y3+z3−3xyz = (x+y+z)(x2+y2 + z2 − xy − yz − zx) and the Cauchy Schwarz Inequality, we find that

1 = (x3 + y3 + z3 − 3xyz)2

= (x+ y + z)2(x2 + y2 + z2 − xy − yz − zx)2

≤[

(x+ y + z)2 + 2(x2 + y2 + z2 − xy − yz − zx)

3

]3= (x2 + y2 + z2)3.

And since x2 + y2 + z2 ≥ 0, the above inequality implies that

x2 + y2 + z2 ≥ 1.

On the other hand, it is clear that for x = 1, y = z = 0, we have x3 + y3 +z3 − 3xyz = 1 and x2 + y2 + z2 = 1. This allows us to conclude that 1 is theminimum of x2 + y2 + z2, and the problem is solved.

?F?

08.37. If a, b, c, d are positive real numbers, then

(a+ b)(b+ c)(c+ d)(d+ a)(

1 +4√abcd

)4≥ 16abcd(1 + a)(1 + b)(1 + c)(1 + d).

484

Page 486: Олимпиадад бэлтгэгчдэд тусламж (Тэнцэтгэл биш 1)

(Ukraine 2008)

First solution: Firstly, we claim that for all x, y > 0, the following inequalityholds

x+ y

(1 + x)(1 + y)≥

2√xy(

1 +√xy)2 .

Indeed, we have

x+ y

2√xy− (1 + x)(1 + y)(

1 +√xy)2 =

(x+ y

2√xy− 1

)−

[(1 + x)(1 + y)(

1 +√xy)2 − 1

]

=

(√x−√y

)22√xy

−(√x−√y

)2(1 +√xy)2

=

(√x−√y

)2(xy + 1)

2√xy(1 +√xy)2 ≥ 0.

This proves our claim. Now, by using the claim and the Cauchy SchwarzInequality, we have

(a+ b)(b+ c)(c+ d)(d+ a)

(1 + a)(1 + b)(1 + c)(1 + d)≥ 4

√abcd(b+ c)(d+ a)(

1 +√ab)2 (

1 +√cd)2 ≥ 4

√abcd

(√ab+

√cd)2

(1 +√ab)2 (

1 +√cd)2

≥ 4√abcd

2√√

ab ·√cd(

1 +√√

ab ·√cd)22

=16abcd(

1 + 4√abcd

)4 .Multiplying both sides of this inequality by (1+a)(1+b)(1+c)(1+d)

(1 + 4√abcd

)4,

we get

(a+ b)(b+ c)(c+ d)(d+ a)(

1 +4√abcd

)4≥ 16abcd(1 + a)(1 + b)(1 + c)(1 + d),

as desired. Note that the equality holds if and only if a = b = c = d.

Second solution: Denote A = (a+ b+ c+ d)(abc+ bcd+ cda+ dab). Thenwe have

(a+ b)(b+ c)(c+ d)(d+ a)−A = (ac− bd)2 ≥ 0.

Hence, it suffices to prove that

A(

1 +4√abcd

)4≥ 16abcd(1 + a)(1 + b)(1 + c)(1 + d).

By expanding, we can rewrite the latter inequality as S1+S2+S3+S4+S5 ≥ 0,where

S1 = A− 16abcd,

S2 = 4A4√abcd− 16abcd(a+ b+ c+ d),

S3 = 6A√abcd− 16abcd(ab+ ac+ ad+ bc+ bd+ cd),

S4 = 4A4√a3b3c3d3 − 16abcd(abc+ bcd+ cda+ dab),

S5 = Aabcd− 16a2b2c2d2.

485

Page 487: Олимпиадад бэлтгэгчдэд тусламж (Тэнцэтгэл биш 1)

To finish the proof, we only need to prove that each term of the above sum isnonnegative. Actually, we will prove that S1 ≥ 0, S2 ≥ 0, S3 ≥ 0, S4 ≥ 0 andS5 ≥ 0. The inequalities S1 ≥ 0, S2 ≥ 0, S4 ≥ 0 and S5 ≥ 0 are obvious fromthe AM-GM Inequality (Actually, we use the inequalities a+b+c+d ≥ 4 4

√abcd

and abc+ bcd+ cda+ dab ≥ 44√a3b3c3d3 to prove S1 ≥ 0 and S5 ≥ 0. We use

abc+bcd+cda+dab ≥ 44√a3b3c3d3 to prove S2 ≥ 0 and a+b+c+d ≥ 4 4

√abcd to

prove that S4 ≥ 0). Now we will prove that S3 ≥ 0. Applying the Maclaurin’sInequality, we get

a+ b+ c+ d

4≥√ab+ ac+ ad+ bc+ bd+ cd

6,

and1

a+

1

b+

1

c+

1

d4

√√√√ 1

ab+

1

ac+

1

ad+

1

bc+

1

bd+

1

cd6

.

Multiplying both two inequalities, we deduce that

(a+ b+ c+ d)(abc+ bcd+ cda+ dab)

16abcd≥ ab+ ac+ ad+ bc+ bd+ cd

6√abcd

,

and hence

A ≥ 8

3

√abcd(ab+ ac+ ad+ bc+ bd+ cd).

Using this inequality, we have

S3 ≥ 6√abcd·8

3

√abcd(ab+ac+ad+bc+bd+cd)−16abcd(ab+ac+ad+bc+bd+cd) = 0.

Thus, we have proved that S1, S2, S3, S4, S5 are all nonnegative and the resultfollows immediately.

?F?

08.38. Let a, b, c be positive real numbers such that a2 + b2 + c2 = 3. Provethe inequality√

a2

a2 + b+ c+

√b2

b2 + c+ a+

√c2

c2 + a+ b≤√

3.

(Ukraine 2008)

First solution: By the AM-GM Inequality, we can conclude that

a+ b+ c ≤ a2 + 1

2+b2 + 1

2+c2 + 1

2= 3.

Now, denote with P the left hand side of the original inequality, then theCauchy Schwarz Inequality implies

P 2 ≤ (a+ b+ c)

(a

a2 + b+ c+

b

b2 + c+ a+

c

c2 + a+ b

)

≤ (a+ b+ c)

∑ a

a2 +1

3(b+ c)(a+ b+ c)

.486

Page 488: Олимпиадад бэлтгэгчдэд тусламж (Тэнцэтгэл биш 1)

Therefore, it suffices to prove that

(a+ b+ c)

∑ a

a2 +1

3(b+ c)(a+ b+ c)

≤ 3.

This is a homogeneous inequality of a, b, c, so we may give up the relationa2 + b2 + c2 = 3 to normalize a + b + c = 1. By this normalization, we canrewrite the last inequality as

a

3a2 − a+ 1+

b

3b2 − b+ 1+

c

3c2 − c+ 1≤ 1.

This is true and we can prove it as follows∑ a

3a2 − a+ 1=∑ 3a

9a2 − 3a+ 3=∑ 3a

(3a− 1)2 + 3a+ 2≤∑ 3a

3a+ 2

=∑(

1− 2

3a+ 2

)= 3− 2

(1

3a+ 2+

1

3b+ 2+

1

3c+ 2

)≤ 3− 18

3(a+ b+ c) + 6= 1.

Note that the equality holds iff a = b = c = 1.

Second solution: According to the Cauchy Schwarz Inequality, we have

(a2 + b+ c)(1 + b+ c) ≥ (a+ b+ c)2.

This implies√a2

a2 + b+ c=

a√

1 + b+ c√(a2 + b+ c)(1 + b+ c)

≤ a√

1 + b+ c

a+ b+ c.

Therefore, it suffices to prove that

a√

1 + b+ c+ b√

1 + c+ a+ c√

1 + a+ b

a+ b+ c≤√

3.

We apply the Cauchy Schwarz Inequality again and obtain

a√

1 + b+ c+ b√

1 + c+ a+ c√

1 + a+ b ≤

≤√

(a+ b+ c)[a(1 + b+ c) + b(1 + c+ a) + c(1 + a+ b)]

=√

(a+ b+ c)[(a+ b+ c) + 2(ab+ bc+ ca)].

This inequality gives us an upper bound of the left hand side of the aboveinequality, that is√

(a+ b+ c)[(a+ b+ c) + 2(ab+ bc+ ca)]

a+ b+ c=

√1 +

2(ab+ bc+ ca)

a+ b+ c.

And so, it suffices to check

ab+ bc+ ca

a+ b+ c≤ 1.

487

Page 489: Олимпиадад бэлтгэгчдэд тусламж (Тэнцэтгэл биш 1)

However, this is true since from the AM-GM Inequality, we have

ab+ bc+ ca

a+ b+ c≤ a+ b+ c

3≤

a2 + 1

2+b2 + 1

2+c2 + 1

23

= 1.

?F?

08.39. Let x, y, z be distinct nonnegative real numbers. Prove that

1

(x− y)2+

1

(y − z)2+

1

(z − x)2≥ 4

xy + yz + zx.

(Vietnam 2008)

Solution: Without loss of generality, we may assume that z = min {x, y, z} .Now, observe that

(x− z)2 + (y − z)2 = (x− y)2 + 2(x− z)(y − z).

Therefore, by the AM-GM Inequality, we get

1

(x− y)2+

1

(y − z)2+

1

(z − x)2=

1

(x− y)2+

(x− y)2

(x− z)2(y − z)2+

2

(x− z)(y − z)

≥ 2

(x− z)(y − z)+

2

(x− z)(y − z)

=4

(x− z)(y − z)≥ 4

xy≥ 4

xy + yz + zx,

as desired.?F?

488

Page 490: Олимпиадад бэлтгэгчдэд тусламж (Тэнцэтгэл биш 1)

Chapter 4

Anexa 1

Tehnici de baza ın demonstrarea inegalitatilor

In compararea cantitatilor observate ın viata reala, relatia de egalitate estelocala si relativa, ın timp ce relatia de inegalitate este universala si absoluta.Esenta inegalitatii este relatia ıntre cantitati inegale.Putem compara ıntotdeauna doua cantitati de acelasi fel, ıncercand sa aratamca una este mai mare decat cealalta: cu alte cuvinte, sa demonstram o inegal-itate. Tehnicile care pot fi utilizate ın demonstrarea unei inegalitati depindde forma si natura inegalitatii, dar si de experienta si abilitatea rezolvitoru-lui, bazandu-se de multe ori pe anumite inegalitati fundamentale, cum ar fiinegalitatea mediilor (ıntre media aritmetica si media geometrica), inegali-tatea Cauchy-Schwarz, inegalitatea Schur etc.; alte tehnici implica anumiterearanjari algebrice mai avansate.In acest capitol introducem cele mai simple tehnici de baza pentru demon-strarea inegalitatilor. Subliniem faptul ca pentru demonstrarea unor ine-galitati mai dificile, adesea trebuie sa aplicam doua sau mai multe tehnici(de baza si/sau avansate). De regula, o demonstratie este frumoasa atuncicand se bazeaza pe un numar redus de tehnici simple.

4.1 Metoda compararii

In mod natural, avem doua cai pentru a compara doua cantitati:(1) Comparare prin scadere: pentru a arata ca A ≥ B, este suficient sa aratamca A−B ≥ 0;(2) Comparare prin ımpartire: presupunand ca B > 0, pentru a arata caA ≥ B, este suficient sa aratam ca A

B ≥ 1.In cadrul metodei compararii, inegalitatea de demonstrat trebuie rearanjataıntr-o forma convenabila, evidenta sau demonstrabila printr-o metoda cunos-cuta, utilizand operatii de reducere, combinare, factorizare, spargere si com-pletare cu patrate, care necesita abilitate, experienta si buna observatie dinpartea rezolvitorului.

Ex. 1 Pentru numerele reale x, y, z care satisfac xy + yz + zx = −10, aratatica:

x2 + 5y2 + 8z2 ≥ 40.

489

Page 491: Олимпиадад бэлтгэгчдэд тусламж (Тэнцэтгэл биш 1)

Proof. Deoarece

x2 + 5y2 + 8z2 − 40 = x2 + 5y2 + 8z2 + 4(xy + yz + zx)

= (x+ 2y + 2z)2 + (y − 2z)2 ≥ 0,

rezulta

x2 + 5y2 + 8z2 ≥ 40.

Egalitatea are loc atunci cand xy + yz + zx = −10, x + 2y + 2z = 0si y − 2z = 0, deci cand x = 4, y = −2, z = −1 sau x = −4, y = 2,z = 1.

Ex. 2 Fie a, b, c ∈ R+. Demonstrati ca pentru orice numere reale x, y, z, avem:

x2+y2+z2 ≥ 2

√abc

(a+ b)(b+ c)(c+ a)

(a+ b

cxy +

b+ c

ayz +

c+ a

bzx

).

[Observatie] Deoarece pentru a = b = c se obtine cunoscuta inegalitatex2+y2+z2 ≥ xy+yz+zx, echivalenta cu (x−y)2+(y−z)2+(z−x)2 ≥0, putem apela la o metoda similara pentru a demonstra inegalitateaceruta.

Proof. Scriem inegalitatea sub forma[b

b+ cx2 +

a

c+ ay2 − 2

√ab

(b+ c)(c+ a)xy

]

+

[c

c+ ay2 +

b

a+ bz2 − 2

√bc

(c+ a)(a+ b)yz

]

+

[a

a+ bz2 +

c

b+ cx2 − 2

√ca

(b+ c)(a+ b)zx

]≥ 0,

sau ∑ab

[x√

a(b+ c)− y√

b(c+ a)

]2≥ 0,

unde∑

este simbolul ınsumarii ciclice. Ultima inegalitate este evidentadevarata. Egalitatea are loc atunci cand

x√a(b+ c)

=y√

b(c+ a)=

z√c(a+ b)

.

Ex.3 Fie a, b, c ∈ R+. Aratati ca:

a2ab2bc2c ≥ ab+cbc+aca+b.

490

Page 492: Олимпиадад бэлтгэгчдэд тусламж (Тэнцэтгэл биш 1)

Proof. Impartind ambii membri prin ab+cbc+aca+b, inegalitatea poate fiscrisa astfel (a

b

)a−b(bc

)b−c ( ca

)c−a≥ 1.

Luand pe rand a ≥ b si a ≤ b, constatam ca(ab

)a−b≥ 1.

Din aceasta inegalitate si din celelelte doua similare rezulta imediat ine-galitatea ceruta. Egalitatea are loc pentru a = b = c.

[Nota] Putem scrie inegalitatea de mai sus sub forma

a3ab3bc3c ≥ aa+b+cba+b+cca+b+c,

sau

aabbcc ≥ (abc)a+b+c

3 .

In general, daca xi ∈ R+, i = 1, 2, · · · , n, avem

xx11 · xx22 · · · · · x

xnn ≥ (x1x2 · · ·xn)

x1+x2+···+xnn .

Demonstratia este similara, scriind inegalitatea sub forma

∏i<j

(xixj

)xi−xj≥ 1.

Ex. 4 Daca a, b, c sunt numere reale nenegative, atunci

a2 + b2 + c2 + 2abc+ 1 ≥ 2(ab+ bc+ ca).

Proof. Printre numerele 1−a, 1− b si 1− c exista doua cu acelasi semn;fie acestea 1− b si 1− c; deci

(1− b)(1− c) ≥ 0.

Avem

a2 + b2 + c2 + 2abc+ 1− 2(ab+ bc+ ca)

= (a− 1)2 + (b− c)2 + 2a+ 2abc− 2(ab+ ca)

= (a− 1)2 + (b− c)2 + 2a(1− b)(1− c) ≥ 0.

Egalitatea are loc daca si numai daca a = b = c = 1.

Ex. 5 Daca a, b, c sunt numere reale nenegative, aratati ca

a3 + b3 + c3 − 3abc ≥ 2

(b+ c

2− a)3

.

491

Page 493: Олимпиадад бэлтгэгчдэд тусламж (Тэнцэтгэл биш 1)

Proof. Scriem inegalitatea sub forma

2(a+ b+ c)[(a− b)2 + (b− c)2 + (c− a)2] ≥ (b+ c− 2a)3.

In cazul netrivial b + c − 2a > 0, deoarece a + b + c ≥ b + c − 2a si(b− c)2 ≥ 0, este suficient sa demonstram ca

2[(a− b)2 + (c− a)2] ≥ (b+ c− 2a)2.

Intr-adevar, cu notatiile x = b− a si y = c− a, avem

2[(a− b)2 + (c− a)2]− (b+ c− 2a)2 = 2(x2 + y2)− (x+ y)2

= (x− y)2 = (b− c)2 ≥ 0.

Egalitatea are loc atunci cand a = b = c, precum si atunci cand unuldintre numerele a, b, c este 0, iar ceilelalte sunt egale.

Ex. 6 Fie a, b, c numere reale nenegative astfel ıncat a2 + b2 + c2 = 3. Aratatica

ab2 + bc2 + ca2 ≤ abc+ 2.

Proof. Fara a pierde din generalitate, presupunem ca b este cuprins ıntrea si c, adica

(b− a)(b− c) ≤ 0.

Avem

2− ab2 − bc2 − ca2 + abc = 2− ab2 − b(3− a2 − b2)− ca2 + abc

= b3−3b+ 2−a(b2−ab+ ca− bc) = (b−1)2(b+ 2)−a(b−a)(b− c) ≥ 0.

Egalitatea are loc atunci cand a = b = c = 1, precum si atunci cand(a, b, c) este o permutatre ciclica a tripletului (0, 1,

√2) .

Ex. 7 Daca a, b, c sunt numere reale pozitive, atunci

a

b+b

c+c

a− 2 ≥ a2 + b2 + c2

ab+ bc+ ca.

Proof. Deoarece inegalitatea este ciclica, fara a pierde din generalitatepresupunem ca c = min{a, b, c}. Avem

a

b+b

c+c

a−3 =

(a

b+b

a− 2

)+

(b

c+c

a− b

a− 1

)=

(a− b)2

ab+

(a− c)(b− c)ac

sia2 + b2 + c2

ab+ bc+ ca− 1 =

(a− b)2 + (a− c)(b− c)ab+ bc+ ca

.

Rezultaa

b+b

c+c

a− 2− a2 + b2 + c2

ab+ bc+ ca

492

Page 494: Олимпиадад бэлтгэгчдэд тусламж (Тэнцэтгэл биш 1)

=

(1

ab− 1

ab+ bc+ ca

)(a− b)2 +

(1

ac− 1

ab+ bc+ ca

)(a− c)(b− c)

=c2(a+ b)(a− b)2 + b2(a+ c)(a− c)(b− c)

abc(ab+ bc+ ca)≥ 0.

Egalitatea are loc atunci cand a = b = c.

Ex. 8 Fie a, b, c ∈ R+ astfel ıncat a2 + b2 + c2 = 1. Gasiti minimul pentru

S =1

a2+

1

b2+

1

c2− 2(a3 + b3 + c3)

abc.

[Observatie] Nu ezitati sa ghiciti raspunsul corect al unei probleme deextremum, deoarece acest lucru va poate ajuta la rezolvarea problemei!Operatia de ”ghicire” a valorii extreme este strans legata de valorilevariabilelor pentru care inegalitatea devine egalitate, de care trebuie satinem seama si ın etapa de demonstrare a inegalitatii.

Proof. Pentru a = b = c = 1√3, rezulta S = 3. Vom ıncerca sa aratam

ca S ≥ 3. Intr-adevar,

S − 3 =1

a2+

1

b2+

1

c2− 3− 2(a3 + b3 + c3)

abc

=a2 + b2 + c2

a2+a2 + b2 + c2

b2+a2 + b2 + c2

c2− 3− 2

(a2

bc+b2

ca+c2

ab

)

= a2(

1

b2+

1

c2

)+ b2

(1

a2+

1

c2

)+ c2

(1

a2+

1

b2

)− 2

(a2

bc+b2

ca+c2

ab

)

= a2(

1

b− 1

c

)2

+ b2(

1

c− 1

a

)2

+ c2(

1

a− 1

b

)2

≥ 0.

Prin urmare, S are valoarea minima 3, care este atinsa atunci canda = b = c = 1√

3.

Ex. 9 Daca x ∈ R, y ≥ 0 si y(y + 1) ≤ (x+ 1)2, atunci

y(y − 1) ≤ x2.

Proof. In cazul 0 ≤ y ≤ 1, avem evident y(y − 1) ≤ 0 ≤ x2. In cazuly > 1, scriem ipoteza y(y + 1) ≤ (x+ 1)2 sub forma

y ≤√

(x+ 1)2 +1

4− 1

2,

iar inegalitatea ceruta y(y − 1) ≤ 0 ≤ x2 sub forma echivalenta

y ≤√x2 +

1

4+

1

2.

493

Page 495: Олимпиадад бэлтгэгчдэд тусламж (Тэнцэтгэл биш 1)

Prin urmare, este suficient sa aratam ca√(x+ 1)2 +

1

4− 1

2≤√x2 +

1

4+

1

2,

adica √(x+ 1)2 +

1

4≤√x2 +

1

4+ 1.

Prin ridicare la patrat, obtinem inegalitatea

2x ≤ 2

√x2 +

1

4,

care este ın mod clar adevarata. Cu aceasta, demonstratia este ıncheiata.Avem egalitate pentru x = 0 si y ∈ {0, 1}.

Ex. 10 Fie a, b, c numere reale pozitive. Aratati ca

a

b+b

c+c

a≥ 3 +

(a− c)2

ab+ bc+ ca.

Proof. Prin dezvoltare si reducere, inegalitatea devine astfel

b2 +ab2

c+a2c

b+bc2

a≥ 2ab+ 2bc.

Scriem aceasta inegalitate sub forma echivalenta(bc2

a+ ab− 2bc

)+

(ab2

c+a2c

b+ b2 − 3ab

)≥ 0.

Din inegalitatea mediilor a doua si respectiv trei numere , avem

bc2

a+ ab ≥ 2

√bc2

a· ab = 2bc

si

ab2

c+a2c

b+ b2 ≥ 3

3

√ab2

c· a

2c

b· b2 = 3ab,

iar prin ınsumarea acestor doua inegalitati obtinem inegalitatea ceruta.Avem egalitate daca si numai daca a = b = c.

Ex. 11 Daca a, b, c sunt numere reale pozitive, atunci

a(a− b)b+ c

+b(b− c)c+ a

+c(c− a)

a+ b≥ 0.

Proof. Deoarece

a(a− b)b+ c

=

(a

b+ c+ 1

)(a− b)− a+ b = (a+ b+ c)

a− bb+ c

− a+ b,

494

Page 496: Олимпиадад бэлтгэгчдэд тусламж (Тэнцэтгэл биш 1)

rezulta∑ a(a− b)b+ c

= (a+ b+ c)∑ a− b

b+ c= (a+ b+ c)

∑(a+ c

b+ c− 1

).

Prin urmare, inegalitatea dorita este echivalenta cu

(a+ b+ c)

(∑ a+ c

b+ c− 3

)≥ 0.

Aceasta inegalitate rezulta imediat din inegalitatea mediilor

∑ a+ c

b+ c≥ 3 3

√a+ c

b+ c· b+ a

c+ a· c+ b

a+ b= 3.

Avem egalitate daca si numai daca a = b = c.

Ex. 12 Pentru n ∈ N+, demonstrati ca

1

n+ 1(1 +

1

3+ · · ·+ 1

2n− 1) ≥ 1

n(1

2+

1

4+ · · ·+ 1

2n).

Proof. Scriem inegalitatea sub forma echivalenta

n(1 +1

3+ · · ·+ 1

2n− 1) ≥ (n+ 1)(

1

2+

1

4+ · · ·+ 1

2n),

saun

2+ n(

1

3+

1

5+ · · ·+ 1

2n− 1)

≥ n(1

4+

1

6+ · · ·+ 1

2n) + (

1

2+

1

4+ · · ·+ 1

2n).

Ultima inegalitate este adevarata, deoarece

n

2≥ 1

2+

1

4+ · · ·+ 1

2n

si1

3+

1

5+ · · ·+ 1

2n− 1≥ 1

4+

1

6· · ·+ 1

2n.

Avem egalitate numai pentru n = 1.

Ex. 13 Fie a, b, c ∈ R+ astfel ıncat abc = 1. Aratati ca

(a+ b)(b+ c)(c+ a) ≥ 4(a+ b+ c− 1).

Proof. Fara a pierde din generalitate, presupunem ca a ≥ 1. Inegalitateadorita este echivalenta cu

a2(b+ c) + b2(c+ a) + c2(a+ b) + 6 ≥ 4(a+ b+ c),

sau

(a2 − 1)(b+ c) + b2(c+ a) + c2(a+ b) + 6 ≥ 4a+ 3(b+ c).

495

Page 497: Олимпиадад бэлтгэгчдэд тусламж (Тэнцэтгэл биш 1)

Deoarece (a+ 1)(b+ c) ≥ 2√a · 2√bc = 4, este suficient sa aratam ca

4(a− 1) + b2(c+ a) + c2(a+ b) + 6 ≥ 4a+ 3(b+ c),

adicaa(b2 + c2) + bc(b+ c)− 3(b+ c) + 2 ≥ 0.

Aplicand inegalitatea mediilor si inegalitatea b2 + c2 ≥ 1

2(b+ c)2, avem

a(b2 + c2) + bc(b+ c) ≥ 2√abc(b2 + c2)(b+ c) = 2

√(b2 + c2)(b+ c)

≥ (b+ c)√

2(b+ c).

Prin urmare, este suficient sa demonstram ca

(b+ c)√

2(b+ c)− 3(b+ c) + 2 ≥ 0.

Cu notatia x =b+ c

2, inegalitatea poate fi scrisa astfel

2x3 − 3x2 + 1 ≥ 0.

Deoarece2x3 − 3x2 + 1 = (x− 1)2(2x+ 1) ≥ 0,

demonstratia este completa. Egalitatea se atinge atunci cand a = b =c = 1.

Ex. 14 Fie a, b, c numere pozitive astfel ıncat abc = 1. Aratati ca

a

b+b

c+

1

a≥ a+ b+ 1.

Proof. Scriem inegalitatea sub forma echivalenta(2 · a

b+b

c

)+

(b

c+

1

a

)+

(1

a+ a

)≥ 3a+ 2b+ 2.

Aplicand inegalitatea mediilor, rezulta(2 · a

b+b

c

)+

(b

c+

1

a

)+

(1

a+ a

)≥ 3

3

√a2

bc+ 2

√b

ca+ 2 = 3a+ 2b+ 2,

adica inegalitatea ceruta. Avem egalitate pentru a = b = c = 1.

[Nota] In aceleasi conditii, urmatoarea inegalitate mai ”tare” are loc:

a

b+b

c+

1

a≥√

3(a2 + b2 + 1).

Aceasta inegalitate este echivalenta cu

a

(1

b+ b2

)+

1

a≥√

3(a2 + b2 + 1),

496

Page 498: Олимпиадад бэлтгэгчдэд тусламж (Тэнцэтгэл биш 1)

care, prin ridicare la patrat, devine

a2(b4 + 2b− 3 +

1

b2

)+

1

a2≥ b2 + 3− 2

b.

Deoarece

b4 + 2b− 3 +1

b2> 2b− 3 +

1

b2=

(b− 1)2(2b+ 1)

b2≥ 0,

din inegalitatea mediilor rezulta

a2(b4 + 2b− 3 +

1

b2

)+

1

a2≥ 2

√b4 + 2b− 3 +

1

b2,

ramanand de aratat ca

2

√b4 + 2b− 3 +

1

b2≥ b2 + 3− 2

b.

Efectuand o noua ridicare la patrat, obtinem

b5 − 2b3 + 4b2 − 7b+ 4 ≥ 0,

adicab(b2 − 1)2 + 4(b− 1)2 ≥ 0.

Ex. 15 Daca a, b, c sunt numere reale pozitive, atunci

1

a(a+ b)+

1

b(b+ c)+

1

c(c+ d)+

1

d(d+ a)≥ 8

(a+ c)(b+ d).

Proof. Scriem inegalitatea sub forma∑ a(b+ d) + c(b+ d)

a(a+ b)≥ 8,

echivalenta cu ∑ b+ d

a+ b+∑ c(b+ d)

a(a+ b)≥ 8.

Vom arata ca ∑ b+ d

a+ b≥ 4

si ∑ c(b+ d)

a(a+ b)≥ 4.

Tinand seama de inegalitatea

1

x+

1

y≥ 4

x+ y,

echivalenta cu (x− y)2 ≥ 0 pentru x, y > 0, avem∑ b+ d

a+ b= (b+ d)

(1

a+ b+

1

c+ d

)+ (a+ c)

(1

b+ c+

1

a+ d

)497

Page 499: Олимпиадад бэлтгэгчдэд тусламж (Тэнцэтгэл биш 1)

≥ 4(b+ d)

(a+ b) + (c+ d)+

4(a+ c)

(b+ c) + (a+ d)= 4.

De asemenea, aplicand de doua ori inegalitatea mediilor x+ y ≥ 2√xy,

echivalenta cu (√x−√y)2 ≥ 0, avem

∑ c(b+ d)

a(a+ b)= (b+d)

[c

a(a+ b)+

a

c(c+ d)

]+(a+c)

[d

b(b+ c)+

b

d(d+ a)

]

≥ 2(b+ d)√(a+ b)(c+ d)

+2(a+ c)√

(b+ c)(d+ a)≥ 4(b+ d)

(a+ b) + (c+ d)+

4(a+ c)

(b+ c) + (d+ a)= 4.

Egalitatea are loc atunci cand a = c si b = d.

Ex. 16 (Cırtoaje) Daca a, b, c sunt numere reale, atunci

a2 − bc4a2 + b2 + 4c2

+b2 − ca

4b2 + c2 + 4a2+

c2 − ab4c2 + a2 + 4b2

≥ 0.

Proof. Deoarece

4(a2 − bc)4a2 + b2 + 4c2

= 1− (b+ 2c)2

4a2 + b2 + 4c2,

putem scrie inegalitatea sub forma echivalenta

(b+ 2c)2

4a2 + b2 + 4c2+

(c+ 2a)2

4b2 + c2 + 4a2+

(a+ 2b)2

4c2 + a2 + 4b2≤ 3.

Aplicand inegalitatea Cauchy-Schwarz (vezi Ex. 38), avem

(b+ 2c)2

4a2 + b2 + 4c2=

(b+ 2c)2

(2a2 + b2) + 2(2c2 + a2)≤ b2

2a2 + b2+

2c2

2c2 + a2.

In mod similar,

(c+ 2a)2

4b2 + c2 + 4a2≤ c2

2b2 + c2+

2a2

2a2 + b2,

(a+ 2b)2

4c2 + a2 + 4b2≤ a2

2c2 + a2+

2b2

2b2 + c2.

Prin adunarea acestor inegalitati rezulta imediat inegalitatea dorita.Avem egalitate pentru a = b = c, precum si pentru a = 2b = 4c, saub = 2c = 4a, sau c = 2a = 4b.

[Nota] In mod similar putem demonstra ca pentru orice k > 0, avem

a2 − bc2ka2 + b2 + k2c2

+b2 − ca

2kb2 + c2 + k2a2+

c2 − ab2kc2 + a2 + k2b2

≥ 0,

cu egalitate pentru a = b = c, precum si pentru a = kb = k2c, saub = kc = k2a, sau c = ka = k2b.

498

Page 500: Олимпиадад бэлтгэгчдэд тусламж (Тэнцэтгэл биш 1)

4.2 Metoda majorarii si minorarii

Daca ne-am blocat ın demonstrarea directa a inegalitatii A ≤ B, putemıncerca sa gasim o cantitate C care actioneaza ca o punte ıntre A si B:daca avem A ≤ C si C ≤ B, atunci ın mod natural avem A ≤ B. Cualte cuvinte, putem majora pe A cu C, apoi pe C cu B (aceeasi idee seaplica la minorare). Important este de a gasi un nivel corespunzator demajorare sau de minorare.

Ex. 17 Demonstrati ca pentru toate numerele reale pozitive a, b, c, avem

1

a3 + b3 + abc+

1

b3 + c3 + abc+

1

c3 + a3 + abc≤ 1

abc.

[Observatie] Cand demonstram inegalitati cu fractii, adesea este binesa aducem fractiile la un numitor comun. Aceasta operatie se poaterealiza inclusiv prin majorarea sau minorarea numitorilor.

Proof. Intrucat a3 + b3 = (a+ b)(a2 + b2 − ab) ≤ (a+ b)ab, avem

1

a3 + b3 + abc≤ 1

ab(a+ b) + abc=

c

abc(a+ b+ c).

Similar,1

b3 + c3 + abc≤ a

abc(a+ b+ c),

si1

c3 + a3 + abc≤ b

abc(a+ b+ c),

Adunand cele trei inegalitati de mai sus, obtinem

1

a3 + b3 + abc+

1

b3 + c3 + abc+

1

c3 + a3 + abc≤ 1

abc,

cu egalitate daca si numai daca a = b = c.

Ex. 18 Fie a, b, c numere reale pozitive astfel ıncat a+ b+ c = 3. Aratati ca

1 + 8abc ≥ 9 min{a, b, c}.

Proof. Fara a pierde din generalitate, presupunem ca a ≤ b ≤ c. Trebuiesa aratam ca

1 + 8abc ≥ 9a.

Din a + b + c = 3 si a ≤ b ≤ c rezulta a ≤ 1, iar din (a − b)(a − c) ≥ 0obtinem

bc ≥ a(b+ c)− a2 = a(3− a)− a2 = a(3− 2a).

Prin urmare, avem

1 + 8abc− 9a ≥ 1 + 8a2(3− 2a)− 9a = (1− a)(1− 4a)2 ≥ 0.

Egalitatea are loc atunci cand a = b = c = 1, precum si atunci cand

(a, b, c) este o permutare ciclica a tripletului

(1

4,1

4,5

2

).

499

Page 501: Олимпиадад бэлтгэгчдэд тусламж (Тэнцэтгэл биш 1)

Ex. 19 Fie a, b, c numere reale pozitive astfel ıncat a ≤ b ≤ c si ab+ bc+ ca = 3.Aratati ca

1

2a+ b+

1

2b+ c+

1

2c+ a≥ 1.

Proof. Vom utiliza cunoscuta inegalitate

(x+ y + z)2 ≥ 3(xy + yz + zx),

echivalenta cu

(x− y)2 + (y − z)2 + (z − x)2 ≥ 0.

Astfel, pentru x =1

2a+ b, y =

1

2b+ csi z =

1

2c+ a, avem

(1

2a+ b+

1

2b+ c+

1

2c+ a

)2

≥ 9(a+ b+ c)

(2a+ b)(2b+ c)(2c+ a).

Ramane sa aratam ca

9(a+ b+ c) ≥ (2a+ b)(2b+ c)(2c+ a).

Scriem aceasta inegalitate sub forma omogena

3(a+ b+ c)(ab+ bc+ ca) ≥ (2a+ b)(2b+ c)(2c+ a),

echivalenta cuab2 + bc2 + ca2 ≥ a2b+ b2c+ c2a,

sau(a− b)(b− c)(c− a) ≥ 0.

Pentru a ≤ b ≤ c, ultima inegalitate este evident adevarata. Egalitateaare loc pentru a = b = c.

Ex. 20 (Schur) Daca a, b, c sunt numere reale nenegative, atunci

(1) a(a− b)(a− c) + b(b− c)(b− a) + c(c− a)(c− b) ≥ 0;

(2) a2(a− b)(a− c) + b2(b− c)(b− a) + c2(c− a)(c− b) ≥ 0.

Proof. Deoarece inegalitatea este simetrica ın raport cu a, b, c, fara apierde din generalitate vom considera ca a ≥ b ≥ c.(1) In ipotez a ≥ b ≥ c, este suficient sa aratam ca

a(a− b)(a− c) + b(b− c)(b− a) ≥ 0.

Intr-adevar, avem

a(a− b)(a− c) + b(b− c)(b− a) = (a− b)2(a+ b− c) ≥ 0.

Egalitatea are loc atunci cand a = b = c, precum si atunci cand unuldintre numerele a, b, c este 0, iar celelalte sunt egale.

500

Page 502: Олимпиадад бэлтгэгчдэд тусламж (Тэнцэтгэл биш 1)

(2) In mod similar, este suficient sa aratam ca

a2(a− b)(a− c) + b2(b− c)(b− a) ≥ 0.

Intr-adevar,

a2(a− b)(a− c) + b2(b− c)(b− a) = (a− b)2(a2 + ab+ b2 − bc− ca) ≥ 0.

Egalitatea are loc atunci cand a = b = c, precum si atunci cand unuldintre numerele a, b, c este 0, iar celelalte sunt egale.

[Nota 1] Inegalitatea Schur de gradul trei are urmatoarele forme echiva-lente, frecvent utilizate ın demonstrarea altor inegalitati:

a3 + b3 + c3 + 3abc ≥ ab(a+ b) + bc(b+ c) + ca(c+ a)

si

(a+ b+ c)3 + 9abc ≥ 4(a+ b+ c)(ab+ bc+ ca).

[Nota 2] Inegalitatea Schur de gradul patru este adevarata pentru oricenumere reale a, b, c si are formele echivalente

a4 + b4 + c4 + abc(a+ b+ c) ≥ ab(a2 + b2) + bc(b2 + c2) + ca(c2 + a2)

si

6abcp ≥ (p2 − q)(4q − p2),

unde p = a+ b+ c si q = ab+ bc+ ca.

Ex. 21 Fie a, b, c numere reale pozitive astfel ıncat a ≤ 1 ≤ b ≤ c si

a+ b+ c =1

a+

1

b+

1

c.

Aratati ca1

a2+

1

b2+

1

c2≥ a2 + b2 + c2.

Proof. Scriem inegalitatea sub forma

(a2 + c2)

(1

a2c2− 1

)≥ b2 − 1

b2,

sau

(a2 + c2)

(1

ac− 1

)(1

ac+ 1

)≥(b− 1

b

)(b+

1

b

).

Din conditia a+ b+ c =1

a+

1

b+

1

c, rezulta

(a+ c)

(1

ac− 1

)= b− 1

b≥ 0.

501

Page 503: Олимпиадад бэлтгэгчдэд тусламж (Тэнцэтгэл биш 1)

Astfel, trebuie sa aratam ca

(a2 + c2)

(1

ac+ 1

)≥ (a+ c)

(b+

1

b

).

Din inegalitatea evidenta (b− c)(

1

bc− 1

)≥ 0, rezulta

c+1

c≥ b+

1

b.

Prin urmare, este suficient sa demonstram ca

(a2 + c2)

(1

ac+ 1

)≥ (a+ c)

(c+

1

c

).

Intr-adevar, avem

(a2 + c2)

(1

ac+ 1

)− (a+ c)

(c+

1

c

)=

(1− a2)(c− a)

a≥ 0.

Egalitatea are loc pentru b = 1 si ac = 1.

Ex. 22 Fie ai ≥ 1 (i = 1, 2, · · · , n). Demonstrati ca

(1 + a1)(1 + a2) · · · (1 + an) ≥ 2n

n+ 1(1 + a1 + a2 + · · ·+ an).

[Observatie] Examinand cele doua parti ale inegalitatii de mai sus, cumputem proceda pentru a obtine 2n si ın membrul stang?

Proof. Avem

(1 + a1)(1 + a2) · · · (1 + an)

= 2n(1 +a1 − 1

2)(1 +

a2 − 1

2) · · · (1 +

an − 1

2).

Intrucat ai − 1 ≥ 0, rezulta

(1 + a1)(1 + a2) · · · (1 + an)

≥ 2n(1 +a1 − 1

2+a2 − 1

2+ · · ·+ an − 1

2)

≥ 2n(1 +a1 − 1

n+ 1+a2 − 1

n+ 1+ · · ·+ an − 1

n+ 1)

=2n

n+ 1[n+ 1 + (a1 − 1) + (a2 − 1) + · · ·+ (an − 1)]

=2n

n+ 1(1 + a1 + a2 + · · ·+ an).

Am obtinut astfel inegalitatea initiala. Pentru n ≥ 2, egalitatea are locatunci cand a1 = a2 = · · · = an = 1.

502

Page 504: Олимпиадад бэлтгэгчдэд тусламж (Тэнцэтгэл биш 1)

Ex. 23 Gasiti cel mai mare numar real α, astfel ıncat

x√y2 + z2

+y√

x2 + z2+

z√x2 + y2

≥ α

pentru orice numere reale nenegative x, y, z, dintre care cel putin douanenule.

Proof. Pentru x = 0 si y = z, membrul stang al inegalitatii este egalcu 2. Prin urmare, α nu poate fi mai mare ca 2. Sa consideram asadarα = 2.

Fara a pierde din generalitate, presupunem ca x ≤ y ≤ z. Vom arata ca

x√y2 + z2

+y√

x2 + z2+

z√x2 + y2

≥√x2 + y2√x2 + z2

+

√x2 + z2√x2 + y2

≥ 2.

Inegalitatea din dreapta rezulta imediat din inegalitatea mediilor. Scriemacum inegalitatea din stanga sub forma

x√y2 + z2

≥√x2 + y2 − y√x2 + z2

+

√x2 + z2 − z√x2 + y2

,

sau

x√y2 + z2

≥ x2√x2 + z2(

√x2 + y2 + y)

+x2√

x2 + y2(√x2 + z2 + z)

.

Deoarece√x2 + y2 + y > 2y si

√x2 + z2 + z > 2z, este suficient sa

aratam ca1√

y2 + z2≥ x

2y√x2 + z2

+x

2z√x2 + y2

.

In cazul netrivial x > 0, avem

x√x2 + z2

=1√

1 + ( zx)2≤ 1√

1 + ( zy )2=

y√y2 + z2

six√

x2 + y2=

1√1 + ( yx)2

≤ 1√2.

Prin urmare, este suficient sa demonstram inegalitatea

1√y2 + z2

≥ 1

2√y2 + z2

+1

2√

2z,

care este echivalenta cu inegalitatea evidenta

√2z ≥

√y2 + z2.

Astfel, inegalitatea ceruta este demonstrata. Prin urmare, αmax = 2. Inipoteza x ≤ y ≤ z, egalitatea are loc pentru x = 0 si y = z.

503

Page 505: Олимпиадад бэлтгэгчдэд тусламж (Тэнцэтгэл биш 1)

Ex. 24 Daca x, y, z ∈ [1, 4], atunci

8

(x

y+y

z+z

x

)≥ 5

(y

x+z

y+x

z

)+ 9.

Proof. Fie

E(x, y, z) = 8

(x

y+y

z+z

x

)− 5

(y

x+z

y+x

z

)− 9.

Deoarece inegalitatea este ciclica, fara a pierde din generalitate, pre-supunem ca x = max{x, y, z}. Vom arata ca

E(x, y, z) ≥ E(x,√xz, z) ≥ 0.

Avem

E(x, y, z)− E(x,√xz, z) = 8

(x

y+y

z− 2

√x

z

)− 5

(y

x+z

y− 2

√z

x

)

=(y −

√xz)2(8x− 5z)

xyz≥ 0.

De asemenea, cu notatia t =

√x

z, 1 ≤ t ≤ 2, avem

E(x,√xz, z) = 8

(2

√x

z+z

x− 3

)− 5

(2

√z

x+x

z− 3

)

= 8

(2t+

1

t2− 3

)−5

(2

t+ t2 − 3

)=

8

t2(t−1)2(2t+1)− 5

t(t−1)2(t+2)

=(t− 1)2(4 + 5t)(2− t)

t2≥ 0.

Avem egalitate atunci cand x = y = z, precum si atunci cand (x, y, z)este o permutare ciclica a tripletului (4, 2, 1).

4.3 Metoda coeficientilor nedeterminati

Demonstrarea unor inegalitati se poate face prin spargerea acestora ıninegalitıti mai simple, cu coeficienti nedeterminati, dar care urmeaza afi calculati astfel ıncat inegalitatile respective sa fie adevarate, sau sa seajunga la rezultatul dorit. In multe cazuri, succesul metodei depinde deigeniozitatea si experienta rezolvitorului.

Ex. 25 Daca x, y, z ∈ [1, 2], atunci

3

(1

x+ 2y+

1

y + 2z+

1

z + 2x

)≥ 2

(1

x+ y+

1

y + z+

1

z + x

).

504

Page 506: Олимпиадад бэлтгэгчдэд тусламж (Тэнцэтгэл биш 1)

Proof. Presupunem ca exista o constanta reala α astfel ıncat

3

x+ 2y− 2

x+ y+ α

(1

x− 1

y

)≥ 0

pentru orice x, y ∈ [1, 2]. Atunci, ınsumand aceasta inegalitate cu cele-lalte doua similare, obtinem inegalitatea dorita. Deoarece

3

x+ 2y− 2

x+ y+ α

(1

x− 1

y

)=

(x− y)[xy − α(x+ y)(x+ 2y)]

xy(x+ y)(x+ 2y),

alegand α =1

6, rezulta

3

x+ 2y− 2

x+ y+

1

6

(1

x− 1

y

)=

(x− y)2(2y − x)

6xy(x+ y)(x+ 2y)≥ 0.

Avem egalitate daca si numai daca x = y = z.

Ex. 26 Fie x, y, z numere reale, nu toate nule. Gasiti valoarea maxima a fractiei

xy + 2yz

x2 + y2 + z2.

Proof. Pentru a gasi valoarea maxima ceruta, este suficient sa aratamca exista o constanta c, astfel ıncat

xy + 2yz

x2 + y2 + z2≤ c,

si ca semnul egal se obtine cel putin pentru un triplet x, y, z dat. Deoareceambii termeni ai sumei xy+2yz contin variabila y, vom sparge termenuly2 al sumei x2 + y2 + z2 ın αy2 si (1− α)y2. Din inegalitatea mediilor,avem

x2 + αy2 ≥ 2√α · xy

si(1− α)y2 + z2 ≥ 2

√1− α · yz,

iar prin ınsumarea acestor inegalitati obtinem

x2 + y2 + z2 ≥ 2√α · xy + 2

√1− α · yz,

x2 + y2 + z2 ≥ 2√α

(xy +

√1− αα· yz

).

Pentru a avea

√1− αα

= 2, trebuie sa alegem α =1

5. Rezulta

xy + 2yz

x2 + y2 + z2≤√

5

2,

cu egalitate pentru x =y√5

=z

2. Prin urmare, fractia

xy + 2yz

x2 + y2 + z2are

valoarea maxima

√5

2.

505

Page 507: Олимпиадад бэлтгэгчдэд тусламж (Тэнцэтгэл биш 1)

Ex. 27 Fie x, y, z numere reale nenegative, dintre care cel putin doua nenule.Demonstrati ca

x√y2 + z2

+y√

x2 + z2+

z√x2 + y2

≥ 2.

Proof. Daca exista o constanta reala α astfel ıncat

x√y2 + z2

≥ 2xα

xα + yα + zα

pentru orice numere reale nenegative x, y, z, atunci obtinem inegali-tatea dorita prin ınsumarea acestei inegalitati cu celelalte doua similare.Deoarece

xα + yα + zα ≥ 2√xα(yα + zα),

inegalitatea de mai sus este adevarata daca

x√y2 + z2

≥ xα√xα(yα + zα)

,

adicayα + zα ≥ xα−2(y2 + z2).

Este usor de observat ca ultima inegalitate este verificata ca identitatepentru α = 2. Cu aceasta, demonstratia este ıncheiata. Inegalitateainitiala devine egalitate cand unul dintre numerele x, y, z este 0, iar cele-lalte doua sunt egale.

Ex. 28 (Cirtoaje) Fie x1, x2, ..., xn numere reale pozitive astfel ıncat x1x2...xn =1. Aratati ca

1

1 + (n− 1)x1+

1

1 + (n− 1)x2+ ...+

1

1 + (n− 1)xn≥ 1.

Proof. Presupunem ca exista o constanta reala k astfel ıncat

1

1 + (n− 1)xi≥ xkixk1 + xk2 + ...+ xkn

pentru i ∈ {1, 2, ..., n}. Atunci, ınsumand toate aceste inegalitati pentrui = 1, 2, ..., n, obtinem inegalitatea dorita. Scriem inegalitatea de maisus sub forma

xk1 + ...+ xki−1 + xri+1 + ...+ xkn ≥ (n− 1)xk+1i .

Din inegalitatea mediilor, avem

xk1 + ...+ xki−1 + xki+1 + ...+ xkn ≥ (n− 1) n−1

√(x1...xi−1xi+1...xn)k

= (n− 1)x−kn−1

i ,

iar pentru k =1

n− 1 obtinem chiar inegalitatea dorita. Cu aceasta,

inegalitatea este demonstrata. Avem egalitate daca si numai daca x1 =x2 = ... = xn = 1.

506

Page 508: Олимпиадад бэлтгэгчдэд тусламж (Тэнцэтгэл биш 1)

Ex. 29 [Cırtoaje] Fie a, b, c, d numere reale pozitive astfel ıncat abcd = 1. Aratatica

1

a+ b+ 2+

1

b+ c+ 2+

1

c+ d+ 2+

1

d+ a+ 2≤ 1.

Proof. Daca exista o constanta reala k astfel ıncat

2

a+ b+ 2≤ ck + dk

ak + bk + ck + dk,

atunci putem obtine inegalitatea dorita prin ınsumarea acestei inegalitaticu celelalte inegalitati similare. Inegalitatea de mai sus este echivalentacu

(a+ b)(ck + dk) ≥ 2(ak + bk).

Deoarece

ck + dk ≥ 2(cd)k/2 =2

(ab)k/2,

este suficient sa aratam ca

a+ b ≥ (ab)k/2(ak + bk).

Aceasta inegalitate este omogena pentru k =1

2. In acest caz, inegalitatea

devinea+ b ≥ 4

√ab(√a+√b),

sau(

4√a3 − 4

√b3)( 4√a− 4√b) ≥ 0,

ultima forma fiind evident adevarata. Egalitatea are loc pentru a = b =c = d = 1.

[Nota] In general, pentru a1, a2, ..., an numere reale pozitive satisfacanda1a2...an = 1, are loc inegalitatea∑ 1

a1 + a2 + ...+ ak + n− k≤ 1,

unde 1 ≤ k ≤ n− 1.

Ex. 30 Fie a, b, c numere reale pozitive astfel ıncat a+ b+ c = 3. Aratati ca

1

a2+

1

b2+

1

c2≥ a2 + b2 + c2.

Proof. Daca exista o constanta reala k astfel ıncat

1

a2− a2 + k(a− 1) ≥ 0,

atunci putem obtine inegalitatea dorita prin ınsumarea acestei inegalitaticu celelalte inegalitati similare. Deoarece

1

a2− a2 + k(a− 1) =

(1− a)[1 + a+ (1− k)a2 + a3)

a2,

507

Page 509: Олимпиадад бэлтгэгчдэд тусламж (Тэнцэтгэл биш 1)

alegand k = 4, rezulta

1

a2− a2 + 4(a− 1) =

(1− a)2(1 + 2a− a2)a2

.

Prin urmare, pentru a ≤ 1 +√

2, avem

1

a2− a2 + 4(a− 1) ≥ 0

In continuare, fara a pierde din generalitate, consideram a ≥ b ≥ c.Avem doua cazuri.

Cazul 1. a ≤ 1 +√

2. Prin ınsumarea inegalitatilor

1

a2− a2 + 4(a− 1) ≥ 0,

1

b2− b2 + 4(b− 1) ≥ 0,

1

c2− c2 + 4(c− 1) ≥ 0,

obtinem inegalitatea dorita.

Cazul 2. a > 1 +√

2. Deoarece b+ c = 3− a < 2−√

2 <2

3, avem

bc ≤ 1

4(b+ c)2 <

1

9,

deci

1

a2+

1

b2+

1

c2>

1

a2+

1

b2≥ 2

bc> 18 > (a+ b+ c)2 > a2 + b2 + c2.

Cu aceasta, inegalitatea este demonstrata. Egalitatea are loc numai ıncazul a = b = c = 1.

Ex. 31 Fie a, b, c, d numere reale nenegative astfel ıncat

a2 + b2 + c2 + d4 = a+ b+ c+ d.

Aratati caa4 + b4 + c4 + d4 ≥ a+ b+ c+ d.

Proof. Pentru orice α ∈ R, inegalitatea dorita este echivalenta cu fiecaredin inegalitatile de mai jos:

a4 + b4 + c4 + d4 +α(a+ b+ c+ d) ≥ a+ b+ c+ d+α(a2 + b2 + c2 + d2),∑a(a3 − αa+ α− 1) ≥ 0,∑

a(a− 1)(a2 + a− α+ 1) ≥ 0.

Alegand α = 3, inegalitatea devine astfel∑a(a+ 2)(a− 1)2 ≥ 0,

fiind evident adevarata. Egalitatea are loc atunci cand a, b, c, d ∈ {0, 1}.

508

Page 510: Олимпиадад бэлтгэгчдэд тусламж (Тэнцэтгэл биш 1)

Ex. 32 (Ostrowski) Fie trei seturi de numere reale a1, a2, · · · , an, b1, b2, · · · , bnsi x1, x2, · · · , xn care satisfac relatiile

a1x1 + a2x2 + · · ·+ anxn = 0,

b1x1 + b2b2 + · · ·+ bnxn = 1.

Demonstrati ca

x21 + x22 + · · ·+ x2n ≥A

AB − C2,

unde

A =

n∑i=1

a2i 6= 0, B =

n∑i=1

b2i , C = (

n∑i=1

aibi)2.

Proof. Oricare ar fi numerele reale α si β, avem

n∑i=1

x2i =n∑i=1

x2i + αn∑i=1

aixi + β(n∑i=1

bixi − 1),

n∑i=1

x2i =

n∑i=1

(xi +αai + βbi

2)2 −

n∑i=1

(αai + βbi)2

4− β,

decin∑i=1

x2i ≥ −n∑i=1

(αai + βbi)2

4− β, (4.1)

cu egalitate daca si numai daca

xi = −αai + βbi2

, i = 1, 2, · · · , n. (4.2)

Inlocuind (4.2) ın relatiile∑n

i=1 aixi = 0 si∑n

i=1 bixi = 1, obtinem

αA+ βC = 0, αC + βB = −1,

deci

α =2C

AB − C2, β = − 2A

AB − C2. (4.3)

Astfel, din (4.1) si (4.3) rezulta

n∑i=1

x2i ≥A

AB − C2,

adica inegalitatea ceruta. Din (4.2) si (4.3) rezulta ca egalitatea are locatunci cand

xi =Abi − CaiAB − C2

, i = 1, 2, · · · , n.

509

Page 511: Олимпиадад бэлтгэгчдэд тусламж (Тэнцэтгэл биш 1)

[Nota] Inegalitatea de mai sus poate fi mai simplu demonstrata utilizandinegalitatea Cauchy-Schwarz (vezi Ex. 38). Astfel, pentru t ∈ R, avem

[

n∑i=1

(ait+ bi)2] · (

n∑i=1

x2i ) ≥ [

n∑i=1

(ait+ bi)xi]2 = 1,

(At2 + 2Ct+B)(x21 + x22 + · · ·x2n)− 1 ≥ 0,

At2 + 2Ct+B − 1

x21 + x22 + · · ·+ x2n≥ 0.

Deoarece A > 0, ultima inegalitate este adevarata pentru orice t realdaca si numai daca ∆ ≤ 0, unde

∆ = 4C2 − 4A(B − x21 − x22 − · · · − x2n)

este discriminantul trinomului de gradul doi ın t din membrul stang alinegalitatii. Din conditia ∆ ≤ 0 obtinem inegalitatea dorita.

4.4 Metoda normalizarii

Cand o inegalitate f(x1, x2, ..., xn) ≥ 0 este omogena de ordinul k, adicaf(tx1, tx2, ..., txn) = tkf(x1, x2, ..., xn), putem presupune, fara a pierdedin generalitate, ca suma variabilelor este constanta, ın scopul de a aduceinegalitatea la o forma mai simpla, mai usor demonstrabila. In gen-eral, putem considera ca g(x1, x2, ..., xn) = const, unde g(x1, x2, ..., xn)este o expresie omogena de ordin arbitrar, sau chiar f1(x1, x2, ..., xn) =f2(x1, x2, ..., xn), unde f1(x1, x2, ..., xn) si f2(x1, x2, ..., xn) sunt expresiiomogene de ordin diferit.

Ex. 33 Daca a, b, c sunt numere reale pozitive, atunci

(2a+ b+ c)2

2a2 + (b+ c)2+

(2b+ c+ a)2

2b2 + (c+ a)2+

(2c+ a+ b)2

2c2 + (a+ b)2≤ 8.

Proof. Deoarece inegalitatea este omogena, presupunem ca a+b+c = 3.Trebuie sa aratam ca

(a+ 3)2

2a2 + (3− a)2+

(b+ 3)2

2b2 + (3− b)2+

(c+ 3)2

2c2 + (3− c)2≤ 8,

adica

f(a) + f(b) + f(c) ≤ 8,

unde

f(x) =(x+ 3)2

2x2 + (3− x)2, x ∈ R+.

Avem

f(x) =x2 + 6x+ 9

3(x2 − 2x+ 3)=

1

3(1 +

8x+ 6

(x− 1)2 + 2)

510

Page 512: Олимпиадад бэлтгэгчдэд тусламж (Тэнцэтгэл биш 1)

≤ 1

3(1 +

8x+ 6

2) =

4(x+ 1)

3.

Asadar,

f(a) + f(b) + f(c) ≤ 4(a+ 1 + b+ 1 + c+ 1)

3= 8,

ceea ce trebuia demonstrat. Avem egalitate pentru a = b = c.

Ex. 34 Daca a, b, c si x, y, z sunt numere reale nenegative, atunci

[(b+ c)x+ (c+ a)y + (a+ b)z]2 ≥ 4(ab+ bc+ ca)(xy + yz + zx).

Proof. Scriem inegalitatea sub forma

2√

(ab+ bc+ ca)(xy + yz + zx) ≤ (b+ c)x+ (c+ a)y + (a+ b)z.

Deoarece inegalitatea este omogena ın raport cu x, y, z, putem consideraca x+ y+ z = a+ b+ c. In aceasta ipoteza, inegalitatea ceruta se obtinedin inegalitatea mediilor, astfel:

2√

(ab+ bc+ ca)(xy + yz + zx) ≤ (ab+ bc+ ca) + (xy + yz + zx)

=(a+ b+ c)2 − a2 − b2 − c2

2+

(x+ y + z)2 − x2 − y2 − z2

2

= (a+ b+ c)(x+ y + z)− a2 + x2

2− b2 + y2

2− c2 + z2

2

≤ (a+ b+ c)(x+ y + z)− ax− by − cz = (b+ c)x+ (c+ a)y + (a+ b)z.

In cazul abc 6= 0, egalitatea are loc daca si numai dacax

a=y

b=z

c.

Ex. 35 Fie a, b, c numere reale astfel ıncat a+ b+ c > 0 si b2 ≥ 4ac. Aratati ca

4 min{a, b, c} ≤ a+ b+ c ≤ 9

4max{a, b, c}.

Proof. Deoarece conditiile date si inegalitatea ceruta sunt omogene ınraport cu a, b, c, presupunem, fara a pierde din generalitate, ca a+b+c =1.

(A) Sa aratam ca

max{a, b, c} ≥ 4

9. (4.4)

Consideram cazul netrivial b < 49 . Din a + c = 1 − b > 5

9 , rezulta caa ≤ 1

9 implica c > 49 , iar c ≤ 1

9 implica a > 49 , inegalitatea ceruta

fiind adevarata ın ambele cazuri. Ramane de analizat cazul b < 49 si

a, c > 19 . Din b2 ≥ 4ac rezulta ac < 4

81 , deci (59 − c) · c < ac < 481 , iar din

(59−c) ·c <481 , obtinem c > 4

9 . Avem egalitate ın (4.4) pentru a = b = 49

si c = 19 , sau b = c = 4

9 si a = 19 .

(B) Sa aratam ca

min{a, b, c} ≤ 1

4. (4.5)

511

Page 513: Олимпиадад бэлтгэгчдэд тусламж (Тэнцэтгэл биш 1)

Consideram cazul netrivial a > 14 si b, c > 0. Din b2 ≥ 4ac > c, rezulta

b >√c. Prin urmare,

√c+ c < b+ c = 1− a < 3

4,

iar din√c + c < 3

4 , obtinem c < 14 . Avem egalitate ın (4.5) pentru

a = c = 14 si b = 1

2 .

Ex. 36 Daca a, b, c sunt numere reale pozitive ıncat

7(a2 + b2 + c2) = 11(ab+ bc+ ca),

atunci51

28≤ a

b+ c+

b

c+ a+

c

a+ b≤ 2.

Proof. Din considerente de omogenitate, fara a pierde din generalitate,

presupunem ca b+ c = 2. Fie x = bc, x ≤ (b+ c)2

4≤ 1. Din relatia data,

rezulta

x =7a2 − 22a+ 28

25,

iar x ≤ 1 implica1

7≤ a ≤ 3. Pe de alta parte,

a

b+ c+

b

c+ a+

c

a+ b=

a

b+ c+a(b+ c) + (b+ c)2 − 2bc

a2 + a(b+ c) + bc

=a

2+

2(a+ 2− x)

a2 + 2a+ x=

4a3 + 27a+ 11

8a2 + 7a+ 7,

iar inegalitatile cerute devin astfel

51

28≤ 4a3 + 27a+ 11

8a2 + 7a+ 7≤ 2.

Aceste inegalitati sunt adevarate deoarece

4a3 + 27a+ 11

8a2 + 7a+ 7− 51

28=

(7a− 1)(4a− 7)2

28(8a2 + 7a+ 7)≥ 0

si

2− 4a3 + 27a+ 11

8a2 + 7a+ 7=

(3− a)(2a− 1)2

8a2 + 7a+ 7≥ 0.

Avem egalitate ın inegalitatea stanga pentru 7a = b = c (sau oricepermutare ciclica). In inegalitatea din dreapta, egalitatea are loc pentrua

3= b = c (sau orice permutare ciclica).

Ex. 37 (Cırtoaje) Daca a, b, c sunt numere reale pozitive astfel ıncat a ≥ b ≥ c,atunci

a+ b+ c− 33√abc ≤ 2(

√a−√c)2

512

Page 514: Олимпиадад бэлтгэгчдэд тусламж (Тэнцэтгэл биш 1)

Proof. Din considerente de omogenitate, fara a pierde din generalitate,presupunem abc = 1. In acest caz, inegalitatea devine astfel

a+ c− b ≥ 4√b− 3.

Din (a− b)(b− c) ≥ 0, rezulta

a+ c− b ≥ ac

b=

1

b2.

Prin urmare, este suficient sa aratam ca

1

b2≥ 4√

b− 3.

Cu notatia x =1√b, rezulta

1

b2− 4√

b+ 3 = x4 − 4x+ 3 = (x− 1)2(x2 + 2x+ 3) ≥ 0.

Cu aceasta, demonstratia este ıncheiata. Egalitatea are loc pentru a =b = c.

Ex. 38 (Cauchy-Schwarz) Daca a1, a2, ..., an si x1, x2, ..., xn sunt numere reale,atunci

(a21 + a22 + ...+ a2n)(x21 + x22 + ...+ x2n) ≥ (a1x1 + a2x2 + ...+ anxn)2.

Proof. Inegalitatea este omogena atat ın raport cu variabilele a1, a2, ..., an,cat si cu variabilele x1, x2, ..., xn. Prin urmare, ın cazul netrivial ın carea21 + a22 + ...+ a2n 6= 0 si x21 + x22 + ...+ x2n 6= 0, putem presupune ca

a21 + a22 + ...+ a2n = n,

x21 + x22 + ...+ x2n = n,

pentru a demonstra ca

n ≥ |a1x1 + a2x2 + ...+ anxn|.

Intr-adevar, avem

n− |a1x1 + a2x2 + ...+ anxn| ≥ n− |a1||x1| − |a2||x2| − ...− |an||xn|

=a21 + a22 + ...+ a2n

2+x21 + x22 + ...+ x2n

2−|a1||x1|−|a2||x2|−...−|an||xn|

=(|a1| − |x1|)2 + (|a2| − |x2|)2 + ...+ (|an| − |xn|)2

2≥ 0.

Avem egalitate atunci cand seturile a1, a2, ..., an si x1, x2, ..., xn suntproportionale, adica aixi+1 = ai+1xi pentru i = 1, 2, ..., n− 1.

513

Page 515: Олимпиадад бэлтгэгчдэд тусламж (Тэнцэтгэл биш 1)

[Nota 1] Putem demonstra mai simplu inegalitatea Cauchy-Schwarzaratand ca

x21 + x22 + ...+ x2n = a1x1 + a2x2 + ...+ anxn

implicaa21 + a22 + ...+ a2n ≥ a1x1 + a2x2 + ...+ anxn.

Intr-adevar,

a21+a22+...+a

2n−a1x1−a2x2−...−anxn = (a21+a

22+...+a

2n)+(x21+a

22+...+a

2n)

−2(a1x1+a2x2+...+anxn) = (a1−x1)2+(a2−x2)2+...+(an−xn)2 ≥ 0.

[Nota 2] O alta demonstratie a inegalitatii Cauchy-Schwarz poate fidata pornind de la inegalitatea evidenta

(a1t− x1)2 + (a2t− x2)2 + ...+ (ant− xn)2 ≥ 0,

echivalenta cu

(a21 +a22 + ...+a2n)t2−2(a1x1 +a2x2 + ...+anxn)t+x21 +x22 + ...+x2n ≥ 0,

unde t este un numar real arbitrar. In cazul netrivial a21+a22+...+a2n 6= 0,inegalitatea Cauchy-Schwarz rezulta din conditia necesara si suficienta∆ ≤ 0, unde ∆ este discriminantul trinomului de gradul doi ın t dinmembrul stang al inegalitatii.

Ex. 39 (Cırtoaje) Daca a1, a2, ..., an si x1, x2, ..., xn sunt numere reale, atunci

a1x1 + a2x2 + ...+ anxn +√

(a21 + a22 + ...+ a2n)(x21 + x22 + ...+ x2n)

≥ 2

n(a1 + a2 + ...+ an)(x1 + x2 + ...+ xn).

Proof. Deoarece inegalitatea este omogena ın raport cu variabilele x1, x2, ..., xn,fara a pierde din generalitate vom considera ca

x21 + x22 + ...+ x2n = a21 + a22 + ...+ a2n.

Inegalitatea de demonstrat devine astfel

(a1+x1)2+(a2+x2)

2+...+(an+xn)2 ≥ 4

n(a1+a2+...+an)(x1+x2+...+xn).

Deoarece

[(a1+a2+...+an)+(x1+x2+...+xn)]2 ≥ 4(a1+a2+...+an)(x1+x2+...+xn),

este suficient sa aratam ca

(a1+x1)2+(a2+x2)

2+...+(an+xn)2 ≥ 1

n[(a1+a2+...+an)+(x1+x2+...+xn)]2.

Cu notatiile yi = ai + xi, i = 1, 2, ..., n, obtinem inegalitatea cunoscuta

n(y21 + y22 + ...+ y2n) ≥ (y1 + y2 + ...+ yn)2,

care rezulta imediat din inegalitatea Cauchy-Schwarz. Egalitatea are locdaca si numai daca seturile (2x−x1, 2x−x2, ..., 2x−xn) si (a1, a2, ..., an)

sunt proportionale, unde x =x1 + x2 + ...+ xn

n.

514

Page 516: Олимпиадад бэлтгэгчдэд тусламж (Тэнцэтгэл биш 1)

[Nota] Pentru xi =1

ai, i = 1, 2, ..., n, obtinem urmatoarea inegalitate

n2 + n

√(a21 + a22 + ...+ a2n)

(1

a21+

1

a22+ ...+

1

a2n

)

≥ 2(a1 + a2 + ...+ an)

(1

a1+

1

a2+ ...+

1

an

).

Ex. 40 (Cırtoaje) Fie a, b, c numere reale pozitive astfel ıncat a ≥ b ≥ c. Aratatica

a+ b+ c− 33√abc ≤ 2(a− c)2

a+ 5c.

Proof. Vom arata ca

a+ b+ c− 33√abc ≤ 2a+ c− 3

3√a2c ≤ 2(a− c)2

a+ 5c.

Inegalitatea din dreapta devine egalitate pentru c = 0. Pentru c > 0, dinconsiderente de omegenitate, vom presupune c = 1. In plus, cu notatiaa = x3, x ≥ 1, inegalitatea devine succesiv astfel:

2(x3 − 1)2 ≥ (x3 + 5)(2x3 − 3x2 + 1),

(x− 1)2[2(x2 + x+ 1)2 − (x3 + 5)(2x+ 1)] ≥ 0,

(x− 1)2(x3 + 2x2 − 2x− 1) ≥ 0,

(x− 1)3(x2 + 3x+ 1) ≥ 0,

ultima inegalitate fiind evident adevarata. Scriem acum inegalitatea dinstanga sub forma

a− b− 3 3√ac( 3√a− 3√b) ≥ 0,

sau( 3√a− 3√b)(

3√a2 +

3√ab+

3√b2 − 3 3

√ac) ≥ 0.

Ultima inegalitate este adevarata, deoarece

3√a2 +

3√ab+

3√b2 − 3 3

√ac ≥ 3

3√ab− 3 3

√ac = 3 3

√a(

3√b− 3√c) ≥ 0.

Cu aceasta, inegalitatea este demonstrata. Avem egalitate pentru a =b = c, precum si pentru a = b si c = 0.

4.5 Metoda omogenizarii

Metoda omogenizarii poate fi aplicata la demonstrarea inegalitatilor nor-malizate, atunci cand acestea devin mai simple prin omogenizare. Un-eori, pentru demonstrarea unei inegalitati normalizate se poate utilizamai ıntai metoda omogenizarii, apoi metoda normalizarii (a doua nor-malizare fiind, desigur, diferita de prima).

515

Page 517: Олимпиадад бэлтгэгчдэд тусламж (Тэнцэтгэл биш 1)

Ex. 41 Fie a, b, c numere reale nenegative astfel ıncat a + b = a4 + b4. Aratatica

a3 + b3 ≥ a2 + b2.

Proof. Scriem inegalitatea sub forma omogena

(a+ b)(a3 + b3)3 ≥ (a4 + b4)(a2 + b2)3,

care este echivalenta cuA− 3B ≥ 0,

undeA = (a+ b)(a9 + b9)− (a4 + b4)(a6 + b6),

B = a2b2(a4 + b4)(a2 + b2)− a3b3(a+ b)(a3 + b3).

DeoareceA = ab(a3 − b3)(a5 − b5)

siB = a2b2(a− b)(a5 − b5),

rezultaA− 3B = ab(a− b)3(a5 − b5) ≥ 0.

Avem egalitate daca si numai daca a = b = 1.

Ex. 42 Fie a, b, c numere reale nenegative astfel ıncat ab+ bc+ ca = 3. Aratatica

1

a2 + 1+

1

b2 + 1+

1

c2 + 1≥ 3

2.

Proof. In urma dezvoltarii, inegalitatea devine astfel

a2 + b2 + c2 + 3 ≥ a2b2 + b2c2 + c2a2 + 3a2b2c2.

Din cunoscuta inegalitate

(a+ b+ c)(ab+ bc+ ca) ≥ 9abc,

echivalenta cu

a(b− c)2 + b(c− a)2 + c(a− b)2 ≥ 0,

rezultaa+ b+ c ≥ 3abc.

Astfel, este suficient sa aratam ca

a2 + b2 + c2 + 3 ≥ a2b2 + b2c2 + c2a2 + abc(a+ b+ c).

Scriem aceasta inegalitate sub forma omogena

(ab+ bc+ ca)(a2 + b2 + c2) + (ab+ bc+ ca)2 ≥ 3(a2b2 + b2c2 + c2a2)

+3abc(a+ b+ c),

516

Page 518: Олимпиадад бэлтгэгчдэд тусламж (Тэнцэтгэл биш 1)

echivalenta cu

ab(a2 + b2) + bc(b2 + c2) + ca(c2 + a2) ≥ 2(a2b2 + b2c2 + c2a2),

sauab(a− b)2 + bc(b− c)2 + ca(c− a)2 ≥ 0.

Avem egalitate atunci cand a = b = c = 1, precum si atunci cand (a, b, c)este o permutare ciclica a tripletului (0,

√3,√

3).

Ex. 43 Fie a, b, c, d numere reale pozitive astfel ıncat a+ b+ c+ d = 4. Aratatica

a

a2 + 1+

b

c2 + 1+

c

d2 + 1+

d

a2 + 1≥ 2.

Proof. Avem

a

b2 + 1= a− ab2

b2 + 1≥ a− ab2

2b= a− ab

2.

In mod similar,

b

c2 + 1≥= b− bc

2,

c

d2 + 1≥= c− cd

2,

d

a2 + 1≥= d− da

2.

Prin urmare, este suficient sa aratam ca

a+ b+ c+ d− ab+ bc+ cd+ da

2≥ 2,

adica4 ≥ ab+ bc+ cd+ da.

Scriem aceasta inegalitate sub forma omogena

(a+ b+ c+ d)2 ≥ 4(ab+ bc+ cd+ da),

echivalenta cu(a− b+ c− d)2 ≥ 0.

Avem egalitate pentru a = b = c = d = 1.

Ex. 44 Fie a si b numere reale nenegative astfel ıncat 2a2 + b2 = 2a+ b. Aratatica

1− ab ≥ a− b3

.

Proof. Pentru a = b = 0, inegalitatea este adevarata. Altfel, scrieminegalitatea ın forma omogena

(2a2 + b2)2

(2a+ b)2− ab ≥ (a− b)(2a2 + b2)

3(2a+ b).

Deoarece(2a2 + b2)2

(2a+ b)2− ab =

(a− b)(4a3 − b3)(2a+ b)2

,

517

Page 519: Олимпиадад бэлтгэгчдэд тусламж (Тэнцэтгэл биш 1)

ramane sa aratam ca

(a− b)[3(4a3 − b3)− (2a+ b)(2a2 + b2)] ≥ 0.

Aceasta este echivalenta cu inegalitatea evidenta

(a− b)2(4a2 + 3ab+ 2b2) ≥ 0.

Egalitatea are loc pentru a = b = 1.

Ex. 45 Fie a, b, c numere reale pozitive astfel ıncat a+ b+ c = 3. Aratati ca

a2 + 3b

a+ b+b2 + 3c

b+ c+c2 + 3a

c+ a≥ 6.

Proof. Prin omogenizare, inegalitatea devine succesiv astfel:∑ a2 + (a+ b+ c)b

a+ b≥ 2(a+ b+ c),

∑(a2 + ab+ b2 + bc

a+ b− a− b

)≥ 0,

∑ b(c− a)

a+ b≥ 0,∑

b(b+ c)(c2 − a2) ≥ 0,

ab3 + bc3 + ca3 − abc(a+ b+ c) ≥ 0.

Ultima inegalitate rezulta din inegalitatea Cauchy-Schwarz, astfel

ab3 + bc3 + ca3 ≥√ab3c+

√abc3 +

√a3bc

c+ a+ b= abc(a+ b+ c).

Cu aceasta, demonstratia este ıncheiata. Avem egalitate pentru a = b =c = 1.

Ex. 46 Fie a si b numere reale astfel ıncat 9a2 + 8ab+ 7b2 ≤ 24. Demonstrati ca

7a+ 5b+ 6ab ≤ 18.

Proof. Observam ca pentru a = b = 1 avem egalitate. Tinand seama deacest fapt, pentru a avea numai expresii omogene de gradul doi ın a si bvom utliza inegalitatile 2a ≤ a2+1 and 2b ≤ b2+1, echivalente respectivcu (a− 1)2 ≥ 0 si (b− 1)2 ≥ 0. Atunci, avem

2(7a+ 5b+ 6ab− 18) ≤ 7(a2 + 1) + 5(b2 + 1) + 12ab− 36

= 7a2 + 5b2 + 12ab− 24 = (9a2 + 8ab+ 7b2 − 24)− 2(a2 + b2 − 2ab)

≤ −2(a− b)2 ≤ 0,

de unde rezulta imediat inegalitatea dorita. Egalitatea are loc daca sinumai daca a = b = 1.

518

Page 520: Олимпиадад бэлтгэгчдэд тусламж (Тэнцэтгэл биш 1)

Ex. 47 Fie a, b, c numere reale pozitive care satisfac abc = 1. Demonstrati ca

81(a2 + 1)(b2 + 1)(c2 + 1) ≤ 8(a+ b+ c)4.

Proof. Deoarece

a2 + 1 = a2 +3√a2b2c2 =

3√a2(

3√a4 +

3√b2c2

),

scriem inegalitatea ın forma omogena

81(

3√a4 +

3√b2c2

)(3√b4 +

3√c2a2

)(3√c4 +

3√a2b2

)≤ 8(a+ b+ c)4.

Pentru a demonstra noua inegalitate omogena, presupunem ca

a+ b+ c = 3.

Inegalitatea devine astfel

3

√(3√a4 +

3√b2c2

)(3√b4 +

3√c2a2

)(3√c4 +

3√a2b2

)≤ 2.

Utilizand inegalitatea mediilor, este suficient sa aratam ca∑(3√a4 +

3√b2c2

)≤ 6.

Utilizand din nou inegalitatea mediilor, avem

∑(3√a4 +

3√b2c2

)≤∑(

a2 + a+ a

3+bc+ bc+ 1

3

)

=(∑a)2 + 2

∑a+ 3

3= 6.

Cu aceasta, inegalitatea este demonstrata. Avem egalitate daca si numaidaca a = b = c = 1.

Ex. 48 Fie a, b, c numere reale nenegative care satisfac realatia

(a+ b)(b+ c)(c+ a) = 2.

Demonstrati ca

(a2 + bc)(b2 + ca)(c2 + ab) ≤ 1.

Proof. Scriem inegalitatea dorita sub forma omogena

4(a2 + bc)(b2 + ca)(c2 + ab) ≤ (a+ b)2(b+ c)2(c+ a)2.

Deoarece inegalitatea este simetrica ın a, b, c, presupunem, fara a pierdedin generalitate, ca a ≥ b ≥ c. Avem

a2 + bc ≤ a2 + ac ≤ (a+ c)2

519

Page 521: Олимпиадад бэлтгэгчдэд тусламж (Тэнцэтгэл биш 1)

si, din inegalitatea mediilor,

4(b2 + ca)(c2 + ab) ≥ (b2 + ca+ c2 + ab)2.

Atunci, este suficient sa demonstram inegalitatea

b2 + c2 + ab+ ac ≥ (a+ b)(b+ c),

echivalenta cu inegalitatea evidenta c(c−b) ≤ 0. Cu aceasta, demonstratiaeste ıncheiata. Avem egalitate atunci cand unul dintre numerele a, b, ceste 0, iar celelalte doua sunt egale cu 1.

Exercise 1

1 Fie x, y, z ∈ R. Aratati ca

(x2 + y2 + z2)[(x2 + y2 + z2)2 − (xy + yz + zx)2)]

≥ (x+ y + z)2[(x2 + y2 + z2)− (xy + yz + zx)]2.

2 Fie m,n ∈ N+, m > n. Aratati ca

(1 +1

n)n < (1 +

1

m)m

3 Fie a, b, c ∈ R+. Aratati ca

1

a+

1

b+

1

c≤ a8 + b8 + c8

a3b3c3.

4 Fie numerele reale a1, a2, · · · , a100 care satisfac:

(1) a1 ≥ a2 ≥ · · · ≥ a100 ≥ 0;

(2) a1 + a2 ≤ 100;

(3) a3 +a4 + · · ·+a100 ≤ 100. Gasiti maximul pentru a21 +a22 + · · ·+a2100.

5 Fie k, n numere ıntregi pozitive, 1 ≤ k < n. Daca x1, x2, · · · , xk suntnumere reale pozitive astfel ıncat

x1 + x2 + ...+ xk = x1x2...xk,

atunci

xn−11 + xn−12 + · · ·+ xn−1k ≥ kn.

6 Daca a, b, c ∈ R, aratati ca

(a2 + ab+ b2)(b2 + bc+ c2)(c2 + ca+ a2) ≥ (ab+ bc+ ca)3.

7 Fie a, b numere reale pozitive date, iar θ ∈ (0, π2 ). Gasiti maximul pentru

y = a√

sin θ + b√

cos θ.

520

Page 522: Олимпиадад бэлтгэгчдэд тусламж (Тэнцэтгэл биш 1)

8 Fie a1, a2, ..., an numere reale astfel ıncat |ai| ≤ 2 pentru i = 1, 2, ..., n sia31 + a32 + ...+ a3n = 0. Aratati ca

a1 + a2 + ...+ an ≤2n

3.

9 Pentru n ≥ 3, fie x1, x2, · · · , xn ∈ [−1, 1] astfel ıncat

x51 + x52 + ...+ x5n = 0.

Demonstrati ca

x1 + x2 + ...+ xn ≤8n

15.

10 Fie x1, x2, · · · , xn numere reale astfel ıncat x1+x2+...+xn = na. Aratatica

n∑k=1

(xk − a)2 ≤ 1

2(n∑k=1

|xk − a|)2.

11 Daca x, y, z ≥ 0, atunci

x(y + z − x)2 + y(z + x− y)2 + z(x+ y − z)2 ≥ 3xyz.

12 Daca x, y, z sunt numere reale pozitive, atunci

1

(x+ y)2+

1

(y + z)2+

1

(z + x)2≥ 9

4(xy + yz + zx).

13 (Cırtoaje, Ji Chen) Daca a, b, c si x, y, z sunt numere reale nenegative,atunci

2

(a+ b)(x+ y)+

2

(b+ c)(y + z)+

2

(c+ a)(z + x)≥ 9

(b+ c)x+ (c+ a)y + (a+ b)z.

14 Fie x, y, z numere pozitive astfel ıncat xyz = 1. Demonstrati ca

x

x3 + y + z+

y

y3 + z + x+

z

z3 + x+ y≤ 1.

15 Daca a, b, c sunt numere reale pozitive astfel ıncat a+ b+ c = 3, atunci

6(a

b+b

c+c

a+ 3 ≥ 7(a2 + b2 + c2).

16 Daca a, b, c,d sunt numere reale nenegative astfel ıncat a+ b+ c+d = 4,atunci

a2bc+ b2cd+ c2da+ d2ab ≤ 4.

17 Daca a, b, c sunt numere reale nenegative, cel mult unul egal cu 0, atunci

1

b2 − bc+ c2+

1

c2 − ca+ a2+

1

a2 − ab+ b2≥ 12

(a+ b+ c)2.

521

Page 523: Олимпиадад бэлтгэгчдэд тусламж (Тэнцэтгэл биш 1)

18 Daca a, b, c sunt numere reale nenegative, cel mult unul egal cu 0, atunci

ab− bc+ ca

b2 + c2+bc− ca+ ab

c2 + a2+ca− ab+ bc

a2 + b2≥ 3

2.

19 (Cırtoaje) Daca a ≥ b ≥ c > 0, atunci

3(a− c)2

4(a+ b+ c)≤ a+ b+ c− 3

3√abc ≤ 4(a− c)2

a+ b+ c.

20 Fie a, b, c numere reale pozitive astfel ıncat abc = 1. Aratati ca

3 + a

(1 + a)2+

3 + b

(1 + b)2+

3 + c

(1 + c)2≥ 3.

21 Daca a, b, c, d sunt numere reale pozitive astfel ıncat a + b + c + d = 4,atunci

1

ab+

1

bc+

1

cd+

1

da≥ a2 + b2 + c2 + d2.

22 Daca 0 < a ≤ b ≤ c ≤ 0, atunci

a

b+b

c+c

a≥ 2a

b+ c+

2b

c+ a+

2c

a+ b.

23 Daca a, b, c, d 6= 13 sunt numere reale pozitive care satisfac relatia

abcd = 1,

atunci

1

(3a− 1)2+

1

(3b− 1)2+

1

(3c− 1)2+

1

(3d− 1)2≥ 1.

24 Daca a, b, c sunt numere reale pozitive care satisfac relatia ab+bc+ca = 3,atunci

bc+ 2

a2 + 2+ca+ 2

b2 + 2+ab+ 2

c2 + 2≥ 3.

25 Fie a, b, c numere reale nenegative, dintre care cel mult unul egal cu 0.Demonstrati ca

a(b+ c)

b2 + bc+ c2+

b(c+ a)

c2 + ca+ a2+

c(a+ b)

a2 + ab+ b2≥ 2.

26 Fie a, b, c, d, e numere reale astfel ıncat a+b+c+d+e = 0. Demonstratica

a2 + b2 + c2 + d2 + e2 ≥ 3(ab+ bc+ cd+ de+ ea).

27 Daca a1, a2, ...an ∈ [1, 2] , atunci

n∑i=1

3

ai + 2ai+1≥

n∑i=1

2

ai + ai+1,

unde an+1 = a1.

522